Sie sind auf Seite 1von 847

Title Page INTERNATIONAL

EMERGENCY
MEDICINE
EDUCATION
PROJECT

iem-student.org

iEmergency Medicine for


Medical Students and Interns
1st Edition, Version 1, 2018

A Free Book For

Emergency
Medicine
Clerkship
Students

Editors
Arif Alper Cevik
Lit Sin Quek
Abdel Noureldin
Elif Dilek Cakal

i
Copyright ©2018 International Emergency Medicine (iEM) Education Project
Copyright Resources shared by iEM Education Project through website, book content,
image and video archive are distributed under the terms of the  Creative
Commons Attribution-NonCommercial-ShareAlike 4.0 International License,
which permits unrestricted use, distribution in any non-commercial medium.
You give appropriate credit to the original author(s) and the source, provide a
link to the Creative Commons license, and indicate if changes were made.

Edited by
Arif Alper Cevik, Lit Sin Quek, Abdel Noureldin, and Elif Dilek Cakal

Cover design
Arif Alper Cevik

Book design and production


Arif Alper Cevik

Published by
iEM Education Project, iem-student.org

First electronic edition,


2018, Version 1

INTERNATIONAL
EMERGENCY
MEDICINE
EDUCATION
PROJECT

iem-student.org
ii
What is iEM? International Emergency Medicine (iEM) Education Project is an
international, non-profit project, endorsed by International Federation for
Emergency Medicine (IFEM) and supported by Emergency Medicine
professionals from all around the world. Currently, there are 146
contributors from 21 countries in the project. It aims to  promote
Emergency Medicine  and  provide free, reusable educational content for
undergraduate medical trainees and educators.

The project focuses mainly on undergraduate curriculum, but learners from


all levels (medical students, interns, residents) and their educators may
benefit from it. It is targeted at all learners around the globe. However,
learners from areas with limited resources have a special place in mind.

The book "iEmergency Medicine for Medical Students and Interns" is a part
of the project content and service. It was created by 133 international
contributors. At the publication time, the book content provided in the iem-
student.org has been visited by more than 20,000 visitors from 150
countries.

The content is entirely free and available in various formats including


website, iBook, pdf, image, video, and audio. Clinical image and video
archives are accessible in Flickr and YouTube accounts. The audio chapters
and podcasts  are available via SoundCloud. Please visit the”blog” for up-to-
date posts. The website is mobile-friendly to maximize accessibility. We
support free open access medical education (FOAMed). All materials are free
to use, download and share.

iEM education project is run by volunteers, and you can join us. Please visit
“how to contribute” page to share your Emergency Medicine experience and
message with future colleagues from all around the world.

Arif Alper Cevik, MD, FEMAT, FIFEM


Founder and Director, iEM Education Project

iii
Preface “if you want to go fast go alone,
if you want to go far go together”
African Proverb

Undergraduate Emergency Medicine Education (UEME) is an undervalued area in


the development of Emergency Medicine around the globe. If you read the articles
regarding Emergency Medicine clerkships or if you travel to different countries and
discuss their undergraduate education with local leaders, you can easily recognize
the gaps between countries.

Today, there are few countries in the world that have appropriately designed
UEME programs in their medical schools. The majority of the countries (even
some developed ones) have no guidelines, curricula, or enough educational
resources. In addition, there are limited resources (textbooks, websites) for
medical students/interns which covers their educational needs based on current
UEME recommendations.

This book is a product of an international collaboration of emergency physicians


and Emergency Medicine enthusiasts. It intends to show that we can produce a
free book and resource if we work collaboratively. It is a product of endless hours
of hard work of all Editors, authors, and contributors. We thank all of them for
trusting us in this journey.

This is just a start to build up better Emergency Medicine resources for medical
students and interns, especially for developing countries. It is a continuous
process, and there are a lot of areas that we need to improve in this book.
Therefore, we are looking forward to your feedback and collaboration.

We also believe that international UEME will reach the minimum required
standards in all countries based on the endless collaboration of emergency
medicine professionals.

iv
Terms of Use This book aims to provide general Emergency Medicine information and dis-
cussion to medical students and interns.The content and discussions found
on this electronic book are not individualized medical advice and can not be
used for this purpose. If you think you need emergency care or any type of
medical care, please contact your doctor or call local/national emergency
number.

Notification The iEM is a global project and may contain various opinions and
approaches. The information and opinions expressed in this book have no
relation to those of any academic, hospital, practice, institution, or worldview
Some images in this book may be disturbing. Some of with which the authors or editors are affiliated, and does not represent
the topics may contain medical-related information institutional policies.

that is sexually explicit. If you find this information The content of this book has been prepared by international authors to
offensive, you may not want to use the book. This provide this content to medical students and interns. The content, informa-
tion, opinions, references, and links to other knowledge resources provided
book includes medical content, particularly emergency
by Editors and authors are only for medical students and interns’
medicine related, for medical students and interns. educational purposes. This book is not intended or advised for public use.
Some of the content in this book may contain elements
The iEM project, its’ Editors, contributors and its’ team do not intend to
that are not suitable for some readers. Accordingly,
establish any physician-patient relationship through the contents of this
viewer discretion is advised. book, nor does it replace the services of a trained physician or health care
professional, or otherwise to be a substitute for professional medical advice,
The book content is not suitable for persons under 18 diagnosis, or treatment. Again, this is an educational book for medical
years of age. Persons under the age of eighteen (18) students and interns, and it is not medical care platform. Therefore, you shall
should not access, use and browse the book. not make any medical or health-related decision based in whole or in part on
anything contained in this book. If you need medical care or advice, you
should consult a licensed physician in your community healthcare office or
hospitals.

Although their main interest is emergency medicine, the content of the book
was prepared by international contributors from different backgrounds.
Medicine itself is changing very fast. Therefore, we can not guarantee

v
providing complete, correct, timely, current or up-to-date our best to get functioning these links, but they may stop
information in the book. Similar to any printed material, the con- functioning any other reason.
tent may become out-of-date and may be changed without
notice. The Editors and authors have no obligation to update any The iEM project Director, Editors, and authors are not responsible
content in the book. The Editors may update the content at any for the content of any linked or otherwise connected websites.
time without notice, based on their absolute discretion. The iEM The iEM project Director, Editors, and authors do not make any
project Director and Editors reserve the right to make alterations representations or guarantees regarding the privacy practices of,
or deletions to the content at any time without notice. or the content or accuracy of materials included in, any linked or
third party websites or resources. The inclusion of third-party
Opinions expressed in the book are not necessarily those of the links in the book does not constitute an endorsement, guarantee,
Editors, authors and iEM project team. These opinions cannot be or recommendation.
applied to an individual case or particular circumstance. The con-
tent should not be used or relied upon for any other purpose, The book content is not suitable for persons under 18 years of
including, but not limited to, use in or in connection with any legal age. Persons under the age of eighteen (18) should not access,
proceeding. Some images in this book may be disturbing. Some use and browse the book.
of the topics may contain medical-related information that is sexu-
Although the Editors and authors have made every effort to
ally explicit. If you find this information offensive, you may not
assure that the information in this book is correct at publication
want to use the book. This book includes medical content,
time, the Editors and authors do not assume and hereby disclaim
particularly emergency medicine related, for medical students and
any liability to any party for any loss, damage, or disruption
interns. Some of the content in this book may contain elements
caused by errors or omissions, whether such errors or omissions
that are not suitable for some readers. Accordingly, viewer
result from negligence, accident, or any other cause.
discretion is advised.
For more information regarding terms of use, please visit website.
This project aims to expand medical students interest in and
knowledge of Emergency Medicine. Therefore, iEM Editors and
team continuously search for valuable third party links. The book
may contain links to other (“third party”) websites, videos, etc.
These links are provided solely as a convenience and not as a
guarantee or recommendation by the Editors or authors for the
services, information, opinion or any other content on such third
party websites or as an indication of any affiliation, sponsorship
or endorsement of such third party resources. If you decide to
access a given link, you do so at your own risk. Your use of other
websites is subject to the terms of use for these sites. We tried

vi
Publishing Process There is continuous work for the iEM book process. We applied multiple
editing and reviewing steps. We continue this process for many chapters
with the feedback from our readers and contributors.

The Editors and authors have made every effort to assure that the
information in this book is correct and appropriate for medical students and
interns. The Editors and authors do not assume and hereby disclaim any
liability to any party for any loss, damage, or disruption caused by errors or
omissions, whether such errors or omissions result from negligence,
accident, or any other cause.

We used original images, illustrations, diagrams provided by the Editors and


authors as much as possible. However, there were chapters that we needed
to use some copyright free material, Creative Commons licensed images,
illustrations, and diagrams with attribution to the original owners. We are
continuously searching for better images, illustrations, and diagrams. If you
have copyright free clinical images, illustrations or diagrams, please share
them with us. We would like to use them with your credentials in the book,
online archive, and website.

vii
Acknowledgement We thank the institutions and organizations which helped this project to see
the light. The United Arab Emirates University, College of Medicine and
Health Sciences supported funding the expenses of iEM Education Project.
The International Federation for Emergency Medicine (IFEM) supported the
project since beginning and officially endorsed on October 2018.

We thank the Council of Residency Directors in Emergency Medicine


(CORDEM) and American College of Emergency Physicians, International
Emergency Medicine Section (ACEP-IEM) for their help finding contributors.

We thank our Editors, authors, and collaborators for their time and fantastic
work during the production of this book.

We also thank Emirates Society of Emergency Medicine (ESEM), Emergency


Medicine Association of Turkey (EMAT), Sociedad Mexicana de Medicina de
Emergencias (SMME) for their continuous support for the project. We like to
specifically acknowledge the support of a young group of Slovenian doctors
for their amazing contributions.

We would like to thank Prof.Dr. Fikri Abu-Zidan for his wise advise and
continuous encouragement during the hardship of delivering the book and
its content.

viii
Editors Editors
Arif Alper Cevik, UAE
Lit Sin Quek, Singapore
Abdel Noureldin, USA
Elif Dilek Cakal, Turkey

Section Editors
Toh Hong Chuen, Singapore
Veronica Tucci, USA
Silvio Aguilera, Argentina
Funda Karbek Akarca, Turkey
Rahul Goswami, Singapore
Mary J.O., USA
Ziad Kazzi, USA
Jesus Daniel Lopez Tapia, Spanish Section Editor, Mexico
Olinda Giselle G. Saenz, Spanish Section Editor, Mexico

Language Editor
Sarah Elizabeth Noureldin, USA

ix
Contributors Abdel Noureldin, USA David Wood, USA
Abdulaziz Al Mulaik, KSA Diana V. Yepes, USA
Aldo E.M. Salinas, Mexico Dejvid Ahmetovic, Slovenia
Alja Pareznik, Slovenia Donna Venezia, USA
Ana Podlesnik, Slovenia David F. Toro, USA
Ana Spehonja, Slovenia Ebru Unal Akoglu, Turkey
Amila Punyadasa, Singapore Eisa AlKaabi – UAE
Arif Alper Cevik, UAE-Turkey Elif Dilek Cakal, Turkey
Arwa Alburaiki. UAE Elizabeth Bassett, USA
Assad Suliman Shujaa, Qatar Emilie J. Calvello Hynes, USA
Ashley Bean, USA Eman Al Mulla, UAE
Aya Dodin, UAE Falak Sayed, UAE
Ayse Ece Akceylan, Turkey Fathiya Al Naqbi, UAE

133 contributors Azizul Fadzi, Malaysia


Bader AlQahtani, UAE
Farhad Aziz, USA
Fatih Buyukcam, Turkey
Begum Oktem, Turkey Feriyde Caliskan Tur, Turkey
are from Bita Abbasi, Iran
Bret Nicks, USA
Funda Karbek Akarca, Turkey
Gregor Prosen, Slovenia
Brian Hohertz, USA Gregory R. Snead, USA
19 different Charlotte Derr, Canada
Chew Keng Sheng, Malaysia
Gul Pamucu Gunaydin, Turkey
Gurpreet Mudan, USA

countries. C. James Holliman, USA


Dan O’Brien, USA
Hamidreza Reihani, Iran
Harajeshwar Kohli, USA
Danny Cuevas, USA Hind Al Dhaheri, UAE
David Hoffman, USA Jabeen Fayyaz, Pakistan
David Wald, USA Jan Zajc, Slovenia

x
Jesus Daniel Lopez Tapia, Mexico Meltem Songur Kodik, Turkey Sadiye Yolcu, Turkey
Joe Lex, USA Michael Butterfield, USA Sarah Attwa, UAE
Jorge Luis Garcia Macias, Mexico Michelle Chan, USA Sara Nikolic, Slovenia
Josepph Pinero, USA Moira Carrol, USA Selene Larrazolo Carrasco, Mexico
Justin Brooten, USA Muneer Al Marzouqi, UAE Sercan Yalcinli, Turkey
Kaja Cankar, Slovenia Murat Cetin, Turkey Serpil Yaylaci, Turkey
Kamil Kayayurt, Turkey Mustafa Emin Canakci, Turkey Shabana Walia, USA
Katja Zalman, Slovenia Nidal Moukaddam, USA Shaza Karrar, UAE
Keith A Reymond, Austria Nik A.S.N. Him, Malaysia Shanaz Sajeed, USA
Kemal Gunaydin, Turkey Nik Rahman, Malaysia Shirley Ooi, Singapore
Khuloud Alqaran, UAE Nur-Ain Nadir, USA Stacey Chamberlain, USA
Khalid Mohammed Ali, Singapore Olinda Giselle Garza Saenz, Mexico Sujata Kirtikant Sheth, Singapore
Kuan Win Sen, Singapore Ozge Can, Turkey Suzanne Bentley, USA
Lamiess Osman, UAE Ozlem Dikme Akinci, Turkey Tanju Tasyurek, Turkey
Linda Katirji, USA Ozlem Koksal, Turkey Tiffany Abramson, USA
Lindsay Davis, USA Pia Jerot, Slovenia Timothy Snow, USA
Lit Sin Quek, Singapore Puneet Sharma, UK Tjasa Banovic, Slovenia
Mahmoud Aljufaili, Oman Qais Abuagla, UAE Toh Hong Chuen, Singapore
Mary J O, USA Rabind Anthony Charles, Singapore Tomislav Jelic, Canada
Maryam AlBadwawi, UAE Ramin Tabatabai, USA Veronica Tucci, USA
Maryam Darwish, UAE Rahul Goswami, Singapore Vigor Arva, Slovenia
Marwan Galal, UAE Rasha Buhumaid, UAE Vijay Nagpal, USA
Matevz Privsek, Slovenia Reza Akhavan, Iran Walid Hammad, USA
Matija Ambooz, Slovenia Rob Rogers, USA Will Sanderson, USA
Matthew Lisankie, Canada Rok Petrovcic, Slovenia Yadira Rubio Azuara, Mexico
Matthew Smetana, USA Rouda Salem Alnuaimi, UAE Yusuf Ali Altunci, Turkey
Mehmet Ali Aslaner, Turkey Ryan H. Holzhauer, USA Ziad Kazzi, USA

xi
Content 1. The Facts of Emergency Medicine
Emergency Medicine: A Unique Specialty     
Will Sanderson, Danny Cuevas, Rob Rogers
Choosing The Emergency Medicine As A Career     
C. James Holliman
Thinking Like An Emergency Physician     
Joe Lex

2. Emergency Medicine Clerkship: Things to Know


The Importance of The Emergency Medicine Clerkship     
Linda Katirji, Farhad Aziz, Rob Rogers
Medical Professionalism: The Dimensions That All Medical Students Should
Know About     
Amila Punyadasa
Communication and Interpersonal Interactions     
Vijay Nagpal, Bret A. Nicks
Data Gathering     
Chew Keng Sheng
Diagnostic Testing in Emergency Medicine     
Yusuf Ali Altunci
Creating Your Action Plan     
Chew Keng Sheng
Documentation     
Muneer Al Marzouqi, Qais Abuagla
Discharge Communications     
Justin Brooten, Bret Nicks

xii
3. General Approach to Emergency Patients Acute Heart Failure     
Walid Hammad
The ABC Approach to Critically Ill Patient    
Donna Venezia Aortic Dissection     
Shanaz Sajeed
Abdominal Pain     
Shaza Karrar Deep Venous Thrombosis     
Elif Dilek Cakal
Altered Mental Status     
Murat Cetin, Begum Oktem, Mustafa Emin Canakci  Hypertensive Emergencies     
Sadiye Yolcu
Cardiac Arrest    
Abdel Noureldin,  Falak Sayed Pulmonary Embolism     
Elif Dilek Cakal
Chest Pain     
Assad Suliman Shujaa
A Child With Fever     
Jabeen Fayyaz
5. Selected Endocrine, Electrolyte
Gastrointestinal Bleeding      Emergencies
Moira Carrol, Gurpreet Mudan, Suzanne Bentley Acid-Base Disturbance     
Headache      Lamiess Osman, Qais Abuagla
Matevz Privsek, Gregor Prosen Hyperglycaemia     
Multiple Trauma      Hong Chuen
Pia Jerot, Gregor Prosen Hypernatremia     
Poisoning      Vigor Arva, Gregor Prosen
Harajeshwar Kohli, Ziad Kazzi Hyponatremia     
Respiratory Distress      Vigor Arva, Gregor Prosen
Ebru Unal Akoglu Hypoglycaemia     
Shock      Rok Petrovcic
Maryam AlBadwawi Thyroid Storm     
Shabana Walia
4. Selected Cardiovascular Emergencies
Abdominal Aortic Aneurysm     
Lit Sin Quek
Acute Coronary Syndrome     
Khalid Mohammed Ali, Shirley Ooi

xiii
6. Selected Environmental Emergencies 9. Selected Neurological Emergencies
Burns      Approach to Patients With Stroke     
Rahul Goswami Matevz Privsek, Gregor Prosen
Drowning      Acute Ischemic Stroke     
Ana Spehonja Fatih Buyukcam
Heat Illness      Intracranial Hemorrhage     
Abdulaziz Al Mulaik Nur-Ain Nadir, Matthew Smetana
Hyperthermia      Seizure     
Puneet Sharma Feriyde Caliskan Tur

10. Selected Pulmonary Emergencies


7. Selected Gastrointestinal Emergencies Asthma     
Acute Appendicitis      Ayse Ece Akceylan
Ozlem Dikme COPD - Chronic Obstructive Pulmonary Disease     
Biliary Disease      Ramin Tabatabai, David Hoffman, Tiffany Abramson
Dan O’Brien Pneumonia     
Massive Gastrointestinal Bleeding      Mary J O
Dan O’Brien Spontaneous Pneumothorax     
Acute Mesenteric Ischemia     Mahmoud Aljufaili
Rabind Anthony Charles
Perforated Viscus      11. Selected Psychiatric Emergencies
Ozlem Dikme Acute Psychosis     
Elizabeth Bassett, Nidal Moukaddam, Veronica Tucci
Stabilization and Management of The Acutely Agitated or
8. Selected Genitourinary Emergencies Psychotic Patient    
Michelle Chan, Nidal Moukaddam, Veronica Tucci
Ectopic Pregnancy     
Dan O’Brien Medical Clearance-Suicidal Thought/Ideation     
Veronica Tucci
Tubo-Ovarian Abscess   
Matthew Lisankie, Charlotte Derr, Tomislav Jelic
Testicular Torsion     
Sujata Kirtikant Sheth

xiv
12. Selected Orthopaedic Problems and 16. Selected Procedures
Injuries Automatic External Defibrillator (AED) Use     
Back Pain      Mehmet Ali Aslaner
Funda Karbek Akarca Arterial Blood Gas Sampling     
Lower Extremity Injuries      Matija Ambooz and Gregor Prosen
Ayse Ece Akceylan Arthrocentesis     
Pelvic Injuries      Tanju Tasyurek
Sercan Yalcinli Basics of Bleeding Control     
Spine Injuries      Ana Spehonja, Gregor Prosen
Ozge Can Cardiac Monitoring     
Upper Extremity Injuries      Stacey Chamberlain
Meltem Songur Kodik Gastric Lavage and Activated Charcoal Application     
Elif Dilek Cakal
13. Selected Infectious Problems Intravenous Line Access     
Epiglottitis     Keith A Reymond
KuanWin Sen Intraosseous (IO) Line Access     
Meningitis      Keith A Reymond
Alja Pareznik Emergency Delivery     
Sinusitis      David F. Toro, Diana V. Yepes, Ryan H. Holzhauer
Katja Zalman, Gregor Prosen Pericardiocentesis     
Sepsis     David Wald, Lindsay Davis
Emilie J. Calvello Hynes Lumbar Puncture    
Khuloud Alqaran
14. Selected Toxicologic Problems Nasogastric Tube Placement     
Opioid Overdose      Sara Nikolic, Gregor Prosen
Aldo E. B. Salinas, Jesus Daniel Lopez Tapia Procedural Sedation and Analgesia     
Nik Rahman
15. Selected Eye Problems Rapid Sequence Intubation     
Eye Trauma      Qais Abuagla
Serpil Yaylaci, Kamil Kayayurt
Reduction of Common Dislocations and Fractures     
Red Eye      Dejvid Ahmetović, Gregor Prosen
David Wood
xv
Splinting and Casting      19. Selected Emergency Drugs
Joseph Pinero, Timothy Snow, Suzanne Bentley
Antidotes     
Urinary Catheter Placement      Hamidreza Reihani, Elham Pishbin
Gul Pamucu Gunaydin
Drugs for Pain Relief    
Nik Ahmad Shaiffudin Nik Him, Azizul Fadzi
17. Selected Diagnostic Tests
Paralyzing Agents     
Arterial and Venous Blood Gases Analyses     
Qais Abuagla
Kemal Gunaydin
Cerebrospinal Fluid Analysis      20. Selected Clinical Rules, Scores,
Arwa Alburaiki, Rouda Salem Alnuaimi
Urine Analysis     
Mnemonics
Jan Zajc Clinical Decision Rules     
Stacey Chamberlain
Whole Blood Cell Count – CBC     
Kaja Cankar Mnemonics     
Ozlem Dikme
18. Selected Imaging Modalities Classifications and Scores     
EFAST - Extended Focused Sonography for Trauma      Sarah Attwa, Marwan Galal
Ashley Bean, Brian Hohertz, Gregory R. Snead
POCUS in Patients with Undifferentiated Hypotension     
Rasha Buhumaid
BLUE Protocol     
Toh Hong Chuen
How to Read C-Spine X-Rays     
Dejvid Ahmetovic, Gregor Prosen
How to Read Chest X-Rays     
Ozlem Koksal
How to Read Head CT    
Reza Akhavan, Bita Abbasi
How to Read Pelvic X-ray     
Sara Nikolic, Gregor Prosen

xvi
Chapter 1

The Facts of
Emergency
Medicine
Section 1

Emergency Medicine: A unique specialty  

Imagine walking into the hospital to start your day – ambulances are blaring, the
by Will Sanderson, Danny Cuevas, Rob Rogers waiting room is clamoring, babies are crying. You stroll through this sea of
humanity and eventually arrive at your workstation. After setting your bag down,
you prepare the basic tools of your trade: a stethoscope, a fresh cup of coffee,
and a sharp mind. Taking a deep breath, you prepare for the routine of yet another
shift. But there is no “routine.” There is only the excitement and variety of what is
about to come through those sliding double doors. That flimsy piece of metal and
glass is the only barrier that separates you from the thousands of people with a
multitude of medical ailments, any one of which could bring them to your
doorstep. With a low hum and an almost silent whoosh, these doors part to reveal
your next patient. To them, it is probably the worst day of their life. For you, it’s
another Tuesday.

Who will be your next patient? Is it the 4-year-old boy with the asthma attack
gasping for that next breath? Will it be the 78-year-old widower who fell at home
while fixing himself a sandwich? Maybe it’s the 31-year-old female who just rear-
ended another vehicle at highway speed; oh, did they also mention she’s 28 weeks
pregnant? You look over and see new patients filling the critical examination rooms
and the trauma bay. No matter what walks through that door, you’ll be ready. You
sit down. You grab a chart. It’s time to get to work. Today is going to be another
Audio is available here routinely awesome day.

18
Why choose a career in emergency medicine? Before discussing d e e p e r l o o k i n t o t h e practice and lifestyle of the modern
where the field is going, it’s important to know a little background emergency medicine physician.
on where it has been. And if you’re reading this and considering a
career in EM, do yourself a favor – take the time to watch this Why EM?
documentary  from the Emergency Medicine Residents’ Emergency medicine is a fast paced, team oriented, dynamic

Association (EMRA). As you’ll see, the specialty of emergency specialty that focuses on the rapid evaluation and treatment of a

medicine has evolved drastically over the last several decades diversified patient population consisting of both pediatric and

and continues to be an increasingly popular choice among adult patients. As the initial provider for many of their patients, the

graduating medical students. Only a few decades ago, emergency medicine physician is charged with the rapid

emergency departments around the country were staffed by assessment and data gathering needed to launch the initial

physicians with a variety of training backgrounds. The vast workup and management of a wide variety of complaints that

majority of these physicians had little to no emergency medicine bring patients to the ED. Their work has an incredible influence in

training at all. General surgeons, family physicians, neurologists, the patients’ care as it generates the driving force for further

and even psychiatrists were among those that staffed emergency medical evaluation; whether the patient is admitted to the hospital

departments around the country and throughout the world. But or discharged home, the emergency physician plays a huge role

since the establishment of the first emergency medicine residency in directing both short and long term care well after their stay in

programs in the 1970’s and the subsequent establishment of the the emergency department. Here’s a look inside the lives of

American Board of Emergency Medicine in 1979, the specialty several emergency physicians from Rob Orman of ERcast.
has continued its rapid development in defining its place in the Variety is the spice of the EM life. There is no set routine or
house of medicine. Walk into anything other than the smallest of expected patient list for the day. In the short span of a shift, you
EDs these days and you’re likely to encounter an emergency may diagnose strep pharyngitis, intubate an unresponsive patient
medicine residency trained physician. A study published in 2008 who overdosed on heroin, reveal a cancer diagnosis to a young
demonstrated that in its relatively short history as a recognized patient with flu-like symptoms, reduce a dislocated hip, place a
medical specialty, the number of physicians staffing departments chest tube in a patient with a hemothorax, and resuscitate a
across the country who had received emergency medicine patient undergoing a cardiac arrest. Your next patient could be a
training soared from 0% to 70%. Why the dramatic shift? To six-year-old or a 75-year-old, both with abdominal pain. In a
understand the answer to this question, you need to take a setting where some may see chaos, EM physicians find order. It’s
19
exciting. It’s energizing. This diversity is a uniquely challenging t h e s h i f t w o r k i n t h e emergency department affords a
aspect of the medicine practiced in the emergency department. level of flexibility not seen in other medical specialties. Emergency
physicians manage the hustle and bustle of their department for a
EM physicians pull from a knowledge base that spans all medical
set number of hours, after which a fresh physician team arrives to
specialties including pulmonology, cardiology, gastroenterology,
take over. After his shift, the previous doctor hands over the care
trauma surgery, nephrology, ophthalmology, psychiatry, and
of his patients to the oncoming team to continue with the
neurology. Jack of all trades? Sure. Master of none? Not even
diagnostic and therapeutic management of the patient. In this
close.   The gap between the medical and surgical specialties is
regard, one can wrap up, sign out, and head home without
bridged within the practice of emergency medicine. The
bringing any of his work with him. The nature of shift work also
combination of a broad knowledge base with the need to develop
allows for trading of shifts amongst the physicians staffing the
a focused procedural skill set makes the EM physician a veritable
department. Want a week off in April to spend some time at the
Swiss Army Knife within the house of medicine. From
beach? As long as you plan in advance, you shouldn’t have any
endotracheal intubation, cricothyroidotomy, fracture reduction,
trouble getting there. With enough planning, it’s quite possible to
and central line placement to pericardiocentesis, thoracotomy,
be at nearly every important life/family event you choose.
chest tube placement, and lateral canthotomy, even the most
enthusiastic proceduralist will find his hands full working in the Within the field of emergency medicine, physicians are employed
ED. in several settings.  These settings range from   hospital-based
and freestanding emergency departments, urgent care facilities,
Variety is a word that not only defines the practice of emergency
observation medicine units, emergency medical response
medicine but also the lifestyle it affords. Are you a morning
services, and even telemedicine locations. Patient volumes, even
person who is up at the break of dawn and thinks best with a
at facilities in close proximity to one another, can vary greatly.
fresh mind after breakfast? Or are you a night owl who gets a
Some facilities are designated trauma centers while others are
burst of energy in the wee hours of the night when most others
not. There are facilities teamed up with a strong academic center
are sound asleep? Are you a weekend warrior who prefers to
to provide numerous subspecialty support and others are
keep your schedule open on those days? Or would you rather
resource-limited community hospitals. No matter what your
work during the day to finish in time to pick up your children after
preference, there are a variety of settings to fit your needs. But
they finish their day at school? Irrespective of your preference,
let’s get to the real question at hand: are emergency medicine

20
physicians satisfied with their career? This is really the crux of any
discussion regarding career choice. How devastating would it be
to realize after spending over a decade in college, medical
school, and residency that working in the emergency department
isn’t for you? Well, in 2015, emergency medicine physicians came
in 4th in overall career satisfaction compared to other medical
specialties. 60% of all emergency physicians surveyed were
satisfied with their income. Emergency physicians typically work
more intensely for fewer total hours compared with other
physicians and enjoy above-average compensation per hour.
Below, Dr. Kevin King of the University of Texas Health Science
Center San Antonio discusses the Pros and Cons of a Career in
Emergency Medicine: Pros and Cons of a Career in Emergency
Medicine.  As you can see, the life of an emergency medicine
physician is not a perfect fit for everyone. EM physicians suffer
from relatively high rates of burnout. However, as the field evolves
and physician wellness becomes a priority for all physicians
within medicine, this will surely improve. If the characteristics
outlined above are consistent with the qualities you are looking
for in a specialty, emergency medicine may well be the perfect fit
for you.

References and Further Reading, click here.

21
Section 2

Choosing the Emergency Medicine As A Career

The specialty of Emergency Medicine (EM) is a great career choice for medical
by C. James Holliman students and interns.  In August 2013, I celebrated my 30th year in full-time EM
clinical practice, and I remain very happy and satisfied with my career choice.  I
have served as a career advisor to medical students and interns for over 30 years
now and am very interested in encouraging people to undertake EM as a career.

Why is EM a great career? The main summary reason is that it is challenging and
very personally rewarding. You can directly and quickly see the benefits and
positive results of your diagnosis and treatment of patients who have emergent
medical conditions. You have the satisfaction of knowing you have made a big
positive difference in patients’ lives and well-being.  EM encompasses a very wide
variety of patients and medical and surgical problems. EM deals with patients of
both genders and all ages. The variety of cases seen by EM is probably greater
than that of any other specialty, and this aspect is part of what makes EM so
interesting and stimulating. The practice of EM encompasses a nice mix of
diagnostic medicine and of performing diagnostic and therapeutic procedures. 
The EM practitioner sees patients with undifferentiated symptoms and so must
make the initial diagnosis of many conditions. EM interacts with all the other
medical specialties, and at most hospitals accounts for the majority of hospital
A video is available here admissions.
Audio is available here

22
Unique subjects routinely taught in EM include: cost-effective intensive care, pediatrics, o b s t e t r i c s , i n t e r n a l m e d i c i n e ,
ancillary test ordering, efficiency in patient flow, managing cardiology, trauma, etc.), and this direct exposure to other
multiple simultaneous patients, coordinating Prehospital and multiple specialties makes EM residencies more interesting. Of
Emergency Department (ED) care, focusing the approach to course, the majority of time in most EM residencies is spent in the
medical problems, speed and efficiency of patient evaluations, hospital ED.    Most EM residencies also offer opportunities to
efficient use of ancillary personnel, efficient recording and participate in prehospital care and EM research.  One validation
transmittal of clinical data, and injury and violence prevention. EM of the strength of EM as a career is that in the U.S. each year it is
is also a young, vibrant specialty with a lot of enthusiastic the first or second most popular choice for residency by medical
practitioners, most of whom have extended interests and talents students, and the overall residency program “fill” rate in the
outside of medical practice, and who serve as role models and National Residency Matching Program is over 99%. For more
mentors. information on EM residencies from the perspective of EM
residents, check the website www.emra.org.
EM also encompasses the supervision of and interactions with
prehospital care. EM receives patients brought to the hospital ED EM has also developed a number of sub-specialties which
by ambulance. EM is responsible for training the prehospital enhance the career options in EM.  Each sub-specialty offers
personnel, and in some countries, EM practitioners may find post-residency fellowship training programs of one to three years
themselves directly staff ambulances, both ground and duration. In the U.S.A., the following EM subspecialties are
aeromedical. EM also is the main specialty involved in the officially recognized and have their own sub-specialty exam
planning for, and management of disasters and mass casualty certification: Pediatric EM, Toxicology, Critical Care, Sports
situations, both of which also require close interaction with Medicine, Hyperbaric Medicine, Emergency Medical Services,
prehospital care. and Palliative Care.  Additional EM sub-specialty fellowship
programs include International EM (or Global Health), Ultrasound,
EM has well-developed residency training programs for medical
Research, Education, Simulation Training, Aeromedical, Disaster,
school graduates in many countries. The length of these training
Trauma, Administration, and Information Technology. Check the
programs varies from country to country but generally is three to
website www.saem.org for the most up to date listing of EM
five years. The EM residencies each have some clinical rotations
fellowship programs.
on other services or specialties (such as anesthesia, surgery,

23
One of the greatest assets of EM as a career is the wide variety of national healthcare system, and there is extensive medical
post-residency career choices or options.  These include literature support for the value and efficacy of EM.
practicing in a variety of hospital types: university, teaching,
Another nice EM career aspect is the opportunity after residency
community, government, military, etc.  EM physicians can also
or fellowship to participate in one or more of the EM specialty
practice in “freestanding” ED’s (not directly connected to a
state, national, regional, or international organizations.  Most
hospital) or in urgent care centers.  If a person does not want to
countries have a national EM organization which carries out some
practice at just one hospital, there is the option to undertake
activities including annual educational conferences.  The
“locum tenens” practice in which the person works clinical shifts
International Federation for EM (IFEM) has a large number of
at multiple different facilities. For those in the military or interested
committees, task forces, and special interest groups which are
in a military career, EM has been shown to be one of the most
carrying out a wide variety of projects which need more
needed specialties in the military.  EM physicians can undertake
individuals to participate and contribute (check www.ifem.cc for
leadership positions in hospital administration, prehospital care,
more information on IFEM).  Becoming involved with one or more
and in the government developing and directing health policy.
of the EM specialty organizations can provide one with career
Unique advantages of EM as a career include the almost satisfaction in helping improve and develop the specialty as well
unlimited opportunities in international EM development, control as obtaining leadership training and experience.
over and predictability of one’s work schedule, usually not having
So in summary, EM is a great career choice with a very wide
to be “on-call” when not directly on duty, and having “geographic
range of post-residency work options, a very safe job market for
flexibility” in the variety of places to practice. There are also a
the future, and the personal satisfaction of knowing one’s work
relatively small number of EM physicians in academic practice, so
directly and quickly helps patients, and that one’s work is a
it is often easy for physicians interested in an academic career to
critical component of the national healthcare system.
rapidly advance up the “academic ladder”.
References and Further Reading, click here.
Another positive aspect for EM is that in most countries it is
projected to be an undersupplied specialty for many years, and
so there will continue to be many open job opportunities in EM. 
EM has also been shown to be a critical component of any

24
Section 3

Thinking Like an Emergency Physician

Why are we different? How do we differentiate ourselves from other specialties of


by Joe Lex medicine? We work in a different environment in different hours and with different
patients more than any other specialty.  Our motto is “Anyone, anything, anytime.”

“Emergency Medicine is the most While other doctors dwell on the question, “What does this patient have? (i.e.,
“What’s the diagnosis?”), emergency physicians are constantly thinking “What
interesting 15 minutes of every other
does this patient need? Now? In 5 minutes? In two hours?” Does this involve a
specialty.” different way of thinking?

– Dan Sandberg, BEEM Conference, 2014 The concept of seeing undifferentiated patients with symptoms, not diagnoses, is
alien to many of our medical colleagues. Yet we do it on a daily basis, many times
during a shift.  Every time I introduce myself to a patient, I never know which
direction things are going to head. But I feel like I should give the following
disclaimer. Hello stranger, I am Doctor Joe Lex.  I will spend as much time as it
takes to determine whether you are trying to die on me and whether I should admit
you to the hospital so you can try to die on one of my colleagues. You and I have
never met before today.  You must trust me with your life and secrets, and I must
trust that the answers you give me are honest. After today, we will probably never
see one another again. This may turn out to be one of the worst days of your life;
for me, it is another workday. I may forget you minutes after you leave the
department, but you will probably remember me for many months or years,
Audio is available here
possibly even for the rest of your life. I will ask you many, many questions. I will do

25
the best I can to ask the right questions in the right order so that I For the most part, this has not changed.  And Lewis Thomas
come to a correct decision. I want you to tell me the story, and for wrote: “The great secret of doctors, learned by internists and
me to understand that story, I may have to interrupt you to clarify learned early in marriage by internists’ wives, but still hidden from
your answers. Each question I ask you is a conscious decision on the public, is that most things get better by themselves. Most
my part, but in an average 8 hour shift I will make somewhere things, in fact, are better by morning.”  Remember, you don’t
near 10,000 conscious and subconscious decisions – who to see come to me with a diagnosis: you come to me with symptoms.
next, what question to ask next, how much physical examination
You may have any one of more than 10,000 diseases or
should I perform, is that really a murmur that I am hearing, what
conditions, and – truth be told – the odds of me getting the
lab study should I order, what imaging study should I look at now,
absolute correct diagnosis are not good. You may have an
which consultant will give me the least pushback about caring for
uncommon presentation of a common disease or a common
you, is your nurse one to whom I can trust the mission of getting
presentation of an uncommon problem. If you are early in your
your pain under control, and will I remember to give you that work
disease process, I may miss such life-threatening conditions as
note when it is time for you to go home?  So even if I screw up
heart attack or sepsis. If you neglect to truthfully tell me your
just 0.1% of these decisions, I will make about ten mistakes
sexual history or use of drugs and alcohol, I may not follow
today. I hope for both of our sakes you have a plain, obvious
through with appropriate questions and come to a totally
emergency with a high signal-to-noise ratio: gonorrhea, a
incorrect conclusion about what you need or what you have.
dislocated knee cap, chest pain with an obvious STEMI pattern
on EKG. I can recognize and treat those things without even The path to dying, on the other hand, is rather direct – failure of
thinking. If, on the other hand, your problem has a lot of respirations, failure of the heart, failure of the brain, or failure of
background noise, I am more likely to be led down the wrong metabolism.
path and come to the wrong conclusion. I am glad to report that
the human body is very resilient. We as humans have evolved You may be disappointed that you are not being seen by a
over millennia to survive, so even if I screw up, the odds are very, “specialist.”  Many people feel that when they have their heart
very good that you will be fine. attack, they should be cared for by a cardiologist.  So they think
that the symptom of “chest pain” is their ticket to the heart
Voltaire told us back in the 18th century that “The art of medicine specialist. But what if their heart attack is not chest pain, but
consists of amusing the patient while nature cures the disease.” nausea and breathlessness; and what if their chest pain is aortic
26
dissection? So you are being treated by a specialist – one who these interruptions derail me from doing what is best for you
can discern the life-threatening from the banal, and the cardiac today.
from the surgical. We are the specialty trained to think like this.
I will use my knowledge and experience to come to the right
If you insist on asking “What do I have, Doctor Lex?” you may be decisions for you. But I am biased, and knowledge of bias is not
disappointed when I tell you “I don’t know, but it’s safe for you to enough to change my bias. For instance, I know the
go home” without giving you a diagnosis – or without doing a pathophysiology of pulmonary embolism in excruciating detail,
single test. I do know that if I give you a made-up diagnosis like but the literature suggests I may still miss this diagnosis at least
“gastritis” or “walking pneumonia,” you will think the problem is half the time it occurs.
solved, and other doctors will anchor on that diagnosis, and you
And here’s the interesting thing: I will probably make these errors
may never get the right answers.
whether I just quickly determine what I think you have by
Here’s some good news: we are probably both thinking of the recognition or use analytical reason. Emergency physicians are
worst case scenario. You get a headache and wonder “Do I have notorious for thinking quickly and making early decisions based
a brain tumor?” You get some stomach pain and worry “Is this on minimal information (Type 1 thinking). Cognitive psychologists
cancer?” The good news is that I am thinking exactly the same tell us that we can cut down on errors by using analytical
thing. And if you do not hear me say the word “stroke” or reasoning (Type 2 thinking). It turns out that both produce about
“cancer,” then you will think I am an idiot for not reading your the same amount of error, and the key is probably to learn both
mind to determine that is what you are worried about. I types of reasoning simultaneously.
understand that, no matter how trivial your complaint, you have a
After I see you, I will go to a computer and probably spend as
fear that something bad is happening.
much time generating your chart as I did while seeing you. This is
While we are talking, I may be interrupted once or twice.  See, I essential for me to do so the hospital and I can get paid. The
get interrupted several times every hour – answering calls from more carefully I document what you say and what I did, then the
consultants, responding to the prehospital personnel, trying to more money I can collect from your insurance carrier. The final
clarify an obscure order for a nurse, or I may get called away to chart may be useless in helping other health care providers
care for someone far sicker than you.  I will try very hard to not let understand what happened today unless I deviate from the clicks
and actually write what we talked about and explained my
27
thought process.  In my eight hour shift today I will click about telling of my wondrous career, I quickly stop short and tell
4000 times. myself “You will just be adding more blather to what is already out
there – what you have learned cannot easily be taught and will
What’s that? You say you don’t have insurance? Well, that’s okay
not be easily learned by others. What you construe as wisdom,
too. The US government and many other governments in the
others will see as platitudes.”
world have mandated that I have to see you anyway without
asking you how you will pay. No, they haven’t guaranteed me any As an author, Norman Douglas once wrote: “What is all wisdom
money for doing this – in fact, I can be fined a hefty amount if I save a collection of platitudes.  Take fifty of our current proverbial
don’t. A 2003 article estimated I give away more than $138,000 sayings– they are so trite, so threadbare.  None the less they
per year worth of free care related to this law. embody the concentrated experience of the race, and the man
who orders his life according to their teachings cannot be far
But you have come to the right place. If you need a life-saving
wrong.  Has any man ever attained to inner harmony by
procedure such as endotracheal intubation or decompression
pondering the experience of others?  Not since the world began! 
needle thoracotomy, I’ll do it.  If you need emergency delivery of
He must pass through fire.”
your baby or rapid control of your hemorrhage, I can do that too. 
I can do your spinal tap, I can sew your laceration, I can reduce Have you ever heard of John Coltrane? He was an astonishing
your shoulder dislocation, and I can insert your Foley catheter. I musician who became one of the premiere creators of the 20th
can float your temporary pacemaker, I can get that pesky foreign century. He started as an imitator of older musicians but quickly
body out of your eye or ear or rectum, I can stop your seizure, changed into his own man. He listened to and borrowed from
and I can talk you through your bad trip. Miles Davis and Thelonious Monk, African music and Indian
music, Christianity and Hinduism and Buddhism. And from these
Emergency medicine really annoys a lot of the other specialists. 
disparate parts he created something unique, unlike anything ever
We are there 24 hours a day, 7 days a week. And we really expect
heard before. Coltrane not only changed music, but he altered
our consultants to be there when we need them. Yes, we are fully
people’s expectations of what music could be. In the same way,
prepared to annoy a consultant if that is what you need.
emergency medicine has taken from surgery and pediatrics,
I have seen thousands of patients, each unique, in my near-50 critical care and obstetrics, endocrinology and psychiatry, and we
years of experience. But every time I think about writing a book

28
have created something unique. And in doing so, we altered the
world’s expectations of what medicine should be.

Now, how can I help you today?

References and Further Reading, click here.

29
Chapter 2

Emergency
Medicine
Clerkship: Things
to Know
Section 1

The Importance of The Emergency Medicine Clerkship

Introduction
by Linda Katirji, Farhad Aziz, Rob Rogers The Emergency Medicine (EM) clerkship typically takes place during the fourth
year of medical school. However, some programs may have an optional elective
during the third year. Whether or not you plan to specialize in Emergency
Medicine, the rotation is an important aspect of your medical education. The
emergency room is a unique learning environment which is different than any other
setting in the hospital. It provides clinical opportunities that are largely unavailable
in other clerkships and rotations. During residency, many specialties will also
spend a significant amount of time in the Emergency Department (ED). This may
be within a structured EM rotation, or while admitting or seeing patients for a
certain medical or surgical service. Therefore, it is important to gain an
understanding of the flow of the ED as well as the particular thought process that
must be employed with emergency department patients. This chapter will discuss
some of the unique aspects of the emergency department, some of the skills to
acquire during the EM clerkship, as well as how to best be successful and take the
most away from your rotation.

Unique Aspects of The Emergency Department


Environment
The high volume and acuity of patients in the ED create a time pressure and forces
Audio is available here
physicians to employ a different style of practice than in other settings. A steady

31
stream of patients, some of whom may require immediate life- s y n d r o m e s , a o r t i c dissection, pulmonary embolism,
saving measures, means that many times there is little to no time pericardial tamponade, pneumothorax, esophageal rupture.
to review history or any medical records prior to seeing the Additionally, ED doctors have to use a different thought process
patient. A majority of the time you will need to assess a patient in determining the disposition, or outcome, of the patient. The ED
without knowing anything about their background.  Therefore, it is doctor essentially wants to avoid sending a patient home that
important to gain an understanding of what the most important should not go home, where as a consultant, or admitting service,
pieces of information to gather are for each patient.  This can be does not want to admit a patient who shouldn’t be admitted. This
difficult since most patients will arrive with completely may seem trivial however this difference in thought process can
undifferentiated complaints. Some common examples of these occasionally create tension between the ED and admitting
undifferentiated complaints are “chest pain” and “abdominal services.
pain”, where the etiology can range from completely benign to
Teaching in the ED is different than most other settings in the
immediately life-threatening, or “weakness”, where the differential
hospital as well. There is usually no time set aside for formal
diagnosis includes essentially the entire spectrum of medical
rounds, so most teaching is done at the bedside or at the time the
pathology.
student or resident presents the patient. Many times, attending
This undifferentiated patient is the standard in the ED. However, physicians will pick out “teaching points” for each patient. Each
they can present in any medical setting. It is important to learn physician will have a different teaching style, and your learning
the thought process and develop a strategy for thinking through will, in general, be more active than passive.
these types of patients whether or not you plan on a career in EM.
Lastly, the ED is a great place for medical students and first-year
Emergency Physicians (EPs) must employ and master a
residents to learn to take responsibility for their patients. Students
completely different style of practice than most physicians. EPs
often time have an increased level of autonomy compared to
must always think worst case scenarios for each chief complaint
other rotations. Many times, the student will be the first person to
and must be knowledgeable and comfortable with associated the
assess the patient, which is a very important role. It is important
workup and treatment. A good example of this is chest pain. Even
to learn to distinguish whether a patient is “sick” or “not sick”,
though many times the complaint of “chest pain” is found to be
and whether or not at first glance you think this patient could go
caused by a non-acutely life threatening etiology, EPs must
immediately think of six fatal causes of chest pain: acute coronary

32
home or needs to be admitted not matter what the diagnosis may variety of procedures. Whether you intend to pursue a
be. career in pediatrics, internal medicine, orthopedics, general
surgery or any other specialty, your rotation through the ED will
Unique Skills To Take Away From EM expose you to a wide array of procedural skills ranging from
Clerkships intubating and placing central lines and arterial lines in the
Emergency medicine is a wonderful rotation that exposes you to critically ill to performing lumbar punctures and fracture
different patient populations but also a variety of pathology. This reductions in children. Autonomy is encouraged with procedures,
diverse collection of patients and pathology lends emergency and you will have the opportunity to improve your skills and
medicine residents and students a unique opportunity to gain a techniques under the guidance of residents and attendings. EM is
mastery of different skills. These skills range from a knowledge of a very hands on specialty. You should take advantage of medical
how to approach critically ill patients, gaining procedural skills, student and resident didactics as a  which may include procedure
reading radiographs and CT scans and performing ultrasounds labs on mannikins or cadavers and simulation. This will give you
and much more. an opportunity to practice and provide better patient care during
your rotation.
Often you may be busy doing different tasks when you have to
drop everything and manage a critically ill patient. This is one of In addition to becoming familiar with a wide array of procedures,
the exciting aspects of emergency medicine. These patients offer your EM clerkship will also allow you to familiarize yourself with a
students a great opportunity to learn the principles of variety of imaging modalities ordered in the ED. There is a
resuscitation, such as managing airways and circulatory collapse, tremendous amount of pathology found in the ED which lends
identifying causes for the patient’s decompensation, and itself to a range of imaging. Whether it be learning to perform
instituting the appropriate treatment. Whether you pursue a bedside ultrasonography on a crashing patient or simply learning
career in emergency medicine or choose to pursue a different how to approach a chest x-ray or a CT scan of the abdomen,
specialty, critically ill patients will always be a part of your patient your EM rotation will give you plenty of opportunities to become
population. Understanding how to approach and stabilize these proficient in a skill you will need later in your career.
patients is an important part of being a physician.
Though your EM clerkship gives you exposure to a tremendous
Though learning the art of resuscitation is a vital part of the EM amount of skills which will help you become a savvy doctor, no
rotation, this is also an opportunity to gain competence in a
33
skill is more important than compassion and humility. Everyday k n o w a n d t a k e t h e opportunity to learn how to diagnose,
you will meet patients on the worst day of their life. Realizing this treat, and manage your patient.
and comforting them and their families is paramount to your
The best way to build confidence during your EM rotation is to
success as a physician. You will also encounter a variety of
gain experience and knowledge. Try to be proactive in learning
consulting physicians. Some are nice and professional while
new procedures or treatments with attending or resident
others are not. Having a general understanding that they all have
assistance. Additionally, it’s very important to keep up with
knowledge that you can learn from will set you up for a successful
reading and studying. In the Emergency Room, you may see
career in medicine.
medical pathology you’ve only read about and will be expected to
How To Be Successful on Your EM Clerkship know how to diagnose and treat these diseases properly.
Many of the of the same qualities that allow you to be successful
Good communication is essential for a physician in any specialty,
in other rotations will help you to be successful in the ED.  It is
and in the ED, it is an imperative skill to have.  You will be working
important to be hardworking, proactive, and knowledgeable.
with a large team of nurses, technicians, consulting doctors,
Keep a close eye on your patients, re-evaluate them frequently,
social workers, and paramedics, just to name a few. When you
and make sure to follow up on any results, including labs,
see a patient, it is a good idea to speak with the nurse before you
imaging, and any recommendations by consultants. The unique
enter the room to gain a better understanding of the patient’s
aspects of the ED and EM clerkship discussed previously mean
complaint, as well as gather any information that was relayed by
the first few shifts may be stressful and seem hectic. For every
EMS. By communicating the plan of care to the nurse and
student and resident that rotates through the ED there is a
supporting staff, you will not only improve patient care and
significant learning curve – with each shift you spend in the ED,
reduce mistakes but also forge relationships that will enrich your
things will feel less and less daunting. It’s important during this
experience in the ED. In acute settings such as a patient code or
time that you know your limitations and what you are comfortable
rapidly decompensating patient, good communication with the
and not comfortable with. Many times you will be the first person
entire team is critical. As a medical student or rotating resident,
to assess the patient. You should have a low threshold for alerting
this is a great time to practice and improve your communication
an upper-level resident or attending if the patient appears to be
skills in these acute settings under the direction of residents and
sick, or if they present with a complaint you are uncomfortable
attendings.
with. At the same time, you should be confident in what you do

34
Your EM rotation will be an exciting, unique experience during
medical school and residency. Whether you plan to specialize in
EM or not, you will learn many procedural skills, improve your
own method of diagnosing and treating patients and be able to
practice a different method of medical decision making.

References and Further Reading, click here.

35
Section 2

Medical Professionalism

Introduction
by Amila Punyadasa It is prudent to commence this chapter with some relevant definitions. A
profession is a specific type of occupation, one that performs work with special
characteristics while competing for economic, social, and political rewards. A
“The Dimensions That All Medical professional, it follows, is a person who belongs to a group (profession) which
Students Should Know About” possesses specialized characteristics (specifically, knowledge, skills, and
attitudes) that have been obtained after a long period of study and are used to
benefit other members of society. Thus, professionalism is used to describe those
skills, attitudes and behaviors. We expect from individuals during the practice of
their profession and includes such concepts as maintenance of competence,
ethical behavior, integrity, honesty, altruism, service to others, adherence to
professional codes, justice, respect for others and self-regulation. In fact, it has
been said that professionalism serves as the basis of the medical professions’
relationship to society and that this relationship is a social contract, underpinned
by professionalism. Medical professionalism thus comprises physicians’ behaviors
that demonstrate they are worthy of the trust the public and patients place in
them.

In essence, our profession involves healing. Kirk (2007) eloquently stated that in
any patient encounter, we consider both a right and good healing strategy for our
patients. The right action is informed by evidence-based medicine, while the good

36
action incorporates the patient’s values and preferences aligned Although there are many definitions of professionalism, most
with the physician’s judgment (or, in other words, requires the contain a list of responsibilities that the physician should exercise;
physician to exercise various dimensions of professionalism). they are based on three fundamental principles:

Interestingly, this ‘judgment’ itself has three concrete steps: 1. The principle of primacy of patient welfare – This is based on a
dedication to serving the interest of the patient with altruism; in
1. The diagnostic question – What is wrong with the patient? –
turn, contributing to trust, the core of the doctor-patient
Incorporates both the clinical assessment and investigations
relationship. This principle should be unsullied by economic,
the patient was subjected to.
administrative or sociocultural exigencies.
2. The therapeutic question – What can be done for the patient? –
2. The principle of patient autonomy – The patient’s autonomy
Informed by evidence and may involve a plethora of treatments
must be respected, and doctors should not only be honest
and interventions.
with their patients but also empower them to make informed
3. The prudential question – What should be done for the patient? decisions about their healthcare, as long as these are within
– This involves the patient in the final decision-making process, the ethical practice and do not lead to demands for
preserves patient autonomy, and ensures a patient-centric inappropriate care.
approach to healthcare provision.
3. The principle of social justice – This includes the fair
The advantages of teaching students to practice professionalism distribution and access to health care resources and the
include imparting a greater sense of purpose, building a elimination of discrimination, whether that be racial, gender-
framework for harmonious and efficacious healthcare provision, based, religious, socio-economic or any other social category.
as well as building trust and mutual respect. Additional benefits
The definition proposed by Wilkinson et al. (2009) is both
include improving patient satisfaction, reducing complaints and
thorough and explicit. His list of specific behaviors necessary for
litigation, improving treatment compliance, and improving clinical
medical professionalism is enumerated. Also incorporated in the
outcomes.
appendix is the Emergency Medicine Clerkship’s core curriculum
learning objectives that pertain to professionalism.

• Honesty/Integrity
37
• Confidentiality • Balancing availability to others with care of ones-self

• Moral reasoning and ethical decision making • Reliability

• Respecting privileges and codes of conduct • Accountability/task completion

• Effective Interactions with Patients and Their Relatives • Punctuality

• Respect for diversity • Assumes responsibility and is conscientious

• Politeness/Courtesy/Patience • Self-Reflectiveness

• Manners/ Demeanor • Recognizing limits

• Patient-centered and involved decision-making process • Life-long learning

• Maintenance of professional boundaries • Dealing with uncertainty

• Balancing availability to others with care of ones-self • Teaching and debriefing

• Effective Interactions with Other Health Care Workers • People management

• Teamwork • Leadership

• Respect for diversity • Using appropriate strategies to improve processes

• Politeness/Courtesy/Patience • Advancing knowledge and one’s field (e.g. via Research)

• Maintenance of professional boundaries Seven deadly sins of professionalism or professional non-virtues.


They are the following:
• Manner/Demeanor – This includes maintaining a professional
appearance. 1. Greed – with respect to money, power, and fame.

38
2. Abuse of power – with respect to colleagues, patients, and explicitly taught but also explicitly tested (or assessment
position in the hierarchy. driven learning and practice). Other motivators include
bestowing rewards and recognition for demonstrating positive
3. Arrogance – towards patients or colleagues
behaviors, a form of positive reinforcement.
4. Conflict of Interest
2. Observing role models are of the utmost importance. This
5. Misrepresentation – for example, lying or being fraudulent. entails not only the incorporation of good behaviors observed
but also the recognition of negative behaviors exhibited by
6. Apathy – pertaining to lack of commitment, irresponsibility or poor role models and purposefully not engaging in such
doing the bare minimum for patients. behaviors. I shall delve into this concept in more detail in part 4
below.
7. Impairment – secondary to illness, alcohol or drugs.
3. Continued exposure to aspects of professionalism is important
Many of the core criteria of professionalism are related to the
to inculcate its tenets. Vertical integration into a spiral
assimilation of good old-fashioned virtues and the development
curriculum, in my opinion, is required to achieve this goal.
of soft skills that must not only be taught but also reinforced
through modeling and active practice. When devising any 4. Reflection and feedback are keys. Self-reflection on aspects of
professional curriculum, take careful consideration of common professionalism, as well as timely and effective feedback of
themes that positively impact necessary behavioral changes. specific behaviors from peers and seniors, will help mold the
Some of those themes include the following: medical student into doctors that demonstrate professional
behaviors. Good feedback, based on observable behaviors,
1. Motivation (or Getting “Buy-In”)
explains not only what should be done but also why it should
a. Intrinsic – Medical students must be convinced of the be done, and both are essential for effective learning.
importance of the desired change in behaviors for it to be a
The importance of teaching medical professionalism to
driving force.
undergraduates is well documented in the literature and is integral
b. Extrinsic – The principal extrinsic motivator for medical to the medical profession. What is perhaps less clear is exactly
students is the knowledge that professionalism will not only be how this teaching should be conducted. The solution lies in

39
understanding how to utilize all aspects of the curriculum between knowledge-based teaching and in-situ experiential
including the formal, informal, and hidden curriculum. learning.

Strategies for Teaching the Formal In my personal undergraduate and postgraduate training years,

Curriculum there has been a scarcity of teaching about professionalism,


The following involves a discussion of the teaching of the “formal especially in the formal curriculum. Professionalism is not a
curriculum,” which is defined as the stated, intended, and gimmick but a set of rules and behaviors that each of us must
formally offered and endorsed curriculum. adhere to and practice; it forms the very foundation of good
clinical practice. Over the last 15 years, the undergraduate
Maudsley & Strivens (2004) have proposed that the ‘situated curriculum has increasingly incorporated medical professionalism,
learning’ theory seems to describe the most effective model to explicitly, as part of their formal curriculum; they do so with the
imbibe the virtues of professionalism. It suggests that learning application of both vertical and horizontal integration. This is the
should be embedded in authentic activities which help to way forward.
transform knowledge from the abstract and theoretical to the
usable and useful. Brown et al. (1989) further noted that there The judicious instructor has a plethora of instructional modalities
should be a balance struck between the explicit teaching of a to choose from. The different teaching/learning strategies and
subject and activities where the implementation of such tools that may be utilized to achieve competence across the
knowledge is utilized in an authentic context. many dimensions of medical professionalism successfully.

Furthermore, there have been two principle approaches • Problem-based reflective practice
described in the teaching of professionalism.
• Role-modeling
• Explicitly Teaching the Basics of Professionalism
• Portfolio based training
• Experiential Learning
• Clinical contacts with tutor debriefs
One should utilize both of these approaches. In fact, to
• Simulation-based training
paraphrase the situated learning theory, a balance must be struck
• Didactics and tutorials
40
The Informal and Hidden Curricula in Medical curriculum. For example, empathy among medical students

Professionalism was seen to decrease as they progress through medical school.


The Informal and hidden curricula are, as the terms themselves
Such surprising findings are noted despite increased emphasis
suggest, not only cloaked in mystery but are also crucial elements
being placed on the teaching of the formal curriculum. Thus, it
of medical professionalism. The informal curriculum is defined as
becomes clear that these discrepancies may be attributed to the
an unscripted and ad hoc yet highly interpersonal form of
influence of the ‘hidden’ curriculum. Interestingly, some students
teaching and learning that takes place between faculty members
believe that certain components of their learning could only be
and students in non-classical teaching settings. Examples of
achieved through the informal and hidden curriculum and that the
these ‘settings’ could include tearooms during clinical breaks or
science of medicine is associated mainly with the formal
even a certain coffee house. Over coffee, I recently taught my
curriculum while the art of medicine is associated mainly with the
4th-year elective student about medical professionalism themes,
informal and hidden curricula.
including respect for the patient and their autonomy, citing
anecdotes from my experience. It is well known that medical students acquire soft skills such as
communication techniques and medical etiquette, both important
The hidden curriculum, on the other hand, is a set of influences
facets of professionalism, from observing mentors, peers, and
that function at the level of organizational structure and culture.
other healthcare workers.
These two components are interrelated; In fact, some authors
refer to both the informal and hidden curricula as one entity. Other modalities of learning via the informal and hidden
Hence, it is clear that in order to acquire the knowledge, skills, curriculum are
and attitudes of professionalism, students must use a wide range
• Rituals
of learning strategies that extend far beyond the intended formal
curriculum. These strategies include interactions with teachers, • Infrastructure
colleagues and various other people around them.
• Chance Observations
Research has shown a distressing downtrend in professionalism,
which has been directly attributed to the influence of a hidden It is quite clear to me that the formal, informal and hidden
curricula are all complimentary. However, there is, unfortunately,

41
an ongoing conflict between the formal curriculum and the Classically, a role model is someone who is admired for the way
informal/hidden curriculum. he acts and for his professionalism and whose behavior is
considered as a standard of excellence to aspire to.
An obvious remedy is to engage the various stakeholders
involved in training medical students in a constructive dialogue on It is important to show students what right practice is, and that
how the hidden and informal curricula can be manipulated to applies to both clinical and professional conduct. This is the
influence student learning positively. This understanding will not essence of role modeling.
only help avoid the visible conflict between formal curriculum and
Paice et al. (2002) described the act of being a role model as
informal/hidden curricula but will also extract the advantages of
serendipitous, a beneficial but chance outcome. I respectfully
the informal/hidden curriculum to produce better physicians.
disagree. Senior tutors and physicians all act as role models and
It is undeniable that medical school faculty, both senior and junior must be cognizant of everything we do in front of our students.
doctors, and other healthcare workers are all role models who Knowing that we will be observed and scrutinized should make us
may influence medical students’ learning. The professionalism ultra self-conscious, and we should try hard to showcase and
demonstrated by all these people is of great importance not only inculcate the virtues of sound clinical practice and
for their patients but also for the next generation of doctors. professionalism at every opportunity.
Hence, we have to keep paramount in our minds that our practice
The vast majority of the literature is in agreement that role
and interpretation of professionalism, and all its dimensions, is
modeling is not only important but also integral to medical
keenly being observed by our students and that we have a huge
education. Role models not only affect the attitudes, behaviors,
role to play in the development and molding of their moral and
and ethics of medical students but also imbibe professionalism in
professional wellbeing.
trainees. I am sure we can all recall a specific role-model that
Role Modelling in Medical Professionalism impressed upon us the virtues of professionalism while
This connects back to the most powerful tool to teach demonstrating punctuality, responsibility, honesty, ethical
professionalism, role modeling. Role modeling involves a reasoning, accountability, collegiality and patient-centric
physician (or role model) who teaches a student by example; its management while embracing diversity with a sense of decorum.
importance is unquestionable and has been documented for Such role models also influence career choices of students and
many years. function in the formal, informal and hidden curricula. However,
42
drawbacks have also been described. Sinclair (1997) wrote that • Collated views of co- workers
he noted medical students being drawn to and indeed emulate
• Simulation
senior doctors who held positions of responsibility and status. He
further noted a warning of their professional ideals and behaviors • Paper tests
as they evolved.
• Patient opinions
Assessment Techniques in Medical
• Ratings by a Superior
Professionalism
Unfortunately, despite the unquestionable importance of • Self-assessment
professionalism to the everyday functioning of every medical
doctor and student, my experiences (spanning two decades and • C r i t i c a l i n c i d e n t re p o r t / R e c o rd s o f i n c i d e n t s o f
three countries) with its assessment has been rather limited. In unprofessionalism
fact, during my postgraduate years of clinical practice, the
assessment of professionalism has been rather rudimentary, with
Social Media and Professionalism
It seems like nearly everyone, certainly from the Generations Y
its evaluation often subordinate to the assessment of clinical
and Z, is using Facebook or Twitter these days for one reason or
competencies.
another. Although not a fan myself, I do concede that when used
If we are to take the assessment of professionalism seriously, with prudence, social media and the Internet is an invaluable
then we must improve our framework for assessment. resource for teaching and learning. It can support physicians’
Specifically, we need to implement a number of different methods personal expression, improve camaraderie and improve the
to effectively measure all levels of Miller’s pyramid, while also dissemination of public health messages. Equally, it risks
covering the multidimensional breadth of professionalism. broadcasting unprofessional content online that reflects poorly on
individuals, their affiliated institutions, and the medical profession
I shall now consider some assessment tools that will enable the alike.
ability to assess the multidimensionality of medical
professionalism. These are; For example, let us consider a hypothetical tweet from a female
doctor to her colleague describing a recent patient: ‘Just saw an
• Assessment of an Observed clinical encounter 18-year-old unmarried G5P0, with Chlamydia, herpes, and
43
gonorrhea. Disgusting!’ This tweet would have contravened a few accurate and appropriate. With regards to interaction with
of Wilkinsons (2009) so-called ‘behaviors inherent to good patients through social media, again, this interaction should fall
medical professionalism.’ This doctor should have had “respect within the boundaries of established professional norms. If a
for her patients’ diversity” and shouldn’t have been so judgmental physician feels that such an interaction transgresses such norms,
(in this case, about the patients alleged sexual promiscuity and he/she should report the matter to the relevant authorities. Finally,
lifestyle). She also should have upheld patient confidentiality (as it is imperative that physicians realize that inappropriate online
although the patient’s name wasn’t tweeted, the descriptors used interactions may have a negative impact on their reputations and
about her obstetric and sexual histories would surely have made that of their institutions, career advancements, and, perhaps most
her easily identifiable amongst her friends and family who might damning, may serve to undermine public trust in the medical
have come across this tweet). The doctor should have, in my profession as a whole.
opinion, had better regard for professional boundaries and
References and Further Reading, click here.
exercised greater judgment and discretion.

Defining unprofessionalism online and policing it has been


challenging. However, with the increase in awareness of such
occurrences, regulatory bodies have published various
documents in an attempt to regulate physician’s activities on
social media sites. The General Medical Council (GMC) has
attempted to do exactly this with its paper. It warns against the
blurring of boundaries between ones public and private lives and
advices that privacy on these sites cannot be guaranteed.
Furthermore, it stresses that physicians must be careful with
regards to patient confidentiality, elaborating that although one
piece of information may not breach confidentiality by itself,
together, a few may certainly do so. In summary, physicians must
be cognizant of patient confidentiality and privacy and monitor
their Internet presence to ensure that information posted is both

44
Section 3

Communication and Interpersonal Interactions

Introduction
by Vijay Nagpal and Bret A. Nicks Emergency Medicine and the situations within the department can present a
stressful, rapidly changing environment where it may feel as though there is too
little time for effective patient communication, patient-centered care or the
opportunity to establish an appropriate provider-patient relationship. It is also an
environment unlike any other in medicine, where a unique team of individuals faces
varying degrees of chaos with limited available information to work together and
address the medical conditions of those presenting to the department. Few would
recommend entering such an environment in the absence of an established care
process and means of clear communication. The tone of the department is set
prior to walking into the ED; from the moment you walk into the department,
preconceived notions and prejudices remain at the door.

It is no surprise that high-functioning emergency departments have high-


performing, well-communicating teams. Clearly defining and communicating to
every team member why we are there and how we care for patients sets the tone
for every interpersonal interaction. This is true not just for our patient-provider
interactions but our interactions with the nursing, ancillary and consultant staff as
well (Gluyas, 2015). Establishing a team mentality and acknowledging the value of
contributions our colleagues and staff bring to the ED is essential to practicing
high-quality, safe emergency medical care. Additionally, the skill set that those in

45
other health professions bring to the team can help us to look while you may not be able to solve the patient’s condition or
from a different perspective to better understand our patients and chronic illness, using effective communication skills and providing
facilitate the best care that can be offered in the ED (Klauer & a positive patient experience will assuage many patient fears
Engel, 2013). (Mole, 2016). Keep in mind, in general, patients remember less
than 10% of the content (what was actually said), 38% of how
Essentials of Communication you say it (verbal liking), and 55% of how you look saying it (body
The approach to providing quality patient care in the ED starts language) (Helman, 2015).
with recognizing the patient-provider mismatched perspective on
what has happened and what is occurring (Helman, 2015). It is Effective provider communicators routinely
essential to recognize the patient-physician relationship starts employ these 5 Steps
with a significant power imbalance. Attempts should be made to 1. Be Genuine
normalize or reduce this power imbalance, to empower the
patients and their families. This will enable an open discussion We know it. People can sense the disingenuous person – whether

about their medical concerns and assist in making informed it is a gut feeling or through other senses. Try to see the situation

decisions about their care. It is important to acknowledge the wait from the patient’s perspective, and it will ensure that you are

or process they have already endured before seeing you. Thank acting in his best interest and with integrity.

the patient (and family) for coming to the ED and allowing you to
2. Be Present
address their medical concerns. Also, take the time to introduce
yourself to everyone in the room with the patient and find out who As emergency providers, we are interrupted more than perhaps
they are in relation to the patient. This can help establish rapport any other specialty. However, for the few moments that we are
with the patient and those around them (Chan 2012, Cinar 2012, engaged with the patient or his family, be all in. If there is a
Hobgood 2002). planned interruption upcoming, make it known prior to starting a
discussion. Be focused on them and the conversation; value what
While many believe the environment of care is the greatest
they have to share. At the end of your encounter, briefly
limiting factor as opposed to quality communication, literature
summarizing what the patient has told you can help to reassure
would suggest otherwise. Establishing a positive patient-provider
the patient that you were listening and also give them the chance
relationship is essential for patient care. One must recognize that
to clarify discrepancies.

46
3. Ask Questions additional information. Doing this also allows the patient to
be more involved in his care and ask further questions regarding
To effectively communicate, one must listen more than he talks.
his workup and treatment plan. Additionally, helping the patient to
After introducing yourself, inquire about the patient’s medical
understand what to expect while in the department can help to
concern; give them 60 seconds of uninterrupted time. Most
alleviate fear associated with unannounced tests or imaging
patients are amazed and provide unique insights that would
studies, especially when these tests may require him or her to be
otherwise not be obtained. Once the patient has provided you
temporarily taken out of the department (e.g., a trip to the CT
with his concerns, begin asking the specific questions needed to
scanner).
further differentiate the care needed. By asking questions and
allowing for answers, you make it about them and give them an Many of these concepts have been identified in patient
avenue to share with you what they are most concerned about, satisfaction and operational metrics. In one study, wait times were
enabling you to address those concerns. not associated with the perception of quality of care, but empathy
by the provider with the initial interaction was clearly associated
4. Build Trust (Helman, 2015). In addition, patient dissatisfaction with delays to

Given the nature of the patient-provider relationship in emergency care is less linked to the actual time spent in the ED and more

medicine, building trust is essential but often difficult. Building with a to set time expectations about the care process, a

trust is like building a fire; it starts with the initial contact and perceived lack of personal attention, and a perceived lack of staff

builds with each interaction. Trust is also built on engaging in communication and concern for the patient’s comfort.

culturally acceptable interactions (Chan, 2012) such as a


Empathy
handshake, affirming node, hand-on-shoulder, or engaging
posture. In the ED, it is essential to understand that much of a patient’s
care relates to empathy – the ability to understand and share
5. Communicate Directly another person’s experiences and emotions. It is recommended

Ensure that at the end of your initial encounter you have to try and understand the patient’s agenda. One can accomplish

established a clear plan of care, what the patient can expect, how this by asking, “Help me understand what brought you in today.”

long it may take, and when you will return to reassess or provide “Help me understand what I can do for you.” “Tell me more.” This

47
will help to normalize the patient’s situation and gain unique • Take Action – discuss and define the care steps (and what to
insights into his care concerns. expect)

There are four easy steps to improve reflective listening and • Express Appreciation – thank the patient for allowing you to
perceived empathy in the ED: care for them

1. Echo – Repeat what the patient says; this gives the message The Approach
that you heard the patient. As with many life circumstances, effective communication is the
glue that helps establish connections to others and improve
2. Paraphrase – Rephrase what the patient says as this gives the
teamwork, decision-making, and problem-solving. It facilitates the
message that you understand the patient.
ability to communicate even negative or difficult messages
3. Identify the feeling – Say, for example, “you seem frustrated,” without creating conflict or distrust. Recognizing this helps
“worried,” “upset.” This produces trust. provide the best foundation and approach for successful patient
communication, an essential element in the ED. In addition to
4. Validation – Validate the patient’s feelings verbally by saying
understanding the five steps of effective communication, ones
statements such as “I can see why you feel that way.”
approach to effective communication must also be guided by the
There is also a great online module and mnemonic for Empathetic individual patient and adjusted accordingly. So, consider seeing
Listening skills development (SMACC, 2016). The RELATE your approach from the patient’s perspective, and set the tone
mnemonic is: with the following three starting points.

• Reassure – share your qualifications and experience The 3 Starting Points:


1. Approach and Appearance:
• Explain – describe in clear, concise language what the patient
can expect • Dress appropriately

• Listen – not just hearing, encourage the patient to ask questions • Sit down next to the patient

• Answer – summarize what they have said and confirm their • Maintain an open posture (avoid crossing your arms)
understanding
48
• Maintain good eye contact, if culturally appropriate Do
• Let the patient tell his/her story (Roscoe, 2016)
• Smile appropriately, nod affirmingly
• Establish what the patient’s agenda is, what his/her fears are
2. How you speak
• Provide the patient with information regarding what will happen
• Speak slowly and quietly (given the constraints of the ED)
during his/her stay. This puts the patient more at ease and
• Use a low tone in your voice improves satisfaction (Hobgood, 2002).

• Empathy can be heard in your tone • Provide expected wait times. Some experts suggest
overestimating the time for results and consultant services
3. What you say (Disney Technique).

• Introduce yourself in a culturally appropriate manner • Explain the reasons for delays and apologize for it

• Use the patient’s last name (helps to minimize power imbalance) • After your history and physical, map out the next steps in the
process (i.e., establish expectations).
• Acknowledge everyone in the room and ask what their
relationship to the patient is (i.e., shake hands if culturally
Don’t
appropriate) • Fold your arms over your chest as this displays an aggressive
posture
• Adjust medical wording based on patient’s medical literacy
• Ask why the patient did not come in earlier
In addition to understanding the five essentials of communication
and setting the tone for the initial care approach, it is important to • Say, “I guess.”
understand a few of the common reasons communication either
fails or succeeds in the Emergency Department. While a single • Repeatedly ask, “why.”
approach framework doesn’t always fit, there are some essential
• Use the words “never” or “always.”
Do’s and Don’ts that must also be considered.

49
The Difficult Patient Frequent Fliers
When facing difficult patients in the emergency department,
High recidivism may be frustrating, but it is important to
understanding the situation and the motivation for the patient
understand that there may be an underlying reason for frequent
may help to navigate better the communication challenges that
ED visits. Socioeconomics and poor access to care are common
are present. A difficult patient encounter in the emergency
reasons. Knowing the available resources (e.g., social workers,
department can often be frustrating for both the physician and
clinical support nursing) can make a difference.
the patient. These patients often present with chronic medical
issues that are superimposed onto individuals with social Combative/Agitated or Intoxicated Patient
disparities (Hull & Broquet 2007, Dudzinski & Timberlake 2016).
It is most important to keep both the patient and the staff
These are just a few examples of types of patients that one may
(including yourself) safe. Redirecting the patient and emphasizing
encounter in the emergency department:
the importance of caring for them medically may help to calm the
Patient Type and Suggestion situation. Psychopharmacological intervention may be necessary
Angry Patient at times.

Don’t ignore the fact that a patient may be angry or upset – often For a deeper dive into effective patient communication related to
it is related to delays, expectations or care concerns. Try to managing difficult patients, listen to Episode 51: Effective Patient
explore this emotion by asking neutral and non-confrontation Communication – Managing Difficult Patients by Anton Helman.
questions. Acknowledgment and a simple apology for process
issues may prove invaluable. The Handoff
Communication between providers and patient care transitions
Manipulative Patient present one of the well-known challenges in patient care and
errors in care management. This handoff communication, often
While these patients may clearly have a secondary agenda, their
perceived as the “gray zone,” has been characterized by
medical complaints may still be legitimate. Approach these
ambiguity regarding the patient’s medical condition, treatment,
patients with an open mind, but be prepared to say no to
and disposition (Akper, 2007). Communication errors, particularly
requests that are not clinically indicated.
related to patient hand-offs, account for nearly 35% of ED-related

50
care errors. Establishing a standardized process to ensure the saturation at 93%. We are currently attempting to wean O2
quality and clarity of transitions in care are essential. One such requirement as tolerated.
example is the I-CAN format that is specifically focused on the ED
patient population. N – Next Steps and Anticipated Disposition
Describe to the receiving provider what will need to be followed
ED-based Patient Handoff Tool (I-CAN) up and the anticipated disposition of the patient. For example, the
patient will need to be admitted for a COPD exacerbation with a
I – Introduction new O2 requirement. He can go to a floor bed if he remains stable
Briefly describe what brought the patient into the emergency
on nasal cannula.
department today. For example, the patient is a 53 yo male with a
past medical history of COPD who presents today with a While many examples for a unified handoff exist, identifying a
productive cough, wheezing, and shortness of breath. defined approach and establishing the expectation for routine
use, especially when integrated into the electronic health record
C – Critical Content & Interventions Performed at transitions of care, ensure improvement with patient care,
Relate information that helps the receiving provider understand quality, and throughput (Akper 2007, Rouke 2016). If the patient
the ED course taken up to this point. and family are involved with this handoff, not only will they

For example: On initial evaluation, the patient was unable to understand care expectations but the will also better understand

speak in full sentences, and O2 saturation was 88% on room air. issues with delays, next steps, and care updates.

We started him on NIPPV, and Nebulizer treatments were given.


Conclusion
Respiratory burst steroids have been given to the patient here in
Most agree that providing patient care in the ED poses many
the department.
challenges. The situations we work with can present a stressful,
rapid environment where it may feel as though we have too little
A – Active Issues
Give the provider an idea of the patient’s current condition at this time for effective patient communication, patient-centered care or
time. For example, the patient improved with an hour of NIPPV opportunity to establish a great patient experience. However, it is
and was transitioned to high flow nasal cannula with O2 also evident that improved communication between the care
team and patients improves not only the care experience but also
patient care outcomes. Quality communication improves patient
51
outcomes, compliance, and satisfaction – not to mention the job
and team satisfaction. While many techniques exist to improve
ED communication, establishing a culture in the ED to habitually
adapt these practices is essential. The ED is an environment
unlike any other in medicine, where a unique team of individuals
works in varying degrees of chaos with limited available
information together to address the medical conditions of those
presenting to the department. Doing so with effective
communication can make a difference.

References and Further Reading, click here.

52
Section 4

Data Gathering

Introduction
by Chew Keng Sheng Although a medical student has always been taught to take a comprehensive
history and a complete physical examination from head-to-toe, she may find this
methodical approach a challenge in the emergency department (ED). Many of the
patients who come to the ED are often first-time patients, unfamiliar with
procedures, and have diverse complaints ranging from a manipulative attempt to
obtain a sick leave certificate to a complex, life-threatening situation. This
challenge is further compounded by the fact that many patients in the ED are
suffering from acute illnesses or injuries that compromise their cognitive capacity
to comprehend and respond.

The Emergency Medicine Approach


Although some studies have shown that history-taking alone can determine the
diagnosis in up to 75-80% of the cases (Hampton 1975, Peterson 1992) obtaining
such a comprehensive history in the ED can be an extremely daunting task
especially if the patient is extremely ill.

In such a situation, the linear clinical approach – history first, followed by physical
examination and investigation – may not be feasible. Rather, data gathering from
the patient’s history, physical examination, and investigation may need to be
performed concurrently. The most important element in the approach to the
patient in emergency medicine is to establish the composite initial impression of

53
the patient. This is based on data gathering from multi-sources Ask the 5-Ws and 1-H q u e s t i o n s : “ W h a t ? ” “ W h y ? ”
including the history, physical findings, and bedside “Who?” “When?” “Where?” and “How?”
investigations. Of particular importance is answering the vital
question: is there any life or limb threat in this patient? And once Pay particular attention to any symptom developed acutely. Acute

a life or limb threat is identified, immediate measures must be onset of a headache, for example, suggests a vascular origin. If a

initiated to reverse the insult before moving on in the data patient has had a chronic, persistent or recurrent condition, the

gathering process. important question to ask is “Is there any difference between the
symptom before and the symptom now?” A patient with a
Activity 1 migraine headache, for example, can present with a sudden
“worst ever headache” suggestive of subarachnoid hemorrhage
Watch a video podcast on General Approach to the Emergency
rather than a chronic migraine. If we do not ask for the symptom
Department Patient.
pattern changes, the patient may not volunteer this information.
Discuss
What
• What are the strengths and limitations you see in this
What is the message that the patient is trying to convey to me
emergency medicine approach model where all processes of
through the words he does and does NOT use? Observe the non-
data gathering (history-taking, physical examination, and
verbal communication cues that he is trying to convey, e.g., a
investigation) may have to occur simultaneously as compared
sense of nervousness, fidgety movements, etc. Often, patients
to the traditional linear clinical approach?
are prone to conceal sensitive information such as sexual history
As tough as it may seem, a doctor working in the ED must still as well as psychiatric/psychological complaints that may only be
establish a good communication rapport with the patient, as detected through non-verbal cues.
much as possible. To attain this, one must utilize open-ended
Why
questions.
Examples: Why does the patient choose to come in the middle of
the night? Why does the patient choose this form of treatment

54
and not another? Why does the patient think that his or her illness Non-verbal cues
is not serious? Be attentive to the patient’s non-verbal cues as well, not just the
verbal contents of his visit. Albert Mehrabian, a professor of
Who
psychology, developed the classic 7-38-55 rule. This rule consists
Examples: Who is/are taking care of the patient at home? Whom of the following: while 7% of what the patient communicates
does the patient seek advice from when he/she is sick? Who else comes from the actual words used (the content), 38% of the
knows about the patient’s illness? Who is/are the eyewitnesses of message comes from the way it is said (the tone), but 55% of the
the accident or the trauma that the patient was involved in? Who message comes from the non-verbal cues including but not
is the patient’s next of kin? Who can be a legitimate surrogate limited to the facial expression, eye contact, etc.
decision maker for the acutely ill patient?
Does the patient appears fearful and defensive? Aggressive?
When Angry? Disinterested? Click here to watch a video on Mehrabian’s
study.
Examples: When does the pain occur? When does the patient
first notice the swelling, the discoloration, etc.? A sudden onset This is especially so when the patient is trying to communicate
of symptoms is a warning sign and may suggest a vascular event. across sensitive information such as his sexual history or
psychological symptoms. Unfortunately, it was found that only
Where between 20 – 40% of doctors responded positively to the
patient’s verbal and non-verbal cues (Beckman 1984).
Examples: Where did the accident happen? Where does the
patient come from? How far from the hospital? Allow the patient to describe his/her concerns using his own
words without interruptions. It has been found that a doctor
How
interrupts his patients as early as 18 seconds into the
Examples: How did the accident happen? Did the patient lose conversation, even though it takes at least 150 seconds for the
his/her consciousness before or after the event? patients to tell his stories (Beckman 1984).

55
Activity 2 alleviate these emotions of fear and anxiety in their clinical
encounters?
Watch this short video: Presenting your patient to your attending
in Emergency Medicine by Dr. David Pierce 3. The speaker also talked about the long waiting time in the ED.
How does the long waiting time affect your data gathering
Reflect: In the video, Dr. Pierce admonishes his residents not to
process?
miss anything important by thinking of 5 other differential
diagnoses. Why is it especially important to adopt a broad-based References and Further Reading, click here.
approach in diagnoses formulation in the ED?

Activity 3

Watch this video: Approach to the ED Patient.

Discuss/reflect on the following questions:

1. In his talk, the speaker stated that “most patients do not take
going to the ED casually.” How does knowing that most
patients do not take going to the ED casually affect the way
you view your patients, especially in the middle of the night?

2. The second thing that the speaker said is that fear and anxiety
are routine emotions experienced by ED patients. Do you agree
with this statement? If yes, why do you think this is so, and
how would this affect your data gathering process? In your ED
rotation or posting, observe whether it is indeed true that fear
and anxiety are routine emotions experienced by the patients
you see. Do you think the doctors have done enough to

56
Section 5

Diagnostic Testing In Emergency Medicine

Case 1
by Yusuf Ali Altunci A fifty-one-year-old male patient is admitted to your
emergency department (ED) with chest pain that started 30
minutes ago. On his ECG, there are 2 mm ST elevations at DII,
DIII, and aVF derivations. Do you need high sensitive troponin
analyzes results for acute management of this patient?

Case 2
A thirty-five year- old female patient presented to your ED with
sudden onset shortness of breath. She has tachycardia. There
is no pathologic finding at auscultation. Her blood pressure is
90/60 mmHg. In history, there is swelling and pain on her left
leg for two days. She is using oral contraceptives. For this
patient can normal D-dimer result rule out pulmonary
embolism?

57
Introduction rules try to answer; therefore, the diagnosis found through the patient’s
The emergency physicians frequently development of these reliable clinical history and physical examination (Wald
make difficult clinical decisions with decision rules is imperative for the 2011). For emergency management, it is
limited information while encountered advancement of modern emergency usually more important to rule out life
with a multitude of demands and medicine (Pines 2012). threating pathologies.
distractions (Kovacs & Croskerry 1999).
“Listen to your patient; he is So why do we need diagnostic tests? For
EDs are crowded places. Usually, you detecting the problem, of course;
telling you the diagnosis.” –
have limited time to diagnose and treat however, the decision to test is impacted
the patients. Today, diagnostic tools are
William Osler (1849-1919) by multiple factors such as clinical
better than they were in the past. This suspicion, persuasion, physician’s
may help provide an easier diagnostic
Diagnostic Testing Approach decision, consultant’s or patient’s request
Polymorbid patients, different diagnostic
approach, but the difficulty is knowing (Wald 2011).
and therapeutic options, more complex
how and when you should use these
hospital structures, financial incentives, Patients often express strong preferences
tools. Even if the technology has become
benchmarking, and perceptional and for medical tests or treatments of their
available more frequently in clinical
societal changes cause pressure on own choosing, even when physicians
practice, clinical expertise and skills are
doctors, especially if medical errors come believe that those interventions are not
still important factors for making correct,
up. This is especially true for the ED beneficial (Brett & McCullough 2012).
timely diagnoses in patients (Wahner-
structure, where patients encounter Patients are also increasingly willing to
Roedler 2007).
delayed or erroneous initial diagnostic or challenge physicians’ intellectual
So this triggers the question: is there one therapeutic actions and expensive authority, often requesting interventions
diagnostic approach for each emergency hospital stays due to sub-optimal triage based on media publicity about new
illness that can render the best result for (Schuetza 2015) research findings, sometimes before
the patient, maximize timeliness and physicians are even made aware of them.
Diagnostic tests should primarily be
accuracy, and limit cost? This is the Internet sources with clinical information
ordered to rule in or out a particular
essential question that clinical decision also empower patients to make medical
condition based on the differential
58
judgments independent of consultations “Medicine is a science of •Sensitivity = True Positive/(True
with physicians (Brett & McCullough Positive + False Negative)
uncertainty and an art of
2012). The Internet continues to create
new, unschooled Internet doctors and, in probability.” – William Osler Specificity refers to the likelihood of the
test being negative or normal in the
turn, new challenges. (1849-1919)
absence of disease
Chosing the test or not test in the ED also
Statistics • Specificity = True Negative/(True
depends on the resources of the hospital.
You decided on one of the diagnostic
Negative + False Positive)
Some hospitals allow easy access to
tests for your patient. Do you think you
radiographic testing and laboratory A test that has high specificity means that
should know some statistics in order to
testing. In other hospitals, obtaining a it has a low rate of reporting false
evaluate the results? Let’s check some
diagnostic test may not be that simple positives. A test that has low specificity
basic statistical terms that we regularly
(Pines 2012). has a high likelihood of false-positive
face as a doctor.
results (Wald 2011).
Questions for diagnostic strategy
Random ordering of laboratory tests and
described by Wald (2011) are Positive predictive value (PPV) refers to
shortcomings in test performance and
interpretation may cause diagnostic the likelihood of the patient truly having
• What am I going to do with the test
results? errors. Test results may be vague with the disease when the test is positive or
false positive or false negative results and abnormal.
• How is this test going to help me generate unnecessary harm and costs.
confirm or exclude the diagnosis? • PPV = True Positive/(True Positive +
Laboratory tests should only be
False Positive)
demanded if results have clinical
• How will the test result affect my
consequences (Schuetza 2015). Negative predictive value (NPV) refers to
diagnostic strategy, management, or
final disposition? the likelihood that the patient does not
Sensitivity refers to the likelihood of a
have the disease when the test is
test being positive or abnormal in the
negative or normal (Wald 2011).
presence of disease.

59
• NPV= True Negative/(True Negative + False Negative) Five causes taxonomy of testing-related diagnostic error
(Epner 2013)
Probability
1. An inappropriate test is ordered.
The other important element in testing is the probability.
Previously, the physicians’ role in emergency medicine was 2. An appropriate test is not ordered.
clinical problem solving by history taking and examination only.
3. An appropriate test result is applied incorrectly.
Now it has changed and incorporates determining the pre- and
post-test probabilities essential for the ordering and interpretation 4. An appropriate test is ordered, but a delay occurs somewhere
of laboratory tests (Schuetza 2015). Probability relates to your in the whole testing process.
concern about a particular patient having an illness or condition
and how that concern may or may not be impacted by the 5. The result of an appropriately ordered test is not accurate.

diagnostic test results (Wald 2011).


Diagnostic Strategy
Diagnostics, including point of care testing in the ED, is still
Testing-related diagnostic error
The EDs are often described as a diagnostic testing center where evolving. As our technology continues to improve, we will have

the results of most diagnostic tests are known within a few hours. greater access to the results of a multitude of diagnostic studies

The importance of diagnostic tests in Emergency Medicine is an in a timely fashion (Wald 2011). It is our responsibility to practice

undeniable fact. For example, there are a lot of diagnostic medicine in a cost-effective manner that benefits our patients and

imaging alternatives available in the ED including USG, CT, and does not overburden them and the health care system with

MRI in the ED. So, the pathologies that were mostly detected at unnecessary and, at times, overused testing (Wald 2011).

autopsies in the past, such as pulmonary embolism or an aortic Blood circulating biomarkers play a crucial role in the present
aneurysm, became a clinical problem for today (Wald 2011). diagnostic workup of ED patients. A biomarker may be
Unfortunately, many “routine” laboratory tests are being ordered considered as any protein or other macromolecules that can be
in “bundles” without any impact on diagnostic or therapeutic objectively measured and evaluated as an indicator of normal
management (Schuetza 2015). biological processes, pathological processes, and course of
diseases or pharmacological responses to a therapeutic

60
intervention. Readily measurable biomarkers give important m a n a g e m e n t t o o u r patients. However, in the same time,
information about etiology of a disease and the necessity for it is our responsibility to use our resources wisely. Therefore,
interventions and prognosis. Diagnostic biomarkers justify the ordering the appropriate tests is very important. The tests which
presence or absence of a disease (Schuetza 2015). you think it will change your management and you know what are
you going to do with the results are the best tests for your
In Emergency Medicine practice, we use algorithms or clinical
patients. In addition, this approach will help to use our resources
decision rules (Ottawa Ankle Rules, PECARN minor head trauma
efficiently and decrease the cost of of unnecessary tests.
algorithm, etc.) to make standard management. These are useful
and practical tools to make an acceptable decision. Clinical References and Further Reading, click here.
decision rules try to make objective criteria that may help you to
distinguish who requires a test or not (Pines 2012). Some people
call it “cookbook” medicine, and, of course, “one size cannot fit
all.” Today, however, they are the most evidence-based
approaches to pathologies. So staying within the rules is one of
the best methods that will assist you when contemplating when
to utilize diagnostic tests.

Comprehending the evidence behind diagnostic testing and using


clinical decision rules to decide when not to test is at the center
of emergency medicine practice (Pines 2012).

Last questions that you should keep in mind:

• Will that test result change your management?

• Do you have any plan if it’s positive, negative, or indeterminate?

These questions should be considered before you order the test.


It is our responsibility that giving the best, correct, and the fastest

61
Section 6

Creating Your Action Plan

Introduction
by Chew Keng Sheng As the patient’s physiologic condition is dynamic and changes from time to time,
we need to remember that the action plan is not static and can change in a
moment. As such, we must not be too fixated with our earlier impression and fail
or refuse to change it in light of discriminating evidence. This is further
compounded by the challenge that the emergency department (ED) can be a high-
acuity clinical environment that does not afford us the luxury of providing care in a
structured manner as a low-acuity outpatient setting does.

Although establishing a definitive diagnosis is the goal in a conventional clinical


approach, that is nearly impossible given the limited available clinical and
laboratory data as well as the limited time we can spend with the patient in the ED.
Unfortunately, establishing the definitive diagnosis may often be an unrealistic
expectation of the general public. In fact, some patients are admitted to the
hospital, and others are discharged home without a definitive diagnosis. Coming to
terms with this unpleasant uncertainty of emergency medicine is necessary. It is,
therefore, important to always maintain a healthy degree of skepticism in patient
management by asking questions like, “What if I am wrong?” “What else could this
presentation be due to?” “Do I have sufficient evidence to support or refute this
diagnosis?”

62
A doctor working in the ED needs to have adequate knowledge of Temperature
emergency conditions commonly presented to the ED. An
Patients in the extreme age group may not mount a sufficient
emergency condition is any medical condition of sufficient
febrile response to an infection to cause an elevation in body
severity (including intense pain) and when the absence of
temperature. Always remember to ask whether the patient has
immediate medical attention could reasonably be expected to
taken any anti-inflammatory or antipyretic medications (e.g.,
result in mortality and morbidity. Hence, unlike in conventional
paracetamol, aspirin, non-steroidal anti-inflammatory drugs)
patient approaches, working in the ED requires a doctor first to
before coming to the ED. The thermoregulatory center is located
ask this important question, “Is there a life or limb threatening
in the anterior hypothalamus; thus, any central nervous system
condition that I must rule out in this patient”? A life-threatening
infection or injury that affects the hypothalamus such as
condition is a threat to the airway, breathing, and circulation.
cerebrovascular accident and subarachnoid hemorrhage may
Once a life or limb threatening condition is identified,
affect thermoregulation. Certain drugs (e.g., anxiolytics,
interventions must be instituted immediately to address it before
antidepressants, oral antihyperglycemics, beta-blockers), adrenal
moving on to another form of examination and investigation.
insufficiency, end-stage renal disease and thyroid disorders can
Importance of Vital Signs also affect basal body temperature or temperature regulation.
In addition to knowing emergency conditions, it is essential not to
Pulse
forget to look at the vital signs chart when formulating your action
plan. Bear in mind that “normal” vital signs can be abnormal When taking the pulse, the rate, regularity, and volume should be
(Markovchick 2011). For example, an elderly patient with BP that noted. The pulse rate should also be interpreted taking into
usually ranges from 140 – 160/90 – 100 mmHg can mean that he consideration the patient’s age. For adolescents and adults, the
is unstable with a BP of 110/70 mmHg and persistent vomiting maximum sustained HR estimation can be calculated with the this
and diarrhea. A patient with severe asthmatic exacerbations who formula: maximum sustained HR = (220 – age in years) × 0.85.
was tachypneic and restless initially does not mean that he is now
stable if he is “calmer” with a respiratory rate reduced to 10 Bradycardia is defined as a heart rate lower than 60 beats/min in
breaths per minute. In other words, noting the trend of the vital adults. However, a well-conditioned athlete may have a normal
signs is much more important than reading an isolated vital sign resting heart rate as low as 30 to 40 beats/min. Ask also if the
measurement. patient is taking any medication that could affect the pulse rate.

63
For example, digitalis compounds, β-blockers, and depth of breathing and the pulse oximetry for the oxygen
antidysrhythmics may alter the normal heart rate and the ability of saturation.
this vital sign to respond to a new physiologic stress.
Respiratory rate of >60 breaths per min in an acutely ill child
Physiologically, for every one-degree increase in Fahrenheit, the under the age of 2 months is a predictor of hypoxia. Respiratory
heart rate increases by ten beats/min. As 1 Celsius equals to 9/5 rate generally increases in the presence of fever; therefore, it can
or 1.8 Fahrenheit, the increase of every one-degree Celsius be difficult to determine whether the tachypnea is a primary
results in an increase of pulse rate by 18 beats/min. This is known finding of respiratory problems or is simply associated with the
as the Leibermeister’s rule. However, there are conditions fever itself. Observe the breathing patterns of the patient as well.
whereby the increase in temperature is not followed by an Look for any abnormal breathing patterns such as Cheyne-Stokes
increased pulse rate. This is known as relative bradycardia (or the breathing (episodes of progressive shallow-deep-shallow cycles
Faget sign). Causes of relative bradycardia can be divided into suggestive of stroke, trauma, carbon monoxide poisoning, and
infective and non-infective causes. Infective causes include the metabolic encephalopathy, etc.) and Kussmaul breathing
following: Legionella, Psittacosis, Typhoid Fever, Typhus, (increased rate and depth of breathing). Click here for a video of
Babesiosis, Malaria, Leptospirosis, Yellow fever, Dengue fever, Cheyne-Stokes breathing and a video of Kussmaul breathing.
Viral hemorrhagic fevers, Rocky Mountain spotted fever, etc. The
Pulse oximetry
non-infective causes beta-blockers (but not an angiotensin-
converting-enzyme inhibitor, ACE inhibitor; calcium-channel Pulse oximetry is a non-invasive measurement of the oxygen
blocker nor digoxin), central nervous system lesions (tumors and saturation. The relationship between SaO2 and the partial
bleeds), lymphomas and drug fever (Cunha 2000). pressure of arterial oxygen (PaO2) is described by the
oxyhemoglobin dissociation curve (ODC). Because of the sigmoid
Respiratory Rate
shape of the ODC, a unit reduction of PaO2 change in this
The respiratory rate only informs us how fast or slow the relatively flat portion of the ODC produces only a small change in
breathing rate is; it does not inform us about the depth of the SaO2 as compared to a unit of reduction of PaO2 in the relatively
breathing or the oxygenation status of the patient. Therefore, steep part of the curve that produces a much greater degree of
besides looking at the rate, we should also pay attention to the reduction of PaO2. The point of intersection between the
relatively flat portion of the curve and the relatively steep portion
64
of the curve is known as the ICU point, and it corresponds to a cool, clammy skin, and delayed capillary refill. However, the
SaO2 of around 92% and the PaO2 of 60 mmHg. Therefore, systolic blood pressure (BP) is still within the normal range even
always attempt to maintain the SaO2 above 92%. PaO2 below 60 though the pulse pressure is decreased. The decrease in pulse
mmHg means that the patient can markedly desaturate. pressure is due to the increased levels of circulating
Conversely, at a PaO2 above 60 mmHg, increasing the PaO2 will catecholamines, causing an increase in peripheral vascular
not result in a marked increase in the SaO2. In fact, giving too resistance, and raising the diastolic BP.
much supplemental oxygen may result in an ever increasing PaO2
For children, the blood pressure measurement varies according to
with a SaO2 maintained at 100%. Hyperoxia (too high PaO2) can
age. A formula for estimating the 95th percentile BP (normal) in
be harmful as it can lead to adverse effects such as generation of
young children is as follows: BP = 80 + (2 x age in years).
reactive oxygen species and release of angiotensin II resulting in
Hypotension is defined as less than the 5th percentile BP that can
vasoconstriction. (Click here to access two articles for more
explanation and diagrams: Hooley J. Decoding the be estimated by the following formula: hypotension = less than

Oxyhemoglobin Dissociation Curve and Brandis, K. Oxygen 70 + (2 x age in years).


Dissociation Curve.
The algorithm of data gathering and creation of an action plan in

Blood pressure the ED is shown below.

Blood pressure, defined as the force exerted by blood on the References and Further Reading, click here.
vessel wall, only indirectly measures perfusion, as blood flow
equals to the change in pressure divided by resistance. But
because peripheral vascular resistance varies, normal blood
pressure does not necessarily mean good tissue perfusion. The
normal blood pressure may be “maintained” by an increase in
peripheral vascular resistance. Furthermore, hypotension is a late
sign of shock; this is especially true in children. For example, in
class II hemorrhagic shock (with a loss of 15%–30% blood
volume), the findings usually include tachycardia, tachypnea,

65
Diagram 2.1 Data gathering and creation of action plan

66
Section 7

Documentation

Introduction may use them for summative or


by Muneer Al Marzouqi and Qais Abuagla Whether you are rotating in the formative assessments as well as a
Emergency Department (ED) or means for feedback. These documents
elsewhere, one of the key skills to learn a re a l s o a n i m p o r t a n t t o o l f o r
is how to write a complete and legible communication between the ED and
patient record. Documentation in the respective physicians, specialties and
ED is usually challenging, as it may be other stakeholders. Appropriate
difficult to adequately capture and note medical documentation improves the
details down in a timely manner. This quality of communication within an ED
happens especially when dealing with and aids in the quality assurance
high acuity or critical case scenarios. process.
Even as a medical student or intern,
your medical record is important on so
It is said that “if something
many levels. It serves to reflect your was not written in the chart,
general approach, thought process, then it never happened.”
the care you provided to patients, as
well as potentially identifying gaps in Having a well-organized and legible
your knowledge and training. chart gives the auditors and reviewers
Attendings, clerkship directors, and a clear picture of the physician’s
faculty usually emphasize and pay thought process, the actions he/she
attention to how notes are written and performed, and provides a real-time

67
snapshot of the patient’s general condition at any given sensitive interventions were done or when medications were
encounter. There is always room to learn about and improve administered (Carrol, 2016a and 2016b).
medical documentation. Therefore, this section will review the key
elements used when documenting in the ED (Murphy, 2001; Components of the history include
CDEM, 2010)

1. Chief Complaint
Emergency Medicine Note
Before writing your note, the nursing triage notes and vital signs, This usually includes the presenting complaint, ideally in the
if available, need to be reviewed. If obvious discrepancies are patient’s words, with the duration (Example: Abdominal Pain – for
seen, these need to be verified with the nurse and patient, as two days).
there may be errors. In addition, any abnormal vitals in triage
must be acknowledged and written in the note. 2. History of Present Illness

Like any other medical record, the ED document will comprise of In general, there are two formats when writing a history of present
the patient’s history, physical exam findings, differential illness (HPI), the narrative format and bullet points format. Both
diagnoses, investigations ordered, lab and imaging findings, are acceptable as long as the history is written in a
assessment and plan. Each component will be discussed comprehensive, concise and coherent manner. It is of added
separately, and suitable examples will be provided accordingly value if pertinent negatives and positives are added when writing
(CDEM, 2010; Carrol, 2016a and 2016b). the HPI, to show the physician’s thought process. This will lead
the person reading the chart towards what differential diagnoses
History to consider and what to rule out, depending on what the patient is
When writing the patient’s history, one needs to be clear, presenting with. Certain mnemonics may be used to aid in writing
thorough, and concise avoiding any long and complex phrases. a systematic HPI (Example: OLDCARS or OPQRST).
Ideally, it needs to flow in a logical and chronological sequence.
Unnecessary details are better avoided as they serve as Example 1: A 45-year-old man, with a history of Coronary Artery
distractors and may confuse other readers. Recording the date Disease and Hypertension, presenting to the ED with chest pain
and time when the patient was seen is crucial, especially in that started 3 hours ago. The pain was of gradual onset while
critical patients, as it helps create a timeline for when time- sitting on his chair, localized in the center of the chest and lasted
68
for 20 minutes. It was described as “a heavy boulder on my 4. Past Medical/Surgical History, Medications, and Allergies
chest.” The pain started when he had a quarrel with his daughter
List any known active illnesses the patient might have or had in
and was relieved with sublingual nitroglycerin. It was associated
the past. Include any surgical procedures he had. State what
with nausea and sweating, but no vomiting. Was localized and
medications he is actively on and whether he has any drug or
not radiating to the shoulders or arms. He claims it was
food allergies.
moderately intense at 4/10 on the pain scale. He denies any
shortness of breath, palpitations, dizziness, or abdominal pain. 5. Family and Social History

Example 2: A 26-year-old male, previously healthy, presents with Document a brief family history that may be relevant to the chief
a sore throat for one week. It is associated with subjective fever complaint (Example: Family history of Diabetes and Cardiac
and fatigue. It is aggravated whenever he drinks or eats but Disease in a patient presenting with chest pain). Social history
denies any difficulty swallowing or drooling. The patient also mainly includes asking about smoking habits, alcohol
denies any chills, runny nose, cough, night sweats, or shortness consumption, sexual history and illicit drug use. It also might be
of breath. No recent travel history reported. Has several sick important and relevant to ask about the patient’s financial and
contacts at home with similar symptoms. health insurance status, particularly in certain healthcare settings,
to avoid ordering unnecessary tests and paying extra costs.
3. Review of Systems

Other organ systems and symptoms that were not mentioned in Physical Examination
the HPI are to be reviewed to make sure the patient does not When recording physical examination (PE) findings start with the

have other complaints or organ system involvement. If the review patient’s general appearance and vital signs, highlighting

or system (ROS) cannot be obtained because of the patient’s abnormal ones. It is important not to document or fabricate any

underlying condition (i.e., unconscious, critically ill, or having findings that were not examined; committing to such findings may

dementia), this should be noted in the chart. Generally, ask have medical and medico-legal implications that are best

patients questions from head to toe (Example: “Do you have a avoided. Document all findings from examined systems including

fever, chills, headache, sore throat, chest pain, abdominal pain, findings from inspection, palpation, auscultation, etc. There is no

urinary symptoms, etc.”). Document all positive ROS symptoms need to document findings that are not pertinent to the chief

and state the remaining ones as otherwise normal. complaint (Example: Neurological examination findings in a

69
patient with a sore throat). Include important positive and instructions should be documented clearly (CDEM, 2010;
negative findings for any given case (Carrol, 2016a). Carrol, 2016a and 2016b).

Example: Patient with abdominal pain Summary of all components in an ED Note:


1. Chief complaint
• Important positive findings: Soft, non-tender abdomen, normal
active bowel sounds 2. History of present illness with pertinent positives and negatives

• Important negative findings: No rebound tenderness, guarding, 3. The brief review of systems
rigidity, or peritoneal signs
4. Focused past medical and surgical history
Assessment 5. Focused pertinent medications and allergies
Should capture the essence of the case and defend the rationale
for potential further investigations. It usually includes an objective 6. Very focused family and social history if required
summary of the case with differential diagnoses based on history
7. Vital signs, highlighting any abnormal readings
and physical examination findings.

8. Focused and pertinent physical exam


Plan
This section includes what investigations, medications, 9. Assessment
procedures, and consultations are to be ordered or performed.
Time of consultation is very important, and the doctor’s name and 10.Plan

his/her recommendations are to be documented in a timely 11.Disposition


manner.
Few helpful hints during documentation
Disposition • Place a date and time for all notes in the medical record
This usually is the last part of the note. It mentions whether the
patient is going to be admitted, discharged, or transferred to • Write notes clearly and legibly
another facility. If discharged, follow-up instructions and return

70
• If you make a mistake, draw one line through it and sign your • If a patient leaves against medical advice (AMA), document that
initials you have explained the specific risks of leaving AMA to the
patient and relatives
• Document a focused but thorough History and Physical
Examination • Document plan for outpatient care and follow-up

• Avoid using unclear abbreviations that are not used commonly • If using an electronic medical record (EMR) instead of a
handwritten one, all of the above sections, components and
• Document vital signs and address abnormalities
hints apply (Murphy, 2001; Dunbar, 2014; Virtual Mentor, 2011)
• Document the results of all diagnostic tests that were ordered
Sample ED Note, please click here.
when appropriate
References and Further Reading, click here.
• When speaking to a consulting service, document the name of
the physician and the time the call was made

• Document the patient’s response to therapy

• Document repeat examinations

• Document your thought process (medical decision-making)

• Avoid writing derogatory comments in the medical record

• Avoid changing or adding comments to the medical record after


completion. It may be appropriate to add an addendum but only
if it is properly timed and dated.

• Document all procedures performed

71
Section 8

Discharge Communications

Introduction
by Justin Brooten and Bret Nicks The process of patient discharge from the emergency department (ED) provides
critical information for patients to manage the next steps of their care. Hospital
accreditation and governmental organizations often require these instructions for
quality or monitoring metrics. However, studies show that many patients do not
fully understand or recall the instructions they receive (Clarke, 2005; Clark, 2005).
Add to this the myriad challenges inherent in every emergency department that
only perhaps compound this lack of comprehension and subsequently impact care
compliance, outcomes, and patient experience.

In many situations, the discharge process is often limited to a brief exchange of


documents, prescriptions and verbal description of the diagnosis, frequently
leaving patients with uncertainty about their care plan. Certainly, understanding
discharge instructions can be very challenging. At the time of discharge, patients
or family members may be experiencing physical and emotional discomfort. They
may be eager to leave, and thus, less interested in the instructions. Moreover, a
significant number of patients have low literacy or health literacy levels (Zeng-
Treitler & Hunder, 2008). Also, the busy ED setting may distract the patient’s
attention from such instructions. Therefore, understanding the challenges around
discharge communications in the ED from the patient’s perspective and having a
clear approach and purpose is essential. Discharge is not an afterthought; it is the

72
first step of a patient’s care transition and greatly impacts quality quite poor (Clarke, 2005; Clark, 2005; Crane, 1997; Engel,
outcomes, litigation, experience and team morale (Henry, 2013; 2012; Sameuls-Kalow, 2015; Taylor, 2000; Zeng-Treitler & Hunder,
Siff, 2011). 2008). This raises significant concerns for care plan adherence
and medical outcomes. Given current trends toward value-based
Understanding the Challenges care and the fact that nearly half of the lawsuits in emergency
Emergency physicians face unique challenges while ensuring medicine revolve around discharge instructions and the discharge
high-quality care due to distractions and time limitations that are program given to patients, ongoing improvements in the
common throughout ED settings. In most cases, emergency discharge communication process is essential (Henry, 2013; Siff,
physicians have little or no previous knowledge of their patients, 2011). While some of this relates heavily to the ability of the
making effective communication paramount when patients are provider to establish a trusting and positive patient-provider
discharged from the ED (Jon, 2013). Recognizing the value of relationship within the ED constraints, several strategies can be
early quality communication continued throughout the patient used to enhance the recall of instructions, improve compliance,
care encounter may carry over to the discharge care processes and minimize litigation.
and, in turn, improve an important aspect of quality and patient-
centered emergency medical care. Discharge Essentials
Effective discharge communication provides an opportunity for
It has been demonstrated that many patients are discharged from
the ED team to summarize a patient’s visit, teach them how to
the ED with an incomplete understanding of the information
care safely for themselves at home and provide specifics
needed to care safely for themselves at home (Clarke, 2005;
regarding the next steps in their care process. It also gives ED
Crane, 1997; Engel, 2012; Sameuls-Kalow, 2015; Taylor &
physicians a chance to address any remaining questions or
Cameron, 2000). Patients have demonstrated particular difficulty
concerns (Jon, 2013), often augmenting patient and family
in comprehending post-ED care instructions regarding
understanding while improving care plan retention. Although
medications, home care, and follow-up expectations. And while
patient education at discharge typically begins with initial
all patients discharged from the ED should be provided
assessments and conversations with the patient and his family,
instructions for ongoing management of their medical condition,
other factors can also influence the success or failure of how
studies have demonstrated that the patient recall and
information is transmitted at discharge (Jon, 2013).
understanding of diagnosis, treatment, and follow-up plan are

73
Common interventions included in an effective ED discharge is needed and why, as well as how to care for oneself until that
process consist of a standardized approach (content), information time, improves outcomes and compliance. Some have phrased
delivery, confirmation of comprehension, post-discharge care these basic tenants of discharge as the ‘rules of the road’;
follow-up planning, review of vital signs and a patient-centered however, this may serve as the basis from which to develop your
closure (Table 2.1) (Taylor, 2000; Zeng-Treitler & Hunder, 2008). process.

Table 2.1 Table 1. Interventions in the ED Discharge Process Rules for the Road
1. Have the right diagnosis
DOMAIN INTERVENTION
Content Standardize approach 2. Time & Action Specific Instructions

Delivery Verbal instructions (language and culture appropriate) • What to do


Written instructions (literary levels)
Basic Instructions (including return precautions)
Media, visual cues or adjuncts • When to do it
Comprehension Confirm comprehension (teach-back method)
3. Provider Specific
Resource connections (Rx, appointment, durable medical supplies,
Implementation follow-up)
Medication review
• Who to contact

• Why and When


Content refers to the education provided to our patients related to
the treatments, tests, and procedures performed during the ED 4. Printed Information, Verbally Explained
visit, as well as further education on diagnosis, treatment plan,
• Verbally confirmed
the expected course of illness and medication reconciliation. It
should also include time-sensitive and specific information The quality and approach of a physician’s delivery of content
associated with their diagnosis and care plan regarding what to cannot be overstated. The ED provider and care team members
do and when to do it. This should include precautions about must consider the wide range of literacy (and health literacy),
when to return to the ED versus waiting for any follow-up cultural backgrounds and access to outpatient resources when
appointments, and what steps have already been taken to assist delivering the ongoing care instructions (Engel, 2012; Sameuls-
with this process. Utmost clarity regarding what type of follow-up Kalow, 2015). In many instances, to improve patient
74
understanding of discharge instructions, EDs attempt to improve high risk of discharge failure and trigger additional resource
patient and family understanding of discharge instructions considerations for these patients.
through standardization and simplification of written and verbal
The discharge process provides an opportunity to ensure the
instructions for patients and those with them. This verbal
patient’s condition is well understood that there aren’t any
discussion can be especially helpful for those with low health
additional medical red flags that need to be addressed, and that
literacy. Also, utilizing interpreter services for those who speak
the care plan and follow-up are fully comprehended. In an online
other languages may be vital. Other approaches that may benefit
video, Dr. Oller (2016) provides another process to engage the
patient outcomes include providing supplemental written
‘moment of safety’ related to discharge and outlines five essential
information and using visual and multimedia adjuncts to support
steps for any ED discharge.
understanding (Taylor & Cameron, 2000; Zeng-Treitler & Kim,
2008). Essential to any successful approach is the patient’s
ED Discharge: Moment of Safety
comprehension of the information provided. After all, if the 1. Has the medical provider discussed the findings, diagnosis,
content and delivery are exceptional but the comprehension is and plan of care (including medications and follow-up plan)?
poor, this should be seen as a discharge failure as it decreases
care compliance and outcome quality. To address this specific 2. Confirm the discharge instructions and prescriptions match the
aspect of the discharge process, instituting a read-back or teach- patient identifiers
back method is recommended.
3. Review all prescriptions and clarify any changes
The implementation of discharge care processes frequently falls
4. Review of current vital signs
short due to unidentified social and medical factors that prevent
the plan from being carried out. Social factors could include 5. Closure
homelessness, low income, uninsured/underinsured status, lack
Watch the video.
of transportation, or lack of primary care. Medical factors could
include concurrent psychiatric illness, substance abuse, cognitive
Barriers to Successful Discharge
impairment, inability to care for self, or young/advancing age.
The barriers to successful discharge are myriad. Some are
Understanding these circumstances will help identify patients at
intrinsic to the ED work environment and the nature of ED patient
arrival and flow. Others relate to the challenging or often
75
unidentified social and medical factors that prevent the plan from of providing discharge information and instruction: basic
being completely carried out. In a recent American College of care instruction note, a pre-formatted illness specific instruction
Emergency Physician Quality Improvement and Patient Safety sheet, and templated software-based discharge product (Taylor,
Section meeting, Dr. Pham (2016) shared a conceptual framework 2000).
for understanding the barriers to success and improving the
discharge process (Figure). Diagram 2.2 Barriers of efective ED discharge

While this framework may not be uniformly representative of all


EDs, it addresses many of the operational failures that occur
outside of the ED and outlines opportunities for hospitals and
health systems to align with improved patient care outcomes.
Moreover, while screening for high-risk discharges in EDs occur,
the additional resources needed to ensure appropriate social
work or case management care coordination are often limited.

Post-discharge follow-up processes for patients at risk for failing


discharge instructions exist in some systems. This may include
flagging a patient’s chart for a social work follow-up to assess
and assist with the patient’s ability to obtain necessary
medications, obtain follow-up appointments, or address other
concerns identified by the provider. Some physician groups
routinely call the patient the next day to see how the patient is
doing and ensure understanding of his/her discharge instructions
and care plan (Sameuls-Kalow, 2015; Taylor 2000). Commonly used, an instruction note is simply a set of instructions
handwritten or typed on plain paper, without the assistance of
Types of Discharge Information Packets computer programs. In settings with limited resources, this may
Discharge instructions vary widely by practice location and be the only means of providing essential care information for the
resources available. However, there remain three primary means
76
patient, their families and the provider with whom they may for the patient to document clearly why the patient left and
follow-up. While uniquely tailored, they may lack substantial attested that the patient had the mental capacity to make such a
content for care, take time to prepare, and are limited by literacy decision at that time (Henry, 2013). While some electronic
and handwriting. documentation systems have templates in place to assist with
this documentation, Table 2.2 provides basic information for
Information sheets are pre-printed education and instruction
against medical advice (AMA) discharge documentation that can
documents that describe care information related to one specific
be used to create a uniform template (Henry, 2013; Siff, 2011;
illness. They can be developed for the most common medical
Levy, 2012; Devitt, 2000).
illnesses for each institution and have essential information
regarding plans of care. Information sheets are immediately An attempt should be made to provide the patient with
available, inexpensive, reproducible, and can be designed to appropriate discharge instructions, even if a complete diagnosis
include simple language and or pictorial education. They are not may not yet be determined. Include advice for the patient to
patient-specific, may not provide adequate instruction in difficult follow up with his physician, strict return precautions, and
or complicated cases and require computer, printer, and copying concerning symptoms that should prompt the patient to seek
capacity. further care. It should also be made clear that leaving against
medical advice does not prevent the patient from returning to the
For settings with an integrated EHR, software products that
emergency department for further evaluation if his symptoms
create discharge packets (including discharge diagnoses,
worsen, or if he changes his mind. Despite a common notion to
medications, medical care instructions and information regarding
the contrary, simply leaving against medical advice does not
the illness, outlined care course after leaving the ED and essential
automatically imply that physicians are immune to potential
contact information for those next steps) are available. These are
medical liability (Levy, 2012; Devitt, 2000). If a patient lacks
highly resource dependent and, therefore, may not be routinely
decision-making capacity to be able to adequately understand
available.
the rationale and consequences of leaving AMA and his condition
places him at risk for imminent harm, involuntary hospitalization is
Against Medical Advice and Elopement
In certain circumstances, patients may request to leave prior to warranted. In unclear circumstances and if available, psychiatry

completion of their medical evaluation and treatment. In this can assist in determining capacity, especially in the case of

situation, it is essential for the last health care professional caring patients with mental health conditions.

77
Elopement is a similar process where patients disappear during verbal instructions remain m o r e e ffe c t i v e t h a n w r i t t e n
the care process. While it is difficult to provide discharge instructions, but both are needed. Be explicit, keep it simple and
paperwork for these patients, documenting the actions taken to have the patients repeat back instructions to ensure
find the patient is essential (e.g., searching the ED, having understanding. These simple steps will improve patient
security check the surrounding areas). In addition, attempt to outcomes, compliance and avoid legal pitfalls.
reach the patient by phone to discuss his elopement and any
References and Further Reading, click here.
additional care issues or concerns. Documentation of these
attempts or any additional conversation is very important (Henry,
2013; Siff, 2011).

Table 2.2 Documentation for Patients Leaving Against


Medical Advice
COMPONENT DESCRIPTION

Establish patient's decision-making capacity, and clarify aspects of


Capacity care which may affect capacity (i.e. Patient is now clinically sober,
etc.)

Specific condition associated risks that were discussed (missed


Risks
diagnosis, potential harms from untreated disease process, etc.)

Verify comprehension Patient's understanding of the risks

Include patient’s decision, and any alternative plans (i.e. patient


Patient's decision refused admission, but agreed to follow up with primary physician
tomorrow.)

Signatures Patient's and provider's signatures

Conclusion
Discharge instructions are a very important part of the ED care
process and record. It is essential to ensure each patient has a
complete understanding of her instructions and to recognize that
78
Chapter 3

General Approach
to Emergency
Patients
Section 1

The ABC Approach to the Critically Ill Patient

Introduction has been incorporated into algorithms


by Donna Venezia History of the ABC’s further to refine the accuracy of initial
treatment in the critically ill.
The basic ABC algorithm was initially
designed and implemented on a large Goals of approaching any
scale in the early 1960’s for those critically ill patients are
requiring cardiac-pulmonary • Rapidly identify and manage life/
resuscitation. The order has recently brain-threatening conditions before
been changed to the CAB for those the exact diagnosis is made.
who have suffered a cardiac arrest
• After initial stabilization, follow with
(See BLS/ACLS – Cardiac Arrest
full history, exam, time-consuming
section). Twenty years later the lab/radiological testing and reach the
American College of Surgeons again final diagnosis.
modified this sequence for patients
with acute traumatic injuries. Over the Identifying A Critically Ill
subsequent years, there have been Patient
many variations and modifications for Triage is a reliable method to quickly
the variety of critically ill patients select from a large group of waiting
presenting to an emergency patients, those who may have a
department. Most recently, the addition potential illness requiring time-critical
of point-of-care testing with ultrasound management to save a life or the brain.

80
As a standard structure, currently, all • acute torso discomfort (may be than minutes, can result in death or
modern emergency departments have a associated with radiation to jaw, brain damage.
triage unit to prioritize the patients. It anterior neck or shoulder/medial upper
• The order is performed sequentially to
aims to select more critical patients as arms) suggestive of an MI/
avoid skipping crucial steps and
e a r l y a s p o s s i b l e a n d c re a t e a n cardiovascular problem.
generally to manage the most serious
appropriate patient flow in the emergency
• severe acute headache first, however, the sequence can and
department. However, triage can be done
should be performed simultaneously
in the field by EMS staff, and patients • intractable seizure (may not show
(horizontal approach) in those with
may directly bring to the resuscitation muscular signs after a period of time)
multiple life-threatening conditions if
room.
• history of significant trauma, drug there are enough team members.
Potential critically ill patients ingestion, exposure, suicidal/homicidal Modify as appropriate to the individual.
may present with: ideation
• Because management may need to be
• altered mental status (unresponsive or
• significant vital signs abnormalities simultaneous, the team approach is
confused/agitated)
(age-dependent) crucial in successfully resuscitating any
• noisy respiration (gurgling, stidor, critically ill patient.
wheezing) Point of Care Testing
• adjunct tests/equipment that help guide • It is also important to emphasize that
• inability to speak normally (acute early decision-making the availability of various treatment
hoarseness or inability to articulate modalities at each medical facility.
words) • results should be back within seconds
to minutes, not hours! Meaning of the letters in the ABCDEF
• respiratory distress (rapid/deep or slow/ sequence:
shallow/agonal respirations) The ABCDEF Sequence
A = Airway Disorders with C-spine control
• Each letter represents a crucial body
• acute weakness or inability to ambulate
system that, if significantly disrupted B = Breathing Disorders
(diffuse/focal muscle weakness or light-
and left untreated over hours rather
headedness/syncope)
81
C = Circulation/Cardiovascular Disorders A – Airway with C-spine Point of Care Testing

D = Disability (Neurological Disorders) Control • generally, none required for complete


Focused clinical assessment for
obstruction
E = Exposure/Decontamination impending/actual airway compromise:
• soft tissue neck X-ray (for potential/
F = F e v e r ( E x t re m e Te m p e r a t u re • noisy respirations (gurgling, stridor, partial airway obstruction only)
Disorders) choking sounds) with or without
retractions • indirect laryngoscopy (for potential/
How to approach the critically ill
partial airway obstruction only)
patient using the ABCDEF algorithm • drooling, inability to swallow secretions,
leaning forward in a tripod position • fiberoptic laryngoscopy (for potential/
For each letter or body system:
partial airway obstruction only)
• throat swelling sensation with or
• obtain a brief, focused history and
without pain • u l t r a s o u n d ( f o r i d e n t i fic a t i o n o f
exam
cricothyroid membrane or assessment
• change in voice associated with proper endotracheal tube placement)
• obtain available point-of-care testing to
symptoms of bacterial infection or
aid in the evaluation/management
allergy (hoarseness, “hot potato” voice) Emergency Equipment for Managing
• initiate management for any acute life or Airway Problems
• active retching or vomiting with an
brain threatening condition
inability to turn or move to protect from 1. nasal or oral airway devices
• then, proceed to the next letter and aspiration
2. suction devices (rigid tip and/or small
repeat
• oral exposure to fire/steam inhalation, flexible tip)
• if no intervention is needed, quickly chemicals, acids/alkali
3. i n t u b a t i o n e q u i p m e n t ( i . e . ,
p ro c e e d t h ro u g h t h e s e q u e n c e .
• neck trauma with crepitus over larynx or laryngoscopes with ET tubes of
(Evaluation of a normal person should
expanding hematoma various sizes)
take just a few minutes or even
seconds.)
82
4. airway adjuncts for difficult airways – Only jaw thrust maneuver is allowed in hereditary angioedema – administer
i.e. LMA, iGel, Bougie, video this situation (see caveat 1) C1 esterase compound.
laryngoscope, fiberoptic laryngoscope,
• To n g u e o b s t r u c t s a i r w a y i n a n • signs of imminent or complete airway
etc.
obtunded patient – perform either head obstruction, unrelieved from above –
5. Magill Forceps for foreign body tilt, chin lift, or use jaw thrust attempt intubation with the most
removal maneuvers if possible. See BLS/ACLS. appropriate device by the most
experienced provider. May attempt
6. Pre-intubation supplies – supplemental • obtunded, without trauma – position
BVM ventilation first, especially in
wall and/or tank oxygen, RSI patient on the side to avoid tongue
children with epiglottis, as a
medication, sedation medication, oral obstruction
temporizing measure.
numbing medication
• Patient unable to be positioned – place
• unable to intubate or BVM –
7. “failed airway” kit – cricothyroidotomy nasal or oral airway. Avoid oral airway if
immediately perform cricothyroidotomy;
kit with appropriate sized Shiley or partially awake since may cause
avoid if laryngeal fracture a concern –
endotracheal tubes gagging/vomiting. Avoid nasal airway if
p r e f e r fib e r o p t i c i n t u b a t i o n o r
midface trauma.
8. capnography and/or ultrasound to tracheostomy in OR if possible. (See
assess for proper endotracheal tube • pharyngeal secretions, blood, and/or cricothyroidotomy technique)
placement vomitus – suction
Caveats
9. appropriately sized cervical collars • obstructing foreign body – perform
1. The airway is always associated with
abdominal thrusts/chest compressions
Management Algorithm for Critical the phrase, “with c-spine control”.
per BLS or if visible, attempt to retrieve
Airway Problems Before performing any airway
with McGill forceps.
procedures, one must quickly assess
• possible c-spine injury – employ the • laryngeal edema; likely anaphylaxis – the likelihood of a c-spine injury. If
second person to immobilize c-spine. administer IV/IM Epinephrine, likely there is a possibility of an injury in an
unresponsive patient, i.e. found at the

83
bottom of the stairs, or on the side of the sequence may improve the mental edema, laryngeal cartilage fractures
the road, unconscious, then assume status, making intubation unnecessary secondary to trauma
an injury and protect the c-spine by such as low blood sugar. Be prepared
• laryngeal cartilage fractures secondary
simply immobilizing as best possible. to log roll quickly if the patient vomits.
to trauma
Typically a C-collar is slid under the
Conditions causing airway
back of the neck while someone • expanding paratracheal hematoma
immobilizes the head. If airway compromise

management is required, the front of • tracheomalacia


• unresponsive patient with tongue
the collar can be opened or removed, blocking the airway an unresponsive • pharyngeal malignancies
as needed, while someone stabilizes patient who is unable to protect from
the head in relation to the torso. aspiration of blood/vomitus, etc. B – Breathing Disorders
Nothing further needs to be done in Focused clinical assessment for
the primary survey to evaluate the c- • an unresponsive patient who is unable evidence respiratory failure
spine. to protect from aspiration of blood/
vomitus, etc. Cyanosis, inability to speak full
2. An unresponsive patient has a sentences without needing a breath,
potential for airway compromise and • infections, i.e. epiglottis,
confused/agitated or unresponsive with:
subsequent aspiration. However, since retropharyngeal abscess, etc.
proper intubation is time intensive, you 1. Rate: too slow, shallow, agonal,
• allergic reactions/anaphylaxis, airway
may avoid intubation in these patients gasping (age-dependent, generally
burns, i.e. steam, chemicals, alkali/
until the primary sequence is rates <10 in an adult are abnormal)
acids, etc.
completed, unless the patient is
2. Rate: too fast and/or deep (again age-
actively retching. Have someone • airway burns, i.e. steam, chemicals,
dependent but >20 in a resting adult is
p re p a re t h e e q u i p m e n t a s t h e alkali/acids, etc.
abnormal, and > 30 is significantly
sequence is being completed and
• other causes of edema, i.e. ACE abnormal)
continue the evaluation since
inhibitors, hereditary angioneurotic
treatment of a condition found later in 3. Abnormal lung sounds:

84
• unilateral decreased breath sounds with very poor ejection fraction, etc. to •fix all upper airway critical issues
(either dull or hyper-resonant) help make a decision about treatment. first

• wheezing or poor air movement If still not clear as to a management • slow, agonal respirations or significant
strategy, add point-of-care testing, i.e., respiratory acidosis on ABG – provide
• rales (fine crepitation) or rhonchi
lung sonography or upright portable CXR. BVM ventilation and administer Narcan.
4. Chest wall abnormalities affecting
Point-of-Care testing • rapid breathing with hypoxia – provide
breathing dynamics – flail chest/open
supplemental O2 by the non-rebreather
punctures • pulse Oximeter mask to keep O2 saturation greater
Obtain as much focused history/exam as • C02 waveform monitor than 94%.
able to help define the need for a
• arterial (ABG) or venous (VBG) blood • sucking chest wound – seal with an
particular emergent treatment strategy for
gas occlusive dressing (3 sides only)
the common causes of critical respiratory
conditions. For example, two common • portable CXR (upright, if possible) • tension pneumothorax – place a 14
causes of severe respiratory distress are gauge needle, immediately followed by
pulmonary edema and COPD. Both may • pulmonary ultrasonography: a chest tube
present with wheezing (“cardiac asthma”
evaluate lung sliding for • massive hemothorax/pleural effusion –
in CHF), pedal edema and/or JVD,
pneumothorax drain fluid, contact trauma surgeon
making the decision for which type of
since may need transfusion/transfer to
emergent management strategy difficult. assess costophrenic angles for
OR for massive hemothorax
Obtain as much focused history/exam in effusion/hemothorax
a brief period of time, i.e. family states • no improvement in oxygenation despite
assess lung field segments for A/B
heavy smoker with similar episodes in the placement of non-rebreather mask or
lines, signs of consolidation
past, all resolved with inhaler therapy or above procedures, either –
the patient has a history of recent ECHO Management Algorithm for Acute
Respiratory Disorders

85
a l l o w t h e p a t i e n t t o b re a t h e Emergency Equipment for Managing b.C o n s i d e r s p e c i fic p o i s o n i n g
spontaneously under tightly held BVM Breathing Emergencies antidotes, i.e., cyanide antidote or
mask with PEEP valve on exhalation hyperbaric/100% O2 for CO
port and 15 L/min nasal cannula O2 1. Noninvasive ventilator NIV poisonings. See the toxicology section.
placed under the mask or 2. BVM (bag-valve-mask) with O2 supply c. Sepsis, Pulmonary embolus, and
provide NIV (non-invasive and added PEEP valve pericardial tamponade management
ventilation) with CPAP/BiPAP 3. additional wall or tank for an additional are discussed in more depth in the

source of O2 (nasal cannula) circulatory section since the primary


• patient agitated and unable to tolerate
c r i t i c a l p re s e n t a t i o n i s u s u a l l y
masks – administer IV Ketamine. 4. 14 gauge catheter-over-the-needle circulatory collapse.
• Signs of obstructive pulmonary disease 5. various sized chest tubes d. Obtain ABG/VBG if metabolic
(COPD/asthma) – administer
acidosis likely, manage most likely
inhalational beta agonist. Consider Caveats
cause. See acid/base section.
additional therapy (i.e., ipratropium,
1. fast RR with clear lungs may be
Prednisone, Magnesium, epinephrine, 2. Severe hypoxia unresponsive to
secondary to psychogenic
etc.). therapy, particularly with clear lungs,
hyperventilation, primary brain lesions,
m a y b e d u e t o s h u n t i n g f ro m
• Signs of acute pulmonary edema with metabolic acidosis, poisoning, sepsis,
congenital heart disease which, in a
adequate BP – administer repetitive or pulmonary embolus or pericardial
neonate, may respond to the
continuous doses of Nitroglycerin SL, tamponade:
administration of PGE1 (prostaglandin).
spray or IV. Consider additional drug
a. consider paper bag breathing if O2
therapy (i.e. Furosemide, etc.) 3. It is important to recognize that
sats normal, no acidosis, and
oxygenation and ventilation are
• respiratory distress unresponsive to hyperventilation syndrome most likely
different. A patient may not be
above therapy – intubate and (i.e., anxious with carpopedal spasm).
hypoxic, especially if given
mechanically ventilate
supplemental O2 but may still be in

86
acute ventilatory failure. (Only 250 cc • pulmonary embolus, air/amniotic fluid/ Skin – i.e., cool, diaphoresis, pale,
of oxygen is used by the resting adult fat embolus poor capillary refill, hives, erythema
per minute. However, 6-10 L of air
• massive hemothorax or massive pleural Mental status changes – i.e.,
must be moved per minute to
effusion confusion, slow responses, agitation
adequately ventilate a normal adult
and prevent the rise in pCO2.) • exhaustion from prolonged Rhythm/quality of pulses in all four
hyperventilation extremities
Conditions Associated with
Respiratory Failure • chronic lung conditions: cancer, Assessment for hidden blood loss,
sarcoidosis, fibrosis, etc. i.e., rectal for melena, pelvic instability,
• Pulmonary edema
pulsatile abdominal mass
• COPD/asthma
C – Circulation Disorders
Poor perfusion, Hypertensive crisis, Acute history: internal/external bleeding/
• severe pneumonia MI trauma, vomiting/diarrhea, oral intake/
urine output, fever, diabetes/renal
• ALI/ARDS from any cause (drugs, Clinically assess for poor perfusion i n s u ffic i e n c y / c a r d i a c f a i l u r e ,
aspiration, etc.) associated with medications, drug abuse/OD, last
menses
• tension pneumothorax • tachycardia: > 100 abnormal in adults,
> 150 frequently clinically symptomatic. Clinically assess for hypertension
• chest wall dysfunction, (flail chest,
muscular weakness, open sucking associated with
• bradycardia: < 60 abnormal, < 30
wound) frequently clinically symptomatic.
• signs of end-organ damage/
• respiratory depressants (narcotic OD, involvement, i.e., encephalopathy and/
• hypotension: systolic < 90
sedative OD) or papilledema, pulmonary edema,
• Perfusion and cardiovascular cardiac ischemia, renal impairment,
• bronchiolitis assessment may include and/or neurological abnormalities

87
• pregnancy (generally 3rd trimester/first Point-of-care testing •Hemocult paper (only needed if any
weeks postpartum); any new elevation question of blood/melena in stools)
of BP >140/90, particularly associated • EKG (perform within 10 minutes of ED
presentation; may include right-sided • Urine beta-HCG for critical childbearing
with a headache, abdominal pain,
leads RV3,4 and posterior leads V8, V9) age females
jaundice, shortness of breath and/or
visual disturbances • Cardiovascular ultrasound to include Emergency Equipment for Managing
assessment of: Cardiovascular Emergencies
Clinically assess torso discomfort for
likely MI LV cardiac contractility – normal, 1. pelvic binders/gauze for compression/
hyperactive, weak tourniquets
• description varies; besides chest
discomfort, symptoms may include the ratio of right to left ventricle size 2. defibrillator/external pacemaker
either/or epigastric discomfort, mid-
back discomfort, radiation to shoulders, p e r i c a r d i a l flu i d / t a m p o n a d e 3. large bore IV’s and 0.9% saline or
anterior neck, jaw or upper, inner arms. physiology Ringer lactate fluids

aortic root dilation/dissection flap 4. various sized IO insertion kits


• Note: There are many serious causes of
torso pain, i.e., simple pneumothorax, IVC collapsibility with inspiration 5. central line kits (only for large bore
cholecystitis, pancreatitis, bowel
sheath placement, if necessary)
perforation, etc.; work-up and evidence of free intraperitoneal fluid
management would be performed 6. thrombolytics or ability to access PCI
evidence of abdominal aortic
during the secondary evaluation unless (percutaneous coronary intervention)
aneurysm and/or dissection
there are signs or symptoms of facility
circulatory collapse. The evaluation of evidence of DVT in femoral veins
7. immediate access to O negative blood
torso discomfort in the initial
assessment should an emergent EKG • Telemetry monitor strip
8. straight catheter/Foley catheter (for
to pick up an MI. • ABG/VBG with electrolytes pregnancy check) and monitoring urine
output
88
9. Sengstaken-Blakemore tube rate (Central line sheaths, if unable to unstable tachydysrhythmia (not
start IO). sinus, multifocal atrial tachycardia,
Management Algorithm for Critical
junctional) – cardiovert per ACLS
Cardiovascular Disorders if no evidence of cardiac failure –
administer bolus 10-20cc/kg 0.9% NS/ unstable bradydysrhythmia –
• Management of Poor Perfusion R i n g e r s s o l u t i o n . ( F u r t h e r flu i d administer meds (i.e., Atropine, etc.)/

place two large bore IV’s and attach administration determined by clinical/ place external pacemaker per ACLS

telemetry monitor to all (may collect sono evaluation, risk/benefit, i.e.,


evidence of rhythm disorder is
various labs including blood cultures, permissive hypotension and clinical
associated with K+ abnormality by the
but should send type and crossmatch response, i.e., urine output).
lab, clinical history (i.e., renal failure,
now) Evidence of unstoppable internal DM) and/or EKG findings – administer

evidence of external bleeding, bleeding – immediate consultation with a p p ro p r i a t e h y p e r / h y p o k a l e m i a /

unstable pelvis – apply pressure/ appropriate specialty, i.e., surgery, OB, magnesemia therapy

binder; in rare cases tourniquet GI, etc. Consider various meds to


dysrhythmia unresponsive, with
attenuate bleeding, i.e., Tranexamic
patient in the 3rd trimester of evidence of thyroid storm or history of
Acid, Terlipressin for esophageal bleed
pregnancy – displace uterus to left/ drug OD, consider thyroid management
or Oxytocin for OB bleed. Consider
wedge under right flankunable to start or specific antidotes: NaHCO2 for fast
various procedures to stop internal
IV – attempt IO (intraosseous) with 300 Na+ channel OD (TCA, tricyclic
bleeding, i.e., Sengstaken-Blakemore
mmHg pressure cuff over the fluid bag antidepressants), Digibind for Digoxin
tube placement for esophageal bleed,
to increase flow rate (Central line toxicity, etc.
uterine massage post-delivery, etc.
sheaths, if unable to start IO). evidence of aortic dissection by
severe blood loss and/or persistent
unable to start IV – attempt IO clinical, sono evaluation – administer b-
unstoppable bleeding – transfuse O-
(intraosseous) with 300 mmHg pressure blocker first, i.e., Esmolol, then
negative units until type specific or fully
cuff over the fluid bag to increase flow antihypertensive, i.e., Nitroprusside,
cross-matched blood available
contact cardiovascular surgery

89
evidence of obstructive shock by 2. drug OD (i.e., b-blocker or calcium Hydralazine, Labetalol, or Nifedipine),
clinical/sono – treat appropriately as channel blocker – treat with high dose immediate OB consult.
guided by diagnosis, i.e., Insulin/glucose)
Management Algorithm for Torso
thrombolytics/interventional radiology
Management Algorithm for Severe Discomfort
for pulmonary embolus,
pericardiocentesis for tamponade, Hypertension associated with
• acute torso discomfort with MI
chest tube for tension pneumothorax,
• e v i d e n c e o f e n d - o rg a n d a m a g e documented by EKG – contact
etc. (ischemia, heart failure, encephalopathy, cardiology for immediate PCI/transfer

no evidence of acute volume loss etc.) – administer IV antihypertensive a n d / o r a d m i n i s t e r t h ro m b o l y t i c s

and/or no response to fluids or (Labetalol, Nitroprusside, etc.) Avoid depending on location and timing of

previous therapies – start pressors, pure beta blockers if suspect cocaine event per ACLS

Norepinephrine, Dopamine, overdose.


• acute MI by EKG (whether eligible or
Epinephrine, Dobutamine, etc., i.e., Epi
• evidence of hemorrhagic stroke, not for PCI/thrombolytics) – monitor for
for anaphylaxis, Dob/Norepi for thrombotic stroke, subarachnoid dysrhythmia, i.e., ventricular fibrillation,
cardiogenic shock, Norepi for sepsis, hemorrhage (See Disability Section) etc., administer Aspirin, follow protocols
etc. per local cardiologist.
• pregnancy and new elevation of BP
if no response to above, consider >140/90 – re-evaluate in 15 minutes • high suspicion of cardiac ischemia but
either: EKG not diagnostic – repeat in 10-15
• pregnancy with evidence of pre-
1. a d r e n a l c r i s i s – s t a r t I V minutes.
eclampsia/eclampsia (i.e., headache,
Hydrocortisone. (Dexamethasone, if nausea/vomiting, abdominal pain, visual Causes of critical cardiovascular
choose to perform testing disturbances, shortness of breath, conditions
concurrently.) and/or hyperreflexia, seizures – with or without
proteinuria) – or severe hypertension BP • dehydration
160/110 – administer MgSO4 and
• acute blood loss (internal and external)
initiate antihypertensive, (i.e.,
90
• sepsis D – Disability (Neurological/ Point-of-Care Adjuncts

• drug toxicity/OD
Psychological Disorders) • fingerstick glucose measurement
Clinically assess for
• cardiogenic shock • non-contrast head CT to be performed
• depressed consciousness (lethargic,
in less than 30 minutes
• anaphylaxis confused, comatose) (may use GCS to
assess the degree of unresponsiveness) • acute malaria screen in appropriate
• neurogenic shock
environments
• pupil size, symmetry, and reactivity
• adrenal crisis
• rapid HIV test
• agitation, delirium (waxing and waning
• thyroid storm
level of consciousness associated with • electrolytes (Na+, and Ca++, in
• obstructive shock confusion/disorientation and/or particular), if available, on ABG/VBG
hallucinations – typically, visual/tactile) assessment, sono for evaluation of
pulmonary embolus papilledema
• acute focal weakness/paralysis, or
pericardial tamponade inability to speak • sono for evaluation of papilledema
tension pneumothorax
• severe, acute headache, nuchal rigidity Emergency Equipment Needed for
gravid uterus compressing IVC Neurological Management
• signs of status epilepticus, including
subtle seizure-like activity (i.e., twitching 1. CT scanner
• tachydysrhytmias/bradydysrhythmias
with or without electrolye disorders e y e l i d s , s t i ffn e s s , p e r s i s t e n t
u n re s p o n s i v e n e s s a f t e r o b v i o u s 2. access or ability to transfer to
• symptomatic hypertensive with or seizure-like activity) neurosurgical equipped OR
without pregnancy
3. LP tray
• acute psychiatric disorder with either
• acute MI suicidal or homicidal ideation 4. leather restraints
• acute aortic dissection/rupture
91
5. stretchers that allow for head elevation • agitation, unable to calm with above •History acute fever, headache,
and/or patient an imminent danger to without focal neurological signs, recent
Management Algorithm for Critical
self/others – call for ‘man-power’ seizure history or impaired immunity
Neurological Disorders support and apply four-point restraints. and exam/sono shows no papilledema

Acute Agitation/Delirium Algorithm (Provide close monitoring of the patient – check malaria smear, rapid HIV test,
and remove restraints as soon as perform LP, initiate empiric antibiotic
• in all patients attempt to talk first to deemed safe) treatment (possible steroids first), based
calm and remove anything that might on age/likely etiology. Before any meds
cause injury Acute Mental Status Depression
given attempt to quickly determine if
Algorithm
allergic, from family, old records, etc.
• agitation, particularly in young patients
or possible drug toxicity/withdrawal – • fix the airway, breathing and circulation
• History acute fever, headache, with
administer Benzodiazepines. Avoid in conditions first
focal neurological signs or seizures,
elderly with dementia; likely to increase impaired immunity and/or exam/sono
• Check fingerstick glucose – if low
confusion. Monitor respirations in all. administer bolus or drip of D50/D25 or shows papilledema – do not perform
D10 depending on patient age. May immediate LP – check malaria smear,
• agitation, with signs of hypoxia,
give IM Glucagon if unable to start IV rapid HIV test, initiate empiric antibiotic
hypoperfusion – consider Ketamine
and patient cannot swallow. Administer treatment (possible steroids first), based
starting dose 1mg/kg with continued
Thiamine with the glucose. (Narcan on age/likely etiology. Before any meds
ABC resuscitation
should have already been given under given, attempt to determine if allergic,
• agitation, with a known history of section B). from family, old records, etc. Follow
psychiatric disorder or likely new-onset with CT and possible LP, ASAP.
psychiatric disease – administer • if GCS < 9 after ABC resuscitation – the
patient likely requires intubation to • consider status epilepticus in all non-
psychotropic agent, i.e., Haldol IV, IM.
protect from aspiration – prepare responsive patients, (motor signs may
with or without Benzodiazepine.
equipment be minimal) or if not awakening
between seizures:

92
check electrolytes – if blood/ xanthochromia, immediate Causes of critical neurological
hyponatremic administer 2cc/kg neurosurgery consultation, control BP < disorders
over 10 min of 3% NaCl (max 160/90. See SAH guidelines.
100cc) • conditions affecting airway, breathing
• Normal CT, likely thrombotic stroke – and/or circulation
Third trimester/post delivery – initiate TPA/endovascular therapy per
administer MgSO4/consult OB protocols, control BP to <185/110. If • metabolic disorders:
unable to use TPA, do not drop BP Hypoglycemia/hyperosmolar
likely INH OD or neonatal
unless >220/120. See thrombotic stroke coma/DKA
dependency – administer
guidelines.
Pyridoxine. thyroid disorders
• New intra-cerebral bleed on CT –
all others – start with electrolyte disorders (primarily Na+
control BP to <140/90; reverse
Benzodiazepines, consult and Ca++)
anticoagulants. See hemorrhagic stroke
neurology
guidelines.
liver/kidney failure, etc.
• if no improvement with above – obtain
• epidural/subdural/nontraumatic SAH on
head CT; follow management in the • drug toxicity/OD or drug withdrawal
CT – immediate neurosurgery
section below. syndromes
consultation for possible OR/IR
Focal Neurological Signs/AMS (with or intervention. • acute psychiatric disorders
without head trauma) and/or a Sudden,
• Evidence of acute herniation – raise the • mass lesions (hemorrhage, tumors,
Severe Headache Algorithm
head of bed 30-45 degrees (assuming abscesses)
• obtain a head CT in all patients, if no spine trauma), consider Mannitol,
3% NS, and/or mild, brief • infections – meningitis/encephalitis
available
(bacterial, fungal, viral, parasitic
hyperventilation. Consider IV
• normal CT, likely SAH by history (onset infections including cerebral malaria)
dexamethasone for a tumor with
> 6 hours), perform LP – nontraumatic herniation. • status epilepticus and post-ictal states

93
• stroke syndromes – thrombotic, • evidence of hidden bleeding – manage F – Fever (Extreme
intracerebral hemorrhagic, SAH as per Section C Temperature Disorders)
Clinically Assess
E – Exposure • evidence of clothes/skin contamination
Clinically evaluate – decontaminate, according to toxicity • skin warmth/coolness
and protect self and others in the
• areas hidden by clothing/body position process (self-protection should be • skin color (pale/red), dryness,
for missed lesions (rashes/stab/gunshot implemented at the onset of patient diaphoresis
wounds) by undressing and log rolling. evaluation)
• muscle rigidity, shivering
• the body for evidence of self/child/ • re-dress patient in a gown to prevent
• thyroid for nodules/enlargement
elder/domestic abuse and evidence of cooling and provide privacy
IV drug abuse. • obtain the history of medications
Equipment Needed for Exposure/
(recent psychotropic/succinylcholine,
• for possible contaminated clothing/skin: Decontamination
anesthetics, etc.), drug abuse,
substances absorbed through the skin
1. shower with containment for water endocrine disease, outdoor exposure,
(i.e., hydrocarbon pesticides), caustics,
runoff excessive exercise
radiation or objects causing continued
burns, etc. • Note: normal temperature is 98.6 F or
2. protective gowns, masks, gloves for
staff 37 C. Any temperature above 100.4 F or
Point-of-care testing
38 C rectally is considered a fever.
• none 3. i s o l a t i o n r o o m w i t h a i r v e n t H o w e v e r, i t i s t h e e x t re m e s o f
containment temperature that require emergent
Management Algorithm for Exposure
management, usually > 105 F (40.5 C)
Disorders 4. shears/metal cutter
or < 95 F (35 C)

• signs of child or self-abuse – provide


safe location and separate from
abusers
94
Point-of-care testing • convection (evaporation) methods, i.e., Hypothermia Algorithm
tepid water spray on skin and fan and/
• Thermometer: oral -affected by mouth • mild to moderate temperature decline –
or
breathing, drinking warm/cold fluids 30-35 degrees – external rewarm, i.e.,
• conduction heat loss by placing ices blankets and initiate warmed IV fluids,
• axillary – add a point to correlate with packs over major vessels, i.e., groin, and heated inspired air heated to 45
rectal temperature.
axilla or neck. (Ice tub immersion degrees

• rectal – most accurately reflects core possible, but not able to easily monitor.)
• severe, <30 degrees – consider
temperature.
• cool IV fluids a d d i t i o n a l c o re re w a r m i n g , i . e . ,

Emergency Equipment Required peritoneal lavage, thoracic lavage,


• if severe temperature elevations –
esophageal tubes, etc.
initiate core cooling: ice water lavage of
• fan
the bladder, thorax, stomach and • evidence of myxedema coma –
• ice packs prevent shivering with drugs such as administer thyroxine and
dexmedetomidine / Butorphanol. hydrocortisone, avoid rapid rewarming
• lavage tubes
• evidence of anticholinergic, Causes of critical heat/cold related
• warning blankets
sympathomimetic, MAOI poisoning – conditions
• rectal temperature probe consider antidotes.
• exposure to extreme environmental
Management Algorithm for Critical • evidence of neuroleptic malignant temperature conditions
Temperature Extremes syndrome – stop neuroleptics, consider
various antidotes See management of • endocrine disorders (especially hyper/
Hyperthermia Algorithm neuroleptic malignant syndrome. hypothyroidism)

• initiate heat loss for all by • evidence for thyroid storm – initiate b- • t o x i n s / O D ’s ( a n t i c h o l i n e r g i c s ,
blockade, cortisone, PTU, then iodine sympathomimetic, MAOI drugs, ASA,

last etc.)

95
• sepsis (for both extremes)

• neuroleptic malignant syndrome

• malignant hyperthermia associated with


anesthetics

After the sequence is completed, quickly


re-evaluate the patient to see if
intervention(s) resulted in improvement.

Then follow the ABC’s with:

• evaluation of past medical history,


medication history, and allergy history, if
not already performed

• perform the secondary survey (i.e.,


detailed history and a complete exam)

References and Further Reading, click


here.

96
Section 2

Abdominal Pain

Case Presentation
by Shaza Karrar A 39-year-old female presented to the emergency department
(ED) complaining of right-lower-quadrant (RLQ) pain; pain
duration was for 1-day, associated with nausea, vomiting, and
loose motions. Abdominal pain started centrally and was
described as diffuse and colic, 3 hours later it gradually shifted
to the RL and became continuous in nature. Her Last-
menstrual- period (LMP) finished a week ago. She denied any
regular medications, known allergies or using any
contraceptive pills. Also, she denied any past surgical history,
travel history, or eating outside. Upon examination, she was
found to be afebrile and vitally stable. The abdomen was soft,
non-distended, with RLQ Tenderness, positive rebound
tenderness, and positive bowel sounds.

Audio is available here

97
Table 3.1 Types of abdominal pain
General Approach and Critical Bedside
Actions

General Approach and Key Concepts


Abdominal pain is a common complaint presented in the ED. It
can range from benign, undifferentiated conditions to life-
threatening ones. The role of the Emergency Physician (EP) is to
identify life-threatening conditions for timely interventions. Life-
threatening conditions can stem from various systemic Critical Bedside Actions
pathologies (i.e. Cardiac, Gastrointestinal GI, Vascular, Urologic, A systematic approach, starting with a Focused Physical
Gynecologic, Infectious, etc.). As well, the EP has to assess Examination and History Taking is key to reaching a diagnosis
properly and dispose of the less serious pathologies; which, if and consequently definitive management.
done poorly, can propagate and lead to delayed presentations
Assessing the hemodynamic stability of the patient, pain
and higher morbidities (i.e., Gastroesophageal reflux disease
score, and overall distress is a primary step in your approach,
GERD, peptic ulcer disease, etc.). Special attention should be
as it would set your pace as a clinician gathering bedside
given to certain subgroups of patients including  the elderly, the
information and ultimately your workup and interventions
immunocompromised, females of childbearing age and children.
(Figure 3.1).
Types of Abdominal Pain
Keep in mind: You progress with assessing your patient’s
Abdominal pain can be either visceral, parietal or referred in
stability, history taking and examination, all simultaneously,
origin  (Table 3.1). In certain abdominal pathologies, abdominal
while advancing with any needed interventions that would
pain can typically begin as visceral, dull and diffuse, and then
stabilize your patient and alleviate his/her pain or distress.
progress to parietal, sharp and localizable, as in Appendicitis.
Understanding the basic physiology of abdominal pain can be of Do No Harm: Awareness of your limitations is extremely
immense aid in your assessment and approach. important. Acknowledging when your patient needs urgent
assessment by your supervising senior, especially in an

98
unstable patient, is part of your oath and a cornerstone of your Figure 3.2 Bedside actions
practice during training years.

Figure 3.1 Approach to abdominal pain

Differential Diagnoses
Abdominal pain can originate from intra-abdominal and extra-
abdominal or non-GI conditions; hence, it’s advisable to be
systematic in your approach to narrow down your differential
Bedside actions are taken in patients presenting with diagnoses. (i.e., Cardiac, GI, infectious, hematologic, urologic,
abdominal pain and are tailored to each patient’s clinical gynecologic, etc.) When diagnosing abdominal pain, the
picture; those include (Figure 3.2), detailed further in the differential diagnoses can be based on anatomic localization of
chapter. pain (Figure 3.3).

99
This, in turn, helps direct your approach. An EP should prioritize
Figure 3.3 Differential diagnoses according to location
possible life-threatening conditions in his differential diagnoses,
and be mindful of other possible extra-abdominal causes
attributable to abdominal pain (Table 3.2).

Table 3.2 Differential diagnoses in abdominal pain

Keep in mind that the following list of differentials is not


exhaustive and should be correlated with the localization of
abdominal pain (Figure 3.3). 

100
History Taking and Physical Examination Associated symptoms: Be systematic – Fever, nausea,

Hints vomiting, loose-motions, melena, jaundice, dysuria, vaginal


discharge/bleeding, shortness of breath, etc.
History Taking Hints
A properly focused history is of paramount importance, as it Past medical, surgical and gynecologic/obstetric history:
guides your physical examination, differential diagnoses, critical Known GI pathologies, chronic comorbid, recent trauma,
interventions, work up, and ultimately your disposition. Specific previous similar complaint, previous surgeries, LMP
history findings can lead to prompt and accurate diagnosis and
Social history and Travel history: Alcohol consumption, illicit
management.
drug abuse, occupational hazards or exposures
History taking should cover the following
Medications and allergies history: Anticoagulants,
General impression: level of distress, stability and pain scale. corticosteroids, contraceptives.

Patient Demographics: Gender, age. Others: Clues of an atypical presentation – considering Extra-
Abdominal and non-GI pathologies including weight loss,
Onset and progression of pain: Sudden, gradual, episodic/ unspecific systemic symptoms, etc.
intermittent, continuous.
Always try to screen for high-risk patients; certain history findings
Localization of pain and radiations (Figure 3.3). can help narrow your differential diagnoses. Table 3 demonstrates
history findings, coinciding with their potential differentials.
The character of pain: Dull, sharp, colicky, stabbing, burning

Severity: Pain Scale – mild, moderate, severe, or a scale from Physical Examination Hints
A focused and systematic Physical Examination (P.E.) aims at
1 to 10
verifying your clinical impression constructed from your history
Alleviating and aggravating factors: Relieved by eating, sitting findings; it also aids in exposing unforeseen findings that may
up, or worsens after eating, lying supine, movement, and make you reassess your differential diagnoses and approach.
coughing, etc. Hence, an EP must have in mind specific working differentials
obtained from the history before examining the patient (Figure 3.3

101
and Table 3.3). Make sure that your patient is comfortable with • Note any instability in vitals, consciousness level, the
adequate pain relief as necessary, suitably positioned and posture of the patient, hydration status and signs of pallor,
appropriately exposed from nipples to lower abdomen/pubic- jaundice.
symphysis in a properly private environment. In the case of
Examination of extra-abdominal systems:
children, the presence of family members can aid your
examination. Assessment of vital signs, hemodynamic stability, • Entails an assessment of the cardiorespiratory functions.
and signs of shock should be noted, keeping in mind that normal
vital signs would not rule out a life-threatening condition. • Other extra abdominal systems of suspicion attributing to the
patient history and clinical picture.

Table 3.3 History findings Focused abdominal examination:

• Inspection: Distention, surgical scars, masses, distended veins,


skin discoloration.

• Palpation:

• The light then deep palpation begins with the opposite non-
tender quadrant, progressing through all quadrants.

• Assess for tenderness, guarding, rebound tenderness,


hepatosplenomegaly, and masses; for instance, a rigid, tense
abdomen with involuntary guarding is highly indicative of
peritonitis. Look for costovertebral angle tenderness and
Henceforth, a P.E. would be based on a focused and systematic
palpate
approach, consisting of:
• Look for costovertebral angle tenderness and palpate
General appearance and vital signs assessment:
bimanually for renal masses.

• Percussion: Dullness for ascites or Resonance.


102
• Auscultation: Hyper/hypoactive or absent bowel sounds and • Examination of hernial orifices: All should be cleared; easily
vascular bruit. missed in the exam.

• Specific signs and maneuvers: (Table 3.4) • Genital and pelvic examination:

• For an overview of the focused Abdominal Examination, • Pelvic exam in all females with lower abdominal pain and
please watch the following video. query pelvic pathologies.

• Serial abdominal examinations are important to reassess • Genital exam in males with possible testicular pathologies
your patient’s progress and response to treatment. and hernias.

• Rectal examination: In suspected GI bleeds, perianal and Emergency Diagnostic Tests and
prostatic disease, foreign bodies, and impacted stools. Interpretation
Table 3.4 Abdominal signs Bedside tests
• 12-lead ECG

• Initial screening for cardiac pathologies (i.e., Acute Coronary


syndrome; MI, AFib, Digoxin toxicity, etc.), especially in
patients presenting with epigastric pain, cardiovascular risk,
and elderlies with poorly localizable pain. Consider follow-up
cardiac enzymes in high-risk patients and abnormal ECG
findings.

What is your opinion about below ECG in a patient with


abdominal pain?

103
Image 3.1 ECG • B l o o d - U r e a - N i t r o g e n (BUN): Assess dehydration.

• Creatinine / Renal Function: Assess renal functions,


dehydration, and record a baseline in case of needed contrast
imaging studies to prevent possible contrast-induced-
nephropathy.

• Liver function test (LFT): Liver and gallbladder pathologies

• Urine Pregnancy test: All females of childbearing age, • Amylase: Increased in most intra-abdominal pathologies.
regardless of history findings
• Lipase: Levels twice the normal is highly indicative of
• Urine analysis: Signs of hematuria or Urinary Tract Infections pancreatitis, joined with elevated LFT, could raise the suspicion
(UTI). of Gallstones pancreatitis.

• Capillary Glucose test: Hyper/Hypoglycaemia and DKA • Inflammatory markers: CRP or Procalcitonin

• Ultrasound • Coagulation profile: Patient on anticoagulation (i.e., Warfarin for


AF) – if reveals sub-therapeutic levels, would raise suspicion of
• Considered the new EP stethoscope – bedside, non-
mesenteric ischemia, or if supra-therapeutic, would raise
invasive, yet operator-dependent
suspicion of active bleed, as well a standard test for possible
• Aids at assessing solid abdominal organs, the presence of surgical interventions; in need of reversal.
intraperitoneal fluid, vascular and hemodynamics of the
• Quantitative hCG Blood: Confirmatory after a urine pregnancy
patient
test

Laboratory tests • TestType and screen: For all patients possibly proceeding for
• Complete-Blood-Count: Leukocytosis, Hemoglobin level, surgical interventions
Platelets count
• Rh Status: All female patients with possible ectopic pregnancy
• Electrolytes: Correct any derangements caused by fluid losses.
104
Imaging modalities Ultrasound (US)
X-rays
• Extended sonographic studies can further your assessment;
• An initial imaging modality, quick yet not very sensitive considered the study of choice in pregnant women.

• Chest X-rays: Delineate Air under the diaphragm in perforated • Abdominal US: Evaluation of biliary tract pathologies,
viscus or pneumonia. intraabdominal organs, and free intraperitoneal fluid,
Intussusception, appendicitis, etc.
• Abdominal X-rays: Usually in erect and decubitus positioning;
looking for Bowel distention, air-fluid levels, obstruction, foreign What is your opinion about below ultrasound samples in a patient
bodies. with abdominal pain?

What is your opinion about below abdominal x-ray in a patient Image 3.3 Abdominal US, RLQ
with abdominal pain?

Image 3.2 Abdominal x-ray

• K i d n e y - U r e t e r- B l a d d e r K U B U S : N e p h r o l i t h i a s i s ,
hydronephrosis, urine retention, etc.

• Pelvic or Obstetric US: Tubo-ovarian pathologies, intrauterine


and ectopic pregnancy, fibroids, etc.

• Testicular US: Testicular torsion, epidedimo-orchitis

105
• Vascular US Studies: assessment of the abdominal aorta
Figure 3.4 Resuscitation of unstable patient
Computed tomography (CT) scan: (With/Without Contrast,
Angiography)

• Highest in sensitivity and specificity in detecting most common


pathologies

• Ionizing and considered limited in the case of contrast allergies,


pregnancy, or pre-existing renal insufficiency predisposing the
patient to contrast-induced nephropathy.

Magnetic resonance imaging (MRI):

• Limited use in the ED, considered in pregnant patients of


inconclusive US findings.

Emergency Treatment Options


Initial Stabilization Appropriate surgical consultations should be sought in a
Primary survey: Hemodynamic stability assessment timely manner, especially in the hemodynamically unstable
subsequently sets the pace of your approach (Figure 3.1). patient and surgical conditions in need of an intervention.

Secondary survey: Identify life-threatening conditions (Table A hemodynamically stable patient should be properly
3.2), and screen for the high-risk patients, and special age worked up and reassessed frequently, as he/she may
groups (Table 3.3). deteriorate and become unstable.

Hemodynamically unstable patients should be resuscitated Patients with possible peptic ulcer disease (PUD) and
without any delay, entailing the following keystones (Figure gastritis can benefit from a “GI cocktail,” typically
3.4).

106
constituted of a combination of antacid, viscous lidocaine, A b d o m i n a l p a i n should be addressed with liberal
and antispasmodics. analgesia, including the use of opioids, as it has been
recently proven that they do not alter physical exam findings
Otherwise, stable patients who are responsive to treatment,
or increase the number of incorrect management decisions.
with conditions of low acuity, can be fit for discharge from
the ED with an appropriate disposition and follow-up plan. Opiate-dependent patients and chronic users may need
higher doses for an adequate response.
Medications
Analgesics should be tailored to each patient’s clinical Always monitor patients for respiratory depression with
picture, pain score, and response, with an aim to relieve his opioids and always consider dose adjustments in geriatrics
distress and pain to a manageable level, making him more and patients with renal and hepatic impairments.
comfortable and cooperative for the abdominal exam and
NSAIDs like Ketorolac are suitable for biliary and renal colic
reassessment (Table 3.5).
but not in PUD.

Procedures: None
Table 3.5 Table 5. Medications in abdominal pain
Pediatric, Geriatric, Pregnant Patient and
Other Considerations
Pediatric Patient
• The list of differential diagnoses tends to rearrange in acuity
according to the age at presentation in pediatric patients. For
example:

• 0 to 3 months: testicular torsion, necrotizing enterocolitis,


incarcerated hernia and pyloric stenosis

• 3 months to 3 years: intussusception, vaso-occlusive crisis, UTI


and toxic megacolon, among those just stated above
107
• 3 years to adolescence: appendicitis, mesenteric lymphadenitis, Pregnant Patient
DKA and toxic ingestion, as well among those just stated above • Females of childbearing age presenting with abdominal pain are
presumed to have an ectopic pregnancy until proven otherwise,
• When it comes to examining for peritoneal sign in pediatrics,
regardless of their history.
children can be asked to jump up and down as an indirect
means of inciting peritoneal irritation. • Pregnant women are at risk of the same abdominal conditions
as all other patients, but they propose special considerations.
Geriatric Patient
• Elderly and immunocompromised patients deserve special • Restricted imaging necessitates astute work up and
attention as a higher risk group compared to their younger reassessment.
counterparts; they are more likely to present late in the disease
• Consider first-trimester abortion in abdominal pain in a
process and with atypical presentations.
pregnancy < 20 weeks of gestation.
• The elderly commonly present with multiple comorbidities,
• In pregnant women well advanced in their pregnancy, consider
difficult communication skills, and vague symptoms that are not
obstetric emergencies, such as Preterm-Labor, Placenta Previa
reflective of the actual severity of their disease.
and abruption, Uterine Rupture and HELLP syndrome.
• It’s important to establish their medication and allergies history
(Anticoagulants, etc.) Other considerations
• Appendicitis should be considered in all patients presenting
• They cannot tolerate volume loss and hence can deteriorate with acute abdominal pain, refer to Alvarado Score.
easily, and their abdominal exam is often unspecific; however,
• Abdominal trauma, blunt and penetrating, results in a myriad of
they’re the most likely population to have surgical emergencies.
intraabdominal injuries that shouldn’t be taken lightly but well
investigated and addressed.

108
Disposition Decisions • Patients with an unclear etiology of pain who are stable
enough for discharge should be reassessed again within 12 to
Admission criteria 24 hours; hence, a follow-up plan should be instituted and
• Hemodynamically unstable patients require admission to the emphasized to the patient.
Intensive Care Unit (ICU), especially post possible surgical
interventions. Referral
• Follow up plans in patients that are discharged decrease high
• Elderly patients with multiple comorbidities or anticipated
morbidity and mortality, a chance of a missed diagnosis, and
clinical course deterioration require an ICU admission as well or
decrease unnecessary ED presentations of benign and low
a High Dependency Unit (HDU) if available.
acuity abdominal pain.

• Hemodynamically stable patients found to have surgical


• Follow up plans are essential and have to be arranged for
conditions can be admitted to a general ward.
certain patients. Those include:

• The conservative and low threshold for admission is strongly


• Patients in need of the re-evaluation of their symptoms’
advised and advocated in the high-risk groups.
progression and resolution; particularly patients at higher

• Certain stable patients might need to be admitted for further risk.


evaluation and planned interventions.
• Patients with an unclear pathology that needs a further
workup by multidisciplinary services – vascular, urology, OB/
Discharge criteria
GYN, gastroenterology, etc.
• Patients who are responding to treatment, with a resolution of
symptoms, without signs of underlying life-threatening
• Patients with known GI pathologies or comorbidities, who
conditions, can be discharged. did not follow-up, are in need of reinstitution of their primary
or specialty care.
• Always instruct your patients to return to the ED in the case of
progression of abdominal pain or worsening general condition,
• Pregnant women should resume their primary care
which is not getting relieved, such as persistent vomiting, obstetricians, with an advised reassessment as soon as
jaundice, fever, etc. possible post discharge.

109
• Yet pregnant women are strongly instructed to return to the
ED in case of recurred, progressing or persistent symptoms,
especially in cases of uncorrected volume loss due to
vomiting and loose motions, vaginal bleeding/discharge, and
abdominal pain.

References and Further Reading, click here.

110
Section 3

Altered Mental Status

Case Presentation
by Murat Cetin, Begum Oktem, Mustafa Emin An 80-year-old female presents to the emergency department
Canakci 
with a tendency to sleep (altered mental status), failure in
recognizing people and answering questions. She is a
nursing-home inhabitant. The caregivers express she was
feverish and fatigued for several days now, but her mental
problems have recently begun. The patient has a history of
hypertension and diabetes mellitus. Her only routine
medications are angiotensin-converting enzyme inhibitors
(ACE inhibitors) and insulins. Vital Signs: Blood Pressure:
110/70 mmHg, Heart rate: 110 bpm, respiratory rate: 20 rpm,
temperature: 38.8 degrees Celsius, peripheral capillary oxygen
saturation: 98%, finger-stick blood sugar: 95 mg/dL. Physical
Exam: She is in mild distress, lethargic and confusional with
no lateralizing signs. The pupils are reactive to light and equal
Audio is available here in size. On a Glasgow Coma Scale, she is registered at 12 (E3,

111
M5, V4) and she had neck stiffness. The heart is • A combination of these two functions.

irregularly tachycardic with no abnormal cardiac The altered mental state may mean coma, confusion, aggression,
sounds. The breath sounds are clear and equal personality alteration, or difficulty in awakening. Approximately
3% of patients in the emergency department have impaired
bilaterally. The abdomen is soft, non-tender, non-
mental status. In the elderly patients, this rate is between 10%
distended. Skin: warm, dry, no rash. A lumbar and 25%. 85% of patients have metabolic and systemic
puncture is performed to diagnose or exclude diseases.

meningitis. Dementia and delirium should be differentiated in patients with


altered mental status. Metabolic, infectious and vascular
General Approach and Critical Bedside pathologies should be evaluated.
Actions
Critical Bedside Actions
The initial evaluation must start with the evaluation of airway,
General Approach and Key Concepts breathing, and circulation (ABC) sequence.  Life-threatening
The state of consciousness is a sum of arousal and cognition.
situations require rapid intervention. During the initial assessment,
Arousal refers to awareness of the self and the environment. The
the reversible causes must be addressed and managed such as
Ascending Reticular Activating System in the brainstem
hypoglycemia or opioid overdose. A continuous cardiac
modulates arousal. The Cognition is the combination of
monitoring and vascular access must be established as soon as
orientation, reasoning, and memory. The cerebral cortex houses
possible.
the cognition centers. In sum, the normal state of consciousness
requires a properly functioning brain stem and cerebral cortex. A (Airway): Hypoxia is a reversible cause of altered mental
status. Ensure the airway is open and protected. If not, secure
Altered mental status may result from any changes in;
the airway and give oxygen if necessary.  We prefer oxygen
• Arousal, saturation maintained above 94%.

• Cognition, B (Breathing): Inadequate ventilation may cause hypercarbia. If


the respirations are inadequate or superficial, perform bag-
112
mask ventilation (BVM) or endotracheal intubation. The h i s t o r y t a k i n g a n d focused physical examination is key
combination of altered mental status and respiratory to reaching a diagnosis and consequently definitive
depression may suggest narcotic/opioid overdose. management.

C (Circulation): Hypoperfusion may cause altered mental status Differential Diagnoses


due to decreased oxygen and glucose in the brain. Check the Metabolic, infectious, toxicological, endocrine, hypoxic conditions
distal pulses, blood pressure, and cardiac rhythm. Capillary may cause altered mental status. The mnemonic “AEIOU-TIPS”
refill time, skin color and temperature may also help to may help to remember most common causes of altered mental
understand possible shock situation. If the patient is status (Table 3.6).
hypotensive, administer IV fluids and investigate the cause.
History Taking and Physical Examination
D (Mini neurological evaluation): Assess Glasgow Coma Score
Hints
(GCS) or AVPU quickly. Check if the pupils are equal and
reactive to light. Administer benzodiazepines (lorazepam or History Taking
diazepam, based on availability) in case of ongoing seizure Obtain patient’s medical history directly from the patient.
activity. Cervical stabilization should be provided if trauma is Determine if the level of awareness is decreased. If the patient is
suspected. unable to provide the necessary information:

E (Exposure): The findings should be evaluated in terms of • Ask a family member, caregiver, or medical personnel
trauma, transdermal drug tapes, dialysis intervention area,
• Check for medical alert identification
sources of infection and petechiae.
• Ask for medical information sheet (i.e., on the refrigerator)
Glucose level, ECG should be performed. Bedside ultrasound
(eFAST or RUSH protocols) should be added to the • Ask surrounding environment (i.e., living quarters, alcohol
investigation of patients with shock or trauma. bottles or drug paraphernalia).

The level of hemodynamic stability of the patient sets the pace


of the investigation. A systematic approach, starting with

113
Altered mental status is a result, not a diagnosis. The diagnosis
Table 3.6 Mnemonic AEIOU-TIPS for Altered Mental Status
is based on clinical suspicion. The cause may be transient
MNEMONIC THINGS TO CONSIDER
(seizure) or permanent (stroke), benign or life-threatening. If not
Alcohol Alcohol levels, serum osmoles treated timely and accurately, most causes may be mortal or
Epilepsy/ Endocrine/ EEG, referral to neurology, TFTs, cause neurologic sequelae. The systematic and structured
Electrolytes/ cortisol, chemistry panel, LFTs/NH3
approach makes diagnosing and management easier.
Encephalopathy
Insulin Glucose
Physical Examination Hints
Oxygen/ Opiates SatO2%, ABG, hypoxia makes A focused and systematic physical examination aims at
agitation, hypercarbia makes
somnolence confirming the clinical impression formed by the history. It also
Look for needle marks aids exposing unexpected findings that may make the clinician
Uremia BUN/Cr reassess the differential diagnoses and approach. Repetitive
Things changing serum osmolarity
examinations should be performed to track changes.
affect mental status. Uremia, Sugar,
Alcohol are common ones
• Vital signs should be evaluated very carefully in terms of
Trauma/ Temperature CT Head, C-Collar, CT C-Spine
hypotension, hidden shock, hypoxia, respiratory rate and
Infection CBC, BCx, UA, UCx, CXR, LP/CSF
pattern, and temperature.
Sepsis and CNS infections are more
important. But, even simple fever may
cause AMS in elderly and kids • Head: Signs of trauma, pupils’ size and reaction to light, cterus,
Poisoning/ Psychosis Drug Levels (e.g. lithium, digoxin) pale conjunctiva

Shock/ Stroke/ SAH/ Space ECG, Troponin, CT Head, LP • The fundoscopic exam may show hemorrhage, papilledema
occupying lesion
• Neck: Rigidity, bruits, thyroid enlargement
If the normal state of the patient is unclear, all changes must be
evaluated as if they are acute. Strokes, seizures, cardiac events, • Heart and Lungs may show heart failure, pneumonia findings
intoxication, psychiatric disorders cause sudden changes,
• Abdomen: Organomegaly, ascites
whereas; infections, metabolic disturbances, or an expanding
intracranial mass may cause gradual changes. • Extremities: peripheral cyanosis
114
• Skin: Diaphoretic/dry, rash, petechiae, ecchymoses, splinter Table 3.8 Full Outline of Un-Responsiveness (FOUR) Score
hemorrhages, needle tracks EYE MOTOR BRAINSTEM
RESPIRATION
RESPONSE RESPONSE REFLEXES
• Neurologic exam should be done in order of GCS (Table 3.7),
4: Eyelids open 4: Thumbs-up, 4: Pupil and 4: Not
FOUR score  (Table 3.8), pupil dimensions, neck stiffness, or opened, fist, or peace corneal reflexes intubated,
lateralizations .  In the secondary evaluation, the full tracking, or sign present regular
neurological exam should be applied. The mental status exam blinking to breathing
command pattern
should be the main part of neurological exam and repeated as
3: Eyelids open 3: Localizing to 3: One pupil 3: Not
needed. but not tracking pain wide and fixed intubated,
Cheyne-Stokes
breathing
Table 3.7 Glasgow Coma Scale pattern
BEST VERBAL BEST MOTOR 2: Eyelids 2: Flexion 2: Pupil or 2: Not
EYE OPENING
RESPONSE RESPONSE closed but open response to corneal reflexes intubated,
to the loud pain absent irregular
4: Spontaneously 5: Oriented and 6: Obeys command
voice breathing
converses
1: Eyelids 1: Extension 1: Pupil and 1: Breaths
3: To verbal 4: Disoriented and 5: Localizes pain
closed but open response to corneal reflexes above ventilator
command converses
to pain pain absent rate
2: To pain 3: Inappropriate 4: Flexion withdrawal
0: Eyelids 0: No response 0: Absent pupil, 0: Breaths at
words; cries
remain closed to pain or corneal, and ventilator rate
1: No response 2: Incomprehensible 3: Flexion abnormal with pain generalized cough reflex or apnea
sounds (decorticate) myoclonus
status
1: No response 2: Extension
(decerebrate) Gujjar AR, Jacob PC, Nandhagopal R, Ganguly SS, Obaidy A, Al-Asmi AR.
Full Outline of UnResponsiveness score and Glasgow Coma Scale in
1: No response medical patients with altered sensorium: interrater reliability and relation to
outcome. J Crit Care. 2013 Jun;28(3):316.e1-8. doi: 10.1016/j.jcrc.
2012.06.009. Epub 2012 Aug 9. PubMed PMID: 22884530. - Please read
Glasgow Coma Score (GCS) (Modified from Teasdale, G., & Jennett, B. this article to get more insight about this score.
(1974). Assessment of coma and impaired consciousness: a practical scale.
The Lancet, 304(7872), 81-84.) - Please read this article to get more insight
regarding GCS. 
115
Emergency Diagnostic Tests and Arterial/venous blood gas investigation is necessary for many

Interpretation critically ill patients to understand acidosis/alkalosis, hypercarbia,


Differential diagnosis of altered mental status is broad, and hypoxemia, carboxy-hemoglobinemia, methemoglobinemia,
diagnostic tests should be targeted to suspected underlying lactate, and base excess situation. 
pathologies. Rather than a single specific algorithm, a ruling in
Complete blood count  can be used to search for Anemia/
and out approach should be followed.
Polycythemia in hyperviscosity related pathologies, Leukopenia/
Leukocytosis in Infection and sepsis, Thrombocythemia /
Bedside tests
• Rapid glucose: Quick and easy. Hypoglycemia is an emergent Thrombocytosis in spontaneous intracranial hemorrhage.
cause and can be ruled out with this simple test, just in
Renal function tests (RFT) for uremia suspicion caused by acute
seconds.  Glucose level should be measured at the bedside to
or chronic renal failure. RFT can also be necessary to understand
understand hypo/hyperglycemia problems including diabetic
patients’ baseline before to order contrast-enhanced imaging or
ketoacidosis and hyperosmolar hyperglycemic state.
use some drugs. 

• ECG may help to understand arrhythmias and some toxic Liver function tests are useful in the suspicion of hepatic failure,
effects of drugs such as TCA overdose.
hepatic encephalopathy or biliary tract problems. 

• Bedside US such as eFAST for trauma patients and RUSH Electrolytes are essential to evaluate altered mental status. There
protocol for nontraumatic hypotensive patients can be valuable are many electrolyte abnormalities can change the level of
to understand the underlying causes. Some other US consciousness such as hypo/hypernatremia, hypo/hyperkalemia,
applications can be used for altered mental status cases such hypo/hypercalcemia.
as optic nerve sheet diameter measurement to understand
increased intracranial pressure. Urinalysis shows ketone bodies of diabetic ketoacidosis and
provides information about urinary tract infection. 
Laboratory tests
Many laboratory tests can help the management of altered mental Thyrotoxicosis and myxoedema coma are important endocrine-
status cases. These tests and their possible findings were given related causes of altered mental status. Therefore thyroid function
below. tests can be valuable in some circumstances. 

116
Cardiac enzymes can be necessary because myocardial Image 3.4 CT head
infarction may cause a low ejection fraction or trigger arrhythmias
and patients may show altered mental status.

Also, when needed:

• Drug screening for a suspected overdose

• Levels of specific medications such as antiepileptics,


antipsychotics, digoxin, warfarin

• Ethanol level, levels of toxic alcohols

• Cerebrospinal fluid (CSF) tests after a lumbar puncture (LP)

• Cultures (Blood, urine, CSF, etc.)

Imaging modalities
• Head CT: Non-contrast CT for ruling out hemorrhage, mass
effect, edema.

What is your opinion about below CT Head image in a patient


with altered mental status?
Case – Motor vehicle accident, head trauma, GCS 9

• Head MRI: In selected patients (e.g., ischemic stroke, sinus


venous thrombosis)

What is your opinion about below MRI images in a patient with


altered mental status?

117
Image 3.5 MRI • E E G : I f t h e r e i s a suspicion for seizure, non-convulsive
status epilepticus, etc.

• Chest radiogram: When indicated, for evaluating suspected


cardiac and pulmonary conditions (e.g., pneumonia, pleural
effusion)

What is your opinion about below chest x-ray in a patient with


altered mental status?

Image 3.6 Chest x-ray.

Case – 3 days history of fever, headache and gradual altered


mental status

Case – 60 yo female, 7 days gradual history of dyspnea, cough, sputum


and fever. Today, altered mental status.
118
Emergency Treatment Options Diagnostic tests and procedures such as performing LP
must not delay antibiotics administration.
Initial Stabilization
As we mentioned above, “ABC” is the first step in the evaluation. Medications
Life-threatening conditions detected during these steps must be • Dextrose: For hypoglycemic patients, 50 mL D50W.
immediately intervened. After initial stabilization, a detailed
• Thiamine: For Wernicke’s encephalopathy, 100 mg IV
examination must be performed and the underlying cause(s) must
be investigated. Treatment strategies vary according to the • Naloxone: As an antidote for opioid overdose, 0.4 mg IV, can be
underlying condition. repeated up to a total dose of 10 mg according to some
resources.
• If hypoglycemia is detected, dextrose (preferably D50W) should
be administered intravenously. • Glucocorticoids: When cerebral edema due to CNS mass
lesions is detected in cranial CT
• In undifferentiated comatose patients, A “coma cocktail” can be
given. Oxygen is also in this category. A mnemonic called • Fluid resuscitation for indicated situations (e.g. hypotension,
DONT stands for dextrose, oxygen, naloxone, and thiamine. dehydration, DKA)
The application of coma cocktail was changed over time. Today,
we have bedside gluco-stick tests. Therefore, application of Specific antidote should be given in intoxications. Choose

glucose mainly depends on the bedside glucose results. appropriate empiric antibiotics for the suspected source of

Similarly, we are using portable saturation devices, depending infection and possible microorganisms. Broad-spectrum

on the SatO2 level measured on the bedside, oxygen antibiotics are an option.

application decisions may change. Therefore, these drugs may Intravenous insulin infusion for diabetic ketoacidosis or
not be routine blindly anymore. In addition, it is better to hyperosmolar hyperglycemic syndrome. Electrolyte imbalances
emphasize that flumazenil (a benzodiazepine antidote) is not in must be corrected using appropriate fluid replacements.
the coma cocktail, and should not be used routinely also.
Procedures
• Empiric antibiotics must be initiated as soon as possible if
• LP: Indicated if central nervous system infection or or
sepsis or central nervous system infection is suspected.
subarachnoid bleeding (with negative CT scan)  is suspected.
119
Pediatric, Geriatric, Pregnant Patient, and A-Accident/abuse (e.g., trauma with a suspicious history)

Other Considerations M-Metabolic (e.g., glycemic abnormalities, electrolyte imbalance)


Advanced age is an independent risk factor for acutely altered
mental status. Almost half of the patients presenting ED with I-Intussusception
altered mental status are elderly. Neurologic etiology is more
N-Neoplasm
common in the geriatric population and is the leading cause.
Having multiple diseases and using multiple medications obscure S-Seizure
the diagnostic process. When renal or hepatic functions are
impaired, patients become more prone to drug intoxication In pregnant patients, in addition to the etiologies mentioned
without overdose. Moreover, drug interactions should be kept in above, eclampsia must be kept in mind. Eclampsia, which is life-
mind, especially when a new medication is initiated. threatening both for the patient and fetus, is usually seen in the
third trimester but can be seen during postpartum period as well.
In the pediatric population, symptoms and clinical findings may All pregnant patients with seizures or altered mental state must
be very subtle and non-specific. Obtaining a detailed history, be investigated for eclampsia. Other conditions with increased
learning the child’s baseline and shifts from that baseline, and risk during pregnancy are cerebral venous thrombosis, ischemic
repeated physical examinations are essential. and hemorrhagic strokes.

Etiology of altered mental status in the pediatric population differs


Disposition Decisions
from the one in the adults. In a child with an altered mental status, The decision on discharging or admitting the patient depends on
the mnemonic “VITAMINS” is helpful: the underlying cause of mental status change and the clinical

Mnemonic VITAMINS situation of the patient. The majority of the patients require
admission, either to the ward or intensive care unit.
V-Vascular (e.g., AV malformations, vasculitis)
If the underlying cause is completely reversed and unlikely to re-
I-Infection (e.g., sepsis, meningitis, encephalitis) occur, the patient reached his/her baseline mental status, vital
signs are normal and stable, preparing a discharge plan may be
T-Toxins (e.g., drugs, environmental toxins)
considered.

120
References and Further Reading, click here.

121
Section 4

Cardiac Arrest

A 23-year-old female was brought into the emergency


by Abdel Noureldin and Falak Sayed
department. Her frantic family members said they found her
on the bathroom floor, not breathing, unresponsive, and with
no pulse. EMS brought the patient to the ED.

Spanish translation is available here

Audio is available here

122
Introduction C a r d i a c A r r e s t Rhythms
Cardiac arrest is a condition that every emergency physician must The pulseless cardiac arrest is caused by 4 different types of
be an  expert in managing.  The EM doctor will face it and have a primary arrhythmias that consist of 2 shockable rhythms
love/hate relationship with it.    You love it when the patient is (ventricular tachycardia and ventricular fibrillation), and 2 non-
resuscitated and breathing on his own; you can then tell the shockable rhythms (pulseless electrical activity and asystole).
family their loved one is alive.    You feel great because, after all,
we are here to save lives, and it’s the reason we joined this Shockable rhythm
specialty.    You hate it when your eyes are tearing up as you
Ventricular Tachycardia
inform the parents of the newborn that he or she did not make it. Ventricular tachycardia (VT) has 2 different types. The most

This illness is due to the lack of effective perfusion of the organs common is the monomorphic (VT) and is defined as 3 or more

of the body 2nd  to the abrupt failure of the heart to pump consecutive ectopic ventricular beats (QRS complexes) of the

blood.    Every year, over a quarter of a million lives are lost same type.

because of cardiopulmonary arrest, and most of these cases


Image 3.7 Ventricular Tachycardia
occur outside of the hospital.   However, the mortality rate can be
improved with the early and effective initiation of cardiopulmonary
resuscitation (CPR) and advanced cardiopulmonary life support
(ACLS).

The 2nd type is the polymorphic (Torsade’s De pointes) that


consists of ectopic ventricular beats (QRS complexes) of different
types of morphology.

123
Image 3.8 Torsade De Pointes N o n - s h o c k a b l e rhythms
Pulseless Electrical Activity
Pulseless electrical activity (PEA) shows organized electrical
rhythm on the electrocardiogram with no mechanical contractions
of the heart muscle (no pulse). It is also called electromechanical
dissociation.

Image 3.10 PEA

Ventricular Fibrillation
Ventricular fibrillation is rapid and unorganized electrical impulses
which makes the ventricles of the heart quiver while no pumping
of the blood occurs.

Image 3.9 Ventricular Fibrillation

Asystole
Asystole is defined as no electrical activity in the heart and no
mechanical contraction of the heart muscle (no pulse). It is also
called flat line or cardiac standstill.

124
Pediatric dosage 5 mg/Kg (can be repeated up to 300 mg)
Image 3.11 Asystole
Adult dosage 300 mg (can be repeated at 150 mg)

Indication shockable rhythm (VT and VF)

Frequency 1st dose after the 3rd shock and repeat dose after the
fourth defibrillation

Mechanism: Class III antiarrhythmic drugs.


Medications for Cardiac Arrest
Thara are only 3 emergency drugs you should now in any cardiac Magnesium
Pediatric dosage 25 to 50 mg/Kg (maximum 2 grams)
arrest patient. These are Epinephrine, Amiodarone, and
Magnesium. Adult dosage 1 to 2 grams

Epinephrine Indication Torsade de Pointes


Concentration 1:10,000
Frequency Once when the diagnosis is made
Pediatric dosage 0.1 mL/Kg (20 kg child = 2 mL)
Mechanism: Shorten the prolonged QT interval
Adult dosage 10 mL or basically 1 mg
Imperative Concepts for The Team During
Frequency Every 2 cycles (3 to 5 minutes)
Cardiac Arrest Management
Indication All pulseless cardiac arrest rhythms
The compressors
Must push hard (2 to 2.4 inches or 5 to 6 cm)
Mechanism : An agonist for the beta and alpha receptor which
increases the perfusion pressure in the coronary and cerebral Should push fast (100 -120/minute)
vessels
Ought to allow the chest to recoil completely
Amiodarone
125
Rotate with another person every 2 minutes Amiodarone 2nd dose: up to 15 mg/kg (max-300) and
150mg adult
Airway
Do not hyperventilate Magnesium: 25 to 50 mg/KG- pediatric and 1 to 2 grams –
adult
2 ventilations to 30 compressions while using ambo bag
Recorder
Give one breath every 6 seconds when the patient is intubated
Must record all the drugs given and the time it was given
Use wave capnography to monitor CPR (CO2 should be >10)
Inform the team members at the end of each cycle
If the advance airway is needed, use supraglottic devices or
Keep track of the total time of resuscitation
endotracheal intubation
Team leader
Confirm endotracheal intubation by wave capnography
Must have mutual respect for all members of the team

Shock delivery Look for ROSC (RETURN OF SPONTANEOUS CIRCULATION)


Biphasic – 200 Joules
• The sudden increase in the PETCO2 (>40)
Monophasic – 360 Joules
• Return of pulse and pressure
Make sure everyone is clear before you shock the patient
Make sure the interruption of chest compression is <10
Attach the patient to the monitor
seconds

Drug therapy Remember the reversible causes (Hs and Ts)


Start IV or IO
• Hypovolemia
Epinephrine: (1:10,000) 0.1ml/KG for pediatric and 10 ml for
adult • Hypoxia

Amiodarone: 5mg/kg for pediatric and 300 mg for adult • Hydrogen Ion (acidosis)
126
• Hypo-hyperkalemia Diagram 3.1 Cardiac Arrest
Management Made Easy
• Hypothermia If the patient developed a pulse at any
time during resuscitation
• Tension pneumothorax
• Stop CPR
• Tamponade
• Intubate the patient and secure the
• Toxins
airway
• Thrombosis (coronary and
• Start post resuscitation care
pulmonary)
• Induced hypothermia
• Trauma
• Admit to ICU

If the patient remained pulseless

• Stop CPR after 20 minutes of


resuscitation of the non-shockable
rhythm or after 20 minutes from the last
shock that was delivered.

References and Further Reading, click


here.

127
Section 5

Chest Pain

Introduction
by Asaad S Shujaa Chest pain is one of the most common symptoms presented in the emergency
department (ED), and it is worrisome because the differential diagnosis widely
range between non-emergent conditions and life-threatening conditions such as
acute coronary syndromes (ACS), pulmonary embolism (PE), aortic dissection,
pericarditis with tamponade, pneumothorax, and esophageal rupture. Chest pain
caused by non–emergent conditions include esophageal reflux, peptic ulcer, biliary
colic, muscle strain, costochondritis, pleurisy, pneumonia and non-specific chest
wall pain.

It is important as emergency physicians to have an approach to chest pain that


enables one to recognize life-threatening conditions from non-emergent
conditions. This chapter aims to discuss how to approach a patient with chest
pain.

Currently, we do not have data regarding how many patients visit the ED with
chest pain in the Middle East; however, in the USA, approximately 6 million
patients visit ED with chest pain, which accounts for almost 9% of all ED cases.
This makes it the second most common complaint in ED visits.

Audio is available here

128
General Approach to Patient with Chest Pain shock. Also, unequal BP in both arm or pulse deficient indicates

in Emergency Department aortic dissection.

The general appearance of the patient

“As a general rule, any chest pain is ischemic in • Looks sick or not sick or

origin until proven otherwise.” • Patient in pain or not in pain

Initial Approach Electrocardiogram (ECG): To interpret myocardial ischemia,


Airway, Breathing and Circulation (ABC) assessment arrhythmias,  pericarditis, and right ventricular strain findings for
PE.
• Assessment of the airway by being able to talk without distress,
no obvious upper airway obstruction such as tongue swelling, Any abnormality found in the initial approach may need
lip swelling, hoarseness, etc. immediate actions.

• Assessment of breathing by listening to the pulmonary sounds. History


Is it equal or wet (basal crackles indicate CHF)? What types of questions would you like to ask?

• Assessment of circulation by listening to heart sounds. Are • Are you having discomfort, chest pain?
there any S3,4 gallop rhythm (CHF) or new murmurs such as
mitral regurgitation (papillary muscle dysfunction). • How would you describe it?

• Checking the pulses, capillary refill to understand the shock • Where is it?
situation.
• Does it radiate anywhere?
Vital signs should be assessed and repeated at regular intervals.
• Frequency?
For example, respiratory distress with low O2 saturation may
indicate pulmonary edema, plus low BP indicates cardiogenic • Time of onset or acute worsening?

• Has there been any progression?


129
• Any aggravating/alleviating factors? Table 3.9 History taking for chest pain

• Any associated symptom?

• Diaphoresis, nausea, vomiting, cough, fever

• History of cardiopulmonary disease?

• Risk factors  for coronary disease such as hypertension,


diabetes, high cholesterol, obesity, male, family history,
smoker, sedentary, post-menopausal, previous history of
ACS and family history of CAD.

• Risk factors for pulmonary embolism  such as travel history,


oral contraceptive use etc.  And risk factors for other critical
diagnoses.

Physical Examination
• Repeat assessment of the airway, breathing, and circulation
with full examination steps.

• Assess abdomen for tenderness and pulsating mass

• Look for swelling in legs (lower limb edema), calf tenderness


(deep vein thrombosis).

130
Bedside test • 1 2 l e a d E C G f o r myocardial infarction and 15 lead
ECG for posterior myocardial infarction
ECG
ECG is the main bedside test for any chest pain patient. • Any ST elevation in 2 contiguous leads should be evaluated
as S.T. Elevation M.I. However, please do remember, there
What is your opinion about below ECGs in patients with different are many other problems can elevate S.T. segment.
type of chest pain?
• Any other changes such as ST depression, T inversion and Q
Image 3.12 Case – 54 yo female presented with 3 days wave should be evaluated
history of right side chest pain (pleuritic).
• ECG is more useful as ‘rule in’ than ‘rule out.’

• In Acute Myocardial Infarction ECG has 50% sensitivity, 90%


specificity.

• 12 lead ECG for PE may show S1 Q3 T3 sign (prominent S


wave in the lead I, Q wave and inverted T wave in the lead III). It
is a sign of acute right ventricular strain (acute pressure and
Image 3.13 Case – 46 yo male presented with
central chest pain. He has nausea and diaphoresis. volume overload of the right ventricle because of pulmonary
hypertension). Other ECG findings noted during the acute
phase of a PE include new right bundle branch block (complete
or incomplete), rightward shift of the QRS axis, ST-segment
elevation in V1 and aVR, generalized low amplitude QRS
complexes, atrial premature contractions, sinus tachycardia,
atrial fibrillation/flutter, and T wave inversions in leads V1-V4.

• The ECG is often abnormal in PE, but findings are neither


sensitive nor specific for the diagnosis of PE. The greatest utility

131
of the ECG in a patient with suspected PE is ruling out other Others
life-threatening diagnoses such as acute myocardial infarction. Complete blood count, ESR, C reactive protein, blood
culture, and lactate may help to rule out some infections such as
• Some aortic dissection cases may also show ST-segment
pericarditis or mediastinitis because of esophageal rupture.  But,
elevation as in acute myocardial infarction.
their value in the acute setting is questionable.
• ECG may also help  to diagnose pericarditis, especially chest
pain patients with fever. Imaging modalities
Chest X-Ray
Laboratory tests • To look for heart failure and evaluate for other cause of chest
Cardiac markers pain such as Aortic Dissection, pneumothorax, pneumonia etc.
• Troponin I or T rise within 3-6  hours and then remain elevated
• Widened mediastinum, abnormal aortic knob, pleural effusions
for about one week
for aortic dissection. These findings are not sensitive for the
• Serial testing improves sensitivity aortic dissection. Only 25% of the patients have wide
mediastinum.
• In acute coronary syndrome suspicion, an increased Troponin is
a marker for increased risk of AMI and death • Esophageal rupture signs in chest X-ray; Hydropneumothorax,
Pneumothorax, Pneumomediastinum, Subcutaneous
• However, cardiac enzymes do not diagnose cardiac ischemia Emphysema, Mediastinal widening without emphysema,
Subdiaphragmatic air and Pleural Effusion.
D-dimer
• Only use is in a low-risk patient What is your opinion about below chest x-ray in a patient with
chest pain?
• A negative test makes PE very unlikely

• A slightly positive test is a positive test

132
Image 3.14 Case – 58 yo male presented with 1 day history CT scan
of sudden onset lef side chest pain radiating to left shoulder. • CT with contrast shows large, central emboli, it is also very
sensitive for aortic dissection.

• In the suspicion of esophageal rupture, contrast-enhanced CT


scan of the chest should be obtained if it is not possible to
obtain a contrast esophagogram, if the esophagogram was
negative, despite a high clinical suspicion, or if seeking to
evaluate for a more likely alternative diagnosis. Perforation may
be suggested by mediastinal air, extravasated luminal contrast,
peri-esophageal fluid collections, pleural effusions, or actual
communication of an air-filled esophagus with an adjacent
mediastinal air-fluid collection. Definitive esophageal
communication with outside structures is often difficult to
visualize.

• The pulmonary angiogram is the gold standard for PE and aortic


dissection but carries a risk of contrast-induced nephrotoxicity
and anaphylactic contrast reaction.

V/Q scan
It is very sensitive but not specific for patients with suspected PE.
Bedside ultrasound 
• RUSH protocol evaluates aorta and pericardial space to rule out Depending on your history, physical exam and bedside
tamponade (video) investigations as well as laboratory and imaging results, the focus
should be given to rule out myocardial ischemia or infarction,
• Consider Doppler ultrasound to see deep vein thrombosis in
pulmonary embolus, pneumothorax, pericarditis with tamponade,
legs (video)
aortic dissection, and esophageal rupture. Each of this specific

133
disease entities has various risk stratification methods, treatment
options, and dispositions. Now, it is time to look to some cases
and discuss more specific management in the ED. Case 1 – Critical Bedside Actions and General
Approach
Case 1 Place the patient in a monitored bed, make sure security
A 46-year-old male with a history of diabetes mellitus, chamber established (monitor, IVs, oxygen, etc.)
hypertension, and coronary artery disease presents to the ED. He
ABC intact
is a smoker. He complains of chest tightness and heaviness. The
symptom started gradually 3 hours ago and lasts 20 minutes Vitally stable except he has tachycardia (HR: 110)
when he was watching TV. The pain scale was 5/10, radiated to
Quick History and Physical Examination as described in the
his jaw. The pain is associated with nausea and sweating. He
text. Chest exam: Equal air entry, no wheeze or crackles
took Nitroglycerin spray, and the pain was relieved. The pain
started again before he reached the ED. The pain scale is 10/10. CVS exam: S1+S2 no additional sound, no murmur, JVP was
The initial assessment at triage: ABC intact, BP: 140/80, HR: 110 normal
RR: 24, O2Sat: 98% on room air, Temperature: 37.3, Random
Blood Sugar: normal. No lower limb edema, pulses for four limb present and equal

12 lead ECG shows inferior ST-elevation myocardial infarction


Image 3.15 ECG case 1
Consult cardiologist as soon as possible

Patient in pain needs analgesia

Aspirin 300 mg was given by EMS

134
Case 1 – Differential Diagnoses • The history does not suggest any past Case 1 – Emergency
There are six life-threatening differential esophageal rupture Treatment
diagnoses for any chest pain patients. • Aspirin should be given immediately
• Physical exam not lead to cardiogenic
These consist of:
shock or pulmonary edema • Great benefit, little risk
1. Myocardial ischemia or infarction (MI)
• No sign of pneumothorax in the exam • Give the minimum of 182 mg
2. Pulmonary embolus (PE)
• Pulses all equal for four limbs and no • Rapid decisions on reperfusion
3. Pneumothorax inequality in BP in both arms, which
• Based on ECG only (PCI vs.
does not go with aortic dissection
4. Pericarditis with tamponade Fibrinolysis)
• ECG suggested Inferior MI, no sign of
5. Aortic dissection • Antiplatelet options:
pericarditis in ECG
6. Esophageal rupture • Heparin (LMWH versus
Case 1 – Emergency Diagnostic
unfractionated)
Case 1 – History and Physical Tests and Interpretation
Examination Hints • ECG suggested Inferior MI, no sign of • Clopidogrel
• The chest pain is typical angina pain pericarditis in ECG
• Symptomatic / pain control
(heaviness radiating to jaw associated
• Portable CXR: normal which rules out
with nausea and sweating), the pain is • GTN    Vasodilator also reduces
pneumothorax and aortic dissection (no
not sharp such as in PE or tearing like preload
wide mediastinum)
in aortic dissection
• Troponin I is high, which suggests • Can give SL or IV
• The patient has cardiac risk factors
Myocardia Ischemia
(DM, HTN, CAD, Smoker, and MI 1 year • Morphine for pain control and reduce
ago) • In bedside echocardiography, there is anxiety and stress
hypokinetic in the inferior wall and no
• No PE risk factors • Secondary prevention
sign of cardiac tamponade

135
• B-Blocker, statins and ACE inhibitor Case 2 Case 2 – Critical Bedside
A 30-year-old male had an open Actions and General Approach
Case 1 – Disposition Decision reduction and internal fixation (ORIF) of • O2 Supply and monitor bed
Assess the risk stratification by using
right ankle fracture 2 weeks ago. C/O
TIMI score • ABC intact
sudden onset of chest pain today. He has

Case 1 – Admission criteria pleuritic sharp chest pain associated with • Vitally stable except he is tachycardia
• Establish risk level using the TIMI short breath, increased during inspiration. (HR 120)
scoring system
Initial assessment at triage • The quick history that suggested the
• Moderate risk: Admit for further • ABC intact patient had a major surgery 2 weeks
evaluation; add beta blockers, ACE ago and was immobilized 2 weeks.
• Vital signs
inhibitors. Follow cardiac enzyme
• Physical examination shows
levels. If MI ruled out, exercise stress • BP 120/80
test before discharge • Chest exam: Equal air entry, no
• Pulse 120
wheeze or crackles
• High Risk: Admit for cardiac
catheterization • RR 40
• CVS exam: S1+S2 no additional
• O2 sat 88% on room air sound, no murmur, JVP was normal
Case 1 – Discharge criteria
• Low-risk TIMI score: May be • T 36.5 • There is calf swelling in right site of
discharged after symptom control and surgery, pulses for 4 limbs present
follow up with cardiologist outpatient • 12 ECG shows sinus tachycardia, T and equal
for the stress test and lipid profile test inversion V2,3 and 4, deep S lead I and
Q and T inversion in the lead III, St • To do 12 lead ECG shows sinus
Case 1 – Referral elevation V1 and V4R suggested tachycardia, T inversion V2,3 and 4,
• Cardiology pulmonary embolism deep S lead I and Q and T inversion in
the lead III, St elevation V1 and V4R
suggested pulmonary embolism

136
• Patient in pain need analgesia • Physical exam not lead to pneumonia Case 2 – Emergency
no crackles in chest exam Treatment
Case 2 – Differential Diagnoses • Heparin (Will limit propagation but does
1. Pulmonary embolus (PE) • No sign of pneumothorax in the exam
not dissolve clot)
2. Myocardial ischemia or infarction (MI) • Pulses all equal for four limbs and no
• Unfractionated: 80 u/kg bolus, 18 h/
inequality in BP in both arms, which
3. Pneumothorax kg/hr.
does not go with aortic dissection
4. Pericarditis with Tamponade • Fractionated (Lovenox): 1 mg/kg SC
• ECG suggested PE, no sign of
BID
5. Aortic dissection pericarditis in ECG
• Fibrinolysis
6. Esophageal rupture Case 2 – Emergency Diagnostic
Tests and Interpretation • Consider with large if the patient is
Case 2 – History and Physical • ECG suggested Pulmonary embolism, unstable
Examination Hints no sign of pericarditis in ECG
• The chest Pain is atypical angina pain • No study has shown a survival
(sharp, pleuritic chest pain increased by • Portable CXR: normal which rules out benefit, but it is very difficult to study.
inspiration and associated with pneumothorax and aortic dissection (no
• Alteplase 50–100 mg infused over 2–
shortness of breath, no radiation), the wide mediastinum)
6 hrs (bolus in severe shock)
pain is not angina pain OR no tearing
• D- Dimer is high
pain as in aortic dissection Case 2 – Disposition
• Cardiac enzymes are negative If there is suspicious of PE, we need to
• There are PE risk factors (major surgery,
do pre-test probability; there are multiple
immobilization 2 weeks) • Bedside echocardiography there is
systems for doing this. Most widespread
signs of right ventricle enlargement and
• The history does not suggest any and validated is Well’s score
strain and no sign of cardiac
previous Esophageal rupture
tamponade There is a difference in Well’s score for PE
& DVT
137
PE – Well’s criteria Case 2 – Referral 200/100 on the left arm, tachycardia
• ICU (HR 110)
• 3 points for:
• Unstable Patient, massive PE, • Quick history which suggested sudden
• PE ‘most likely diagnosis onset central chest pain, described as
Bilateral PE
• Signs and symptoms suggesting ripping his chest and radiating to the
• Medical Ward
DVT back, no associated symptoms.

• Stable patient with Small PE


• 1.5 points for: • Physical examination shows:

• PR>100,
Case 3 • Chest exam: Equal air entry, no
A 60-year-old male patient presented to wheeze or crackles
• history (PE/DVT), the ED with sudden onset central chest
pain, described as ripping his chest and • CVS exam: S1+S2, a grade 2/6
• immobilization in 2 weeks s y s t o l i c m u r m u r, a n d a s o f t
radiating to the back, no associated
symptoms and patient, previous history decrescendo diastolic murmur are
• 1 point for:
with HTN, CAD, and smoker. Initial heard at the second right intercostal
• Hemoptysis or malignancy assessment by EMS was ABC intact. space. JVP was normal
Vitals were BP 190/95 Right arm, Pulse
Risk Stratification • There is radial to radial pulsation
110, RR 20 , T 37 , O2sat 98%.
delay
• <2 low risk (10%), D-Dimer is good to
rule out PE Case 3 – Critical Bedside • There are abdominal and bilateral
Actions and General Approach femoral bruits, with absent distal
• 2-6 medium risk (25%), Spiral CT chest • O2 Supply and monitor bed
pulses.
with contrast to rule out PE
• ABC intact
• 12 lead ECG shows no ST, T wave
• >6 high (50%), start changes, no sign of MI
• Vitally stable except he is high BP
anticoagulation(LWMH) and Spiral CT
185/85 mmHg on the right arm and
chest with contrast
138
• Portable CXR shows wide mediastinum, • Physical exam not lead to pneumonia, •Bedside Echo has no sign of
no sign of CHF, pneumothorax or no crackles in chest exam tamponade
pneumonia
• No sign of pneumothorax in the exam • CT scan is the most accurate and
• Patient in pain need analgesia fastest option
• Pulses delay in radio –radio pulsation
Case 3 – Differential Diagnoses and different BP in both arm and Case 3 – Emergency Treatment
1. Aortic dissection abdominal and bilateral femoral bruits, • Involve Cardio-Thoracic surgery as
with absent distal pulses with going soon as possible.
2. Myocardial ischemia or infarction (MI)
with aortic dissection
• Control the blood pressure
3. Pulmonary embolus (PE)
• ECG no sign of ischemic changes, no
• SBP goal is 120-130 mmHg
4. Pneumothorax sign of pericarditis in ECG

• Beta blockers are first-line agents


5. Pericarditis with Tamponade • Patient in Pain need analgesia
(Labetalol and Esmolol), they control
6. Esophageal rupture Case 3 – Emergency Diagnostic blood pressure and heart rate
Tests and Interpretation
Case 3 – History and Physical • Depending on the patient’s vitals you
• 12 lead ECG shows no ST, T wave
can add vasodilators such as
Examination Hints changes, no sign of MI
• The chest Pain is sudden onset central nitroprusside
ripping chest pain radiating to back as • Portable CXR shows wide mediastinum,
no sign of CHF, pneumothorax or Case 3 – Disposition
in aortic dissection; the pain is not
pneumonia • Patients should be admitted to ICU,
angina pain.
• Emergency surgery is needed for
• There are risk factors: HTN, CAD, • The cardiac enzyme was negative rule
ascending dissections
smoker, and age out MI

• If dissection is only descending,


• The history does not suggest any • D-Dimer was negative
management is only supportive.
previous esophageal rupture
139
Case 4 • Chest exam: decrease air entry in the Case 4 – History and Physical
A 55-year-old alcoholic with persistent left side, and there is subcutaneous Examination Hints
vomiting presents with sudden onset of emphysema in the left side of the • The chest pain is sudden onset
Chest Pain followed by hematemesis. chest followed by hematemesis. The chest
The chest pain is sudden onset, sharp in pain is sharp in nature radiating to the
• CVS exam: S1+S2. No additional
nature, radiating to the back. It is back; it is associated with shortness of
sound, JVP was normal, pulses equal
associated with shortness of breath for 3 breath for 3 hours. A history of repeated
in four limbs
hours. Past medical history: DM, HTN, vomiting and associated with short of
alcoholic, and smoker. Vitals: BP 120/80 • 12 lead ECG shows no ST, T wave breath and vomiting blood
equal bilateral arm, pulse 90 regular and changes, or ischemic changes (hematemesis).
equal on four limbs, no pulse deficit, RR
• Portable CXR shows left pleural • There is Risk factors, HTN, CAD,
40, T 38, O2sat 96% on room air.
effusion and pneumomediastinum and smoking, and alcohol use

Case 4 – Critical Bedside normal width of the mediastinum.


• There is strong history suggested of
Actions and General Approach Esophageal rupture
• O2 Supply and monitor bed Case 4  – Differential Diagnoses
1. Esophageal rupture
• Physical exam shows decreased air
• ABC intact
2. Aortic dissection entry in the left side, and there is
• Vitally stable except he is febrile (T 38) subcutaneous emphysema in the left
3. Myocardial ischemia or infarction (MI) side of the chest
• The quick history which suggested the
4. Pulmonary embolus (PE) • No sign of pneumothorax in the exam
sudden onset of Chest Pain followed by
hematemesis. The chest pain is sudden 5. Pneumothorax • ECG no sign of ischemic changes, no
onset, sharp in nature radiating to the
sign of pericarditis in ECG
back; it is associated with shortness of 6. Pericarditis with Tamponade
breath for 3 hours.

• Physical examination shows


140
Case 4 – Emergency Diagnostic initiated when the initial diagnosis is
Tests and Interpretation suspected.
• 12 lead ECG shows no ST, T wave
• Parenteral nutritional support
changes, no sign of MI
• Nasogastric suction – This should be
• Portable CXR shows left pleural
maintained until there is evidence to
effusion and right pneumomediastinum
indicate that the esophageal perforation
and normal width of the mediastinum.
has healed, is smaller or is unchanged
No sign of pneumothorax, no sign of
CHF, no sign of pneumonia • Narcotic analgesics

• Cardiac enzymes were negative, which • Admission to a medical or surgical


rule out MI intensive care unit (ICU)

• D-Dimer was negative • Outcome: survival 65-90%, poor


survival with delayed diagnosis >48hrs
• Bedside Echo: no sign of tamponade
References and Further Reading, click
Case 4 – Emergency Treatment
here.
and Disposition
• Nothing by mouth, NPO

• Broad-spectrum antibiotics – No
randomized clinical trials exist for
antibiotics and esophageal perforation;
h o w e v e r, e m p i r i c c o v e r a g e f o r
anaerobic and both gram-negative and
gram-positive aerobes should be

141
Section 6

A Child With Fever

Case Presentation
by Jabeen Fayyaz A 2-month-old female child was brought in with a history of
cough and fever for 2 days. As per mother, the fever was high
grade, documented as 38.5ºC with an inability to drink for the
last 4 hours. There was history of an episode of cyanosis at
home with coughing an hour ago. On examination, the child
was looking dull and lethargic. Her vital signs were: Temp
39ºC, HR 170/ min, RR 65/ min, SPO2 89% in room air, BP
75/50mm of Hg, and Capillary refill time 4 sec. Chest on
auscultation has bilateral crepitation. The child was taken
immediately in the resuscitation area and was put on high flow
oxygen. The blood work up and CX-ray ordered showed right
middle zone consolidation. IV antibiotic, Cefotaxime was
administered. The child was kept on IV fluids and cardiac
monitoring. The child was admitted to the high dependency
unit.

142
Overview c h i l d r e n r e s u l t s i n s i g n i fic a n t p a r e n t a l a n x i e t y.
Fever is one of the most common reasons for the Pediatric Management decisions about febrile children are further
Emergency Department (PED) visits. It accounts for almost 10% complicated by the fact that parents and physicians weigh the
to 25% of PED visits annually. Febrile illness in children is caused risks and costs differently.
mostly by viral infections, but a significant proportion, especially
In a study (Byington 2004), common sources of bacterial infection
in children who are less than 3 months old, are caused by serious
in children less than 90 days were found UTI, bacteremia, soft
bacterial infection (SBI). As an ED physician, the goal is to identify
tissue infection, meningitis, and pneumonia.
this population at risk and to promptly manage them.
Fever is defined as temperature ≥38°C measured rectally or
SBI has been reported to affect 6-10 % of infants who are
tympanic/axillary temperature of approximately 37.5°C. If parents
younger than 3 months and 5-7% of children who are between
state that fever is documented at home by a thermometer, it
3-36 months of age. Therefore, you should always be very careful
should be considered as fever recorded in the ED and should be
when evaluating a child with fever under 3 years old. The infant’s
evaluated in the same manner. Another important consideration
immune system is relatively immature during the first 2 to 3
mainly in neonates is hypothermia. Neonates may respond to SBI
months of life. This puts them in a very high risk group.
with hypothermia rather than hyperthermia, so they need to be
SBI can even be found in the presence of viral infection evaluated carefully for any other sign of toxicity.
concomitantly, 5% of patients with confirmed viral sources having
Temperature in children can be measured at the axilla, rectally,
urinary tract infections or other SBIs. Infants and children
orally or via the ear (tympanic). Younger children (<5 years old)
presenting with a fever and signs of a viral illness should have
cannot manage the glass thermometers because it can break
investigations to confirm the viral etiology, but should also be
easily. Therefore, this method is  not recommended for this age
assessed for other sources of bacterial infections. Details of this
group. To check the temperature in newborns and young children,
approach can be found in Policy Clinical Guideline.Children with
axillary measurement is an acceptable method. However, children
an apparent focus or are sick looking are easy to manage.
under 2 years of age may need confirmation with a rectal
However, it is very challenging and many gray zones in managing
temperature. Rectal temperature is considered the gold
the well-appearing infants and children with febrile illness without
standard. Bundling a young child may increase the skin
any source in the chaotic ED environment. Febrile illness in
temperature but not the core temperature. It should also be
143
considered in neonates and children less than 2 years of age Table 3.11 Recommended methods to measure temperature
where other methods are not reliable. by age

Table 3.10 Normal temperature ranges in children measured RECOMMENDED


by different method AGE OPTIONS
TECHNIQUE
MEASUREMENT
NORMAL TEMPERATURE RANGE
METHOD Rectal (for exact
Birth -2 years First Choice
temperature)
Rectal 36.6°C to 38°C (97.9°F to 100.4°F)
Axillary (to check for
Second Choice
Ear 35.8°C to 38°C (96.4°F to 100.4°F) fever)

Oral 35.5°C to 37.5°C (95.9°F to 99.5°F) Between 2 and 5


First Choice Rectal
years

Axillary 34.7°C to 37.3°C (94.5°F to 99.1°F)


Second Choice Tympanic
Temperature measurement in pediatrics - MicroLife USA, http://
www.microlifeusa.com/pdfs/therm/taking_an_infants_temp.pdf (accessed
June 27, 2016).
Third Choice Axillary

Studies have shown a good correlation between the tympanic


temperature and rectal temperature, especially in children more Older than 5 years First Choice Oral
than 2 years of age. On the contrary, axillary temperatures have a
lower correlation. If there is any doubt about a child’s
Second Choice Tympanic
temperature, rectal temperature measurement should be
considered for confirmation. Rectal temperature should be
avoided in neutropenic and immunocompromised children Third Choice Axillary

provided by author.

144
History and Physical Examination Hints Examination of skin is very important. So, skin color, cyanosis or
The detailed history and physical examination are the most vital in jaundice, rashes should be evaluated. Although the skin may give
the assessment of the febrile child. It is critical to pay attention to a clue about the degree of hydration, tears during crying,
the history provided by parents for documented fever at home as moisture on the oral mucosa/lips and tongue should be checked.
studies have shown it is moderately accurate; further evaluation For the neonate, “gentle” palpation of the anterior fontanelle
should always be carried out because a subjective fever at home indicates current the fluid status. If the fontanelle is sunken, this
may be the only indicator of a possibly serious bacterial infection shows hypovolemia/dehydration.
in a child who is afebrile in the ED.
An assessment of the child’s overall appearance is critical.
Focused history on fever characteristics should be asked, as it Although there is an imperfect correlation between physical
may provide useful clues. There is an increase in the rate of examination findings and serious bacterial illness, ill-appearing
pneumococcal bacteremia with a rise in temperature, especially children are more likely than well-appearing children to have
in young children. Studies suggest that the incidence of SBI is serious bacterial infection, and most well-appearing children do
higher in patients who have higher temperatures. The duration of not have serious bacterial infection.
the fever at the time of ED presentation does not help to predict
Toxic appearence includes lethargy with poor perfusion (delayed
occult bacteremia. The response to antipyretic medications does
capillary refill, cold hands and feet, pale or mottled skin) or
not predict bacterial or viral infection. Other important data to be
cyanosis or respiratory distress findings. Grunting is considered
considered include associated signs and symptoms, underlying
one of the most important signs to identify a sick child and may
medical conditions, exposure to ill contacts, and immunization
indicate an impending respiratory failure.
status.

In the exam, evaluate the quality of the cry? High pitched, or


weak in effort? Does the child appear fearful of the doctor, nurse?  
It is normal to see healthy young children’s fear of strangers.
Therefore, if you expose to a child who lies on the exam bed and
not interacting things around him or not showing his/her fear, then
you need to think about more serious illness on those.

145
The physical examination may reveal focal infection, and if so
Table 3.12 Assessment of Clinical Condition in Children on
Physical Examination the need for additional testing decreases. For example, the
children who have clinically obvious viral illness such as croup,
WELL UNWELL TOXIC chicken pox have lower rates of bacteremia than the children who
have no obvious infection source. Except for neonates and
Wakes only with
Strong cry or Drowsy / prolonged young infants, if a child has a nontoxic appearance, a more
not crying / decreased stimulation/ unable selective approach can be undertaken. When a child has an
Alertness/ smiles / stay activity / poor to arouse/ weak cry
Activity awake/ normal smile/response / high pitched cry/ identifiable cause, the treatment and disposition should generally
response to to social cues/ continuous cry/ be tailored to this specific infection.
social cues irritable bulging fontanelle /
grunting
NICE green light system.
Chest indrawing /
Normal work of RR more than age
Breathing
breathing
Nasal flaring
specific rates/ Emergency Treatment Options
grunting Airway, breathing, circulation (ABC) is the priority for all patients.
Normal lips, Supplemental oxygen or advance airway measure can be
Color / Pale per Pale / mottled /
skin, tongue necessary. Open intravenous access to draw blood samples, fluid
Circulation caregiver blue / ashen
color
infusion, and medications. Monitor the patients’ vitals. Early
Poor feeding in
Normal skin infants / dry treatment of fever is important. This gives the patient comfort as
Reduced skin
Fluid/ Urine and eyes/ mucus well as optimal physical examination condition for the physician.
turgor / bilious
output moist mucus membranes /
vomiting
membranes reduced urine
Acetaminophen  and  ibuprofen  both can be used.  They can
output
overlap during the treatment period to control fever. Some studies
Appears very
New lump >2 favor acetaminophen because of its fast effect. Other studies
Others unwell to health
cm
care professional found that combination of acetaminophen and ibuprofen is very
provided by author effective regime. Recommended doses are acetaminophen 15
mg/kg and ibuprofen 10 mg/kg.

146
Empiric antibiotic regimes day with the primary physician are factors •No underlying or chronic illness
should also be considered affecting admission decisions. However,
• No previous admissions
Age 0-28 days: ampicillin + admission is warranted for febrile infants
gentamicin  or a third-generation 28-56 days old regardless of the above • CSF – WBC < 8/hpf
cephalosporin factors.
• WBC – 500-15000/mm3, PMNL < 0.2
Age 29-56 days: Ill appearing children If the patient meets all of the following
can receive same regimen above. The low-risk criteria, they may be discharged • Urine WBC <10/hpf
children who can discharge home do home.
• No infiltration on chest x-ray
not need empiric antibiotics.
• Full-term birth
• Fecal leukocytes < 5/hpf
Ceftriaxone  is a preferred agent by
• Not hospitalized longer than the mother Red Flags to be explained to parents at
some clinicians before ED discharge.
• No toxic appearing the time of discharge. The parents should
Age 2-24 months: Empiric antibiotic
be instructed to follow-up after 24-48
therapy is not indicated for well- • Not received antibiotic within 48 h hours as per clinical condition in the
appearing children if there is no
primary health care system. A detailed
defined bacterial source and will be • No dehydration
account of danger signs should be
managed as outpatients.
• No lethargy explained to parents and if possible given
Finding venous access, waiting the lab a handout. It should be emphasized to
• No irritability
results and availability of the antibiotics them that if they notice or observe any of
are couple obstacles to apply timely • No wheezing the following, they should come back to
antibiotics to children with fever. the ED immediately as it indicates
• No infections in the ear, skin, soft worsening of the child’s condition:
tissue, skeletal
Disposition Decisions
• Have breathing difficulty
Toxic appearance, need for monitoring,
• No focal infection source
need for fluid treatment, poor social • The lips, tongue or nails appear blue
condition, follow up chance for the next • No hyperbilirubinemia
147
• Crying continuously and inconsolable

• Refuse to eat or drink or appear too


sick to eat or drink

• Vomiting whatever eating

• Has headache

• Has stiffness

• Develop skin rash

• Has severe abdominal pain

• Anything that worried parents from his


baseline

References and Further Reading, click


here

148
Section 7

Gastrointestinal Bleeding

Case Presentation
by Moira Carrol, Gurpreet Mudan, and Suzanne A 61-year-old man with a history of liver cirrhosis secondary to
Bentley
chronic EtOH abuse presents to the Emergency Department
(ED) with a complaint of vomiting bright red blood that began
prior to arrival. He arrives actively vomiting; a significant
amount of blood is noted in his emesis basin. He is now
complaining of dizziness and appears pale.

Audio is available here

149
Overview result of diverticulitis, the most common cause of LGIB, or
Gastrointestinal bleeding (GIB) can be generalized into two from hemorrhoids, colitis, anal fissures, inflammatory bowel
categories based on the site of bleeding. Upper GIB (UGIB) is disease including Crohn’s disease and ulcerative colitis, colon
defined as any bleeding that occurs proximal to the ligament of cancer or angiodysplasia.
Trietz near the terminal duodenum. Lower GIB (LGIB) is any
bleeding that occurs distal to the ligament extending to the Table 3.13 List of upper and lower G.I. bleeding causes
rectum. Most GIB seen in the ED is attributed to UGIB with an
incidence of 90 per 100,000 population. LGIB, on the other hand,
presents with a rate of 20 per 100,000 population. LGIB is more
commonly seen in the elderly but has a wide range of
presentations and causes. As a result, the approach to LGIB has
been less standardized.

Upper Gastrointestinal Bleeding (UGIB) Critical Bedsides Actions and Emergency


The most common causes of UGIB include peptic ulcer disease
Department Approach
(PUD), erosive gastritis or esophagitis, esophageal or gastric
Just as the causes of acute GIB are diverse, so too are the
variceal bleed and Mallory Weiss tears. Among these, PUD is the
possible presentations. As severe GIB can have a high risk of
most common cause of UGIB presentations. Other less common
morbidity and mortality, patients with possible GIB should be
causes include gastric malignancy, aortoenteric fistula,
identified quickly. In the ED, the patient should be evaluated for
hematobilia and Dieulafoy’s lesion, which is a large tortuous artery
hemodynamic stability, as patients with brisk GI bleeding can
that can run very close to the gastric mucosa and can cause
rapidly decompensate. Assessment of patient stability includes
devastating bleeding.
evaluating the patient’s general appearance, volume status, and
vital signs. If deemed hemodynamically unstable upon initial
Lower Gastrointestinal Bleeding (LGIB)
LGIB is less common than UGIB in the ED. In fact, UGIB is clinical evaluation, begin resuscitation of the patient immediately.

identified in 11% of cases, whereas a lower GI source is found Please check for the general approach in Figure 3.5 (F1).

only in 9%. Diagnosis is sometimes elusive. LGIB can be the

150
Figure 3.5 Approach to GIB Indications for transfusion include hemodynamic instability
despite crystalloid resuscitation, Hemoglobin (Hb) Hb < 9 g/dL in
high-risk patients, Hb < 7 in low-risk patients. High-risk patients
are considered those who are likely to rebleed or have severe
hemorrhage, whereas low-risk patients are less likely. Various
decision tools exist to help risk stratify patients based on multiple
clinical criteria and lab values. Consider FFP to correct
coagulopathy if present in a patient on anticoagulation or with
severe liver dysfunction. Placing the patient on a cardiac monitor
continually to assess changes in heart rate, blood pressure, and
oxygen saturation is imperative.

History and Physical Examination Hints


After initial stabilization, the next step is to determine the
probable cause of the bleeding in order to treat appropriately and
disposition the patient. The history and physical can guide you
towards identifying the likely source of the bleeding and direct the
necessary treatments and consultations. UGIB can often be
definitively managed by gastroenterologists. Lower GI bleeding,
however, might require interventions by general surgeons,
gastroenterologists or interventional radiologists.
First, do a rapid assessment and intervention of airway, breathing,
Certain complaints are unique to GIB. One of the most important
and circulation (ABCs). Then, place two large bore IVs in
pieces of history is to assess complaints of active bleeding.
preparation for IV crystalloid fluids infusion and possible blood
Hematemesis is virtually diagnostic of UGIB. Hematemesis is
product transfusion. Draw initial labs including a complete blood
defined as bloody vomit, either appearing as bright red blood or
count, type and screen and type and crossmatch in case the
as coffee ground emesis. Melena, or dark, tarry stool, is also a
patient will require blood product transfusion.

151
sign of UGIB. In patients with UGIB, Similarly, a patient who complains of expected blood transfusion. Initially,
between 90-98% presented with either blood in the stool with a history of Hb and Hct may be within normal limits.
melena or hematemesis. Alternatively, constipation suggests bleeding caused The values might not immediately reflect
hematochezia is defined as blood within by the diverticular disease. Recent blood loss after an acute hemorrhage
o r a r o u n d t h e s t o o l . H o w e v e r, diarrheal illness can be found in infectious and, therefore, should be repeated.
hematochezia can sometimes be the colitis. An elderly patient presenting with Higher mortality and incidences of
result of a brisk UGIB. Diagnosis can also weight loss or anorexia is concerning for rebleeding were found in patients with Hb
be confounded if there is slow peristalsis malignancy. The duration and timing of < 10 g/dL. Additionally, many recommend
in the setting of an LGIB. the bleeding are important to determine. using the Hb and Hct to inform the
Brisk or continued bleeding can alert to decision to type and crossmatch blood
Therefore, start by assessing the context
the need for resuscitation or emergency versus only drawing a type and screen.
of the bleed as it can give you clues to its
intervention. Finally, the provider must
origin. For example, patients who have a In a patient without kidney disease, a
characterize and quantify the bleeding. In
bleed secondary to PUD might have a BUN to Creatinine ratio that is elevated to
a complaint with multiple pathologic
history of an ulcer, might complain of acid greater than or equal to 36 is strongly
causes, a good history and physical
reflux or have a recent history of frequent associated with UGIB. As blood is
exam are paramount.
NSAID use. Patients with gastric or digested, the BUN is reabsorbed into the
esophageal varices might describe a Emergency Diagnostic Tests circulation leading to elevated serum
history of or risk factors for liver disease, and Interpretation levels. Below 36, however, the ratio has
such as daily alcohol abuse, or have no positive or negative predictive value.
other pathognomonic signs of portal Laboratory Studies BUN/Cr >36 can be helpful in the
hypertension. A history of intractable The most important lab tests for risk diagnosis of an occult UGIB in those
vomiting in the setting of hematemesis stratification for patients with acute GIB patients who present without classic
may suggest Mallory Weiss tears as the are the hemoglobin (Hb) and hematocrit signs of GI bleeding.
cause. (Hct), coagulation studies, and BUN to
Creatinine ratio. A type&screen is The role of nasogastric (NG) lavage and
recommended as well in case of aspiration in the diagnosis of GI bleeding

152
has been controversial. NG aspiration suspected UGIB can undergo endoscopy Tagged Red Blood Cell Scan
positive for blood is highly predictive of a as an inpatient. Early endoscopy, within Tagged Red Blood Cell Scan is a second
UGIB. However, it has not proven to be the first 24 hours of presentation, is line study that can assist in the diagnosis
sensitive. Placement of an NG is not a associated with shorter hospital stays of more indolent and continued bleeding.
benign procedure as there are risks and early instigation of appropriate Scanning within the recommended two-
including perforation and discomfort. treatment. However, most UGIB resolves hour window after the injection has high
without this intervention.  Figure shows a rates of positive diagnosis in 95-100% of
Fecal occult blood test duodenal  ulcer (Deep demarcated cases but after the recommended time
Performing a fecal occult blood test via a
ulceration with a visible vessel on base period the test is significantly less
rectal exam is important in the setting of
(Forrest Iia) Source: Lai, WEO Endoscopy effective.
a GIB. This bedside test can confirm
Atlas, Date: 2012-12-25.
whether or not blood is present in the Medications
stool, confirming the presence of a GIB. Colonoscopy Only a few medications have been shown
Unfortunately, it is not specific to UGIB or Colonoscopy can be helpful in the to be influential in the acute management
LGIB; however, often the presence of diagnosis of LGIB and is an effective first- of GIB. Pantoprazole is indicated for a
melena or bright red blood can help guide line test, but it is not a gold standard in UGIB in the setting of PUD. It is given as
diagnosis. the diagnosis of LGIB. A diagnosis is an 80 mg bolus followed by an infusion at
made by colonoscopy in 75% of cases. a rate of 8 mg/hour. If variceal bleeding is
Upper Endoscopy Typically, lower GI scopes are not known or suspected, consider starting
Upper endoscopy is overwhelmingly
performed emergently but can be Octreotide or other somatostatin analog.
diagnostic and usually therapeutic for
performed later during hospitalization or Octreotide is given as a 25-50 mcg bolus,
UGIB. Consultation with gastroenterology
a s a n o u t p a t i e n t .  F i g u r e s h o w s then 25-30 mcg/hr infusion. In patients
is necessary for the emergent scope of
colonoscopy, bleeding from multiple with cirrhosis, antibiotics such as
patients with continued bleeding and
diverticular outpouchings. Source: Ceftriaxone, Amoxicillin-clavulanate or
suspected UGIB. These specialists can
American Family Physicians, Wilkins et al. Quinolone should be given.
immediately diagnose and treat the
Diverticular bleeding (please see Figure in
source of bleeding. Stable patients with
their manuscript).

153
Procedures behaviors that may increase the risk of
Sengstaken-Blakemore Tube is a device bleeding again, such as NSAIDs and
that is inflated in the esophagus to alcohol.
tamponade uncontrolled bleeding caused
References and Further Reading, click
by varices. It is used as a measure of last
here
resort because of the high complication
rate. EM CRIT – VIDEO – Blakemore Tube
Placement for Massive Upper GI
Hemorrhage.

Disposition of Patient with


Gastrointestinal Bleeding
Finally, the patient with GI bleeding will
need to have a disposition based on the
resuscitation and findings of the workup.
Unstable patients or those with active
GIB and rapidly decreasing Hb and Hct
levels on reassessment should receive a
consult from the intensive care unit. If a
patient and Hb/Hct remain stable,
admission to a regular medical floor or
possible discharge home with close,
appropriate follow-up may be considered.
Appropriate follow-up should be timely
with a gastroenterologist. Prior to
discharge, patients should be
encouraged to avoid medications and
154
Section 8

Headache

Case Presentation
by Matevz Privsek and Gregor Prosen A 52-year old male comes to the ED with a severe headache.
A triage nurse gives you his chart and says that his vital signs
are normal, but he does not look well. You start to question
the patient, and the following history is obtained: his
headache started approximately six hours ago. He was
working in his office when he started to feel squeezing-like
sensation in his head. The pain has gotten worse since then,
but it is still tolerable. It is independent of any physical activity
or position. He already had a few similar episodes of this kind
of headache in the past two years, but now the pain does not
go away after aspirin as it did previously. He denies trauma as
well as any associated symptoms, e.g. no visual disturbances,
hearing loss, weakness, dizziness, stiff neck, loss of
consciousness. He is otherwise a healthy, non-smoker, with
Audio is available here no regular therapy or known allergies. His clinical exam is

155
unremarkable. Conscious, GCS back in 2 hours and are Introduction
Headache is a subjective feeling of pain,
15, alert and oriented, normal completely normal. The patient
crushing, squeezing or stabbing
skin color. Blood pressure now feels much better, with anywhere in the head. They are typically
135/82 mm Hg, pulse 78/min, almost no headache at all. divided to primary and secondary

14 breaths/min, SpO2 99%, Repeated vital signs and headaches. The most important task
emergency physicians have is to exclude
body temperature 36,4 °C. clinical exam are again
any potentially lethal causes of headache.
Neurologic exam shows no unremarkable. You explain to
According to some data, around 85% of
declines from normal, as well the patient that most likely he
the adult population experience
as the rest of the physical had a tension headache, warn headaches at least occasionally, and
exam. him about red flags regarding 15% does so regularly. A headache is the
chief complaint in around 3-5 % of all
headaches, and discharge him
You set up an intravenous emergency departments (ED) visits.
home with a prescription for
cannula, draw blood for
peroral analgesics with a Pathophysiology
testing, and gave the patient The pain in the head originates either
follow-up at his general from the meninges, blood vessels or
some parenteral analgesics
physician. surrounding tissues; the brain
(metamizole 2.5 g, ketoprofen
parenchyma itself is insensitive to pain.
100 mg) along with 500 ml of Because most of the pain is mediated
normal saline. You put him into through the fifth cranial nerve, the
patient’s ability to localize the pain is
the observation room. Lab
often poor. More specific localization of
results (complete blood count, the pain is associated with specific
basic biochemistry panel) came
156
inflammation in a specific structure (e.g. sinusitis). unknown. It is estimated that less than 1% of patients with
headache have a serious, life-threatening underlying disease.
Etiology
A vast number of diseases can cause a headache. We divide Management
them into primary headaches, in which the headache is the
disorder in itself, and secondary headaches, in which headaches
Critical Bedside Actions And General Approach
Regardless of the patients’ chief complaint, an emergency
are caused by various exogenous disorders (Table 3.14).
physician’s first task is assessing a patient’s condition and vital
Table 3.14 Etiology of headaches signs, and stabilize him/her, if necessary. After the patient is
stabilized, we continue with establishing the chief complaint,
precise history, and physical exam, setting the working diagnosis
and list of differential diagnoses, and diagnostic and treatment
plan.

Differential Diagnosis
An emergency physician has to exclude life-threatening causes of
headache, based on history, physical exam, and diagnostic tests.
Subarachnoid hemorrhage (SAH), meningitis, encephalitis, carbon
monoxide poisoning, and temporal arteritis considered as critical
diagnoses. Other emergency causes of headache are shunt
failure, dub or epidural hemorrhage, tumor/mass lesions,
mountain sickness, glaucoma, sinusitis, brain abscess, anoxic
hedache, anemia, and hypertensive crisis.

Around 50% of patients with a headache in the ED have a


tension-type headache, 10% have a migraine-type headache, 8%
have a secondary headache, and 30% of headaches remain

157
History And Physical Examination Hints • patient’s activity at the onset of the pain (headaches
The leading symptom has to be thoroughly “dissected” and a associated with exertion are suggestive of vascular bleeding),
focused medical history must be obtained. Useful mnemonics for
• history or possibility of head trauma suggests possible epidural
this are “SOCRATES” and “SAMPLE”  (Table 3.15).
or subdural hematoma, traumatic SAH, skull fracture, and
closed-head injury (e.g. diffuse axonal injury),
Table 3.15 Getting fast and effective history about chief
complaint
• in immunocompromised or HIV-infected patients one must
consider for brain abscess, toxoplasmosis, or cryptococcal
meningitis,

• the severity of headache is not useful to accurately distinguish


differential diagnoses; it is more helpful in monitoring the
patients’ response to treatment. However, relieving headache
with pain medications will not eliminate severe, secondary
causes.

• the character of the pain alone is not enough to adequately


differentiate one type of headache from another,

• location of headache is only helpful when the patient identifies a


When taking the history of presenting illness, one must be specific, well-localized area,
especially focused on:
• nausea and vomiting are completely nonspecific and points
• marked variation in headache pattern (if already had previous towards the intensity of the complaint.
episodes of headache),
Tension headache is the most common. Usually, it is bilateral,
• sudden onset of “thunder-clap” or “lightning strike” headache non-pulsating, and mild to moderate severity, not related to
(possible SAH),

158
physical activity. Pain is blunt, squeezing-like. It is most common
Table 3.16 Emergency diagnostic tests for headache
in the afternoon or evening.

Migraine starts gradually and can last up to three days. Pain is


unilateral, pulsating, and severe, often worse with physical
activity. Vomiting, as well as photo and noise sensitivity, often
accompany it. Aura (transitional vision disturbances,
paresthesias, speech disturbances) can be present before an
attack, but it is rare. It is more common in females.

Cluster headache is extremely rare (overall prevalence around


0,4%). Its characteristics are extremely severe, unilateral pain,
limited to orbital, supraorbital or temporal regions. It is of short
duration (few minutes up to two hours), during which the patient
cannot be still.

Emergency Diagnostic Tests And Interpretation Image 3.16 SAH


The majority of patients with headache do not require any
additional testing. The emergency physician has the following
options: blood tests, head CT, lumbar puncture and cerebrospinal
fluid analysis (Table 3.16).

It is important to know that head CT scan misses 6-8 % of


patients with SAH, its’ sensitivity for detecting SAH is reduced by
10% if the symptoms began over 12 hours ago, and almost by
20-30% if they began 3-5 days ago.

159
On the other hand, head CT scan within 6 hours of onset of the Table 3.18 Initial treatment options for primary headaches
pain has almost 100% sensitivity for SAH; therefore, lumbar
puncture is reasonable only when head CT does not confirm the
diagnosis, yet the clinical picture is highly suspicious for SAH.

Emergency Treatment Options


Pain relief is the first and most important important thing of
management, at least initially in an undifferentiated headache.
Depending on the severity and associated symptoms (e.g.
vomiting) we can choose between paracetamol and non-steroid
anti-inflammatory drugs, metamizole, and opiates. Analgesics can
Treatment for secondary headaches is directed towards the
be given per oral or parenterally (Table 3.17). For specific
cause (e.g. surgery, antibiotics, antihypertensives), but we must
headache treatment options, see Table 3.18.
not forget about pain relief!

Table 3.17 Analgesics for headache


Pediatric, Geriatric, Pregnant patient, and
Other Considerations
All the principles for adults can be safely applied to pediatric,
geriatric or pregnant patients with headache, with regards to
differential diagnosis (e.g. geriatric patient has increased
likelihood for a vascular headache). Venous sinus thrombosis
should be kept in mind for pregnant and peripartum patients.
Sinusitis is one of the common cause of childhood headaches.

Disposition Decisions
The vast majority of patients with headache can be discharged
home with a prescription for analgesics and a close follow-up.
These patients are those in whom ED therapy was successful in
160
pain relief, have normal clinical exam and vital signs, and no
serious illness has been identified or suspected. All other patients
require additional work-up or admission.

References and Further Reading, click here

161
Section 9

Multiple Trauma

Case Presentation
by Pia Jerot and Gregor Prosen A 28-year old male was a restrained driver in a head-on motor
vehicle collision. He was entrapped and extricated from the
vehicle. Transient loss of consciousness was reported. He
complains of severe chest pain, abdominal pain, and right
upper leg pain.

Audio is available here

162
Critical Bedside Actions and General D: disability
Approach E: exposure
Multiple trauma patients are primarily stabilized by the ambulance
crew on the field according to Prehospital Trauma Life Support In case of massive external hemorrhage, “C” takes advantage
(PHTLS) or International Trauma Life Support (ITLS) algorithm. over A and B.  When the bleeding is controlled, we can continue
This chapter is about the approach to multiple trauma patients in with A and B.
the resuscitation room and mainly focuses primary survey.
A – Airway and cervical spine control
Preparing For Patient Arrival When the patient arrives at the trauma center, talk to the patient
The ambulance service has to provide information to the trauma and quickly asses his consciousness and airway. If the patient is
center for the arrival of the seriously injured patient. The trauma conscious and talking with a normal voice, his airway is adequate
team puts on protective clothing (rubber latex gloves, plastic for a period. If it is obstructed, the airway has to be secured.
aprons, eye protection, etc.). A team leader should brief the team While securing the airway, C-spine has to be protected, especially
and make sure that every member knows his role and all when we are dealing with a patient with neck pain, focal
necessary equipment is ready. neurological signs, coma, suspected head injury or a history of
high-speed impact.
Primary Survey
In the first few minutes, a primary survey has to be done. The Do not delay C-spine and vertebra stabilization in a trauma
primary survey is a structured assessment in which we identify patient with proper size of c-collar, side pads, and trauma board..
and immediately treat conditions that are life-threatening. The
The airway can be temporarily opened with a jaw thrust or basic
primary survey should always be the same, following the ABCDE
adjuncts such as nasopharyngeal or oropharyngeal airway. If the
algorithm listed below.
patient’s airway is still compromised, it should be secured by
A: airway and cervical spine control endotracheal intubation or surgical airway. Indications for
endotracheal intubation are listed in Table 3.19. Intubated
B: breathing patients should be monitored by continuous capnography.

C: circulation

163
Table 3.19 Indications for ET intubation C – Circulation and hemorrhage control
The main objective of this step is to identifying shock situation in
the patients. If there is massive external hemorrhage, it should be
controlled before we proceed with A and B assessment. Radial
pulses, heart rate and blood pressure (BP) should be assessed at
that stage. The patient should be attached to the monitor. If we
are dealing with a major trauma patient, at least two large bore
(14G or 16 G) IVs have to be established. As an alternative,
intraosseous line can be used.
B – Breathing
“Permissive hypotension” with systolic BP 80-90 mmHg should
All trauma patients should be given 15 L O2 via non-rebreather
be maintained when bleeding is not controlled (internal bleeding).
mask. The respiratory rate has to be evaluated. Expose the chest,
If the patient is hemodynamically unstable, fluid resuscitation
and inspect for any deformities, wounds, bruising, asymmetrical
should begin with 1 L of warm isotonic fluid, either normal saline
movement or flail chest. The chest has to be palpated for any
or lactated Ringer’s. At this moment, it is better to keep in your
crepitus or subcutaneous emphysema. The lungs should be
mind that normal vitals are considered stage I shock which
auscultated to assess the presence of breathing sounds
patients could lose up to 750 cc blood. So, starting fluid support
bilaterally.
to patients suspected multiple trauma is a standard approach
Chest injuries that can impair breathing have to be identified and regardless of their vital signs. If the patient is still unstable after 1
treated. Life-threatening thoracic conditions are  tension Liter of fluid or is having ongoing blood loss, we should
pneumothorax, massive haemathorax, cardiac tamponade, open administer a transfusion of O-negative blood and order type-
chest wound, flail chest (video). If any of those conditions are specific blood. The patients who have an identified bleeding
found, immediate action is needed. source will usually require surgical intervention. Do not delay
surgery departments’ involvement. Internal bleeding in the thorax,
We can also use ultrasound at this moment to identify
abdomen, pelvis or around fractures of long bones (particularly
pneumothorax, haemathorax, and cardiac tamponade.
femur) should also be evaluated. For identifying bleeding into
abdominal, pleural, or pericardial cavities, we can use the bedside

164
ultrasound (e-FAST exam).  In the hemodynamically unstable If there is time, Glasgow Coma Scale (GCS) can be used to
patient with normal e-FAST, one should think pelvic injuries. assess the level of consciousness. Otherwise, GCS is assessed in
the secondary survey. GCS is listed in Table 3.20.
Image 3.17 Positive eFAST exam (peri-splenic free fluid)
Table 3.20 Glasgow Coma Scale

And finally, patients should be evaluated for any lateralized motor


deficits which motor part of the GCS may help for this. An
important clue, the patients can not move their extremities
because of focal injuries.

In this stage, blood glucose and body temperature should also be


checked.

E – Exposure
D – Disability All clothes have to be removed from the patient so that hidden
In the primary survey pupil size, symmetry and reactivity should
injuries and bleeding can be identified. The patient has to be log-
be assessed. To detect the level of consciousness, the patient
rolled. In a log-roll, back of the head, neck, posterior chest, lower
has to be assessed by AVPU scale. It stands for A: Awake, V:
back should be inspected, palpated and auscultated as
responds to voice or verbal commands, P: responds to painful
appropriate. If necessary, the rectal examination can be applied at
stimuli, U: unresponsive.
this moment. We have to avoid hypothermia during this stage.

165
Secondary Survey • C:  Shock because of cardiac tamponade, intra-abdominal,
The secondary survey can be done after the primary survey and intrathoracic, intrapelvic hemorrhage, external hemorrhage
when the patient is responding to resuscitation. It consists of
• D: Head injury
taking history (see “SAMPLE” mnemonic below), head to toe
assessment (including log-roll), interpreting results of • E: Fractures, dislocations and small wounds
investigations, formulating a management plan for the patient and
documenting all findings. History and Physical Examination Hints
History and physical examination hints to help “rule in or rule out”
S: symptoms – social differential diagnoses are listed in Tables 3.21. 

A: allergies
Table 3.21 Lorem Ipsum dolor amet, consectetur
M: medications

P: past medical history

L: last meal

E: environment and events

Differential Diagnoses
There are critical conditions which should have been diagnosed
during the primary survey. These are;

• A: Airway obstruction caused by c-spine injury, foreign body,


etc.

• B:  Tension pneumothorax, flail chest, massive haematothorax,


chest wound As an example, any patients with head, neck, facial injury or
having abnormal breath sounds should be suspected of an

166
obstructed airway. Any patients with severe shortness of breath, A trauma patient should be radiographically evaluated with chest
decreased or absent breath sounds on the lung, and hypotension and pelvic radiographs. C-spine X-ray was traditionally used.
is a tension pneumothorax until proven otherwise. Nowadays, any low-risk criteria violation is considered for
computerised tomographic evaluation for the cervical spine.
E m e r g e n c y D i a g n o s t i c Te s t s a n d Cervical spine imaging can be delayed if there are no neurologic
Interpretation findings or persistent moderate hypotension. For assessing C-
spine and other injuries, CT scan is the best choice in the acute
Laboratory tests
setting, when the patient is stable.
Laboratory evaluation of the trauma patient is used for assessing
the adequacy of resuscitation, for determining the proper Imaging studies of the thoracolumbar spine and extremities can
transfusion products and the onset of coagulopathy and for be delayed until higher priority assessments and interventions are
baseline values for follow-up studies. complete.

When IV access has been established, 20 mL of blood should be


Emergency Treatment Options
taken for a full blood count, urea, electrolytes and for blood group
type or full cross-match. An arterial blood should also be taken Initial stabilization
for blood gas and pH analysis, but it can be taken at the end of Initial stabilization should be done during primary survey. The
the primary survey.  If there is a very limited amount of blood algorithm is shown in Diagram 3.2. The goal is every abnormality
taken, blood group type and cross-match should be the only test detected during the primary survey should be fixed.
for the patient.

Imaging
The e-FAST (Extended Focused Assessment with Sonography for
Trauma) should ideally be a part of the primary survey, especially
for unstable patients. It can be used for evaluating pneumothorax,
free fluid in thorax and abdomen and to identify cardiac
tamponade.

167
Diagram 3.2 Initial stabilization Medications
Drugs needed for RSI
For RSI, you will need a pretreatment agent (mainly fentanyl),
induction agent (mainly ketamine or etomidate) and paralytic
agent (mainly rocuronium). Dosages and characteristics of drugs
are listed in Table 3.22. Doses must be adjusted in the
hypotensive or shocked patient.

Analgesia
For analgesia, opioids or ketamine can be used. Dosages and
characteristics of drugs are listed in Table 3.22.

Table 3.22 Drugs for RSI and analgesia

168
Procedures the neck and the floppy upper airway which can result in
All procedures for initial stabilization should be done in the occlusion of the airway. To prevent this, place a pad under the
primary survey. torso of children younger than eight years.

Airway: If the patient’s airway is compromised after a jaw thrust, Breathing: Normal respiratory rates vary in different ages.
nasopharyngeal or oropharyngeal tube placement, then Newborn respirates 25-50 breath per minute while 6 months to 12
orotracheal intubation is indicated. Rapid sequence intubation year kids respirate 15-30 breath per minute.
should be done. If it is unsuccessful, the  surgical airway should
Circulation: Brachial and femoral pulses are usually easy to feel. A
be established.
weak, rapid pulse with a rate over 130 is a sign of shock in
Breathing: If tension pneumothorax is found in the primary survey, children all ages except neonates. Children have a strong
it should be decompressed immediately. Tube thoracostomy compensatory mechanism in early shock and later deteriorate
should be followed this procedure. If massive haemathorax is very quickly. When giving fluid resuscitation, give 20 mL/kg in
found, thoracostomy should be performed.  However, in this each bolus.
situation, please make sure that the patient was supplied with
Disability: When evaluating an injured child, the care provider
enough volume and blood. Informing trauma surgery or thoracic
should remember that children of various ages have different
surgery for potential thoracotomy risk is a wise approach.
cognitive skills and interact differently. For assessing child`s
Circulation: If massive bleeding is found, it should be controlled neurological status, special GCS is used.
immediately. If cardiac tamponade is found, it should be treated
with pericardiocentesis.
Geriatric Patient Considerations
The geriatric patient should be assessed and treated by ATLS
protocol for adults. Because of multiple comorbidities in this age
Pediatric, Geriatric, and Pregnant Patient
group, the patients may need special considerations such as
Considerations
medications which affect vital signs and basal laboratory results
Pediatric Patient Considerations which affect the decision on imaging modalities (high kidney
Airway: Airway should be checked the same as in an adult functions). However, life-threatening situations have priority, and
patient. When positioned flat on a stretcher, the occiput can flex immediate actions should be taken as with normal adults.

169
Pregnant Patient Considerations • Negative e-FAST and CT scans for free fluid in body cavities.
Primary survey is the same for the pregnant patient as for other
• Minimal head injury with GCS 15 and normal CT scans without
patients.
other body injuries and with normal neurologic status (with
The normal heart rate of a pregnant patient is 10-15 beats faster instructions to return for any changes in mental status,
than usual, and the blood pressure is 10-15 mmHg lower; so, vomiting, or worsening headaches).
normal vital signs can be mistaken for a shock. A blood loss of
• Uncomplicated rib or sternal fractures.
30-35% can occur before there is a significant fall in blood
pressure. Referral
Multiple trauma patients who do not need admission for opetaion
Disposition Decisions or observation are usually referred to a surgeon for the follow-up
Admission criteria or management of their injuries.
The majority of patients will be admitted to the hospital following
References and Further Reading, click here
major trauma for the management of their injuries. Some of them
need operation while other just need observation.

Admission criteria for ICU are cardiovascular resuscitation, airway


protection and mechanical ventilation, invasive monitoring, severe
head injury, organ support and correct coagulopathy.

Discharge criteria
The discharge decisions of trauma patients differ between
institutions and systems. However, below list are cover the
general agreement about the patients who can discharge after
trauma.

• Minor blunt trauma, hemodynamically stable on serial


assessments.

170
Section 10

Poisonings

Case
by Harajeshwar Kohli and Ziad Kazzi An 18-year-old, previously healthy female, presents to the
Emergency Department with nausea, vomiting, and tremors.
She states 45 minutes ago she ingested an unknown number
of diphenhydramine tablets (25 mg) in a suicidal gesture. Past
Medical History: Depression, Medications: none. Social
History: As per family member, she does not smoke or use
illicit drugs. She is single and unemployed. Vital Signs: HR 110
bpm, BP 151/92 mmHg, RR 20 / min, Temp 38.5 degrees
Celsius. Physical Exam: General Appearance: Mild distress,
awake, appears to be hallucinating. Eyes: Dilated pupils
bilaterally but reactive. Cardiovascular: Tachycardic, normal
sounds, and no murmurs. Lungs: Clear to auscultation
bilaterally. Abdomen: Soft, non-tender, non-distended,
decreased bowel sounds. Neurologic: Normal motor power,
Audio is available here normal cranial nerves, normal cerebellar exam, alert and

171
oriented to self. Not oriented to Initial Approach a defective space heater or electrical
The initial approach to any patient generator can cause altered mental
location or date. Attention level
presenting to the emergency department status. Remember to always check a
waxes and wanes. Skin: warm, begins with airway, breathing, and glucose level in an altered patient!
dry, no rash. Musculoskeletal: circulation (the ABC’s). The physician can
proceed to a more thorough history and Common Toxidromes
No deformities, no clonus, A toxidrome is a constellation of signs,
physical examination after the ABC’s are
normal deep tendon reflexes. symptoms and vital signs findings that
secured. Oftentimes, patients who
clinically correlate with exposure to a
present after an overdose have altered
toxin or class of toxins. The following list
mental status or try to conceal their
includes common toxidromes:
ingestion. This highlights the need for the
physician to gather collateral history from
Sympathomimetic (cocaine,
Emergency Medical Services (EMS)
amphetamines, phencyclidine)
providers, bystanders, family, and friends. Hypertension
The physician should try to find out if the
patient has any psychiatric history or Tachycardia
access to medications. The physician
Diaphoresis
should check the patient’s clothing for
empty bottles or paraphernalia of drugs Mydriasis
of abuse. The physician should also
Agitation
inquire about the physical environment
where the patient was initially found. Anticholinergic (tricyclic
Various environmental toxins can lead to antidepressants,
altered mental status and should be diphenhydramine,
considered in the initial assessment. For antihistamines, jimson weed,
example, carbon monoxide released from atropine)

172
Tachycardia A- Abdominal cramps Sedative (benzodiazepines,
gamma-hydroxybutyric acid)
Hyperthermia M- Miosis
Typically normal vital signs
Dry skin Cholinergic (organophosphates, Depressed mental status
carbamates, nerve agents) –
Mydriasis
MTWThF (mnemonic for Bradypnea
Diminished bowel sounds nicotinic effects – days of the
week) Vital Signs
Urinary retention M- Mydriasis Vital signs can help guide the physician’s
differential diagnosis. The following table
Delirium, agitation T- Tachycardia lists some toxins and their effect on vital
signs:
Cholinergic (organophosphates, W- Weakness
carbamates, nerve agents) –
TH- Hyperthermia
SLUDGEBAM (mnemonic for
muscarinic effects) F- Fasciculations
S- Salivation, seizure
Opioid (opiates, opioids,
L- Lacrimation
clonidine)
U- Urination Miosis

D- Diarrhea Hypotension

G- GI distress (diarrhea and vomiting) Bradypnea

E- Emesis Bradycardia

B- Bronchorrhea Hypothermia

Depressed mental status


173
Table 3.23 Vital sign abnormalities and related toxins

Diagnostic Evaluation
The initial diagnostic workup for an overdose patient should be
guided by clinical presentation and can be broad. Please note
that this is an introductory chapter and the following is a basic
initial approach and not meant to be exhaustive by any measure.
As an initial suggestion, the following should be ordered initially:

• Complete Metabolic Panel (to assess electrolytes, anion gap,


renal function, liver function)

Physical Exam Findings • Complete Blood Count (to assess for hematologic disturbances)
Physical exam findings can guide a physician’s initial assessment
of a possible overdose patient. Track marks could be a clue to • Serum acetaminophen (paracetamol) levels (which is a common
intravenous drug abuse. The following table lists some key cause of overdose, does not cause any clinical manifestations
physical exam findings associated with certain toxins: initially and can be lethal)

• Ethanol level and other drug levels based on history or


Table 3.24 Physical exam findings and related toxins suspicion

• Urine pregnancy test in any female of child-bearing age

• Serum salicylate levels are frequently obtained in overdose


patients although salicylate toxicity can be suspected on initial
clinical examination.

• Urine drug screens are not useful in the initial clinical


assessment and management of overdose patients because

174
they are not able to detect a large number of drugs and can Table 3.25 Causes of high anion-gap metabolic acidosis
have false positives.

• For example, cocaine metabolites are detected for 3 days after


use, synthetic opioids like fentanyl are not detected on routine
urine drug screens, and pseudoephedrine produces a false
positive screen for amphetamines.

• Urine drug screens are often requested for the psychiatric


evaluation.

The anion gap is calculated as follows: Na – (HCO3 + CL), normal


< 14 typically. An elevated anion gap means there is an acidotic
process going on and can support the diagnosis of a toxic
ingestion. Many toxins and conditions can cause an elevated
anion gap metabolic acidosis. The following table lists these If a patient has altered mental status, always initially check a
substances and processes and the means by which they cause blood glucose level and consider empiric administration of
an anion gap acidosis (note the mnemonic CATMUDPILES[Table dextrose, naloxone, and thiamine in the primary exam along with
3.25]) ABCs.. The physician should consider a non-contrast Computed
Tomography scan of the brain to exclude intracranial pathology.
An Arterial Blood Gas can be helpful to evaluate acid/base status,
and co-oximetry can assess methemoglobin (MetHb) and
carboxyhemoglobin (COHb) levels. If toxic alcohol ingestion is
suspected, serum levels for methanol or ethylene glycol can be
obtained but are not readily or rapidly available. Measured serum
osmolality can be obtained to calculate the osmolar gap, which is
the difference between the measured osmolality and calculated
osmolarity (= 2Na + BUN/2.8 + Glucose/18 + Ethanol/3.7). A
175
normal osmol gap is between -12 and 10. Common Treatments and •Whole Bowel Irrigation
An elevated osmol gap can be caused by Antidotes
methanol, acetone, ethanol, mannitol, • Can be used with toxins that do not
A few common treatment modalities and
bind to charcoal (metals or lithium),
sorbitol, isopropanol, lactic acid or antidotes will be outlined below.
and drug packets
ethylene glycol ingestion.
GI Decontamination • Recommended within 6 hours of
Additional diagnostic tests and imaging • Gastric lavage
ingestion
should be considered based on the
history and presentation. For example, an • Within an hour of potentially lethal
• Administer polyethylene glycol 1 liter/
EKG can be ordered to assess the impact ingestion, particularly if no antidote
hour PO or per NG in adults and stop
of a certain drug on heart rate, rhythm treatment is available (i.e., colchicine,
when rectal effluent is clear
and interval length. calcium channel or beta blocker)
• Multi-dose Activated Charcoal
A chest radiograph can be ordered to • Charcoal
assess for pneumonitis after an ingestion • Useful for drugs that have
• Binds toxins in the stomach enterohepatic and enteroenteric
or inhalation. An abdominal radiograph
(KUB) can help identity radiopaque • Does not bind metals, lithium, iron circulation (i.e., Digoxin,
ingestions, including calcium carbonate, Theophylline, Carbamazepine)
chloral hydrate, heavy meals, iron, • Should not be used in caustic
ingestions and if there is a risk of • Useful for drugs with long
phenothiazines, enteric coated and
vomiting and aspiration (altered g a s t ro i n t e s t i n a l t r a n s i t t i m e s ,
sustained released drugs (mnemonic
mental status) including sustained release products
CHIPES).
and drugs that impede GI motility
Always consider consulting a clinical • The dose is 1g/kg with sorbitol or (i.e., anticholinergics, opiates)
toxicologist or a poison center for 25-50g in children
• Dose is 25 g every 4-6 hours for 2-3
assistance with managing toxic
• Recommended within 1 hour of the doses
overdoses. ingestion

176
• Enhanced Elimination References and Further Reading, click here

• Urinary alkalinization

• Enhances excretion of weak acids

• Recommended for salicylic acid and phenobarbital


overdoses

• Hemodialysis

• Can be used for ethylene glycol, methanol, isopropyl


alcohol, salicylic acid, and lithium. Best with small-sized
substances that are not protein bound and that have a
small volume of distribution.

Table 3.26 Common Toxins and Antidotes

177
Section 11

Respiratory Distress

Case Presentation
by Ebru Unal Akoglu A 40-year-old female with a history of diabetes mellitus
presents with a complaint of 6 days cough and muscle aches.
Patient has right-sided chest pain with deep breathing. Her
vitals are the following: temperature 37.1 degrees Celcius;
blood pressure 150/97 mmHg; heart rate 120 bpm; respiratory
rate 19/min; and pulse oximetry 89%. On physical
examination, she has diminished breath sounds and ronchi at
the right bases. Her chest X-ray is shown in Image 3.18.

Audio is available here

178
What are the diagnostic considerations? What is your next move? I n t h e e m e r g e n c y department, respiratory distress is a
What is the most appropriate management strategy? challenging chief complaint and diagnosis, and you should
evaluate, examine and ease (treat) the patient simultaneously. You
have to act quickly with limited information, or your patient can
Image 3.18 Chest x-ray.
decompensate in front of you.

Objectives of this chapter are listing the causes of respiratory


distress, describing the initial approach to a patient with
respiratory distress and discussing the initial management plan
for a patient with respiratory distress.

Introduction
Respiratory emergencies are common presentations to
emergency departments. Appropriate assessment and timely
interventions may be crucial in dyspneic patients. Respiratory
distress is responsible for nearly 4 million ED visits each year and
is one of the most common presenting complaints in the
elderly. Management of acute respiratory distress is a challenging
task. Good patient outcomes rely on your ability to assess
ventilation, oxygenation, work of breathing, lung function, airway
resistance and air flow.

When a patient presents with dyspnea, the primary task of the


emergency physician is to assess for and ensure the stability of
the patient’s airway, breathing, and circulation (ABC).

Respiratory distress is used to describe varying degrees of


problems in the respiratory system.

179
Rapid assessment may necessitate intubation, BiPAP (Bilevel Healthy lungs are the cornerstone of fluid regulation among
Positive Airway Pressure), nebulizations, decompression or other the interstitium and alveoli, which can be destroyed by lung injury.
therapies in the immediate period following the patient’s
Lung injury can cause abnormal gas exchange, impaired
arrival. Sometimes, it may be hard to decide whether your patient
compliance, and pulmonary pressure. Normal lung function
needs medication, suctioning, airway management, intubation,
requires dry, patent alveoli assisted by proper capillary perfusion
mechanical ventilation support (invasive, non-invasive) or just
and patent endothelium.
close observation.
Respiratory distress is a consequence of an alveolar injury
Respiratory distress is a term utilized to summarize a complex of
producing diffuse alveolar damage. Tumor necrosis factor,
clinical features. These are tachypnea, hypoxemia (peripheral
interleukin (IL)-1, IL-6, and IL-8, are the pro-inflammatory
arterial oxygen saturation [SpO2] <90% on room air), increased
cytokines released after injury and recruit neutrophils to the lungs.
work of breathing (intercostal, subcostal, or suprasternal
Activation of neutrophils causes endothelium damage that ends
retractions; nasal flaring; grunting; use of accessory muscles)
with impairment of hydrostatic and oncotic forces of membranes.
apnea, altered mental status, and cyanosis which is characterized
by >5gr/dL of deoxygenated hemoglobin. At this moment, it is Damage to the capillary endothelium causes the escape of
better to share some terms and definitions. Please check Table proteins from intravascular space. The membranous hydrostatic
3.27. and oncotic forces are lost, and the interstitial space fills with
fluid. Also, the clearance ability of the membranes may be lost.
Table 3.27 Terms and definitions in respiratory distress
Increase in interstitial fluid, combined with damage to the alveolar
epithelium, causes the air spaces to fill with bloody,
proteinaceous edema fluid and debris from degenerating cells.
Besides, the functional surfactant is lost, resulting in alveolar
collapse.

Lung injury has numerous consequences including impairment of


gas exchange, decreased lung compliance, and increased
pulmonary arterial pressure.
180
Patients with acute respiratory distress tend to progress through Airway
three relatively discrete pathologic stages. These are 1) Exudative Abnormal breath sounds often point to the obstruction. Snoring
stage: diffuse alveolar damage, 2) Proliferative stage: resolution of indicates obstruction of the airway, usually by the tongue. Simple
edema, squamous metaplasia, deposition of collagen, and 3) interventions can lead to marked improvement. For example,
Fibrotic stage: diffuse fibrosis and cyst formation. head tilt maneuver or a nasopharyngeal/oropharyngeal airway
often eliminates snoring. Inspiratory stridor suggests obstruction
Acute respiratory distress is the clinical consequence of lung
above the vocal cords (a foreign body obstruction or epiglottitis).
injury. Many predisposing factors may lead to lung injury (Table
Any foreign body should be removed immediately. Expiratory
3.28). Associated abnormalities increase the risk for adverse
stridor often comes from below the cords (as in croup or a deeper
outcomes.
foreign body).

Initial Stabilization
The following three assessment questions guide management:
Breathing
Coarse lung sounds, formerly called rhonchi, generally result from
secretions in the airway. Nasotracheal suctioning of accumulated
Table 3.28 Predisposing factors
secretions using a soft, flexible catheter clears coarse-sounding
lungs. Wheezing suggests flow restriction below the level of the
trachea, whereas crackles (or rales) indicate the presence of fluid
or atelectasis at the alveolar level. Administration of an inhaled
bronchodilator significantly reduces wheezing.

The most difficult management part of a patient with respiratory


distress is ventilation support. Unfortunately, unrecognized
inadequate breathing, failure of ventilator support or unassured
1. Is the airway patent? (A) airway will ultimately lead to cardiopulmonary arrest. If breathing
is inadequate, ventilation must be provided immediately.
2. How adequate is breathing? (B) Ventilation can be non-invasive or invasive.

3. Is oxygenation sufficient? (C)


181
Non-invasive refers to ventilator support is oxygen support using an oxygen mask, B – Breathing: Oxygenation (nasal
provided through the patient’s upper nasal cannula or bag-valve mask, cannula, non-rebreather mask) if not
airway, usually using an oxygen mask, depending on the patient’s need. If responding, next step is non-invazive
nasal cannula or bag-valve-mask (BVM) oxygen saturation still does not increase, mechanical ventilation support or
depends on the patient’s need. airway devices should be applied, or the intubation
patient should be intubated. The aim is
Invasive refers to ventilator support D – Drugs: Depending on the patient’s
the correction of hypoxia ideally to
provided by passing the upper airway primary problem causing respiratory
maintain saturation at 94-98% by titration
with an endotracheal tube, supraglottic distress, appropriate drugs should be
of oxygen carefully. The treatment
airway (e.g., laryngeal mask airway or implemented to management
depends on the condition that causes
laryngeal tube) or tracheostomy accordingly.
respiratory distress. However, in a general
depending on the need. Unassured
approach, CAB+D – circulation, airway, The three signs of impending respiratory
airway patency is the most important
breathing, and drugs is the perfect arrest are:
indication for invasive ventilation.
treatment protocol in the ED.
• Decreased level of consciousness;
Circulation Some critical actions should be done at
Hypoxia is the lack of sufficient oxygen in every stage if necessary. • Inability to maintain respiratory effort;
the body. Hypoxia may result from an
C – Circulation: two large bore IV access, • Cyanosis.
airway patency problem, failure of
ventilation support, or an intact airway fluids if hypotensive, monitorization
Presence of one or more of these needs
with good breathing but poor perfusion immediate intervention. The untreated
A – Airway: oropharyngeal or
and oxygenation. In airway management, respiratory arrest will lead to cardiac
nasopharyngeal airway devices, LMA,
the first approach is the clearance of arrest eventually. Life-threatening
some maneuvers (Head Tilt, Heimlich),
airway and positioning; this maneuver conditions, such as airway obstruction,
suction, medication for an allergic
often increases oxygen saturation and acute coronary syndrome, pneumonia,
reaction
improves ventilation. If oxygen saturation cardiac tamponade, pulmonary
does not increase, the second approach embolism, asthma, anaphylaxis, trauma,

182
and exacerbation of chronic obstructive common symptoms. Not only family Pulse oximetry is a valuable
p u l m o n a r y d i s o rd e r m a y l e a d t o members, but also a brief conversation monitoring tool for the management of
respiratory distress and arrest. These with the paramedics, who transferred the respiratory distress patients. It is useful
critical problems should be treated during patient, can give you useful information for either making a decision when to
the assessment. about the patient and the surrounding administer oxygen or titration of oxygen
area they took the patient from. to avoid patient harm from too much
While you are examining the patient,
oxygen.
other staff members (such as intern, Physical Examination Hints
nurse, paramedic) may measure vital Although we do a focused and goal- For respiratory distress patients without
parameters and monitor the patient, directed physical exam in critical patients, immediate life threats, your next
obtain intravenous access and do ECG. a detailed physical examination also assessment focus should be to determine
Te a m w o r k w i l l a c c e l e r a t e y o u r provides important guidance. the patient’s work of breathing and
assessment process and allow you to respiratory pattern (video), looking for any
The general appearance of patient –
formulate a treatment plan while others tripoding or retractions. Retractions can
c o n f u s i o n , c y a n o s i s , d ro w s i n e s s ,
obtain a history from family or friends. be visualized during the assessment of
tachypnea, and pallor – can guide your
chest movements, and they are more
History Taking and Physical management. Also, respiratory rate and
valuable than lung sounds in the decision
Examination Hints oxygen saturation are two vital sign
of the respiratory distress severity.
measurements that are helpful in
History Taking Hints assessing and monitoring the degree of Lung sounds (video) such as wheezing,
Acute respiratory distress is one of the respiratory distress. The higher the rales, ronchi, and stridor further guide the
most common chief complaints in the ED. respiratory rate, the greater the work of differential diagnosis. Decreased sounds
The differential diagnosis includes many breathing and the more likely the patient or hyperresonance may also provide
disorders, so a careful history can be will eventually get tired. Oxygen additional clues. Lung sounds should be
helpful to narrow this wide differential. In saturation is important not only in examined from both sides of the chest
addition, past medical and family history, assessing but also following the progress wall even in supine positioned patients
trauma, travel, medications, allergies and of the patient. (video). Orthopnea, or the inability to lie
exposures should be considered with
183
flat, is not a test, but rather, a question to 5) restlessness, agitation, decreased level distress.  Generally, laboratory and
ask the patient. Sweating and diaphoresis of consciousness. radiological tests take a long time; you
in an environment where others are not should start the treatment before getting
sweating, suggests significant distress. Differential diagnosis results.
Having a wide differential diagnosis list
Jugular venous distension (picture), S3 for respiratory distress will allow you to Bedside tests
gallop, and peripheral edema indicate sort through the possible causes more • ECG, especially in elderly patients who
that the patient has fluid overload. Heart rapidly.  In the ED, you must think the usually present atypically with dyspnea
sounds such as murmur, or decreased worst case scenarios first, and you in acute coronary syndrome, is easy
sounds, guide the differential and also should try to rule out them. Respiratory and practical.
management. Pulses must be assessed distress differential diagnoses list has
What are your diagnosis and next action
bilaterally. various critical diseases. These are
about the ECG in a patient with shortness
anaphylaxis, asthma/COPD, acute
It is important to remember that anxiety is of breath and palpitation (Image 3.19)? -
coronary syndrome, pulmonary edema,
common in patients with significant Case – 68 yo female presented with
pulmonary embolism, pneumonia,
medical problems, just as in trauma. palpitation, dyspnea, unable to lay down.
pericardial tamponade, tension
COPD patients have it more often than Vitals are BP: 80/43 mmHg, HR: 160
pneumothorax, and upper airway
the general population. Secondly, even bpm, RR: 32 pm, Temp: 37 Celsius,
obstruction.
healthy, young patients may have a SatO2: 87%. Patient diaphoretic, cool,
medical cause for hyperventilation. A The above diagnoses are crucial and anxious. Chest auscultation revealed
thorough assessment is important not to should be treated immediately. Other basal to mid zone crackles on both sides.
miss clues of a medical or traumatic causes of respiratory distress should also Heart sound irregular. Bilateral 1+ pitting
condition. be assessed and managed properly. edema.

Key findings of severe respiratory distress • Bedside glucose level should be


Emergency Diagnostic Tests
are 1) retractions and use of accessory obtained in cases of a decreased level
and Interpretation
muscles, 2) inability speak full sentences, of consciousness and suspected
Multiple tests are available to narrow the
3) inability lie flat, 4) extreme diaphoresis, metabolic acidosis.
differential diagnosis of respiratory
184
Image 3.19 Imaging
• Chest X-ray and Computed Tomography are generally indicated
to detect and differentiate pathologies. Pneumothorax,
pneumonia, pulmonary embolism, pleural effusion, cardiac
tamponade, etc. are the important causes of respiratory
distress.

What are your diagnoses about the chest x-rays (Images 3.20 and
3.21) in patients with shortness of breath?

Image 3.20

Case – 68 yo female presented with palpitation, dyspnea, unable to lay


down. Vitals are BP: 80/43 mmHg, HR: 160 bpm, RR: 32 pm, Temp: 37
Celsius, SatO2: 87%. Patient diaphoretic, cool, anxious. Chest auscultation
revealed basal to mid zone crackles on both sides. Heart sound irregular.
Bilateral 1+ pitting edema.

Laboratory tests
• Arterial blood gas analysis is useful, quick and important to
determine metabolic and/or respiratory cause of respiratory
distress.

• Besides these, complete blood count (CBC), troponin, renal


panel, BNP, and D-dimer can be used to assess differential
diagnosis of respiratory distress.

185
Image 3.21

Emergency Treatment Options


Because of a variety of diseases can cause respiratory distress
and specific diseases need specific approaches such as tension
pneumotorax needs decompression of the air from the chest;
asthma and COPD need bronchodilator treatments. Please check
each critical diagnosis separately in other chapters.

Patients often die from the complications of respiratory distress.


The initial assessment is crucial, and essential interventions
should be made immediately.

Respiratory distress symptoms usually have a wide range of


ineffective breathing or respiratory arrest and difficulty in
speaking, accompanied by cyanosis and diaphoresis.

Immediate assessment priorities for any difficulty breathing


include quick determination of circulation, airway, and breathing
as described above.

Disposition
• Currently, ultrasonography is another option in the evaluation of
if the patient’s condition or blood gas analyze does not improve
respiratory distress (watch this video). Ultrasonography
despite therapy admission should be considered to appropriate
provides valuable information about the origin of symptoms and
clinics.
often diagnosis in the initial assessment of the patient. Also,
ultrasonography is faster than laboratory tests and other References and Further Reading, click here
imaging modalities, repeatable, and portable so that it can be
used for unstable patients. It is also cost-effective.
186
Section 12

Shock

Case presentation
by Maryam AlBadwawi A 61-year-old male with fever, shortness of breath and
vomiting was brought to the ED by ambulance. He also
complained of dizziness, malaise, and reduced urination. His
symptoms started one week earlier and got progressively
worse in time. However, he did not seek any medical
assistance before. His medical history includes diabetes
mellitus and hypertension.

On examination, he appeared ill. His vital signs were: BP:


80/50 mmHg, HR: 140 bpm, T: 38.6°C, RR: 30 bpm, SPO2:
90% on room air. His blood sugar was 5.3 mmol/dL, and
capillary refill was four seconds. He responds to verbal
comments on the AVPU score. On auscultation, coarse
crackles were heard on the right side. Heart sounds were
normal. There was 1+ pitting edema on bilateral legs.
Audio is available here

187
Abdominal examination was unremarkable, and His MAP improved to 65 and remained stable.
there was no focal neurological deficit. He was transferred to the ICU.

The pre-diagnosis was the septic shock. He


needed emergent resuscitation and antibiotics
administration.

Two large bore IV cannulas were inserted. CBC,


urea, electrolytes, creatinine, LFT, cardiac
enzymes, lactate, and ABG were ordered.
Bedside US scan revealed a collapsing inferior
vena cava. A urinary catheter was placed for
urinalysis and to monitor the output.

While waiting for the results, one liter of isotonic


saline, one dose of Vancomycin (15-20 mg/kg)
and Piperacillin-Tazobactam ( 80-100 mg/kg)
were administered. His BP did not improve. After
the second liter of isotonic saline, his HR settled
to 120, but his mean arterial pressure (MAP)
remained below 60 mmHg. A central line was
inserted, and epinephrine (2-10 mcg/min) started.
188
Introduction Table 3.29 Shock types
Shock, in simple terms, is a reduced circulatory blood flow state SHOCK MECHANISM/ CLINICAL
PHYSIOLOGY
within the body. The inadequate circulation deprives the tissues TYPE CAUSES EXAMPLES
of its oxygen and essential nutrients. Mitochondria are the first Hypovolemic The decrease in External bleeding Trauma, AAA
plasma or RBC + Internal bleeding rupture, ectopic
cellular structure to be affected by hypoxia. As a result of plasma pregnancy rupture,
Third spacing loss, nausea and
anaerobic respiration, they start to produce lactate, leading to
GI, renal, or vomiting,
lactic acidosis. Initially, the shock is reversible. Extended shock insensible losses inadequate intake,
(hemorrhagic) renal diuresis,
state leads to cellular and organ dysfunction and ultimately death. paraesthesia,
burns.
The shock is a life-threatening medical emergency with a Cardiogenic A primary cardiac Pump failure from Ischemia and
problem myocardial injury or infarct,
mortality rate up to 20%. The primary goal of resuscitation is to decreasing dysfunction Myocarditis,
optimize organ perfusion. It may result from a variety of causes. It cardiac output. Arrhythmias Cardiomyopathy
Valvular disease VT, VF
is divided into four categories according to the mechanism of the Ventricular septal Aortic regurgitation
defect
disease and treatment (Table 3.29). Differentiating the type
Obstructive Non-cardiac Impaired RV diastolic Cardiac
facilitates treatment, however, all categories of shock lead to the obstruction filling tamponade,
same result. affecting cardiac Impaired RV filling Constrictive
filling or emptying due to obstructed pericarditis
venous return Tension
Increased right pneumothorax
ventricular afterload Pulmonary
Increased left embolism
ventricular afterload Aortic dissection

Distributive Extreme Peripheral pooling Sepsis or SIRS


peripheral Capillary leak Anaphylaxis
vasodilation Myocardial Neurogenic shock
depression Rewarming in
severe
hypothermia
Endocrinological

Adopted from following references. Please read (Marx, J. A., Hockberger, R. S., & Rosen, P. (2014).
Rosen's emergency medicine: Concepts and clinical practice (8th ed., Vol. 1). Philadelphia, PA:
Mosby Elsevier and Avegno, J. CDEM Self-Study Modules. The approach to shock. Retrieved May
11, 2016) references to get more information.

189
Critical Bedside Actions and General • C a r d i o g e n i c s h o c k occurs when more than 40% of the
Approach myocardium undergoes necrosis from ischemia, inflammation,
Early and accurate management is essential as it reduces toxins or immune destruction. It induces the same impairment
mortality significantly in certain types of shock. Heart rate, blood as hemorrhagic shock. Patients have evidence of ventricular
pressure, and partial oxygen saturation must be continuously dysfunction earlier in the disease.
monitored. Optimizing airway, breathing, and circulation (ABC) is
• Obstructive shock should be considered in patients with chest
the priority. Check the airway of the patient, and consider
pain, shortness of breath, and altered mental status. The
intubation in case of inadequate oxygenation and ventilation.
physical examination may reveal jugular venous distention,
Maximizing arterial oxygen saturation through proper oxygenation
muffled heart sounds, pulsus paradoxus, tachypnea,
is crucial. Central venous oxygenation should be monitored with a
tachycardia, cold extremities, friction rub, new murmur, and
target of minimum 70%. Reducing the work of breathing lessens
signs of deep vein thrombosis.
the metabolic load.
• Distributive shock findings depend on the cause. Sepsis and
History and Physical Examination Hints septic shock cause signs of infection. Other symptoms and
Common features of the shock include hypotension, altered signs include hypo/hyperthermia, tachycardia, tachypnea, wide
mental status, and oliguria, regardless of the etiology. The patient pulse pressure, warm extremities, altered mental status,
history is significant to diagnose the type of shock and accurately oliguria, and skin rash. Anaphylactic shock is characterized by
treat the patient. skin and mucosa manifestation such as urticaria, flushing,
pruritis, and angioedema. Respiratory symptoms may include
• Hypovolemic shock might have a history of trauma, pregnancy,
rhinitis, bronchospasm, dyspnea, and stridor (pharyngeal/
gastrointestinal losses or burn. Initially, heart rate and force of
laryngeal edema). The patient may experience dysrhythmias,
contraction increase. Vasoconstriction causes elevated diastolic
hypotension, presyncope, and syncope. Additionally, GI
BP, and pulse pressure (the difference between systolic and
symptoms such as nausea, vomiting, and diarrhea may be
diastolic BP) narrows. The blood flow to the noncritical organs
present.
decreases so that cells produce lactic acid. As bleeding
continues, ventricular filling and cardiac output (CO) decrease, • Blood pressure may be normal or even high in the early course
resulting in decreased BP. Hypotension is a late sign of shock. of shock; however, if left untreated, it may proceed to
190
tachycardia and hypotension. Shock Index (heart rate divided information about prerenal causes showing a volume or
by systolic blood pressure), may reveal obscure shock. The blood loss.
normal shock index ranges from 0.5 to 0.7. A value of >0.9 is
• Urinalysis
considered abnormal and associated with higher mortality.
• Urosepsis is one of the common sepsis causes.
Emergency Diagnostic Tests and
Interpretation • Hepatic function tests
The suspected cause of shock, attributed from the history and
• Hepatic functions impair because of low perfusion.
physical examination, should guide diagnostic testing. The goal is
to determine the involvement of organ hypoperfusion and • Lactate
damage. The following are helpful investigations in shock:
• Lactate gives an opinion about the hypoperfusion status. Its
• Complete blood count and coagulation profile levels considered normal between 0.5-1 mmol/L. Lactate
levels more than 2 considered as abnormal in the critically ill
• Anemia, infection, hypo-coagulopathy related abnormalities
patients. Levels more than 4 shows increased the risk of
can be seen
mortality and morbidity. Therefore, those levels are used for
• Electrolytes some institutions to decide ICU admission.

• Some of the cases may show electrolyte disturbance • Urine pregnancy test
because of their comorbidities or continuous medical
• Considering every female patient in childbearing age as a
problems affecting their nutrition or metabolism.
pregnant patient is essential thinking in the ED.
• Renal function tests
• Chest x-ray
• Blood urea nitrogen/creatinine
• For pneumonia, pleural effusion, and other possible shock
• Renal functions impair because of low perfusion. High causes such as cardiac tamponade, aortic dissection,
blood urea nitrogen compared to creatinine may provide pneumothorax (tension).

191
What are your diagnoses in hypotensive patients with below • ECG
chest x-rays?
• For arrhythmias, MI, cardiomyopathy and other findings

Image 3.22 Image 3.24 • US (RUSH protocol to find the cause of the shock (video)

• Other invasive tests:

• Arterial blood gas analysis for O2 pressure and pH level,

• Some centers may prefer to measure systemic vascular


resistance, central venous oxygen, and cardiac output in the
ED.

• Certain etiologies of shock will require additional investigations:

Image 3.23 • SIRS and sepsis: Cultures (blood, sputum, urine, or wound),
head CT and lumbar puncture

• Cardiogenic: ECG, cardiac enzymes, and echocardiography

• Obstructive: CT or V/Q scan for PE, echocardiography for


cardiac tamponade

192
Table 3.30 Important Physiological Changes in Shock
CENTRAL SYSTEMIC TISSUE
SHOCK TYPE HEART RATE VENOUS CONTRACTILITY EXTREMITIES VASCULAR PERFUSION/
PRESSURE RESISTANCE SCVO2

Hypovolemic Increased Decreased +/- Increased Cool Increased Decreased

Cardiogenic Increased Increased Decreased Cool Increased Decreased

Increased in
Tamponade and
Decreased or
Obstructive Increased +/- Increased +/- Cool PE, but decreased
Increased
in tension
pneumothorax

Distributive Increased Decreased +/- Warm Decreased Decreased

In any ill-appearing patient with tachycardia and hypotension or high shock index, the shock must be considered. The mentioned signs,
symptoms and relevant diagnostic tests often help to arrive at a diagnosis and initiate appropriate treatment. However, certain disease
processes can complicate the picture and lead to an alternate diagnosis. Therefore, understanding the shock physiology is important
(Table 3.30). The labs should be completed, but it is important not to wait for the results before initiating treatment. 

Using certain criteria to help make the diagnosis and point to specific types of shock is more beneficial.

193
Septic shock Tachypnoea: respiratory rate greater 2. Heart rate >100 beats/min
• SIRS (Systemic Immun Response than or equal to 22 breath per
3. Respiratory rate >20 breaths/min
Syndrome) – Two or more of the minute. 2 or more criteria violation in
or PaCO2 <32 mmHg
following: Q sofa score is considered the poor
outcome predictor. 4. Arterial base deficit <-4 mEq/L or
1. Temperature >38°C or <36°C
lactate >4 mmol/L
• Septic shock
2. Heart rate >90 beats/min
5. Urine output <0.5 mL/kg/hr
• Sepsis and hypotension despite
3. Respiratory rate >20 breaths/min
adequate fluid resuscitation. 6. A r t e r i a l h y p o t e n s i o n > 3 0
or PaCO2 <32 mmHg
continuous minutes
Hemorrhagic shock
4. WBC >12,000/mm3, <4,000/
• Simple hemorrhage Cardiogenic shock
mm3, or >10% band neutrophilia
• Cardiac failure
• Suspected bleeding with normal
• Sepsis
vitals and normal base deficit • Clinical evidence of impaired forward
• SIRS with finding the source of flow of heart, including presence of
• Hemorrhage with hypoperfusion
infection and associated with organ dyspnea, tachycardia, pulmonary
damage or hypoperfusion. • Suspected bleeding with base deficit edema, peripheral edema, or
<-4 mEq/L or persistent pulse >100 cyanosis.
• Sequential Organ Failure Assessment
beats/min
(S.O.F.A. or sofa) was recently • Cardiogenic shock
described and created multiple • Hemorrhagic shock
• Cardiac failure and at least four
discussions in emergency and critical
• Suspected bleeding with at least four criteria of that similar to hemorrhagic
care journals. q (Quick) sofa score
of the following criteria: shock.
includes Hypotension: systolic blood
pressure less than or equal to 100 1. Ill appearance or altered mental
mmHg, Altered mental status, and status

194
Emergency Treatment acceptable. Additionally, controlling the through beta-1 adrenergic agonist

Options source of bleeding is critical. In special effect, and it may cause mild peripheral
cases like hereditary or acquired bleeding vasodilation through beta-2 adrenergic
Fluid Resuscitation diathesis, platelet transfusion is indicated agonist effect. Dopamine at moderate
Two large bore IV access should be if platelet count <50,000/μL. FFP doses (5-10 μg/kg/min) has alpha and
obtained to support the circulatory transfusion is indicated for patients on beta-1 adrenergic effects.
system. A central line is also very warfarin with an elevated INR and
b e n e fic i a l i n d e l i v e r i n g flu i d a n d significant bleeding, liver failure, or Treatment Success
m e d i c a t i o n , e s p e c i a l l y i n o t ro p e s . Monitoring fluid status is encouraged by
massive transfusion (>10 units PRBC in
Crystalloid fluids (normal saline or using a urinary catheter, intra-arterial
24 hours). PCC is used for warfarin
Ringer’s lactate) should be used in blood pressure measurements, and
reversal (FFP, if not available). In a
boluses (2-3 L bolus in 5-20 min – 20ml/ central venous pressure monitoring.
massive transfusion, plasma, platelets,
kg in neonates and pediatrics). Pay close and red blood cells should be given in When patients’ hemodynamic status
attention to patients in cardiogenic shock. 1:1:1 ratio. become normal (blood pressure, heart
Do not administer I.V. fluids rapidly to
rate and urine out put) and necessary
patients with signs of pulmonary Inotropes
volume restored. These help to maximize
congestion. Small fluid boluses such as If volume resuscitation does not improve
tissue oxygenation, resolution of acidosis
250 mL should be preferred in those the patient’s hemodynamic status and
and decrease lactate levels. These are
cases. MAP remains below 65, inotropes may be
the findings of successful resuscitation.
used. Inotropes are also used in
Blood Products Resuscitation cardiogenic shock for depressed LV
Medications
Blood transfusion is considered if there is function. Norepinephrine (2-10 mcg/min) • Distributive shock
no response to two liters of fluid boluses, stimulates alpha and beta-adrenergic
ongoing hemorrhage, or impending receptors, increasing peripheral vascular • The treatment depends on the
cardiovascular collapse. O-negative tone. Dobutamine (2.5-15 mcg/kg/min) specific cause of shock.
blood is standard for child-bearing may improve myocardial contractility and
women and O-positive in men is augment diastolic coronary blood flow

195
• In septic shock, it is important to • Pediatrics: 0.01 mg/kg, max 0.3 Image 3.25 Chest X-ray shows
start early broad-spectrum mg of 1:1000 solution IM q5-10 position of the chest tube in a patient
with pneumothorax.
antibiotics: minutes

• Neonates: Ampicillin (150-200 mg/ • Histamine blockers.


kg) with Cefotaxime (150-200 mg/
• Diphenhydramine (H1 blocker):
kg)
(25-50 mg IM/IV, pediatric 1
• Children: Vancomycin (15-20 mg/ mg/kg IM/IV)
kg) and Cefotaxime (150-200 mg/
• Cimetidine or ranitidine (H2
kg)
blockers).
• Adult: Vancomycin (15-20 mg/kg)
• Aerosolized albuterol and
with Piperacillin-Tazobactam
ipratropium are important in
( 80-100 mg/kg)
anaphylactic shock. An intra-aortic balloon pump decreases
• Consider low-dose corticosteroids afterload and increases diastolic BP and
• Steroids should be considered
to treat relative adrenal bridge to revascularization or valvular
suppression if refractory
Procedures repair. Also, cardiogenic shock may
hypotension is present despite In the management of obstructive shock, necessitate emergent angiography or
fluids and inotropes. it is essential to remove obstruction by surgical procedures such as bypass or
pericardiocentesis in cardiac tamponade valve repair.
• Anaphylactic shock
tamponade, by needle thoracocentesis
• Epinephrine is the first line of followed by tube thoracostomy in tension Disposition Decisions
treatment. pneumothorax, by thrombolysis in Despite proper treatment, the mortality

massive P.E., and afterload reduction until rates from severe shock can exceed 50
• Adults: 0.3-0.5 mg of 1:1000 percent. Even after aggressive treatment
definitive treatment in aortic dissection.
solution IM q5-10 minutes in the ED, ICU admission is required.

196
References and Further Reading, click here

197
Chapter 4

Selected
Cardiovascular
Emergencies
Section 1

Abdominal Aortic Aneurysm (AAA)

Case Presentation
by Lit Sin Quek A 75-year-old obese man comes to the emergency
department. He has history C.O.P.D., hypertension. He is a
smoker and on regular follow-up with primary care. He
describes sudden onset severe flank and back pain for past 2
hours. He denies any chest pain or dyspnea. He informs the
physician about his chronic abdominal pain. His initial vital
signs are HR 98 bpm, RR 24/min, BP 190/105 mmHg, T
36.9C. His examination revealed mild abdominal pain without
rigidity or rebound tenderness. Bedside ultrasonography
performed and the result is shown below.

Image 4.1

Audio is available here

199
Introduction The leading causes of AAA are; •Cardiovascular risk factors and
Abdominal Aortic Aneurism (AAA) rupture vascular bed affection: People with
• Atherosclerosis coronary artery disease and peripheral
is one of the serious problems which
should be suspected in every • Genetic predisposition (weakening vs. artery disease are more prone to have
hypotensive elderly with abdominal pain. occlusion) AAA.
This chapter’s learning objectives are;
• Connective tissue diseases • Family history: A family history of AAA
• Understand the epidemiology and increases the risk of developing AAA.
pathogenesis of AAA • Marfan’s, Ehlers-Danlos The risk of developing an AAA may
reach 20% among brothers of a patient
• Appropriate diagnostic measures • Infection (Syphilis, salmonella, others)
with a known AAA.

• Clinical Key Points Epidemiology


• Smoking: It is a risk factor for many
There are many factors affecting AAA
diseases. Number of years of smoking
• Recognize indications for referral development. These factors are very
is related to high risk.
critical to reaching the diagnosis and
Abdominal Aortic Aneurysm
knowing those can help you during the • Diabetes mellitus: There is a negative
(AAA)
history taking, even in the management of association with diabetes mellitus and
AAA is defined by International Society
the patient. Factors Associated with AAA AAA.
for Cardiovascular Surgery and Society
are as follows;
for Vascular Surgery as “a focal dilation • Hypertension: It is a poor predictor for
(widening) of the abdominal aorta where • Old age: generally above 55 considered AAA development but important risk
the diameter is at least 50% larger than is a risky cut-off for AAA. factor for expansion and rupture.
the expected normal diameter for that
individual.” However, most clinicians will • Gender: Men develop AAA 4-5 times • Lipid: There is no and weak correlation
consider the diagnosis of an AAA if the more often than women between risk for AAA and high serum
diameter greater than 3 cm. triglyceride and cholesterol,
• Ethnicity: White people develop AAA
respectively.
more frequently than other ethnicities

200
• Emphysema: It is the strongest Presentation •Myocardial infarction
independent risk factor for rupture. Abdominal/back pain, a pulsatile mass,
• Musculoskeletal back pain
Prevalence is 5% to 7% of people over and hypotension are known as the classic
the age of 65 in the United States. triad, but only seen 1/3 of the patients. The patients may have a variety of
There is a 3:1 ratio of men to women. So, you have to lower your threshold to differential depending on their symptoms.
After age 65, the prevalence of 3 cm be suspicious for cases showing The important clue to keep in mind is
aneurysms in men increases by epidemiologic warnings described above. each of these specific diseases shows
approximately 6% per decade. their specific symptoms, and as a rule of
Critical Bedside Actions and thumb, these symptoms may be indirectly
Types of AAA General Approach mimicking AAA, especially elderly
• Saccular aneurysm – is an outpouching As described in many other chapters
patients and patient who have risk
arising from one part of the aorta, has a (e.g., Shock), the primary goal is the
factors.
neck, and does not involve the entire re s u s c i t a t e a n y u n s t a b l e p a t i e n t .
circumference of the aorta. Therefore, airway, breathing, circulation History and Physical
should be evaluated immediately and Examination Hints
• Fusiform aneurysm – is tubular in
resuscitative measures implemented. If Many of the patients are elderly. Because
shape, involves the entire
the rupture is suspected, immediate of their pain sensation affected by
circumference of the localized aorta,
s u rg i c a l c o n s u l t a t i o n a n d b l o o d multiple comorbidities, AAA patients may
and has no neck.
transfusion to the patient is a must. Do not give clear history hints to physicians.
• Pseudoaneurysm – dilatation is only at not delay the definitive treatment which is Most of the times, symptoms are very
the outside layer of the aorta (tunica surgery. subtle unless hypotension and shock
adventitia) situation in rupture. The patients showing
Differential Diagnoses epidemiologic risk factors should be
• Mycotic aneurysm – a rare aneurysm • Renal colic
questioned very carefully.
caused by a fungal infection which may
be associated with immunodeficiency, • Diverticulitis
Physical examination of the patients
IV drug abuse, heart valve surgery. should include relevant organ systems
• GI bleeds
201
that patient having risk factors or symptoms.  Specific attention Laboratory tests
should be given to understand instability. The most important issue for these patients is bleeding.
Therefore, type and cross-match blood is the most critical test.
Specific exam for AAA includes deep gentle palpation, above the
CBC, Urea/Creatinine, coagulation studies and urinalysis are
umbilicus, left of midline, continuous over several heartbeats.
other tests.
Bleeding into retroperitoneum may create doughy abdomen.
Hypotension also minimizes pulsations. Imaging modalities
• Ultrasound provides low cost, reliable, fast and safe approach.
Some facts;
However, it is operator dependent modality. Poor imaging
• 38% patients AAA initially detected by physical examination above renal vessels, obesity, intestinal gas, or very painful
abdomen may affect the proper investigation. Please see RUSH
• 62% found incidentally on imaging studies done for other protocol chapter to learn more about aortic ultrasonographic
indications
evaluation.

• AAA detected by physical examination had lower BMI but there


Image 4.2 US - AAA
was no difference in AAA size

• 43 % of AAA detected on radiologic examination is palpable


and should have been detected on physical examination.

• 23% AAA were not palpable on pre-operative physical


examination, even when the diagnosis was known.

• Obese patients had only 15% of AAA detected by physical


examination, and only 33% were palpable.

E m e r g e n c y D i a g n o s t i c Te s t s a n d
Interpretation

202
Abdominal aorta investigation with ultrasound. Tutorial in 3 • Abdominal X-Ray/KUB may incidentally show findings of
minutes take a look this video. AAA. AAA can be seen in 60-75% of cases in the x-ray with the
calcification of aortic wall or paravertebral mass.
• CT Scan with contrast is a gold standard (Image 4.3). It shows
better demonstration extent and complications of an aneurysm, • Cross-table lateral most helpful view and a negative study is not
retroperitoneal blood because of rupture, and dissection. helpful.
However, the patient instability affects the usage of this imaging
modality. Emergency Treatment Options
Medications
Image 4.3 CT scan - AAA There are no specific medications for AAA patients. However,
some patients may require blood pressure and arrhythmia
management. In the unstable patients, intubation with rapid
sequence intubation (RSI) protocol, fluid and blood replacement
should be considered.   Analgesics also an important part of the
treatment.

Procedures
Any critically ill patient who diagnosed AAA (potentially rupture)
should immediately be intubated and airway secured. This also
prepares the patient for the operation theatre. Some patients may
have no peripheral IV access because of their shock situation.
These patients require an intraosseous line or central I.V
placement. Although these resuscitative measures keep the
• MRI has no advantage over CT scan.
patient alive and any ruptured patient should directly go to the
• The angiogram is not preferred for diagnosis but good for pre- operation theatre, you should also know some other red flags for
op “mapping.” the indications for repair of AAA.

203
• Size more than 5.5 cm. However, 5.0 Additional Information: see the video on are associated with similar mortality
cm still used in common practice by repairs – link up to 10 years.
many surgeons.
Clinical Key Points – putting Patients treated with endovascular
• Symptoms such as abdominal or back it all together repair require long-term surveillance
pain, to groin in some cases or Abdominal aortic aneurysms are owing to a small risk of aneurysm sac
tenderness of AAA. asymptomatic until they rupture, reperfusion and late rupture.
resulting in a mortality of 85 to 90%.
• Risk of Rupture: Emphysema, smoking, Decisions regarding prophylactic
hypertension increase likelihood of Urgent repair is the only definitive repair — whether to pursue and what
rupture. Regarding Powell et al.’s study option for symptomatic patients. type of repair to perform must take
aneurisms less than 5.5 cm in diameter into account anatomy (not all
has less than 1% of rupture in one year. Although the optimal group to be situations can undergo endovascular
Above 5.5 cm risk is between 9.4% to screened remains controversial, repair), operative risk, and patient
32.4 (more than 7 cm). preference.
Smoking men or women 65 to 75

• Rupture (“Leak”) as we discussed years of age and 65 to 75 years of age


Disposition Decisions
above. non-smoking man should undergo
screening and selective screening, Admission criteria
Other treatment options and respectively. • All unstable patients should be
management strategies should be transferred to operation theatre
thought in stable AAA patients such as; The threshold for elective repair is an
immediately.
aortic diameter of 5.5 cm in men and
• Observation: Small aneurysms < 5 cm 5.0 cm in women, but this may vary • Stable patients with high risk of rupture,
with practices. if they are not going to operation
• Elective repair: Open surgical repair or
theatre, they can be admited into ward
endovascular (stent-graft) repair Endovascular repair results in lower
or ICU depending on institution
perioperative morbidity and mortality
protocols.
than open repair, but the two methods

204
Discharge criteria
• Asymptomatic patients only patient group can be discharged if
they do not have any risk factor for rupture. Patient with risk
factors should be evaluated carefully. If they are decided to
discharge, close follow-up in the clinic should be arranged. The
outpatientclinic folow-up for other patients must also be
arranged before their discharge from the emergency
department. Instruction specific to AAA should be given to
patients.

References and Further Reading, click here

205
Section 2

Acute Coronary Syndrome (ACS)

Case Presentation
by Khalid Mohammed Ali, Shirley Ooi A 46 years old man with a past medical history of
hypertension and hyperlipidemia developed central crushing
chest pain associated with sweating and shortness of breath
while driving. He presented to the emergency department 1
hour after the onset of chest pain. On physical examination,
his vital signs were as follows; pulse rate: 60 beats/min, blood
pressure: 100/50 mm Hg, respiratory rate: 20/min, SpO2 98%
on room air. Patient has no leg edema, new murmur or
features of heart failure.

His ECG is as is given.

What are the ECG features? What is the diagnosis?

Audio is available here

206
• Chest pain described as pressure, squeezing or a burning
Image 4.4
sensation across the precordium and may radiate to the neck,
shoulder, jaw, back, upper abdomen, or either or both arms.

• Shortness of breath especially exertional dyspnoea

• Palpitation

• Diaphoresis from sympathetic stimulation

• Nausea from vagal stimulation


Answer
• Decreased effort tolerance
ECG features are ST elevation more than 2 mm in inferior leads (II,
III, AVF) with reciprocal changes (deep ST depression in lateral It is important to ascertain the onset of chest pain as it will affect
leads of I, AVL) and ST depression in lead V1, V2 which indicate the management of STEMI. Other important questions to ask are
the involvement of posterior wall. The ECG is diagnostic of the chest pain character, radiation, associated symptoms,
inferior and posterior wall ST elevation MI. relieving and exacerbating factors, especially exertion.

Definition Do not forget to ask about the risk factors. You can use the TIMI
Acute coronary syndromes (ACS) include conditions that share score.
the same pathophysiology of myocardial ischaemic states, i.e.,
TIMI (Thrombolysis in Myocardial Infarction) investigators have
unstable angina (UA), non-ST elevation myocardial infarction
developed a 7-variable risk stratification tool that predicts the risk
(NSTEMI) and ST-segment elevation myocardial infarction
of death, re-infarction, or urgent revascularization at 14 days after
(STEMI).
the presentation:

History and Physical Examination Hints • ≥ 65 years of age


The patient may experience the following symptoms:
• Presence of ≥ three cardiac risk factors

207
• Prior coronary artery stenosis of ≥ 50% the base of the lung may indicate either existing heart failure or
an acute one secondary to acute myocardial ischemia
• >= two angina events in the preceding 24 hours
• A new cardiac murmur may indicate acute valvular insufficiency
• Aspirin use in the previous seven days
or rupture interventricular septum
• ST-segment deviations of ≥ 0.5 mm on ECG at presentation
• Distant heart sound on auscultation of precordium may indicate
• Positive cardiac biomarkers acute pericardial effusion secondary to rupture of a free
ventricular wall or acute aortic dissection with extension to
Patients are considered to be high risk if their TIMI risk score is ≥ precordium.
5 and low risk if the score is ≤ 2. High-risk patients have a more
significant benefit from early percutaneous coronary intervention In addition to the above, it is essential to check the vital signs
and use of adenosine phosphate inhibitor and low molecular carefully. If the patient has hypotension with acute myocardial
weight heparin than lower risk patients. ischemia, this may indicate cardiogenic shock. Tachycardia may
range from sinus tachycardia to ventricular tachycardia;
Most of the cases with ACS have a normal cardiovascular bradycardia, on the other hand, may range from sinus
examination. In a busy emergency department where time is of an bradycardia to third-degree heart block.
essence, targeted physical examination in the patient with ACS is
important to rule out complications and possible differential Differential Diagnosis
diagnosis. There are many critical differential diagnoses when we consider
A.C.S. Please look for all in the given table. However, Unstable
The following are essential components:
angina, Acute myocardial infarction, Acute pulmonary embolism,

• Differential pulse and BP between both arms, which if present Acute aortic dissection, Tension pneumothorax, Oesophageal
may indicate the possibility of aortic dissection rupture (Boerhaave’s syndrome) are the life-threatening ones. We
advise you to read these chapters form multiple resources to feel
• Tachypnoea, pitting leg edema and raised jugular venous confident.
pressure with crackles in the base of the lung or only crackles in
See the following table:

208
• Arrhythmias include tachyarrhythmias and
Table 4.1 ACS/Chest Pain Differential Diagnosis bradyarrhythmias
LIFE REFERRED
CVS RESPIRATORY GI
THREATENING PAIN
• Cardiogenic shock
Unstable Stable angina Pneumonia Gastro- Sub phrenic
angina oesophageal abscess /
Acute Simple reflux inflammation
• Cardiac arrest in the form of ventricular fibrillation
Acute pericarditis pneumothora
myocardial x Oesophageal Hepatobiliary
infarction Myocarditis spasm disease Decision Making Process and Reaching The
Acute
Diagnosis
pulmonary There are three pillars of diagnosis: history, ECG, and cardiac
embolism
enzymes.
Acute aortic
dissection Clinical features of unstable angina include the following:
Tension
pneumothora • Unstable angina differs from stable angina in that the chest pain
x
is usually more intense, easily provoked, more prolonged, more
Oesophageal
rupture
frequent and more severe. All first presentation of angina should
(Boerhaave’s be regarded as unstable. In unstable angina typically there is
syndrome)
either no ECG changes or non-specific ECG changes, the
patient is usually chest pain-free on presentation to the
Acute Complications emergency department, and the cardiac enzymes will be
The acute complication which we may see in the emergency
normal.
department includes the following:
• NSTEMI should be diagnosed in any patient whose cardiac
• Acute pulmonary edema due to acute myocardial ischemia
enzymes are raised without evidence of ST elevation MI. An
which leads to decrease effective ejection fraction and heart
NSTEMI does not need to have ECG changes at the time of
failure
presentation. The ECG may show the following:
• Mechanical complications include rupture of papillary muscles,
1. ST-segment depression
free left ventricular wall, and interventricular septum
209
2. The transient ST-segment elevation that resolves
Table 4.2 ST Segment Changes And Its Anatomical Relation
spontaneously or after glyceryl trinitrate treatment In Acute Myocardial Infarction
3. T-wave inversion ST SEGMENT
LOCATION LEADS
CHANGES
4. Evidence of previous myocardial infarction

5. Left bundle branch block Anterior wall V1 -V4 Elevation

6. Minor non-specific changes

The ECG can also be normal. It should not show persistent acute Inferior wall II, III, aVF Elevation

ST-segment elevation.
V8 and V9 Elevation
• STEMI (ST-segment elevation MI) is a true cardiac emergency. Posterior wall
V1-V3 Depression
The criteria of diagnosing ST-segment elevation MI on ECG
are: New ST elevation at the J point in at least two contiguous
Right Ventricular wall
V4R, V5R, V6R Elevation
leads of ≥ 2 mm in leads V2–V3 and/or of ≥ 1 mm in other MI
contiguous chest leads or the limb leads.

Lateral wall I, aVL, V5 and V6 Elevation

210
There are other causes of ST elevation should be known to acute pulmonary congestion or indicate the diagnosis
differentiate it from the ST elevation of myocardial infarction. of other conditions like pneumothorax or acute aortic dissection.
These are;
Emergency Treatment Options
• Acute pericarditis
Initial Stabilization
• Benign early repolarization In typical emergency medicine room, once a patient presents with
chest pain suspecting of acute myocardial ischemia should be
• Brugada’s Syndrome
seen in the monitored area, the patient should remain under
• Hyperkalemia continuous cardiac monitoring, HR, BP, and SpO2.

• Left Bundle Branch Block The proper approach will consist of all following:

• Left ventricular aneurysm ECG should be done immediately or within the first 10
minutes by the emergency room staff nurse, which should
• Left ventricular hypertrophy be interpreted by a senior doctor.

• Normal variant Targeted history of onset of chest pain, associated

• Osborn wave of hypothermia symptoms and risk factors.

• Prinzmetal’s angina Targeted examination to exclude potential differential


diagnosis and complications of acute myocardial ischemia.
• Ventricular paced rhythm
IV cannula will be set, and blood will be sent for full blood
In general, the difference between unstable angina and NSTEMI/ count, urea and electrolytes, cardiac enzymes (CKMB,
STEMI is an absence of cardiac enzymes abnormalities. troponin I or T).

Cardiac enzymes (CKMB, Troponin T or I) are highly sensitive to Radiological examination of the chest will be required only
cardiac muscle injury. Another lab investigation is full blood to diagnosis acute pulmonary edema, rule out possible
count, urea, and electrolyte. A chest x-ray may give clues to differentials like pneumothorax or aortic dissection.
211
Antiplatelet typically aspirin 300 mg and either Ticagrelor
Table 4.3 Advantages And Disadvantages Of Thrombolysis
180 mg or Prasugrel 60 mg. Versus PCI
GTN sublingually or spray to relieve chest pain, if chest pain
THROMBOLYSIS PCI
persists after two sublingual GTN tablet, proceed with GTN
infusion especially if the patient has concomitant Advantages Rapid administration Better clinical efficacy i.e.
Widely available superior vessel patency,
hypertension or heart failure. Convenient TIMI grade 3 flow rates
and reduced occlusion
IV morphine with anti-emetic if chest pain persisted and rates
Less haemorrhage
titrated according to the response of the patient. Early definition of
coronary anatomy allows
tailored therapy and
Oral beta blocker if no contraindication within 24 hours. more efficient risk
stratification
Definitive treatment depends on which condition within the
acute coronary syndrome is diagnosed. Disadvantages Patency ceiling, i.e. Delay limits efficacy
infarct-related artery is Less widely available
restored in only 60-85% Requires expertise
STEMI should undergo reperfusion therapy preferably of patients, with a normal
TIMI grade 3 epicardial
percutaneous coronary intervention (PCI) or intravenous coronary flow in only
thrombolytic therapy 45-60% of patients

Less clinical efficacy, i.e.


Unstable angina and NSTEMI: medical therapy of optimal reperfusion is not
achieved in more than
antiplatelet, anticoagulant and beta blocker followed by 50% of patients, and re-
occlusion of infarct
admission to hospital and arranging of urgent PCI vessel occurs in 5-15%
of patients at week 1 and
For STEMI, time is muscle, the sooner the PCI or 20-30% within 3 months

thrombolysis, the better prognosis is. Risk of haemorrhage

212
In the absence of PCI, thrombolysis is alternative, and the In the acute management of acute coronary syndromes, please
following should present for the patient to be a candidate of take note of the following:
thrombolysis:
If the patient is in shock, always look for precipitating causes:
• Typical chest pain of AMI
1. Do a gentle rectal examination to look for gastrointestinal
• ST-segment elevation fulfilling the criteria stated above bleeding.

• Chest pain <12 hours from onset 2. Is the patient bradycardic? Treat according to ACLS guidelines.

• Patients <75 years of age 3. Is the patient tachycardic? Treat according to ACLS guidelines.

Table 4.4 Contraindications Of Intravenous Thrombolysis 4. Does the patient have a right ventricular infarct?

a. Do right-sided leads in the presence of ST elevation in II


ABSOLUTE RELATIVE and III and aVF as in inferior AMI (Look for at least 1 mm
ST elevation in V4R, V5R, and V6R.

History of intracranial haemorrhage Severe hypertension (blood pressure >


b. If so, give the fluid challenge of 100-200 ml normal saline
History of ischaemic stroke in the past 180/ 110 mm Hg)
3 months (except acute ischaemic History of ischaemic stroke >3 months over 5 to 10 minutes and assess response.
stroke within 3 hours) Presence of dementia
Presence of cerebral vascular Known intracranial disease that is not
malformation or intracranial an absolute contraindication c. This can be repeated if the patient does not become
malignancy Traumatic or prolonged
Suspected aortic dissection cardiopulmonary resuscitation (CPR)
breathless and there are no clinical signs of pulmonary
Bleeding diathesis or active bleeding that lasted more than 10 minutes edema.
(except menses) Major surgery within 3 weeks
Significant head trauma or facial Presence of active peptic ulcer
trauma in the past 3 months Internal bleeding within the last 2 to 4 d. Start inotropes (IV dobutamine/dopamine 5-20 µg/kg/min)
weeks
Non-compressible vascular punctures. if the blood pressure remains low despite IV fluid
Pregnancy administration of 500 ml.
On warfarin therapy
For streptokinase, prior exposure
(more than 5 days ago) or history of
allergic reaction

213
5. Is the patient in cardiogenic shock because of mechanical
complications, e.g., papillary muscle dysfunction or rupture,
septal rupture or cardiac tamponade from free wall rupture?

a. Call the cardiologist and cardiothoracic surgeon.

b. Meanwhile, start inotropic support, e.g. IV dobutamine/


dopamine 5-20 µg/kg/min

c. Catheterize the patient to measure the urine output.

d. Admit to the coronary care unit or send the patient to the


cath lab.

References and Further Reading, click here

214
Section 3

Acute Heart Failure (AHF)

Case Presentation
by Walid Hammad An ambulance crew rushes into your emergency department
(ED) with a 56-year-old man. He is severely short of breath,
sitting upright on the stretcher, using his accessory respiratory
muscles, and gasping for air. You find that he is diaphoretic,
tachypneic, and in severe respiratory distress. You ask him,
“What’s going on?” He replies: “I…can’t…(pauses and inhales
a shallow breath)…breathe!”

The paramedics inform you that they received a call from the
patient’s wife about 6:30 that morning, saying that her
husband was short of breath and sweaty and that he had
vomited once. The wife told them that she and her husband
had returned from a long trip the night before and that her
husband had not taken his “water pills” because he did not
want to stop for frequent restrooms breaks during their drive.
Audio is available here
When they got home, he still did not take his pills because he

215
wanted to sleep through the night. His breathing Critical Bedside A c t i o n s a n d G e n e r a l
problems woke him during the night, and he tried Approach
The first step in managing such a patient, as for most ED
to get more comfortable by adding pillows under patients, is measuring vital signs. This information will help you
his head to the point that he was almost sitting identify a part of the pathology. For example, if the patient is
up in bed. hypertensive, he could be in acute heart failure; on the other
hand, if the patient is hypotensive, he could be in shock. Similarly,
You thank the paramedics and turn back to the if the patient is tachycardic, his symptoms could be caused by
the very fast heart rate; conversely, if he is bradycardic, he could
patient, who now looks even worse. He is more
have symptomatic bradycardia. The vital signs will guide your
short of breath, and you sense that he is getting treatment options. When you examine the patient described in
tired, about to give up. He looks like he is about this case report, you find his blood pressure (BP) to be 265/145

to collapse. What is your next step? mm Hg, his heart rate (HR) to be 138 beats/min, his respiratory
rate at almost 40 breaths/min, and his pulse oximetry reading is
92% on 4 liters of oxygen delivered by nasal cannula. (Note: A
patient who is severely short of breath might be breathing through
his or her mouth, so a nasal cannula may not be of great benefit;
in these patients, the use of a face mask might be prudent.)

This patient’s clinical presentation and vital signs represent a


clinical pattern of acute heart failure (AHF) with severe acute
pulmonary edema (APE) secondary to acute left ventricular failure
(LVF). The LVF and subsequent pulmonary edema are secondary
to any or a combination of (1) an increased preload, (2) a
decreased left ventricular ejection fraction, or (3) an extremely
elevated blood pressure.

216
On the other hand, a patient who presents with the same clinical by the increased systemic vascular resistance (SVR) that is
picture but with a low BP instead of a high one could have APE or generating an extremely high BP in the aorta. With such elevated
acute LVF secondary to cardiogenic shock. In this scenario, your BP, the left ventricle is unable to offload the suitable amount of
treatment choices will change, and your strategy will be directly blood with each stroke, i.e., stroke volume (SV) is decreased,
opposite that for a patient with elevated BP. It is crucial to make leading to a decrease in cardiac output (CO). With time, if the
this distinction early because the administration of vasodilators in preload does not decrease and the afterload continues to
high doses to a patient in cardiogenic shock could have a increase, the blood will back up behind the left ventricles (in the
devastating outcome. lungs), causing the lungs to become engorged with accumulated
blood and thus increasing transudation of protein-poor fluid into
The second step in management is the clinical examination. In a
the interstitial space and the alveoli. This cascade compromises
patient with AHF with consequential severe APE, the clinical
the air exchange mechanisms, causing the patient to manifest
picture might resemble severe Stage D congestive heart failure
signs and symptoms of respiratory failure, presenting clinically as
(CHF), but with a swifter, more acute onset. Generally, the patient
tachypnea and hypoxia.
is in moderate to severe distress, is uncomfortable, and is usually
diaphoretic, with jugular venous distention (JVD) and bilateral The First 5 to 10 Minutes
rales on lung examination. Depending on how long the patient The main pathology in AHF is the extremely high SVR in the heart,
has been in severe CHF, the rales might be basal initially but then so the treatment modalities should focus on decreasing the
heard in all lung fields, up to the apices, in late stages or even resistance, i.e., decreasing the blood pressure (afterload) or
audible without a stethoscope. decreasing preload. The mean arterial pressure (MAP) can be
used as a treatment guideline. It is determined by the cardiac
Hepatic engorgement, a positive abdominojugular test, and
output (CO) and SVR: MAP=CO x SVR. Note that about 50% of
bilateral lower extremity edema are signs of chronic CHF and
patients presenting with APE are euvolemic rather than
might not have developed yet in a patient with acute left heart
hypervolemic and that the treatment options should focus on
failure. These signs start to manifest when the pulmonary arterial
volume redistribution rather than volume removal.
wedge pressure increases. Think about the pathology: the heart
(the pump or engine), specifically the left ventricle, is unable to Since you do not want to decrease CO in a patient who is barely
pump the blood against the overwhelming resistance generated perfusing because of the elevated BP, your best bet is to work on

217
decreasing preload and/or SVR and thus decrease the MAP. This • Myocardial Infarction
can be achieved by several means. However, in this scenario, in
• Nephrotic Syndrome
which the patient is extremely ill and needs the MAP to be
dropped quickly, we head directly to the rapid-onset options— • Neurogenic Pulmonary Edema
nitro derivatives (fast-acting nitroglycerin derivatives) and BIPAP
or CPAP. • Pneumothorax

• Pulmonary Embolism
Differential Diagnoses
• Acute Kidney Injury • Respiratory Failure

• Acute Respiratory Distress Syndrome • Venous Insufficiency

• Bacterial Pneumonia • Viral Pneumonia

• Cardiogenic Pulmonary Edema (secondary to cardiogenic • Others


shock)
History and Physical Examination Hints
• Chronic Obstructive Pulmonary Disease (COPD)
• JVD might be present, indicating increased central venous
• Cirrhosis of the liver pressure (CVP) resulting from pulmonary edema and increased
right ventricular pressures.
• Community-Acquired Pneumonia
• An extremely elevated BP in a previously relatively healthy
• Emphysema patient presenting with APE points to acute heart failure.

• Goodpasture Syndrome • Bilateral pulmonary rales are typical of APE but not specific.
Rales on only one side could suggest other causes such as
• Idiopathic Pulmonary Fibrosis
pneumonia or emphysema (dry crackles).
• Interstitial (nonidiopathic) Pulmonary Fibrosis

218
• APE can also present as bilateral wheezing (cardiac asthma); output (CO=HR X SV). The ECG could also reveal a left
however, this presentation should not be confused with the ventricular strain pattern.
wheezing associated with pure reactive airway disease or
3. With proper training, emergency physicians can reliably obtain
asthma.
the following information with an ultrasound examination
• If the patient presents with altered mental status or has an performed at the bedside:
abnormal neurologic exam, the APE might be neurogenic
• Determine left ventricular ejection fraction as a broad
pulmonary edema.
categorization (normal, moderately reduced, severely
• If the patient is receiving dialysis or has nephrotic syndrome, reduced) – link
cirrhosis, or other causes of volume overload, the backbone of
• Check for pulmonary congestion/edema – link
treatment will be diuresis rather than redistribution (preload or
afterload reduction). • Evaluate volume status by examining the inferior vena
cava – link
• If lung sounds are unequal, the patient might have a
spontaneous pneumothorax. If the patient is as sick as the one
Laboratory Tests
in our scenario, he or she could be experiencing cardiac 1. Complete Blood Count (CBC): May show an elevated white
tamponade (pay attention to the position of the trachea). blood cell (WBC) count, which may indicate an infectious
cause rather than a cardiac cause. However, stress in itself can
Emergency Diagnostic Tests and induce hypoxia and shortness of breath, which can cause
Interpretation margination of WBCs, leading to an elevated WBC count. A
Bedside Tests patient with severe chronic obstructive pulmonary disease
1. By placing the head of the stretcher at a 45-degree angle, you (COPD) or Goodpasture syndrome is likely to be taking
should be able to assess the patient for JVD. corticosteroids, which can raise the WBC count.

2. An electrocardiogram (ECG) might show sinus tachycardia, 2. Comprehensive Metabolic Panel (CMP): May indicate renal
atrial fibrillation, or another arrhythmia (tachycardia or failure if the SVR is so high that it is causing severe spasm in
bradycardia), suggesting the reason for a decrease in cardiac the renal artery and thus impairing kidney function, especially if

219
the patient has other comorbidities that predispose him/her to
Image 4.5
kidney injury. The patient might be alkalotic in response to
tachypnea, which presents as low CO2. He may also present
with acidosis due to elevated lactate levels resulting from
tissue hypoxia. Liver function test results could be elevated,
especially if the patient has long-standing CHF that is causing
hepatic engorgement.

3. Pro-BNP: The b-type natriuretic protein (BNP) concentration


will be elevated in a patient with acute heart failure, but usually
the values are not very high. Be careful: pro-BNP could also be
elevated in patients with large pulmonary embolisms that are
causing right ventricular strain. Cardiac enzymes might be
slightly elevated due to the myocardial strain; significantly
elevated numbers warrant consideration of Acute MI.

Imaging Modalities
A chest x-ray (obviously a portable frontal view in our patient)
would show pulmonary congestion, with cephalization of the Emergency Treatment Options
pulmonary vessels, Kerley B lines, peribronchial cuffing with air Initial Stabilization
bronchograms, a “bat wing” pattern, and possibly, though not 1. If the patient is lying down flat, move him/her into a sitting
always, an increase in cardiac shadow size. Note that not all position, which should lessen the pooling of blood in the lungs
these findings may be present on the chest x-ray. and allow the utilization of the superior lung fields for aeration
and gas exchange.

2. Place the patient on supplemental oxygen.

3. Obtain intravenous access quickly.

220
Non-invasive positive pressure ventilation (NIPPV) • N i t r o g l y c e r i n might cause hypotension, which is
usually temporary and associated with overall clinical
NIPPV for cardiogenic pulmonary edema is an effective and safe
improvement. However, the persistence of the
modality in adult patients with APE. There is a potential benefit of
hypotension after the nitroglycerin is stopped might
NIPPV in reducing mortality.
indicate a right ventricular malfunction, e.g., right
ventricular MI or volume depletion.
Medications
• Nitro derivatives: Nitrogen inhibits the motor function of the • Intravenous nitroglycerin can be administered, starting
smooth muscles in the systemic vasculature, leading to as a drip at 0.5‒0.7 mcg/kg/min and then increased by
vasodilation and a decrease in SVR. 10‒20 mcg/min q3-5 minutes up to 200 mcg/min. The
BP must be monitored closely during administration.
• Nitroglycerin: Reduces preload.

• Nitroglycerin has been found to be safe for use in • Transdermal nitroglycerin (1‒5 cm) can be applied to the
patients with acute heart failure and improves short- chest wall.

term outcomes in EDs and ED-like settings.


• Pediatric dosing: 0.25‒5 mcg/kg/min IV, to a maximum
dose of 20 mcg/kg/min
• Nitroglycerin is a rapid-onset, short-acting smooth
muscle relaxant that reduces preload through venous
• Safety profile: Class C in pregnancy, Safety during
dilation and, in high doses, reduces afterload through lactation is unknown
arteriolar dilation.
• Nitroprusside: Reduces afterload.
• Sublingual tablets dissolve under the tongue and are
manufactured in 400-mcg minitablets. After you have • Is a more potent arteriolar vasodilator than nitroglycerin
confirmed the diagnosis by exam and the monitor is
• Poses the potential for thiocyanate toxicity
indicating a high blood pressure, place one, two, or
even more tablets under the patient’s tongue (up to one • If further afterload reduction is needed after
tablet per minute). administration of nitroglycerin, a nitroprusside drip can
be started.

221
• Dosage: 0.3–10 mcg/kg/min doses, is more helpful than the oral route; diuresis will begin
within 15 to 20 minutes after administration.
• Pediatric dosing: 0.25‒4 mcg/kg/min to a maximum
dose of 10 mcg/kg/min • The dose in emergency circumstances can be up to 2.5
times the patient’s regular dose. If the patient is loop diuretic
• Safety profile: Class C in pregnancy. Possibly unsafe
naïve; give 40 mg IV of furosemide or the equivalent of
during lactation
another loop diuretic.
• Nesiritide:
• Check the safety profile for each individual medication that is
• Has not been found to increase or decrease the rate of considered or administered.
hospitalization or death. It might have a negative effect,
• Inotropics:
causing hypotension. Its use in acute heart failure is still
being investigated. • Severe left ventricular dysfunction or acute valvular problems
may cause hypotension in some patients. Therefore, using
• Dosing: 0.01 mcg/kg/min IV to a maximum of 0.03 mcg/
the above agents can be harmful to these patients.
kg/min
Therefore, inotropic medications such as dopamine and
• Safety profile: Class C in pregnancy. Safety during dobutamine can be inevitable to preserve normal blood
lactation is unknown pressure.

• Loop diuretics: Reduces preload. Procedure


• Loop diuretics may be the first line of treatment for chronic Bedside ultrasound
heart failure. However, Cotter and colleagues found that for Please read RUSH protocol chapter and watch its’ videos to learn
patients with acute heart failure accompanied by severe more about US evaluation of pulmonary edema.
pulmonary edema and respiratory distress, high-dose nitro
derivatives combined with a small dose of furosemide were Disposition Decisions
more effective than high-dose furosemide with a small dose • All patients who present to an ED with acute heart failure and
severe pulmonary edema should be admitted to either an
of nitro derivatives. IV dosing of diuretics, especially in large

222
intensive care unit or an intermediate care unit based on the
institution’s ability to handle the necessary therapeutic
modalities, especially drips and noninvasive positive-pressure
ventilation (NIPPV).

• Some patients might be stable enough on a telemetry floor/


ward, especially patients who had resolved symptoms and
findings.

• Risk-stratifying patients with acute failure in the ED is difficult


and requires additional investigation.

References and Further Reading, click here

223
Section 4

Aortic Dissection

Case Presentation
by Shanaz Sajeed A 56-year-old male presented to the emergency department
with sudden onset of severe tearing chest pain radiating to the
back. He had a history of hypertension and hyperlipidemia. He
was a smoker. Upon arrival, he appeared to be diaphoretic
and in severe pain. He denied any prior history of chest pain.
He had been without any infective symptoms lately. He was
compliant with his medications, namely, amlodipine and
simvastatin. At triage, his blood pressure was noted to be
80/60 mmHg with a pulse rate of 130 bpm. His oxygen
saturation was 95% on room air, and his respiratory rate was
22 breaths per minute. On examination, he had muffled heart
sounds, jugular venous distention, and radio-radial pulse
delay.

Audio is available here

224
Introduction Illustration 4.1 De Bakey and Stanford classification of
Aortic dissection carries high morbidity and mortality. Although aortic dissection.
patients generally present with acute symptoms and classic
signs, a subset of patients may present with syncope, GI
bleeding, and neurological deficits. The clinician needs to remain
vigilant for such atypical presentations. There are two standard
anatomical classifications – Stanford and De Bakey. Stanford type
A dissections (De Bakey I and II) involve the ascending aorta.
Stanford type B (De Bakey III) dissections arise distal to the left
subclavian artery (Illustration 4.1). Stanford A dissections are
more common than Stanford B dissections (62% vs. 38%). The
aortic wall consists of 3 layers – the intima, media and an outer
layer known as the adventitia. Classic nontraumatic aortic
dissection is usually due to a tear in the intimal layer of the aorta,
leading to an intimal flap. The bridge between the media layer and
the aortic lumen causes a subintimal hematoma. Then intima gets
separated from the underlying media and adventitia. This false
lumen of varying size may result in complete occlusion of major
arteries that branch from the aorta leading to major ischemic
complications such as limb ischemia, paralysis, stroke, renal
failure as well as cardiac events. Critical Bedside Actions and General
Approach
The initial management of a patient with chest pain and
hypotension warrants observation in a monitored area with
continuous SpO2 and cardiac documentation. The physician
needs to assess airway, breathing, and circulation. A compromise
in any of these necessitates immediate action. In this case
225
scenario, for example, the next priority History Taking and Physical Other symptoms include dyspnea,
would be to determine the etiology of this Examination Hints dysphagia, focal weakness and altered
patient’s hypotension and to initiate mental status. Type B dissections present
resuscitative and stabilizing measures. History Taking Hints s i m i l a r l y. O c c l u s i o n o f t h e m a i n
The physician should As initial resuscitation is going on, a abdominal aortic branches may lead to
focused history should be obtained at the mesenteric and solid organ ischemia.
• Administer supplemental oxygen and bedside. Inquire about the site, onset, Patients may present with gastrointestinal
assess the patient’s airway, breathing, nature, duration, radiation as well as bleeding, oliguria or anuria. Therefore,
and circulation to determine the need aggravating and relieving factors of the asking about gastrointestinal symptoms
for any immediate critical interventions. chest pain. Patients with acute Type A is an essential part of the history. Risk
dissection classically present with : factors for aortic dissection should be
• Secure venous access by inserting two
large bore IV cannulas into the assessed. These are;
• Sudden onset of ‘ripping’ or ‘tearing’
antecubital fossa. chest pain (85%) and/or interscapular • An aortic aneurysm
back pain (46%)
• In the hypotensive patient, administer
• Atherosclerosis
an initial IV fluid bolus of 20 ml/kg • Pain is usually maximal at onset, unlike
crystalloid. MI where pain usually gradually • Chronic Hypertension
increases in intensity.
Note: A small initial bolus of fluid would • Coarctation of the aorta
not be harmful. Even in cardiogenic • Pain may migrate distally to the
• Congenital aortic valvular defects (e.g.,
shock, it is likely to improve cardiac abdomen as dissection progresses
bicuspid aortic valve)
output and blood pressure transiently.
Thus, one should not be hesitant to give • However, a minority of patients may
• High-intensity weight lifting
an initial bolus of fluid in the hypotensive present atypically with abdominal pain

patient even if the etiology of shock is not (22%), syncope (13%) and stroke (6%). • Increasing Age
immediately apparent.
• Patients often describe the pain as • Infection leading to aortitis
knife-like.

226
• I n fla m m a t o r y p ro c e s s e s d u e t o • Vasculitis facilitate diagnosis as soon as
vasculitis. e.g., Takayasu’s aortitis possible. Assess for pulse deficits. Pulse
The physician should also assess the
deficits are diminished or absent pulses
• Inherited connective tissue disease patient’s anticoagulants use for increased
caused by compression of the true lumen
(e.g., Marfan’s, Ehlers Danlos Syndrome bleeding risk, such as Warfarin/Novel Oral
by the false lumen. Blood pressure
Type IV, familial forms of a thoracic Anticoagulants or antiplatelet agents,
difference between the left and right arm
aneurysm and dissection) which would increase their bleeding risk.
suggests aortic dissection. BP difference
It is also vital to elicit any specific
• Male gender >20 mmHg between the two limbs is
allergies that may affect therapy or
significant. Data analysis from the
• Substance abuse such as cocaine, prohibit the use of IV contrast for imaging
International Registry of Acute Aortic
methamphetamine, MDMA purposes. Elicit any relevant social history
Dissection (IRAD) revealed that fewer
including questions about illicit drug use.
than 20% of patients with proven acute
Past medical history findings that should Note: Cocaine use is associated with
aortic dissection had reported pulse
prompt consideration of aortic dissection acute dissections.
d e fic i t s . T h e c a r d i o p u l m o n a r y
include:
examination should focus on signs of
Physical Examination Hints
• A family history of aortic dissection, As initial stabilizatio n and initial cardiac ischemia, aortic insufficiency,
aneurysm, or sudden death resuscitation take place, a focused cardiac tamponade and cardiogenic
clinical examination should shock. When listening to the heart
• Chronic hypertension (most common
simultaneously be performed. The sounds, listen for:
predisposing factor)
physician should assess for signs of
• Distant/muffled heart sounds suggest
• Documented aortic pathology shock (e.g., cold extremities, delayed
pericardial effusion.
capillary refill, weak, thready pulse.) and
• History of tuberculosis or syphilis
aim to determine the etiology of • Gallop rhythm, S3, S4 suggestive of
• Known connective tissue disorder hypotension if present. The physician heart failure.
should always consider bedside
• Previous cardiac surgery (especially ultrasonography (RUSH protocol) to
valve repair) or vascular surgery
227
• Diastolic murmur indicating aortic insufficiency. It is seen up to • Acute coronary syndrome
75% of Type A dissections.
• Pericarditis/Myocarditis
In the setting of the hypotensive patient, pulsus paradoxus and
• Pulmonary embolism
distended neck veins suggest cardiac tamponade. Examine for
pulmonary findings of: • Pneumonia/Pleural effusion

• Rales or Ronchi suggestive of heart failure • Pneumothorax

• Tachypnea and retractions indicating respiratory distress • Rib fractures

Examine for signs of stroke or paraplegias/paresthesias • Esophageal rupture


suggestive of spinal cord infarcts due to occlusion of the spinal
arteries. Examine the abdomen for pulsatile expansile masses • Mediastinitis
suggestive of aneurysmal dilation. Signs of peritonitis may
suggest mesenteric ischemia, a known complication of an aortic
Emergency Diagnostic Tests and
dissection.
Interpretations
Bedside Tests
Differential Diagnosis • Electrocardiogram(ECG) – may be normal, show nonspecific ST
Aortic dissection should be considered in all patients with chest
changes, or changes suggestive acute coronary syndrome. The
pain. The typical pain in acute thoracic aortic dissection is abrupt
most common coronary artery involved is the right coronary
onset, severe and with radiation to the back. However, there is
artery, leading to an inferior STEMI.
significant overlap between patients with myocardial infarction,
and the two can be difficult to distinguish. A patient with aortic • Bedside transthoracic echocardiography may yield useful
dissection may experience ischemic pain due to the involvement information such as the presence of pericardial fluid suggestive
of the coronary arteries. There is a wide range of differential of tamponade. It can also give information on cardiac
diagnosis in a patient with chest pain. Potentially life-threatening contractility. A dilated aortic root or dissection flap may be
causes of chest pain include: visualized on the parasternal long axis view.

228
• Chest X-ray – Abnormalities suggestive of dissection are
Image 4.6 Chest x-ray showing aortic dissection findings.
present between 60-90% of cases.

These are;

• Depression of the left mainstem bronchus

• Displaced intimal calcification

• Indistinct or irregular aortic contour

• Left apical pleural cap

• Opacification of the “AP window” (i.e., clear space between the


aorta and the pulmonary artery)

• Pleural effusion (left > right)

• Tracheal or esophageal deviation

• Widened aortic knob or mediastinum (present in only 63% and


56% of patients with type A and type B dissections, Laboratory Tests
respectively). Blood investigations should include a full blood count, urea and
electrolytes, coagulation, cardiac enzymes and crossmatch.

Imaging Modalities
• Contrast-enhanced CT aortogram (Figure 2 and Figure 3) is
usually the investigation of choice. In cases where CT poses a
significant risk (e.g., pregnancy), MR Angiography of the aorta
can be done.

229
• Transesophageal echocardiogram (TEE) can be done at the
Image 4.8 CT scan - Stanford Type B dissection
bedside where there is a risk of contrast-induced nephropathy
(Patients with impaired renal function) or contrast allergy, or in
unstable patients.

Image 4.7 CT scan - Stanford Type A dissection

Type B aortic dissection with dissection flap in descending aorta. (Case


courtesy of Dr. Avni K P Skandhan, Radiopaedia.org, rID: 25409)

Emergency Treatment Options


Investigations and diagnostic workup should be done in parallel
with the resuscitation of the patient. After initial assessment and
stabilization and the definitive diagnosis made, reassess the
patient and determine further management. Patients with aortic
Type A dissection involving the ascending aorta demonstrating dissection dissection may be hypotensive, normotensive or hypertensive.
flap as well as the true and false lumen. (Case courtesy of Dr. Frank Gaillard,
Radiopaedia.org, rID: 8886)
Hypotensive Patient
The priority is to maintain organ perfusion until definitive
management:

230
Administer IV crystalloid bolus of 20 ml/kg blockers or Calcium Channel Blocker therapy. Avoid using
as sole therapy as it can cause reflex tachycardia.
Consider vasopressors (if needed) to maintain a MAP:
70-80 mmHg Practical Point: Hypotensive and drowsy patients need secured
airway or intubation before any advanced imaging. Induction
If pericardial tamponade is present, emergent
agents with cardiovascular stability are advised. Push dose
pericardiocentesis is indicated
vasopressors should be available in case of a precipitous drop in
Blood transfusion is indicated if the hypotension is due to blood pressure. Investigations and diagnostic workup should be
internal bleeding done in parallel with the resuscitation of the patient. Emergent
surgical/interventional consult should be sought for definitive
Hypertensive Patient management.
Aggressive blood pressure control is essential to reduce shear
stress: • Type A dissections are usually managed surgically. The principal
objectives are 1) relieve the symptoms, 2) reduce the
Target a systolic pressure between 100-120 mmHg or complications, and 3) prevent aortic rupture and death. The
MAP 70-80 mmHg. Heart rate should be between 60-80. affected layers of the aorta are sutured together, and the aorta
is reinforced with a graft. Endovascular therapy is now
IV ß-blockers are the first-line therapy.
becoming increasingly popular.
• Labetalol: 20 mg IV slow injection, then 40-80 mg IV
• Type B dissections are usually managed medically with
q10 min PRN, up to 300 mg IV total. An infusion of 0.5
aggressive blood pressure and pain control as well as continual
to 2.0 mg/min can also be run.
monitoring for signs and symptoms of complications. Surgical
Calcium channel blockers such as diltiazem or verapamil management indications include:
may be used in patients with contraindication to ß-
• Signs of bowel ischemia, limb ischemia or solid organ
blockers
ischemia
Sodium Nitroprusside may be added as adjunctive
• Persistent pain
therapy for elevated blood pressure refractory to ß-

231
• Expanding hematoma or impending rupture

• Associated aneurysmal dilatation of the aorta

Disposition Decision
Patients with acute aortic dissection should be managed in a high
dependency or intensive care unit. The overall in-hospital
mortality of aortic dissection is 27%. 30-day mortality of type A
dissection with and without surgery is 26% and 58%,
respectively. Type B dissection treated medically has 11% and
surgically has 31% 30-day mortality.

References and Further Reading, click here

232
Section 5

Deep Vein Thrombosis (DVT)

Case Presentation
by Elif Dilek Cakal An 85-year-old woman, with a history of congestive heart
failure, presented with right leg pain and swelling of 2 days’
duration. She had been hospitalized for pneumonia one week
earlier. Her vitals on arrival were: Blood Pressure: 138/84
mmHg, Pulse Rate: 65 beats per minute, Respiratory Rate: 14
breaths per minute, Body Temperature: 37°C (98.6°F), Oxygen
Saturation: 96%. On examination, her right calf was reddish,
tender, edematous and 4 cm greater in circumference than the
left when measured 10 cm below the tibial tuberosity. Her
Wells’ Score for deep vein thrombosis (DVT) was 4 and
suggested high-risk for DVT. Compression ultrasonography
showed a thrombus in the popliteal vein. Enoxaparin (1 mg/kg,
twice a day, SC) was started. No signs and symptoms of
pulmonary embolism were observed. The patient was referred
Audio is available here to a cardiovascular surgeon as an outpatient after discussion

233
and confirmed understanding of discharge Introduction
The annual incidence of DVT is 92 cases per 100000 persons.
instructions.
The rate steadily advances with increased age (32/100000 if age
< 55 years, 282/100000 if age 65-74 years, 555/100000 if age
>74). While 90% of DVT occurs in lower extremities, 10% of DVT
occurs in upper extremities. Up to more than 40% of patients
with lower extremity DVT have concomitant pulmonary embolism
(PE), whether they may have related complaints or not.

Critical Bedside Actions and General


Approach
DVT is mostly a relatively benign disease; nevertheless, it may
cause severe symptoms and limb- or life-threatening
presentations. Emergency physician (EP) must check for signs of
adverse outcome. Therefore patients should be evaluated for
airway, breathing, circulation sequence and EPs try to understand
possible immediate life-threatening problems. Concentrating on
the patient focal complaint should be followed after the initial
evaluation. Check vitals for instability and fever. Check for arterial
pulses and signs of acute arterial thrombosis immediately in the
case of every limp pain. Also, an extremely or entirely swollen
limb indicates total or near total obstruction at a more proximal
level. Increased compartment pressure may potentially disrupt
the arterial flow. Diagnosing DVT in the emergency department
(ED) is crucial. A timely started treatment may prevent the
subsequent pulmonary embolism (PE) and chronic morbidities
like chronic venous stasis and recurrent clots.
234
Some patients may ignore PE-related mild symptoms, or they
may give priority to DVT-related ones. EP must concentrate on Table 4.5 Differential Diagnoses of DVT
subtle PE-related sign and symptoms. In the spectrum of DVT,
phlegmasia alba dolens, phlegmasia cerulea dolens and venous UNILATERAL LEG SWELLING BILATERAL LEG SWELLING
AND/OR PAIN AND/OR PAIN
gangrene are vascular emergencies. They should be managed
surgically, by endovascular interventions or thrombolytic
Abscess/Necrotizing Fasciitis Bilateral DVT
treatment, in a time-sensitive manner. Upper extremity DVT has Arterial insufficiency Congestive heart Failure
its own risk factors and consequences. It should be managed in Baker cyst rupture/inflammation Inferior vena cava compression
Cellulitis Liver Failure
its own context. Claudication Renal Failure
Compartment Syndrome
DVT
Differential Diagnosis Erythema Nodosum
Table 4.5 summarizes differential diagnoses of DVT. Unilateral and Hematoma
bilateral leg swelling and pain are two categories in order to Lymphangitis
Lymphedema
differentiate the various causes.  Bilateral leg swelling is more Musculoskeletal trauma
likely a clue for congestive heart failure, liver or renal failure, Myositis
Polyarteritis Nodosa
inferior vena cava compression than the bilateral DVT. However, Postphlebitic Syndrome
patients symptoms and findings should be considered for ruling Superficial thrombophlebitis
Tendinitis/Achilles tendinitis
out these causes. Varicose Veins
Please read Courtney DM. Venous Thrombosis. In: Adams JG, Barton ED,
Collings JL, DeBlieux PMC, Gisondi MS, Nadel ES, editors. Emergency
Medicine: Clinical Essentials. Philedephia: Elsevier; 2013:611-617. and Ferri
FF. Deep Vein Thrombosis. In: Ferri FF, et al, editors. Ferri’s Clinical Advisor
2015: 5 Books In 1. Philedephia: Elsevier; 2015:348-350. for more
information.

235
History and Physical ankle. It is insensitive and • Prothrombin mutation
Examination Hints nonspecific, therefore, useless.
• Hyperhomocysteinemia
• Neither medical history nor physical
• Because only history and
examination is specific to DVT. Clinical • Deficient levels of clotting factors
examination are indeterminate, risk
presentation may range from nearly
factors for DVT are essential to • Congestive heart failure
asymptomatic to severely symptomatic
predict clinical probability. Known
or limb- or life-threatening. • Chronic obstructive pulmonary
risk factors for DVT are as follows:
disease
• As a general rule, unilateral limb pain
• Previous history of PE or DVT
and swelling imply DVT. • Air travel
• Recent Trauma or surgery
• Lower extremity DVT • Obesity
• Cancer
• Unilateral leg pain and swelling are • Phlegmasia alba dolens and
indicators of lower extremity DVT. • Central or long-term vascular phlegmasia cerulea dolens are vascular
Some patients may define fullness or catheter surgical emergencies. The features of
cramping in the posterior aspect of these conditions are summarized in
• Age
t h e l o w e r e x t re m i t y. B i l a t e r a l Table 4.6.
symptoms are more likely in the • Oral contraceptives
course of other diseases. However,
simultaneous bilateral DVT or • Hormone replacement therapy

obstruction of the inferior vena cava


• Pregnancy
may cause bilateral symptoms.
• Immobility
• Edema, redness, and tenderness are
possible signs. None is specific. • Factor V Leiden mutation
Homans sign refers to calf pain
• Antiphospholipid antibody
elicited by passive dorsiflexion of the
syndrome
236
• Upper extremity DVT:
Table 4.6 Surgical Emergencies Secondary To DVT
• Upper extremity DVT is infrequent and accounts for
PHLEGMASIA ALBA PHLEGMASIA
DOLENS CERULEA DOLENS approximately 10% of all DVTs. Its prevalence is increasing
due to indwelling central catheters.
Appearance Pale, cool, edematous Cyanosed, edematous,
(An example is available purple ecchymosis
at https:// (An example is available • Primary Upper extremity DVT is rare. A well-known form of
www.thrombosisadviser. at http:// primary upper extremity DVT is effort-related thrombosis
com/html/images/library/ circ.ahajournals.org/
vte/deep-vein- content/125/8/1056/ named Paget-Schroetter syndrome. Paget-Schroetter
thrombosis-right-leg- F1.expansion.html)
HR.jpg) syndrome generally occurs in otherwise healthy young men,
Distal pulses Poor/Absent Hard to palpate because after vigorous arm exercise or repetitive overhead activities.
of edema Patients with effort-related upper extremity DVT suggests an
Absent if advanced
underlying venous thoracic outlet syndrome. In the absence
Pain Positive Positive of an obvious risk factor or underlying venous thoracic outlet
syndrome, it is called idiopathic DVT.

Mechanism Massive iliofemoral Arterial flow disruption • Catheter-associated DVT is the predominant secondary
venous thrombosis and due to venous
associated arterial congestion and upper extremity DVT. Indwelling central venous lines, port
spasm increased tissue pressure
systems and pacemaker or defibrillator are leading
Thrombus location In major veins (collaterals In major veins and
are generally spared) collaterals predisposing factors in descending order. Cancer, surgery,
trauma, immobilization, pregnancy, oral contraceptive use

Advances to Phlegmasia cerulea Venous gangrene


and the ovarian hyperstimulation syndrome are the other
dolens predisposing factors for secondary upper extremity DVT.

Treatment: IV Fluid + systemic anticoagulation + catheter-directed thrombolysis/systemic


• Severe upper extremity DVT may result in superior vena cava
thrombolysis/surgical thrombectomy/mechanical thrombectomy. syndrome.
Please read following references for more information (http://lifeinthefastlane.com accessed at
10.05.2016, https://www.thrombosisadviser.com accessed at 10.05.2016, and Mumoli N, Invernizzi
C, Luschi R, Carmignani G, Camaiti A, Cei M. Phlegmasia Cerulea Dolens. Circulation. 2012; 125:
1056-1057.)

237
Emergency Diagnostics Tests and Emergency Treatment Options
Interpretation • The mainstay of medical therapy in ED is anticoagulation.
• Approximately 90% of DVTs occur in lower extremities.
• Possible anticoagulation options are summarized in Table 4.7.
Determination of pretest probability (PTP), D-dimer testing and
bedside compression ultrasound are the milestones of
management in ED.
Table 4.7 Medication For Anticoagulation in DVT
• Wells’ Criteria for DVT (link) stratifies patients according to their
CLASS OF
DVT risk. Scores ≥2 qualify a patient as “High Risk.” DOSE COMMENTS
AGENT

• A diagnostic algorithm is shown here (accessed at 10.05.2016) Unfractionated heparin 80 U/kg IV bolus, then Consider in inpatient therapy
18 U/kg/h IV infusion and in severe renal failure
(Dose adjustment based on APTT)
• D-dimer is useful for its negative predictive value. When
negative, it rules out DVT in the low-risk group. It does not Low Molecular Weight
Heparins
confirm DVT when positive.
Dalteparin 100 IU/kg, twice a day, SC A standard treatment for DVT,
200 IU/kg, once a day, SC preferred in outpatients as a
first line therapy if not
• Many ultrasound protocols for DVT are available. Related contraindicated
ultrasound videocasts can be found here  (accessed at Enoxaparin 1 mg/kg, twice a day, SC A standard treatment for DVT,
1.5 mg/kg, once a day, SC preferred in outpatients as a
10.05.2016) first line therapy if not
contraindicated

• Upper-extremity DVT is diagnosed by Doppler ultrasonography. Tinzaparin 175 IU/kg, once a day, SC A standard treatment for DVT,
preferred in outpatients as a
first line therapy if not
Watch - A tutorial about diagnosing DVT with US. contraindicated

Factor Xa inhibitors
Watch - Normal and Abnormal US findings for DVT Fondaparinux < 50 kg - 5 mg, once a day, SC Do not use in renal failure
50-100 kg - 7.5 mg, once a day, SC
> 100 kg - 10 mg, once a day, SC

Please read http://emedicine.medscape.com (accessed at 10.05.2016) for


more information.

238
The indications for more advanced therapies like catheter- • <2 months: 1.5 mg/kg/ dose SC, twice a day
directed thrombolysis, percutaneous mechanical thrombectomy,
• >2 months: 1.0 mg/kg/dose SC, twice a day
conventional surgery or systemic thrombolysis are as follows:

• Phlegmasia cerulea dolens Geriatric Considerations


DVT management does not alter in the elderly. Frequency and
• Inferior vena cava thrombosis severity of DVT increase. Anticoagulation complications are more
frequent than younger counterparts. Concomitant diseases and
• Subacute and chronic iliofemoral DVT
possible drug interactions complicate the management.
• Acute iliofemoral or femoropopliteal DVT
Pregnant Patients
Though all are useful, endovascular interventions are preferred DVT management does not alter in pregnant. Pregnant women
over more invasive interventions in capable centers so as to are susceptible to DVT. LMWHs are the drug of choice during
minimize the consequent risks. (Bleeding or perioperative pregnancy. All pregnant patients with DVT should be admitted to
complications, etc.) hospital.

The pain medication is advised for patients who are suffering from Patients With Isolated Calf Vein Thrombosis
severe pain. The need for treatment is controversial.

Pediatric, Geriatric, Pregnant Patient and Disposition Decisions


Other Considerations
Admission Criteria
Pediatric Considerations Most patients with DVT can be treated as outpatients. EP can
DVT is infrequent in children and almost always associated with decide the patients that need admission based on four questions
risk factors. Central venous catheter-associated upper extremity (link).
DVT is relatively common in children. LMWH is the mainstay of
1. Does the patient have massive DVT?
the therapy.
2. Does the patient have symptomatic pulmonary embolism?
• Enoxaparin:

239
3. Is the patient at high risk for anticoagulant-related bleeding? • No contact telephone

4. Does the patient have major comorbidity or other factors that • Geographic location (too far from the hospital)
warrant in-hospital care
• Patient/family resistant to outpatient therapy
One or more positive answers should lead EP to admission.
Discharge Criteria
Consider admission if any is present: All patients lacking admission criteria may be treated as
outpatients after a confirmed understanding of discharge
• Suspected or proven concomitant PE
instructions. Several discharge instructions are available online.
• Significant cardiovascular or pulmonary comorbidity
Referral: Patients must be referred to cardiovascular surgeons.
• Iliofemoral DVT
References and Further Reading, click here
• Contraindications to anticoagulation

• Familial or inherited disorder of coagulation

• Familial bleeding disorder

• Pregnancy

• Morbid obesity (>150 kg)

• Renal failure (creatinine >2 mg/dL)

• Unavailable or unable to arrange close follow-up care

• Unable to follow instructions

• Homeless patient

240
Section 6

Hypertensive Emergencies

Case Presentation
by Sadiye Yolcu A 68-year-old man with tearing chest pain presented to the
emergency department. He had a history of coronary artery
disease and hypertension. BP: 220/160 mmHg, HR: 105 bpm,
RR: 20/min, T: 37, SpO2: 96% in room air. In the initial
evaluation, airway and breathing were intact. Diastolic murmur
was heard on cardiac auscultation, and pulses were positive in
all extremities. He has a normal mental state (GCS 15) and no
lateralized motor deficit. A difference in systolic blood
pressure was measured between upper extremities (220/160
vs. 180/140 mmHg). ECG showed nonspecific ST-T changes
and sinus tachycardia.

Audio is available here

241
Introduction urgencies are defined as situations •M a l i g n h y p e r t e n s i o n w i t h o u t
Systemic hypertension is a common requiring actions within 24 hours and yet complication
medical problem. It affects over 1 million do not compromise the risk of developing
• Perioperative hypertension
people worldwide. ER clinicians complications within that period.
commonly encounter this problem. Rapid • Pheochromocytoma,
Hypertensive emergencies include
diagnosis, evaluation, differentiation of
• Sympathomimetic drug use (cocaine,
hypertensive emergencies and • Acute aortic dissection
etc.)
hypertensive urgencies, and appropriate
treatment of these conditions are required • Acute coronary syndrome
Critical Bedside Actions and
to prevent morbidity and mortality.
• Acute heart failure General Approach
The levels above 180 systolic BP and 110 The priority should be given to initial
• Acute renal failure
diastolic BP are considered very stabilization of the patient (C-A-B) as
dangerous which may cause end-organ • Eclampsia other critically ill patients. Depending on
damage such as intracranial bleeding, patients’ symptoms in addition to high
• Hypertensive encephalopathy
aortic dissection, renal failure, etc. Having blood pressure, the cardiac
end-organ damage is the hypertensive • Intracerebral/subarachnoid hemorrhage monitorization, oxygen (if necessary), two
emergency. Having high blood pressure l a rg e b o re I V a c c e s s s h o u l d b e
• Pheochromocytoma, established and blood samples (CBC,
without any signs of end-organ damage
is the hypertensive urgency. Retinal BUN, Cr, coagulation, cardiac markers,
• Sympathomimetic drug use (cocaine
hemorrhage or exudates/papilledema type, and cross-match) sent to the
etc.),
associated with hypertension is defined laboratory. ECG and chest x-ray should
as malignant hypertension. • Stroke be ordered.

Hypertensive emergencies require action Hypertensive urgencies include Lowering BP should be balanced with the
within one hour to abolish the risks of level of BP, patient’s symptoms as well as
• Diastolic tension ≥140 mmHg without
developing complications. Hypertensive harm-benefit situation.
complication
242
Differential Diagnosis Each of these hints was g i v e n i n t h e s p e c i fic d i s e a s e
The most critical step in the differential diagnosis is the definition chapters. Therefore, we advise you to review those chapters too.
of the hypertensive situation (emergency or urgency). Suspicion
of hypertensive emergencies aligns with hypertension and end- Emergency Diagnostic Tests and
organ damage. Depending on patient symptoms and findings, Interpretation
An electrocardiogram (ECG) and chest X-ray should be
hypertensive emergencies differentials include severe problems
performed. ECG may show arrhythmias, nonspecific ST-T
such as intracranial hemorrhage, ischemic stroke, aortic
changes or obvious acute MI findings. The chest x-ray may give
dissection, acute MI, AAA rupture, heart failure, renal failure, limb
hints about aortic dissection, aneurysm, pulmonary edema.
or organ ischemia, etc. In addition to these end-organ damages,
other differentials (seizure, brain tumor, encephalitis, What is your opinion about the chest x-ray (Image 4.9)?
encephalopathy, drug overdose, etc.) should also be considered.
Bedside ultrasonography may help to diagnose some critical
History and Physical Examination Hints pathologies timely. These are pulmonary edema, aortic aneurysm
The previous medical history of the patient (chronic diseases, or dissection, heart failure, and increased intracranial pressure.
antihypertensive drugs usage, previous end-organ compromise,
What is your opinion about the transthoracic ultrasound here?
etc.) should be taken. Chest pain for myocardial infarction, aortic
dissection, dyspnea for pulmonary edema, headache, mental Blood urea nitrogen (BUN), electrolytes, complete blood count
status, seizure for hypertensive encephalopathy should be asked. (CBC), liver-renal function tests, coagulation parameters, cardiac
enzymes and urine analyses should be checked. BUN and Cr
The patients present mostly with ischemic stroke, pulmonary
may show renal impairment. Hematuria and proteinuria in the
edema, hypertensive encephalopathy, or congestive heart failure.
urine should also be checked.
Therefore, history and physical exam should be focused on these
problems during the initial and secondary evaluation. In the Some patients may require further investigations with CT or MRI
physical examination, measure the blood pressure from both depending on their symptoms and findings.
arms and assesses the patient for end-organ compromise
(neurologic-ophthalmologic-cardiac). What is your opinion about the CT (Image 4.10)?

243
Image 4.9
Image 4.10

ignore pain medication because some of them require effective


pain control.
Emergency Treatment Options
Medications in specific problems
Initial Stabilization
Aortic dissection
Support C-A-B and stabilize the patient as needed. Cardiac
monitoring, pulse oximetry, oxygen administration, and IV access The aim is to reducing shearing forces by decreasing the heart
required for all hypertensive emergency cases. Key precaution in rate to 60-80 beats/min, and the systolic pressure to 140 mmHg
the control of hypertensive situations is to maintain the balance of and below, then to 120 whether the patient can tolerate. Organ
the benefits of immediate decreases in BP against the risk of a perfusion should be monitored carefully. Na nitroprusside (0.3-0.5
significant decrease in target organ perfusion. Therefore, IV μg/kg) is a potent agent, and the dose can be arisen by 0.5 μg/
agents are preferred because of their titration option. Do not kg/min each time till the maintaining the expected effect on blood
244
pressure. Along with Na nitroprusside, mmHg. Nitroglycerin or oral metoprolol Subarachnoidal Hemorrhage
Esmolol (300 μg/kg IV bolus, then 50 μg/ (50-100 mg/12 hrs or IV 5mg on every
The systolic blood pressure and the MAP
kg /min infusion) or labetalol (20-40 mg IV, 5-15 mins up to 15 mg)
should be lower than 160 mmHg and 130
then 20 mg IV on every ten mins, the
Acute Sympathetic Crises mmHg, respectively. Esmolol and
maximum dose is 300 mg) helps to
nicardipine can be used.
control heart rate. If beta blockers are Benzodiazepines are the initial treatment.
contraindicated, verapamil (5-10 mg IV or Nitroglycerine can be considered if Ischemic Stroke
diltiazem 0.25 mg/kg IV can be used. benzodiazepines are not effective.
If the fibrinolytic will be used, the systolic
Phentolamine is another choice (5-15 mg
Acute Hypertensive Pulmonary Edema blood pressure should be lower than
IV).
185/110 mmHg. If the patient will not take
The blood pressure shouldn’t be
Acute Renal Failure a fibrinolytic treatment, then it is
decreased by more than 20-30%. The
important to maintain the BP lower than
first choice is nitroglycerin (5-100 μg/min The blood pressure decreased up to 20%
220/120 mmHg. Nitroglycerin and
IV infusion). Start with 5 μg/min; then it if it is higher than 180/110 mmHg.
nicardipine can be used.
can be increased up to 200 μg/min by Nicardipine, labetalol, or fenoldopam is
increasing 10 μg on every five mins. recommended agents. Hypertensive Encephalopathy
Enalaprilat (0.625-1.25 mg IV in 5 mins
every 4-6 hours) and nicardipine 5 mg/hr Intracerebral Hemorrhage The first agent is Na nitroprusside and
IV infusion, if no control in 15 mins 2.5 followed by labetalol, nicardipine,
The mean arterial pressure (MAP) should
mg/hr dose can be added on every 15 fenoldopam. The systolic blood pressure
be decreased to130 mmHg if the patient
mins). shouldn’t be decreased by more than
has increased intracranial pressure
25% of the total. A 160-170 mmHg
Acute Coronary Syndrome findings. If no suspicion of increased
systolic blood pressure is expected in
intracranial pressure, the MAP can be
first 2-3 hours.
Maximum 20% of the blood pressure decreased to 110 mmHg or the systolic
should be acutely decreased if the blood pressure to 150-160 mmHg.
systolic blood pressure is higher than 160 Esmolol and labetalol can be used.

245
Asymptomatic Situations Discharge Criteria
Hypertensive urgencies (Absence of end-organ damage
Oral antihypertensives (hydrochlorothiazides 25 mg/day,
symptoms and findings, known to have hypertension, reversible
Metoprolol 25 mg/day, angiotensin receptor blockers, ACE
causes, etc.)
inhibitors) should be given in the ED and prescribed to the
patients whose systolic blood pressure is higher than 180-200 Referral
mmHg and the diastolic blood pressure higher than 110/120 Patients should refer to their primary care physician or
mmHg. hypertension clinic in 7 days.

Pediatric, Geriatric, and Pregnant Patients


In pregnant patients who have underlying hypertension may
References and Further Reading, click here
present with severe preeclampsia, stroke, pulmonary edema, fetal
decompensation, etc. IV hydralazine and oral nifedipine are
equally effective in pregnant patients. In the pediatric population,
the hypertensive emergency with end-organ effects requires
immediate, and gradual decreasing of the BP. Metoprolol is
effective and safe in the pediatric population.

Disposition Decisions
Admission Criteria
All patients with hypertensive emergencies, signs of end-organ
damage are admitted to the intensive care or high dependency
care unit.

246
Section 7

Pulmonary Embolism

Case Presentation
by Elif Dilek Cakal A 45-year-old female with no prior medical history presented
to the emergency department (ED) with three days of constant
shortness of breath. She was suffering from left-sided sharp
chest pain, which is stronger during inhalation. She had felt
breathless while she was climbing upstairs during the previous
week. She had no cough or expectoration. She was a non-
smoker; her only drug was daily oral contraceptive. Vitals at
arrival were as follows: Blood Pressure: 116/72 mmHg, Pulse
Rate: 102 beats per minute (bpm), Respiratory Rate: 18
breaths per minute, Body Temperature: 37°C (98.6°F), Oxygen
Saturation: 95%. Physical examination revealed no
abnormality except for the left-sided basilar crackles. Chest X-
ray was unremarkable. The emergency physician (EP)
proceeded to investigate differential diagnoses. Her Well’s
Audio is available here Score for pulmonary embolism was 4.5 (moderate) because of

247
increased heart rate and lack of alternative Introduction
The incidence of pulmonary embolism (PE) is approximately 1.5
diagnosis. The laboratory results showed
new cases per 1000 persons. Patients with chest pain, shortness
negative β-HCG, normal renal function test, of breath and syncope should have pulmonary embolism
platelet number and a D-dimer measurement of excluded. Atypical presentations include mental deterioration in

751 ng/ml (cutoff = 550 ng/ml). EP explained patients with prior dementia. EP must maintain a high index of
suspicion as the potential outcome of a misdiagnosis is
these results to the patients and suggested a
catastrophic. The mortality of untreated PE is estimated to be
computed tomographic pulmonary angiography 30% whereas the all-cause 30-day mortality of diagnosed PE is
(CTPA). CTPA showed filling defects within the only 8%.

left pulmonary artery, left anterior and lateral Critical Bedside and General Approach
segmental artery associated with pulmonary First, the EP must determine whether the patient is stable or
unstable. Instability and shock warrant stabilization in addition to
embolism. Enoxaparin, 1.0 mg/kg, twice a day
simultaneous diagnostic and therapeutic effort. If the patient is
(80 mg = 0.8 ml, each dose for approximately 80 hypoxic, administer oxygen. Severe hypoxemia or mental
kg patient), was started. Her Pulmonary deterioration necessitate intubation. If the patient is hypotensive,
Embolism Severity Index (PESI) was 65 (class I) administer only 500 mL IV bolus saline. Aggressive IV fluid may
increase the right heart strain and shock. If hypotension persists,
and implied a very low risk. As a shared decision
give IV vasopressors, particularly norepinephrine or epinephrine.
with the patient and respiratory physician, the
Obtain an ECG to exclude STEMI and dysrhythmia. Perform a
patient was referred to the respiratory physician
thorough bedside ultrasound. EP can rapidly exclude pericardial
as an outpatient after discussion and confirmed tamponade, pneumothorax, and intraabdominal bleeding via
understanding of discharge instructions. bedside ultrasound. Right ventricular enlargement or the
presence of deep venous thrombosis (DVT) gives hints of
pulmonary embolism (please check RUSH Protocol chapter).

248
What is your diagnosis in US given here? presentation. Even highly qualified EPs may miss the diagnosis
because of vague signs. Previously healthy young patients tend
• If the patient is stabilized, the patient should directly undergo
to be mildly symptomatic with normal vital signs. Prior
CTPA.
cardiopulmonary disease and cognitive dysfunction generally
• If the patient remains unstable or CTPA is unavailable, bedside obscure the diagnosis. Therefore, atypical presentations are
ultrasound is the only diagnostic tool. frequent in elderly patients. Proximal clots cause dyspnea via
ventilation-perfusion mismatch. Pneumonia-like presentation
• If CTPA confirms or bedside ultrasound strongly suggests and pain due to pulmonary infarction are more often in distal
pulmonary embolism, thrombolysis is indicated. clots. The presence or absence of sudden onset symptoms
neither increase nor decrease the probability of PE. Fewer than
• If the patient is stable with high suspicion of PE, but the
half of patients describe sudden onset.
diagnostic measures are expected to delay, administering of
one dose low molecular weight heparin (LMWH) is • Most patients with PE complain of dyspnea (82-85%), chest
recommended. pain (40-49%), pre-syncope or syncope (10-14%), and
hemoptysis (2%). Other PE-related signs and symptoms include
Differential Diagnosis functional or mental deterioration, arterial hypotension, cough,
Potentially life-threatening differential diagnoses of pulmonary
flank pain, abdominal pain, dizziness, light-headedness,
embolism are summarized in Table 4.8.
tachypnea (30-60%), fever, diaphoresis, and anxiety. DVT-
Non-Life-Threatening Causes are Bronchitis, Chest wall pain/ related symptoms may accompany. Some cases are
Costochondritis, Pleuritis/Pleurisy, GI Abnormalities (GERD, asymptomatic and diagnosed incidentally.
Peptic Ulcer, Gastritis), Panic Attack/Anxiety Disorder, Rib
• Shortness of breath, vague or apparent, is the most common
Fracture.
symptom. A patient with PE typically presents with 2 to 3 days
of new-onset shortness of breath that is not explained by a
History and Physical Examination Hints
• The character and severity of the clinical presentation may vary known medical condition, now worsened enough to seek care.
tremendously from being asymptomatic to sudden death. The Because the embolic burden is loading gradually, most patients
patient’s prior condition, clot’s size, and localization affect describe dyspnea on exertion days to weeks before dyspnea at

249
Table 4.8 Potentially Life-Threatening Differential Diagnoses Of Pulmonary Embolism

POTENTIALLY LIFE- HISTORY AND COMMENTS/


THREATENING PHYSICAL BED-SIDE ACTIONS IMAGING LABORATORY PEARLS AND
CAUSES EXAMINATION PITFALLS
ACS – STEMI Typical chest pain ECG Troponins Central PE causes angina-
like chest pain and cannot
be excluded only based on
the nature of pain.

ACS – NSTEMI Typical chest pain ECG Troponins Troponins may be elevated
in PE.

Cardiogenic Shock/ History of CHF Bed-side ultrasound Pulmonary edema on chest BNP BNP and pro-BNP may be
Congestive Heart Failure (e.g. RUSH protocol) X-ray Pro-BNP elevated in PE.

Cardiac Dysrhythmias Personal/Family history of ECG


cardiac dysrhythmias, new-
onset “convulsions”

Pneumothorax Trauma? Bed-side ultrasound Pneumothorax on chest X-


ray

Cardiac Tamponade Malignancy? Bed-side ultrasound Malignancy increases both


PE and cardiac tamponade
incidence.

Pneumonia Cough with sputum, fever, Bed-side ultrasound Pneumonic infiltration on WBC
immunosuppression chest X-ray CRP
Procalcitonin

Esophageal Rupture Chest X-ray


CT/CTPA

Pulmonary Malignancy History Chest X-ray Malignancy increases


CT/CTPA cardiac tamponade
incidence.

Asthma History
Bronchospasm on
examination

Aortic Dissection Bed-side ultrasound CT/CTPA

Pericarditis/Myocarditis History of flu-like symptoms ECG Troponins


Bed-side ultrasound

The most helpful diagnostic ways to establish the diagnoses are in bold. Original by author.
250
rest. secondary to PE or other diagnoses may cause crackles.
Bronchospasm primarily dictates other diagnoses. However, the
• Contrary to common misbelief, PE may cause both pleuritic and
EP must consider that underlying PE exacerbates Chronic
angina-like chest pain. Distal emboli induce atypical, pleuritic,
Obstructive Pulmonary Disease (COPD). Treatment-resistant
stabbing-like chest pain due to pleural irritation. Central emboli
COPD exacerbations may imply PE.
may present as typical angina-like chest pain, possibly
associated with RV ischemia. Thus, exclusion solely based on • The combination of history and physical examination is
the quality of chest pain is impossible. frequently insufficient to diagnose. Thus, the EP must
investigate risk factors to determine the likelihood of PE. The
• The frequency of syncope and pre-syncope among the ED
risk factors in the emergency setting differ from the general
patients with confirmed PE remains highly variable in different
population or longitudinal risk factors.
studies (4-22%). On the other hand, only in a minor group of
patients presented with pre-syncope and syncope, the final
Table 4.9 Selected Risk Factors of Pulmonary Embolism In
diagnosis is PE. Yet, patients with PE who present with syncope The ED Setting
tend to have major PE.
INDICATORS OF PE IN THE ED MAY BE LESS SIGNIFICANT IN
• Haemoptysis is not common but is more specific to PE. SETTING THE ED POPULATION

• Vital signs are variable. Most patients have relatively normal Age > 50 Pregnancy
Recent Surgery Smoking
vitals. Some are in shock and shock is a predictor of bad Recent Major Trauma Family History of VTE
outcome. Heart rate > 100 bpm and oxygen saturation <95% Immobilization Inactive cancer
Estrogen Travel
increase the probability. Fever does not exclude PE, though an Prior VTE
oral temperature >39.2°C (102.5°F) greatly decreases the Postpartum
Inherited Thrombophilia
possibility. Mild or severe increase in respiratory rate may be Active cancer
present. Normalization of vital signs with treatment or time does
Lorem ipsum dolor sit amet, consectetur adipisicing elit, sed do
not change the likelihood of PE.
tempor incididunt ut labore et dolore magna aliqua.
• No single examination sign confirms or excludes PE. DVT-
related signs increase the possibility. Pulmonary infarction
251
Table 4.9 summarizes selected common risk factors in the ED • B e d s i d e E C G s h o w s tachycardia and non-specific ST-T
setting. For a more detailed discussion, please refer to the changes in most of the cases. Acute S1Q3T3 finding in the ECG
relevant sections of “Emergency Evaluation For Pulmonary is seen only increased right ventricle enlargement and pressure
Embolism, Part 1: Clinical Factors That Increase Risk” at this link which seen massive emboli. These finding can also be seen in
and “Clinical Features From the History and Physical Examination core pulmonale.
That Predict the Presence or Absence of Pulmonary Embolism in
Symptomatic Emergency Department Patients: Results of a Image 4.11 ECG shows S1Q3T3
Prospective, Multicenter Study” at this link.

• Up to 30% of adult patients are without risk factors at the time


of the diagnosis.

• Clinical prediction rules stratify patients according to their


pretest probabilities.

• Wells’ Criteria for PE – link • Pulmonary embolism should come to mind in a large number of
patients because of the changeable and vague nature of its
• Revised Geneva Score (RGS) – link presentation. Proper assessment of clinical probability, D-dimer
testing and CTPA are cornerstones of management. Other
• Experienced physician’s gestalt and clinical prediction rules
d i a g n o s t i c t o o l s l i k e l u n g s c i n t i g r a p h y, b e d s i d e
showed similar performance in some studies, but the use of
echocardiography and compression venous ultrasonography of
clinical prediction rules are strongly recommended for
bilateral lower extremities may prove useful in special
inexperienced physicians.
circumstances such as pregnancy, unavailability of CTPA,
• Pulmonary Embolism Rule-Out Criteria (PERC) is recommended instability or shock.
for the bedside exclusion of low-risk patients – link
What is your diagnosis in CT given here?

Emergency Diagnostic Tests and


Interpretation
252
The high frequency of patients with Pulmonary Embolism, Part 2: dilatation and hypokinesis, septal
symptoms implying PE results in the Diagnostic Approach. flattening and paradoxical septal
dilemma of underdiagnosing or motion, diastolic left ventricular
• High-risk PE: The clinical approach
overtesting for PE. Underdiagnosing PE impairment. Other findings include
for unstable patients is demonstrated
increases mortality and morbidity. direct visualization of pulmonary
at this link as part of 2014 ESC
Overtesting raises emergency medicine embolism, pulmonary arterial
Guidelines on the diagnosis and
crowding and potential complications due hypertension, right ventricular
management of acute pulmonary
to chosen diagnostic modality. A true hypertrophy and patent foramen
embolism.
understanding and strict application of ovale. A normal right ventricular
proven and reliable clinical rules and • Once PE is confirmed accordingly, function does not exclude PE.
algorithms are recommended. prognostic assessment is the next step.
• Elevated brain natriuretic peptide
Clinical parameters and right ventricular
• The first step is the evaluation of the (BNP), N-terminal (NT)-proBNP,
function via imaging and biomarkers
patient’s stability. The diagnostic and troponins, creatinine, and D-dimer
help the EP to predict prognosis.
therapeutic measures in stable and predict higher mortality. Negative
unstable patients are different. The • The mostly studied prognostic biomarkers and D-dimer indicates a
presence of shock or hypotension prediction rules are Pulmonary good prognosis.
immediately indicates high-risk PE Embolism Severity Index (PESI) –
• Patients are classified into four
while the absence of those implies non- link. and its simplified version (sPESI)
groups based on early mortality risk.
high-risk PE. The proposed algorithms – link.
Patients who are hypotensive or in
for both are as follows:
• Right ventricular dysfunction has shock are defined as high-risk
• Non-high-risk PE: The clinical been reported ≥ 25% of patients. It regardless of other evaluations.
approach for stable patients is implies a low cardiac output and Normotensive patients who are PESI
demonstrated at this link as a part of adverse outcome. Echocardiographic class III-IV or sPESI class ≥ I are at
“Emergency Evaluation For fin d i n g s o f r i g h t v e n t r i c u l a r intermediate risk. Intermediate risk
dysfunction include right ventricular group divides into intermediate-high

253
risk and intermediate-low risk • Intubate, if necessary. Beware of dobutamine/dopamine, except for
groups, according to RV function and high intrathoracic pressure for it may a selected group of patients with
cardiac biomarker values. Lastly, a worsen the right ventricular failure. known congestive heart failure.
PESI class I-II or sPESI class 0 Therefore; aim:
• Confirm PE with whether CTPA or
normotensive patient is defined as a
• Low tidal volumes (about 6 mL/ bedside ultrasound as the patient’s
low-risk patient. The proposed risk
kg lean body weight) status permits. Prepare for thrombolytic
stratification (link) and risk-adjusted
treatment.
management strategies in acute PE • limited positive end-expiratory
is demonstrated at this link as part of pressure Medications
2014 ESC Guidelines on the Parenteral anticoagulation for stable
diagnosis and management of acute • to keep end-inspiratory plateau
patients remains the mainstay of therapy
pulmonary embolism. pressure < 30 cm H20
in ED. Low-Molecular-Weight Heparins
• Use non-invasive mechanical ventilation (LMWHs) and fondaparinux are preferred
Emergency Treatment over unfractionated heparin (UFH)
with caution.
Options because of lower major bleeding and
• Mental deterioration and coma may heparin-induced thrombocytopenia risk.
Initial Stabilization
dictate intubation.
Stabilizing interventions, diagnostic and
• LMWHs
therapeutic effort must begin immediately • If the patient is hypotensive
and continue till admission for an • Enoxaparin: 1.0 mg/kg, every 12
unstable patient. Development of shock, • Give 500 mL normal saline IV hours, SC
hypotension or hypoxemia in the course bolus. Avoid excessive IV fluids for
of ED stay warrants prompt stabilization. it may increase right ventricular • Tinzaparin: 175 U/kg, once daily, SC
strain.
• If the patient is hypoxemic • Dose reduction is required in renal
• If fluid bolus does not help, start impairment.
• Administer oxygen vasopressors. Norepinephrine and
• Fondaparinux
epinephrine are preferred over

254
• 7.5 mg, body weight 50-100 kg, Thrombolytic treatment must be reserved Pediatric, Geriatric,
for unstable patients. Streptokinase, Pregnant Patient, and Other
• 5 mg, body weight <50 kg
urokinase and recombinant tissue
Considerations
• 10 mg, if body weight >100 kg, once p l a s m i n o g e n a c t i v a t o r ( r T PA ) a re
daily, SC) approved thrombolytic agents for PE. As Pediatric considerations
a general rule, LMWHs, fondaparinux and The pediatric pulmonary embolism is
• C o n t r a i n d i c a t e d , i f c re a t i n i n e UFH infusion must be stopped during relatively rare, but widespread use of
clearance <30 mL/min t h r o m b o l y t i c t h e r a p y. C u r r e n t l y, CTPA showed that it is more frequent
recombinant tissue plasminogen activator than previously thought.
• Dose reduction by 50%, creatinine
(rtPA) is the most widely used agent and
clearance is 30-50 mL/min. Up to 30% of adult patients have no
its dose is 100 mg over 2 hours OR, 0.6
identifiable risk factors. Unlike adults,
• UFH mg/kg over 15 minutes, the maximum
96-98% of pediatric patients have
dose of 50 mg.
• 80 Units/kg IV bolus, then 18 Units/ identifiable risk factors, 88% have two or
kg/h continues IV infusion. Vitamin K antagonists (VKAs, e.g., more. Infants and neonates bear the
warfarin) and new oral anticoagulants highest risk. In all age groups, a central
• Recommended, if the patient is venous catheter is the most common risk
(NOACs, e.g., dabigatran, rivaroxaban)
should be started in the inpatient setting factor. Other common risk factors include
• a candidate for thrombolytic
after initial therapy. dehydration, septicemia, peripartum
treatment
asphyxia in neonates. Malignancy, lupus
• severely obese Surgical embolectomy, percutaneous erythematosus, renal disease, congenital
catheter-directed treatment, and venous thrombophilia, surgery and major trauma
• Recommended, if creatinine filters are rarely applied after admission to are common predisposing factors in older
clearance <30 mL/min ICU. c h i l d re n . O v e r a l l , i m m o b i l i z a t i o n ,
• Advantages: The ease of hypercoagulability, central venous
monitoring and reversal of effects catheter, excess estrogen state, and
by protamine. concurrent deep venous thrombosis are

255
associated with pediatric PE. Deep • >2 months: 1.0 mg/kg/dose SC, from ionizing radiation, the fear of
venous thrombosis in children is twice a day missing a life-threatening diagnosis and
predominantly associated with upper the need for quick decisions harden the
extremity and central venous catheter Geriatric considerations management of a pregnant woman with
The management and treatment do not
rather than lower extremity as in adults. suspected PE. A clinical pathway is
change in geriatric patients. However, the
recommended at this link as a part of
Pleuritic chest pain (84%), hemoptysis EP should consider a few issues. The
“Emergency Evaluation For Pulmonary
and shortness of breath are the main incidence of PE increases with age.
Embolism, Part 2: Diagnostic Approach.
symptoms. D-dimer and prediction rules Atypical presentations are common;
are not studied in children. CTPA remains comorbid illnesses and dementia obscure The first step is bilateral lower extremity
the primary diagnostic tool in the the diagnosis. The treatment does not venous ultrasound. If the ultrasound is
emergency setting. The segmental change, but complications of positive, the treatment starts without
arteries are affected 52%. The main or anticoagulation occur more frequently. further investigation. If the ultrasound is
central arteries are affected 6%. Children The EP must adjust dose according to negative, the EP must assess the pretest
tend to compensate for relatively large comorbid situations like renal dysfunction p r o b a b i l i t y ( P T P ) . T h e t r i m e s t e r,
clots well out of their cardiopulmonary of cachexia. physician’s gestalt or clinical prediction
reserve. rules are available methods to assess
Pregnant patient considerations PTP. Note that no prediction rule is
LMWH is the mainstay of the therapy. PE and pregnancy form an ominous validated in pregnant. In the non-high risk
Hemodynamically unstable patients couple for apparent reasons. Pregnant group, PERC negative patients are further
should receive thrombosis. The prognosis and postpartum women are susceptible stratified with D-dimer. If D-dimer is under
is generally good. Shock is the predictor to PE. Peak times are the third trimester cutoff values according to trimesters, PE
of an adverse outcome. and the first 4 weeks following the labor, can be excluded to a reasonable degree
particularly after cesarean section. of medical certainty. High risk, PERC
• Enoxaparin
Moreover, breathlessness is a common positive or D-dimer positive patients
• <2 months: 1.5 mg/kg/dose SC, c o m p l a i n t d u r i n g p re g n a n c y. T h e should undergo imaging. On the imaging
twice a day potential harm to fetus and woman breast

256
branch, shared decision-making should be pursued between Referral
CTPA and ventilation-perfusion scan. • Patients must be referred to respiratory or internal medicine.

LMWH is safe during pregnancy [Pregnancy Category (PC) B] and Pearls And Pitfalls
lactation and so is standard treatment and is preferred over Use validated clinical prediction rules to estimate pretest
heparin (PC C). Fondaparinux (PC B) is not recommended due to probability in patients with suspected PE.
lack of data. VKAs (PC X) and new oral coagulants are
contraindicated in pregnancy. Pregnancy does not alter the Do not proceed to D-dimer measurements or imaging
dosage. studies in patients with a low PTP and negative Pulmonary
Embolism Rule-Out Criteria.
Disposition Decisions
A high sensitivity D-dimer is the initial test in patients with
Admission Criteria intermediate PTP or low PTP but a positive Pulmonary
• All high-risk patients, including those in shock, who are Embolism Rule-Out Criteria. Imaging studies are not the
hypotensive, post-CPR, intubated, or who have received initial test in patients with low or intermediate PTP.
thrombolytic treatment must be admitted to ICU.
Use age-adjusted D-dimer thresholds in patients older
• Intermediate-high risk patients should be observed in monitored than 50.
beds and possibly in ICU
CTPA is the initial test in patients with high PTP.
• Intermediate-low risk and low-risk patients should be admitted Ventilation-perfusion scans are alternative if CTPA is
to the ward. contraindicated or unavailable. D-dimer cannot exclude
PE in patients with high PTP.
Discharge Criteria
• A very selected group of low-risk patients may be treated as
outpatients. A proposed algorithm for outpatient management
References and Further Reading, click here
of PE is available at this link

• Several discharge instructions are available online (I and II).

257
Chapter 5

Selected Endocrine,
Electrolyte
Emergencies
Section 1

Acid-Base Disturbance

Case Presentation
by Lamiess Osman, Qais Abuagla A 15-year-old female presented with dyspnea, polyuria, and
polydipsia for the last 3 days. She was slightly lethargic with
dry oral mucosa. Vitals were BP 92/45mmHg, RR 27/bpm,
HR119/bpm, Temp 37°C, SpO2 99% on INO2 1L/min.
Physical examination revealed normal findings except there
was a mild abdominal tenderness without guarding.
A bedside arterial blood gas revealed the following:

• pH: 7.19

• PaO2: 105mmHg

• PaCO2: 19mmHg

• HCO3: 7mmol/L

• Na: 124mmol/L

• K: 3.4mmol/L

259
• Cl: 91mmol/L Introduction
• Gluc: 310 mg/dL Definitions
Acid: a substance that is capable of donating a hydrogen ion to
• BUN: 13mmol/L
another substance
The patient was put under close monitoring, and Base: the substance that is capable of receiving a hydrogen ion.
intravenous fluids were initiated. Urine dipstick
Acidemia is serum pH < 7.35.
showed glucose 4+ with ketones. A diagnosis of
diabetic ketoacidosis was made. The arterial Alkalemia is serum pH > 7.45.

blood gas (ABG) was evaluated at the end of the Acidosis refers to physiologic processes that cause acid
chapter for this case. accumulation or alkali loss.

Alkalosis refers to physiologic processes that cause alkali


accumulation or acid loss.

Defense Mechanisms
The body has three defense mechanisms to maintain normal pH:

1. The buffering system.

2. The respiratory system.

3. The renal system.

Among the three systems, the  buffering is the fastest means of


preventing disturbances in pH. It was likened to a ”sponge” that
soaks up excessive hydrogen ions and releases them when
there’s a deficient concentration. Several buffering agents bind
260
hydrogen ions reversibly. These include blood gas (ABG) and manage it in the a r e made up of carbon dioxide; as an
bicarbonate, ammonia, hemoglobin and clinical context. end product of the metabolism of
plasma proteins. carbohydrates and fats. Although CO2 is
Critical Bedside Actions and General
not an acid in itself, when combined with
The respiratory system is a key player in Approach
water it forms carbonic acid, which is
acid-base regulation, albeit acting slightly
The emergency department is where you capable of dissociating to hydrogen ions
slower than the buffer system. Central
will first encounter sick patients having and bicarbonate ions. Lungs can excrete
and peripheral chemoreceptors, when
critical situations including acide-base CO2 it is known as volatile acids.
stimulated, increase the rate and depth of
disorders. Thetefore, you must have the However, many acids are referred to as
respiration when carbon dioxide (CO2) or
ability to diagnose and manage acid-base fixed acids such as ketoacidosis and
hydrogen ions levels are elevated. This
derangement. However, every critically ill lactic acid, and they depend on the
increases the rate  of CO2 elimination
patient should be evaluated for airway, kidneys for their excretion.
from the lungs, resulting in less CO2
breathing and circulation in the initial
available to form carbonic acid. The body concentration of hydrogen ions
assessment phase and necessary
must be maintained within a strict range
The renal system is a much slower resuscitation effort should be applied.
for optimal cellular function, and even a
process for dealing with hydrogen ion Many of the patients with acid-base
small deviation can significantly affect the
concentration change, taking hours to problem require cardiac monitorization, IV
patient. This underscores the importance
days. Therefore, it is more important for lines, supplemental fluid and oxygen
of being able to evaluate a blood gas for
the long-term maintenance of acid-base during the initial evaluation phase.
an acid-base disorder. Essentially, this is
balance. The kidneys can reabsorb accomplished by these laboratory values:
An acid-base derangement may
bicarbonate and excrete hydrogen ions. pH, PaCO2, and serum HCO3. Na and Cl
contribute to the patient symptoms and in
In clinical practice, the recognition of one some cases signifies an immediate life are also required in the anion gap

acid-base disorder must prompt the threat. analysis.

search for other concurrent disorders.


The body produces acid as a byproduct
Due diligence must be exercised to
of basal metabolism. Respiratory acids
complete the evaluation of the arterial
261
5 Simple Steps to Solve an Acid-Base Metabolic Acidosis • Hyperalimentation/
Problem • Calculate the Anion Gap. AG = Na- (Cl Hypoaldosteronism
+ HCO3)
Illustration 5.1 • Acetazolamide
• Normal anion gap is 8-16.
• Renal tubular acidosis
• Causes of high anion gap metabolic
• Diarrhea
acidosis (i.e., AG >16) can be
remembered using mnemonic: “MUD • Uretero-Pelvic shunt
PILES.”
• Post hypocapnia
• Methanol
• Spironolactone
• Uremia
Respiratory Acidosis
• DKA From a pathophysiological perspective,
the two broad categories are V/Q
• Paraldehyde
mismatch with/out increased CO2
• Isoniazid production, and alveolar hypoventilation
due to either central causes or chest wall-
• Lactic Acidosis
neuromuscular disorders. Causes can
• EtOH/Ethylene glycol also be classified according to its acuity.

• Salicylates • Acute causes:

• Causes of a normal anion gap • CNS depression (cerebrovascular


metabolic acidosis (i.e., AG <16) can be accident/drugs)
remembered using the mnemonic:
• Airway obstruction
“HARDUPS”

262
• Pneumonia • Post-hypercapnia Step 2: Evaluate the primary process
that accounts for the deranged pH.
• Pulmonary edema • Diuretics*
• The low HCO3 with a low PaCO2
• Hemo/pneumothorax (* associated with high urine Cl level)
indicates that the main primary disorder
• Myopathy is metabolic acidosis.

• Chronic cause: Respiratory Alkalosis Step 3: Evaluate for appropriate, over or


This is secondary to excessive under compensation: indicating the
• Chronic obstructive pulmonary concurrent primary acid-base disorder in
ventilation, i.e., excessive respiratory rate
disease (COPD)
and/or depth from the following causes: the compensating system.

• Restrictive lung disease


• CNS disease • Using the Winter’s formula {expected
PaCO2 =1.5[HCO3]+ 8 (±2)}, the
Metabolic Alkalosis • Hypoxia
Obtain a urine Chloride level and expected PaCO2 is 18.5. The patient’s

remember the following mnemonic: actual PaCO2 (19mmHg) lies within this
• Anxiety
range. This means that the respiratory
“CLEVER PD”
• Mechanical ventilation compensation is appropriate and there
• Contraction (due to blood loss) was no concurrent respiratory acid-
• Progesterone
based disorder.
• Licorice *
• Salicylates
Step 4: Calculate the Anion Gap, Anion
• Endocrine (Conn’s/ Cushing’s/ Batter’s)*
• Sepsis Gap and delta HCO3
• Vomiting/nasogastric suction
Looking Back To Our Case AG = Na – (HCO3 + Cl) ±4). The patient’s
• Excess Alkali* Step 1: Interpretation of pH AG is 26±4 {i.e. 124 – (7 + 91 ) ±4}.

• Refeeding Alkalosis* • The pH <7.35 indicating acidosis AG = AG-12. The patient’s AG is 14±4
{i.e.26-12(±4)}.

263
Delta HCO3 = 24-HCO3. The patient’s delta HCO3 is 17 {i.e.
24-7}

Since this drop in HCO3 (17) lies within the increased in AG


(14±4), there is no second metabolic disorder.

In conclusion, the patient’s ABG is a pure High Anion Gap


Metabolic Acidosis. As a Step 5, look for the causes of defined
acid/base situation. In this case, use MUD PILES mnemonic to
find specific problem.

References and Further Reading, click here

264
Section 2

Hyperglycemia

Case Presentation
by Toh Hong Chuen A 58-year-old lady presented with right foot pain for 3 days,
associated with high fever, lethargy, polyuria, and polydipsia.
At triage, air hunger was noted. Her vital signs were: BP 82/46
mmHg, PR 131/min, RR 28/min, T 38.7 and SpO2 98%. She
was brought to the resuscitation room for further
management.

Clinically, she was dehydrated and confused with GCS


E4V4M6. Her neck was supple, and lungs were clear. Crepitus
was noted on the dorsum of the right foot. Point of care blood
tests showed: capillary glucose 40 mmol/L, capillary ketone
7.2 mmol/L, pH 7.22, HCO3 8 mmol/L, pCO2 20 mmHg, Na
130 mmol/L, Cl 95 mmol/L, K 5.5 mmol/L and lactate 6.9
mmol/L.

Audio is available here

265
A diagnosis of septic shock Image 5.1 Critical Bedside Actions
secondary to gas gangrene and General Approach
Diabetic ketoacidosis (DKA) and
complicated by diabetic hyperosmolar hyperglycemic state (HHS)
ketoacidosis was made. She are potentially life-threatening diabetic
was aggressively resuscitated emergencies. In acutely ill patients with
hyperglycemia, blood and urine tests
with fluid and started on IV
must be performed, preferably at the
insulin infusion. Potassium point of care, to evaluate for the presence
replacement was withheld as K of DKA or HHS (Table 5.1).

was elevated. Urinary Key management principles for DKA and


catheterization was performed HHS include initiation of IV fluid therapy
and IV insulin infusion while ensuring
for strict input-output
normokalemia and avoiding
monitoring. Broad-spectrum hypoglycemia. The precipitating cause
antibiotics and intramuscular should be identified and treated. Patient’s

tetanus toxoid were given. X- vital signs, mental status and biochemical
The patient was sent directly to response to therapy (glucose, ketones,
ray (Image 5.1) of right foot Na and K) and input-output must be
the theatre and underwent
confirmed subcutaneous air. closely monitored. As DKA and HHS
extensive debridement for the resolve, overlap with s/c insulin prior to
gas gangrene. She had an stopping the insulin infusion. See
appendix 1 for management details.
uneventful recovery and was
discharged 1 week later.

266
Differential Diagnoses D K A a n d H H S a r e distinguished as follow:

Hallmarks of DKA are anion-gap acidosis, ketosis, and


Table 5.1 Diagnostic Criteria For DKA And HHS
hyperglycemia.
DKA
DKA
TERIA DKA MILD MODERAT HHS
SEVERE • Usually associated with type 1 diabetes; can occur in type 2
E
diabetes during a serious illness.
Serum
glucose ≥14 ≥14 ≥14 ≥33
(mmol/L)
• A short duration of symptoms, usually <1-2 days.

pH 7.25-7.30 7.00-7.24 <7.00 >7.3 • Up to 10% of DKA are “euglycaemic” (glucose <14). They can
HCO3 be seen in pregnant patients, those with restricted food intake,
15-18 10-14 <10 >18
(mmol/L) or had initiated insulin therapy (though insufficient) prior to
Anion gap presentation.
>10 >12 >12 Variable
(mmol/L)
Serum Hallmarks of HHS are profound hyperosmolality, hyperglycemia,
ketone ≥0.6 ≥0.6 ≥0.6 Small/none and dehydration.
(mmol/L)
Urine • Associated with type 2 diabetes; can occur with type 1 as a
Positive Positive Positive Small/none
ketone simultaneous occurrence with DKA.
Effective
serum
Variable Variable Variable >320
• Like DKA, the circulating amount of insulin is inadequate to
osmolality
prevent hyperglycemia. Unlike DKA, this amount is sufficient to
(mOsm/kg)
prevent lipolysis and ketoacidosis.
Adapted from Kitabchi AE, Umpierrez GE, Miles JM, Fisher JN.
Hyperglycemic crises in adult patients with diabetes. Diabetes Care.
2009;32(7):1335-1343. doi:10.2337/dc09-9032. Please read the article for • Without significant ketoacidosis, HHS develops slowly and
more information. subtly over several days, contributing to more severe water
deficit at around 7-12 L, compared to 4-6 L in DKA.

267
• The older terms “hyperglycemic • Bleeding GI • DKA patients often present with
hyperosmolar nonketotic coma” (HHNK) a i r h u n g e r a n d k u s s m a u l ’s
and “hyperglycemic hyperosmolar History and Physical breathing secondary to acidosis,
nonketotic state” (HHNS) should not be Examination Hints unlike HHS.
used. Patients often present without 1. DKA and HHS can present similarly
with malaise, anorexia, thirst, polyuria, 4. Abdominal symptoms
coma, and ketonemia may found in
some. and polydipsia. In addition, they can
• In DKA, nausea, vomiting and
be triggered by similar precipitants.
abdominal pain are associated
Up to 1/3 of patients have an overlap of D i ffe r e n c e s i n c l u d e a l o n g e r
with the severity of ketoacidosis.
DKA and HHS. presentation and more severe
dehydration in HHS. • Conversely, an acute abdominal
Any significant stress can precipitate
process such as pancreatitis can
DKA/HHS, remembered as “7 ‘I’s have 2. Neurological symptoms
precipitate DKA. Search for
bled!”.
• HHS patients often have some intraabdominal precipitants if
• Iatrogenic (drug interaction, e.g. degree of altered mental state or abdominal pain when DKA is mild,
steroids) other neurological disturbances. persistent pain despite
These are related to the severity improvement of acidosis or signs
• Idiopathic (new onset DM)
and rate of development of of peritonism.
• Illegal (substance abuse) hyperosmolality.
• HHS (no significant ketoacidosis)
• Infarction (e.g. AMI, stroke, bowel • Conversely, since hyperosmolality is not associated with abdominal
ischemia) is absent or insignificant, DKA pain. Evaluate for an abdominal
patients have normal neurological precipitant if there are abdominal
• Infection (e.g. pneumonia, UTI, cellulitis) symptoms.
status. Only severe DKA presents
• Infraction (i.e. noncompliance) with coma.
5. Physical findings may be unreliable for
3. Respiratory symptoms estimating the degree of dehydration,
• IUP (i.e. pregnancy)
particularly in children
268
• In DKA, patients may appear more • Note that the severity of metabolic •AAc and BHB fully dissociate in
dehydrated from the drying of oral acidosis can be masked by metabolic physiological pH and contribute to
m u c o s a d u e t o K u s s m a u l ’s alkalosis from vomiting. HAGMA. Ac, which does not dissociate,
respiration. does not.
• Interestingly, most patients change from
• Hyperosmolality in HHS may HAGMA to NAGMA while recovering • BHB is the most abundant ketone in
“preserve” intravascular volume from treatment. This is due to urinary DKA, with a ratio of 10:1 compared to
(even though it leads to urinary loss of ketones earlier during osmotic Aac, and Ac is least abundant.
losses) and mask signs of volume diuresis. Ketones can be metabolized to
• Insulin reduces overall ketone level but
depletion until hemodynamic bicarbonate when adequate insulin is
also converts BHB AAc. As
deterioration suddenly occur. provided; hence the loss of urinary
nitroprusside-based urine test detects
ketones is equivalent to losing
Emergency Diagnostic Tests only Ac and AAc, urinary ketones may
bicarbonate, resulting in NAGMA.
and Interpretation not improve or paradoxically worsen
Ketosis with treatment. Therefore, serum BHB
High Anion Gap Metabolic • The small amount of ketones is should be used to monitor resolution of
Acidosis (HAGMA) normally present (<0.6 mmol/L) acting ketosis.
• The most important feature of DKA. as an alternative energy source if
• Note that blood test for ketones can be
• Anion gap = Na – (Cl + HCO3). Use glucose is not available.
falsely positive in a patient taking
measured sodium in the calculation of sulfhydryl drugs.
• In DKA, relative or absolute insulin
anion gap.
d e fi c i e n c y a n d t h e s u r g e o f
counterregulatory hormones (especially Serum osmolality
• pH in venous blood gas is sufficient as
• The effective serum osmolality should
it correlates with arterial pH. Perform glucagon and catecholamines) cause
be used in the diagnosis of HHS, not
arterial blood gas only if concomitant unrestrained ketogenesis. All three
measured osmolality. Measured Na
respiratory failure is suspected. ketones, acetone (Ac), acetoacetate
should be used to determine the
(AAc) and beta-hydroxybutyrate (BHB)
osmolality.
are elevated.
269
• Effective serum osmolality = 2 x Na low normal or mild hyponatremia Emergency Treatment
(mmol/L) + Glucose (mmol/L). (dilutional). Hypernatraemia, therefore, Options
signifies severe dehydration.
• Urea travels freely across a cell Fluids
membrane and does not contribute to • Use the measured Na when calculating • Patients with DKA and HHS are
osmolality in vivo. the anion gap and serum osmolality. invariably volume depleted. Start IV
0.9% NaCl at 10-20 mmol/kg/hr during
Serum potassium • Corrected Na = serum Na + 2 [ (serum
the first hour.
• Check K before and after starting glucose – 5.5)/5.5 ]
insulin. • After BP and perfusion normalizes,
• After the initial fluid challenge with 0.9%
continue infusion at a rate of 250-500
• Total body potassium depletion occurs NaCl, use corrected sodium to decide
ml/hr with 0.45% NaCl if the calculated
through urinary (and occasionally on the choice of saline for infusion.
Na is normal or high; or 0.9% NaCl if
gastrointestinal) loss. Serum K,
Others the calculated Na is low
h o w e v e r, m a y b e n o r m a l o r
• Leukocytosis is present due to the
paradoxically elevated due to • The total body fluid deficit should be
elevated levels of stress hormones. Up
transcellular shift (acidosis, insulin slowly corrected over 24 hrs.
to 15K may be expected for DKA.
deficiency), volume contraction and
reduced renal function. • Serum amylase, lipase, hepatic
IV insulin
• Mainstay treatment of DKA and HHS.
enzymes, creatinine kinase and CRP,
• Hypokalemia at presentation signifies
can be mildly elevated. These are • Before initiating IV insulin,
profound K loss. This generally worsens
nonspecific findings.
with treatment and may precipitate
• Initiate fluid replacement. With
threatening arrhythmia and profound
insulin, glucose is taken up by cells,
respiratory muscle weakness.
drawing fluid out of intravascular
space and can cause hypotension.
Serum sodium
• As hyperglycemia draws fluid into the
intravascular space, most patients have
270
• Correct hypokalemia, if present, with • Add D5% to replacement fluid, •Add 20-30 mmol of K in each liter of
IV KCl at 20-30 mmol/hr until K>3.3 keep serum glucose between 8-11 fluid to maintain normokalemia.
mmol/L. mmol/L until ketoacidosis
• Withhold K is elevated above the
resolves.
• Give as a continuous infusion of 0.1 U/ upper limit (or >5.2 mmol/L).
kg/hr, and not as a bolus as this may • Once resolved and the patient is
• Check K 2 hourly after initiating fluid
cause severe hypokalemia and may risk a b l e t o t a k e o r a l l y, s t a r t
and insulin therapy
hypoglycemia. subcutaneous rapid-acting insulin
at 0.1 U/kg around 30-60 min
• Resolution of DKA as indicated by Phosphate
b e f o re s t o p p i n g t h e i n s u l i n • DKA patients have total body
serum glucose <11 mmol/L plus any 2
infusion, given the delayed onset phosphate though serum levels may be
of the following:
of the s/c preparation, to prevent normal or elevated. Treatment with
• pH >7.3, rebound hyperglycemia. insulin drives phosphate intracellularly
and worsens hypophosphatemia.
• HCO3 >18 mmol/L • Similarly in HHS, when capillary glucose
reaches 16 mmol/L: • However, routine phosphate
• Anion Gap ≤ 12 mmol/L
replacement is not indicated as no
• Reduce IV insulin rate to 0.02-0.5
• As the resolution of ketoacidosis in DKA studies demonstrated benefit.
U/kg/hr, keep serum glucose
often lack behind hyperglycemia (mean Treatment could also precipitate
between 11-16 mmol/L.
duration of 12 hours versus 6 hours hypocalcemia.
respectively), IV insulin should be • IV insulin can be stopped once the
• Consider phosphate replacement (IV
continued with dextrose replacement to patient is alert, taking orally and
K2PO4 at 4.5 mmol/hr) in DKA if:
clear the ketones. When capillary started on their oral hypoglycemic
glucose reaches 11 mmol/L: or subcutaneous insulin. • Cardiac dysfunction

• Reduce IV insulin rate to 0.02-0.5 Potassium • Respiratory depression


U/kg/hr • Correct hypokalemia if present as
above. • Anemia
271
• Severe hypophosphatemia (<1 • Presents similarly to non-pregnant patient is hypotensive, use IV 3%
mg/dL). women, but 1/3 of patients may have NaCl (5-10 ml/kg over 30 min).
euglycaemic DKA.
• Unlike DKA, there is no evidence for Geriatric (HHS)
replacement of phosphate in HHS. • Treated similarly, but include fetal • Prone to develop HHS due to altered
monitoring. thirst mechanism, restricted access to
Sodium Bicarbonate water and decreased pancreatic
• NaHCO3 should not be given in DKA as Pediatric (DKA) reserve. Always check blood sugar in
it is Independent risk factor for cerebral • As the clinical judgment of dehydration
patients, especially the elderly, with an
edema, and can worsen hypokalemia, is unreliable, set maintenance fluids
altered mental state.
create excessive sodium load, induce based on of 6% total water deficit.
paradoxical CSF acidosis or even • Managed similarly to other population.
• The most feared complication is
impairing release of O2 to tissues via
cerebral edema Disposition Decisions
shifting the oxygen-hemoglobin curve
to left. All patients with DKA or HHS should be
• Occurs in 1% of DKA episodes, but
admitted. They should be sent to the ICU
high mortality of 40-90%.
• Consider only if the patient is severely or high dependency unit if there were
acidotic (pH <6.9). • Major risk factors include treatment severe symptoms, refractory
with sodium bicarbonate and severe hypotension, refractory oliguria or
Pediatric, Geriatric, hypocapnia and high urea at persistent mental state changes.
Pregnant Patient and Other presentation.
Considerations
• Typically presents 4-12 hours after
Pregnancy (DKA) References and Further Reading, click
initiating treatment, but can be delayed
• Low maternal mortality (<1%), but here
up to 48 hours. Once recognized:
perinatal mortality can be as high as
elevate the head of the bed and reduce
35%.
IV fluids by 1/3. If BP is normal, start IV
mannitol (0.5-1 g/kg over 20 min). If

272
Section 3

Hypernatremia

Case Presentation
by Vigor Arva, Gregor Prosen A 79-year-old man was brought to the emergency department
(ED) by his wife. She complained that the patient had general
weakness and was feeling ‘unwell’ for the last two days. He
had a history of dementia, diabetes, renal failure, and
hypertension. He was on diabetic and antihypertensive
medication.

On examination, his vital signs were as follow HR 115/min, BP


135/90 mmHg, RR 17/min, and afebrile with normal oxygen
saturation. He was confused and disoriented, but there was
no other deficits or localizing signs on neurological exam. He
was clinically dehydrated with dry oral mucosa. Lab results
showed a serum sodium concentration of 160 mEq/L, with
elevated glucose, creatinine, urea, and osmolality. Point of
care ultrasound demonstrated a small and almost totally-
Audio is available here
collapsed inferior vena cava. Upon further history taking, the

273
patient’s wife reported that he had not been Critical Bedside Actions and General
drinking much for the last few days, even though Approach
Hypernatremia should be excluded in any patient who presents
he did not complain about thirst. with altered mental status, particularly, the very young or old and
those with abnormal basal cognition.

Once diagnosed, the next step is to assess for volume status and
acuity of symptoms, as they influence treatment plan. It is
considered acute if symptoms develop within 48hrs, and chronic
if it is longer. Then, the cause should be determined and treated.

Differential Diagnoses
Hypernatremia usually results from relative water losses, and
rarely secondary to sodium overload. The causes can be
categorized into the following three groups:

A. Hypernatremia with hypovolemia: Often found with dehydration


and fluid losses such as heatstroke, excessive sweating, burns,
and gastrointestinal losses with diarrhea and vomiting

B. Hypernatremia with euvolemia: Occurs mostly as a result of


ADH deficiency, diabetes insipidus, reset of osmostat,
hypothalamic dysfunction, primary hypodipsia, renal diseases,
drugs and alcohol (e.g., lithium, amphotericin, phenytoin,
aminoglycosides)

C. Hypernatremia with hypervolemia: Can occur because of


iatrogenic causes infusion of hypertonic sodium solutions (3%

274
saline, or large volumes of sodium bicarbonate), ingestion of salt History and Physical Examination
water or large amounts of salt, Cushing’s and Conn’s syndrome Hints
The signs and symptoms of hypernatremia are nonspecific
Diabetes insipidus (DI) refers to an absolute or relative antidiuretic
including lethargy, irritability, restlessness, hyperactive reflexes,
hormone (ADH) deficiency. Absolute ADH deficiency occurs in the
and increased muscle tone. Severe symptoms usually occur after
setting of inadequate ADH secretion and is called central DI.
the serum Na has risen acutely above 158 mEq/L, and may
Relative ADH deficiency occurs in lack of renal response to ADH
include seizures, coma or even death.
and is called nephrogenic DI.
Rapid and severe hypernatremia developing over minutes and
Malignant diseases, trauma or surgery on the pituitary, infiltrative
hours can result in brain hemorrhage, due to the accompanying
diseases, familial diseases or idiopathic conditions may cause
rapid decrease in brain volume causing ruptures of cerebral veins.
central DI. Chronic renal insufficiency, tubulointerstitial diseases,
It is also associated with hypocalcemia, for unclear reasons.
polycystic kidney disease, hypercalcemia, hypokalemia, lithium
toxicity, or familial diseases may cause nephrogenic DI. If the hypernatremia is chronic, the brain can adapt by generating
intracellular osmogenic compounds, or idiogenic osmoles, which
Symptomatic hypernatraemia (e.g., polyuria and polydipsia,
increases the osmolality in the cells and thus maintaining brain
lethargy and weakness) with an inappropriately low urine
volume by resisting shrinkage.
osmolality (<300 mOsm/kg) should suggest DI in the ED. The
formal diagnosis requires a water deprivation test, which is often Patients should be asked about their fluid and salt intake, urine
not performed in the ED given the long duration required. output, and concurrent medical and medication history. The
patient’s caregiver should be interviewed, especially if the patient
Central and nephrogenic DI are further distinguished by the
is mentally altered, to see if there are mental or behavioral
response to desmopressin (synthetic ADH). With desmopressin,
changes (e.g., excessive water intake).
urine osmolality will rise to more than 800 mOsm/kg in patients
with central DI, while this rise is absent in nephrogenic DI. Patients should be examined for their volume status by checking
skin turgor, capillary refill, looking for edema and raised jugular
Psychogenic polydipsia can be distinguished from DI by water
venous pressure, measuring heart rate, blood pressure and
restriction. Following water restriction, urine osmolality will rise in
looking for a postural drop, mental and neurological status.
psychogenic polydipsia and remain unchanged in DI.
275
Emergency Diagnostic Tests and After stabilizing the patient, proceed to evaluate for and treat the

Interpretation underlying cause of hypernatremia (e.g., fever, new medicine,


Serum sodium level (Normal reference: 135-145 mEq/L) and gastrointestinal disturbances and DI).
serum osmolality (Normal reference: 275-295 mOsm/L) should be
In patients with acute hypernatremia (symptoms occurring
checked. Both can be done at the bedside. Other electrolytes
<48hrs), it is relatively safe to correct sodium at a rate of 1mEq/L/
(mainly potassium and calcium) should be checked. Patients
hr, with correction of the total water deficit in 24 hours. This is
urine output and urine osmolality should be measured.
because the brain did not have enough time to produce idiogenic
Total body water (TBW) deficit can be calculated as a function of osmoles, lowering the risk for cerebral edema.
sodium concentration:
In patients suspected of having chronic hypernatremia, the
sodium should be corrected gradually, and no more than 0.5
TBW deficit = TBW x [(serum sodium/140) – 1]
mEq/hr or 10-12 mEq/day. Rapid overcorrection can result in
Average total body water is obtained by multiplying the patient’s cerebral edema, especially in children. Oral rehydration with water
body mass by 0.6. However, this holds mostly for children and may be the safest option.
adult men. A multiplier of 0.5 for elderly men and adult women
Sodium concentration in different solutions
and 0.45 for elderly women should be preferred.
• 0.9% NaCl: 154 mmol/L
Emergency Treatment Options
Following the initial ABC evaluation, hypernatremic patients with • Ringer’s Lactate: 130 mmol/L
hemodynamic instability should be resuscitated with intravenous
• 0.45% NaCl: 77 mmol/L
fluids. The aim at this point is to address the underlying
hypovolemia and tissue hypoperfusion. Unless this is corrected, • 5% Dextrose in water (D5W): 0 mmol/L
the body’s normal response is to increase the sodium
concentration even further to maintain intravascular volume, To determine the volume and rate of correction, one can calculate
which will worsen the hypernatremia. Fluid resuscitation can be it manually using the formulas given above. There are also several
achieved using normal saline (0.9%) or Ringer’s lactate. free online calculators available as well, such as the one by
Medcalc.
276
Patients with known central DI should be given desmopressin Hemodynamically stable and asymptomatic patients with mild
(DDAVP) which should improve symptoms. Initial doses are 1 – 2 hypernatremia from benign causes may be discharged with
micrograms. The dose should be up-titrated if necessary. advice and follow up with the primary physician.

Pediatric, Geriatric, Pregnant Patient, and


Other Considerations References and Further Reading, click here
Infants can develop hypernatremia if they are not given adequate
fluids, or are given hypertonic fluids. Older children can also
develop hypernatraemia with volume depletion following after
severe diarrhea or vomiting.

Pregnant patients may develop hypernatremia if vomiting


secondary to hyperemesis gravidarum is severe. Cases of
gestational diabetes insipidus have been documented and should
not be confused with polyuria that can normally occur during
pregnancy.

Hypernatremia is common in the elderly. The causes are likely


multi-factorial and inter-related, including the decrease in thirst
sensation, polypharmacy, and pre-existing co-morbidities.

Disposition Decisions
Patients with symptomatic hypernatremia should be admitted for
evaluation and treatment, as the free water deficit is generally
replaced gradually. Those with severe neurological symptoms will
require admission to a closely monitored unit.

277
Section 4

Hyponatremia

Case Presentation
by Vigor Arva, Gregor Prosen A 72-year-old man was brought to the emergency department
(ED) by his daughter. She reported that he had nausea,
vomiting, and confusion and had been unwell for the last few
days. He had hypertension and heart failure for the previous
ten years and was on ACE-inhibitor, beta-blocker and thiazide
diuretic.

At triage, the patient’s vital signs were usual: BP 110/70


mmHg, HR 95/min, RR 15/min, temperature 36.1°C and SpO2
100% on room air. He appeared lethargic and walked with an
unsteady gait. He had no focal neurological deficit. He had a
normal skin turgor and no edema. Postural BP revealed mild
orthostatic hypotension. The lab results showed a serum
sodium concentration of 115 mEq/L.

Audio is available here

278
Critical Bedside Actions and changes of sodium concentration usually Isotonic hyponatremia (osmotic
General Approach present more dramatically with more pressure 275-295 mOsm/L) (Also known
Hyponatremia is the most frequent neurologic involvement. The management as pseudo-hyponatremia) occurs with
electrolyte disturbance and refers to differs based on the rate of change. high levels of osmotically inactive
when the serum Na is <135mEq/L. The substances.
management principles are as follow:
Differential Diagnoses
The type of hyponatremia has to be • Hyperlipidemia
1. Assess severity and acuity determined to narrow down the
• Hyperproteinemia (multiple myeloma
differential diagnoses. Based on serum
2. Determine type (based on plasma etc.)
osmolality, we distinguish between
osmolarity and volume status)
hypertonic, isotonic, and hypotonic Hypotonic hyponatremia (osmotic
3. Identify and treat the underlying cause hyponatremia. pressure < 275 mOsm/L) is subdivided
based on clinical evaluation of volume
4. Prevent complications Hypertonic hyponatremia (osmotic
status and urine sodium concentration.
pressure > 295 mOsm/L) occurs when a
Hyponatremia can be classified as large concentration of osmotically active A. Hypovolemic
substances pull additional water and
• mild (125-134 mEq/L), • Urinary sodium > 20mEq/L:
dilute sodium concentration.
renal losses (diuretics, salt-
• moderate (120-124 mEq/L)
• Hyperglycemia (for each 1mmol/L rise w a s t i n g n e p h r o p a t h y,
• severe (<120 mEq/L) in blood glucose, the serum sodium mineralocorticoid deficiency)
decreases by 0.3mmol/L)
The physician should carefully evaluate • Urinary sodium <20mEq/L:
the underlying cause before attributing • Administration of osmotic agents, such extrarenal (hypotonic fluids,
the symptoms to hyponatremia alone as as mannitol, glycerol, sorbitol, and GI, and third space loss,
mild and moderate hyponatremia are radiocontrast infusion sweating in CF patients)
often asymptomatic. Determining the rate
of change is important since rapid B. Euvolemic

279
• Syndrome of inappropriate • Hormone deficiency (hypothyroidism, develop acutely (<24h) or may
ADH secretion (SIADH) adrenal insufficiency) develop gradually and subtly over days.

• Drugs • Others Ask patients about their fluid intake


(polydipsia) and losses, comorbidities
• Hypothyroidism • Pulmonary
(e.g., heart and renal diseases,
• Pain, stress, nausea, Drugs that commonly cause malignancies), medications (e.g., thiazide
psychosis, water intoxication hyponatremia include ADH, nicotine, d i u re t i c s ) a n d a n y re c e n t i l l n e s s
MDMA, SSRI, sulfonylureas, opioids, (gastrointestinal diseases with vomiting or
C. Hypervolemic diarrhea), or use of recreational drugs.
barbiturates, NSAIDs, paracetamol,
• Urinary sodium > 20 mEq/L: carbamazepine, phenothiazine, TCA,
During the physical examination,
renal failure colchicine, cyclophosphamide, and
attention should be focused on
MAOI.
neurological examination and volume
• Urinary sodium < 20 mEq/L:
status. The latter can be assessed by
CHF, nephrotic syndrome, History and Physical
checking skin turgor, capillary refill, and
liver cirrhosis Examination Hints
whether pitting edema, raised jugular
The signs and symptoms of
If SIADH is suspected, evaluate for its venous pressure, and orthostatic
hyponatremia are usually vague and
cause. The acronym MADCHOP lists the hypotension are present. The signs of the
nonspecific. Patients may present with
causes of SIADH: common causes of hyponatremia, such
nausea, vomiting, anorexia and general
as cirrhosis and heart failure, should be
• Malignancy malaise. Pay attention to neurologic signs
carefully elicited.
including a headache, confusion,
• ADH secretion (ectopic) l e t h a r g y, s e i z u r e s , a n d c o m a .
Emergency Diagnostic Tests
Hyponatremia can also manifest as
• Drugs (e.g., SSRIs, ecstasy) and Interpretation
muscle cramps, ataxia, hemiparesis, and
If hyponatremia is suspected, check the
• CNS disease focal weaknesses. Symptoms may
serum sodium level (Normal reference:
135-145 mEq/L) and serum osmolality
280
(Normal reference: 275-295 mOsm/L), preferably at the bedside if s t i l l s y m p t o m a t i c . Additional doses are not advised
available. Serum osmolality can also be calculated by using the given the risk of overcorrection. If 3% NaCl is not available,
following formula: consider using NaHCO3 solution.

Osmolality (in mmol/L) = 2 Na + Glucose + Urea If the patient is stable, evaluate the patient’s volume status to
determine the management strategy.
After the type of hyponatremia, Checking the urine sodium and
osmolality to evaluate the etiology further. Note to monitor these Hypovolemic patients benefit more from increasing volume rather

values after treatment for hyponatremia is initiated. than sodium correction per se. If the patient’s serum sodium is
<130 mEq/L, Ringer’s lactate solution (sodium concentration =
If pseudohyponatremia is suspected, verify if the serum sodium 130mEq/L) may be used. It has the advantage of treating
measure is affected by protein and lipid concentration. If this is concurrent hypokalemia if present – correcting this often
the case, add serum protein and lipid levels to exclude improves serum sodium. Otherwise, use Normal Saline (sodium
pseudohyponatremia. concentration = 154 mmol/L) for volume correction.

Exclude other electrolytes abnormalities, especially potassium. Sodium concentration in different solutions
These disorders often co-exist in conditions such as adrenal
insufficiency. • 0.9% NaCl: 154 mmol/L

Point of care ultrasound can be used to evaluate volume status. • Ringer’s Lactate: 130 mmol/L
Nerve sheath diameter can be measured using ocular ultrasound
• 0.45% NaCl: 77 mmol/L
if cerebral edema is suspected.
• 5% Dextrose in water (D5W): 0 mmol/L
Emergency Treatment Options
Initiate therapy with 100 mL of 3% NaCl over 10-15 minutes in In general, patients with euvolemic and hypervolemic
patients with severe acute hyponatremia (sodium < 120 mEq/L) hyponatremia should be fluid restricted. For patients with SIADH
and neurological symptoms, such as seizures, confusion or and congestive heart failure, consider adding loop diuretics. For
coma. At this point, recheck the serum sodium. Give a second those with glucocorticoid deficiency, hydrocortisone should be
dose of 3% NaCl if hyponatremia is still severe and the patient administered. In these three conditions, a vasopressin receptor
281
antagonist may be considered in consultation with the inpatient presentation can take place a f t e r 2 - 6 d a y s a n d m a y b e
specialists if the hyponatremia is refractory. irreversible. Symptoms range from ataxia and paresis to the
“locked-in syndrome.” As the prognosis is poor, care should be
The amount and speed at which sodium is corrected must be
taken to avoid this complication in the first place when correcting
determined before initiating replacement therapy. This can be
hyponatremia.
calculated manually or via several online calculators, such as
Medcalc. Diagram 5.1

The rules of 6s and 100s can be used to avoid over-correction.

• Rule of 6s:

• “Six in six hours for severe symptoms, then stop.” If


symptoms are not severe, then “six a day makes sense for
safety.” In other words, for patients with severe symptoms,
raise the sodium level by a maximum of 6 mEq/L over 6
hours. For those without severe symptoms, the maximum
is 6 mEq/L over 24 hours.

• Rule of 100s:

• If the output is more than 100 mL/hr, and the osmolality is


less than 100 mosm/kg H2O, consider administering IV
DDAVP 1 microgram to slow down the correction rate.

Rapid overcorrection is a risk factor for osmotic demyelination


syndrome. As it has a predilection for the pons, it is also
previously known as central pontine myelinolysis. Diagnosis is Treatment algorithm for hyponatremia (Conceived by Dr. Edward
Etchells). Please read the whole article for more information (link).
initially clinical, but later changes can be seen on the MRI. Clinical

282
Pediatric, Geriatric, Pregnant Patient, and Asymptomatic patients with mild hyponatremia from benign

Other Considerations causes can be discharged with advice. They should be followed
The most common cause of hyponatremia in the pediatric up by their primary physicians or referred to a specialist.
population is the gastrointestinal fluid loss (emesis or diarrhea)
and inappropriate rehydration with a hypotonic solution. Ingestion
of overly diluted formula and excessive water are other causes. References and Further Reading, click here
Diagnosis and treatment are as described above.

Exercise-induced hyponatremia, a form of dilutional


hyponatremia, may happen when patients consume excessive
water or hypotonic liquids during exercise. Endurance sports and
training, such as ultra-marathons, Ironman triathlons, and
intensive military training deserve particular attention. Ironically,
the patient may wrongly attribute the symptoms of lethargy
(caused by hyponatremia) to dehydration, prompting further fluid
intake. The management of these patients should follow the
schema given above.

If hyponatremia is an incidental finding in an otherwise


asymptomatic patient, water restriction can be advised until
voiding occurs, to avoid development of symptomatic
hyponatremia5

Disposition Decisions
Patients with neurologic symptoms should be admitted to the
ward or the ICU. Patients requiring excessive sodium correction
should be placed under close monitoring.

283
Section 5

Hypoglycemia

Case Presentation
by Rok Petrovčič A 75-year-old woman was brought to the emergency
department (ED) by her relatives for “not being her usual self”
for a day. She was on insulin therapy for her diabetes, but
otherwise healthy.

On examination, she appeared confused and disoriented. Her


vitals were as follow HR 95/min, RR 18/min, BP 141/85mmHg,
T 37.7°C and SpO2 99% on room air. Given her past medical
history, capillary blood glucose test was performed by the
bedside. It was 2.6 mmol/L (47 mg/dl), and hypoglycemia was
diagnosed.

She was given a bolus dose of intravenous glucose and much


to the relatives’ relief and amazement; she returned to her
normal behavior within 5 minutes. The patient herself reported
Audio is available here lower urinary tract symptoms with a low-grade fever for the

284
last two days. In addition, Critical Bedside Actions and •Cardiogenic Shock

blood investigation showed General Approach • Hypopituitarism (Panhypopituitarism)


Every patient who is critically ill or has
that her renal function had also any neurological derangement should • Inadequate intake of food
deteriorated significantly since have their blood glucose checked at the
• Insulin Resistance
her last primary care visit while bedside. Blood glucose level is akin to a
“vital sign” for any patient with • Insulinoma
continuing on the same insulin
neurological symptoms or signs, such as
regime. The patient was weakness, confusion, seizures or even • Poisoning

subsequently admitted to a coma. If the level of glucose is low (i.e.,


• Pseudohypoglycemia
below 3,5 mmol/L), the patient should be
general ward for further
promptly given glucose therapy, either • Renal failure
evaluation and management. orally or intravenously. The neurological
• Sepsis
d e fic i t s h o u l d r e v e r s e r a p i d l y i f
hypoglycemia is the only reason for the • Stress
deficit. The cause of the hypoglycemia
should be investigated, and the patient The above causes can apply to diabetic

monitored for recurrence of further patients as well. In addition, one has to

episodes. exclude inappropriate administration of


diabetic medication, especially insulin.
Differential Diagnoses However, physicians should keep in mind
Hypoglycemia can be iatrogenic or that suicidal attempt with oral anti-
secondary to an underlying disease diabetic agents can be more dangerous.
process. These include:
History and Physical
• Addison Disease Examination Hints
• Adrenal Crisis
285
Hypoglycemia must be excluded in any Emergency Diagnostic Tests T – Toxicologic/Trauma
patient with coma, altered behavior, and and Interpretation I – Infection
any other neurological symptoms or Venous or capillary blood is checked with
signs. Signs and symptoms of a glucose oxidase strip. If the level of P – Psychiatric/poisoning
hypoglycemia are combinations of glucose is <3.0 mmol/L, take a venous
neuroglycopenic and adrenergic effects: S – Stroke / shock
blood sample for formal blood glucose
level. The differential diagnosis and
• Neuroglycopenic symptoms are due to Emergency Treatment
clinical picture direct additional
low glucose levels in CNS: blurred Options
diagnostic tests. When an overdose of
vision, weakness, tremor, seizures, Patients with hypoglycemia should be
diabetic medications are considered,
paraesthesias, focal neurologic signs, placed in a monitored area. The means of
additional studies such as serum insulin,
and confusion. reversing the hypoglycemia depends on
C-peptide, cortisol, and glucagon may be
the patient’s mental status, ability to
• Adrenergic symptoms are tachypnea, indicated.
cooperate with oral intake, availability of
t a c h y c a r d i a , s w e a t i n g , h u n g e r,
If neurologic/behavioral symptoms persist intravenous access and medical and
headache, and anxiety.
after treatment with glucose, evaluate for medication history.
Ask the patient about the time of last concurrent causes of altered mental
• If the patient is conscious and can
meal, exercise, and alcohol consumption. status (mnemonic: “TIPS AEIOU”). A CT
cooperate with oral intake,
Detailed history on the use and dosage brain may be warranted.
administration of food or liquid rich in
regime of diabetic medication is needed if
A – Alcohol simple carbohydrate (e.g., a sugary
the patient has diabetes. For patients
drink, sugar, candies, glucose tablets) is
with hypoglycemia secondary to a drug E – Endocrine/Electrolyte/Epilepsy preferred.
overdose, such as with oral hypoglycemic
agents, suicide risk assessment has to be I – Insulin • If the patient is unconscious or unable
performed. to cooperate with oral intake and
O – Overdose/opioids/oxygen
intravenous access is available, give
U – Uremia 50mL of IV dextrose 50% (equivalent to

286
25g of dextrose) over a few minutes. A Pediatric, Geriatric, monitored area. Consider
second dose can be administered if the Pregnant Patient, and Other consultation with toxicologist and
patient’s mental status does not psychiatrist for patients who overdose on
Considerations
improve. Children should receive 5 mL/kg of 10% their diabetic medication.
glucose or 2.5mL/kg of 25% dextrose.
• If intravenous access is not available, Discharge criteria
Avoid using 50% dextrose in this
IM/SC glucagon 1mg can be given. The patient should only be discharged if
population as it may easily result in
Glucagon takes a longer time to the cause of the hypoglycemia is
thrombophlebitis.
normalize mental status (around identified and deemed benign, have fully
7-10mins), and its effect tends to be Up to half diabetic pregnant patients on recovered, taking well orally and have no
short-lived. As glucagon raises blood insulin will experience an episode of recurrence of hypoglycemic episodes
glucose by releasing the hepatic severe hypoglycemia during pregnancy. after a period of observation. Discharge
glycogen reserve, it is not helpful in Careful titration of insulin is paramount to advice should be given.
patients with depleted glycogen stores prevent recurrence of hypoglycemia while
(e.g., liver failure or chronic alcoholism) Referral
attempting to achieve optimal sugar
If discharged from the ED, the patients
control.
• For patients with sulfonylurea overdose, should be referred to their primary
commence therapy with IV dextrose physician or specialist to follow up.
Disposition Decisions
until the patient can tolerate orally. If
episodes of hypoglycemia recur despite Admission criteria
glucose therapy, consider the addition Patients with hypoglycemia generally
References and Further Reading, click
of SC octreotide 50-100 micrograms. require admission to an observation unit
here
Note that octreotide should only be or the general ward, for evaluation and
used for recurrent sulfonylurea-induced treatment of underlying cause and
hypoglycaemic episodes despite titration of diabetic medication.
glucose therapy.
Patients with unexplained or recurrent
hypoglycemia should be admitted to a

287
Section 6

Thyroid Storm

Case Presentation
by Shabana Walia A 68-year-old female with hypertension presented to the
emergency department (ED) with worsening of lower extremity
swelling for the last few months. She appeared to be
confused over the last three days according to her husband.
He also noted that she had a fever. She had intermittent chest
discomfort and was feeling “anxious.” She was compliant with
the prescribed antihypertensive (lisinopril and
hydrochlorothiazide). She used no tobacco or illicit drug. She
had a family history of hypertension and hyperthyroidism.

Her vitals at triage were as follows: BP 170/86mmHg, HR 136/


min, RR 18/min, Temp 40.2°C and SpO2 100% on room air.
She appeared agitated and flushed, with bilateral
exophthalmos and lid lag. Her thyroid was diffusely enlarged
with bruit noted. Her pulse was irregularly irregular. She had
pitting edema up to the mid-shin. Bilateral plantar reflexes

288
were 3+. The rest of the physical examination Free T3: > 30 pg/ml (2.5- 3.9)
was unremarkable.
Free T4: > 6 ng/dL (0.58-1.64)
Bedside ECG is below
Troponin: 0.10 (<0.04)
Image 5.2
Pro-BNP: 3,000 pg/mL (0-100)

A diagnosis of hyperthyroidism was made, and


she was evaluated for possible thyroid storm.

Her blood test results were as follow:

Normal CBC and renal function.

Calcium: 11.5 mg/dL (8.6-10.2)

Thyroid stimulating hormone (TSH) < 0.01 mlU/L


(0.34-5.6)

289
Introduction Initiate aggressive supportive care, •Encephalitis/meningitis
Thyrotoxicosis occurs when there is an including temperature control. Treatment
• Heat stroke
excess of circulating thyroid hormone in of the thyrotoxic state is aimed at
the body, whereas hyperthyroidism refers inhibition of thyroid hormone release, • Hypertensive encephalopathy
to thyrotoxicosis that arises from a inhibition of new hormone synthesis,
inhibition of peripheral conversion of T4 • Malignant hyperthermia
hyperfunctioning thyroid gland. Thyroid
storm, a true endocrine emergency, is the to T3, and lastly blockage of peripheral
• Neuroleptic malignant syndrome (NMS)
most extreme form of thyrotoxicosis. It beta-adrenergic receptors.
consists of a triad of severe hyperthermia, • Sepsis
Evaluate the patient for precipitants (e.g.,
cardiovascular dysfunction and altered
sepsis, noncompliance of anti-thyroid • Serotonin syndrome
mental state. Although it occurs in less
medications, trauma to the thyroid,
than 2% of patients with thyrotoxicosis, • Sympathomimetic overdose
radioactive iodine therapy, chemotherapy,
Emergency Physicians must maintain a
recent surgery, and molar pregnancy) and
high index of suspicion for thyroid storm History and Physical Exam
complications of the thyroid storm (e.g.,
because mortality approaches 80-100% Hints
high output cardiac failure, atrial As thyroid hormones act on almost every
if untreated. Prompt identification and
fibrillation). The underlying precipitant has cell in the human body, thyroid storm will
appropriate treatment can reduce the
to be addressed early and concurrently result in multi-organ dysfunction.
mortality to 15-50%.
with treatment for complications of the
thyroid storm. A thorough history and physical exam are
Critical Bedside Actions
keys to diagnosing thyroid storm.
A patient with suspected thyroid storm
Differential Diagnosis Patients often have a personal or family
should be placed in the resuscitation
Differentials of thyroid storm include: history of thyrotoxicosis. Initial symptoms
area. Evaluate the patient’s ABCs and
m a y b e v a g u e a n d n o n s p e c i fic .
establish intravenous access. Vitals signs, • Acute psychosis
Symptoms of weight loss, ravenous
including temperature, must be closely
• Alcohol or benzodiazepine withdrawal appetite, emotional labiality or irritation,
monitored.
and heat intolerance suggest
• Anticholinergic overdose
290
thyrotoxicosis. In a patient with prequel symptoms, Table 5.2 Criterion for Diagnosing Thyroid Storm
hypermetabolic state, and deranged vital signs, the diagnosis of
thyroid storm should be considered. CATEGORY SITUATION SCORE

While the differentiation between thyrotoxicosis and thyroid storm


Thermoregulatory 99-99.9 5
is a clinical one, the diagnostic criteria by Burch and Wartofsky Dysfunction 100-100.9 10
101-101.9 15
can help to identify a thyroid storm (modified in Table 5.2). A key 102-102.9 20
103-103.9 25
diagnostic criterion is severe hyperthermia, as heat production 104 30

becomes excessive and unregulated. Temperature above 38.5°C CNS Effects Absent 0
Mild-Agitation 10
is common, and can even exceed 41°C. Patients also present Moderate-Psychosis, 20
Delirium, Fatigue
with altered mental state and severe cardiovascular dysfunction. Severe-Seizures/Comatose 30

Gastrointestinal or Liver Absent 0


Dysfunction Moderate- Diarrhea/N/V/ 10
Other common physical exam findings of thyroid storm include Abdominal pain
Severe-Jaundice 20
sweating, tachycardia, hypertension with a widened pulse
pressure, anxiety, atrial fibrillation, tremors, exophthalmos (Image Cardiovascular Dysfunction Tachycardia 5
99-109 10
5.3), goiter, and hyperreflexia. Note that the tachycardia seen in a 110-119
120-129 15
thyroid storm is often out of proportion to the fever. 130-139 20
140 25

Congestive Heart Failure 0


Absent 5
Mild-Pedal edema 10
Moderate-Bibasilar rales 15
Severe-Pulmonary edema
Image 5.3
Atrial Fibrillation 0
Absent 10
Present

Precipitant History 0
Negative 10
Positive

Criterion for diagnosing thyroid storm, Modified from the original Burch HB,
Wartofsky L. Criteria: Life-threatening thyrotoxicosis: Thyroid storm.
Endocrinol Metab Clin North Am 22:263-277, 1993. Please read the article
for more information. 291
SCORING SYSTEM adds total points Emergency Diagnostic Tests consider a head CT scan to rule out a
based on patient’s history and physical and Interpretation precipitating or concurrent intracranial
exam: The following investigations are indicated process.
in patients with suspected thyroid storm:
• >45: Highly suggestive of thyroid storm Use the Burch and Wartofsky criteria to
CBC, renal function test, liver function
evaluate for the likelihood of thyroid
• 25-44: Suggestive of an impending test, thyroid function tests, metabolic
storm. If the diagnosis is suspected,
thyroid storm or thyrotoxicosis panel (including calcium), ECG, CXR.
treatment with the medications listed in

• <25: Unlikely thyroid storm based on While confirming the presence of Table 2 should be initiated. The primary
presentation thyrotoxicosis, thyroid function tests goals are blocking the peripheral effects
alone cannot be used to rule in or rule out of thyroid hormone, preventing the
  The patient in our case presentation synthesis of T3 and T4, and inhibiting the
thyroid storm reliably. Thyroid stimulating
above hormone (TSH) will be low or release of preformed thyroid hormone.
u n d e t e c t a b l e , w i t h e l e v a t e d f re e
• Temp: 30 Emergency Treatment
triiodothyronine (T3) and its prohormone,
• CNS effects: 10 thyroxine (T4). Hypercalcemia, elevated Options
Initiate supportive care expeditiously.
alkaline phosphatase, and hyperglycemia
• GI-Liver: 0 Administer intravenous fluid to correct
are other common lab abnormalities seen
volume depletion, supplemental oxygen
• Tachycardia: 20 due to bone resorption, bone remodeling,
for hypoxia and exter nal cooling
a n d g l y c o g e n o l y s i s , re s p e c t i v e l y.
• CHF: 5 measures for severe hyperthermia.
Abnormalities on ECG include premature
Acetaminophen alone is not helpful as the
atrial, premature ventricular contractions,
• Atrial Fibrillation: 10 hyperthermia in thyroid storm is not a
atrial fibrillation, or atrial flutter The CXR
central, hypothalamic regulatory problem.
• Precipitant History: 0 may show cardiomegaly and pulmonary
Cooling blankets, ice packs or even cold
vascular congestion – indicating heart
• TOTAL POINTS: 75 intravenous fluids can be used. In
failure. If global or focal neurological
patients with airway compromise, rapid
deficits are found, it is reasonable to
292
sequence intubation and paralysis should be considered with a
Table 5.3 Medical Treatment for Thyroid Storm
secondary aim of temperature control. Aspirin and other non-
MECHANISM EXTRA
steroidal anti-inflammatory drugs (NSAIDs) should be avoided as GOAL MEDICATION
OF ACTION CONSIDERATIONS
they can increase peripheral free T3 and T4 due to their protein
STEP 1: Block Propranolol: Beta blockade, Use propranolol
binding properties. peripheral 60-80mg PO q4hrs shorter half life with with caution in
adrenergic effects or 0.5-1 mg IV q1h esmolol those with signs
of thyroid hormone (slow infusion) and symptoms of
Propranolol has the congestive heart
Esmolol: additive mechanism failure, as this can
250-500micrograms of blocking cause cardiogenic
/kg IV bolus, then conversion of T4 to shock and collapse
50-100micrograms/ T3 in peripheral
kg infusion tissues Esmolol is a
selective B-1
blocker, thus can be
used in patients
with bronchospasm
or asthma

STEP 2: Prevent Propylthiouracil Central mechanism PTU also inhibits


synthesis of thyroid (PTU): 600-1000mg of PTU and MMI: peripheral
hormone PO initial dose, then Inhibits thyroid conversion of T4 to
200-250 q4h peroxidase. Thyroid T3 by inhibiting the
peroxidase oxidizes enzyme
Methimazole: iodide to create 5’deiodinase
20-30mg PO initial iodine, which is
dose, then 20-30 then added to
q6h tyrosine residues on
thyroglobulin,
Dexamethasone: creating T3 and T4
2mg IV q6h
Steroids decrease
Hydrocortisone: the conversion of
300mg IV initial T4 to T3 in the
dose, then 200 mg periphery
q8h

STEP 3: Inhibit Lugol’s Solution Decreases the Must be given at


release of thyroid (Iodine): 8 drops PO release of thyroid least one hour after
hormone q6h hormone from the PTU or it can
thyroid gland increase thyroid
SSKI: 5 drops PO stores by iodinating
q6h tyrosine

Table produced by author.

293
Pediatric, Geriatric, Pregnant Patients And Admit the patients with partially-controlled thyrotoxicosis

Other Considerations symptoms for further management. Patients with mild controlled
Pediatric patients may not present with the classic symptoms of symptoms and stable vital signs may be discharged with close
hyperthyroidism. They may only demonstrate jitteriness, agitation, follow up with the primary care physician or endocrinologist.
restlessness without typical ophthalmologic or other systemic
References and Further Reading, click here
findings.

The presentation in geriatric patients may be even more subtle.


These vague complaints may be easily dismissed by the patients
themselves or care provider, or even mistaken as normal aging.
Alternatively, they may also present with primarily cardiovascular
complaints, such as palpitations, chest discomfort or signs of
heart failure. Therefore, a high index of suspicion should always
be maintained in the elderly.

Pregnancy can trigger thyrotoxicosis in up to 10% of patients,


even up to 6 months postpartum. High estrogen and human
chorionic gonadotropin level have weak TSH-like effects and
contribute to an overall hyperthyroid state. Propylthiouracil (PTU)
is preferred over methimazole (MMI) in this population, as it does
not cross the placenta as readily. Methimazole is also associated
with choanal atresia and aplasia cutis. Both drugs are FDA
category D in pregnancy.

Disposition
Admit all patients with thyroid storm to the ICU or high
dependency unit for ongoing monitoring and treatment.

294
Chapter 6

Selected
Environmental
Emergencies
Section 1

Burns

Introduction mechanism is burns. This chapter will


by Rahul Goswami The skin is the largest organ in the l o o k a t t h e d i ffe r e n t t y p e s o f
body. Its physiological purpose is to mechanisms along with their
protect the body contents from foreign assessment and management.
pathogens and maintain
thermoregulation. Thermal Burns
When excessive heat is applied to the
The skin layers consist of: skin, the destruction of the tissue is
known as a thermal burn. This may
• Epidermis – outermost layer
involve inflammation and healing but if
containing epithelial cells
enough heat is applied, there is
• Dermis – middle thicker layer coagulative necrosis and that area of
comprising connective tissue, nerve skin is dead. The type of burn depends
endings, blood vessels and sweat on many factors and only careful
glands. assessment with appropriate treatment
can lead to good outcomes.
• Subcutaneous tissue – lies just above
the muscle layer and contains fat
cells and connective tissue.

Damage to the skin can be temporary


or permanent, and a common

296
• Estimation of the area can be • Genital bur ns – fertility
Image 6.1
done in the following ways: consequences

• Rule of Nines. • Facial burns – cosmetic and


psychological sequelae
• Palm area estimation – This
utilizes the fact that the palm • Ocular burns – lasting visual
(including fingers) of patients is impairment
a p p ro x i m a t e l y 1 % o f t h e
• Airway burns or inhalational
patient’s body surface area.
burns – this requires eventual
Estimate how many “palms” is
Assessment airway protection as the
burnt to get the approximate
• Depth swelling will occlude the
total burn surface area % (or
ability to breathe in a few
• How deep the injury is can be TBSA). This is especially useful
hours
estimated from the surface in children.
condition as well as the • Special consideration in the
• Pediatric assessment is more
mechanism. assessment of burns in
difficult as their surface area is
children must be given to
• Area unlike adults. This link is
Non-Accidental Injury (NAI)
helpful.
• The area affected is a good or child abuse. Certain
predictor of outcomes. Here is a • Location patterns of burns are
calculator to remind you of the suspicious in nature and if
• certain areas of the body are more
importance of surface area burnt seen, should alert the
susceptible to edema and
for survival. Most calculators are physician to the safety of the
constriction and hence aggressive
only applied to 2nd and 3rd- child. Examples of these
management is warranted.
degree burns. patterns include:
Examples include:

297
• Glove or stocking
Table 6.1 Burn Degrees
distribution indicating
forced immersion EXAMINATION FINDINGS
DEGREE EXAMPLE
OF SKIN
• C i g a re t t e m a r k s o r
other implement 1st degree or Red, painful Sunburn
contact marks superficial

2nd degree is divided into: Sometimes blistered, painful, Scald or flash burn
Management - superficial partial thickness moist
1. First aid – the primary treatment is - deep partial thickness
cooling the burn area with running 3rd degree or Dry, white, insensate, leathery Flame or immersion
water. This not only gives pain relief full thickness and decreased sensation
but also halts the thermal coagulative
4th degree or Damage extends to fat, Chemical or electrical
process and prevents more area of deep full thickness muscle or bone
skin being damaged. Following this,
burns should generally be wrapped in treatments, applications, and • Give 1/2 of the total in 1st 8 hours,
dry cling film or dressing before dressings available. and then give 2nd half over next
transport to a medical facility. This 16 hours
reduces pain from surrounding clothes 3. Fluids – When skin is lost, a large

and wind. The author was once asked amount of fluids may leak into the • Hartmann’s solution is the ideal
to explain minor injuries including injured space and also evaporate. replacement fluid

burns and this is the video for it. Regimens may vary but a good rule to
• Pediatric patients require even
follow is the Parkland formula. Here is
2. Analgesia – simple oral analgesia more specific control of fluid
an online calculator for this. In
(paracetamol or NSAID) followed by replacement and have their own
essence, give in mls:
opioids if need be. This is the most formulae.

important step in hospital. This is a • 4 x weight of patient x TBSA burnt


good video for the different types of (%)

298
4. Specific dressings and burns ointments have changed over the • Airway burns (if the unit has an ICU built in)
last decade. Here’s a good, updated site for such information.
Examples of units are in these links (1, 2)
5. Blister management – whether to remove or puncture the
blister or leave it alone has been controversial for a long time. Electrical Burns
The current consensus is to de-roof them. Electricity contains a large amount of energy, and when passing
through the body, it causes damage along its path and usually
6. ATT – Anti-tetanus toxoid for anyone not immunized or whose manifests itself as a burn at the entry and exit points.
last dose was more than 10 years ago.

Referral to the Burn Unit Image 6.2 Image shows the entry wound of an electrical
The burn unit is a specialized unit which deals specifically with injury
burns patients’ needs (dressing changes, escharotomy,
debridement, physio, etc.). They are essential units which provide
specialist care and dedicated rehabilitation.

Criteria for referral vary between countries but in general patients


who require referral or review include:

• Total Burn Surface Area (TBSA) greater than 10% in adults

• TBSA greater than 5% in children

• Full thickness burns of any area

• Circumferential burns of limbs or chest/abdomen

• Facial burns or inhalational burns

• Scrotal/genital burns

299
the body at small points but does
Image 6.3 Image shows the exit Image 6.4 Image shows urine of
wound on the leg. tremendous damage along the route of the patient who has
the current all the way to the exit point. rhabdomyolyis after electrical
Thus a careful scan of the body and an burn.
ECG must be done to elicit damage.
Muscle damage leads to breakdown
called rhabdomyolysis and this, in turn,
leads to renal failure and multi-organ
failure if not treated promptly. Shoulder
dislocations from being jolted from the
There are a few modes of injury: electricity and head injuries are also
common.
• Power points – The amount of damage
caused by home/industrial mains Lightning strikes have far more energy
depends on contact time, voltage and transmitted than household mains.
current. Lichtenberg flowers are classically (but
rarely) seen in lightning strike burns. Here
• Lightning strikes, on the other hand, are
rare but cause devastating injuries. lies a good write-up and picture of it.

• A new phenomenon these days might


Management
be TASER burns in patients shot by
ED management includes:
police.
1. Analgesia – simple oral tablets to IV
Assessment opioids depending on pain score
A similar approach to assessment as
thermal burns but beware of the lack or 2. Fluids – to prevent and treat renal
area affected. Electrical energy arcs into failure

300
3. Bicarbonate – in cases of rhabdomyolysis, alkalinization of the Chemical/ Radiation Burns
urine will help draw products of muscle break down out of the
body. Image 6.6 Image shows chemical burn on the hand.

4. Treat other injured joints or organs sustained from the jolt when
shocked (e.g., shoulder dislocations)

Image 6.5 Escharotomy in a severely injured patient with


high voltage electricity

Image 6.7 Image shows chemical burn on the foot.

301
Like most toxic ingestions and lamps. A good explanation of radiation Management
exposures, the extent of injury depends damage and risks can be found in this Probably the most important
on: pdf. interventions to get right are the first aid
processes:
• Type of toxin Assessment
A similar calculation for TBSA burned can • Removal of offending agent
• Concentration of toxin
be used as in thermal burns shown in the
• Irrigation with water to dilute and
• Length of exposure to it section above. However, the location of
neutralize
the burn and concentration of the agent
• Immediate decontamination and first have far more impact. • Transport to a medical facility
aid
For example: ED management includes;
A good explanation of chemical burns
from common household items can be • Alkali burns of the cornea can result in 1. Analgesia – IV opioids most commonly
found here. permanent blindness as they are deep dermal burns

Although radiation burns sound very • Hydrofluoric acid burns can result in 2. More irrigation of the affected area if
ominous, they are handled similarly as all systemic fluorosis which is life- symptoms or pain persists
other burns. The myth that exposed threatening
3. Irrigation of the eyes is of utmost
patients can contaminate the whole
• Corrosive burns to the esophagus can importance and the only way of
hospital is unjustified and applies to only
cause permanent swallowing difficulties removing the offending agent.
a specific scenario involving “dirty
bombs.” Hence a thorough examination is more 4. Local anesthetic also helps during
important than any lab or imaging test. irrigation. A good video of this is here.
Most common radiation burns are
Geiger counters can be used to detect
actually from medical facilities such as 5. Anti-tetanus toxoid
contaminated patients exposed to
cancer treatment or x-ray imaging
radionuclides. This needs to be done by
centers as well as tanning booths and 6. Specific antidotes:
experts in the field.

302
• Hydrofluoric acid – this acid is one Inhalation injuries •A c u t e R e s p i r a t o r y D i s t r e s s
of the most corrosive known and Syndrome (ARDS) or acute lung injury
its systemic effects lower calcium
Image 6.8 Smoke inhalation
to a life-threatening level. Hence
• Particulate matter in smoke causes
calcium is essential not only for
airway and bronchial inflammation
cardiac stability but also for
which can lead to pulmonary edema,
analgesia. Calcium can be given in
bronchospasm, and even ARDS. Signs
oral, topical gel and IV forms.
of such injury include soot in the
• Radionuclide poisoning – oropharynx, singed nasal hairs,
decontamination is once again hoarseness, stridor or confusion/
dealt very comprehensively in this agitation. Watch this video.
pdf.
Chemical inhalational injury
6. Referral to the burn unit When patients are thermally injured, the
environment they were in can cause harm
as well. Here are two of the most
common inhalation injuries that patients
trapped in burning environments receive.
Thermal inhalational injury
• Airways can become swollen due to • CO poisoning – This gas is a by-
inflammation. Prompt airway protection product of combustion. Symptoms can
measures need to be initiated before range from confusion to coma. The only
the swelling becomes too severe way to detect it is a high degree of
(impairs breathing) suspicion and via arterial blood gas.
The treatment is 100% oxygen. In some
• Tracheobronchial edema / inflammation
cases with neurological symptoms and

303
coma, hyperbaric oxygen therapy is recommended.

• CN poisoning – This gas is produced when furniture and other


plastics are combusted (usually house fires). It is a pulmonary
irritant and can cause tissue hypoxia which may lead to cardiac
arrest. Treatment is decontamination, oxygen and specific
antidotes (not in the scope of this section).

References and Further Reading, click here

304
Section 2

Drowning

Case Presentation
by Ana Spehonja A 6-year-old previously healthy male was brought to the
emergency department (ED) after he fell into a freshwater lake
while playing on the dock. Eyewitnesses found his body
floating face down in the water. He was unaccounted for
10-15 minutes. They started basic life support right after they
pulled him out of the water. He was cyanotic, apnoeic,
pulseless with fixated and dilated pupils and tympanic
temperature of 26,7 at arrival to ED. CPR was continued. After
established airway and assessment for other injuries, they
began to warm him up. ABG showed combined respiratory
and metabolic acidosis with severe uncorrected hypoxemia.
15 minutes post mechanical ventilation and the return of a
spontaneous heart rate with adequate blood pressure 110/67,
SpO2 was 96%, pupils small and reactive to light and the
tympanic temperature was 32,2°C. There was no spontaneous

305
respiratory effort. ABG analysis showed d r o w n i n g a s t h e ˝ t h e process of experiencing respiratory
impairment from submersion/immersion in liquid. Drowning
uncompensated metabolic acidosis with
outcomes are defined as death, morbidity, and no morbidity.˝
corrected hypoxemia. He was stable enough for
transfer to Paediatric ICU. After two days, ICU Drowning happens due to closed glottis, hypoxia,
reported him to be stable with normal and cardiac arrest.
temperature. People don’t inhale water; it gets into the lungs
Definition and Terminology later.

Previous Definitions Immersion syndrome is syncope resulting from cardiac


Drowning – submersion injuries that resulted in death in under 24 dysrhythmias on sudden contact with water that is at least 5C
hours lower than body temperature. Vagal simulation leads to asystole
and ventricular fibrillation secondary to QT prolongation after a
Near drowning – those that survived over 24 hours.
massive release of catecholamines on contact with cold water.
Dry drowning – a state caused by laryngospasm, followed by Loss of consciousness leads to secondary drowning. We can
convulsion, hypoxia that leads to loss of consciousness and prevent this from happening with wetting our face and head
death without entry of fluid to lungs. It accounts for 10-20% of before entrance into the water.
submersion injuries.
Drowning incidence is most common in toddlers and young
Wet drowning – is a state caused by the aspiration of water. This children as they can drown in toilets, buckets, and bathtubs. The
leads to dilution and washout of the pulmonary surfactant. The second are adolescents and young adults and third are the
result of this situation is diminished gas transfer across the elderly.
alveoli, atelectasis, and ventilation-perfusion mismatch.
The fluid medium in which a submersion happened has little
Currently, all submersion injuries are defined as drowning. In 2005 clinical relevance, pulmonary injury and hypoxia are caused by
World Health Organisation published a new policy defining the amount of water aspirated and the duration of submersion. In

306
both cases, the effect of the osmotic gradient on the very delicate I f t h e p a t i e n t i s n o t breathing give 5 rescue breaths
alveolar-capillary membrane increases its permeability and immediately, followed by 30 chest compressions and continuing
exacerbates fluid, electrolyte shifts and plasma. with 2 rescue breaths and 30 compressions until signs of life
reappear. Positive pressure bag-valve-mask ventilation should be
Rapid CNS cooling before significant cardiac dysrhythmia
administered.
provides cerebral protection in cold water submersion.
If the heart is beating give only breaths, not CPR. Victims with
In submersion victims amount of swallowed water is much
only respiratory arrest usually respond after a few rescue breaths.
greater than aspirated, as a consequence 60% of patients vomit
after a submersion event. Well-known complication with If the patient is spontaneously breathing, let him cough, place him
aspiration of gastric contents is pulmonary injury and increased in the recovery position and administer high-flow oxygen mask
possibility for acute respiratory distress syndrome. (15 liters of oxygen per minute).

Type of the water does not matter If it is possible, resuscitate in the water
When the patient is rescued alive is the clinical picture
(ventilation alone)
determined predominantly by the amount of water has been Endotracheal intubation and positive pressure ventilation are
aspirated and its effects. Osmotic gradient effects the very necessary if there is no recovery of spontaneous respiratory
delicate alveolar capillary membrane increases its permeability effort.
and exacerbates fluid, plasma and electrolyte shifts.
Obligatory transportation to ED is for the patients who have a loss
We should also consider the precipitants of submersion injury or depressed consciousness, an observed period of apnoea and
which may be drugs or ethanol intoxication, cardiac arrest, those who require a period of artificial ventilation.
hypoglycemia, seizure and attempted suicide or homicide.
Emergency Department Care
Prehospital Care First steps are assessing and securing the airway, providing
The most important thing is early CPR, as it optimizes the oxygen, determine core temperature and assisted ventilation as
outcome. This is the reason why there is a need to train necessary.
laypersons in CPR.
307
If the patient is in cardiac arrest you should follow ACLS • A v o i d s u c t i o n a s i t disrupts oxygenation and do not
guidelines. extubate early (lung injury may present later). Medications
should not be administered through the endotracheal tube.
The following management shows the steps when the patient is
not in cardiac arrest: C. Circulation

A. Airway • We are looking for the presence of significant dysrhythmias, QT


prolongation or ischemia with cardiac monitoring and an
• When the pulmonary examination is abnormal or there is
electrocardiogram.
oxygen requirement, admission or transfer to monitored bed is
needed. • If there is cardiopulmonary arrest or asystole when the patient
comes to ED, we should consider discontinuing resuscitation
• When the GCS is under 13 the patient should receive
efforts, because of profound neurologic handicaps.
supplemental oxygen and ventilator support if needed.
• There was not shown benefit in efforts to control cerebral
• The patient should be intubated when high-flow oxygen cannot
edema, with use of mannitol, loop diuretics, hypertonic saline,
maintain an adequate partial pressure of arterial oxygen.
fluid restriction and mechanical hyperventilation.
B. Breathing
• Continuous cardiac monitoring, pulse oximetry, temperature
• Arterial saturation should be between 92-96%. Treatment monitoring.
should be administered like ARDS: add PEEP, low tidal
Varieties of tests in the ED include
volumes, and permissive hypercarbia.
• Gastric tube, laboratory studies (frequent arterial blood gas
• Supranormal levels of positive end-expiratory pressure are
measurements)
beneficial to recruit fluid-filled lung units. Special consideration
should be taken to avoid lung over-distention and ventilate- • Chest radiography (initial chest radiographs may be
associate lung trauma. unremarkable, even in the setting of severe and evolving
pathologic process)

308
Risk Groups If ED resuscitation/CPR was required the patient should be
• Drowning is the leading cause of injury mortality in children 1 to a d m i t t e d t o t h e i n t e n s i v e c a re u n i t f o r c o n t i n u o u s
4 years of age. cardiopulmonary and frequent neurologic monitoring.

• Children can develop dilutional hyponatremia and seizures in The patient can be discharged after 4-6 hour observation period if
freshwater near-drowning. the Glasgow Coma Scale is 15, oxygen saturation over 94% on
room air and if pulmonary examination does not reveal rales,
• Children have hypothermia more quickly because of a lower rhonchi, wheezing, or retractions. We should warn the patient to
ratio of body mass to surface area. There was no shown benefit return if mental status changes, pulmonary symptoms or fever
in controlled hypothermia, barbiturate coma, and intracranial occur.
pressure monitoring.

Prognosis
There is no prognostic scale that accurately predicts long-term References and Further Reading, click here
neurologic outcome. There is documented normal neurologic
recovery even with fixed and dilated pupils, cardiovascular
instability, prolonged submersions and persistent coma.
Complete recovery within 48 hours is expected if there was no
need for cardiopulmonary resuscitation on the scene or in ED. If
there are no continuous neurologic and cardiovascular deficits
shown, the patients should recover completely. Those who
needed CPR in ED have a poor prognosis, because of significant
anoxic or ischemic insult to the brain and other vital organs.

Disposition Decisions
Apnoea, hypoxia, unconsciousness, dysrhythmia or abnormal
chest radiograph are signs for admission.

309
Section 3

Heat Illness

Case Presentation
by Abdulaziz Al Mulaik A 57-year-old male is brought to the emergency department
by EMS during Hajj. The patient as stated by the paramedics
was “found face down” in the street under direct sunlight,
where outside temperature is 45°C. Initial vitals are BP: 91/55,
HR 130 . O2Sat 95% on room air, RR 25 , Axillary temperature
39°C, and his Glucose check was 8 mmol/l. On examination,
the patient is not oriented nor alert but he moans to painful
stimuli, and he is maintaining his airway with no drooling. You
ask a member of your team to repeat temperature
measurement rectally, and he finds it to be 42.3°C. The rest of
your physical examination is unremarkable. You remove all of
the patient’s clothes and spray him with lukewarm tap water,
you then turn on a fan and raise the head of the bed and the
side rails. A continuous temperature probe is inserted rectally.
A cardiac monitor with pulse oximetry is connected, and

310
blood samples were drawn for laboratory testing. Basics of Heat Transfer is explained in this video.

After reaching a rectal temperature of 39°C, you Critical Bedside Actions and General
direct your team to dry him and cover him with a Approach
Heat stroke is a devastating disease that might have permanent
light bed sheet. On subsequent examination, the
sequelae if lifesaving interventions are delayed. It is defined
patient is conscious, alert and oriented. Vitals are simply as a failure of thermoregulatory mechanisms to cope with
HR 105, O2Sat 96% on room air, RR 20. Labs either internal heat production known as exertional heat stroke
reveal multiple abnormalities including respiratory (EHS) or external environmental heat, known as classic heat
stroke (CHS). Watch this video.
alkalosis and elevated liver enzymes. Your
disposition includes appropriate medical To diagnose a heat a stroke, the patient has to have a Central
Nervous System (CNS) impairment and core temperature of more
consultation and admission to a medical ward for
than 40°C. The spectrum of neurological abnormalities ranges
further management. from mild confusion to full-blown coma with GCS of 3. Core
temperature has to be measured and continuously monitored
using rectal or esophageal probes as peripheral measures of
temperature are unreliable and does not correlate with core
temperature.

The essence of heat stroke management is to “Cool first, then ask


questions!”, as it is a time-sensitive condition, where cooling
takes precedence over everything else including confirmation of
the diagnosis. ABCs are the way emergency medicine
practitioners approach every patient rightfully, so, to ensure that
all critical decisions are made in a timely fashion. Heat stroke is
not an exception to this role, as the disturbance in consciousness
could result in significant airway complications. A complete
311
airway assessment should be immediately performed when the will continue warming up to the new set point. No harmful effect
patient arrives in the emergency department while cooling is exerted directly by fever alone, and clinical focus should be
measures being set up. Other physical examination details are directed toward the cause of the fever.
important but should not delay cooling.
Heat stroke, on the other hand, is a failure of the thermoregulatory
Two important observations are worth taking note of: the first system to cool the body. Therefore, all antipyretics are potentially
relates to how difficult intravenous peripheral access might be harmful interventions as they work primarily on thermal set point
during early stages of resuscitating in a dehydrated victim which reduction which is already normal in heat stroke patients.
is going to be compounded by cooling as this will result in Reverting patients to a normal thermal level will reboot the
peripheral vasoconstriction. The second is a phenomenon related thermoregulatory system, hence the urgency of cooling.
to excessive watery diarrhea during cooling, and this mandates
Minor heat illnesses are expected in the right environmental
adequate preparation by the nursing and janitorial team. There
conditions. These illnesses are considered the milder side of the
are two possible explanations for this phenomenon; the first is the
spectrum of heat illnesses with heat stroke on the other side. A
compensatory vasoconstrictive changes to the splanchnic
list of these disorders with their classic presentations and
vasculature as blood pools peripherally for cooling. The second
recommended management strategies can be found below (Table
proposed mechanism is the multisystem failure caused by
1).
heatstroke which includes the gastrointestinal system where
epithelial cells shed and propagate an inflammatory diarrhea.

Differential Diagnoses
In a febrile illness, be it infectious or otherwise, circulating
pyrogens resets the normal temperature in the thermoregulatory
control center to a new set point above normal. The entire
thermoregulatory pathways in the body will work to achieve the
new set point; hence febrile patients will have behavioral changes
where they will seek warmer environments. Cooling febrile
patients by cold towels or showers has a mild effect as the body

312
by sweating is another risk factor for developing heat illness.
Table 6.2 The list of minor heat illnesses with their clinical
features and treatment Generally speaking, CHS is not common in geographical areas
where average temperature throughout the year is high, as
MOST PROMINENT
MINOR HEAT communities living in these places will develop behavioral tactics
CLINICAL TREATMENT
ILLNESS
FEATURES to avoid the heat.
Prickly heat Very pruritic vesicular Chlorhexidine lotion
rash on an Intense exercise, military training, sports competitions or
erythematous base prolonged labor might induce another type of stroke known as
Heat syncope Standing in heat for No specific treatment exertional heat stroke (EHS), which differs from CHS in laboratory
long time with no required
previous indices and long-term complications. A third less common type of
acclimatization to heat stroke is confinement hyperpyrexia where the patient is
heat
exposed to moderately high ambient temperatures for a long
Heat cramps Muscular cramps Oral 0.1% salt time.
AFTER working in solution
heat
Measuring body core temperature is perhaps the most important
Heat edema Swollen feet and No specific treatment
ankles in healthy required physical assessment, whenever heat illnesses are considered in
patients after the differential diagnosis of a given patient. Peripheral
standing in heat for
long time temperature measurements correlate poorly with core
temperature. The two methods of measuring core temperature
are either through the esophagus or the rectum with the latter
History and Physical Examination Hints
representing the majority of clinical practice. A common pitfall in
Situational awareness is a vital skill to emergency physicians, as
using rectal temperature is inserting the probe to an insufficient
one should be aware of high ambient temperatures and high
depth which will render readings to be inaccurate in both
humidity days as they are perfect conditions for classical heat
directions especially if ice packs have been applied to the groin.
strokes. Lonely elderly community members with low
Rectal probes, in general, have to be inserted 15 cm inside the
socioeconomic status are particularly vulnerable to CHS as they
rectum to mitigate the effects mentioned above, but
have poor access to good air conditioning and ventilation. Usage
manufacturers may recommend different depths.
of some medications which impairs adrenergic response to heat

313
Tachycardia and hypotension are Emergency Diagnostic Tests immersion can do this, and although
commonly seen and represent the and Interpretation this is theoretically the best cooling
physiologic response to heat, as E H S a n d C H S h a v e d i ffe r e n t method, it is clinically challenging as it
peripheral vascular resistance decreases derangements in laboratory studies with poses a risk of aspiration and renders
to allow the blood to be cooled at the some similarities. For instance, patient’s accessibility quite difficult.
surface of the skin, leading to a high respiratory alkalosis is a physiologic Convection, which is the thermal loss due
output status. This phenomenon might response to heat stress, which is to gas movement around the body,
explain other heat illnesses like heat profoundly represented in CHS, whereas combined with evaporation can achieve
syncope and heat edema. lactic acidosis is the prominent acid-base the similar speed of cooling to full body
disturbance in EHS. Moreover, liver immersion. This combination can be
Heat stroke is a multisystem disease
enzymes should be elevated in both EHS achieved by spraying the patient with
affecting almost every organ in the body.
and CHS with numbers in the tens of lukewarm water followed by fanning with
CNS effects might range from simple
thousands above normal cutoffs, to the warm air. Mist fans are very convenient
confusion to deep coma. Seizures, in
degree that their elevation is a cardinal and have the added benefit of their ability
general, are common and might be
diagnostic criterion, and their absence to fan multiple patients at once. Cooling
confused with shivering during cooling,
will render the diagnosis of heat stroke units with intermittent water sprays from
but both disorders need to be treated, the
unlikely. Another common difference all directions around the patient are costly
former for neural protection and the latter
between CHS and EHS is glucose level and not available in most hospitals, and
to prevent heat generation. Heat stroke
as it might be low in the latter but not the they have recently fallen out of favor,
patients might have derangements in
former. even in Hajj despite their availability,
their hemostasis represented clinically as
mainly due to safety concerns as they
melena, hemoptysis, conjunctival
Emergency Treatment limit access to patients.
hemorrhage or epistaxis. Prickly heat
Options
patients will have pruritic vesicles on an Invasive cooling procedures such as cool
The fastest way to transfer heat and to
erythematous base. These vesicles are IV fluids have not been proven to change
cool patients is through conduction,
sweat glands with blocked pores by the outcomes as their evidence remains
which is the direct transfer of heat
macerated stratum corneum. to be weak. On the other hand, thoracic,
between molecules. Full body water
314
bladder, rectal and peritoneal lavage and should take priority on any other A mass casualty incident of heat stroke
should only be used when all other diagnostic or therapeutic procedures. and heat exhaustion patients should be
measures fail. We suggest abandoning expected in preparing for a mass
Peripheral blood pooling is in the heart of
them even in the very sickest of patients, gathering event and mitigation measures
heat stroke pathology, so hypotension is
as a neurologically meaningful recovery is should be sought in advance. Public
common in these patients and fluid
highly unlikely. education to seek shade, drink enough
administration should be very judicious
fluids, use umbrellas and installing mist
as the blood pressure usually will pick up
Image 6.9 monitor showing the as the core temperature drops down. Image 6.10 The row of beds with
current vitals while the patient is Aliquots of 250 cc of crystalloids should mist fans in a sunstroke unit. A
cooled. cooling unit can be seen at the far
be used when fluids are needed, and
right.
repeated dosing should take place after
volume status assessments.

Special Patient Groups and


Situations
Pediatric heat stroke patients are usually
victims of confinement hyperpyrexia.
A protected airway should be maintained Attention should be paid if further
Image 6.11 Fiberglass grooved
in heatstroke patients, and efforts should interventions are needed to protect a beds with waterproof mattresses in
be taken to resist the urge to intubate child from abuse or maltreatment. a sunstroke unit.
them. Intubating solely for the low level of Geriatric patients, on the other hand, are
consciousness is ill-advised and will usually the victims of CHS. Emergency
expose patients to unnecessary adverse physicians should play their role in
effects of intubation. With that being said, advocating for closer community ties and
airway protection is of utmost importance socioeconomic support by appropriate
authorities for those patients.

315
pipes, venting fans, cooling stops in a path of a riot or a race are
few examples.

Disposition Decisions
Heat stroke patients usually require higher care upon admission
as their stability is not certain and further assessment of heat
stroke complications should take place in the hospital. Stable,
conscious heat exhaustion patients may be discharged with
education and close follow up. Minor heat illnesses should be
treated as a case by case scenario, but they rarely require
inpatient care.

References and Further Reading, click here

316
Section 4

Hyperthermia

Introduction The action of the pre-optic region of


by Puneet Sharma Hyperthermia is elevated body the anterior hypothalamus in response
temperature. It generally due to failed to infection sets the core temperature.
thermoregulation which occurs when a Hyperthermia, however, occurs when
body produces or absorbs more heat the body temperature rises without a
than it disseminates. Extreme change in the heat control centers.
temperature elevation then becomes a Watch this video.
medical emergency requiring
immediate treatment to prevent Causes of Hyperthermia
disability or death. Failure of thermal
Temperature greater than 37.5–38.3 °C
homeostasis and increased
heat production
(99.5–100.9  °F) depending on the
• Exercise-associated hyperthermia:- a
reference used is, hyperthermia, with
continuum of heat-related conditions
severe hyperthermia being greater than
due to environmental heat and
40°C(104°F). Different sources have
exercise.
different cut-offs.
• Heat stress/cramps
Hyperthermia differs from fever in that
the body’s temperature set point • Heat exhaustion
remains unchanged. When the core
temperature is set higher, fever occurs. • Heatstroke

317
• Seizures • Status epilepticus, linked to the RYR1 gene located on
chromosome 19q. Triggering drugs cause
• Agitation • Stroke,
a release of sarcoplasmic reticulum Ca2+.
• Uncoupling of oxidative • Brain trauma, Resulting ca2+ stimulated glycolysis,
phosphorylation- e.g., Salicylate muscle contraction, uncoupling of
• Neoplasms, oxidative phosphorylation leading to
overdose.
hyperthermia. Drugs causing MH are
• Acute intermittent porphyria,
• Hepatic Metabolism stimulation-e.g - volatile inhalation halogenated
sympathomimetic drugs • Tetanus, a n e s t h e t i c s a n d m u s c l e re l a x a n t
suxamethonium.
Hyperthermia secondary to • Thyroid Storm
other processes Pathophysiology
• Neuroleptic Malignant Syndrome(NMS) • Heat stroke,
– link The usual body temperature of humans is
• Sepsis.
between 36° C and 37.5°C.
• Serotonin Toxicity/syndrome(SS) –
• SSRI toxicity and other drug toxicities,
Serotonin TOXICITY – link When Core Body temperature (Rectal
• pheochromocytoma. temperature/esophageal temperature) is
• Malignant Hyperthermia (MH) – link greater than about 41.5°C it results in:
Malignant Hyperthermia (MH)
Differential Diagnoses Incidence is about 1:10000-15000. All • Progressive denaturing of number of
There are multiple differentials to the races are affected. vital cellular proteins.
cause of hyperthermia. A good history
from the patient (if possible), carers or Mortality rates have fallen from 70-80 % • Failure of vital energy-producing
relatives is crucial to the diagnosis. Few to 2-3 % due to increase awareness, process in the cells like oxidative

important differentials to consider in ED monitoring standards, and Dantrolene. phosphorylation and failure of enzyme

are: function.
Genetically inherited disorder (autosomal
• Central nervous system infections, dominant). About 70% of families are

318
• Loss of cell membrane function with • Failure to achieve muscle relaxation There is Dopamine depletion/
increasing permeability. following succinylcholine, e.g., master dopamine receptor (D2) blockade in the
spasm impeding intubation and hypothalamus, nigrostriatal pathways and
Tissues most at risk are:
persisting for 2 minutes. spinal cord which leads to increased

• Vascular endothelium muscle rigidity and tremor via


• Signs of increased metabolism:
extrapyramidal pathways.
• Nervous tissue Tachycardia, Tachypnoea, dysrhythmia,
increased CO2 production noted on the Hypothalamic D2 blockade leads to
• Hepatocytes end-tidal CO2 monitor elevated temperature set point and
impairment of heat dissipation. This is an
At organ level hyperthermia manifests • Metabolic acidosis, rhabdomyolysis
idiosyncratic reaction to neuroleptic
as : and hyperthermia and DIC.
agents. All classes of antipsychotics have
• Rhabdomyolysis • Signs and symptoms may be delayed been associated with neuroleptic
or may reappear after successful malignant syndrome, most frequently in
• Electrolyte disturbance patients taking haloperidol and
treatment.
chlorpromazine. It occurs in response to
• Renal and Liver failure
Neuroleptic Malignant a single agent and may occur in
• Cardiovascular dysfunction Syndrome(NMS) therapeutic dosages. It maybe dose-

• Acute Pulmonary edema with ARDS


and Serotonin Toxicity/ related and more commonly seen in

(acute respiratory distress syndrome) syndrome(SS) patients on higher dose, depot


Occurs due to increased motor activity neuroleptics.
• Disseminated intravascular coagulation and central resetting of hypothalamic
Clinical Features
( DIC) thermostat
Usually develops in patients who have
• Neurological damage Neuroleptic Malignant
recently started a neuroleptic treatment or
Syndrome(NMS)
Clinical Features have recently increased the dose. The
onset of the symptoms is 4-14 days after
319
the first day of therapy; most of the cases • Diaphoresis ,sialorrhea ,tachycardia ,tac diethylamide (LSD), mescaline-
occur within 10 days. Usually associated hypnea, respiratory containing cacti (peyote and others).
with almost all antipsychotics and also in distress ,hypertension / labile blood
• Direct 5HT release from stored vesicles
patients in whom dopaminergic agents pressure ,hypoxia.
— Amphetamines, MDMA, cocaine,
have been withdrawn (e.g., in Parkinson
Good drug history may help to reserpine, levodopa, MAOIs(monoamine
’s). Similar to Serotonin syndrome, there
differentiate between the NMS and oxidase inhibitors), codeine,
is a latent period of several hours to days.
Serotonin syndrome. dextromethorphan, pentazocine.
Four Classic Signs
Serotonin syndrome • Increased availability of 5HT precursors
1. Hyperthermia >38° C There is excess CNS (Central Nervous — L-tryptophan.
System) Serotonin (5HT- 5Hydroxy
2. Severe Muscular rigidity (typically “lead • Decreased 5HT reuptake — SSRIs(slow
Tryptamine) due to multiple mechanisms
pipe” rigidity) serotonin reuptake inhibitors),
based on different 5HT receptors in the
trazodone, nefazodone, venlafaxine,
3. Altered mental state brain.
T C A s ( Tr i c y c l i c a n t i d e p re s s a n t s ) ,
4. Autonomic instability SS is most often caused by simultaneous dextromethorphan, tramadol,
ingestion of 2 or more serotonergic meperidine, (, Hypericum species (St.
There is a broad spectrum of clinical J o h n ’s w o r t ) , a m p h e t a m i n e s ,
medications.There may be a recent
illness; carbamazepine, methadone, linezolid.
history of dose increase. Excess
Diaphoresis,Pallor ,Dysphagia ,Dyspnea ,
serotonergic activity can be precipitated
Tremor ,Incontinence ,shuffling gait, • Decreased 5HT degradation — MAOIs,
by any of the following mechanisms and
agitation ,delirium progressing to lethargy, St. John’s wort.
drugs (ones in bold seen commonly in ED
stupor, coma
as overdose): Clinical Features
E x a m i n a t i o n fin d i n g s s h o w i n g
• Direct 5HT-receptor stimulation — CNS, Autonomic and Motor Dysfunction
autonomic dysregulation include: Buspirone, triptans, lithium, related features
carbamazepine, lysergic acid

320
• Agitation, anxiety, confusion, decreased picture, and exclusion of alternative AKI(Acute Kidney Injury) and
level of consciousness, seizures diagnoses. Drug history is very important rhabdomyolysis.
and clinical suspicion is paramount.
• Clonus, Hyperreflexia, Hypertonia, • Cardiac enzymes.
Investigations are directed towards the
Incoordination, Myoclonus, Tremor
exclusion of other causes of pyrexia, e.g., • C h e s t X - r a y - To i n v e s t i g a t e
• Diaphoresis, Diarrhoea, Hypertension, sepsis and other disorders complications and rule out the
Hyperthermia, Tachycardia differential diagnosis.
Investigations are done to rule out
• mydriasis, piloerection, and muscular complications and guide treatment. • Specialist investigation:
rigidity
• Serum electrolytes- to check imbalance • Muscle biopsy using in-vitro
• CVS features include sinus tachycardia, and supportive treatment contracture test (IVCT) which is
flushing, hypertension, and hypotension the gold standard for MH
• Creatinine Kinase – Guides treatment of
(rare). diagnosis. This is done in
Rhabdomyolysis.
specialized MH centers. 8 to 10
• Citalopram causes dose-dependent QT
• Serum Glucose – Rule out hypo/ muscle specimens are taken and
prolongation.
hyperglycemia as the cause of altered considered positive if muscle
Develops after a latent period, ranging mental state. contracts to halothane and/or
from few hours to several days. Most caffeine
• ECG- arrhythmia, electrolyte
patients are mildly affected, but the
abnormalities. Treatment
disease spectrum is very broad. Most
cases resolve within 24-48 hours after • Urine- Toxins Supportive and cooling
withdrawal of the precipitating agent. measures for hyperthermia in
• Multiorgan dysfunction workup
general
Workup • Clotting screen- coagulopathy • Evaporative cooling- Remove all
Diagnosis of hyperthermia disorders is clothing, and spray the patient with
(DIC)LFT’s, Renal functions-
based on a detailed history, clinical tepid water while blowing air with a fan.
detect complications such as

321
Areas with increased vascular beds, • Severe SS: Neuromuscular •Bromocriptine as dopamine agonist
e.g., neck, axillae, groins should be paralysis should be considered can be given orally/NG tube, 2.5-10mg
asked with ice packs. early especially in cases with a TDS.
low GCS.
• Iced water immersion: – Can cause Malignant Hyperthermia
awkward patient access and difficulty in • Antiserotoninergic drugs: • Avoiding the triggering agents prevents
monitoring. Not very practical in the MH.
• Chlorpromazine- 12.5-50mg
Emergency Department.
IM/IV • Using inhalation agent free machines
• Invasive methods: Cold IV fluids, urinary during anesthesia.
• C y p ro h e p t a d i n e - 4 - 8 m g
bladder lavage, peritoneal/pleural
orally 8 hourly. • Dantrolene Sodium- Inhibits the release
lavage with cold fluid.
of calcium from the sarcoplasmic
Neuroleptic Malignant
Serotonin Syndrome( SS) reticulum.
• Mild cases- May need observation in
Syndrome
• Benzodiazepines for anxiety and • 2.5mg /kg IV initially repeated
ED for a few hours and safely
agitation. every 15 minutes to maximum
discharged if asymptomatic.
30mg/ kg.
• Stop all neuroleptics
• More serious cases would need
• AAGBI guidelines for treatment of MH
supportive treatment and • Correct volume depletion and
poster link
pharmacological therapy with hypotension with intravenous fluids
observation and treatment for
• Reduce hyperthermia (see above)
Prognosis and Disposition
complications in ICU.
• Early intensive care referral is indicated.

• Pharmacological Therapy • Alkalinization of urine with sodium


• Prognosis is worse with complications
bicarbonate for prevention of renal
and multi-system failure.
• Mild SS- No treatment is needed failure following rhabdomyolysis.
or small doses of
• Mortality remains high in this group if
benzodiazepines. untreated.
322
• Malignant hyperthermia: Modification of anesthesia in the future
with inhalation agent free machines and no use of
suxamethonium. Family members should be tested for
susceptibility.

References and Further Reading, click here

323
Chapter 7

Selected
Gastrointestinal
Emergencies
Section 1

Acute Appendicitis

Case Presentation
by Ozlem Dikme A previously healthy 22-year-old male was brought to the
emergency department (ED) with recently-started abdominal
pain. He had not eaten anything since that morning due to
loss of appetite. He was nauseated and vomited three times.
His abdominal pain started around the umbilicus and
epigastric area. His pain increased as it moved towards his
right lower quadrant (RLQ). The maximum pain was felt on the
right iliac fossa. He had not taken any medication. His social
history revealed that he was non-drinker, non-smoker and did
not use any illicit drugs. His diet mostly consisted of
carbohydrates. The past and family histories were
unremarkable. His blood pressure was 120/70 mmHg, pulse
rate was 100/min, the temperature was 37.8°C (100°F), and
respiration rate was 22/min. Physical examination showed
Audio is available here normal bowel sounds, tenderness and voluntary guarding,

325
particularly over the right iliac Can you name the finding in the given diffuse abdominal infection, systemic
ultrasound in this video? infection, even sepsis/septic shock can
fossa. The costa-vertebral
be another priority for the physician. After
angles were not tender. Oral Introduction the ABC evaluation, focused
About 7% of the population develops
intake was stopped, gastrointestinal and pelvic orientation
appendicitis in their lives. Males are follows. Depending on the patient needs,
intravenous (IV) catheter was affected 1.4 times higher than females, critical actions necessary in the initial
inserted, blood and urine tests and teenagers more than adults (3:2). The ABC evaluation can be applied. However,
were planned, and fluid therapy incidence rises gradually from birth, placing IV catheters, starting fluid therapy
peaks in the late teens, and declines in are the priority in most of the cases. Oral
was started. The urinalysis was
the elders. It occurs in all age groups but intake should be stopped. Pain
normal. White blood cell (WBC) most frequently between the age of 10 medication and application of antibiotics
count was 14,500 with 89% and 30. Prevalence is higher in countries may be considered in the early phase
with diet habits low in fiber and high in
polymorphous and 11% depending on the patient situation.
refined carbohydrates. Low dietary fiber
lymphocytes. The causes fecalith formation and obstruction The possibility of acute appendicitis must
ultrasonography (USG) showed of the appendicular lumen. be explained, and the patient’s approval
should be obtained for further steps. The
a non-compressible tubular
Critical Bedside Actions and evaluation should include laboratory tests
structure of 9 mm in diameter General Approach and imaging. Count blood cell (CBC) and
at RLQ. He admitted to the The general approach to a patient with c-reactive protein (CRP) are generally not
possible acute appendicitis must start specific to diagnose, but they may be
surgical ward with the
with the patient stabilization. Fortunately, useful to confirm or exclude the
diagnosis of acute the most of the patients come with stable differential diagnoses. USG or computed
appendicitis. clinical presentation except pain. Some tomography is possible imaging
patients may present late. In this situation modalities.
perforation is a possibility. Therefore

326
History and Physical peritoneal irritation are triggered RLQ •Inflammatory Bowel Disease (Crohn
Examination Hints pain with palpation of the left lower Disease, Ulcerative Colitis)
Abdominal pain is the most common quadrant (Rovsing sign), with internal and
• Inguinal hernia
complaint. It typically starts periumbilical external rotation of the flexed right hip
or epigastric, then migrates to the RLQ. It (Obturator sign), with the extension of the • Intussusception
is the most discriminating feature of the right hip (Psoas sign), with cough
(Dunphy sign) or with dropping from • Meckel Diverticulum
patient’s history. Its’ sensitivity and
specificity are approximately 80%; the standing on toes to the heels (Markle
• Mesenteric adenitis
positive likelihood ratio is 3.18, the Sign).
negative likelihood ratio is 0.5. Patients • Mesenteric ischemia
typically avoid moving because it
Differential Diagnoses
Many different specific diseases cause • Omental torsion
worsens their pain. The classic history of
abdominal pain. The below list is given in
anorexia, periumbilical pain followed by • Pancreatitis
alphabetical order. We advise you that
nausea, RLQ pain, and vomiting occurs in
look for other specific disease entity • Perforated viscus
only 50% of cases. Nausea is present in
chapters to understand presentation,
61-92% of patients, anorexia in 74-78%. • Rectus sheath hematoma
diagnosis and treatment differences.
Vomiting almost always follows the pain.
Diarrhea or constipation is noted in as • Tubo-ovarian pathologies (Ectopic
• Acute Cholecystitis or Biliary Colic
many as 18% of patients. In up to 50% of pregnancy, Pelvic inflammatory disease,

cases, local tenderness of Mc Burney’s • Acute Gastritis or Peptic Ulcer Disease Abscess, Endometriosis, Ovarian cyst/

point and rebound tenderness may be torsion, Uterine leiomyomata)


• Colonic carcinoma
present. Typical physical findings are
• Typhoid Fever
rebound tenderness, pain with • Diverticulitis
percussion, guarding and rigidity. RLQ • Ureterolithiasis or Urinary tract infection
• Gastroenteritis
tenderness is seen in 96% of patients,
but it is nonspecific. Other signs of

327
Emergency Diagnostic Tests T h e r e f o r e , i t i s n o t s p e c i fic t o more likely urinary tract infections.

and Interpretation appendicitis. Studies show that sensitivity Additionally, proteinuria and hematuria in
Appendicitis is a clinical diagnosis. of CRP is between 93% and 96.6% for urinalysis usually suggest genitourinary or
However, some laboratory tests may help acute appendicitis. A normal CRP level hematological disorders. Women of
emergency physicians in the decision- has a negative predictive value of childbearing age must have pregnancy
making process. Each test has some pros 97-100% for appendicitis in the patients evaluated. Ectopic pregnancy should be
and cons. Therefore, your clinical history with symptoms longer than 24 hours. in your mind always.
and exam should be the main part of your Investigators have also studied the
Computed tomography (CT) has 94%
decision-making process. Relying on combinations of WBC count, CRP and
sensitivity and 95% specificity and shows
laboratory tests may mislead you in some neutrophil count to reliably rule out the
higher diagnostic accuracy over USG
cases. diagnosis of acute appendicitis. Patients
(Sensitivity: 88%, specificity: 94%) for
with a WBC count below 10000/mm3 and
Count Blood Cell (CBC) is an easily acute appendicitis. A large, single-center
a CRP below 6 to 12 mg/dL are unlikely
accessible and inexpensive test, but it is study found that CT has a high rate of
to have acute appendicitis (Negative
nonspecific. Studies consistently show sensitivity and specificity (98.5% and
likelihood ratio: 0.09). Patients with a
that WBC count is greater than 10500/ 98%, respectively) for acute appendicitis.
WBC count above 10000/mm3 and a
mm3 of 80-85% adult patients with acute Though the use of IV and oral contrast
CRP above 8 mg/dL were likely to have
appendicitis. Also, the neutrophil count is may increase sensitivity, it may prolong
acute appendicitis (positive likelihood
higher than 75% in 78% of patients. CBC ED stays, cause allergic reactions and
ratio: 23.32).
shows different likelihood ratios (LR) for vomiting. Therefore, in adults, abdominal
different WBC levels. LR of WBC of Urinalysis may differentiate diagnoses and pelvic CT may be performed with or
9-11000 is 0.29. However, WBC of such as urinary tract infections. However, without contrast.
11-13000 has 2.8 LR. the appendix has a relationship with the
A healthy appendix usually cannot be
right ureter, and in some cases, pyuria
C-reactive protein (CRP) is useful, and it viewed by Ultrasonography (USG). In the
may not refer to only urinary infections.
usually is higher than 1 mg/dL. However, case of acute appendicitis, the USG
Pyuria may occur in cases of
it cannot detect the site of infection. typically demonstrates a non-
appendicitis, but severe pyuria marks

328
compressible tubular structure of 7-9 mm in diameter in the RLQ. T h e u l t r a s o u n d v i d e o shows transverse and longitudinal
However, USG is not as accurate as CT. USG is the first choice, views of  appendicitis in the same sequence. The appendix is
especially in pediatric patients, pregnant females, and slender located 3-4 cm deep from the skin surface. This video does not
patients. Additionally, if a gynecologic pathology is more likely include measurement. However, the reported diameter was 8 mm,
than acute appendicitis in females, USG can be the initial test to the diameter reaches more than 1 cm (10 mm) in some slices.
detect gynecologic pathologies such as ectopic pregnancy,
Plain radiographs are not specific or cost-effective. It may
ovarian cysts or other female reproductive system pathologies. If
visualize an appendicolith (It is highly suggestive of appendicitis
the operator is an experienced ultrasonographer, it may be the
but only seen in fewer than 10% of patients) or air-fluid level on
first imaging method. It is also important to emphasize that USG
RLQ location.
is an operator-dependent modality.
If USG is equivocal, magnetic resonance imaging (MRI) should be
Ultrasound images show the increased size of appendicitis
considered in pregnant patients. Its’ disadvantages are long scan
(below). More than 6 mm is considered abnormal (Image 7.1).
times, high cost, and limited availability. Some researchers
suggest MRI instead of USG in pediatric patients. MRI’s (100%)
Image 7.1 sensitivity is found higher than USG (76%) in pediatric patients
with acute appendicitis.

Emergency Treatment Options


Very few patients require aggressive resuscitation during the initial
evaluation (ABC phase). All patients with suspected dehydration
or septicemia must receive IV access and aggressive crystalloid
therapy. Additionally, parenteral antiemetics and analgesics
should be administered. Prophylactic antibiotics should be given
to cover gram-negative and anaerobic organisms.

One of the important global discussion is analgesic use in


appendicitis. There is still disagreement between different

329
physician groups, especially emergency Elders initially relate their symptoms to and tenderness may occur in the first
physicians and surgeons in some their comorbidities. As a result, late trimester, but RUQ or flank pain may
facilities. 2011 Cochrane review reported presentation to ED may cause diagnostic dominate later. The symptoms are similar
that “The use of analgesia for acute delays. Additionally, ongoing drugs’ side t o t h e fir s t - t r i m e s t e r p r e g n a n c y
abdominal pain does not mask clinical effects may mask their acute condition. symptoms such as nausea, vomiting, and
findings, nor does it delay diagnosis.” Therefore, a late presentation or anorexia. The physicians should consider
However, only recommended analgesics insignificance of symptoms should not appendicitis if these symptoms reappear
are opioids in these patients. dissuade the clinician from the diagnosis. later in pregnancy. However, WBC count
The diagnostic delay relates to increased is not reliable in pregnancy because of
Pediatric, Geriatric, and mortality and morbidity. The mortality the physiologic leucocytosis. Imaging
Pregnant Patient rates range from 0.1% to 1% in children, modalities USG or MRI can use for the
Considerations and it rises above 20% in patients older diagnosis.
Appendicitis has relatively high than 70 years. Overall, the perforation
misdiagnosis rates at both extremes of rate varies from 16% to 40%. Younger Decision Making
age. In children, the misdiagnosis rate is children have a higher perforation rate Clinical findings guide risk stratification.
25-30%. The most common between 50-85%. Diagnostic delays may Risk stratification scores guide diagnostic
misdiagnoses are gastroenteritis and increase perforation rates up to 55-70% modalities and disposition decisions such
respiratory tract infections. The early in patients older than 50 years. as discharge, observation or surgical
symptoms like loss of appetite or consultation. The Alvarado score is a
vomiting are non-specific. They may The appendicitis incidence in the well-known classification for appendicitis
easily lead the physician to other pregnant remains unchanged compared (Table 7.1).
diagnoses such as gastroenteritis, urinary to the general population, but the
or respiratory infections. changes in the presentation may delay
the diagnosis. During pregnancy,
Ten percents of the appendectomies are appendix replaces toward the right
performed in the elderly. Misdiagnosis kidney and rises above the iliac crest at
rates are high in this age group too. about 4.5 months of gestation. RLQ pain

330
Score
Table 7.1 Alvarado Score In Acute Appendicitis
1-4 Appendicitis unlikely
CATEGORY EXPLANATION SCORE
5-6 Appendicitis possible

Symptoms Migration 1 7-8 Appendicitis probable

Anorexia or acetone 9-10 Appendicitis very probable


1
(in the urine)
Disposition Decisions
Nausea or vomiting 1 Patients with minimal physical findings and a strong alternative
diagnosis or previous multiple episodes of similar pain are
Tenderness in right considered low risk. In low-risk patients, the best course of action
Signs 2
lower quadrant is advising on signs of appendicitis and arranging close follow-up
in 12 to 24 hours. Discharged patients should start on a liquid diet
Rebound pain 1 and advance to solids when their symptoms improve. Patients
with non-specific abdominal pain, who require significant doses
Elevation of
temperature (>37.3°C 1 of opiates should be considered for admission. Equivocal patients
measured orally) mostly consist of women of childbearing age, men, and children
Leukocytosis with atypical signs. They should be considered for diagnostic
Laboratory 2
(>10,000/mm3) testing or active observation. Men and children with classic
presentations are at high risk and gain little benefit from further
Shift to the left
1 imaging. Emergency physician should consult the patient with
(>75% neutrophils)
general surgeon without delay. Acute appendicitis is the most
Total Score 10 common reason for emergent abdominal surgery and
appendectomy remains the only curative treatment. Antibiotic
Adopted from Alvarado A. A practical score for the early diagnosis of acute treatment without appendectomy may be sufficient therapy for
appendicitis. Ann Emerg Med. 1986;15(5):557-564. Please read the original
article for further information.
331
uncomplicated appendicitis, especially in the pediatric
population.

References and Further Reading, click here

332
Section 2

Biliary Disease

Case Presentation
by Dan O’Brien A 35-year-old woman presents to the emergency department
with right upper quadrant pain of two hours duration. She
awoke several hours after eating a large meal. Based on
increasing pain and nausea she presents for evaluation. She
denies vomiting, fever or dysuria. Her past history is notable
for diet-controlled type II diabetes, dyslipidemia, and essential
hypertension. Her BMI is 33. Her only medication is lisinopril
10 mg daily. She has never had surgery. Her social history is
unremarkable. She neither drinks alcohol nor uses tobacco.
She has begun to diet and reports recent weight loss.

Her temperature is 37ºC, blood pressure: 110/70 mmHg,


pulse: 90 beats per minute. Physical exam reveals an
overweight female in mild distress secondary to right upper
quadrant pain. She cannot find a position of comfort and
Audio is available here
describes the pain as similar to labor pains. Pertinent exam

333
findings include: chest exam normal, cardiac Image 7.2
exam normal, abdominal exam demonstrates
normal bowel sounds and no rebound in any
quadrant. She has guarding to inspiration with
palpation over the gallbladder (positive Murphy’s
sign). Rectal exam normal, stool is hemoccult
negative for blood. Pertinent lab values: glucose
110 mg/dl, alkaline phosphatase 120 U/L, alanine
aminotransferase (ALT) 25 U/L, aspartate
aminotransferase (AST) 25 U/L, gamma glutamyl
transferase (GGT) 20 U/L, direct bilirubin 0.1 mg/
dL, total bilirubin 0.5 mg/dL, lipase 20 U/L.

The emergency physician performs a focused


An IV was established, and the patient received
right upper quadrant ultrasound (Image 7.2) and
an isotonic fluid bolus. In addition, ketorolac 30
finds gallstones without associated gallbladder
mg IV and ondansetron 4 mg IV were
wall thickening or pericholecystic fluid. In
administered. Over the course of an hour
addition, the patient has a “sonographic Murphy
symptoms resolved. Absent evidence of
sign”: there is maximal abdominal tenderness
gallbladder inflammation or infection, she was
when the ultrasound probe is pressed over the
discharged from the emergency department and
visualized gallbladder.
334
referred to a general surgeon with a history of hypertension may physicians should think early pain
suggest a dissecting abdominal aortic medication to comfort the patient.
for elective cholecystectomy.
aneurysm, but the colicky pain
She was advised that her pain associated with abdominal distention Differential Diagnosis
Pain in the right upper quadrant can be of
might return but if it is may suggest bowel obstruction. Fever,
p ro t r a c t e d v o m i t i n g , s y n c o p e o r biliary origin including cholelithiasis:
prolonged, is associated with g a l l s t o n e s w i t h o u t i n fla m m a t i o n ,
gastrointestinal blood loss should all raise
fever or jaundice she is to the suspicion of serious illness. In cholecystitis: inflammation or infection of

return to the emergency addition, it is important to exclude the gallbladder wall, or cholangitis:

pregnancy and its complications in any inflammation or infection of the biliary


department.
woman of childbearing years who ducts. Pancreatitis independent of, or as

presents with abdominal pain. In this a consequence of gallstone obstruction


Critical Bedside Actions and
case, the patient had classic biliary colic of the common biliary duct,
General Approach
and documented gallstones. The pain choledocholithiasis, may present in a
Abdominal pain is a common complaint
was due to crystals or a small stone similar fashion as well. Hepatitis, gastritis,
in the emergency department and can be
passing and or blocking the cystic duct. dyspepsia, peptic ulcer disease are other
challenging to diagnose. Presenting
Based on the resolution of pain, the potential gastrointestinal causes of right
illnesses may range from benign self-
absence of abnormalities on ultrasound upper quadrant pain. Appendicitis,
l i m i t e d d i s e a s e s t o t r u e s u rg i c a l
exam such as pericholecystic fluid or wall especially in pregnant patients may
emergencies. The priority is to assess the
thickening, and normal laboratory values present with symptoms of right upper
stability of the patient. Use history, the
it would be safe to discharge this patient quadrant pain. Non-abdominal diseases
likelihood of disease, vital signs, and the
for elective cholecystectomy. During the such as pneumonia or pleurisy on the
physical exam to assist in determining
course of the management, it is very right lung may present with right upper
whether a patient may have a serious
i m p o r t a n t t o d i ffe re n t i a t e c r i t i c a l quadrant pain.
illness or surgical emergency. For
example, a sudden onset of tearing pain situations from uncomplicated
radiating to the back in an older patient gallbladder disease. In addition, the

335
History and Physical are at increased risk for pancreatitis as described as colic, the pain may be

Examination Hints well. Diets low in fiber and high in more constant as it is caused by an
“A 35-year-old woman presents to the carbohydrates and fat have been obstruction of bile flow with subsequent
emergency department with right upper associated with gallstone formation. This distention. Patients may appear restless
quadrant pain of two hours duration. She may, in part, explain regional differences and unable to find a comfortable position.
awoke several hours after eating a large in gallstone formation. Murphy’s sign (the sudden cessation of a
meal. Based on increasing pain and deep inspiration when the inflamed
“ H e r t e m p e r a t u re i s 3 7 º C , b l o o d
nausea she presents for evaluation. She gallbladder descends and reaches the
pressure: 110/70 mmHg, pulse: 90 betas
denies vomiting, fever or dysuria.Her past examiners’ fingers palpating the right
per minute. Physical exam reveals an
history is notable for diet-controlled type subcostal area) is 65% sensitive and 87%
overweight female in mild distress
II diabetes, dyslipidemia, and essential specific for acute cholecystitis. Fever is
secondary to right upper quadrant pain.
hypertension. Her BMI is 33. Her only not typical, and jaundice is rarely seen
She cannot find a position of comfort and
medication is lisinopril 10 mg daily. She unless there is obstruction of the
describes the pain as similar to labor
has never had surgery. Her social history common bile duct from
pains. Pertinent exam findings include:
is unremarkable. She neither drinks choledocholithiasis or extrinsic
chest exam normal, cardiac exam normal,
alcohol nor uses tobacco. She has begun compression due to mass or
abdominal exam demonstrates normal
to diet and reports recent weight loss.” inflammation.
bowel sounds and no rebound in any
Gallstones are two to three times more quadrant. She has guarding to inspiration Emergency and Diagnostic
common in women, especially during with palpation over the gallbladder Tests and Interpretations
childbearing years. The risk of also (positive Murphy’s sign). Rectal exam
gallstones increases with age. Obesity or normal, stool is hemoccult negative for Laboratory Tests
blood.” “Pertinent lab values: glucose 110 mg/dL,
Body Mass Index (BMI) greater than 30 is
alkaline phosphatase 120 U/L, alanine
associated with increased gallstone
Patients with biliary colic have moderate aminotransferase (ALT) 25 U/L, aspartate
formation. Type II diabetes is associated
to severe right upper quadrant colicky aminotransferase (AST) 25 U/L, gamma
with obesity, hyperlipidemia, and
pain without peritoneal signs. Although glutamyl transferase (GGT) 20 U/L, direct
gallbladder hypomotility. Diabetic patients
336
bilirubin 0.1 mg/dL, total bilirubin 0.5 mg/ cholangitis. ALT may briefly spike during gallbladder wall thickening or
dL, lipase 20 U/L.” acute obstruction, but it usually is not pericholecystic fluid. In addition, the
elevated unless there is secondary liver patient has a “sonographic Murphy sign”:
Alkaline phosphatase (ALP) is
parenchymal damage. An AST level there is maximal abdominal tenderness
synthesized by the bile duct epithelial
greater than the ALT level suggests when the ultrasound probe is pressed
cells. Its production is stimulated by bile
alcoholic liver disease, cirrhosis or over the visualized gallbladder.”
duct obstruction and is elevated in a
metastatic disease.
majority of patients with cholestasis. Plain radiography is often not helpful in
However, isoenzymes are found in the Gamma-glutamyl transpeptidase (GGT) is assessing biliary stones as most do not
liver, bone, placenta, small bowel and a membrane-bound peptidase that contain enough calcium to be visible on
leukocytes; it is therefore not specific for hydrolyzes peptides to amino acids and plain x-ray. Plain imaging may be useful
the biliary tract. smaller peptides. Although serum activity to identify gas in the biliary tree or
is primarily from the liver, it is found in the evidence of intestinal obstruction.
Bilirubin is a breakdown product of heme.
renal proximal tubule, pancreas, and
Unconjugated bilirubin is hydrophobic X-ray (Image 7.3) shows relatively normal
intestine. Its circulating half-life is usually
and is transported in the blood bound to findings in a RUQ and abdominal pain
7-10 days but may increase to 28 days in
albumin. It is taken up by the hepatocyte, patient. The CT scan of the same patient
alcohol-associated liver disease. The
conjugated, and actively secreted into the is shown below. It revealed cholecystitis
cholestatic disease may elevate GGT
biliary tract. Cholestasis may elevate (Image 7.5).
significantly. A complete white blood cell
serum bilirubin.
count, serum electrolytes, glucose renal
The aminotransferases; aspartate function studies, and urinalysis, may
aminotransferase (AST) and alanine assist in diagnosis and management.
aminotransferase (ALT) are found in the
liver, cardiac and skeletal muscle, and
Imaging Modalities
“The emergency physician performs a
cerebral nerve cells. Levels of these
focused right upper quadrant ultrasound
enzymes are typically only mildly elevated
and finds gallstones without associated
but may be markedly increased in
337
pericholecystic fluid, C T imaging (Image 7.5) is not nearly as
Image 7.3
helpful as the right upper quadrant
sonographic Murphy’s sign,
ultrasound in evaluating the biliary system
common duct dilatation. for evidence of cholecystitis. Gallstone
sensitivity is about 75%, and common
In acute cholecystitis, gallstones are duct stones may be missed. It may be
present in 95-99% of cases. Emergency helpful to reveal complications of
physicians, performing focused, limited cholecystitis such as gangrenous or
bedside ultrasound and taking into emphysematous cholecystitis as well as
account the context of the patient’s to exclude other pathologies in the
h i s t o r y a n d c l i n i c a l p i c t u re h a v e abdomen.
documented a sensitivity of 90-96%, a
specificity of 88-96% as well as a positive Image 7.5
predictive value of 88-99% and a
negative predictive value of 73-96% for
cholecystitis. (Image 7.4)

Ultrasound imaging of the right upper


quadrant is the principal study used to
evaluate biliary-type pain and detect
Image 7.4
gallbladder disease and biliary dilatation.

There are several sonographic criteria for


acute cholecystitis;

the presence of gallstones,

thickened gallbladder wall,

338
Emergency Treatment inflammatory drugs (NSAIDs) are first-line o f metronidazole and a fluoroquinolone.

Options therapies. In fact, studies suggest that Most patients will improve over 24 to 72
“An IV was established, and the patient NSAIDs have similar efficacy as opioids hours before surgical intervention.
received an isotonic fluid bolus. In with fewer complications. Opioids may be
Cholangitis, an infection of the bile duct,
addition ketorolac, 30 mg IV and used to control pain. Although there were
is a life-threatening disease that requires
ondansetron 4 mg IV were administered. historical concerns about morphine
aggressive resuscitation, timely
Over the course of an hour symptoms causing greater sphincter of Oddi spasm
antibiotics, and early drainage via either
resolved. Absent evidence of gallbladder relative to other opioids, all opioids to
endoscopic retrograde
inflammation or infection she was some degree increase sphincter of Oddi
cholangiopancreatography (ERCP)
discharged from the emergency pressure and biliary pressure. If a
guided sphincterotomy or stent
department and referred to a general patient’s pain is resolving and controlled
placement or percutaneous drainage to
surgeon for elective cholecystectomy. with oral agents, they may be discharged
stabilize the patient prior to definitive
She was advised that her pain might and referred to a general surgeon for
surgery.
return but if it is prolonged, is associated consideration of elective laparoscopic
with fever or jaundice she is to return to cholecystectomy.
Pediatric, Geriatric,
the emergency department.” Pregnant Patient and Other
Acute cholecystitis is best managed in
Asymptomatic gallstones do not require the hospital with surgical consultation. Considerations
Early laparoscopic cholecystectomy is None other than mentioned above.
any treatment. Most remain
asymptomatic for years after diagnosis. often the treatment of choice. Patients
should be given nothing by mouth. About
Disposition Decisions
About 1-2% may become symptomatic
annually. 20% of patients develop gallbladder or Admission
biliary duct infection. Appropriate Patients with suspected cholecystitis or
Biliary colic or biliary pain typically has a antibiotics regimens include second- and cholangitis should be admitted to the
definitive onset with a duration ranging third-generation cephalosporins, hospital. For suspected cholangitis,
from 15 minutes to up to four hours. carbapenems, ß-lactam/ß-lactamase emergency consultation, and if need be,
Antiemetics and nonsteroidal anti- inhibitor combinations or a combination transfer to a facility that can emergently

339
establish biliary drainage either via ERCP-guided sphincterotomy
or percutaneous stenting.

Discharge
Patients with biliary colic may be discharged once their
symptoms have resolved with follow up with a general surgeon.
They should be informed that there may be symptom recurrence
and should be instructed to return if they experience prolonged
pain, fever or jaundice.

Referral
Asymptomatic gallstones need not be referred to a general
surgeon. The patient should be informed of their findings and
instructed to follow up with their primary care physician.

References and Further Reading, click here

340
Section 3

Massive Gastrointestinal Bleeding

Case Presentation
by Dan O’Brien A 68-year-old female presents to the emergency room at
midnight, with a chief complaint of vomiting “coffee grounds”
earlier that evening. She has a history of congestive heart
failure, hypertension, and a mild stroke. Her medications
include lisinopril 20 mg, Lasix 20 mg, aspirin 325 mg, and
clopidogrel 75 mg daily. Recently, she has taken ibuprofen
several times daily for arthritic pains. Family history is
significant for peptic ulcer in her mother and a brother.
Pertinent Exam: blood pressure is 98/65 mmHg, heart rate
110 bpm and regular, respiratory rate 14, non-labored, and
temperature 37 ºC. She appears pale, has a poor capillary refill
and is mildly confused but oriented to person, place and time.
Heart and lung sounds are normal; her abdomen is soft, non-
tender, without organomegaly, and without bruits. She has
Audio is available here trace pedal and pretibial edema. Her neurological exam is

341
grossly normal. While being examined, she asks hemoglobin of 11 g/ dL and creatine of 1.0 mg/
for a bedpan and vomits a cup full of bright red dL.
blood. Her blood pressure systolic is now 85 mm
Initial resuscitation is successful. Upper
Hg.
endoscopy reveals a bleeding duodenal ulcer that
She has hemodynamically significant upper GI is successfully stopped with hemoclips. The
bleeding. Her hypotension and tachycardia patients H. pylori stool antigen is positive.
indicate loss of more than 20% total blood Cardiology and Neurology agree to stop aspirin
volume. The most likely working diagnosis is and clopidogrel. Oral iron was started, and the
active upper GI bleeding likely from peptic patient was discharged. With the avoidance of
ulceration secondary to nonsteroidal anti- NSAIDs and with H. pylori eradication the risk of
inflammatory drugs (NSAIDs) with likely rebleed is less than 5%.
Helicobacter infection.

Bedside testing reveals hemoglobin of 6 g/dL.


Transfusion with packed red blood cells is begun
with a goal of hemoglobin of 8.0-10.0 g/dL.
Additional labs demonstrate a normal albumin
and prothrombin time (PT). Her creatinine is 2.0
mg/dL. Old records document baseline

342
Introduction • Hematochezia from upper GI source history of liver cirrhosis also give one
Despite advances in diagnosis prevention gram IV Ceftriaxone or 400 mg
• Hemoglobin <8 g/dL
and treatment, nonvariceal upper norfloxacin orally twice daily. Antibiotics
gastrointestinal bleeding is still a serious • Liver cirrhosis, coagulopathy are of benefit in cirrhosis by decreasing
problem in clinical practice. The infectious sequelae as well as the
• Orthostatic incidence of encephalopathy. The
incidence ranges from 48 to 160 cases
per 100,000 population per year. Upper reduction of bacterial products in the
• Resting tachycardia (>100 bpm)
GI bleeding causes mortality ranges from p o r t a l c i rc u l a t i o n re s u l t s i n l e s s
5% to 14%. • Syncope (systolic < 90 mmHg) vasodilation, which lowers the rebleeding
risk. After initial stabilization efforts,
• Transfusion > 1 unit/8hrs or 6 units total
Critical Bedside Actions and consult an endoscopist.
General Approach Initiate resuscitation: insert two large bore
“She has hemodynamically intravenous catheters infuse lactated
“The most likely working
significant upper GI bleeding.” ringers, type and crossmatch, obtain diagnosis is active upper GI
complete blood count with platelets, PT, bleeding likely from peptic
Indicators of Major Blood Loss and INR, as well as routine blood
ulceration secondary to
• Acidosis chemistries to assess for renal and
hepatic function. Start intravenous nonsteroidal anti-inflammatory
• Anticoagulation
octreotide, a somatostatin analog, at 50 drugs (NSAIDs) with likely
• Antiplatelet medications mcg/hour. Proton pump inhibitor by
Helicobacter infection.”
continuous drip, as a pH of 7 or greater is
• Azotemia (BUN > 40 mg/dL needed for platelet function and clot
adherence.
• Chest pain or dyspnea
Intravenous erythromycin 250 mg if given
• Continued bleeding or re-bleeding
within 30 minutes of planned endoscopy
• End stage renal disease can improve visualization. If there is a

343
The goal of therapy is to stop bleeding to
Figure 7.1 Figure 7.2
prevent end organ ischemic damage.
Medical treatment alone is successful for
most cases of lower GI bleeding with a
third of upper GI bleeding cases requiring
emergent endoscopic therapy.

Table 7.3 Presentations of Upper and


Lower GI Bleeding

UPPER GI LOWER GI
BLEEDING BLEEDING
Differential Diagnosis Table 7.2 Comparison of Upper and
Abdominal pain may or may Large volume hematochezia
While it is important to know what is Lower GI Bleeding not be present in peptic or maroon stool with
ulcer orthostasis indicates
bleeding to determine prognosis and bleeding from right sided
UPPER GI LOWER GI Chest pain with esophageal diverticulae or ateriovenous
guide management, it is most crucial to BLEEDING BLEEDING ulcer malformations

think of anatomy and pathophysiology: Sudden fullness with nausea Small volume hematochezia

larger vessels bleed faster and more often 35% present with 19% present with due to blood in GI tract without orthostasis indicates
hemorrhoidal bleeding
shock shock Hematemesis or coffee (usually painless) if painful
require urgent intervention. The internal ground emesis followed by with dyschezia indicates
melena anal fissures or proctitis
diameter and pressure in vessels above 65% require 36% require
the ligament of Treitz are greater than transfusion transfusion Hematochezia in 10% of Bloody loose stools with low
rarpidly bleeding upper abdominal pain present in
lesions infectious colitis,
vessels associated with lower GI inflammatory bowel disease
30% require >90% stop Coughing followed by or ischemic colitis
bleeding. intervention to stop spontaneously hematemesis in Mallory
Weiss tear

Valsalva may prompt


bleeding from esophageal or
gastric varices

Historical presentations vary in upper and lower


GI bleeding

344
Upper GI bleeding represent 65 to 80% • gastric antral vascular ectasias (GAVE Lower GI bleeding is most often
of all GI bleeding, and includes or “watermelon stomach”) or caused by right-sided diverticula,
arteriovenous arteriovenous malformations, colonic
• esophageal or gastric varices,
adenocarcinoma, ischemic colitis,
• malformations above the ligament of
• duodenal or gastric ulcer, inflammatory bowel disease, infectious
Treitz.
colitis, or anorectal lesions including
• erosive gastritis, hemorrhoids, fissures, and proctitis.
Worldwide, upper GI bleeding from peptic
• erosive or ulcerative esophagitis, ulcer is most prevalent, although persons
with portal hypertension may represent
History and Physical
• Mallory Weiss tears, the majority who present with massive Examination Hints
Elderly patients and those with valvular
upper GI bleeding. With advanced age
• gastrointestinal cancers, heart disease or renal failure have an
and atherosclerotic disease, more
increased risk for arteriovenous
Rarer causes are patients are using anticoagulants or
malformations anywhere in the GI tract.
antiplatelet medications that impair clot
• hemobilia, Chronic NSAID use causes gastric
formation, and augment bleeding.
mucosal erosions in at least one-third of
• splenic artery pseudoaneurysms, Gastrointestinal bleeding from non-
daily users or significant ulceration in 2%.
steroidal induced peptic ulcers is on the
• Dieulafoy lesions, Alcohol consumption, chronic viral
rise, with up to 1 in 2 adults taking these
hepatitis, non-alcoholic hepatitis (NASH)
medicines. Although with industrialization
• gastrin-secreting tumors (Zollinger- can result in cirrhosis with portal
Ellison syndrome), and improved hygiene the prevalence of
hypertension. A family or prior history of
Helicobacter pylori has declined, the
peptic ulcer suggests Helicobacter pylori
• arteriovenous fistulae, infection and associated conditions are
infection. H. pylori is a spiral-shaped
still major causes of upper GI bleeding in
• penetrating foreign bodies, flagellated bacterium that lives in the
many parts of the world including the
human stomach and interrupts the
Middle East, Asia, and South America.
protective mucous bicarbonate layer, thus
exposing the epithelium to hydrochloric
345
acid leading to chronic inflammation. Most persons with duodenal
Image 7.6
ulcer report sharp epigastric pain worsened by eating while less
than 50% of patients with gastric ulcer report abdominal pain that
improves post meals, as the acid is then buffered by the food and
duodenal bicarbonate secretion. The color of vomitus or stool is
also predictive of severity: hematemesis suggests ongoing
bleeding, whereas “coffee grounds” indicate partially digested
hematin or “old blood.” Abrupt symptom onset associated with
hypotension suggests acute bleeding whereas a history of weeks
of intermittent dark melanic stools suggests chronic blood loss.
Presenting vitals signs are most predictive of the magnitude of
blood loss; other important physical clues for portal hypertension
include abdominal ascites, enlarged liver or splenomegaly.
Evidence of hyperestrogenemia in males with cirrhosis includes
gynecomastia, testicular atrophy, and spider telangiectasias on
the chest or upper body. Palmer erythema and bounding pulses
in the fingers from peripheral vasodilation are indicators of
advanced cirrhosis. The bedside physical exam is unreliable in
females as palmer erythema and telangiectasias are normal Emergency Diagnostic Tests and
findings. Ascites determination is difficult in every patient unless it Interpretation
is massive. If available, a bedside ultrasound may confirm When abdominal pain is present, a plain film with upright chest x-
suspected ascites, coarse echotexture of the liver, or show ray may reveal significant atherosclerotic disease, ingested
reduced or reversed (hepatopedal) flow in the hepatic veins in foreign bodies, or subdiaphragmatic free air from a perforated
advanced cirrhosis. ulcer.

“Bedside testing reveals hemoglobin of 6 g/dL.


Transfusion with packed red blood cells is begun
346
with a goal of hemoglobin of 8.0-10.0 g/dL. Emergency Treatment Options
Additional labs demonstrate normal albumin and “Initial resuscitation is successful.”
PT. Her creatinine is 2.0 mg/dL. Old records
Table 7.4 Emergency Management of GI Bleeding
document baseline hemoglobin of 11 g/dL and
TREATMENT
creatine of 1.0 mg/dL.” COMMENTS
OPTIONS

Patients with low albumin, prolonged PT or INR, or platelet count Goal:


Resuscitation
Hgb 7.5 to 8.0 in those with portal HTN
less than 150K, consider underlying cirrhosis and avoid with packed red
blood cells
transfusion above hemoglobin of 7.5 to 8.0, to avoid increasing Hgb 9.5 to 10 in the elderly, MI, CHF, stroke
portal pressure and increasing the risk of rebleeding. A platelet 50mcg/hour for active bleeding anywhere in the GI
IV octreotide
transfusion would be indicated for a critically low platelet count tract (both upper and lower bleeding)
(<50K) from consumptive coagulopathy or splenic sequestration. IV continuous
give IV erythromycin 250mg 30 minutes prior to
For significant coagulopathy, (INR >1.8) transfusing fresh frozen PPI for all upper
EGD to improve visualization
GI bleeders
plasma (10-15 cc/kg) corrects factor seven deficiency to allow
coagulation to occur. Cryoprecipitate or other specific factors ceftriaxone 1 gram IV or norfloxacin 400mg IV
Antibiotics
twice daily for all GI bleeders with cirrhosis
may be indicated for patients with known factor deficiencies. Red
blood cell indices are useful as a low mean corpuscular volume It depends on magnitude of bleeding; goal is to
Timing of
(MCV) suggests iron deficiency from chronic GI blood loss stop bleeding as soon as possible to prevent end
endoscopy
organ ischemic damage. Involve consultants early!
whereas in acute bleeding MCV is often increased due to the
release of reticulocytes from the bone marrow. Less than 5% cases require angiography, less
Other
than 1% surgery

It is not necessary to insert a nasogastric tube (NG) as the


information obtained rarely changes management and may
compound problems by causing pain, gagging, and epistaxis. A
negative aspirate does not exclude active bleeding, and a positive

347
aspirate does not affect the timing of Colonoscopy is used less often as an
Figure 7.3
endoscopy or additional interventions. interventional therapeutic technique to
Gastric lavage is no longer considered stop bleeding from hemorrhoids, fissures,
useful. Consider endotracheal intubation arteriovenous malformations or diverticuli.
to decrease aspiration risk before elective
endoscopy for any patient with upper GI Figure 7.4
bleeding who is unconscious, in
significant respiratory distress, or with
recurrent witnessed hematemesis.

Urgent Endoscopy: Call as soon as


possible for endoscopy in patients with of endoscopy in upper GI bleeding.
hemodynamically significant bleeding. Outcomes for endoscopic intervention
Endoscopy is portable and can be safely have shown reduced rebleeding and
performed in the emergency department transfusion requirements, with improved
providing immediate information morbidity. With excellent clinical care and
Interventional Radiographic Techniques
regarding diagnosis, treatment, combined with endoscopic therapies,
are required in less than 5% of all cases
prognosis, and disposition. mortality from non-variceal GI bleeding is
of non-variceal GI bleeding. Angiography
10%, and mortality from variceal
Large studies have shown that can arrest bleeding from arteriovenous
hemorrhage is 25-30%. Colonoscopy for
endoscopy can safely be performed in malformations of the upper or lower GI
diagnosis of lower GI bleeding cause is
patients with bleeding leading to acute tract, as well as selective embolization of
most often performed electively after
myocardial ischemia or infarction, with arterioles from bleeding tumors or ulcers.
resuscitation; usually within 24 to 48
improved outcomes for interventions that Early use of transjugular intrahepatic
hours. Visualization of the lower GI tract
stop further bleeding. The magnitude of portosystemic shunts (TIPS) to definitively
requires cleansing with large volume
bleeding, signs of continued bleeding, or reduce portal pressure is beneficial after
balanced electrolyte solutions taken orally
suspicion of varices predicate the timing initial endotherapy, and as first-line
o r b y N G f o r q u i c k e r d e l i v e r y.

348
therapy for select patients with high risk of variceal bleeding.
Table 7.5 Glasgow-Blatchford Risk Score
Emergency Surgery for GI bleeding is required in less than 1% of CATEGORY SCORE
all cases including surgery for a peptic ulcer, and total or subtotal
BUN in mg/dL
colectomy for shock associated with bleeding diverticuli.
18.2 to 22.4 2
Emergency shunt surgery for liver cirrhosis is almost never
22.5 to 28 3
performed, as the mortality is unacceptably high compared to
endoscopic and angiographic techniques. 28.1 to 70 4
70.1 or greater 6
Pregnant Patients and Other Considerations Hemoglobin, men g/dL
In pregnant patients with GI bleeding, monitor for fetal distress,
12 to 13 1
and consult Obstetrics. Avoid erythromycin in the third trimester
10 to 11.9 3
otherwise treat the same as any other adult with GI bleeding.
Emergency upper endoscopy is safe in all trimesters. Lower 9.9 or less 6

endoscopy may be difficult depending on the size of the fetus/ Hemoglobin, women g/dL
uterus but is not contraindicated. The endoscopist will use safe 10 to 12 1
sedation medications for pregnancy. For GI bleeding in patients 9.9 or less 6
with acute MI, significant heart disease, stroke or significant
Systolic Blood Pressure, mmHg
neurovascular disease consult cardiology or neurology for help
100-109 1
with management. Often they will agree to urgent endoscopy to
90-99 2
clarify and treat bleeding lesions without interruption or reversal
of anticoagulation. <90 3
Heartrate >100 peats per minute 1
Disposition Decisions Melena 1
There are several valuable bleeding scoring systems help to
Syncope 2
guide disposition.
Hepatic Diseases 2
Heart failure 2
349
Glasgow-Blatchford Risk Score is useful for predictive of inpatient AIM65 GI Bleeding Score is practical, easy to remember, assists
mortality, blood transfusions, re-bleeding, ICU monitoring, and with level of care, and timing for endoscopy. Scores less than 1
hospital length of stay. Patients with a score of zero may be predict good outcome, scores above 2 require hospitalization and
discharged home, those with score 2 or higher are usually treatment.
admitted, and those with score of 10 or more are at highest risk
for morbidity and resource utilization. Maximum score is 23. Admission Criteria
Patients with GI bleeding presenting in shock, requiring
transfusion or with bleeding scores (AIM65>2 or Glasgow-
Table 7.6 AIM65 Bleeding Score
Blatchford (GB) >10) have significant predictable morbidity and
RISK FACTOR SCORE mortality requiring ICU admission and treatment. Patients with
AIM65 of 1 or less or GB score 2 or less have predictably mild GI
Albumin <3.0 1
bleeding (melena without hematemesis and who are
INR > 1.5 1
hemodynamically stable) and may be cautiously admitted to a
Altered mental status 1
medical floor.
SPB < 90mm Hg 1
Age > 65 1 Discharge Criteria
Patients with AIM or GB score of zero may be discharged home
MAXIMUM SCORE 5
without outpatient gastroenterology evaluation within two weeks.
Prescribe twice daily PPI, avoidance of NSAIDS and alcohol for
Point Mortality %
those with upper GI symptoms. Instruct them to return
0 0% immediately for syncope or signs of bleeding. Those with history
1 0.9% and findings consistent with minor lower GI bleeding and stable
2 7.4% hemoglobin should also be referred for outpatient consultation

3 42% within 2 weeks.

4 75% “Upper endoscopy reveals a bleeding duodenal ulcer that is


5 100% successfully stopped with hemoclips. The patients H. pylori stool
antigen is positive. Cardiology and Neurology agree to stop
350
aspirin and clopidogrel. Oral iron was started, and the patient was
discharged. With the avoidance of NSAIDs and with H. pylori
eradication the risk of re-bleed is less than 5%.

References and Further Reading, click here

351
Section 4

Acute Mesenteric Ischemia

Case Presentation
by Rabind Antony Charles A 75-year-old woman presents to your Emergency
Department (ED) with diffuse abdominal pain for the past day,
associated with diarrhea and vomiting. She says the pain is
increasingly worse and has failed to respond to paracetamol
and charcoal tablets. She has a history of hypertension,
hyperlipidemia, and atrial fibrillation. She has no history of
laparotomy. She is alert and oriented. However, she is in
distress because of her abdominal pain. The pain score is 9
out of 10. Blood pressure: 96 over 56 mmHg, pulse rate: 125
(irregularly, irregular), respiratory rate 20, pulse oximetry: 98%
on room air, tympanic temperature: 37.5 degrees Celsius.
Heart sounds: (irregular) S1S2 positive. Lungs sounds are
bilateral equal and clear. Abdominal exam reveals diffuse
tenderness; it is worse in periumbilical region, no guarding,
Audio is available here

352
bowel sounds are sluggish. No scars or hernias Introduction
Acute mesenteric ischemia is a life-threatening cause of acute
noted. Per rectal exam: brown stool.
abdominal pain which occurs predominantly in patients over 50
What do you think about patient’s ECG? years old with the underlying cardiovascular disease. It is caused
by inadequate flow through the mesenteric vessels resulting in
Image 7.7 bowel ischemia and eventually gangrene of the bowel wall.
Mortality rates can be between 60-80% especially in patients with
greater than a 24-hour delay in diagnosis or presentation. This
underscores the importance of early detection in the ED, and the
need for aggressive management to reduce morbidity and
mortality. Surgical intervention in 6 hours of symptoms increases
survival rate.

Mesenteric artery embolism is responsible for 50% of patients.


Symptoms are sudden onset of abdominal pain with bloody
diarrhoea if infarct develops. Arrhythmias (e.g., Atrial fibrillation),
valvular disorders and recent myocardial infarction are the
predisposing factors for embolism.

Mesenteric artery thrombosis is around 25% of the patients.

Patients are generally shown atherosclerotic disease symptoms.

“Abdominal angina” for preceding months, which is a pain on


eating with loss of weight, then sudden severe pain episodes.

In 20% of the reason is non-occlusive mesenteric ischemia.


Hypotension, Congestive Heart failure, dialysis, use of
vasoconstrictors or digoxin are predisposing factors. This
353
situation happens typically intubated, sick t h e p a t i e n t s a r e s u ffe r i n g f r o m •Acute pancreatitis
ICU patients on vasopressors who gastroenteritis.
• Peptic ulcer disease
deteriorate with bloody diarrhea and
One of the key features to look out for is
worsening hypotension. • Bowel perforation
pain that is “out of proportion” to the
If the patient has hypercoagulable states abdominal findings. This is due to visceral • Diverticulitis
or his story of prior thromboembolic ischemia with sparing of the parietal
events, mesenteric venous thrombosis peritoneum in the initial stages. Peritonitis • Bowel obstruction
should be considered (5% of the is a late finding and points towards
• Ureteric calculi
patients). Patients represent with severe bowel ischemia and necrosis. At
nonspecific abdominal pain with diarrhea this stage, there may be abdominal Emergency Diagnostic Tests
1-2 weeks after the event which may distension associated with reduced and Interpretation
resolve spontaneously. bowel sounds.
Laboratory Tests
History Taking and Physical Mesenteric ischemia also can be more There are “no” sufficiently sensitive or
Examination Hints subacute in its presentation with the specific serum markers to identify acute
This is a difficult condition to diagnose, insidious onset of less severe and vague mesenteric ischemia.
especially in the early stages. However, it abdominal pain, abdominal distension,
Complete Blood Cell Count: may show
should be considered in those over 50 and occult gastrointestinal bleeding.
haemoconcentration and leukocytosis
years old with severe abdominal pain,
and predisposing factors for the
Differential Diagnoses ( WBC count > 15,000/mm3) – lacks
Depending on the clinical presentation, specificity.
condition.
the differential diagnoses can be quite
Arterial/Venous blood gas analysis:
In the early stages, patients may present broad and include the following:
metabolic acidosis is seen late in the
with severe poorly localized abdominal
• Acute gastroenteritis disease. Presence of metabolic acidosis
pain, nausea, vomiting, and diarrhea with
which cannot otherwise be explained
no signs of peritonism. These symptoms • Acute cholecystitis
may mislead physicians into assuming
354
should prompt the clinician to suspect mesenteric ischemia in the
Image 7.8 The CT image shows bowel wall thickness.
appropriate clinical context.

Serum lactate: nearly 100% sensitive when bowel infarction is


present but lacks specificity and is not often increased without
infarction.

Serum amylase may be moderately elevated in more than half of


the cases (lacks specificity).

Imaging Modalities
Erect Chest x-ray/Abdominal x-ray series used mainly to exclude
other causes of abdominal pain or look for complications of acute
mesenteric ischemia (e.g., free gas/bowel obstruction). They are
often normal in the early stages of acute mesenteric ischemia.
However, there are some early and late findings in the X-rays.
Early findings are adynamic ileus, distended air-filled bowel loops
and bowel wall thickening from submucosal edema or
hemorrhage. Pneumatosis of the bowel wall and gas in the portal
venous system strongly suggest bowel infarction as late findings.

Multi-detector CT angiography is the primary imaging modality to


diagnose acute mesenteric ischemia in the ED. The recent meta- The CT angiogram may show edema of the bowel wall and
analysis shows a sensitivity of 82.8- 97.6% and specificity of mesentery, abnormal gas patterns, intramural gas, ascites and
91.2-98.2 when compared to conventional angiography, which is occasionally direct evidence of mesenteric venous thrombosis. It
still considered the gold standard but is rarely available in ED. will also determine other causes for the abdominal pain. If the CT
is non-diagnostic and clinical suspicion for acute mesenteric
ischemia remains high, there may be a need for angiography or

355
diagnostic laparotomy depending on monitoring. IVC assessment with bedside decompress stomach and bowel is
institutional practice. U l t r a s o u n d c a n g u i d e t h e flu i d often necessary. Correct any electrolyte
resuscitation before invasive procedures abnormalities and acidosis.
Ultrasound has a limited role in the
(RUSH protocol).
diagnosis of acute mesenteric ischemia. Urgent surgical consultation should be
It is more useful for ruling out other A quick tutorial on IVC measurement with obtained in the ED as this is a time-
causes of abdominal pain, e.g., ultrasound (video) sensitive condition. Delays to definitive
cholecystitis, acute abdominal aneurysm treatment will result in increased
The ultrasound video shows collapsible
rupture, ureteric colic. morbidity and mortality. It is best to get a
and non-collapsible IVC. If the IVC
surgical consult when suspicion is high
Emergency Treatment collapse, this means the patient may
for acute mesenteric ischemia even
Options benefit from fluid resuscitation. (video)
before a CT angiogram has been done.

Initial Stabilization and If needed, correct any arrhythmias or


Specific Treatment
Aggressive Resuscitation CHF which may have contributed to the
In general, the definitive treatment of
Evaluation of patients with ABC approach bowel hypoperfusion. Because one of the
acute mesenteric ischemia depends on
gives the physician a chance to recognize predisposing factors is vasoactive
the underlying etiology and the presence
immediate life-threatening problems. The agents, discontinue these medications. If
or absence of necrotic bowel signs. This
most of the patients require supplemental pressors are required to support the
ultimately is decided by the surgeons and
oxygen. But, consider securing airway if patient’s blood pressure, it is preferable
is one of the reasons why it is extremely
needed. Correction of hypovolemia and to avoid alpha agonists. In this
important to get an urgent surgical
hypotension (secondary to third space circumstances, use inotropes at the
consult when confronted with these
loss and/or bleeding) with normal saline/ lowest possible dose. Start broad-
patients. In the presence of necrotic
crystalloids is very important. Because spectrum IV antibiotics early, because of
bowel/peritonitis, bowel resection will
most of the patients have multiple the high risk of bacterial translocation
need to be done regardless of which of
comorbidities, e.g., CHF; there may be a across the bowel wall. Patients should
the four types of the acute mesenteric
role for invasive hemodynamic monitoring not receive anything orally and
ischaemic bowel.
with arterial lines, central venous pressure nasogastric tube placement to
356
In addition, there are some other specific options. Mesenteric results were unremarkable. ECG showed atrial fibrillation (see
artery embolism may benefit prom embolectomy then distal picture given under the case presentation)
bypass graft. Mesenteric artery thrombosis needs bypass graft or
A bedside ultrasound excluded other causes of abdominal pain
stenting. Nonocclusive mesenteric ischemia requires to remove
(e.g., ruptured abdominal aortic aneurysm, acute cholecystitis).
the underlying stimulus and correction of the underlying medical
Assessment of IVC collapsibility and cardiac ejection fraction
condition. Occasionally direct transcatheter papaverine
gave the clues on aggressive fluid resuscitation. An NG tube was
(vasodilatory) infusion will restore normal blood flow. Mesenteric
inserted and broad-spectrum IV antibiotics (ceftriaxone and
venous thrombosis showing mild ischemia may be treated with
metronidazole) were given. Portable CXR and AXR series were
anticoagulation.
unremarkable. Based on her presenting complaint, a high
Disposition Decisions suspicion for acute mesenteric ischemia (possible acute
These patients are critically ill with potentially high mortality rates, mesenteric embolism- due to her underlying atrial fibrillation) was
and as such, they should be admitted and managed in intensive entertained and urgent surgical consult was sought. A
care after surgery. multidetector CT angiogram showed thickened small bowel wall,
dilated bowel loops, and superior mesenteric artery embolism.
The Conclusion of Case She was rushed to the operation theatre for exploratory
This patient was brought to the resuscitation area of the ED and laparotomy as her abdomen was noted to be more tender and
was put on cardiac and blood pressure monitors and pulse had some guarding.
oximetry. After ascertaining that her airway was intact, and
providing supplemental oxygen with intranasal oxygen, 2 large
bore IV cannulas were established and one liter of normal saline
was started, with care taken not to tip her into fluid overload by
serial assessment of IVC collapsibility with bedside ultrasound.
CBC, renal panel, VBG, and serum lactate, group and
crossmatch, and coagulation profile were sent off. Her leukocytes
were 12,000 and serum lactate was elevated while the rest of the

357
As a summary, the role of the ED physician is
to
• resuscitate the patient as needed,

• make an early diagnosis based on clinical suspicion,

• understand the limitations of laboratory tests in ruling out acute


mesenteric ischemia,

• give priority to aggressively resuscitation and

• get urgent surgical involvement

References and Further Reading, click here

358
Section 5

Perforated Viscus

Case Presentation
by Ozlem Dikme A previously healthy 42-year-old male presented to the
Emergency Department (ED) with a 3-day history of worsening
abdominal pain. He felt nauseated and vomited twice. His
pain started around the umbilicus, moved to the left side of his
abdomen and then become generalized. It peaked the last few
hours, and the painkillers did not work. His social history
revealed that he was non-drinker, non-smoker and did not use
any illicit drugs. The past and family histories were
unremarkable. His blood pressure was 100/60 mmHg, pulse
rate 120/min, the temperature 37.8°C (100°F), and respiration
rate 24/min. Physical examination showed diffuse abdominal
tenderness and voluntary guarding. Bowel sounds were not
heard. Bedside ultrasonography (USG) exhibited increased
echogenicity of the peritoneal stripe, with corresponding
Audio is available here horizontal reverberation artifacts over the liver. Plain chest

359
radiographs confirmed the presence of free Critical Bedside Actions and General
abdominal air. Oral intake was stopped, Approach
All critically ill patients with acute severe abdominal pain is a
intravenous (IV) catheter was inserted, fluid candidate to have a viscus perforation. The first step is always
therapy was started, and cefoperazone sodium patient evaluation with ABC approach and stabilization. These
was administered intravenously. Blood type and patients present to the ED with a severe abdominal pain and
discomfort. Because of the pain severity, they may not let you
cross, complete blood count and coagulation
touch their abdomen. They prefer to stand still because of any
were ordered. He transferred to the operation movement trigger pain. They look sick. Our first priority is to
theater with the diagnosis of the perforated ensure there is no immediate life or organ-threatening situation. If
so, immediate actions should be done at the bedside during the
viscus.
initial evaluation. Airway, breathing, and circulation evaluations
Can you identify free air on the X-ray? are completed. However, quick, focused abdominal examination
can be done before the full secondary evaluation. Opening two
Image 7.9 large bore IV lines, fluid therapy, stopping oral intake are some of
the routine actions. Patients are attached to the cardiac monitor.
Necessary blood samples are collected and sent. The pre-
diagnosis of perforated viscus must be explained to the patient,
and his approval should be obtained for further evaluation and
treatment. The US can be used at the bedside as an adjunct to
focused history and physical exam.

Differential Diagnoses
During the initial evaluation, emergency physicians try to
understand possible differential diagnoses in a patient with
severe abdominal pain. The below list is given in alphabetical
order.
360
• Abdominal Aortic Aneurysm Abscess, Endometriosis, Ovarian cyst/ o f pain helps to diagnose. Typical pain
torsion, Uterine leiomyomata) starts around a limited location, then
• Acute Cholecystitis or Biliary Colic
expands to all abdomen in a short time. A
• Acute Gastritis or Peptic Ulcer Disease
History and Physical history of frequent abdominal pain may
Examination Hints suggest the patient has a predisposing
• Acute MI Thorough medical history usually reveals condition. Free air under the diaphragm
predisposing factors or possible etiology may cause referred pain to the either or
• Acute Pancreatitis
of perforation. Predisposing chronic both shoulders. Vomiting is present in
• Aortic Dissection conditions include peptic ulcer disease, 50% of patients. Shock, sepsis,
inflammatory bowel disease, malignancy. gastrointestinal or intraabdominal
• Appendicitis Acute conditions like acute appendicitis, bleeding may accompany perforation.
acute diverticulitis, infections (e.g.,
• Diabetic Ketoacidosis
typhoid fever), intestinal ischemia, Ulcer perforation refers to when the ulcer
• Diverticulitis necrotizing vasculitis and penetrating or erodes through the wall and leaks air and
blunt injuries may cause perforation. peptic contents into the peritoneal cavity.
• Gastrointestinal carcinoma The anterior wall of the duodenum is the
Additionally, caustic substance and
foreign body (e.g., toothpicks) ingestions, most common site. Approximately 2 to
• Inflammatory Bowel Disease (Crohn
Disease, Ulcerative Colitis) endoscopic interventions and some 10% of patients with peptic ulcer
m e d i c a t i o n s a re a s s o c i a t e d w i t h undergo perforation once in their lives.
• Mesenteric ischemia perforation. Most common medications
Contamination of the sterile peritoneal
are aspirin, nonsteroidal anti-
• Omental torsion cavity with the chemical and bacterial
inflammatory drugs (NSAIDs) and
intestinal content causes inflammation,
• Rectus sheath hematoma steroids.
infection, and sepsis. Therefore, patients
• Tubo-ovarian pathologies (Ectopic The patient typically presents with may become tachypneic, tachycardic and
pregnancy, Pelvic inflammatory disease, sudden and severe abdominal pain. hypotensive in short time.
Asking patients about the characteristics Immunocompromised or critically ill

361
patients with other comorbidities pose a to diagnose but may exclude the other gas analysis, lactic acid, liver and renal
greater risk for perforation. Obscure differential diagnoses. function tests, lipase/amylase and
presentations in these patients may delay urinalysis can guide to diagnose and
the diagnosis. Bedside Tests post-surgical care. WBC count usually
The US is a highly sensitive modality in
elevated owing to peritonitis. Amylase
Fever and tachycardia are common. scanning for peritoneal free air. Recently
may be elevated; liver function test
Typically, initial low-grade fever increases attention of this technique has been a
results are variable.
over time. Peritoneal findings are almost rise, and it became a popular rapid
always present. Diffuse guarding and diagnostic test in EDs. Characteristic US Imaging Modalities
rebound tenderness are likely. “Boardlike” signs of pneumoperitoneum are the ring Plain radiography has a sensitivity
abdomen is a late sign. Bowel sounds are down artifact and enhancement of the demonstrating pneumoperitoneum
found decreased in the majority of the peritoneal stripe over the liver often ranging from 30 to 80%, thus making it is
cases. caused by fluid trapped between gas a questionable initial study when a
bubbles. When experienced hands use perforated peptic ulcer is considered
Emergency Diagnostic Tests the US its sensitivity of likely. Free air rises to its highest elevation
and Interpretation pneumoperitoneum achieves an almost in the body when the patients sit upright
An essential step of the evaluation is 93%. The US video. or in positions of left lateral decubitis for
imaging and laboratory tests. Erect chest at least 10 minutes. Thus it results in
or left lateral decubitus radiographs may Laboratory Tests increased sensitivity of the radiography.
reveal pneumoperitoneum. Bedside US Laboratory studies are generally not
Perforation suggestive findings include
may shorten the time to diagnosis and specific to diagnose. Use of these tests is
subdiaphragmatic free air, visible
surgical consultation. If these methods do valuable for the preparation before the
falciform ligament and air-fluid level.
not confirm pneumoperitoneum, the surgery and information about the patient
Radiography can be used as an initial
physician should proceed with b a s a l s t a t u s . Ty p e a n d c r o s s ,
screening exam. Thus, a patient may
computerized tomography (CT) or hemoglobin/hematocrit, platelet and
more expediently go to surgery with
laparotomy options by discussing with coagulation studies are the minimum
positive plain radiography. It also has the
surgery. Laboratory tests are not specific tests for this purpose. Additionally, blood
advantage of being obtainable portably at

362
the bedside with little interruption in contrast CT scans has shown as 95
Image 7.10
patient monitoring or care. On the other to 98% sensitivity in many protocols.
hand, free air cannot be identified in 30%
Can you identify free air on the CT?
of patients approximately. Thus plain
radiography is not sufficiently sensitive to
rule out perforation. Image 7.11

Can you identify free air on the X-ray?

The CT is the most sensitive and specific


imaging test in diagnosing a perforated
viscus. CT scan has the additional
fin d i n g s o f a c c o m p a n y i n g i n t r a -
abdominal abnormalities and etiological
changes. It has numerous advantages,
first of all, it can detect the small volume
Emergency Treatment
of pneumoperitoneum or retroperitoneal
Options
The initial management focuses on
free air. Secondly, it can point out that the
resuscitation, appropriate antibiotic
potential location of the perforation site
selection, and immediate surgical
and known of this may help the surgeon
consultation. Regardless of the cause, if
in operation and finally it can provide
signs of intestinal perforation with
alternative diagnoses if no
peritonitis are present, prompt emergent
p n e u m o p e r i t o n e u m i s i d e n t i fie d .
laparotomy is indicated. The critically ill
Perforated viscus detection of oral and IV
patient with a suspected perforated

363
viscus should be in the ED resuscitation area with two large IV Pediatric, Geriatric, and Pregnant
line, oxygen, and close monitoring. Crystalloid fluids and Patient Considerations
antibiotics are medical treatment essentials. In the emergency In the pediatric population, two etiologies of perforation are
setting, antibiotics should cover gram-negative, gram-positive prominent: Blunt trauma and intussusception. Vehicle-related
and anaerobic pathogens. Two sample regimens are below. Fore trauma, bicycle handlebar injuries, and seatbelt syndrome are
more regimens, and please visit given links under the references common causes of perforation secondary to blunt trauma in
and further reading. children. Intussusception refers to invagination or “telescoping” of
a part of the small intestine into itself. Most cases are children
Table 7.7 A Sample Antibiotic Regimen In Perforated Viscus younger than two years. It leads to venous and lymphatic
ANTIBIOTIC PREGNANCY congestion and subsequent intestinal edema. As a result,
DOSAGE
REGIMEN CATEGORY intestinal ischemia and perforation may occur.
Ceftriaxone B Adult: 1-2 gr IV (bid)
Pediatric: 50-75 mg/kg/day IV Perforated viscus incidence increases with advancing age.
and (bid) History of peptic ulcer disease or diverticular disease is common
Metronidazole B Adult: Loading dose: 15 mg/kg IV in elderly. Medicine-related perforation is common in the geriatric
(max: 4 grams), 7.5 mg/kg IV (bid
or tid) population. NSAIDs increase the risk of colonic perforation in
Pediatric: 15-30 mg/kg/day IV patients with diverticular disease. In an elderly with lower
(bid or tid) (Check dosage for
neonatal of children <2 kg) abdominal pain, the physician should suspect perforated
diverticulitis or appendicitis.
Meropenem B Adult: 1-2 gr (tid)
Pediatric: 20 mg/kg – 1 gr (tid)
(Check dosage for children <3 Disposition Decisions
months)
All patients require intensive care unit admission. The majority of
patients with perforated viscus require laparotomy to explore the
whole gastrointestinal system, remove spilled ingredients and
repair the lesion. Selected self-closing lesions such as a duodenal
perforation covered by omentum may be an exception. They may
not need emergent laparotomy but close monitoring and

364
intravenous large-spectrum antibiotics treatment. Alternative
methods of source control such as the use of endoscopic clips
for iatrogenic colon injury during colonoscopy are under
investigation.

References and Further Reading, click here

365
Chapter 8

Selected
OB&GYN and
Genitourinary
Emergencies
Section 1

Ectopic Pregnancy

Case Presentation
by Dan O’Brien A 24-year-old woman presents to the emergency department
with the complaint of lower abdominal pain and vaginal
spotting. She has never been pregnant. Her last normal
menstrual period was two months ago. She had light spotting
last month and states that her period this month is late.

Her history is notable for one episode of lower abdominal pain


two years ago thought to be the pelvic inflammatory disease
that responded to a two-week course of oral antibiotics. She
has no medical allergies and is not on any medications.
Review of systems and family history are unremarkable. Her
social history is significant in that she is in a monogamous
relationship and is not using birth control.

Her general appearance is that of a well-developed female


Audio is available here with a temperature of 37ºC, a blood pressure of 110/70 mm

367
Hg and a pulse of 90 bpm. An examination of her consulted. Treatment options were
abdomen reveals normal bowel sounds, no discussed.
masses, distension, organomegaly or rebound
tenderness. She is mildly tender to palpation in
the left lower quadrant. Pelvic exam reveals left
adnexal tenderness without palpable masses.
The rectal exam is normal with hemoccult
negative stool.

Pertinent lab values: urine dip pregnancy testing


is positive, quantitative serum B-hCG is 2000
mIU/mL, hemoglobin 13 gr/dL, hematocrit 40%.
She is Rh-positive. A transvaginal ultrasound
performed by the emergency physician during
the pelvic exam fails to demonstrate an
intrauterine pregnancy. There is a small amount of
fluid in the rectouterine cul-de-sac. 2 cm ectopic
pregnancy was identified.

Two large-bore IV’s were started, the patient was


crossmatched for blood and OB-GYN was

368
Introduction excluded. Once the diagnosis is It is important to determine the Rh factor
An ectopic pregnancy occurs when a entertained, the first step is to determine status of the mother. An ectopic
fertilized egg implants somewhere other whether the patient is hemodynamically pregnancy can sensitize an Rh factor
than the main cavity of the uterus. The stable. Most ectopic pregnancies are negative mother to Rh-positive fetal
true incidence worldwide is uncertain; stable on presentation. Alternatively, any blood. Obtaining a type and screen on a
however, in the United States, the woman of childbearing years with stable patient is the most efficient
incidence ranges from 2.7 to 6 deaths per abdominal or pelvic complaints and method. A baseline complete blood count
10,000 live births. Approximately 1%-2% unstable vitals should be considered to is warranted.
of pregnancies in the United States are have a ruptured ectopic pregnancy.
Other causes of abdominal or pelvic pain
ectopic and ectopic pregnancy accounts
An essential step is to determine if the with vaginal bleeding in the first twenty
for 3%-4% of pregnancy-related deaths.
patient is pregnant. The easiest method is weeks of pregnancy include abortion,
Ectopic pregnancy remains the leading
to determine the presence of the ß implantation bleeding, and gestational
cause of maternal death in the first
subunit of human chorionic gonadotropin trophoblastic disease. Abdominal or
trimester of pregnancy and is the second
(ß-hCG) in the urine or serum. Qualitative pelvic pain causes without bleeding may
leading cause of maternal mortality. Early
urines tests can be easily performed at include gallbladder disease, appendicitis,
diagnosis and appropriate management
the bedside. Urine testing is positive and hyperemesis. Urinalysis, electrolytes,
may prevent serious adverse outcomes
when ß-hCG is greater than 20 mIU/mL in and liver function studies should be
and potentially improve subsequent
the urine. Although dilute urine may considered.
fertility.
reduce sensitivity, at this level of
detection, the false negative rate will be Differential Diagnosis
Critical Bedside Actions and Abdominal or pelvic pain is a common
less than one percent. Quantitative serum
General Approach complaint in the emergency department
testing should be obtained as well since
Given the consequences of missing an
the serum ß-hCG level may assist with an extensive differential. All patients
ectopic pregnancy, all women of
subsequent disposition. who are  in childbearing age should be
childbearing years with abdominal or
suspected and investigated for
pelvic pain with or without vaginal
appendicitis, endometriosis, ovarian cyst,
bleeding must have ectopic pregnancy
369
ovarian torsion, pelvic inflammatory From the emergency physician sounds, no masses,
disease, renal  colic, and urinary  tract perspective, the differential diagnosis of a
distension, organomegaly or
infection. In the pregnant patients, woman with a positive pregnancy test
however,   intrauterine pregnancy, and abdominal or pelvic complaint is rebound tenderness. She is
implantation bleeding, threatened broad and will require, in almost all mildly tender to palpation in the
abortion, inevitable abortion, corpus instances in the first trimester, an
left lower quadrant. Pelvic
luteal cyst, molar pregnancy, and abdominal and pelvic exam.
ectopic pregnancy possibilities should be exam reveals left adnexal
Although abdominal pain is reported in
evaluated. tenderness without palpable
90% of ectopic pregnancies, vaginal
bleeding in more than half and menstrual masses. The rectal exam is
History and Physical Exam
Hints irregularities in up to 70%, none of these normal with hemoccult
symptoms narrow the diagnosis enough negative stool.”
“A 24-year-old woman
to include or exclude the diagnosis of
presents to the emergency ectopic pregnancy reliably. The classic Unfortunately, the physical exam may not
department with the complaint triad of abdominal pain, vaginal bleeding be helpful in distinguishing the ectopic

of lower abdominal pain and and amenorrhea is not specific for p re g n a n c y f ro m o t h e r c a u s e s o f


ectopic pregnancy and, in fact, occurs abdominal or pelvic symptoms. In cases
vaginal spotting. She has never more frequently with other more common of ruptured ectopic pregnancy, a patient
been pregnant. Her last normal complaints such as spontaneous may present in shock, with peritoneal
menstrual period was two abortion. Therefore, ectopic pregnancy signs on the abdominal and pelvic exam.
should be considered in any women of Vital signs are likely normal. The
months ago. She had light
childbearing age who presents with an abdominal exam may be nonspecific, the
spotting last month and states abdominal or pelvic complaint. pelvic exam may reveal normal or minor
that her period this month is cervical motion tenderness. The uterus
“An examination of her may be normal size, and there may be
late.”
abdomen reveals normal bowel minimal bleeding in the vaginal vault. In

370
fact, patients with unruptured ectopic pregnancies may present If the vaginal exam is delayed or the patient is judged low
identically as a healthy pregnancy. risk, a transabdominal exam to identify an IUP may suffice.

Emergency and Diagnostic Tests and


Image 8.1 Intrauterine pregnancy. Transvaginal image.
Interpretations Normal early pregnancy. Note yolk sac (red) and
“Pertinent lab values: urine dip pregnancy testing intrauterine gestational sac (yellow)

is positive, quantitative serum B-hCG is 2000


mIU/mL, hemoglobin 13 gr/dL, hematocrit 40%.
She is Rh-positive.”
The patient is pregnant. The primary goal at this point is to
determine if an intrauterine pregnancy (IUP) is present. From the
emergency physician perspective, an intrauterine pregnancy
proven by transabdominal or transvaginal sonography may safely
rule out an ectopic pregnancy. She is Rh positive and therefore
not at risk for alloimmunization.

Advances in ultrasound imaging and the capability of emergency


physicians to perform transabdominal and transvaginal imaging UP had been considered to exclude ectopic pregnancy. The
have enhanced patient safety and improved clinical accuracy. incidence of heterotopic pregnancy may be as low as 1 in 30,000
There is no definitive guideline regarding sequencing the pregnancies. However, it is as high as 1 in 100 in pregnants who
transabdominal and transvaginal study. In a stable patient, it is have undergone in vitro fertilization or have taken ovulation-
reasonable to perform the transvaginal ultrasound exam with the inducing drugs.
vaginal exam. The exams complement the other. If there is no IUP
identified, it is reasonable to search for free fluid in the abdomen.

371
“Transvaginal ultrasound Video shows Ectopic Pregnancy –
Image 8.3 Evidence of living embryo
on M-mode. M-mode pictorially Transvaginal Ultrasound
performed by the emergency
describes temporal changes at a
physician during the pelvic given depth on one axis while
measuring time in the second axis. Image 8.5 A pseudo-gestational
exam fails to demonstrate an The fluttering noted is cardiac sac (red), is a collection of
activity. intrauterine fluid and may be
intrauterine pregnancy. There is confused with a true gestational
a small amount of fluid in the sac. A true gestational sac is
normally embedded in the
rectouterine cul-de-sac. 2 cm endometrium rather than in the
uterine cavity, contains a yolk sac
ectopic pregnancy was typically seen at 5.5 weeks and has
a characteristic double ring or
identified.” double decidual sign at 4-6.5
weeks.

Image 8.2 Transvaginal ultrasound


of embryo in the adnexa next to the Image 8.4 A small amount of free
empty uterus fluid is noted in the cul-de-sac
(red). Don’t confuse endometrial
reaction typical of pregnancy
(yellow) for a gestational sac. An
ectopic pregnancy (green) is noted
in the adnexa.

Laparoscopy should be considered in


patients with suspected ectopic
pregnancy and nondiagnostic vaginal
ultrasound. It is both diagnostic and
therapeutic.

372
Culdocentesis involves extracting fluid consulting OB-GYN physician may I f the serum ß-hCG is below 1500 mIU/
from the rectouterine pouch posterior to consider medical management with mL, the patient is at low risk, and with the
the vagina through a needle. It has been methotrexate. The surgeon, not the concurrence with OB-GYN consultant,
supplanted by the ß-hCG and ultrasound emergency physician, should decide the the patient may be discharged with
but may be useful when ultrasound is not treatment. follow-up in two days for reexamination
available. and repeat ß-hCG levels.
The Discriminatory Zone
Emergency Treatment If the urine ß-hCG is positive, but the ß-hCG levels rise rapidly during the first

Options transvaginal ultrasound does not ten weeks of pregnancy then plateau.
“Two large-bore IV’s were started, the demonstrate an IUP, the emergency Although pathologic pregnancies often
patient was cross-matched for blood and physician should consider a concept have lower ß-hCG levels than normal
OB-GYN was consulted. Treatment known as the “discriminatory zone.” The pregnancies, there is significant overlap,
options were discussed.” discriminatory zone is the level of serum and absolute levels are not helpful in
ß-hCG above which an examiner should distinguishing a normal from abnormal
Ectopic pregnancy requires consultation be able to see an IUP. With transvaginal pregnancy. A general advisory rule is that
with OB-GYN. If the patient is unstable; ultrasound, an IUP should be seen with a ß-hCG levels double every 48 hours in a
resuscitation, urgent consultation, and ß-hCG level above 1500 mIU/mL and normal pregnancy. However, even here
laparoscopic or open surgery are with transabdominal above 6000 mIU/mL. there is significant variation and some
indicated. In this instance, IV access was If the serum ß-hCG is above 1500 mIU/ controversy. In stable patients, serial
established, the patient was typed and mL and transvaginal sonography does measurements and repeated sonography
cross-matched for blood. The OB-GYN not identify an IUP, consultation with OB- may be used to raise or lower suspicion
surgeon elected laparoscopic surgery. GYN is essential. These patients should of an occult ectopic pregnancy.
Ectopic pregnancy was confirmed in the be presumed to have an ectopic
left fallopian tube which was successfully The ß-hCG level representing the
p re g n a n c y. A d d i t i o n a l d i a g n o s t i c
removed. discriminatory zone is dependent on the
techniques may include laparoscopy or
technique and capabilities of the
If the patient is stable and the ectopic is dilation and curettage.
examiner and equipment. The
early (ß-hCG levels < 3000 mIU/mL) the
373
discriminatory zone should not be used to determine viability or A pregnant patient with a ß- hCG below the discriminatory zone
treatment plan associated with an IUP. and without evidence of IUP may be discharged with the
concurrence of the consulting OB-GYN surgeon for close
Documented ectopic pregnancies have presented with a ß-hCG
outpatient follow up and serial exams. A portion of these patients
level below test resolution. Therefore do not forgo transvaginal
will subsequently be diagnosed as an IUP, an ectopic pregnancy,
ultrasound investigation in any pregnant patient with a serum ß-
or a threatened, incomplete or completed miscarriage.
hCG below 1500 mIU/mL.

Disposition Decision
If an ectopic is diagnosed in an unstable patient, that patient will References and Further Reading, click here
require resuscitation, urgent consultation, and surgical
intervention.

If an unruptured ectopic is diagnosed in a stable patient, the


consulting OB-GYN surgeon may consider surgical or medical
intervention based on the characteristics of the ectopic, patient
risk factors and stability. Many of the patients managed medically
may be discharged with close follow up and strict return
instructions. Patients treated with methotrexate who return may
represent a unique challenge as the pain associated with
methotrexate-induced tubal abortion may not be readily
distinguishable from a ruptured ectopic.

A consulting OB-GYN should evaluate a pregnant patient with a


ß-hCG above the discriminatory zone but without evidence of IUP.
Laparoscopic surgery is often diagnostic and, in the case of an
ectopic pregnancy, is often therapeutic.

374
Section 2

Tubo-Ovarian Abscess

Case Presentation
by Matthew Lisankie, Charlotte Derr, Tomislav A 19-year-old female presents to the emergency department
Jelic
(ED) complaining of 48 hours of worsening, stabbing left lower
quadrant abdominal pain. The patient notes an intermittent,
foul-smelling vaginal discharge for the past week. She also
endorses fever, nausea, vomiting, dyspareunia, dysuria, and
generalized fatigue. The patient is sexually active with one
male partner and uses combination OCPs in conjunction with
inconsistent utilization of condoms. She denies vaginal
bleeding, fevers, jaundice, vomiting, constipation, or diarrhea.
Her last menstrual period (LMP) ended 16 days ago and was
typical of her usual menses. The patient has a history of
menarche at 14 and coitarche at 17. She denies any use of
tobacco but admits intermittent alcohol and marijuana use.
She has no past medical or relevant family history. There are
Audio is available here no known drug allergies.

375
Physical exam reveals a well- with no visible blood products. Introduction
Tubo-ovarian abscess (TOA) is a walled-
developed female in mild Cervical motion tenderness
off infection of adnexal structures,
discomfort but no acute and pain on palpation of typically the fallopian tubes or ovary and
distress. Her vitals are bilateral adnexa are present. occasionally adjacent intra-abdominal

unremarkable except for a Left adnexa is more tender and structures. It is a potentially life-
threatening progression of the pelvic
temperature of 38.5 and a has a palpable mass on it.
inflammatory disease (PID). Thus, TOA
heart rate of 102. Her and PID share a great deal of
abdominal exam reveals pathophysiology and clinical
manifestations. TOA is common in
moderate tenderness to
women of childbearing age, who have
palpation, worse in the left multiple sexual partners and a history of
lower quadrant, with no PID [3]. Transvaginal ultrasound is the first
choice to diagnose TOA. But, CT remains
rebound tenderness. There is
an important tool in determining further
no costovertebral angle management. [2] Up to 70-80% of
tenderness, Rovsing sign or appropriately selected TOA cases resolve
McBurney point tenderness. with appropriate antibiotics alone.
However, many patients require either
External genitalia is
image-guided drainage or surgical
unremarkable. A pelvic exam exploration for resolution.
demonstrates foul purulent
Critical Bedside Actions and
discharge in the vaginal vault
General Approach
emanating from the cervical os Assessment of the undifferentiated
patient with a high suspicion for tubo-
376
ovarian abscess begins with the Consider the following critical actions to catastrophes can often lead to severe
measurement of vital signs and make a diagnosis and initiate effective metabolic derangements.
establishment of vascular access. treatment:
• Consider checking hepatic and
Continuous cardiac and pulse oximetry
• Obtain a urine specimen to rule out pancreatic function assays. Abnormal
monitoring is often prudent, especially if
cystitis and pyelonephritis. It may values may suggest other etiologies
the patient appears distressed or toxic, or
provide evidence for or against including biliary obstruction,
has vital signs that fulfill Systemic
nephrolithiasis. It may determine pancreatitis, Fitz-Hugh-Curtis
Inflammatory Response Syndrome (SIRS)
pregnancy status and therefore, change syndrome, or hepatitis.
criteria.
the choice of radiologic modalities.
Next, prepare for the pelvic examination
Rapid determination of the patient’s
• Obtain basic lab work, namely by obtaining:
pregnancy status is critical. A positive
complete blood count (CBC), blood
result warrants immediate rule out of • A lighted speculum to inspect the
urea nitrogen (BUN) and creatinine.
ectopic pregnancy and septic abortion. vagina and cervix
CBC may provide information on the
Additionally, it determines the appropriate
infection and anemia. BUN and • Chlamydia/Gonorrhea PCR swabs
interventions and diagnostic modalities. A
creatinine determine if the patient can
thorough history and physical including
safely undergo contrasted imaging • Wet prep swab
pelvic exam are crucial to timely
studies if required.
diagnosis and intervention. If available, • Lubricant
bedside transabdominal and • Check serum lactate and venous blood
• Gloves
endocavitary ultrasound can be a gas if there is a concern for sepsis.
powerful adjunct to the initial assessment A chaperone/assistant is recommended
of the patient with undifferentiated low • Obtain blood and other indicated for both male and female examiners.
abdominal or pelvic pain. cultures if the patient is exhibiting signs
Always be sure to discuss the major
of SIRS
points of and rationale/risks/benefits/

• Check electrolytes as hemorrhage, alternatives for the exam with the patient.
intra-pelvic, and intra-abdominal
377
The initial pelvic exam is critical as it diagnose these may lead to increases in should bear a high index of suspicion
leads the investigation and provides morbidity and mortality. More common in females of reproductive age.
valuable information to consulting but less immediately-threatening
The emergency physician should inquire
physicians. At the minimum, the diagnoses include constipation,
about the sexual history of the patient.
emergency physician should note the gastroenteritis, colitis, diverticulitis,
Multiple sexual factors and non-safe sex
general appearance of external genitalia, ruptured ovarian cyst, uncomplicated
practices are among the risk factors.
any bleeding, discharge, or odors, the pelvic inflammatory disease,
appearance of the cervix and caliber of nephrolithiasis, urinary tract infection. Symptoms related to TOA are abdominal
the os, presence or absence of any Finally, consideration of pelvic pain, fever, vaginal discharge, nausea,
cervical motion tenderness, and malignancy, particularly in the post- and abnormal vaginal bleeding. Physical
characteristics of the bilateral adnexa, menopausal patient with suspicion for examination features related to TOA are
making note specifically of mass, TOA is recommended. mucopurulent discharge, cervical motion
unilateral tenderness, and description of tenderness, and uterine or adnexal
ovaries if palpable. History and Physical Exam tenderness.
Hints
Differential Diagnosis Presentation of the patient with TOA can Emergency Diagnostic Tests
A chief complaint of acute lower vary from the post-menopausal woman and Interpretation
abdominal pain in the female of with only vague GI complaints to the Ultrasound is the first imaging modality to
reproductive age necessitates a rapid rule teenage patient with septic shock and evaluate the female reproductory system
out of multiple surgical and gynecologic peritonitis from a ruptured abscess. due to low-cost and lack of ionizing
emergencies. The emergency physician radiation. Developing a facility with
The typical presentation of TOA consists
should consider ruptured ectopic bedside ultrasound can have a profound
of abdominal pain, pelvic mass on
pregnancy, appendicitis, and TOA in the impact on the patient’s course in the ER.
examination, fever, and leukocytosis.
undifferentiated patient. Likewise, A skilled operator with access to an
However, a significant portion of patients
diagnoses including bowel obstruction, endocavitary probe can incorporate
with TOA may lack one or more of these
ovarian torsion, urinary obstruction diagnostic imaging into the initial pelvic
features. Therefore, emergency physician
should be excluded early as failure to exam within the first minutes of
378
evaluation, and potentially shorten the Pregnancy testing is perhaps the most •Unasyn (cat B) 3g IV q6h +
time to effective antibiotics, definitive guiding first step in both diagnosis and Doxycycline (cat D) 100 mg IV/PO q12h
imaging, consultant evaluation, and treatment. A positive result limits the use
• Imipenem-Cilastatin (cat C) 500 mg 16h
disposition. of CT, raises the possibility of ruptured
ectopic pregnancy, and limits the
The computerized tomography (CT) with Procedures
clinician’s armamentarium of antibiotics. Evacuation of the abscess will typically
oral and IV contrast has improved
be performed by either an interventional
sensitivity. The other advantages of CT Medications
radiologist or gynecologist, depending on
are to show more detailed anatomy and The mainstay of the medical therapy is
abscess characteristics and specific
rule in or rule out other differential antibiotics. TOA is typically a
institutional policies.
diagnoses. polymicrobial infectious process and
necessitates initial broad coverage for
Emergency Treatment Disposition Decisions
anaerobes, aerobes, gram-positive, and Signs of peritonitis, sepsis, or toxic
Options gram-negative bacteria. appearance suggest ruptured abscesses.
Initial management of patients with TOA
A summary of common empiric antibiotic These unstable patients need immediate
includes stabilization and timely
regimens and respective pregnancy surgical intervention. Stable patients with
diagnosis. Access and frequently
categories is as follows: a high suspicion or radiographic evidence
reassess airway, breathing, and
of TOA warrants ward admission for IV
circulation (ABC). Establish IV access to
• Cefotetan (cat B) 2 g IV q12h + antibiotics and serial evaluation by a
draw blood, enable intravenous contrast
Doxycycline (cat D) 100 mg IV/PO q12h surgeon or gynecologist. Discharge from
CT and administer medications. Sound
the emergency department and
medical management is the primary • Cefoxitin (cat B) 2 g IV q6h +
outpatient follow up are not
concern of the emergency physician. Doxycycline (cat D) 100 mg IV/PO q12h
recommended because of the risk of
Medical management primarily includes
• Clindamycin (cat B) 900 IV q8h + sepsis, peritonitis, and loss of fertility.
supportive care (e.g., fluid resuscitation,
Gentamicin (cat D) 2mg/kg IV (load)
antiemetics, analgesics.) and broad- References and Further Reading, click
then 1.5 mg/kg q8h
spectrum antibiotics. here

379
Section 3

Testicular Torsion

Case Presentation
by Sujata Kirtikant Sheth A 16-year-old male was sleeping when he suddenly started to
feel left sided lower abdominal pain. He continued to bear
through the pain for another 30 minutes until he started to
vomit. At this time he decided to go to the nearest hospital,
which is about 15 minutes away. When he reached the
hospital his vital signs were as follows: BP: 120/60 mmHg,
HR: 120 bpm, RR: 20 bpm, Temp 36.5C, Pain 10/10 and
SpO2 was 100% on room air. Physical shows a swollen right
scrotum with significant tenderness. What is the next step?

Audio is available here

380
Critical Bedside Actions and • W h a t a re t h e o t h e r a s s o c i a t e d Nausea or vomiting, pain duration of

General Approach symptoms? less than 24 hours, high position of the


• Perform full physical examination testis or abnormal cremasteric reflex.
• Are you sexually active?
including a genital examination
• Are all your immunizations up to date?
Differential Diagnosis
• Induce cremasteric reflex- stoke the • Epididymal appendage torsion
inner thigh to see if the cremasteric Signs and Symptoms
• Epididymitis or orchitis
muscle contracts and the testicle The diagnosis of testicular torsion is
elevates based on signs and symptoms. The most • Hydrocele
classic presentation is a severe testicular
• Perform ultrasound but do not delay • Idiopathic scrotal edema
pain within 6 hours. However, some
care
patients have milder, less acute pain or • Idiopathic testicular infarction
• Detorse the testis from medial to lateral no scrotal pain. Alternatively, abdominal
pain or inguinal pain may be present. The • Testicular torsion
• Consult urology pain may start at rest, while asleep, or
• Testis tumor
with physical activity. Testicular torsion
History and Physical Exam
rarely may be bilateral. • Traumatic hematoma
Hints
Key questions to establish an accurate The most common physical findings are • Urolithiasis
diagnosis are: testicular tenderness and absence of the
Acute scrotum warrants the exclusion of
cremasteric reflex. Note that 10% of
• What time did this occur? the testicular torsion. The emergency
proven testicular torsion cases have a
physician should bear a high index of
• What was the patient doing at the time? cremasteric reflex. A negative Prehn’s test
suspicion because the clinical signs and
(relief of pain with elevation of the testes)
• Have they had pain on the same side symptoms vary widely between patients.
is another sign to look for, but it is not
before? No single details in the history, physical
100% reliable. All patients with testicular
or imaging confirm or exclude the
• Was there any trauma involved? torsion have one or more of the following:
diagnosis with 100% certainty.
381
Pathophysiology Urine analysis may prove other
Image 8.6
Two main types of testicular torsion are diagnoses. However, the presence of
intravaginal and extravaginal. The infection in urine does not exclude the
extravaginal torsion occurs in the testicular torsion.
perinatal period before the fixation of the
Pre-operation labs if required by the
tunica vaginalis. Intravaginal torsion refers
surgical team
to when the testis twists inside the tunica
vaginalis. Bell Clapper deformity refers to
Emergency Treatment
a partial or complete fusion of the tunica
Options
vaginalis along the epididymis. It causes
an excessive testicular movement and Initial Stabilization
about 12% of the testicular torsions. There are a few steps to perform in the
Which testicle has no “blood flow” on the ED. The emergency physician may try to
Emergency Diagnostic Tests below ultrasound image? detorse the testicle manually at the
and Interpretation bedside if the diagnosis is likely. Most
Scrotal ultrasound is the method of testicular torsions are from lateral to
choice. The ultrasound shows a medial, so the physician should move the
Image 8.7
hypoechoic and enlarged testis in testicle from a medial to a lateral position
patients with testicular torsion. Reduced to detorse the testicle. The patient’s pain
b l o o d fl o w a n d p a r e n c h y m a l should relieve if the detorsion is
heterogeneity are the other signs of successful. Rarely, if the patient’s torsion
testicular torsion. is from medial to lateral, the physician
should detorse in the opposite direction.
The testicular ultrasound shows bilateral
Turning the testicle in the wrong direction
normal blood supply in doppler
increases the pain.
investigation (below).

382
Torsion of the appendix is more common the intervention in patients with high Referral
than torsion of the spermatic cord. clinical suspicion. The emergency If you have deemed the patient as not
Torsion of the appendix is managed physician may attempt a bedside having a testicular torsion you can refer
conservatively unlike the torsion of the ultrasound while waiting for the urologist. them to urology a week later to see if
spermatic cord. Torsion of the spermatic Consider drawing blood for operation. their symptoms have resolved. Please
cord requires early surgical exploration provide patients with strict information on
because this will result in ischemia, Disposition Decision when to return to the emergency
damage or loss. department. If they start having pain
Admission Criteria
Patient with testicular torsion present again, increased vomiting, inability to
Medications urinate, fever, any other worrisome
within 6 hours should undergo an
Testicular torsion is a painful condition.
emergent surgery. Patients with testicular symptoms they need to return.
Please do not ignore and treat the
torsion for more than 48 hours should be
p a t i e n t ’s p a i n w i t h p r o p e r p a i n
admitted to the urology ward unless the
medication, paracetamol, ibuprofen or
patient is hemodynamically unstable. References and Further Reading, click
with stronger alternatives. In a pediatric
here
patient, the physician may consider
Discharge Criteria
options such as intranasal fentanyl. Patients with testicular torsion should not
I d e a l l y, o r a l m e d i c a t i o n s a re n o t be discharged from the emergency
preferable as the operation is likely. department. In a patient with intermittent
symptoms and a negative ultrasound, if
Procedures
the urologist does not admit the patients
If the emergency physician suspects
for observation, it is safer to observe the
testicular torsion, an emergent urological
patient in the ED for repeating symptoms.
consult is indicated. The urologist
If the symptoms occur again, repeating
determines the need for ultrasound or
the ultrasound and urology consultation is
emergency surgery. Ultrasound or any
sensible.
other diagnostic tests should not delay

383
Chapter 9

Selected
Neurological
Emergencies
Section 1

Approach To Patient With Stroke

Case Presentation
by Matevž Privšek and Gregor Prosen A 56-year old female is brought to the ED by the paramedics
due to weakness in her left arm and left leg. She is conscious,
GCS 15, painless, normal skin color. Vitals are: BP 132/84,
pulse 78/min, 14 breaths/min, SpO2 99 %, temperature 36,4
°C, blood glucose 5,4 mmol/L. She says that weakness
started about 2 hours ago, while she was watching TV when
she suddenly realized she was unable to pick up a glass of
juice. She wanted to stand up and almost fell because her
right leg did not move. She thought it would go away, but it
did not, so she called an ambulance. She denies dizziness,
vertigo, nausea, vomiting, headache, visual disturbances. She
is otherwise healthy, not taking any medications or drugs. She
smokes half a pack of cigarettes daily.

The focused neurological exam is performed: pupils are equal


and reactive, bulbomotorics and facial mimic seem

385
appropriate except slight drift of right mouth Introduction
Stroke or cerebrovascular accident (CVA) is a syndrome of any
angle. She also has decreased muscle power in
vascular injury that diminishes cerebral blood flow (CBF) to a
her left arm as well as slightly decreased muscle specific region of the brain, causing ischemia and thereby
power in her left leg. She denies any sensory consequently causing focal neurologic impairment. Emergency

deficits. The rest of physical exam is physicians’ main goals are early recognition of stroke symptoms,
objectification of complaints and prompt diagnostics and
unremarkable.
treatment.

You set up an intravenous cannula, draw some According to some data, stroke is the third leading cause of death
blood for testing, and order emergency non- and a leading cause of long-term disability in the United States.
Around 2-4 % of hospital admissions are due to potential strokes.
contrast head CT scan, due to a high suspicion of
Depending on the cause of stroke, in-hospital mortality rates vary
an acute CVA. The results of the CT scan are between 5-10 % for ischemic stroke and up to 45 % for
back in 35 minutes: radiologist describes no hemorrhagic stroke. Up to 50-70% of stroke survivors regain
functional independence, while 15-30% be permanently disabled
intracranial hemorrhage or ischemic areas. A
and another 30% eventually require institutional care.
neurologist is consulted; upon repeated
Etiopathogenesis. 80 % of all strokes are ischemic in origin; the
examination, he advises highly for acute ischemic
rest are hemorrhagic. In the ischemic stroke, a clot stops the
stroke, most likely due to occlusion of the right blood supply to a specific area of the brain. However, in
middle cerebral artery. You immediately start with hemorrhagic stroke, blood leaks into brain tissue. It is highly
thrombolysis and transfer her to the neurology important to differentiate between them since treatment is
completely different.
ward.
In the ischemic stroke, a causative clot can originate from large
blood vessels of the brain (thrombus) or elsewhere in the body
(usually from the heart due to atrial fibrillation; embolus). Rarely,
386
the cause of ischaemic stroke is irreversible changes have not yet should have their complaints
hypoperfusion of the brain, due to a o c c u r re d . W i t h f u r t h e r o c c l u s i o n objectified by a focused neurologic exam,
systemic problem (e.g., myocardial irreversibility and scope of cerebral and efforts should be made to perform
infarction, dysrhythmias). In hemorrhagic infarction increase. Studies have shown urgent diagnostics.
stroke, the main causes are intracerebral that occlusion longer than 6 hours leads
(ICH) and subarachnoid hemorrhage to irreversible neurologic deficits. Differential Diagnosis
(SAH). A physician must be well aware of “stroke
In hemorrhagic stroke, events beside mimics,” which are defined as non-
Brains are highly sensitive to any alterations in CBF, such as red blood cells vascular diseases that present with
alterations in the blood supply of oxygen lysis and increased permeability of the stroke-like symptoms. Since the majority
and glucose that are needed for their blood-brain barrier lead to brain edema of strokes are treated with thrombolysis,
metabolism. Immediate alterations in CBF and secondary injury. accurate diagnosis due to harmful effects
and cellular homeostasis follows a stroke. of thrombolytics (significant intracranial
A complete interruption of CBF (rare) Critical Bedside Actions And bleed in 1 %) is essential.
causes loss of consciousness within 10 General Approach
seconds and death of pyramidal cells Regardless of the patients’ chief Possible stroke mimics which may be

follows within minutes. More often, complaint emergency physicians’ first misdiagnosed as a stroke;

collateral circulation helps to maintain task is to rapidly asses patients’ condition


• Migraine
some CBF to the ischaemic region. When and vital signs (do not forget blood
CBF drops below a certain point, loss of glucose!), and stabilize them, if • Seizures
electrical activity of the affected area necessary. After patients’ condition is
stable, we continue with establishing • Psychiatric
occurs, which is clinically seen as a
neurologic deficit (but the brain cells chief complaint, focused, but thorough
• Syncope
remain viable; integrity and function of the history and physical exam, setting the
neuronal membrane are intact). Area of working diagnosis and list differential • Sepsis

the brains with electrical silence but diagnoses, and diagnostic and treatment
• Brain tumor
viable cells is called penumbra; plan. All patients with a suspected CVA

387
• Metabolic P = Psychiatric R = Rhythm disturbance

• Transient global amnesia S = Sepsis, stroke, shock, Syncope T = ventricular Tachycardia

• Labyrinthitis

Depending on the affected area of the HEAD HEART VESSELS: Vascular causes are VESSELS:
brain and type of stroke CVA can present Syncope causes, by system
V = Vasovagal
with a vast list of chief complaints: altered
mental status, confusion, syncope, E = Ectopic (reminds one of hypovolemia)
CNS causes include HEAD:
weakness, dizziness, vertigo, ataxia,
aphasia, diplopia. S = Situational
H = Hypoxia/ Hypoglycemia

Differential diagnosis of some stroke- S = Subclavian steal


E = Epilepsy
related chief complaint
E = ENT (glossopharyngeal neuralgia)
A = Anxiety
AEIOU TIPS: Causes of altered L = Low systemic vascular resistance
mental status D = Dysfunctional brain stem
(Addison’s, diabetic vascular neuropathy,
A = Alcohol (basivertebral TIA)
calcium channel blockers, anti-
E = Epilepsy, electrolytes hypertensives)

I = Infection Cardiac causes are HEART: S = Sensitive carotid sinüs

O = Overdose H = Heart attack History And Physical


E = Embolism (PE)
Examination Hints
U = Urea
The thorough history must be quickly
T = Trauma A = Aortic obstruction (IHSS, AS or obtained and focus neurological, and
myxoma) general clinical exam must be performed.
I = Insulin H i s t o r y s h o u l d i n c l u d e t h o ro u g h

388
“dissection” of the complaint (use
Table 9.1 Guide For A Quick Neurological Exam
modified SOCRATES and SAMPLE),
especially the exact time and rate of
STEP COMMENTS
symptom onset (e.g., sudden onset
suggest an embolic or hemorrhagic
mental status “fogs”
cause, while gradual onset suggests family history, orientation, general info, spelling (back & forth), also
thrombotic stroke or hypoperfusion). It is count backwards from 100 by 3, repeat 7 digit number, recall 3 objects
after few minutes
also essential to identify any risk factors
cranial nerves CN 1: smell tobacco or soap
for thrombotic (hypertension, diabetes, CN 2: visual acuity, gross visual field, ophthalmologic (background)
coronary artery disease) or embolic cause exam
CN 3, 4, 6: pupillary light response, lateral and vertical gaze
(atrial fibrillation, valve replacement, CN 5: double simultaneous stimulation, also corneal reflex
recent MI). CN 8: does he/she hear fingertips moving near ears
CN 9, 10: gag reflex
The focused neurological exam can be CN 11: shoulder elevation
CN 12: stick out tongue
performed within 4 minutes:
motor drift of upper (and lower, if indicated) extremity
hand grasp
• check mental status, toe and foot dorsiflexion
additional: assessment of individual muscles
• cranial nerves,
sensory double simultaneous stimulation with needle pin on hands and feet
proprioception in big toe
• motor and sensory function, additional: check involving dermatomes, light touch, vibration

• coordination, and coordination finger-to-nose


heel-to-sheen
• reflexes rapid alternating movements of hand and feet
additional: Romberg, tandem gait
reflexes biceps (C5-6), triceps (C6-7), knee (L2-4), ankle (S1)
Babinski response
additional: Kernig and Brudzinski signs

provided by authors
389
In comatose patients, we can perform modified (neurological)
exam: vital signs, drop hand overhead, pupils, abnormal eye Table 9.2 Cincinnati Prehospital Stroke Scale
movements, grimacing, withdrawal from noxious stimuli, Babinski
response. FACIAL DROP ARM DRIFT SPEECH

How to clinically differentiate between ischemic and hemorrhagic person should person should close eyes person should repeat
stroke? Despite clues and suggestions for one cause of smile or show and straight out arms in a simple sentence
his/her teeth front for 10 seconds
symptoms or another, clinical differentiation alone is unreliable! A
patient with hemorrhagic stroke typically complains about normal: both normal: both arms move normal: repeats the
sides of face equally or not at all sentence using the
headache, sudden onset of symptoms that are gradually move equally correct word and no
worsening, nausea and vomiting. Clinical exam often reveals slurring
decreased level of consciousness, hypertension, bradycardia,
seizures, meningism, fever. Often patients with hemorrhagic Emergency Diagnostic Tests And
stroke present with similar focal deficits as in ischemic stroke, but Interpretation
tend to look sicker. Other clues suggestive of hemorrhagic cause As soon as possibility of stroke has been established, the patient
are uncontrolled hypertension, use of anticoagulants, has to be transferred to a facility where emergency non-contrast
coagulopathies (advanced liver disease), known vascular head CT scan can be performed, mainly to exclude hemorrhage
malformations, brain tumors. as a cause of symptoms so proper treatment can commence.

In prehospital setting Cincinnati Prehospital Stroke Scale (CPSS) Laboratory tests are directed to exclude possible stroke mimics
is highly useful tool to diagnose a potential stroke; if any of tests and should include blood glucose, complete blood count, basic
is abnormal, it suggests possible stroke and this patient should metabolic panel, cardiac enzymes, and coagulation studies, as
be transferred to hospital as soon as possible. CPSS with 1 well as EKG and chest X-ray.
abnormal finding has 72 % probability of ischemic stroke and 85
American Heart Association recommends that workup should be
% probability if all 3 tests are abnormal.
completed within 3 hours between symptoms onset and
beginning of thrombolysis.

390
An NIH Stroke Scale/Score calculator is a
Table 9.3 Recommended Time Frame Table 9.4 Criteria To Become A
In Management Of Ishemic Stroke useful tool for quantifying neurologic Candidate For Thrombolysis
deficit.
QUESTIONS YES NO
ACTION TIME FRAME
A diagnosis of an acute stroke is often all answers must be »yes«
based solely on the patients’ history and is the time of onset of YES
Symptom onset to < 3 hours symptoms clearly defined?
physical exam, since head CT does not
ER doors
show an acute infarction until at least 6 will thrombolysis be possible YES
within 4,5 hours of onset?
hours after the occlusion (but it helps to
Door to lab work 45 minutes has the patient had a good YES
completed rule out intracranial hemorrhage). quality of life until now?

Emergency treatment options. Treatment


Door to non- 25 minutes
contrast head CT of stroke is based on the cause of all answers must be »no«
ordered symptoms. If an ischemic stroke is seizures at the beginning of NO
confirmed (or suspected and hemorrhage symptoms?
Door to CT being 45 minutes
read has been ruled-out) the next step is to is this minor/isolated disability NO
(e.g. just disarthria, ataxia)?
determine if a patient is a candidate for
are symptomes rapidly NO
Door to decision to 45 minutes thrombolysis. improving?
give thrombolysis
gastro-intestinal or genito- NO
urinary bleed within the last 3
weeks?
Door to drug 60 minutes (& < 3
administration hours from onset) larger surgery within the last 2 NO
weeks?

American Heart Association recommendations prior ischemic CVA or severe NO


head injury in the past 3
months?

prior intracranial bleed anytime NO


in the past?

BP > 185/110 (despite therapy)? NO

391
Contraindications for • Recent arterial puncture (at the o f stroke increases with age, so
noncompressible site) physicians have to maintain a high level
thrombolysis
Absolute contraindications If a patient fulfills above criteria for of suspicion for stroke when managing
thrombolysis, recombinant t-PA is given undifferentiated geriatric patient who is
• Hemorrhagic (or unknown) CVA anytime at a dose of 0,9 mg/kg IV up to a “just unwell.” Stroke in pregnant patient
• Ischemic CVA within the past six maximum of 90 mg. 10 % of the dose is can occur due to predisposition to
months
given as a bolus, followed by a 60 hypercoagulability, but one must be
• Malignancy of CNS minutes infusion. Also, blood pressure aware that thrombolysis is
must be treated before thrombolysis if it contraindicated in pregnancy until the
• Major (head) trauma or surgery within
the past three weeks exceeds 185/110 (use captopril 12,5 mg first week after Labor has passed.
SL).
• Gastrointestinal bleed in the last month
Disposition Decisions
• Known coagulopathy When hemorrhagic stroke is suspected All patients suffering acute stroke should
(or confirmed), one must do an urgent be admitted to stroke care unit or
• Aortic dissection
consultation with neurosurgeon to decide intensive care unit, depending on local
on further treatment options and plan policy, abilities and patients’ condition.
Relative contraindications (e.g., craniotomy and evacuation of Patient has to be on a monitor and have
hematoma, endovascular aneurysm frequent assessment of neurologic
• TIA in the past six months repair). system.
• Peroral anticoagulation therapy
Pediatric, Geriatric,
• Pregnancy up to less than a week
postpartum Pregnant Patient, And Other
References and Further Reading, click
Considerations here
• Refractory hypertension
Stroke in pediatric population is an
• Advanced liver disease extremely rare occasion, but all the

• Infective endocarditis principles for adults apply for the


pediatric population. Generally, incidence
• Active peptic ulcer
392
Section 2

Acute Ischemic Stroke

Introduction
by Fatih Büyükcam Patients with stroke present with sudden onset of paresis, sensory deficits, visual
loss or visual field defects, diplopia, dysarthria, facial droop, ataxia, vertigo,
aphasia and altered mental status. These symptoms and signs may be seen alone
or in combination.

Acute ischemic stroke is a type of brain ischemia due to thrombosis, embolism or


systemic hypoperfusion. Atherosclerosis, dissection, fibromuscular dysplasia,
arteritis, vasculitis, non-inflammatory vasculopathy, and vasoconstriction may
cause thrombosis. The most common source of emboli is cardiac or aortic
diseases.

Management principles include stabilization, diagnosing and reversing the cause


and decreasing harm to the patient. All patients with acute ischemic stroke have to
be evaluated for suitability to thrombolytic therapy or invasive procedures.

393
Eligibility criteria for • Recent intracranial or intraspinal •Heparin use within 48 hours and an
treatment of acute ischemic surgery abnormally elevated aPTT*

stroke with recombinant • Arterial puncture at a noncompressible • Current use of a direct thrombin
tissue plasminogen activator site in the previous seven days inhibitor or direct factor Xa inhibitor with
(alteplase) evidence of anticoagulant effect by
Clinical
laboratory tests such as aPTT, INR,

• Symptoms suggestive of subarachnoid ECT, TT, or appropriate factor Xa


Inclusion Criteria
hemorrhage activity assays
• Clinical diagnosis of ischemic stroke
causing measurable neurologic deficit Head CT scan
• Persistent blood pressure elevation
• Onset of symptoms <4.5 hours before (systolic ≥185 mmHg or diastolic ≥110
• Evidence of hemorrhage
beginning treatment; if the exact time of mmHg)
stroke onset is not known, it is defined • Evidence of a multilobar infarction with
• Serum glucose <50 mg/dL (<2.8 mmol/
as the last time the patient was known hypodensity involving >33 percent of
L)
to be normal the cerebral hemisphere
• Active internal bleeding
• Age ≥18 years Relative Exclusion Criteria
• Acute bleeding diathesis, including but • Only minor and isolated neurologic
Exclusion Criteria not limited to conditions defined in signs
Historical
Hematologic'
• Rapidly improving stroke symptoms
• Significant stroke or head trauma in the
Hematologic
previous three months • Major surgery or serious trauma in the
• Platelet count <100,000/mm3 * previous 14 days
• Previous intracranial hemorrhage
• Current anticoagulant use with an INR • Gastrointestinal or urinary tract bleeding
• Intracranial neoplasm, arteriovenous in the previous 21 days
>1.7 or PT >15 seconds*
malformation, or aneurysm

394
• Myocardial infarction in the previous warfarin, a direct thrombin inhibitor, or a •Severe aphasia: ≥2 on NIHSS
three months direct factor Xa inhibitor), (3) use of question 9, or
anticoagulants is not known. For patients
• Seizure at the onset of stroke with • Visual or sensory extinction: ≥1 on
without recent use of oral anticoagulants
postictal neurologic impairments NIHSS question 11, or
or heparin, treatment with intravenous
• Pregnancy tPA can be started before availability of • Any weakness limiting sustained effort
coagulation test results but should be against gravity: ≥2 on NIHSS question 5
Additional Relative Exclusion discontinued if the INR, PT, or aPTT or 6, or
Criteria for Treatment from 3 to exceed the limits stated in above.)
4.5 Hours from The Symptom • Any deficits that lead to a total NIHSS
Onset The available data suggest that under >5
• Age >80 years some circumstances – with careful
consideration and weighting of risk-to- Differential Diagnosis
• Oral anticoagulant use regardless of benefit – patients may receive fibrinolytic Syncope, hypoglycemia, drug toxicity,
INR therapy despite one or more relative s e i z u re , i n t r a c r a n i a l h e m o r r h a g i c
contraindications. In particular, there is conditions can be misdiagnosed as acute
• Severe stroke (NIHSS score >25)
now consensus that patients who have a ischemic stroke.
• Combination of both previous ischemic persistent neurologic deficit that is
stroke and diabetes mellitus potentially disabling, despite History and Physical
improvement of any degree, should be Examination Hints
(* Although it is desirable to know the Most important data of the history is the
treated with tPA in the absence of other
results of these tests, thrombolytic time onset of symptoms because this is
contraindications. Any of the following
therapy should not be delayed while the main data that determine the eligibility
should be considered disabling deficits:
results are pending unless (1) there is for thrombolytic therapy. If the symptom
clinical suspicion of a bleeding • Complete hemianopsia: ≥2 on NIHSS onset time is not known, the time the
abnormality or thrombocytopenia, (2) the question 3, or patient was last awake and free of stroke
patient is currently on or has recently symptoms is accepted as symptom onset
received anticoagulants (eg, heparin,
395
time. Also, other important data have to controlled for suspicion of trauma. Emergency Tests and
be asked like co-morbidities, Neurological examination findings give us Imaging Studies
medications, head trauma and prior some clues about the affected or Hypoglycemia may mimic a stroke.
stroke. obstructed vascular region. Therefore, fingertip blood glucose
measurement is one of the initial essential
Stabilization is a priority in every critically Next step is to test motor and sensory
steps. In case of hypoglycemia (blood
ill patient. Once the patient is stable, a function. Muscle strength is assessed
glucose level <60 mg/dL), immediate
focused neurologic examination should against resistance. Pronator drift can be
intravenous glucose administration (slow
be performed. Level of consciousness tested by having the patient sit with eyes
intravenous push of 25 mL of 50%
(LOC), speech, cranial nerve (CN) closed and arms outstretched, with
dextrose) is indicated. Whether the
function, motor and sensory function, and palms toward the ceiling, for 10 seconds.
patient is hypoglycemic or not, stroke
cerebellar function are the main abilities Double simultaneous stimulation may be
probability should be kept in mind until
to assess. The physician may assess performed by simultaneously touching
exclusion.
LOC and speech in a dialogue with the the right and left limbs. The patient with
patient. The physician should also check sensory neglect may feel the right and left The basic workup should include an
pupillary size, reactivity, and eye sides individually but may ignore one side electrocardiogram, complete blood
movements to assess CN III through CN when both are touched simultaneously. count, plasma urea nitrogen, creatinine,
VI. Additionally, eyebrow elevation and electrolytes, cardiac enzymes,
The last step is to assess cerebellar
squinting, smiling, gag reflex, shoulder coagulation parameters like prothrombin
function, reflexes, and gait. Finger-to-
elevation and tongue protrusion are parts time, activated partial thromboplastin
nose and heel-to-shin evaluations,
of CN evaluation. time and an international normalized ratio
asymmetry of the deep tendon reflexes or
(INR). If there is a suspicion for other
As we are still in differential diagnosis unilateral Babinski’s sign and observing
specific diseases liver function tests,
process, we have to do full systemic routine ambulation are al informative
toxicology tests, urinalysis, blood culture,
examination including especially parts of the neurologic examination.
β-HCG, arterial blood gases, lumbar
cardiopulmonary and neurological
puncture, etc. could be evaluated.
examination. Whole body skin should be

396
The physician should send the patient to • Hypoattenuation of a focal parenchymal
Image 9.2 Ischemic stroke
imaging without waiting for the laboratory region
results.
• Obscuration of the silvian fissure and
The evaluation of a suspected stroke insular ribbon
patient starts with non-contrast brain
• The gray-white matter differentiation
computed tomography (CT). A non-
defect of basal ganglia
contrast brain CT is the fundamental
imaging to differentiate hemorrhagic or
ischemic stroke. This information Image 9.1 Hyperdense MCA sign
determines subsequent treatment.

In the acute ischemic stroke, the


sensitivity of standard non-contrast CT Computed tomographies show two regions
increases after 24 hours. However, there of ischemic stroke in the territory of the left
middle cerebral artery, involving the regions
may be some early signs of infarction in supplied by both the anterior and posterior
branches.
the first six hours.
Case courtesy of Dr David Cuete,
Radiopaedia.org. From the case rID: 26882
Early signs of acute ischemic
stroke
• Hypodensity of the lentiform nucleus
The prevalence of these signs is 61%
obscuration
Case courtesy of Dr Mohammad Taghi (±21 standard deviation). The mean
Niknejad, Radiopaedia.org. From the case
• Hypoattenuation of 1/3 or more of the rID: 20784
sensitivity and specificity of early CT
middle cerebral artery region signs are 66% (range 20-97) and 87%
(range 56-100) respectively. If the
• Hyperdensity of large vessels diagnosis is unclear, a diffusion-weighted

• Sulcal effacement magnetic resonance imaging (DW-MRI)

397
may visualize the affected area. DW-MRI
Image 9.4 CT imagining of at 48th Image 9.6 ADC view of the patient
can show ischemic changes within 3 to hour
30 minutes of onset. The apparent
diffusion coefficient (ADC) view of DW-
MRI gives a quantitative measure of the
water diffusion. The decreased water
diffusion in cytotoxic edema of acute
ischemic stroke causes a hyperintense
DW-MRI signal and a hypointense ADC
signal. Please see couple examples
below.

79-year-old female with left parieto-


occipital cortex subcortical white matter
infarct. CT imagining of at admission. Image 9.5 DW-MRI imagining of the Emergency Department
patient Management
Oxygen is not recommended for all
Image 9.3
patients. If the pulse saturation is below
94%, supplemental oxygen should be
given.

Fluids should be individualized according


to patients’ cardiovascular status, plasma
glucose, and electrolyte levels. Isotonic
fluid is the best choice because
hypotonic fluids may aggravate cerebral
edema. Both hypoglycemia and
hyperglycemia worsen the prognosis;
398
normoglycemia (60–126 mg/dL) is the • <50 kg: 12.5 mg/kg IV q4hr OR 15 mg/ •20 mg IV over 2 minutes initially,
best target point. American Diabetes kg IV q6hr; not to exceed 750 mg/dose then 40-80 mg IV q10min; total dose
Association suggests achieving glucose or 3.75 g/day not to exceed 300 mg
targets of 140 to 180 mg/dL in acute
• ≥50 kg: 650 mg IV q4hr OR 1000 mg IV • Alternative: 1-2 mg/min by continuous
ischemic stroke.
q6hr; not to exceed 4 g/day Infuse IV IV infusion; total dose of 300 mg has
Thrombolysis is done in patients eligible over at least 15 minutes been used
for thrombolytic therapy by alteplase,
Reducing the blood pressure does not Nicardipine
(Figure 10) which is a tissue plasminogen
affect the early and long-term outcome.
activator. Within 4.5 hours of symptom • Start with 5 mg/hr
However, systolic blood pressure should
onset, thrombolytic therapy may be
be kept under 185 mmHg and diastolic • If necessary, increase infusion rate 2.5
applied.
pressure under 110 mmHg to administer mg/hr every 5 minutes to a maximum
Apart from brain injury of acute stroke, thrombolytic treatment. If thrombolytic dose of 15 mg/hr.
central nervous system infection, therapy is contraindicated, the physician
subdural empyema, brain abscess and should not interfere unless systolic blood Nitroprusside sodium
any concomitant infection may cause pressure >220 mmHg or diastolic blood
• Initial: 0.25-0.3 mcg/kg/min IV infusion;
fever. Fever is associated with the pressure >120 mmHg or the patient has
may increase by 0.5 mcg/kg/min every
increased mortality rate, disability, and active coronary artery disease, aortic
few minutes to achieve desired results
extended hospital stays. The physician dissection, hypertensive encephalopathy,
should find and treat the source of fever. acute renal failure, pre-eclampsia, and • Usual range: 3-4 mcg/kg/min IV
Additionally, antipyretics may be used for eclampsia. infusion, not to exceed 10 mcg/kg/min
febrile patients, but its effect on
The first-line antihypertensive agents are Aspirin reduces the 14-day recurrence of
prognosis is still unclear.
labetalol and nicardipine, and the ischemic stroke and total mortality.
Drug of choice for fever: second-line choice is nitroprusside. Clopidogrel and aspirin-extended-release
dipyridamole can be used alone or with
Acetaminophen Labetalol
aspirin. Aspirin is the only medication that

399
is effective for the very early treatment of Prognosis
acute ischemic stroke. Presence of facial paresis, arm weakness
or drift and abnormal speech are the main
Aspirin (acetylsalicylic acid)
predictors of outcome. The NIHSS
• 160 to 325 mg/day PO (National Institutes of Health Stroke
Scale) score on admission gives a clue
• Aspirin may be given rectally for
about stroke outcome. The use of NIHSS
patients with acute stroke who cannot
score is recommended for all patients
take by the oral route.
with stroke.

• To the Asian patients with high-risk TIA


(i.e., ABCD2 score of ≥4) or minor
stroke (NIHSS score ≤3) clopidogrel References and Further Reading, click
and aspirin as dual antiplatelet therapy here
could be given for 21 days 35

Anticoagulation with heparin or low


molecular weight heparin is not
recommended. It is associated with
higher mortality and poor outcomes
compared to aspirin alone in 48 hours.
However, secondary stroke prevention is
recommended for patients with atrial
fibrillation and in case of risk for
cardiogenic embolism. For this treatment,
warfarin may be given in the first 24 hours
and continue for two weeks.

400
Section 3

Intracranial Hemorrhage

Case Presentation
by Nur-Ain Nadir and Matthew Smetana As you start your 3rd night shift in a row, paramedics bring in
a 70-year-old female with altered mental status. Patient has a
history of hypertension and diabetes mellitus. She is on
Coumadin for atrial fibrillation. She was last seen normal three
hours ago when she went to sleep. Her husband called
emergency medical services (EMS) because she was difficult
to arouse. Her blood pressure in the emergency department
(ED) is 240/120 mmHg, heart rate 45 bpm, respiratory rate 22
bpm, pulse oxygen saturation 96% and temperature 99°F.1
rectally. On physical examination, she is diaphoretic and
unresponsive to any commands. She has dilated fixed pupils.
During your assessment, she begins to vomit. What should be
your next step in management?

401
Critical Bedside Actions and • Pulses?
Image 9.7 GCS
General Approach • Blood pressure?
General Assessment: Is the patient Stable
Glasgow Coma Scale
or Unstable or in Acute distress? • Skin temperature/quality/moisture/
color?
EYE OPENING
Primary Survey
Disability – Check; 4: Spontaneously
Obtain brief chief complaint and history
3: To verbal command
of present illness from EMS providers or
• The patient consciousness – Is the 2: To pain
bystanders. Include time of onset and
patient awake, alert and oriented? 1: No response
preceding symptoms, i.e., a headache,
nausea, vertigo, syncope, chest pain, BEST VERBAL RESPONSE
• The patient’s score on the Glasgow
trauma. Coma Scale (GCS)? 5: Oriented and converses

4: Disoriented and converses


O b t a i n V i t a l S i g n s . D o n ’ t f o rg e t • For lateralizing neurological deficits? 3: Inappropriate words; cries
temperature and blood glucose.
2: Incomprehensible sounds
• For gaze preference?
Airway – Is the patient protecting the 1: No response

airway? If not – intubate. • For posturing? – Is decerebrate or BEST MOTOR RESPONSE


decorticate posture present?
6: Obeys command
Breathing – Is the patient breathing
spontaneously? If not – intubate. Exposure – Completely expose patient. 5: Localizes pain

• If the patient is breathing spontaneously Secondary Survey 4: Flexion withdrawal

but is hypoxic, provide supplemental Perform secondary survey which includes 3: Flexion abnormal (decorticate)

oxygen. complete examination, history taking


2: Extension (decerebrate)
following the primary survey and initial
Circulation – Place two large-bore stabilization. 1: No response

intravenous lines and check; Glasgow Coma Score (GCS) (Modified from Teasdale, G., &
Jennett, B. (1974). Assessment of coma and impaired
consciousness: a practical scale. The Lancet, 304(7872),
81-84.) - Please read this article to get more insight regarding
GCS. 402
History and Physical • Age- Directly proportional to ICH risk. Skin – Signs of trauma? Needle

Examination Hints tracks? medication patches? rashes?


• Structural Abnormalities- aneurysms,
Obtain a more detailed medical history
connective tissue diseases, congenital Neurological – complete full neurexam.
including surgical, travel, social and
arterio-venous malformations (AVMs)
medication history. Pay specific attention Differential Diagnoses
and family history of subarachnoid
to risk factors for cerebral hemorrhage. • Hemorrhagic stroke
hemorrhage (SAH) increases ICH risk.

Modifiable Risk Factors • Ischemic stroke


Physical Exam
• Hypertension- Results in small vessel
HEENT exam – Pupils equal round and • Infectious – abscess/meningitis
damage to deeper structures such as
reactive? Papilledema/Venous pulsations
basal ganglia and thalamus • Medications – overdose
present on fundoscopic examination?
• Alcohol- Impairs platelet function and Bruising? Raccoon’s eyes? Battle sign?
• Metabolic- HONK, hypoglycemia,
damages endothelial cell wall Hemotympanum? Ottorhea? Rhinorrhea?
electrolyte abnormalities

• Coagulopathy- Warfarin increases ICH Neck Exam – C Spine tendernes?


deformity? Bruising?
Emergency Diagnostic Tests
risk up to 2x-4x. Inherent coagulopathic
and Interpretation
situations such as liver failure increase
Cardiovascular Exam – Bradycardia? A non-contrast head CT is typically the
ICH risk
Hypertension? Chest Trauma? first neuro-imaging performed in the ED
• Tobacco Use- promotes vascular due to availability, low cost, and high
Respiratory – Spontaneously breathing?
damage. s p e e d .  A c u t e b l e e d i n g a p p e a r s
Crackles/Rales? – pattern?
hyperdense in a pre-contrast CT scan.
Non-modifiable Risk Factors Abdomen – Signs of trauma? Then the blood starts to appear isodense
• Cerebral Amyloid Angiopathy- Risk and hypodense in weeks,
Hepatomegaly? Ascites? Signs of
increases with age. Amyloid protein respectively. Radiologic signs of elevated
cirrhosis?
deposition weakens vessels’ structural intracranial pressure (ICP) include loss of
integrity compressible spaces (basal cisterns,

403
ventricles, cortical sulci), midline shift, can be performed. Protect the patient • Mannitol (1 g/kg) and
herniation, and loss of grey-white matter from hypoxia during the procedure. hypertonic saline reduce
junction. cerebral edema by producing
Impending Herniation an osmotic gradient that
Magnetic Resonance Imaging is equally • Clinical signs of elevated ICP may be
prevents water from moving
effective for the detection of acute ICH. subtle due to the brain’s intrinsic
into the cells during membrane
 However, scanning typically takes longer autoregulatory mechanisms or more
pump failure and drawing tissue
to perform. It may show the underlying severe including Cushing’s triad
water into the vascular space.
cause of ICH such as a tumor or a (Hypertension, Bradycardia, Irregular
vascular aneurysm. respirations), altered mental status, • Hyperventilation to produce
headache, vomiting or focal vasoconstriction.
Blood tests should include a
neurological deficit.
comprehensive chemistry panel, • Hyperventilation should be
complete blood count, coagulation • When elevated ICP is suspected rapid closely monitored. The goal
profile, urinalysis. A chest X-ray should be treatment must be performed to PCO2 is between 30-35 mmHg.
ordered. decrease the risk of herniation and E x t re m e v a s o c o n s t r i c t i o n
secondary ischemia. secondary to PCO2 less than
Emergency Treatment 20 mmHg may cause brain
Options • Steps to prevent pending herniation:
ischemia. This should be a

• Head elevation to 30 degrees temporizing step as injured


Medications and Procedures
cerebral blood vessels lose
Intubation • Ensure midline head position to their responsiveness to
• Airway protection is the priority. Assess maximize venous drainage. hyperventilation-induced
the patient neurologic status briefly hypocarbia within 12 hours.
• Titrate sedation to reduce
before sedation/paralysis. Prefer short- Surgical management of the
agitation
acting sedative and paralytic agents so elevated ICP should be the
that frequent neurological examination • Osmotic Agents- Mannitol definitive measure.

404
Neurosurgical Consultation 2. SBP >180 mm Hg or MAP >130 mm Seizures
• ICH dictates immediate neurosurgical Hg and the possibility of elevated • Patients with ICH are at an increased
consultation. If hydrocephalus is ICP, consider monitoring ICP and risk of developing seizures. This risk
present, a ventricular drain will allow reducing BP while maintaining a i n c re a s e s w i t h l o b a r h e m a t o m a
both measurement of ICP and drainage cerebral perfusion pressure ≥60 mm location, hemorrhage size, depressed
of cerebrospinal fluid (CSF). Additional Hg. mental status, history of epilepsy,
hematoma evacuation or history of cirrhosis and penetrating
3. SBP >180 mm Hg or MAP >130 mm
decompressive craniotomy may be trauma.
Hg and no evidence of elevated ICP,
performed.
then consider reduction of BP (e.g., • The current guideline recommends
Hypertension a MAP of 110 mm Hg or target BP against the routine administration of
• Blood pressure should be closely of 160/90 mm Hg). antiepileptic medication. However, in
monitored. An invasive arterial catheter case of seizure, antiepileptics such as
• While the decrease in blood pressure
may be needed. The blood pressure fosphenytoin (20mg/kg loading dose)
may reduce the hematoma formation
targets in ICH is controversial and should be initiated.
and risk of re-bleeding, it may also
depends on the specific type of ICH.
reduce cerebral perfusion. One should Anti-coagulation
Recommended guidelines from the
avoid over-lowering blood pressure as • Coagulopathy should be reversed.
AHA/ASA are illustrated below.
s i g n i fic a n t d ro p s m a y m i n i m i z e Common anti-coagulants and their
• Recommended Guidelines from the perfusion to the ischemic penumbra. reversal agents are illustrated in Table.
AHA/ASA for Treating HTN in ICH Rapidly titratable antihypertensive
drugs are recommended. Hypotension
1. SBP >200 mm Hg or MAP >150 mm may be managed by crystalloid fluid,
Hg, consider aggressive reduction blood or vasopressors to maintain a
of BP with a continuous intravenous systolic blood pressure of >90 mmHg.
infusion

405
Table 9.5 Common anti-coagulants and their reversal agents Pediatric, Geriatric, Pregnant
Patient, and Other Considerations
DRUG
MECHANISM OF REVERSAL • Geriatric patients: The elderly are particularly at risk for
ACTION MEDICATION
spontaneous and traumatic ICH due to higher hypertension and
brain atrophy prevalence, and frequent use of anticoagulation.
Warfarin Inhibits Vitamin K Vitamin K
clotting factors FFP
(2,7,9,10) Prothrombin • Pregnant Patients: During pregnancy, ICH risk increases in case
Complex of preeclampsia, eclampsia and gestational hypertension.
Concentrates
Unfractionated Binds to antithrombin Protamine • Pediatric Patients: The majority of childhood ICH is secondary
Heparin 3
to trauma. The physician must always look for the signs of non-
Low Molecular Inhibits factor Xa Protamine accidental injury in pediatric patients with ICH even if the history
Weight Heparin incompletely reverses
factor Xa inhibition suggests otherwise.
Aspirin Irreversibly blocks Platelet transfusion to
cyclooygenase increase normal Disposition Decisions
platelet count by Patients are typically admitted to an ICU.
50,000
Clopidogrel Inhibits ADP receptor Platelet transfusion
on platelet
membrane References and Further Reading, click here
Dabigatran Direct thrombin No specific reversal
inhibitor
Rivaroxaban, Inhibit factor Xa No specific reversal
apixaban
provided by authors

406
Section 4

Seizure

Case Presentation
by Feriyde Caliskan Tur A 20-year-old female patient presented to the ED with
shoulder pain beginning in the morning at work. She had no
history of trauma. However, her right shoulder had a deformity.
Her X-ray showed a posterior shoulder dislocation, and she
could not explain how it occurred. While the emergency
physician was making preparations to reduce the shoulder, the
nurse shouted that the patient was having a generalized tonic-
clonic convulsion.

407
Introduction L/hour oxygenation was started via an air History and Physical
The seizure is a frequent neurologic mask, and the vascular access was Examination Hints
emergency in the emergency department established. Lorazepam 2mg, IV was History of seizures, head injury, recent
(ED), accounting for 1-2% of all given by slow injection to stop the fever (suggests infection or drug
emergency department visits. The highest seizure. reaction), anticholinergic and
incidence of seizure is among infants and sympathomimetic syndromes (mainly
The seizure stopped in a few minutes.
individuals aged > 75 years. The infantile depending on street drug-abuse) are
The patient’s blood glucose was
seizure occurs due to the high prevalence essential clues of the etiology of seizure.
measured at 122 mg/dL. Her vital signs
of febrile seizures, and in the elderly, it is
were: blood pressure 116/80 mmHg, A full neurological examination should be
mostly secondary to structural brain
heart rate 60 beats per minute, made. Motor movements and the
damage. Most seizures (49%) are related
respiratory rate 12 breaths per minute, accompanying eye movements during the
to alcohol or drugs, head injury, and pre-
oral temperature 98.6 °F, and a pulse seizure may distinguish the seizure from
existing diagnosis of epilepsy. Less
oximetry 100 % on room air. psychogenic seizures (pseudoseizures or
frequent etiologic pathologies are brain
nonepileptic seizures). 12% to 18% of
tumors (3%), metabolic abnormalities Her physical examination was normal
patients with transient loss of
(3%), stroke (3%), and neuro- except for the right shoulder. When the
c o n s c i o u s n e s s a re d e s c r i b e d a s
cysticercosis (1%). The reason for the patient regained consciousness, she
psychogenic seizures. It can exist
rest (41%) is unknown. Managing wanted to know what had happened. It
concomitantly in patients with neurogenic
patients with seizure and no apparent was her first witnessed seizure. There
seizures. Psychogenic seizures are a
etiology may be challenging for the was no drug or substance abuse, and her
manifestation of psychological distress.
emergency physician. menstrual status was normal. She had
frequent headaches for the last month, Neurologic deficits may be secondary to
Critical Bedside Actions and and she had been evaluated by a an old lesion, new intracranial pathology,
General Approach neurologist. or postictal neurologic compromise
The patient was placed immediately in
(called Todd paralysis, the physician may
the left lateral recovery position to avoid
rule out a new structural lesion).
aspiration of vomitus. Simultaneously, 4
408
Seizure is defined as a sudden change in A generalized seizure is related to both Emergency Tests and
behavior, characterized by an alteration in hemispheres accompanied by Interpretation
sensory perception or motor activity, convulsions in the entire body took place. Adult patients with new-onset seizures
resulting from an abnormal, excessive, who are otherwise healthy and have
Focal seizures took place in certain parts
and synchronous electrical firing in returned to baseline require only simple
of the body remain localized in a single
groups of neurons, caused by tests including serum glucose, sodium
hemisphere of the brain so may be easily
disequilibrium of the neuronal cell level, and pregnancy test. In patients with
overlooked. A simple focal seizure may or
membrane, normally kept stable by f e v e r, c o m o r b i d d i s e a s e , o r n e w
may not cause a depressed mental
inhibitory mediators such as gamma- neurological deficit further testing is
status, but a complex focal seizure
aminobutyric acid (GABA). indicated.
causes changes in consciousness.
Convulsions are the motor manifestations
Generalized status epilepticus is seizures Bedside test
of this abnormal electrical activity. The • Capillary glucose level (stick glucose)
that prevent the return to conscious state
clinical manifestations of seizures
with frequent recurrences or last more • Blood gases: It may show an anion gap
include  focal or generalized motor
than 20 minutes. According to these metabolic acidosis secondary to lactic
activity, altered mental status, sensory or
definitions, our case had a generalized acidosis. Lactate elevates within 60
psychic experiences, and autonomic
seizure. This definitions of seizure can seconds of a convulsive seizure and
disturbances.
change the patient management. normalizes within 1 hour.
Epilepsy is referred to convulsive seizures
Pregnant patients with seizure: Pregnant • Electrocardiography (ECG) should be
without any recurring or provocative
patients of more than 20 weeks’ gestation obtained in patients with new-onset
reason.
(and up to 6 weeks postpartum) with seizure, or with the suspicion of
The postictal period is a change in eclampsia is the major consideration in decreased CNS perfusion secondary to
consciousness that start before the presentation with new-onset seizures. a cardiac cause. In addition to
seizure and last after for a while. Gestational epilepsy is diagnosed in ischemia, conduction abnormalities and
approximately 25% of patients with new- dysrhythmias are important disorders to
onset seizures during pregnancy. be excluded, see below.
409
• Differential diagnosis by ECG • W o l ff- P a r k i n s o n - W h i t e Imaging modalities
syndrome Brain CT is indicated in all first-time
• Conduction Disorders That Can
s e i z u re s . A d d i t i o n a l l y, n e w f o c a l
Be Cause of Seizure-like Activity Laboratory tests neurological deficit, history of trauma,
• CBC (reveal anemia or infectious
(Adapted from: Seizure: Emergency Medicine, toxic drug and substance use
Second Edition. Editor; Adams, James G., MD, process)
necessitates a brain CT, see below.
2013, 2008 by Saunders, an imprint of Elsevier
Inc. Book chapter 99) • Electrolytes (especially Na, and Ca/Mg),
Indications for CT Scanning of the
• A seizure may also result in
• Serum glucose Brain
dysrhythmia-related
syncope. • Urea-nitrogen, creatinine, Adapted from: Teran F, Harper-Kirksey K,
Jagoda A. Clinical decision making in seizures
and status epilepticus. Emerg Med Pract. 2015
• Brugada syndrome: Right • Pregnancy tests in women of Jan;17(1):1-24.
bundle branch block with ST- childbearing age (rule out eclamptic
• A Persistent change in mental status
segment elevation in leads seizures),
V1-V3 • Advanced age
• Antiepileptic drug levels,
• Long QTc interval • History/clinical evidence of trauma
• Liver function tests, and
• Short QTc interval • H u m a n i m m u n o d e fic i e n c y v i r u s /
• Drugs-of-abuse screening acquired
• Sodium channel blockade
with cyclic antidepressants, • Spinal tap is useful to evaluate • Immunodeficiency syndrome (HIV/AIDS)
lidocaine, anticholinergics suspected CNS infection (patients with
fever, severe headache, or persistent • Infection (neurocysticercosis)
• Torsades de pointes altered mental status) or HIV/AIDS
• New focal neurological deficit
p o p u l a t i o n ( s t ro n g s u s p i c i o n o f
• Widening of QRS complex
immunodeficiency). • Suspicion of parasitic central nervous
system

410
First-onset seizures or seizures with • The absence of a history of alcohol Emergency Treatment
persistent mental status change, focal abuse Options
neurologic deficit, or suspicion of organic Maintenance of adequate cerebral
• The focal onset of the seizure
intracerebral lesion necessitates brain perfusion, oxygen and glucose supply to
computed tomography (CT). It will help to MRI can reveal additional diagnosis like the brain, is the goal of initial treatment.
diagnose epidural or subdural brain abscess and central vascular Airway must be preserved. Continuous
hemorrhage, a brain mass or infections. events.  MRI is more sensitive than CT pulse oximetry and capnography should
and can successfully diagnose temporal be monitored. Jaw thrust and
Factors Associated With Abnormal
sclerosis, cortical dysplasia, vascular nasopharyngeal airway ensure an
Computed Tomography Findings
malformations (e.g., AV aneurysms), and improved oxygenation. Preventing
(Adapted from: Teran F, Harper-Kirksey K, some tumors in addition. Its use is limited aspiration in the postictal phase, seizure
Jagoda A. Clinical decision making in seizures
and status epilepticus. Emerg Med Pract. 2015,
in emergency setting. control (administration of
page 9) anticonvulsants), correction of
E l e c t ro e n c e p h a l o g r a p h y ( E E G ) i s
• Altered mental status hypoglycemia, IV line placement, and
important to monitor intubated patients or
administration of oxygen can be
• Closed head injury patients with persistent altered mental
addressed together with coordinated
status. (Suspicion of nonconvulsive
• Focal abnormality on neurological team care. If there is trauma signs
status epilepticus).  EEG records brain
examination secondary to the seizure, cervical spine
electrical activity and is used for definitive
precautions (immobilization with a collar)
• History of cysticercosis diagnosis. The need for EEG in the
should be initiated.
emergent setting is limited and must be
• Malignancy saved for when seizure activity is Medications
uncontrollable or difficult to diagnose. Traditionally, pharmacologic therapy of
• Neurocutaneous disorder
(e.g., patients who are under sedation or seizure has been divided into three steps
• Patient aged > 65 y are intubated). (Table). Generally, benzodiazepines are
the initial choice, followed by phenytoin
• Seizure duration > 15 min
or valproic acid. Levetiracetam is the
411
second-step choice in patients with liver • If the patient needs intubation, and aggressive cooling. Phenytoin is
disease. Third step interventions are pretreating with lidocaine (1.5 mg/kg) not effective in substance-related
infusions of benzodiazepines (midazolam and a low dose (defasciculating dose) seizure, and also may be harmful to
or long-acting lorazepam), propofol, or of a nondepolarizing paralytic agent drug intoxications such as tricyclic
barbiturates (e.g., pentobarbital; pay (e.g., vecuronium, 0.01 mg/kg) is antidepressants and antiarrhythmics.
attention to hypotension). In up to 30% of preferable to control intracranial
• The clinician should be aware that
the patients, the first- and second-step pressure from trauma or intracranial
administration of phenytoin and
therapies fail. bleeding. Short-acting paralytic agents
phenobarbital is rate-dependent and
such as succinylcholine is
• The drug choice is same for that patients may continue to seize for
recommended during rapid sequence
nonconvulsive seizures. 30 minutes before effective serum
intubation.
levels are reached.
• Secondary causes for seizure must be
• Remember that phenytoin (effective
considered for the treatable etiologies • Timely administration of antibiotics is
dose 20 mg/kg) must be infused with
(e.g., intracranial infections and lesions, important for the survival of patients
saline solution (not dextrose due to
metabolic abnormalities, drug toxicities, with infectious problems
precipitation). Its main adverse effect is
and eclampsia).
arrhythmia due to QT prolongation. • Prophylactic medication is not indicated
Many considerations on the use of T h e re f o re , t h e p a t i e n t m u s t b e to prevent late posttraumatic seizures.
medications should be remembered; monitored during the infusion.
• New generation drugs such as
• Use of benzodiazepine for active • Alcoholic seizure and seizure secondary lamotrigine (FDA category C) is used for
seizure in the prehospital setting is to isoniazid in tuberculosis treatment partial, generalized, and absence
strongly supported. are treated with 5 gr IV vitamin B6 in seizures for maintenance therapy.
adults and 70 mg/kg IV pyridoxine
• IM midazolam is the best option for the infusion in children.
prehospital treatment of seizure,
especially when no intravenous access • Seizures due to ecstasy or cocaine
is immediately available. abuse are treated with benzodiazepines
412
Table 9.6 Antiepileptic drugs and doses for seizures therapies
MEDICATION LOADING DOSE IV MAINTENANCE DOSE PEDIATRIC DOSE COMMENTS
Diazepam 10 mg over 2 min, or 10-20 mg Repeat q 5-10 min 0.15 mg/kg IV; 0.2-0.5 mg/kg respiratory depression,
hypotension

Lorazepam 2-4 mg IV Repeat once in 10-15 min <13 kg: 0.1 mg/kg IV (max 2 mg); respiratory depression,
13-39 kg: 2 mg IV; hypotension
>39 kg: 4 mg IV

Midazolam 0.1-0.2 mg/kg (also IM, IN 0.001 mg/kg/min 0.2 mg/kg IV, IN (max 5 mg); 0.5 mg/kg respiratory depression,
rectal or buccal) buccal (max 5 mg); <13 kg: 0.2 mg/kg IM hypotension
(max 5 mg); 13-39 kg: 5 mg IM; >39 kg: 10
mg IM

Phenytoin 18-20 mg/kg, max rate of 50 100 mg IV/PO q 6-8 h, 20 mg/ 20-mg/kg IV infusion hypotension, ataxia
mg/min kg IV infusion

Pentobarbital 5-20 mg/kg, 25 mg/min 1-3 mg/kg/ respiratory depression

Phenobarbital 10-15 mg/kg bolus, 60 mg/min 120-240 mg q 20 min respiratory depression,


hypotension

Propofol 1-2 mg/kg IV over 5 min 2-4 mg/kg/h respiratory depression,


acidosis (in children)

Valproic acid 20 mg/kg at 20 mg/min Repeat if needed subtherapeutic dosages

Magnesium sulfate 4-6 g over 15 min 2 g/h respiratory depression, loss of


deep tendon reflexes

Calcium gluconate, 10 mL of 10% calcium Only indicated for hypocalcemia


calcium chloride (has gluconate in 50-100 mL or hyperpotassemia
three times more Ca+2) of D5W, over 5-10 min

3% NaCl (hypertonic 300-500 mL of 3% NaCl in 20 Repeat if needed Only for hyponatremia


saline solution) min

Pyridoxine 5 g (50 ampoules of 100 mg of Only for some drug-induced


vitamin B6) seizure

413
Pregnant patients with seizure Disposition Decisions
Seizure related hypoxia and acidosis have a greater teratogenicity Admission criteria
potential than anticonvulsant medications. Therefore, actively- Patients with persistent seizures, change of mental or neurologic
seizing pregnant patients may be managed the same as status, or underlying medical conditions that require hospital
nonpregnant. Magnesium sulfate is the therapy of choice in the treatment (e.g., sepsis, overdose, brain trauma) should be
treatment of acute eclamptic seizures and for prevention of admitted. Patients with status epilepticus should be admitted to
recurrent eclamptic seizures. Additionally, a seizing chronic the intensive care unit. Patients with subtherapeutic drug levels
epileptic pregnant can be treated with midazolam. Remember should receive an additional dose before discharge. First-onset
that midazolam (FDA category D) is the safest; valproate and seizures should have follow-up arranged with the neurology
phenytoin are the most harmful (both FDA category D) service/consultant for further investigations. A second attack
antiepileptic drugs in first-trimester pregnancy. occurred in 1 month in 32% of patients with a first-onset seizure.
Risk factors such as alcoholism, comorbidities or known
Emergency Procedures cardiovascular disease, age > 60 years, history of cancer, or
Airway management is the most important procedures during a
history of immune-compromise should be considered for
seizure activity. The majority of the patients, however, will not
admission.
need definitive airway protection. The basic airway maneuvers or
adjunct devices may overcome the temporary airway obstrcution Discharge criteria
risks. Simple seizures are self-limiting in most cases. Maintaining Discharge decision should be based on final underlying
the airway by jaw trust/chin lift maneuvers, inserting an diagnosis. Chronic seizures can be discharged after return to the
oropharyngeal airway, and oxygen mask ventilation are first baseline neurologic levels.
measures to prevent the tongue bite, airway obstruction, and
apnea. A corkscrew is useful to open the jaw. In cases that
oxygen inhalation and intubation fail, a surgical airway is References and Further Reading, click here
indicated.

414
Chapter 10

Selected
Pulmonary
Emergencies
Section 1

Asthma

Case Presentation
by Ayse Ece Akceylan A 50-year-old male with a history of asthma presents to the
emergency department (ED) with shortness of breath,
tachypnea, and audible wheezing. The patient has taken his
prescribed medications at home, but his symptoms did not
relieve. His vitals were as follows: BP 130/90 mmHg, HR 120
bpm, RR 40 bpm, SpO2 92% on room air. Physical exam
revealed accessory respiratory muscle use, expiratory
wheezing and decreased breath sounds with expiratory
rhonchi. Nebulized short-acting beta2-agonists (SABA) and
systemic corticosteroid were ordered. Peak expiratory flow
(PEF) measurements before and after treatment were 125 and
360, respectively. Auscultation after initial treatment revealed
much-improved airflow. The patient was discharged following
clinical improvement, with a prescription of oral
corticosteroids in addition to his current medications.

416
Introduction obstruction and airflow limitation. The spectrum are patients with a “silent
Asthma is a chronic inflammatory repetitive airway inflammation leads to chest,” which reflects very severe airflow
disorder of the airways characterized by persistent structural changes in airways, obstruction and air movement insufficient
recurrent episodes of variable expiratory called airway remodeling. This results in to promote a wheeze.
airflow limitation. Asthmatic patients have increased airway resistance and a
The exacerbation begins with coughing
hyperresponsive airways that constrict decrease in forced expiratory volumes
and a sensation of chest constriction. As
when exposed to various stimuli. and flow rates. As a result, the lungs
the attack advances, expiration is
Symptoms and airflow limitation are often become hyperinflated. Ventilation-
prolonged, wheezing becomes
reversible, either spontaneously or with perfusion mismatch develops despite
prominent, and accessory respiratory
treatment. However, reversibility may be increased work of breathing. The
muscles are used. To decrease the effort
incomplete in some patients. Although interaction of these features determines
of breathing, the patient may sit upright
patients appear to recover completely the clinical manifestations, the severity of
or lean forward. The appearance of
clinically, some asthmatic patients may asthma and the response to therapy.
p a r a d o x i c a l r e s p i r a t i o n s r e fle c t s
have chronic airflow limitation. The Watch this video.
impending ventilatory failure. Alteration in
diagnosis of asthma should be based on
Presenting Signs and mental status heralds respiratory arrest.
the history of characteristic symptom
patterns and evidence of variable airflow Symptoms
The classic symptoms include the triad of
Critical Bedside Actions
limitation documented by bronchodilator 1. Ensure adequate oxygenation
dyspnea, wheezing and coughing.
reversibility testing or other tests.
Physical findings during an asthma 2. G i v e S A B A t o r e v e r s e a i r flo w
Pathophysiology exacerbation can be variable. A patient obstruction
The mediators released in response to with a mild exacerbation may merely be
coughing and complaining of chest 3. Give systemic corticosteroids to relieve
allergens and nonallergic stimuli cause
tightness, whereas a patient with a severe inflammation
inflammation, edema, mucus production,
and airway smooth muscle hypertrophy. exacerbation will be in respiratory 4. Carry out serial assessments to
All of these lead to bronchoconstriction distress, with tachypnea and loud monitor the response to therapy
and hyperreactivity, aggravating airway wheezing. At the other end of the
417
General Approach intravenous magnesium sulfate if the History/Physical
• O b t a i n p a t i e n t h i s t o r y, a s s e s s patient is unresponsive to intensive Examination Hints
exacerbation severity and initiate initial treatment.
treatment simultaneously. Respiratory History
• Do not routinely perform chest x-ray or • P o s s i b l e c a u s e s o f t h e c u r re n t
rate, dyspnea, pulse rate, oxygen
blood gases, or prescribe antibiotics. exacerbation
saturation and lung function reflect
exacerbation severity. • Prescribe ongoing controller treatment • The severity of symptoms compared
before discharge to reduce the risk of with previous exacerbations
• Keep alternative causes of
future exacerbations. Provide follow up
breathlessness in mind. • Other comorbidities
for all patients, preferably within a
• Start treatment with repeated doses of week.
• Current asthma medications and
inhaled SABA by a puffer, spacer, or adherence to therapy
nebulizer. Give early oral corticosteroids Differential Diagnosis
Wheezing, coughing, and dyspnea may • Any use of potential asthma triggering
and controlled flow oxygen in life-
be caused by many other conditions, medication
threatening exacerbations or if Forced
including pneumonia, bronchitis, croup,
Expiratory Volume in 1 Second (FEV1) is
b ro n c h i o l i t i s , c h ro n i c o b s t r u c t i v e • Aspirin
less than 30% of the predicted. Target
pulmonary disease, congestive heart
oxygen saturation is 93-95% in adults • Beta-blockers
failure, valvular heart diseases, pulmonary
and adolescents, and 94-98% in
embolism, allergic reactions, • Angiotensin-converting enzyme
children between 6-11 years.
g a s t ro e s o p h a g e a l re flu x d i s e a s e , inhibitors
• Monitor symptoms and oxygen exposure to odors, dust, and gas, and
• Any risk factors for asthma-related
saturation frequently or continuously. upper airway obstruction from vocal cord
death
Measure lung function after one hour.3 dysfunction, edema, neoplasm or a
foreign body. Any of these alternative • poor adherence to asthma
• Add ipratropium bromide to treatment
diagnoses may also be found together medications
in severe exacerbations. Consider
with asthma.
418
• psychosocial problems • Anaphylaxis Respiratory rate is increased, and the
pulse rate is around 100-120 bpm.
• history of near-fatal asthma requiring • Pneumonia
Oxygen saturation on room air is 90-95%,
intubation and mechanical ventilation
• Pneumothorax and PEF > 50% predicted or best.
• hospital visit for asthma in the past
• Look for signs of alternative conditions Patients with a severe attack can only talk
year
that could explain acute dyspnea in words, sit hunched forwards, use
• currently using or having recently accessory muscles and show agitation.
• cardiac failure Respiratory rate is >30/min. Pulse rate
stopped using oral corticosteroids
exceeds 120 bpm. Tachypnea and
• pulmonary embolism
• not currently using inhaled tachycardia are associated with severe
corticosteroids • PEF in patients older than 5 years obstruction, but a lower rate does not
r u l e o u t s e v e re a s t h m a . O x y g e n
• over-use of SABAs The most common finding on physical
saturation on air is <90% and PEF≤50%
examination is expiratory wheezing. With
• food allergies predicted or best.
severe airway obstruction, it decreases or
Physical Examination vanishes because air movement velocity Patients with the life-threatening attack
• Check vital signs is insufficient to produce sound. Crackles are drowsy or confused and have silent
and inspiratory wheezing are not features chest.
• Look for signs of exacerbation severity
of asthma. They are more likely to be

• use of accessory muscles seen in pneumonia. Inspiratory and Emergency Diagnostic Tests
expiratoy crackles, however, are seen in And Interpretation
• mentation pulmonary edema. • Pulmonary Function Tests: Bedside
spirometry is used both for initial
• sitting position Patients with the mild or moderate attack assessment and for evaluating the
can talk in full sentences or phrases. response to therapy. If possible, and
• level of consciousness
Patients may prefer sitting to lying, but without delaying treatment, record PEF
• Look for complicating factors accessory muscles are not used. or FEV1 before treatment. Patient
419
cooperation is essential for these tests >45 mmHg) indicate respiratory failure. of 93-95% (94-98% for children 6-11
to be reliable. Monitor lung function at Fatigue and somnolence suggest that years).
intervals until a clear response to pCO2 may be increasing and airway
• Inhaled short-acting beta2-agonists
treatment has occurred, or a plateau is intervention may be needed. Do not
(SABAs): Use 4-10 puffs pMDI with a
reached. When possible, management wait for arterial blood gas confirmation
spacer in mild or moderate attacks. For
decisions should be guided by the treat ventilatory or respiratory failure.
severe attacks, administer 1 nebule
patient’s personal best PEF or FEV1
• Other Blood Testing: Laboratory studies every 20 minutes for 1 hour.
value or, if unknown, predicted values.
are rarely helpful in an acute asthma
• Epinephrine (for anaphylaxis): Indicated
• Pulse oximetry: is a noninvasive, attack.
only if acute asthma is associated with
convenient and continuous method for
• Radiology Studies: Radiography is only anaphylaxis and angioedema.
monitoring oxygen saturation before
indicated if there is the possibility of
and during treatment. • Systemic corticosteroids: enhance the
pneumothorax, pneumomediastinum,
resolution of exacerbations and prevent
• Arterial Blood Gas (ABG) analysis: is pneumonia, or other medical
recurrence. They should be utilized
helpful if there is a concern for conditions.
within 1 hour of presentation. Oral and
hypoventilation with carbon dioxide
• Electrocardiogram and Cardiac intravenous administrations are equally
retention and respiratory acidosis. It is
Monitoring: A routine electrocardiogram effective. However, the oral route is
not indicated in the majority of patients
is unnecessary. Older patients and preferred because it is less invasive and
with mild to moderate asthma
patients with coexistent heart disease less expensive. Intravenous
exacerbation. Consider ABG analysis
or with severe exacerbation should corticosteroids can be administered
for patients with a PEF or FEV1 <50%
undergo continuous cardiac monitoring when patients are too dyspneic to
of the predicted, or for those who do
to detect dysrhythmias. swallow if the patient is vomiting, or
not respond to initial treatment or
when patients require non-invasive
continue deteriorating despite
Treatment Options ventilation or intubation. Corticosteroid
treatment. A PaO2 <60 mmHg and
• Oxygen: Administer by nasal cannula or dose is 1 mg/kg (max. 50 mg)
normal or increased paCO2 (especially mask. Target arterial oxygen saturation

420
prednisolone for adults and 1-2 mg/kg NIMV is still controversial in asthma. It comorbidities, concurrent treatments,
(max. 40 mg) for children. improves work of breathing, gas medication side effects (cardiotoxicity
exchange. However, increase the risk of with beta2-agonists; skin bruising,
• Inhaled corticosteroids: are well
barotrauma. osteoporosis, cataracts with
tolerated. However, the cost is a limiting
corticosteroids) and lack of self-
factor. The effectiveness, dose, and This video demonstrates treatment of
management skills should be taken into
duration of treatment in the ED remain asthma. However, in the ER, the actions
account while managing asthma in the
unclear. should be a lot faster.
elderly.
• Ipratropium bromide: Use for moderate- Special Populations Children
severe exacerbations, along with SABA.
Management of asthma exacerbation for
Pregnancy
• Magnesium: Intravenous magnesium The advantages of actively treating adults and children >5 years are mostly
sulfate is not recommended for routine asthma in pregnancy markedly outweigh similar. This section points to the
use in asthma exacerbations. It reduces any potential risks of the usual controller management of asthma exacerbations in
hospital admissions in patients who fail and reliever medications. To avoid fetal children 5 years and younger.
to respond to initial treatment and have hypoxia, acute asthma exacerbations
The presence of any one of these
persistent hypoxemia. during pregnancy should be aggressively
features means a severe exacerbation:
t re a t e d w i t h S A B A , o x y g e n , a n d
• Helium-oxygen therapy: May be
administration of systemic • altered consciousness
considered for patients who do not
corticosteroids.
respond to standard therapy. • oxygen saturation from pulse oximetry
Elderly of <92% on presentation
• Non-Invasive Mechanical Ventilation
The elderly may not describe asthma
(NIMV) and Intubation: If the patient • central cyanosis
symptoms or may associate
begins to exhibit signs of acute
b re a t h l e s s n e s s w i t h t h e i r a g e o r • silent chest
respiratory failure including progressive
comorbidities (cardiovascular disease,
hypercapnia and acidosis, intubation
o b e s i t y, e t c . ) . T h e i m p a c t o f
and mechanical ventilation is indicated.
421
• i m p a i re d m e n t a t i o n ( t h e n o r m a l intravenous methylprednisolone 1 mg/kg decision should be made according
developmental capability of the child 6-hourly. to the patient’s risk factors and the
must be taken into account) availability of follow-up care.
Magnesium sulfate: If the child is not
• pulse rate (>200 beats/min for children responding to standard therapy, consider Risk factors associated with the need for
0-3 years, >180 beats/min for children nebulized isotonic magnesium sulfate admission:
4-5 years). (150mg) 3 doses in the first hour of
• Female sex, older age, and non-white
treatment OR intravenous magnesium
Oxygen: target oxygen saturation is race
sulfate (in a single dose of 40-50- mg/kg
94-98%.
(max 2g) by slow infusion (20-60 min) for • Use of more than 8 beta2-agonist puffs
Bronchodilator therapy: Give 2-6 puffs of children aged ≥2 years with severe in the previous 24 hours
salbutamol by a spacer, or 2.5mg exacerbation.
• Severity of the exacerbation (e.g.need
salbutamol by nebulizer, every 20 min for
the first hour, then reassess severity. If
Disposition Decisions for resuscitation or rapid medical
If pre-treatment FEV1 or PEF is <25% of intervention on arrival, respiratory rate
symptoms persist or recur, give an
the predicted or personal best, or post- >22 breaths/minute, oxygen saturation
additional 2-3 puffs per hour. For children
treatment FEV1 or PEF is <40% of the <95%, final PEF <50% predicted).
with moderate-severe exacerbations and
predicted or personal best,
a poor response to initial SABA, • History of severe exacerbations
hospitalization is recommended.
ipratropium bromide may be added, as 2 requiring admission to hospital
puffs of 80 mcg (or 250mcg by nebulizer) If post-treatment lung function is >60% of
every 20 minutes for 1 hour only. • Previous healthcare facility visits
the predicted or personal best, discharge
requiring the use of oral corticosteroids.
is recommended after considering risk
Systemic corticosteroids: Systemic
factors and availability of follow-up care. An asthma exacerbation does not resolve
corticosteroids: Give initial dose of oral
prednisolone (1-2 mg/kg up to a c o m p l e t e l y o n d i s c h a rg e ; a i r w a y
Patients with post-treatment lung
maximum 20 mg for children <2 years inflammation and peripheral obstruction
function 40-60% of the predicted are the
old; 30 mg for children 2-5 years, OR, may take hours to days to dissipate.
gray zone. Hospitalization or discharge

422
• Prescribe at least a 5-7 day course of oral corticosteroids
(prednisolone or equivalent 1 mg/kg/day to a maximum of 50
mg/day), along with inhaled corticosteroids and reliever
medication.

• Implement strategies to reduce modifiable risk factors (irritant or


allergen exposure, incorrect inhaler skills, inadequate long-term
treatment, problems with adherence, or lack of a written asthma
action plan).

• Arrange follow-up appointment within one week

References and Further Reading, click here

423
Section 2

Chronic Obstructive Pulmonary Disease (COPD)

Case Presentation
by Ramin Tabatabai, David Hoffman, and Tiffany A 68-year-old male presents to the emergency department
Abramson
(ED) with audible wheezing, and he is in severe respiratory
distress. He is speaking in 2-3 word sentences, and he is
diaphoretic and slightly confused. Per the paramedic report,
the patient is a two pack per day smoker. On physical
examination, the patient demonstrates poor air movement,
and you note that he has a “barrel chest.” As you pick up the
phone to call the respiratory therapist for airway management,
you wonder, “What other interventions should I initiate and are
there other diagnoses I should be considering?”

Audio is available here

424
General Approach and treatment in the majority of the patients. limitation. COPD is, therefore, a

Critical Bedside Actions BiPAP therapy for moderate to severe chronic, progressive disease, usually with
Although COPD patients may frequently exacerbations should be kept in mind. an indolent course of gradual decline in
visit ED, some of these presentations Antibiotic therapy should be started for airflow and physical activity level
may require critical interventions such as any acute exacerbation requiring secondary to dyspnea.
intubation. Therefore, the ABC sequence admission or discharged patients with
The etiologies of acute exacerbation can
should be followed in all these cases to increased sputum purulence.
be classified into four different groups
understand an immediate life-threatening
Differential Diagnoses (infectious, pollution, destabilizers,
situation.
During the initial evaluation and ongoing idiopathic). Although approximately 70%
The most patients require Oxygen bedside treatments, emergency physician of exacerbations are due to infection
therapy to keep pulse oximetry 88-92%. lists causes of this attack in his/her mind. (Viral or Bacterial), it is important to
Establishment of intravenous (IV) line and Two major challenges exist in evaluating consider other potential triggers or
fluid replacement may be necessary for the patient with suspected COPD. First, etiologies such as Pneumothorax,
severe attacks. Cardiac monitor and the differential diagnosis for dyspnea is Pulmonary Embolism (PE), Congestive
electrocardiogram (ECG) to assess broad and distinguishing COPD from H e a r t F a i l u re ( C H F ) , P n e u m o n i a ,
cardiac ischemia or arrhythmia is alternative causes can be difficult. Pericardial Effusion, Lobar Atelectasis,
mandatory for every case. While these Second, patients with COPD may harbor Anaphylaxis, Airway Obstruction, and
activities are going on simultaneously, the concomitant cardiopulmonary disease. Trauma.
emergency physicians’ primary role is to
COPD should be considered in anyone Acute exacerbation of COPD is often
rule out other life-threatening causes of
with risk factors and dyspnea, chronic confused with pulmonary edema
dyspnea. Inhaled beta-agonist
cough or sputum production. Major risk secondary to CHF. Cardiac “wheeze” is
bronchodilator (e.g., Albuterol), Inhaled
factors include smoking and easily mistaken for the wheeze classically
anticholinergic bronchodilator (e.g.,
environmental exposures. Pathological heard in acute COPD exacerbation.
Ipratropium), and oral glucocorticoid
changes that occur in the lung causes air Further complicating matters, these
therapy (IV steroids only if unable to
t r a p p i n g a n d p ro g re s s i v e a i r flo w diagnoses are not mutually exclusive and
tolerate PO) are the mainstay of the
425
can often present together in a mixed picture. A thorough speak in full sentences, c o n f u s i o n , a g i t a t i o n , u s e o f
evaluation of clinical evidence of CHF is therefore critical in the accessory respiratory muscles, paradoxical chest wall
evaluation of the wheezing acute COPD exacerbation patient. movements, worsening or new onset central cyanosis,
development of peripheral edema, or hemodynamic instability.
Additional diagnoses should be considered when an acute COPD
exacerbation is more severe than previous or if the patient A thorough examination will involve cardiopulmonary evaluation
deteriorates rapidly. One such disease is PE, which can occur in to assess for the presence of wheeze and auscultation to
COPD patients due to sedentary lifestyles, increased venous estimate the degree of tidal volume that occurs with each
stasis, and increased blood viscosity. Pneumothorax is another ventilation. Markedly decreased air movement indicates severe
critical consideration as COPD patients. As a traditional disease. Other findings in chronic COPD may include a thin,
knowledge, COPD patients have increased risk for ruptured barrel-chested appearance or plethoric, cyanotic appearance.
bullae. Other lethal causes of exacerbation and dyspnea are not
One important sequela of COPD is cor pulmonale. Long-standing
limited to but include pneumonia and lobar atelectasis secondary
increased pulmonary pressures can lead to right-heart strain and
to bronchial plugging.
eventual right heart failure. Patients can therefore present with
History and Physical Examination Hints acute COPD exacerbation along with CHF findings of jugular
In the ED, providers are predominately concerned with acute venous distention and peripheral edema.
COPD exacerbation. An exacerbation is defined as an acute
Finally, a thorough history should be obtained by evaluating risk
event that leads to a worsening of the patient’s respiratory
factors, previous exacerbations, the frequency of exacerbations,
symptoms, beyond normal day-to-day variation and leads to a
and prior intubations. While there are many predictors of a COPD
change in medication.
exacerbation, the best is a history of prior exacerbations.
The physician’s first action in the evaluation of a dyspneic patient
with suspected acute COPD exacerbation is airway, breathing
and circulation, and assessment of vital sign abnormalities. These
are used to determine whether the patient will require immediate
intervention. Any of the following signs on initial visual inspection
indicate severe acute COPD exacerbation: “tripoding,” inability to
426
Emergency Diagnostic Tests diagnosis of COPD in the outpatient
Image 10.1
and Interpretation setting, there is no role for its use in the
Every patient in respiratory distress emergency room.
should be placed on continuous pulse-
Laboratory Tests
oximetry and cardiac monitoring.
Blood tests have little utility because their

Bedside Tests results do not change treatment or


An electrocardiogram is useful in disposition. The argument can be made
identifying classic patterns (cor for the measurement of brain natriuretic
pulmonale and dysrhythmias) associated peptide (BNP), which reflects the
with COPD as well as evaluating for stretching of myocardial tissue and can,
ischemia. therefore, indicate decompensated heart
failure.
The chest radiograph (CXR) should be
ordered to evaluate the presence of other Emergency Treatment
treatable diagnoses. CXR findings in Ultrasound should be used when Options
COPD may include a small cardiac available for the rapid information it
silhouette, hyperinflated lung fields, or
Bedside Critical Actions and
provides. A cardiac and lung ultrasound
bullae. Alternative findings may consist of
Stabilization
can help both establish and rule out The emergency physician must first
a n e n l a r g e d h e a r t , e ffu s i o n o r diagnoses by evaluating for pericardial decide whether the patient requires
parenchymal consolidation. effusion, cardiac squeeze, B-lines, and respiratory assistance. Oxygenation and
lung sliding. v e n t i l a t o r y s u p p o r t a re m a i n s t a y
What is your diagnosis in the given Chest
X-ray of a dyspneic patient? The US video shows A (normal) and B- therapies. Patients with hypoxemia need

lines in the lung. supplemental oxygen with a targeted


oxygen saturation goal of 88-92%. This
It is important to note that while target has been set because high flow
spirometry is essential to the formal oxygen has been associated with carbon
427
dioxide retention, hypercapnia, treatment with beta-agonist Antibiotic administration is
respiratory acidosis and respiratory bronchodilators (e.g., albuterol) and controversial. To simplify this debate,
failure. Oxygen delivery is dependent on anticholinergics (e.g., ipratropium) is antibiotics for patients with both
the severity of the patient’s respiratory generally safe and is considered 1st line i n c re a s e d s p u t u m p u r u l e n c e a n d
status and mentation, ranging from the therapy. Efficacy between MDI and dyspnea or those requiring hospitalization
nasal cannula to mechanical ventilation. nebulizer is equivalent, however are recommended. The antibiotic choice
Noninvasive positive pressure ventilation moderate to severe patients may be should be selected based on the
via BiPAP is an effective treatment for unable to use MDI. Nebulized dosing for suspected pathogen and whether the
patients in moderate to severe respiratory albuterol should be 2.5 to 5mg and 0.5mg patient has risk factors for Pseudomonas.
distress. It decreases treatment failure, for ipratropium. Combination of both First line outpatient antibiotic choices
reduces complications, shortens medications is synergistic and relatively include doxycycline, a beta-lactam, and
hospitalizations, and improves mortality safe; however, caution is advised in sulfamethoxazole-trimethoprim (for 5-7
rate. Specifically, a Cochrane review in patients with cardiac disease. days) and for hospitalized patients
2004, demonstrated the use of BiPAP led c o n t a i n s flu o r o q u i n o l o n e s ( e . g . ,
Corticosteroids are the other first-line
to decreased mortality (number need to levofloxacin) or a beta-lactam with
treatment in acute COPD exacerbation.
treat, NNT=10), reduction in treatment pseudomonas coverage (e.g., cefepime).
Studies have demonstrated shorter
failure (NNT=5) and decreased need for
recovery time, length of stay in the Pediatric, Geriatric,
intubation (NNT=4). Patients with BiPAP
hospital and an NNT=10 to avoid Pregnant Patient and Other
failure, however, require intubation and
treatment failure. Oral and IV
mechanical ventilation. Once intubated, Considerations
glucocorticoids have similar efficacy, with COPD is a chronic disease with a peak
ventilator management and strategy
a recommended dose of prednisone 40 incidence in the 5th to 6th decade of life.
should focus on a prolonged Inspiration
mg PO and methylprednisolone 1-2mg/ It has been linked to many comorbid
and Expiration ratio and low respiratory
kg IV. A 5-day course is sufficient and conditions making the geriatric
rate with small tidal volume.
preferable to the side effects caused by population high–risk. Additionally, studies
For patients with a high suspicion for longer regimens. have demonstrated that morbidity in
acute COPD exacerbation, empiric COPD increases with age. Providers,
428
should, therefore, demonstrate greater References and Further Reading,
Image 10.2 COPD Admission and
caution with geriatric patients and have a Discharge Criteria (GOLD) click here
lower threshold to admit.
Admission Criteria

COPD is not a well-defined disease in the Ward

pediatric population and is extremely rare Failure to respond to initial medical management

in pregnant patients. Even children with A marked increase in the intensity of symptoms

α1-antitrypsin deficiency do not develop Severe underlying COPD, severe comorbidities or


old age
symptoms until 20-50 years of age.
Insufficient home resources
Interestingly, however, the diagnosis of
ICU
“asthma” in childhood, has been
Persistent or worsening hypoxia despite
associated with a 10-fold risk for COPD supplemental oxygen
in the future and there is mounting Need for invasive mechanical ventilation
evidence that genetics has a
Change in mental status
c o n s i d e r a b l e ro l e t o p l a y i n t h e
Hemodynamic Instability
development of the disease.
Discharge Criteria

Disposition Decisions Clinically stable


Criteria for hospitalization include an
The frequency of medication use <4hrs
exacerbation failing to improve back to
baseline. The GOLD collaborators have Dyspnea does not negatively effect sleeping,
eating or basic functions
outlined the criteria for admission and
discharge and further delineated criteria Able to complete ADLs and understands
medications
for ICU admission versus ward (Table).
Adapted from the Global Strategy for the
Criteria for ICU admission revolve around Diagnosis, Management, and Prevention of
respiratory failure and altered mental COPD, Global Initiative for Chronic Obstructive
Lung Disease (GOLD) 2015. Available from:
status. http://www. goldcopd.org/.

429
Section 3

Pneumonia

Case Presentation
by Mary J. O. A 74-year-old male with a history of hypertension and
diabetes presented to the emergency department with a
cough productive of rust-colored sputum. His complaints
started approximately three days earlier and progressively
worsened. The patient reported difficulty in breathing, shaking
chills, and fever up to 39ºC. He had no sick contacts. On
examination, the physician noticed an elderly gentleman in
mild respiratory distress. His vital signs were: BP: 110/70
mmHg, HR: 102 bpm, RR 20 bpm, T 38.4ºC and SpO2 91%
on room air. Auscultation revealed rales at the right lung base.

430
Introduction Classification v i r u s ( R S V ) , p a r a i n flu e n z a ,
Pneumonia is an acute respiratory Community-acquired pneumonia (CAP) coronaviruses, adenoviruses, and
infection of the lung parenchyma, occurs in patients with no recent rhinoviruses. Recently, a number of new
particularly the alveoli. The healthy alveoli hospitalization or exposure to the viral pathogens have emerged, including
fill with air when a person breathes; healthcare system. The most common coronaviruses that causes severe acute
however, in pneumonia, the fluid or pus in bacterial cause of CAP is Streptococcus respiratory syndrome (SARS) and the
the alveoli makes breathing painful and pneumoniae. Its incidence is declining Middle East respiratory syndrome
inhibits air exchange. Despite modern due to vaccination. Other common (MERS-CoV).
research and the development of a bacteria are Haemophilus influenzae,
Hospital-acquired pneumonia (HAP)
variety of antimicrobial agents, Moraxella catarrhalis, and
refers to pneumonia newly-contracted at
pneumonia remains a leading cause of Staphylococcus aureus. These so-called
least 48-72 hours after hospitalization. It
death worldwide, especially in the very “typical” pneumonia agents generally
is the second most common type of
young and the elderly. present with primarily respiratory
nosocomial infection (after urinary tract
symptoms and a lobar consolidation on
The most common causes of pneumonia infections), and a common cause of
chest radiograph. Mycoplasma
are bacteria and viruses, but fungi, death in the intensive care unit. Ventilator-
pneumoniae, Chlamydophila
protozoans, and parasites can also cause a s s o c i a t e d p n e u m o n i a ( VA P ) i s
pneumoniae, and Legionella sp. are
infection. These organisms, typically pneumonia that occurs 48 hours or more
among the common atypical causes of
found in the nasopharynx, can infect the after a patient receives mechanical
bacterial pneumonia. They may present
lungs by inhalation. Additionally, airborne ventilation through an endotracheal tube
with a subacute onset, more generalized,
droplets (such as from a cough or sneeze) or tracheostomy. Intubation allows oral
non-respiratory symptoms and respond
or blood-borne infections (such as from and gastric secretions and
to different antibiotics than the typical
mother to baby during delivery) may microorganisms to enter the lower
organisms. The radiographs may not
spread the disease. respiratory tract. Multidrug-resistant
show an infiltration.
(MDR) organisms usually cause VAP.
For an overview of pneumonia, watch this
Common viral causes of pneumonia
video.
include influenza, respiratory syncytial

431
H e a l t h c a re - a s s o c i a t e d p n e u m o n i a presenting with sepsis. Empiric antibiotic tachycardia and tachypnea are
(HCAP) is a particular subset of therapy should be started once the usually present. Patients may be hypoxic
nosocomial pneumonia in which patients diagnosis of pneumonia is established, and hypotensive. Auscultation of the
come from the community but have even before the definite identification of a chest may reveal coarse rales or
frequent interactions with the healthcare microbial cause. bronchial breath sounds. There may also
system. It includes patients who were be dullness to percussion and increased
hospitalized within the last 90 days, Differential Diagnosis tactile fremitus. No single clinical finding
reside in a long-term care facility such as • Asthma is reliable in establishing a diagnosis of
a nursing home, receive hemodialysis or pneumonia.
• Bronchitis
wound care, have contact with a family
For examples of lung sounds, please see
member with MDR pathogens, or are on • Chronic obstructive pulmonary disease
(COPD) the this video.
chemotherapy or intravenous antibiotics.
Like HAP and VAP, patients with HCAP
• Lung cancer Emergency/Diagnostic Tests
are at risk for multidrug-resistant
and Interpretation
pathogens. • Pulmonary edema • Pulse oximetry to screen for hypoxia.
Hypoxia is an indication for admission.
Critical Bedside Actions and • Pulmonary embolism
General Approach • Upper respiratory tract infections • Chest x-ray (CXR): generally the most
Initial evaluation should focus on ensuring important study to determine the
adequate ventilation and oxygenation. Clinical Presentation presence of pneumonia, although it
H y p o x i c p a t i e n t s s h o u l d re c e i v e The classic symptoms of pneumonia are cannot establish the causative agent.
supplemental oxygen. Endotracheal fever/chills, cough (often productive of The absence of findings on CXR should
intubation may be required in patients purulent sputum), pleuritic chest pain, not preclude the use of antibiotics in
with severe respiratory distress. Early and and shortness of breath. Elderly patients patients thought to have pneumonia
a g g r e s s i v e flu i d r e s u s c i t a t i o n i s may present with nonspecific symptoms, based on clinical presentation.
necessary for patients who are such as general malaise, anorexia, and
hemodynamically unstable or who are confusion. On physical examination,
432
peptide (BNP) may help distinguish
Image 10.3
between CHF and pneumonia.
• Computed tomography (CT) of the
chest: more sensitive than CXR, but • The utility of routine blood cultures has
often not necessary. been questioned due to the low yield,
but they should be drawn before the
• Point-of-care ultrasound is becoming
initiation of antibiotics in patients who
more widely used for the rapid
will be admitted, particularly in severely
diagnosis of pneumonia. It can be more
ill patients. Cultures are positive in
sensitive than CXR, though findings of
20-25% of pneumonia caused by S.
consolidation on ultrasound are not
pneumoniae, but the percentage is
specific for pneumonia. The accuracy of
even lower in pneumonia due to other
ultrasound is operator-dependent.
causes. Positive blood cultures may
• This ultrasound video shows help determine local antibiotic
consolidation and pleural effusion resistance patterns.

Image 10.4 related to pneumonia.


• Sputum Gram stain and culture can
• A complete blood count (CBC) may help identify a bacterial pathogen, but
show the presence of leukocytosis, but like blood cultures, the usefulness in the
this often does not affect overall emergency department setting is very
management. limited. With an adequate specimen,
sputum studies are positive in more
• An elevated lactate level may indicate than 80% of cases of S. pneumoniae,
the need for more aggressive fluid but that percentage is much less for
resuscitation. other pathogens.

• If congestive heart failure is in the


differential diagnosis, B-type natriuretic

433
• Urine antigen tests are available for The following recommendations are • I f the patient is allergic to penicillin, use
Legionella pneumophila serotype 1 adapted from: the EMRA Antibiotic aztreonam and levofloxacin instead
(74% sensitivity) and pneumococcus. Guide; Musher et al., NEJM; and World
• If Pseudomonas is likely, use double
Health Organization (WHO) guidelines.
• Rapid diagnostic tests are available for coverage until susceptibilities are back
many viruses, including RSV and Outpatient therapy (adults): – levofloxacin or gentamicin, with the
influenza. addition of an anti-pseudomonal beta-
• Amoxicillin/clavulanate, with the lactam: cefepime, piperacillin/
Treatment Options addition of azithromycin or doxycycline
tazobactam, or aztreonam
Once pneumonia has been diagnosed, if atypical organisms are suspected
antimicrobial therapy should be started HCAP, HAP, or VAP (adults) [choose
• L e v o flo x a c i n o r m o x i flo x a c i n
as soon as possible, as early initiation one from each category below]:
monotherapy may be used instead
leads to better outcomes. Antimicrobial
• Beta-lactams: cefepime, ceftazidime,
therapy should be tailored to the most • If influenza is suspected, treat early with
piperacillin/tazobactam, aztreonam
likely causative organisms in order to oseltamivir
avoid drug toxicity, decrease the rate of • Levofloxacin, or azithromycin plus
resistance to broad-spectrum antibiotics, Inpatient therapy (adults):
gentamicin
and reduce cost. The empiric treatment
• Ceftriaxone or cefotaxime, with the
of CAP has been made more difficult by • MRSA coverage: vancomycin or
addition of azithromycin or doxycycline
t h e e m e rg e n c e o f d r u g - re s i s t a n t linezolid
Streptococcus pneumoniae (DRSP). • Levofloxacin or moxifloxacin may be
Outpatient therapy (pediatrics):
used instead
Risk factors for resistant S. pneumoniae
• Amoxicillin (preferred in low-resource
include age > 65 years; recent treatment • If influenza is suspected, treat early with
settings) or amoxicillin/clavulanate
or repeated therapy with beta-lactams, oseltamivir
macrolides, or fluoroquinolones; and • If atypical pneumonia suspected,
• If MRSA is suspected, vancomycin or
medical comorbidities, including azithromycin
linezolid should be added
immunosuppression.
434
Inpatient therapy (pediatrics): although it more often causes COPD myringitis, though it is actually not
exacerbations than pneumonia. common and is nonspecific.
• Ampicillin
• Klebsiella pneumoniae rarely causes • The Legionella genus is comprised of
• Ceftriaxone or cefotaxime CAP in a healthy host but can cause over 50 species of intracellular

• If MRSA is suspected, add clindamycin severe pneumonia in patients with o rg a n i s m s t h a t l i v e i n a q u a t i c


or vancomycin chronic illnesses, such as alcoholism or environments and cause infection when
diabetes. The sputum produced is often inhaled. There have been no
• If atypical organisms suspected, mixed with blood, giving it the color of documented cases of person-to-person
azithromycin red currant jelly. transmission. Elderly patients are at risk
of severe pneumonia, often with
Special Considerations • S. aureus can cause severe, necrotizing
gastrointestinal symptoms such as
pneumonia with cavitary lesions. It can
Specific Organisms and abdominal pain and diarrhea.
often occur after an infection with
Associations Hyponatremia is often present as well.
influenza. Intravenous drug users can
• S. pneumoniae is the single most
also get hematogenous spread. • Anaerobic infections can result from the
commonly identified organism in both
aspiration of oropharyngeal contents.
adults and children with CAP. It is • C. pneumoniae is a relatively common
These infections are often
classically associated with rust-colored cause of CAP, causing subacute
polymicrobial. Risk factors are patients
sputum and lobar consolidation. systemic symptoms, such as fever,
with decreased level of consciousness
malaise, myalgias, and non-productive
• H. influenzae is a common pathogen in or severe periodontal disease.
cough.
patients with chronic obstructive
• Fungi are a rare cause of pulmonary
pulmonary disease (COPD), diabetes, • M. pneumoniae is a common cause of
infections. Histoplasma capsulatum
m a l i g n a n c y, a l c o h o l i s m , a n d CAP, particularly in younger patients. It
lives in soil that contains large amounts
malnutrition. can be associated with rashes, sore
of bird or bat droppings and causes
throat, and ear pain. The classic finding
• M. catarrhalis is another common disease when spores are inhaled. It is
associated with this pathogen is bullous
pathogen in patients with COPD, found most commonly in the central
435
and eastern United States, near the may cause pneumonia. Viruses are a The most common cause of community-
Ohio and Mississippi River valleys. H. common cause of pneumonia in infants acquired pneumonia in the elderly
capsulatum can also be found in and young children. RSV and remains S. pneumoniae. However,
Central and South America, Asia, Africa parainfluenza infections most often occur geriatric patients are more likely to have
and Australia. Coccidiodes immitis is in the wintertime. Symptomatic treatment resistant types of this organism.
found in the southwestern United is recommended for cases of viral
States and Central and South America. pneumonia. In school-age children, M. Pregnant Patients
Pneumonia is the most frequent cause of
pneumoniae and C. pneumoniae
• Other rare causes of pneumonia are: non-obstetric infection in the pregnant
infections are more common. Pediatric
tularemia (caused by the bacterium p a t i e n t . T h e p re g n a n t w o m a n i s
patients often do not present with the
Francisella tularensis) is spread by predisposed to infection due to an
classic signs of pneumonia, such as a
contact with infected mammals, alteration in immune status, making her
productive cough. Often, the only signs
especially rabbits. Psittacosis (caused more susceptible to infection. In the
are tachypnea and increased work of
by Chlamydia psittaci) can be spread to postpartum period, pneumonia can occur
breathing.
humans from infected birds. Sheep, from aspiration as a complication of
cattle, and goats are the natural Geriatric Patients obstetric anesthesia. Additionally, there is
reservoir of Q fever (caused by Coxiella Elderly patients are more susceptible to an increased risk of aspiration due to
burnetii). pneumonia than younger patients and increased intragastric pressure from the
have a higher rate of morbidity and gravid uterus and relaxation of the
Pediatric Patients mortality. Even after recovery from gastroesophageal sphincter due to
Pneumonia is the leading cause of death
pneumonia, they have a higher rate of progesterone.
of children worldwide, killing more than
mortality in one year. They often do not
malaria, measles, and AIDS combined. In Evaluation and treatment are more
present with the classic signs and
neonates, organisms that colonize the complicated because there are two
symptoms of pneumonia, such as fever,
maternal vaginal canal, such as Group B patients to care for simultaneously. Chest
cough, sputum production, or
Streptococcus (GBS), Listeria radiography should be performed with
leukocytosis. Frequently, the only sign of
monocytogenes, and Escherichia coli the protection of the fetus. Appropriate
pneumonia may be confusion or malaise.

436
antimicrobial therapy should be selected choice for PCP, although alternatives The PSI is the most widely studied clinical
t o a v o i d t e r a t o g e n i c i t y. F e t a l (such as pentamidine, dapsone, and prediction rule for pneumonia. It stratifies
complications are common as fever and atovaquone) are often needed due to patients into five classes for risk of death
hypoxemia are harmful to development. allergic reactions, adverse effects, or (Risk Class I to V) from all causes within
Preterm labor is a known complication of treatment failure. Adjunctive therapy with 30 days of presentation based on medical
pneumonia, and tocolytic therapy may be corticosteroids has been shown to history, physical examination, and
r e q u i r e d . Te s t i n g f o r G r o u p B improve survival, especially in patients laboratory/radiologic findings. All-cause
Streptococcus before delivery and who are hypoxic. mortality ranges from 0.1% for Risk Class
intrapartum administration of antibiotics I to 27.0-29.2% for Class V. As points are
Another pathogen to consider in
can prevent the transmission of the assigned by age, it may underestimate
immunocompromised patients is
bacteria to the neonate. severe pneumonia in otherwise young,
Mycobacterium tuberculosis.
healthy patients and may overestimate
Immunocompromised Patients Cytomegalovirus (CMV) and varicella
severity in older patients (any patient over
Pneumocystis jirovecii (formerly P. carinii) zoster are rare causes of viral pneumonia.
50 years of age is automatically classified
is a fungal agent that does not cause
into Risk Class II).
infection in healthy people but is an Disposition Decisions
A number of clinical prediction rules and Click on the link to access a calculator for
important cause of opportunistic infection
guidelines have been developed to
in immunocompromised hosts. It remains PSI.
determine whether patients with CAP
the most common AIDS-defining illness in
individuals with human immunodeficiency
should be admitted or can be safely CURB-65 severity score
treated as an outpatient. As with all The PSI score uses twenty variables and
virus (HIV). The classic symptoms are
clinical prediction rules, these scores may be cumbersome to use in the
fever, nonproductive cough, fatigue,
should be used as a guideline and should emergency department. The CURB-65
shortness of breath especially with
not override the judgment of the score only requires five variables and is
exertion, bilateral interstitial infiltrates,
physician. easier to compute.
a n d h y p o x i a . Tr i m e t h o p r i m /
sulfamethoxazole (TMP/SMX), also • Confusion
Pneumonia Severity Index (PSI)
known as cotrimoxazole, is the drug of

437
• Urea > 7 mmol/L (in the United States, References and Further Reading,
blood urea nitrogen > 19 mg/dL) click here
SMART-COP
• Respiratory rate ≥ 30 breaths/minute The SMART-COP rule is a clinical rule
that predicts which patients with
• Blood pressure (systolic < 90 mmHg or
community-acquired pneumonia may
diastolic ≤ 60 mmHg)
need intensive care, such as mechanical
• Age ≥ 65 years ventilation or inotropic support. A
SMART-COP score of ≥ 3 points
One point is given for each variable and identified 92% of patients who required
patients can be stratified according to intensive care measures. While this score
increasing risk of mortality, ranging from was superior to CURB-65 for predicting
0.7% mortality for a score of 0 to 57% whether a patient would need intensive
mortality for a score of 5. Consider an respiratory or vasopressor support, like
ICU admission for patients with a score of the PSI, the sensitivity of SMART-COP is
4 or 5. reduced in younger patients and was
noted in one study to stratify 15% of
Compared to the CURB-65 score, the PSI
young adults incorrectly.
identified a greater number of patients as
low-risk (68% vs. 61%). The low-risk To see the SMART-COP tool, follow the
patients according to the PSI had a link.
slightly lower 30-day mortality (1.4%)
compared to the CURB-65 (1.7%). The Ultimately, the decision to admit a patient
clinical relevance of the slightly improved depends on the physician’s judgment, but
accuracy of the PSI is unknown. all the factors listed in the above scoring
systems should be considered.
To calculate a CURB-65 score, click on
the link.

438
Section 4

Spontaneous Pneumothorax

Case Presentation
by Mahmoud Aljufaili A 26-year-old male, with no significant medical history,
presented to the emergency department with acute shortness
of breath and associated right-sided chest pain. The pain
started suddenly while the patient was at rest, it was sharp
and worsening with inspiration. He denied a history of trauma,
fever, cough or any other constitutional symptoms. In the ED,
apart from tachypnea, his vitals were within normal limits. He
was not in distress. The trachea did not deviate. Breath
sounds were markedly diminished on the right side, with
normal breath sounds on the left side. No wheeze or crackles
were appreciated. The chest x-ray is shown below. What is
your diagnosis and plan for this case?

439
Image 10.5 Introduction
Pneumothorax refers to the presence of air in the pleural cavity. It
can impair oxygenation/ventilation. There are two types of
spontaneous pneumothorax 1) primary, and 2) secondary. Primary
refers to no underlying disease. Secondary refers to underlying
pulmonary disease which has a worse prognosis.

Causes of secondary pneumothorax are various as follows;

• Vascular: Pulmonary infarct

• Airway disease: Asthma, COPD, Cystic fibrosis

• Neoplasm: Primary or metastatic

• Infective: Tuberculosis. Pneumocystis carinii pneumonia, lung


abscess

• Interstitial lung disease: Sarcoidosis, Idiopathic pulmonary


fibrosis

• Miscellaneous: Endometriosis

Risk factors increasing spontaneous pneumothorax risk;

• Family history

• Smoking

• Change in atmosphere pressure

440
• Marfans – in the absence of lung symptoms are the driven factor for the • more than 2 cm, then it is a large
disease treatment options. However, knowing the pneumothorax
pneumothorax size is useful to decide the
Critical Bedside Actions and • The American College of Chest
next step in majority of the cases. At
General Approach Physicians
least, today, the algorithms are still
Assess the stability of the patient: Our designed to the size of the • If the distance from apex to cupola is
first responsibility is to evaluate the pneumothorax.
patient vitals and control to airway, • Less than 3 cm, then it is a small
breathing, circulation abnormalities. In How to estimate the size of the pneumothorax
any instability, immediate actions are pneumothorax?
• More than 3 cm, then it is a large
needed to stabilze the problem. Although
• On the bedside, this can be done with pneumothorax
there is a low chance to have a tension
Chest x-ray. It is ideal to get upright,
pneumothorax in spontaneous
  postero-anterior, and inspirium- Image 10.6
pneumothorax, this can be the worst
expirium x-rays. However, good quality,
case scenario for those patients. Oxygen,
s i t t i n g a n t e ro - p o s t e r i o r b e d s i d e
IV lines and cardiac monitorization may
portable x-rays may guide us well.
b e n e c e s s a r y. N e e d l e o r t u b e
thoracostomy can be necessary • British Thoracic Society guidelines:
immediately on the bedside. These
• If the interpleural distance at the level
critical bedside actions are rarely needed
of the hilum is
for asymptomatic or mildly symptomatic
 patients. Therefore, we may have time to • less than 1 cm, then it is a small
use proper diagnostic techniques for pneumothorax
differential diagnoses.
• 1 to 2 cm, then it is a moderate MacDuff A, Arnold A, Harvey J Management
Size of pneumothorax: When we pneumothorax of spontaneous pneumothorax: British
Thoracic Society pleural disease guideline
diagnosed pneumothorax, the patient
2010 Thorax 2010;65:ii18-ii31.

441
• Primary or secondary • Esophageal rupture t h e difference in small pneumothorax,
especially in a busy and noisy ED
Differential Diagnoses • Toxin ingestion
environment.
The patients present mostly with
• Epiglottitis
shortness of breath (SOB). Therefore, Hypotension (think tension
pulmonary, cardiac and other causes of • Anemia pneumothorax!). This is very important
SOB should be considered first. “red flag” for a pneumothorax patient.
If the pneumothorax is made clinically or The patients are generally agitated
Pulmonary radiologically, then the types should be because of hypoxemia and low blood
• Airway obstruction confirmed as spontaneous, traumatic, supply to the brain. This finding should
primary, secondary or tension etc. warn physicians to immediate action to
• PE
treat the pneumothorax.
• Pulmonary edema
History and Physical
Examination Hints Emergency Diagnostic Tests
• Anaphylaxis Sudden pleuritic chest pain. Most often
and Interpretation
occur at rest.
• Asthma Chest X-ray
Increased work of breathing and • Displaced visceral pleural line without
• Cor pulmonale
tachypnea can be seen in moderate and lung markings between pleural line and
• Aspiration severe pneumothorax. chest wall

Hypoxemia can be seen in severe cases. • Deep sulcus sign on supine x-ray
Cardiac
• MI Normal oxygen saturation does not rule
out pneumothorax. The Chest x-ray shows left side large
• Tamponade pneumothorax with fully collapsed lung
Reduced breath sound on the affected tissue. If the patient is vitally unstable
• Pericarditis side is more obvious with the  increased (hypotensive, tachycardic, hypoxemic)
size of pneumothorax (moderate or and agitated, then this x-ray means
Others
severe). Auscultation may not appreciate “tension pneumothorax.” If the patient
442
vitally stable, there is no tension. There is Ultrasound Image 10.8
very important teaching point for all • No sliding lung sign
physicians. tension pneumothorax is a
• Barcode (instead of the wave on the
clinical diagnosis, not imaging diagnosis.
beach) appearance on M-mode (video)
Therefore, if the patient is clinically
unstable and there is no breath sounds This video shows normal (left) and
on the left side, this is tension abnormal (right) lung findings. Left side
pneumothorax until proven otherwise, shows normal pleura and lung tissue
and this x-ray should not be ordered. relation and called seashore sign. Right
Nowadays, yes, you can use ultrasound one shows no clear differentiation
in seconds to diagnose if you are in between these structure and look like a
doubt. barcode. This finding is a warning for
pneumothorax presence.

This video shows “lung Point.” Please


Emergency Treatment
Image 10.7
pay attention to hyperechogenic (white)
Options
Initial Stabilization with oxygen, needle or
line (pleura). Half of the pleura is moving,
tube thoracostomy (if necessary) is the
but other half is standing still (no lung
the first priority for unsatble patients.
sliding). The connection point is the
border of pneumothorax line. Oxygen: increase pleural air resolution by
3-4 times, and alone helps to absorbtion
Computerized Tomography of the air by 1-2% a day. Therefore, some
Very sensitive and specific
institutions may prefer to observe small
CT image shows left side pneumothorax, pneumothoraxes with supplemental
collapsed lung tissue. oxygen.

443
Procedures This video demonstrates chest tube Small pneumothorax with no
Needle or catheter aspiration as effective insertion symptoms and  normal findings can be
as chest tube for small pneumothorax. discharged.
The image shows left side chest tube
Therefore, they are both appropriate
location. Please see the below algorithm below to
treatment options. Although large
understand possible treatments and
pneumothoraxes may require tube
Image 10.9
thoracostomy, choosing the narrow tube
Image 10.10 Management of
size is effective as wide tubes. Most of Spontaneous Pneumothorax
the spontaneous pneumothorax are easily
and safely treated with pig-tale catheters.

N e e d l e d e c o m p re s s i o n ( v i d e o ) i s
necessary for tension pneumothorax. The
classical teaching was 2nd intercostal
space, mid-clavicular line, over the rib
insertion. However, this location is
recently controversial, especially in obese
patients. Therefore, for adults, the new MacDuff A, Arnold A, Harvey J Management
of spontaneous pneumothorax: British
location is mid-anterior axillary line Thoracic Society pleural disease guideline
2010 Thorax 2010;65:ii18-ii31.
crossing with 4-5 th intercostal line. This
location is also entry side of the chest Disposition Decisions disposition decisions.
tube. The below video demonstrates old Recurrent pneumothorax, the patient with
version of needle decompression. abnormal vitals, bilateral pneumothorax, If patients were discharged no flying for a
However, this location is still acceptable and all secondary pneumothorax should week after resolution and no diving are
in skinny adults and children. be admitted. standard recommendations.

References and Further Reading, click

444
Chapter 11

Selected
Psychiatric
Emergencies
Section 1

Acute Psychosis In The Emergency Department

Case Presentation
by Elizabeth Bassett, Nidal Moukaddam, and A 25-year-old female is brought in by police after being found
Veronica Tucci
in a gas station, behaving bizarrely, talking to herself. The
patient has no identification, cannot provide her name, and no
medical history is available. She is responding to internal
stimuli, responds to questions with inappropriate laughter and
illogical statements such as “look, a bird, I am queen, meow,
what, Jesus, leave me alone,” and not making eye contact
with staff. She is noted to be paranoid, repeatedly looking at
the air vent above her bed. Initially calm, she became violent
after attempts to establish IV access. Verbal de-escalation and
redirection were not fruitful. The patient was placed in
physical restraints and eventually required emergency
pharmacologic intervention. Initial vital signs are Temperature
100.1 Fahrenheit (38.4 degree Celsius), HR 120 bpm, BP
Audio is available here 110/75 mmHg, RR 24 per minute, O2 saturation 98%. A liter

446
of IV fluids is given, and an hour later the patient Introduction
Acute psychosis may be encountered on a daily basis in the
is sleeping, physical restraints are removed, and
emergency department (ED). Psychosis is characterized by
all vital signs are within normal limits. disorganized thinking, delusions (false, unshakable beliefs), and
hallucinations, often auditory, visual. Acute psychosis can also be
The patient wakes up again, slightly calmer, but is
accompanied by behavioral changes and agitation that are not
perseverating on being pursued by a dark necessarily commensurate with the severity of psychotic
organization which can read her mind, and wants symptoms. The role of the emergency physician, in addition to
medically stabilizing and treating acute agitation/psychosis, is to
to cast her in a pornographic video. She insists
determine whether the patient is experiencing symptoms related
that the technician assigned to the ED is an agent to primary psychiatric diagnosis or secondary to a medical illness.
of evil, and refuses further vital signs. She also This can be challenging given both the limited time and history
available to the emergency physician, and the often-noted lack of
refuses oral medications, and when asked if
cooperation of acutely psychotic patients, known to be amongst
pregnant, lowers her voice and says “that’s why the hardest amongst patients in the emergency room. The
they’re after me, help me please.” She refuses to distinction, however, is critical, as incorrectly diagnosing a
provide further history, and from that point on, patient’s behavior as primarily psychiatric in nature, and missing
causes of altered mental state (AMS), can lead to dire
becomes mute.
consequences for the patient. This chapter will cover
management aspects of psychosis in the ED.

General Approach and Critical Bedside


Actions
1. Ensure the safety of the patient and the medical staff. Have a
low threshold to call security personnel early as unsafe
behavior can escalate quickly. Physical restraints and
medications may be required early on.
447
2. Recognize abnormal vital signs. 4. Substance-induced psychosis (think routinely has markedly elevated
stimulants, e.g., amphetamines, blood pressure values)
3. Have the patient put on a telemetry
synthetic cannabinoids)
monitor and establish IV access. 12.Thyrotoxicosis
5. Infection – either systemic or central
4. Assess for immediately life-threatening History Taking and Physical
nervous system (think sepsis or
and reversible causes of psychosis. Examination Hints
encephalitis)
Check a point of care glucose for
hypoglycemia, check oxygen 6. C e n t r a l n e r v o u s s y s t e m l e s i o n History Taking Hints
saturation for hypoxemia, and expose (particularly in patients with a history of A patient with acute psychosis may be a

the patient to look for evidence of cancer of AIDS) poor historian and collateral information

trauma. ( f ro m p o l i c e , E m e rg e n c y M e d i c a l
7. Intracranial bleed (especially if Services, or patient’s family) is often
5. Perform a head to toe complete evidence of trauma or anticoagulated helpful. Make every effort to meet with
physical exam with attention to patient) them at the time of arrival, as such
neurologic exam. information may not be available later on.
8. Hyperthermia/heat stroke (unlikely with
For our patient, no history is available;
Differential Diagnosis a temperature of 100.1, however,
however, in general, the key information
1. P r i m a r y p s y c h i a t r i c e t i o l o g y : checking a rectal temperature may
sought includes:
schizophrenia versus schizoaffective reveal significantly higher core body
disorder, depression with psychotic temperature) 1. Pick up location: Street, home, nursing
f e a t u re s , b i p o l a r d i s o rd e r w i t h home? Get contact information of the
9. Hypoxia/hypercarbia (unlikely given
psychotic features family or nursing home if available,
this patient’s pulse oximetry reading)
including prescribed medications. If
2. Hypoglycemia
10.Vitamin deficiencies (Wernicke’s) the patient was from home or nursing
3. Drug or alcohol intoxication or home, was any medical history
11.Hypotension/hypoperfusion (unlikely
withdrawal provided such as a history of mental
with systolic of 110 unless the patient
illness or past similar episodes. The

448
absence of previous psychiatric history and waning course? Is the patient’s o r medical reasons for the symptoms at
strongly suggests a medical cause of mental status improving or declining? hand.
the behavior.
6. H a s t h e p a t i e n t b e e n s t a r v i n g 1. HEENT: Look for evidence or recent
2. Timing: When was the patient last seen themselves because of psychotic trauma – lacerations, abrasions,
at baseline? Has the onset of abnormal beliefs? hematoma, basilar skull fracture
behavior been gradual or rapid? Rapid (raccoon eyes, battles sign, CSF or
7. How paranoid is the story the patient is
onset suggests underlying medical blood in the ears). Look for evidence of
telling you? While individuals may
condition or drug use. past traumatic brain injury as
indeed be the target of mysterious
evidenced by old neurosurgical scars.
3. What did the scene look like: Empty organizations, most paranoid,
pill bottles, alcohol, illicit drugs, a persecutory delusions are extremely 2. Eyes: Pay special attention to the
potential for other toxic exposures? unlikely. ocular exam. Assess for pupil size and
We r e t h e r e a n y p r e s c r i p t i o n responsiveness to light, presence or
8. Do they have pre-existing medical
medications at the scene? Obviously, absence of extra-ocular eye
conditions they have been neglecting
the presence of prescription anti- movements, and presence of
because of their psychosis? Many
psychotics can help with the diagnosis nystagmus. Ocular findings can be a
patients with mental illness have
but the entire medication list can help clue towards various toxidromes or
medical comorbidities, often poorly
with determining the patient’s past space-occupying lesions.
treated.
medical history and may be the key in
3. Neck: Assess for meningismus and
diagnosis if the etiology is medical in
Physical Examination Hints thyromegaly
nature. It is imperative that a head to toe exam
be performed. This may be the only full 4. Pulmonary: The presence of rales,
4. Vitals and blood sugar of the patient en
physical exam that the patient receives wheezing can be a clue that the patient
route to the hospital if available.
while in the hospital and therefore the is experiencing hypoxia secondary to
5. The mental status of the patient at the only opportunity to assess for traumatic CHF, COPD, or asthma. Hypoxia can
scene vs. on arrival. Is there a waxing be a cause of the patient’s altered

449
mental status although unlikely if the 8. Neurologic: A complete neurologic a n abnormal neurologic exam should
patient’s oxygen saturation is high. exam is often difficult to perform as it prompt a CT head to assess for a bleed
Rales or diminished breath sounds requires cooperation on the part of the or lesion. However, note that inpatient
may be a clue, especially in elderly patient. Assess for cranial nerves, psychiatric facilities may require testing
patients, that the AMS is secondary to strength, sensation, coordination, independent of clinical status.
a pulmonary infection. reflexes, and gait; focal deficits
suggest a medical cause of psychosis. Emergency Treatment
5. Cardiac: This exam is unlikely to aid in Options
the diagnosis; however the presence Emergency Diagnostic Tests These are divided into medical and
of an irregularly irregular heartbeat may and Interpretation psychiatric.
indicate that the patient is The lab and radiologic studies are
anticoagulated and therefore at dependent on the clinical presentation. Initial Stabilization
increased risk of spontaneous or As always, ensure airway, breathing, and
Indicated studies may include
traumatic intracranial bleed. circulation. Ensure a safe and if possible,
electrolytes, acetaminophen levels,
Tachycardia or bradycardia may also a low-stimulus environment to minimize
salicylate levels, LFTs, ammonia, PT/INR,
indicate various toxidromes. agitation. This may be done by
thyroid studies, HIV, pregnancy test, ECG,
attempting to redirect the patient verbally
CXR, UA, urine culture, blood culture, CT
6. A b d o m e n : A s s e s s f o r r i g i d i t y and tur ning down the lights and
head, LP. Increasing age, preexisting
suggesting trauma or infection. Look decreasing the number of people in the
medical comorbidities, the absence of
for evidence of encephalopathy: room. Temporary physical restraints and
past psychiatric history warrant, whereas
hepatomegaly, ascites, caput medusae emergency pharmacologic intervention
young patients with a known psychiatric
may be needed if the patient escalates.
7. Skin: Assess for rashes, petechiae, history, normal vitals/physical exam, a
Psychotic patients will benefit from an
track marks. This can be a clue to classic toxidrome or admitted drug use
antipsychotic agent.
infection, trauma, intoxication, or may not require lab or imaging studies
withdrawal. unless the patient’s mental status fails to
Medications
improve on serial assessments. Evidence Antipsychotics and benzodiazepines are
or trauma, the use of anticoagulants, or the mainstays of treatment for acutely
450
psychotic patients who are agitated or When possible, get a baseline ECG o f time, have returned to their baseline
violent. These may be given alone but are prior to administration. mental status.
often given in conjunction and will be
3. Atypical Antipsychotics: Olanzapine, Admit to the psychiatric facility: This is
beneficial not only to the patient with
Ziprasidone, Quetiapine, Risperidone. appropriate for patients with higher
psychosis secondary to primary
These drugs also cause QT symptom burden who have been
psychiatric condition but also in various
prolongation but generally have fewer medically cleared; dangerousness to self
sympathomimetic toxidromes as well.
extrapyramidal side effects. or others, or inability are unable to care
1. Benzodiazepines: For the acutely for themselves due to psychosis warrant
agitated patient, the benzodiazepines Disposition Decisions admission.
of choice are typically midazolam and Admission to the hospital: This is

lorazepam, given intravenously or appropriate in patients with an identified

intramuscularly. Midazolam may be underlying medical cause of psychosis or


References and Further Reading, click
preferred as it has a shorter time of patient’s in whom no underlying medical
here
onset as well as a shorter duration of cause has been found, but a medical

action compared to lorazepam. condition is suspected, for example,

Generally, diazepam is avoided due to patients with no prior psychiatric history

its long half-life. In the elderly or or patients with abnormal vitals. Inpatient

pediatric patients, use lower doses. admission may be necessary to follow up

Watch for paradoxical disinhibition. on studies, particularly cultures.

2. First-generation antipsychotics:
haloperidol and Droperidol. Potential Discharge to home: This may be
side effects include extrapyramidal appropriate for patients with substance-
side effects, e.g., dystonic reaction, as induced psychosis who, either with the
well as potential arrhythmias, help of medications or simply the tincture
especially long QT leading to torsades.

451
Section 2

Stabilization and Management of the Acutely Agitated or


Psychotic Patient
Case Presentations
by Michelle Chan, Nidal Moukaddam, Veronica
Tucci Case 1
It is a quiet Wednesday night in the emergency department
when you suddenly hear someone coming down the hall
continuously spouting out a string of profanities.  You leave
the comfort of your chair to see what the commotion is all
about only to find a 37-year-old female brought in by police
for altered mental status.  She is acutely agitated on
presentation, spouting obscenities non-stop, refusing to
answer questions and uncooperative with a physical exam.

Case 2
As you are pondering your next step, you see the paramedics
wheeled an older gentleman past you and into the next room.
 You step into the next room to get a report.  The family is at
the bedside and states the patient is an 82-year-old male with
Audio is available here
a history of hypertension and BPH who has been increasingly

452
confused and aggressive over the past two days. General Approach and Critical Bedside
 You note that he is mildly tachycardic when you Actions
hear the PA system announce, “Security is General Approach and Key Concepts
The first steps in evaluating any patient who presents to the
needed in the critical care hallway.”
emergency department are to assess and ensure that the
Case 3 patient’s airway, breathing, and circulation is intact.   However,

A nurse pops her head into the room and when presented with an acutely agitated or psychotic patient,
even before assessing the ABC’s, you should ask yourself if the
requests your immediate assistance.  You follow patient poses an immediate threat to the safety of both your
him down the hall and see your charge nurse patient or your medical staff.
along with three security officers trying to hold If you determine that the patient poses an imminent threat to self
down a male patient.  The patient, who appears or staff, a number of factors should be considered prior to
to be in his late twenties, is actively kicking and administration of medications in order to achieve rapid
stabilization without over-sedation or use of undue force.  These
trying to bite and spit at the medical staff.  He
factors include age, known psychiatric history, known or
appears flushed and diaphoretic. suspected substance abuse, and severity of agitation.   Case 3,
above, is a common scenario in the ED where a patient clearly
poses an immediate threat.  Clearly call out the medication(s) and
dosage(s) you would like, and while it is being prepared, attempt
de-escalation techniques.   Appropriate pharmacologic and non-
pharmacologic agents will be further discussed under Emergency
Treatment Options.

Once immediate safety has been established, make sure the


patient is placed on monitors and a full set of vitals, including
temperature, is acquired.  This should be followed by a thorough
453
primary survey.   A commonly used Common Causes of Acute Agitation or History and Physical
mnemonic is “ABCDE,” where A = airway, Psychosis Examination Hints
B = breathing, C = circulation, D = History is often limited in patients
disability, and E = exposure.   Disability Medical
presenting with acute agitation.
refers to assessing a patient’s level of Whenever possible, gather collateral
• Hypercalcemia
consciousness and quickly screening for information from family, emergency
reversible causes of altered mental • Hypercapnia personnel and police.  Key information to
status.   This includes examining pupils obtain include:
• Hypoxia
and checking blood glucose.  Hypoxemia
and hypotension should be noted in the • Infection (encephalitis, meningitis, • Past medical history
vitals.   Exposure is a critical bedside sepsis)
• Past psychiatric history (include current
action that is easily neglected.   All
psychotropic medications)
patients who present with acute agitation • Substance-related (alcohol,
hallucinogens, steroids, stimulants,
or psychosis should be fully exposed and • Allergies
synthetic marijuana)
changed into a hospital gown.   This
• Home medications
allows for rapid visual assessment for Psychiatric
overt signs of trauma, as well as the • Social history (alcohol use, substance
opportunity to check for concealed • Bipolar disorder abuse, living situation, caregivers, etc.)
weapons.
• Posttraumatic stress disorder • Recent health status (i.e., Has patient
Differential Diagnoses • Psychotic depression
reported headaches in the past few
The differential for patients presenting days suggestive of a possible
with acute agitation or psychosis is long • Schizophrenia spectrum disorders intracranial process or has an elderly
(see the list for common causes), but is patient reported dysuria suggestive of
best divided into organic (primarily possible urosepsis)
m e d i c a l ) a n d i n o rg a n i c ( p r i m a r i l y
• Baseline mental status
psychiatric) causes.

454
Although it is often said that history is Emergency Diagnostic Tests Emergency Treatment
80% of the diagnosis, in cases of acute and Interpretation Options
agitation, a thorough physical Blood work, diagnostic tests, and The goals for the treatment of acute
examination is all the more important due imaging should be guided based on agitation is early recognition, intervention
a limited HPI and review of systems.  The history and physical exam.   Depending before escalation into more violent
physical exam of every acutely agitated on your institution, laboratory tests behavior, and stabilization of life-
patient should include a full neurologic required for medical clearance should threatening conditions.   Therapy should
exam and head-to-toe visualization for also be taken into account.   A basic be aimed at decreasing agitation and
obvious signs of trauma or injury.   A full metabolic panel and CBC, although not psychosis to the greatest extent possible
neurologic exam may not always be standard, is typically ordered for most without oversedation in order to allow for
possible during the initial assessment but patients with acute undifferentiated further assessment of the patient to
should be completed as soon as feasible agitation or psychosis. determine the underlying cause.
and prior to disposition. Interventions can be divided into
A urine pregnancy should be ordered on
nonpharmacologic and pharmacologic
When dealing with acute undifferentiated all women of childbearing age.   When
strategies.
agitation due to a limited history and urine is unobtainable, consider a
physical, a number of key signs and qualitative hCG or substituting blood for Non-pharmacologic strategies include
symptoms may help to at least narrow urine on a point-of-care pregnancy test. environmental interventions, de-
the differential to organic vs. inorganic escalation techniques, mechanical
causes.  Organic causes of agitation tend In cases of suspected ingestion or
restraints, and seclusion.   Environmental
to be associated with abnormal vital substance abuse, consider checking
conditions are often difficult to control in
s i g n s , d i s o r i e n t a t i o n , flu c t u a t i n g acetaminophen and salicylate levels, as
the ED, but the concept is simple: create
symptoms, or signs of trauma, whereas, well as an EKG and measurable levels of
a safe space that minimizes stimulation.
inorganic causes lack these features. prescription drugs to which the patient
This means screening patients for
has access.
weapons, removing objects that could be
used as weapons (pens, chairs, or other
loose objects), finding space away from
455
the noise and activity of the ED when Although environmental and de- Whereas, in patients with
possible, and dimming room lights.   De- escalation strategies can be very undifferentiated agitation or psychosis,
escalation techniques involve both verbal effective, many times, acutely agitated or monotherapy with a benzodiazepine may
and non-verbal methods.   Once again, psychotic patients will require some form be a better option due to its added
the concepts are simple and should be of pharmacologic intervention.   The two anxiolytic effects and usefulness in cases
applied as first-line techniques in the major classes of drugs used for this of substance-related psychosis such as
management of acutely agitated or purpose are antipsychotics and phencyclidine (PCP) use or alcohol
psychotic patients.  Successful use of de- benzodiazepines.   They may be used in withdrawal.
escalation techniques will vary from combination or as monotherapy and are
When the above interventions fail to
situation to situation, but many times available in many formulations.   In the
stabilize an acute agitated or psychotic
hinges on the ability to establish rapport case of acute agitation or psychosis,
patient, physical restraints and/or
with the patient quickly.  Often times this these medications are most often given
seclusion may be necessary.   It is
can be accomplished by addressing a parenterally, either intramuscularly or
important to understand, however, that
patient’s basic needs of safety, hunger, intravenously for rapid tranquilization.
these are methods of last resort and
and comfort.   Provide reassurance that  Refer to Table 2 for common agents and
should never be used out of convenience
the patient is in a safe place, offer food, doses.
or as a form of punishment.   Seclusion
water or warm blankets, and make sure
Choice of medication(s) varies greatly differs from placing the patient in a safe
to address pain management.   It is
depending on personal preference; and less stimulating environment in that
equally important to be mindful of
however, the two most commonly used seclusion involves involuntary
personal space and avoid the threatening
agents are haloperidol and lorazepam.  In confinement.   Both seclusion and
or confrontational behavior.   Verbal de-
patients with a known history or high physical restraints are associated with
escalation techniques involve maintaining
suspicion for underlying psychosis, increased morbidity and mortality.
a calm and respectful demeanor while
monotherapy with an antipsychotic such
acknowledging the patient’s anger,
as haloperidol may be considered.
frustration or agitation.

456
Pediatric, Geriatric, substance-induced psychosis may be

Pregnant Patient and Other eligible for discharge if the patient is no


longer clinically intoxicated and back to
Considerations
baseline mental status.
Geriatrics
With age comes an increased risk for If after medical evaluation is complete,
dementia which is a potential cause of and no medical cause of agitation or
acute agitation and psychosis in the psychosis can be determined, patients
geriatric population.  Although effective in must then be evaluated for psychiatric
the management of psychosis, FDA causes.   Admission criteria for acute
issued a black box warning for psychosis due to an underlying
olanzapine and ziprasidone regarding psychiatric disorder primarily involves
their use in elderly patients with whether or not the patient is at high risk
dementia-related psychosis due to for continued harm to self or others.
increased mortality in this population.  Admission is also strongly considered for
first episodes of psychosis due to
Disposition Decisions inorganic causes.
Once stabilization of a patient’s acute
agitation or psychosis is achieved,
disposition should be guided by the References and Further Reading, click
underlying cause of agitation.   Organic here
causes of agitation or psychosis such as
sepsis, acute intracranial hemorrhage or
severe metabolic disturbances warrant
hospitalization until stabilization of the
underlying cause is also achieved.
  Whereas, patients who present with

457
Section 3

Medical Clearance – Suicidal thought/ideation

Case Presentation
by Veronica Tucci A 35-year-old female presents to the ED after the family called
the paramedics for “bizarre behavior.” She notes that her
family persuaded her to seek evaluation; however, they are not
with her currently. She seems somewhat paranoid and
tangential and is difficult to obtain a history from. On review of
systems does endorse some mild abdominal pain and
diarrhea. Her vital signs on arrival as recorded in triage are as
follows: heart rate 135, blood pressure 110/90, respiratory rate
24, oxygen saturation of 96% on room air, temperature 100.7.
When you speak with the family, they state that she has been
agitated and paranoid. They are also concerned that she
made suicidal threats while with friends.

On physical examination, you see an anxious appearing


woman. She is tachycardic with an irregular pulse. Her
Audio is available here
extremities are dry with 1+ lower extremity edema. Her

458
abdominal is soft and non- Introduction that this did not hold true in the two
Many medical conditions can present as areas that are mandatorily evaluated in
tender. She has no meningeal
psychiatric complaints. The case below the ED – suicidal ideation and acute
signs. She is tachypneic and will demonstrate the importance of the psychosis. This suggested that the
has crackles in the lung bases. medical evaluation of these patients, as increase in visits was related to non-
well as the need to keep a broad emergent psychiatric complaints that
Her neurologic examination is
differential diagnosis. There are also might be better managed by outpatient
non-focal. She reports suicidal mental health professionals.
medical problems which may exacerbate
ideation without a plan. psychiatric symptoms or need to be
ED physicians are often tasked with
addressed in order for a patient to be
evaluating these patients for medical
able to be transferred and safely
problems prior to clearing them for
managed at a psychiatric center. In
possible psychiatric evaluation and
addition to these scenarios, the patient’s
admission. They must control the acute
underlying psychiatric disorder may lead
symptoms, attempt to determine the
to an emergent medical condition, such
etiology of complaints (particularly
as an overdose or a self-inflicted trauma.
functional vs. organic), provide
Psychiatric complaints are common appropriate initial treatment, and
presentations for our ED (emergency determine disposition. Of the utmost
department) patients and are ever importance is identifying and treating
increasing. Mental health-related visits immediate life-threatening problems.
increase from 1992 – 2001, most Historical data, mental status
significantly in the areas of substance- examination, physical examination, and
related disorders, mood disorders, and appropriate ancillary testing are indicated.
anxiety. Pediatric mental health visits are
This process has previously been termed
also increasing. Interestingly, one study
“medical clearance.” While the
which showed this increasing trend found

459
importance of this process cannot be and treatment or using the term medically family member with them at all times.
stressed enough, is fraught with both stable. There has also been shown to be Importantly, she should not be allowed to
intrinsic and extrinsic difficulties. wide variation in the comprehensiveness leave the ED until the evaluation is
Nevertheless, a thorough medical of medical clearance examinations. complete.
assessment is imperative to taking
excellent care of this high-risk patient General Approach and History Taking and Physical
population. Critical Bedside Actions Examination Hints
The most important first step in the For this patient, you will want to obtain
One challenge is with the term “medical assessment of this patient is to assess for further history including prior episodes,
clearance” itself, which can be abnormalities in the airway, breathing or p a s t m e d i c a l h i s t o r y, a s s o c i a t e d
misleading. It means different things to circulation which may require immediate c o m p l a i n t s ( c o u g h , f e v e r, h e a t
different providers, and its overuse can stabilization. A rapid blood glucose level intolerance, headache, neck pain/
result in poor patient care. No standard should be obtained early on. The patient stiffness, changes in hair or skin, etc.),
criteria exist for what medical clearance should be placed on a cardiac monitor prior medications, drug and alcohol use,
consists of, or even what the status of a and continuous pulse oximetry, and IV prior hospitalizations. It is important to
medically cleared patient truly is. access should be established with tubes obtain collateral for this patient who may
Complicating this further, different collected for blood work. Place the be unable or unwilling to provide a full
specialties have their own approaches to patient on oxygen by nasal cannula, and and accurate history for you.
this evaluation. In addition, receiving consider IV fluid.
psychiatric facilities often have their own A thorough history and physical are the
requirements, irrespective of what the With regards to the family’s concern starting point of any patient evaluation.
treating ED physician and psychiatrists about suicidal ideation, some precautions Several studies looking at missed medical
believe to be medically indicated. Some should be taken. This may include diagnoses in patients with psychiatric
have suggested modifying the term or removing items and clothing from the complaints have shown that these should
replacing it instead with a thorough room that could be used for self-harm. have been identified if a proper history
discharge summary. Another suggestion The patient should not be left alone and and physical were performed.
is to provide a summary of the evaluation should have a staff member or a reliable Unfortunately, studies looking at the

460
thoroughness medical evaluations of History should be obtained from the The mental status examination (MSE)
these patients have often found them to patient in addition to those close to them plays a crucial role in the evaluation of
be incomplete. An incomplete medical like family and caregivers, and an effort these patients. The MSE needs to be
evaluation can lead to missed medical s h o u l d b e m a d e t o c o n fir m t h e focused and brief, and evaluate seven
diagnosis, which can be dangerous for information obtained from outside major areas (affect, attention, language,
patients. One study found that “medically sources whenever possible. Sudden orientation, memory, visual-spatial ability,
clear” had been documented in 80% of onset in changes in behavior, mood, or and conceptualization). Again, this should
patients where a medical diagnosis thought in a previously normal patient, or be structured and evaluate changes in
should have been identified. One a deterioration in a patient with a chronic alertness, cognition, behavior. Remember
retrospective chart review found that disorder should be suspicious for an delirium, dementia, and psychiatric illness
complete vital signs were only underlying medical etiology. h a v e s i g n i fic a n t d i ffe r e n c e s i n
documented 52% of the time. management and outcomes, and thus
Assessing for substance abuse, use, and
need to be identified. Delirium in the ED is
All patients require a complete history, changes are important. Also, inquire
associated with decreased survival. There
physical and mental status examination. about adherence to their current
are also alternatives to the traditional
This should be approached in an medication regimen. Family and social
mental status examination. The quick
organized fashion in order to determine stressor should be assessed. It is
confusion scale is a scoring system that
the etiology of their complaints as important to find out about medical
was published and is quickly obtained,
functional or organic. The medical comorbidities, or physical symptoms and
easily calculated, readily interpreted
evaluation of these patients should be no complaints as these might also indicate a
score.
different than of those presenting with medical etiology (trauma, fever, etc.). Be
medical complaints. The history and aware that many medications can lead to Differential Diagnosis
physical should guide laboratory and changes in behavior, especially in at-risk In the above patient, the following is a list
other diagnostic testing and imaging. The groups. Physical complaints, abnormal of possible etiologies for her symptom:
information gathered from this will form exam findings, and abnormal vitals must sepsis, diabetic ketoacidosis, pneumonia,
the clinical picture. be evaluated and addressed. pulmonary embolism, meningitis,
encephalitis, hyperthyroidism/thyroid

461
storm, schizophrenia, bipolar disorder, of abnormal vital signs in comparison to organic etiology of their symptoms. A
psychosis, salicylate ingestion, acute patients admitted to medical units. study looking at 658 psychiatric
intoxication, alcohol withdrawal/delirium outpatients receiving medical and
Some special groups are at increased risk
tremens. There are several abnormalities biochemical evaluation found the
of having a medical etiology of their
in history and physical examination which incidence of medical disorders producing
complaints, and care should be taken
suggest that the patient’s symptoms are psychiatric symptoms at 9.1%. The
when evaluating these patients. Several
not primarily psychiatric in origin. etiologies included infectious, pulmonary,
prior studies have identified these as the
thyroid, diabetic, hematopoietic, hepatic
Alterations in mental status may elderly, those with substance abuse,
and CNS. Another study of 100
incorrectly be attributed to psychiatric those without a prior psychiatric history,
psychiatric patients who had been
diagnoses. A review looked at 64 cases and those with pre-existing or new
previously medically screened found that
of patients admitted to the psychiatric medical complaints. Intoxicated patients
46% had a medical illness that caused or
ward, whom were later found to actually represent a particular challenge. In
exacerbated their symptoms and 80% of
have a medical diagnoses that explained addition to often providing a limited
these required treatment. They concluded
their symptoms. The etiologies identified history, they may express certain
that a battery of laboratory and ancillary
included intoxication, withdrawal complaints (like thoughts of self-harm),
testing would have identified the majority
syndromes, overdose. In this, they noted only while intoxicated. A study looked at
of these.
that none had an appropriate medical 100 consecutive alert patients with new
screening examination performed. In psychiatric complaints. They excluded Emergency Diagnostic Tests
another study looking at factors which those obviously intoxicated, prior and Interpretation
may have contributed to a patients diagnosis of abnormal behavior, those The following diagnostic testing should
symptoms being attributed to a with medical complaints and overdose or be considered in the above case.
psychiatric problems instead of a medical suicide patients. For all patients, they
one, found that these patients had a then performed a history, physical, panel
lower rate of complete history, physical of laboratory tests, CT scan of the head
examination, cognitive assessment, and lumbar puncture if febrile. They
indicated ancillary testing and treatment concluded that 63/100 patients had an

462
While everyone can agree that these b e the basis of beginning the psychiatric
Table 11.1 Medical Clearance -
Suicidal Thought/Ideation Diagnostic patients deserve a complete history and assessment and recommend considering
Tests physical, the role of laboratory and a period of observation to determine if
TEST COMMENT ancillary testing is less well delineated symptoms resolve as intoxication
and is often viewed differently among ED resolves. A study looked at patients with
Complete blood Anemia, hematologic
count abnormality and psychiatric physicians. What studies isolated psychiatric complaints and past
Complete Metabolic abnormality, are required for medical clearance of the medical history of psychiatric disorder.
metabolic panel uremia, liver failure, renal psychiatric patient, and whether this None of these had positive screening
failure
process should be standardized, or be laboratory or radiograph results. The
Electrocardiogram Arrhythmia, evaluation of
tachycardia and irregular performed on a case by case basis, is the remaining patients had a presenting
pulse source of much controversy. medical complaint as well, and these
Chest X-ray Pneumonia, heart failure, complaints directly correlated with the
other etiology of The American College of Emergency
tachypnea need for labs and radiography. They
Physicians published a clinical guideline
Urinalysis/Urine Source of sepsis concluded that patients with a psychiatric
on the subject. They suggest that
culture complaint with a documented past
diagnostic evaluation should be directed
Blood cultures Sepsis evaluation psychiatric history, negative physical
by the history and physical and routine
Troponin and BNP Heart failure findings and normal vital signs, who deny
laboratory testing of all patients is of very
current medical problems did not require
Thyroid function Hyperthyroidism / thyroid low yield. Routine urine toxicology
studies storm further labs or testing in the ED.
screens in awake, alert, cooperative
CT scan of the Abscess, meningitis,
brain patients do not affect ED management, Another systematic review of the
mass
and using this screening in the ED literature indicated that history, physical
Lumbar puncture Meningitis/encephalitis
because of the requirement of receiving examination, review of systems, and tests
Alcohol level/urine Intoxication, may be
drug screen psychiatric facilities or service should not for orientation had relatively high yield for
required at psychiatric
facility delay evaluation or transfer. They also say detecting active medical problems.
Acetaminophen Commonly ingested in that patients’ cognitive abilities rather Routine laboratory testing was relatively
and salicylate suicide attempts than a specific blood alcohol level should low yield. However, four groups were at
levels

Original by author 463


serious risk for medical problems, and more pressing concern is stabilizing this I n this case, the appropriate disposition
these included the elderly, substance patient from a medical standpoint. of the patient is the intensive care unit. In
users, patients with no psychiatric history, general, In patients presenting with
During her stay, she becomes more
and patients with pre-existing medical psychiatric complaints, the medical
tachypneic and supplemental oxygen is
disorders and/or concurrent medical screening exam should identify medical
placed. Her EKG reveals atrial fibrillation
complaints. A retrospective chart review problems which require admission, or
with a rapid ventricular response. Her
of pediatric patients presenting to an concurrent medical issues which may
troponin and BNP are mildly elevated.
academic pediatric emergency inhibit their ability to go to a psychiatric
Her CXR reveals signs of heart failure and
department for medical clearance for an facility. However, if after the evaluation
pulmonary edema. Her TSH < 0.01 and
acute psychiatric emergency found that none of these medical issues are
her T4 level is > 100. Overall, this is
screening laboratory tests resulted in few identified, or they are adequately treated
suggestive of hyperthyroidism/thyroid
management or disposition changes in in the ED, the patient needs to be
storm as the etiology of her psychiatric
patients if they had a noncontributory evaluated for the need for psychiatric
complaints.
history and physical, but did result in an admission. In a patient with suicidal
increased length of stay. The costs of Propanolol is promptly administered in a ideation, there are several components to
routine testing given the low yield also dose of 1 mg IV over 10 minutes. be considered.
should be considered. Propylthiouracil (PTU) is then
S e v e re a n x i e t y, p a n i c a t t a c k s , a
administered in a dose of 200 mg and
Emergency Treatment depressed mood, a diagnosis of major
iodine is administered 1 hour later.
Options affective disorder, recent loss of
Steroids are given to prevent the
The patient presenting above has several interpersonal relationship, recent alcohol
conversion of T3 to T4. She is admitted to
abnormal findings on history and physical or drug abuse coupled with feelings of
the intensive care unit for thyroid storm.
which make a medical etiology more hopelessness, helplessness,
She has a prolonged and complicated
likely. While safety precautions for self- worthlessness, global or partial insomnia,
hospital course and is discharged several
harm and levels of commonly ingested anhedonia, inability to maintain a job and
weeks later. Her psychiatric symptoms
medications should be obtained, the recent onset of impulsive behavior are
resolved with her medical treatment.
predictors of suicidal behavior.

464
evaluation is ultimately needed. A study mental health crisis, the discovery of
Diagram 11.1
looking at several risk assessment scales mental health issues in ED patients, and
found that in general, they overestimated approaches to advocating for improved
suicide risk. They did note that they might recognition and treatment in mental
help highlight important concepts and illness in children. The ED evaluation of
risk factors. This may be particularly pediatric mental health is crucial to the
useful for non-psychiatric medical child’s long-term care and treatment.
personnel or junior residents. They may
It is important to note the overall
help identify high-risk patients in the ED
significance of medical problems in the
early in assessment, and those that may
population of patients with psychiatric
need psychiatric referral.
disorders, and the challenges that they
Conclusion face interfacing with and accessing the
The role of the ED provider in psychiatric medical community. A study of this
care is increasing, and external resources population out of Nova Scotia showed
Original by author
are often inadequate. A study of increased mortality from cancer, which
There are several clinical rating scales in California EDs showed that there are may be attributed to delays in detection
suicide risk assessment. An example of limited mental health resources for or initial presentation and difficulties in
one is the SAD PERSONS scale. It stands suicidal patients. It suggested the need communication and access to healthcare
for: Sex, Age, Depression, Previous for more regional solutions including contribute to this finding. Another study
attempt, ethanol abuse, rational thinking improved access to mental health looked at compulsory community
loss, social support lacking, organized personnel and follow of suicidal patients treatment in this patient population. They
plan, no spouse, sickness. One point is and community mental health resources saw a reduction in all-cause mortality in
given for each. for patient referrals. The same is true in their intervention, group which they
the pediatric patient. The ED physician stated that might be partially explained by
Assessing the risk of suicide is
plays an important role in the pediatric ED increased contact with health services in
complicated, and complete psychiatric
in the stabilization and management of a the community. Looking at 200 patients

465
re c e i v i n g p s y c h i a t r i c c a re i n t h e challenging. The ED physician is tasked
outpatient setting for schizophrenia and with the initial assessment both of
affective disorder diagnosis, both groups psychiatric risk and medical clearance.
had greater odds of having comorbid Care should be taken to stabilize any life-
medical conditions than those in the threatening condition and then to try to
general population. differentiate a functional versus organic
cause of the patient’s symptoms. They
In addition to the challenges of
must also assess for any underlying
assessment, these patients present
medical problems that may exacerbate
logistical difficulties in the ED setting.
the patient’s symptoms or need to be
Patents with psychiatric related
managed at a psychiatric facility. The
complaints have long lengths of stays in
psychiatric disorder itself may also lead
the ED. Older individuals, the need for
to a life-threatening medical condition
hospitalization, restraint use and
that needs to be threatened or treated.
diagnostic testing prolonged the length of
This is a very important part of the care of
stay. Drug and alcohol screening also led
this challenging patient population.
to delays. They also tend to have high
rates of readmission. Predictors of 12-
month readmission and ED revisits for
References and Further Reading, click
patients with substance abuse, and
here
mental health-related complaints were
highest in those with dementia, psychotic
disorders, autism, impulse control
disorders and personality disorders.

The care of patients with psychiatric


complaints is complicated and

466
Chapter 12

Selected
Orthopaedic
Problems
Section 1

Back Pain

Case Presentation
by Funda Karbek Akarca A 45-year-old age male presented to the emergency
department with severe back pain after lifting a heavy object.
He described the pain radiated to the right leg. He had
difficulty with walking. His medical history revealed no
additional diseases except for occasional back pain. The vital
signs were normal. The physical examination showed palpable
peripheral pulses, no motor or sensory deficit, no drop foot or
murmur in the abdomen. Straight leg raising test is positive at
45 degrees. Palpation of the vertebrae revealed no tenderness
on spinous processes but paravertebral muscles spasm. The
patient’s pain decreased after resting in the supine position,
muscle relaxants, and analgesics. The patient was discharged
with a recommendation of neurosurgery visit in ten days.

468
Critical Bedside Actions and corticosteroid or anticoagulant •Abdominal aortic aneurysm
General Approach use, cancer
• Aortic dissection
Back pain is a common problem and
Make an orderly and thorough
affects up to 90% of the general • Upper Urinary Tract Infection, renal
physical examination
population at some point in their lives. It infarction, renal colic
is the fifth leading cause and accounts for Order necessary imaging and labs
• Abdominal infection (cholecystitis,
2% to 3% of emergency department
Assess the risks and consider the cholangitis, pancreatitis, retroperitoneal
visits. Although most back pain is due to
potentially life-threatening or abscess)
a benign and self-limiting reason, a
debilitating diagnoses.
minority of patients may face a risk of • Abdominal neoplasm
permanent neurological damage or death.
Differential Diagnoses
History and Physical
Acute, non-traumatic low back pain can
Spinal origin Examination Hints
be divided into three groups: • Musculoligamentous In many patients, a thorough history and
musculoskeletal causes with no
physical examination are essential and
neurologic deficits, musculoskeletal • Discopathy
s u ffic i e n t f o r d i a g n o s i s . T h e t i p s
causes with neurologic deficits and other
• Fracture indicating severe pathologies should be
causes that can present with back pain.
investigated (red flags are shown in Table
• Spondylolisthesis 1). Additional questions are whether the
Check vital signs; especially fever
• Vertebral osteomyelitis patient has a similar pain before, has any
Learn the history of current illness; prior diagnosis related to this complaint
pain duration, how the pain • Spinal epidural abscess or receive any treatment.
started and spread.
• Spinal epidural hematoma
Ta k e m e d i c a l h i s t o r y ; d i s c
• Neoplasm/metastatic disease
herniation history, recent spinal
a n e s t h e s i a o r s u rg e r y,
Nonspinal causes
469
Table 12.1 Red Flags In Back Pain Illustration 12.1
HISTORY PHYSICAL EXAMINATION
Pain duration more than 6 Fever
weeks
Age;child or elderly Major motor weakness esp. bilaterally

Fever, malaise, weight loss Saddle anesthesia

IV drug use Perianal sensory loss

Corticostreoid use Urinary retansion

Cancer history (especially Anal sphincter laxity


bone metastasis)
Trauma esp. in elderly Fecal incontinance

Recent instrumentation or
spinal anesthesia

provided by author

The typical back pain from muscles or ligaments is generally


easily localizable, increases with movement and decreases after
rest. The pain rarely radiates, but when it does, it radiates to the
pelvis. It the pain radiates below the knee, it may point to L3
nerve root radiculopathy. However, about %90 of the disc
herniations relate to L4-5, L5-S1 regions (Illustration 12.1). The The duration of pain is important. The musculoskeletal pain
location of pain and nerve root innervation). generally limits itself in 4-6 weeks. Consider malignancy if the
pain lasts longer.

The patients under 18 and over 50 years old are under risk for
non-musculoskeletal pathologies.

470
The systemic symptoms are another tip for non-musculoskeletal
Illustration 12.2
pain. Fever, tremor, night sweating, anorexia, unexplained weight
l o s s a r e s i g n i fic a n t f o r i n f e c t i o n a n d m a l i g n a n c y.
Immunocompromised patients (diabetic, the corticosteroid use, IV
drug use) may not develop a healthy inflammatory response and
accordingly the systemic symptoms; therefore, they may require
further investigations. Consider spine infections in IV drug users.

The prior cancer diagnosis should make the physician consider


spine metastases. Especially breast, lung, thyroid, kidney,
prostate cancers and lymphoma tend to metastasize to the spine.

The physical examination aims to detect the risk factors and


neurologic deficits. Check the vital symptoms at the beginning.
Fever may point to spinal infections, but its sensitivity is low.

Lying flat will decrease the musculoskeletal pain. If the pain


increases with lying, consider nephrolithiasis, spine infection,
abdominal aortic aneurysm. Check all patients for abdominal
tenderness, a palpable pulsatile mass, and murmur.

Assess vertebral tenderness; erythema, increased heat, purulent


lesion over the adjacent skin. Evaluate each vertebral spinous
processes individually. If the pain occurs at 30-35 degree angle, it is considered
significantly positive. The radicular pain worsens with ankle
Apply straight leg raise test (Lasegue’s test) by elevating each leg dorsiflexion and improves with plantar flexion. Straight leg raise
slowly while the patient in supine position. Increased pain or test highly sensitive but not specific. Contralateral (opposite or
reproduced sciatic symptoms during test means positive well-leg) straight leg raise is highly specific but poorly sensitive for
(Illustration 12.2). Straight leg raise test).
471
L4-5 or L5-S1 radiculopathy. In other words, a negative I n fla m m a t o r y m a r k e r s (Eritrosit sedimentation rate or C-
contralateral straight leg raise is useful to exclude disk protrusion reactive protein) are highly sensitive but not specific for epidural
(see videos 1 and 2). abscess or cancer.

Evaluate ankle and the first toe’s dorsiflexion and plantar flexion Lumbosacral anterior-posterior and lateral X-rays are indicated in
for L5-S1 nerve roots. Examine patella and ankle deep tendon case of suspected fractures, especially in patients over 50 years
reflexes. Evaluate bilateral dermatomes and check for saddle old.
anesthesia. Test the sensation of light touch along dermatomes
from L1 to S1. Standard dermatomal charts can be helpful, but
there is variability between individuals, and this test is highly Image 12.1 Normal lumbosacral X-ray. Lateral (left), AP (right)
subjective. In the upper lumbar roots, there is often a significant
overlap. The L4, L5 and S1 nerve roots are the most discrete
levels for testing. Additionally, these are the most often affected
lumbar discs.

The rectal examination is not routinely indicated. However, in case


of bladder or bowel incontinence, it is mandatory. Decreased
rectal tone and the sensorial defect may make the physician
consider epidural compression syndrome.

Emergency Diagnostic Tests and


Interpretation
History and physical examination are essential. Laboratory testing
is generally not useful.

Elevated white blood cell (WBC) may point to the infectious


diseases. However, WBC is high only in the two-thirds of the
patients with a spinal epidural abscess.

472
Image 12.2 Lumbar flattening Image 12.3 Normal spinal CT lateral Image 12.4 L2 compression fracture
view

Bedside ultrasound should be performed


if the patient has urinary retention or
suspected abdominal aortic aneurysm

The computed tomography can be useful dissection.

in fractures or facet joint pathologies.


473
Magnetic Resonance Imaging (MRI)
Image 12.6 L2 compression fracture Image 12.7 Pneumonia secondary to
visualizes abscess, metastatic lesions, spinal abscess
and hematoma. Additionally, patients with
neurological deficits requiring urgent
surgery may necessitate MRI.

Image 12.5 Discopathy

Emergency Treatment
Options
Initial Stabilization
Structured management is essential in
the emergency department. Stabilization
is a priority. A critical abnormality in the
vital signs and clinical may lead to the

474
early intervention in the life-threatening diseases and permanent For children: Ibuprofen; Infants and Children <50 kg: Limited
neurological damage is at stake. After stabilization, pain control data available in infants <6 months: 4 to 10 mg/kg/dose every 6
should be provided. to 8 hours; maximum single dose: 400 mg; maximum daily dose:
40 mg/kg/day. Children ≥12 years: Refer to adult dosing.
Medications
• Pain is the main symptom. Non-steroidal anti-inflammatory Elderly considerations
drugs (NSAIDs) are considered as first-line therapy for acute Consider fractures in elderly patients with relatively minimal
back pain. Ibuprofen has less adverse effects and toxicity. trauma. Additionally, consider non-musculoskeletal causes of
back pain, such as abdominal pathologies, aortic aneurysm or
• Acetaminophen may be another choice.
dissection.
• Opioids analgesics should not be administered more 1-2
Pregnant considerations
weeks.
Back pain is frequent in later pregnancy. The neurological deficit
• The muscle relaxants are another treatment choice. is infrequent. Pain control via analgesics and back strengthening
exercises are recommended. Paracetamol is considered safe in
• The use of steroids is not recommended due to lack of pregnancy and should remain the first-line treatment for pain and
evidence. fever. General Dosing Guidelines: 325 to 650 mg every 4 to 6
hours or 1000 mg every 6 to 8 hours.
• The patients should return to their daily activities after a few
days of bed rest.
Disposition Decisions
Pediatric, Geriatric, Pregnant Patient, and Admission Criteria
Other Considerations • Patients with uncontrolled pains

Pediatric considerations • Patients with progressive neurological deficit


In the pediatric age group, back pain is unusual. Consider the
infectious causes. Ibuprofen is the preferred analgesic in this • Patients with symptoms of cauda equina syndrome
population.
• Patients with infectious, vascular or malignant pathologies

475
Discharge Criteria
Patients with musculoskeletal pain without neurological deficits
may be discharged after pain control

Referral
Patients should be referred to neurosurgical or orthopedic surgery
departments.

Pearls and Pitfalls


• Musculoskeletal causes with no neurologic deficits include
degenerative spine disease, muscular or ligamentous injury and
mostly acute disc pathologies. These patients have normal
neurologic examination but have severe pain.

• If the patient has positive neurologic findings during the


examination, consider sciatica with radiculopathy and the
severe other diagnoses.

• Don’t forget that new neurologic physical findings suggest


severe disease and serious diseases have normal neurologic
examination findings. Emergency physicians must think broadly
and consider nonspinal causes of back pain like an aortic
aneurysm.

References and Further Reading, click here

476
Section 2

Lower Extremity Injuries

pelvis x-ray showed a


by Ayse Ece Akceylan

Hip femoral neck fracture. The


patient was admitted to the
orthopedics ward for surgical
Case Presentation
A 75-year-old male with a repair.
history of osteoporosis Critical Bedside Actions
presented to the emergency and General Approach
department after falling on P re s e r v i n g f u n c t i o n , p re v e n t i n g
infection and assuring perfusion of the
his right side. He complained
limb should be the goals. Proper
of pain on the right hip.  His diagnosis and treatment are essential
vital signs were normal. His for establishing these goals.

right leg was in abduction • Check vital signs


and external rotation and
• Learn mechanism of injury
shorter than the left leg.
• Take medical history
Distal pulses were palpable.
An anteroposterior (AP)

477
• Make an orderly and thorough • Visual inspection and palpation: look for • In anterior dislocations, the limb is
examination t e n d e r n e s s , p a l l o r, e c c h y m o s i s , abducted, externally rotated, and
deformity, abrasions, lacerations, and shortened. 
• Order necessary imaging and labs
open wounds.An open fracture is a
• Check neurovascular status: Femoral
• Noncritical orthopedic injuries should fracture associated with overlying soft
nerve and artery may be injured with
be treated only after more threatening tissue injury, creating communication
anterior hip dislocations. The sciatic
injuries have been addressed. between the fracture site and the skin.
nerve may be injured with a posterior
Even a puncture wound extending to
hip dislocation or a hip fracture. Check
Differential Diagnosis the depth of an underlying fracture is
The patient might have one or more of pinprick sensation, light touch and
considered an open fracture. Open
the following: motor function. Also, check  femoral,
fractures are usually classified by their
popliteal, dorsalis pedis, and posterior
severity, based on the size of the
• Hip fracture tibial pulses.
overlying laceration, the extent of tissue
• Hip dislocation damage, lack of bone coverage, the • Dislocations and fracture-dislocations
kinetic energy of the injuring force, and of the hip are two true orthopedic
• Acetabular fracture evidence or likelihood of significant emergencies. The hip joint possesses
• Neurovascular injury contamination. impressive inherent strength and
stability; therefore, considerable force is
• In a femoral fracture, the limb is
History and Physical required to produce these injuries. It is
shortened and externally rotated.
Examination Hints highly recommended that in the
• Note syst emic illnesses, known • Most hip dislocations are posterior. In presence of this type of injury, patients
metabolic disorders and medications. posterior dislocations, the limb is be managed as major trauma victims.
These may provide clues that lead to adducted, internally rotated, and
uncovering the reason behind what may shortened.  Emergency Diagnostic Tests
seem like a simple trauma. (I.e., a fall and Interpretation
may be the result of a cardiovascular • Anteroposterior (AP) and lateral
event.) radiographs of the hip are usually
478
sufficient to diagnose hip dislocations • Significant pain with weight bearing in angiography or CT Angiography are
and fractures. (See Image 12.8 and the face of normal radiographs should necessary. 
12.9)  raise suspicion for occult fracture,
especially at the femoral neck or E m e r g e n c y Tr e a t m e n t
acetabulum.  Options
Image 12.8 Right hip dislocation • Most femoral and hip fractures need
• If there is a suspicion of fracture but operative repair. Consult an orthopedic
plain radiographs appear negative, surgeon. Meanwhile, immobilize the
computed tomography (CT, See Image extremity to prevent it from further
12.10) or magnetic resonance imaging damage.
(MRI) may be used for diagnosis.
• If a fracture is suggested clinically but
radiographic films appear negative, the
Image 12.10 Left acetabular fracture patient should initially be treated with
immobilization as though a fracture
were present.
Image 12.9 Fracture of the femoral • Patients with a traumatic fracture of the
neck and peritrochanteric fracture
hip or femur may lose about 2 to 3 units
of blood at the fracture site and require
blood transfusions. Therefore, order
blood type and crossmatch for at least 2
units of blood.

• If there is suspicion of vascular injury, • Dislocated hips need to be reduced as


cardiovascular surgery consultation and soon as possible, under procedural
D o p p l e r flo w u l t r a s o u n d , p l a i n sedation and analgesia (see videos 1
and 2).

479
• The sooner a joint is relocated, the • In case of open dislocation/fracture, injuries, in which contamination with
b e t t e r, t o a v o i d n e u r o v a s c u l a r remove gross contaminants from the Clostridium perfringens can be
compromise. Also, delays cause wound and irrigate the injury thoroughly. present.    Early surgical intervention for
swelling and muscle spasm, which Apply saline-soaked sterile gauze, and debridement and irrigation is crucial, so
hinder reduction. Use adequate splint the injured leg. If a significant emergency orthopedic consultation is
analgesia or conscious sedation before deformity is present, immediate indicated. Administer analgesics as
attempting relocation. The emergency reduction before splinting is indicated. necessary (Morphine  sulfate: 2–10 mg
physician sometimes may be unable to Administer tetanus immunoprophylaxis (pediatric dose: 0.05–0.1 mg/kg per
reduce a dislocation. Orthopedic as appropriate (Tetanus booster: 0.5 ml dose IV or equivalent analgesic)).
consultation is necessary in such cases. (Tdap) IM, Tetanus immunoglobulin: 250
• In case of neurovascular injury, surgical
IU IM if not previously immunized
• For hip dislocations, after reduction, the consultation is necessary.
against tetanus). Start the patient on
legs are immobilized in slight abduction
intravenous antibiotics. For injuries with
with a pillow between the knees, and Special Populations
mild to moderate contamination, a first-
the patient should be sent for • Treatment options are mostly the same
generation cephalosporin such as for children, elderly and pregnant
radiographs.  Check neurovascular
cefazolin 1–2 g (pediatric dose: 20 mg/ patients.
status before and after all reductions
kg IM/IV)is usually sufficient.2Heavily
and after administration of
contaminated wounds require the • In a fall, elderly patients may have
immobilization. sustained additional injuries; most
addition of gram-negative bacterial
coverage, typically an aminoglycoside commonly, these injuries involve a
• Withholding Oral Intake: Any patient
who might go under general anesthesia such as gentamicin  1.5–2 mg/kg fracture of a vertebral body or wrist.

or procedural sedation should not be IV(pediatric dose: 2–2.5 mg/kg IV). Cervical spine and intracranial injuries

allowed to eat or drink from the moment Adding either penicillin G  4–5 million U also are considered.

of arrival until the need for, and timing IV (pediatric dose: 50,000 U/kg IV)or, if
• The dislocation reduction methods for
of, such a procedure has been penicillin allergic, clindamycin or
patients with hip arthroplasty are the
ascertained. metronidazole as a third antibiotic is
same as with a native hip.
necessary for farm-or soil-related crush
480
• Fractures involving the physis, the be reduced in the operation room under normal. The x-ray revealed a
cartilaginous epiphyseal plate near the general anesthesia.
comminuted patellar fracture
ends of the long bones of growing
• Hip fractures and hip dislocations (even (Image 12.11). The patient was
children, are called Salter fractures.
if reduced in the ED) need to be
Damage to the epiphyseal plate during admitted to the orthopedics
admitted to the orthopedics ward.
a child’s growth may result in an
ward for surgical repair.
aborted or deformed growth of the limb.

• Children who have sustained trauma at


or near a joint may need comparison
studies of the opposite extremity to
Knee Image 12.11

differentiate fracture lines from normal Case Presentation


epiphyseal plates or ossifying growth A 60-year-old female presented
centers.
to the emergency department
• The elderly are more prone to serious with pain and swelling on her
injury from low-energy mechanisms.
right knee after a fall. Her vital
The elderly are more susceptible to
adverse outcomes following trauma signs were normal, and she did
because of comorbid diseases and not hit any other part of her
physiologic changes that arise with
body. Upon physical
aging.
examination, there was
Disposition Decisions tenderness and deformity on
(admission, discharge,
the right knee. The
referral)
• Hip dislocations that cannot be reduced neurovascular examination was
in the emergency department need to
481
Critical Bedside Actions and • Visual inspection and palpation – of the dorsum of the foot and by

General Approach mentioned above in the topic “hip.” dorsiflexion of the ankle. The posterior
These steps are the same as those tibial nerve may also be injured. This
• See Image 12.12 for open knee
mentioned above in the topic “hip.” manifests with diminished plantar
dislocation.
sensation and plantar flexion of the
Differential Diagnosis foot.
The patient might have one or more of
Image 12.12
the following. • Check vascular status: knee trauma
may cause vascular injury. Check the
• Distal femoral, proximal tibial, proximal popliteal, dorsal pedal and posterior
fibular fracture tibial arteries.

• Knee dislocation • Palpate the extensor mechanism for


tenderness and crepitation: quadriceps
• Ligamentous injury
tendon, patella, patellar tendon, and
• Meniscal injury tibial tubercle.

• Popliteal artery injury • Palpate the joint line (for meniscal or


collateral ligament injuries)
• Peroneal, tibial (less common) nerve
injury • Check the range of motion of the knee

History and Physical • Perform knee stability testing and


meniscal testing (see videos 3 and 4)
Examination Hints
• Note syst emic illnesses, known • Check peripheral nerves: knee trauma, • Comparison with the uninjured knee is
metabolic disorders and medications, especially knee dislocations may cause helpful, especially for assessment of
as mentioned above in the topic “hip.” peroneal nerve injury. Examine the ligamentous laxity.
peroneal nerve by testing the sensation

482
A grossly unstable knee after a traumatic the joint above and the joint below the I n acute knee trauma, the goal of
injury should be assumed to be a injury, not to miss associated injuries. radiography is to rule out frac¬ture.
re d u c e d d i s l o c a t i o n u n t i l p ro v e n Because radiographs are not 100%
The joints above and below a fracture
otherwise. sensitive, knee immobi¬lization and
should generally be imaged for coexisting
orthopedic referral for reevaluation are
(For more information, videos 5 and 6 will fractures.
options. When suspicion for a fracture is
be helpful)
Pre-and-post-reduction radiographs are extremely high, CT or MRI can be used.

Emergency Diagnostic Tests advisable both before and after reduction


If there is suspicion of vascular injury, the
and Interpretation of dislocations and fractures.
same rules apply as mentioned above in
The Ottawa Knee Rule and the Pittsburgh the topic “hip.”
See Image 12.13 for comminuted tibial
Knee Rule are useful for deciding when to
plate fracture involving the knee joint.
order plain radiographs. Both criteria are E m e r g e n c y Tr e a t m e n t
sensitive for fractures, but the Pittsburgh Options
criteria are more specific and can be Image 12.13 Most fractures concerning the knee joint
applied to both children and adults. This need operative repair. Consult an
approach is associated with an orthopedic surgeon. Meanwhile,
approximately 1% chance of a missed immobilize the extremity to prevent
fracture. Therefore patients should be further damage.
reevaluated in the event of persistent or
If a fracture is suggested clinically, but
progressive symptoms.
radiographic films appear negative,
Ottawa Knee Rule video immobilize the limb as though a fracture
is present and consult an orthopedic
If plain radiographs are indicated, obtain
surgeon.
a minimum of an antero¬posterior (AP)
and a lateral view. Remember to examine P a t e l l a r F r a c t u re s : N o n d i s p l a c e d
fractures usually heal with a long leg cast

483
for 4 to 6 weeks. Displaced fractures are ice, non–weight-bearing status, and application and analgesic use and an
treated surgically. referral for an orthopedic follow-up. orthopedics outpatient follow-up.

Patellar Dislocation: After reduction, In case of neurovascular injury, urgent • Give the patient instructions about
immobilize the knee in full extension for 3 surgical consultation is necessary. splint care, crutch use, range-of-motion
to 6 weeks. Ice, elevation, non–weight exercises, weight-bearing status,
Controlling Pain and Swelling: The early
bearing, and analgesia are beneficial in war ning signs for neurovascular
application of cold and elevation are
the acute setting. The patient can be impairment and compartment
effective in minimizing swelling or at least
discharged with a referral for a follow-up syndrome and follow-up.
deterring its progression. Administer
within 2 weeks. Watch this video.
analgesics as necessary. • The patient can begin exercises when
Knee Dislocation: To avoid tissue the pain subsides and can return to full
Withholding Oral Intake: same as
damage, the reduction should be activity when full pain-free motion and
mentioned above in the topic “hip.”
attempted as soon as possible.. After equal strength are attained in both
reduction, immobilize the knee and call limbs.
Special Populations
for an orthopedic consultation. Watch this Treatment options are mostly the same

Ankle
video. for children, elderly and pregnant

For open fractures/dislocations, the same patients.

rules apply as mentioned above in the


Disposition Decisions
topic “hip.”
• All dislocations (even if reduced in the Case Presentation
Meniscal Injuries: Unless the knee is ED) and most fractures need to be A 25-year-old male presented
locked and cannot be extended or flexed admitted to the orthopedics ward.
to the emergency department
(which requires orthopedic consultation),
a patient with a meniscal tear should be
• Ligamentous and meniscal injuries can with right ankle pain after a fall
be sent home, with the advice of
managed with analgesics, immobilization, during a soccer game. His
immobilization, elevation, ice
medial right ankle was swollen,
484
and palpation revealed •Soft tissue injury
Image 12.14 Fracture of fibula and
tenderness on the medial fracture of medial malleolus
History and Physical
ligaments and the medial Examination Hints
malleolus. He could not bear • Note systemic illnesses, known
metabolic disorders and medications,
weight on his right foot. as mentioned above in the topic “hip.”
Anteroposterior and lateral
• Visual inspection and palpation:
ankle x-ray revealed a lateral mentioned above in the topic “hip.”
and medial malleolar fracture • Check neurovascular status.
(see Image 12.14) involving the • Evaluate weight-bearing ability only if
joint. The patient was admitted clinical suspicion of a fracture is low.
Critical Bedside Actions and
to the orthopedics ward for General Approach • The patient with a sprain may complain
of hearing a “snap” or a “pop” at the
elective surgical repair. These steps are the same as those
mentioned above in the topic “hip.” moment of injury. Examine the joint for
abnormal motion.
Differential Diagnosis • Examine the proxi¬mal fibula in all
The patient may have one or more of the
medial ankle injuries. A medial ankle
following.
disruption (deltoid ligament tear or
• Fracture (distal tibia, distal fibula, talus, medial malleolar fracture) can cause
calcaneus) complete tearing of the tibiofibular
syndesmotic ligament and fracture of
• Ankle dislocation the proximal fibula (Maisonneuve
fracture).
• Neurovascular injury

485
• See Images 12.15 and 12.16 for ankle Emergency Diagnostic
Image 12.16
open fracture plus dislocation. Tests and Interpretation
• The blunt ankle trauma evaluated within
48 hours of injury, the Ottawa Ankle
Image 12.15
Rules (OAR) can be used to determine
necessity of x-rays. The OAR does not
apply to the hindfoot or forefoot. Finally,
the OAR is not applicable to intoxicated
patients, patients with head injuries,
multiple injuries, or diminished
sensation related to neurologic deficits.

• Views of the ankle should include AP,


lateral, and mortise views. The mortise
view allows a reasonably good image of
both the mortise and the talar dome.

• See video for ankle examination • OAR video


• Achilles tendon may be ruptured in
• Plain radiography may miss subtle
patients exhibiting posterior ankle pain
ankle fractures. If plain radiography is
after a shortfall or jump onto a slightly
negative, but there is suspicion of a
plantar-flexed foot. The Thompson test
fracture, other imaging modalities or
is used to assess the integrity of this
orthopedic consultation is advisable.
tendon. (see video)

486
Emergency Treatment Special Populations
Options Treatment options are mostly the same
Ankle Dislocations: See video for for children, elderly and pregnant
reduction maneuvers. Reassessment of patients.
the neuro¬vascular status, splint
immobilization, ankle elevation, and post- Disposition Decisions
reduction radiography should follow. • Ankle dislocations and most ankle
fractures should be admitted to the
Watch this video.
orthopedic ward. Consult an orthopedic
Ankle fractures: Displaced intraarticular surgeon.
fractures require surgery.
• Soft tissue injuries can be discharged
Achilles Tendon Rupture: Splint the leg in with the recommendation of ice
plantar flexion; arrange orthopedic follow application, elevation, immobilization,
up as an outpatient. and analgesic use.

Sprains: Application of ice, elevation, and


analgesia are recommended.
Nonsteroidal anti-inflammatory drugs References and Further Reading, click

(NSAIDs) are effective in many patients.3 here

Immobilization of the limb for the first 48


to 72 hours provides protection and
comfort. For complete or nearly complete
ligamentous disruption, orthopedic
consultation is mandatory.

A neurovascular injury requires urgent


surgical evaluation.

487
Section 3

Pelvic Injuries

Introduction Illustration 12.3 Anterior view of


by Sercan Yalcinli Pelvic fractures constitute 1-3% of pelvic bones
pelvic skeletal fractures. In younger
adults, a pelvic fracture is generally
due to high energy pelvic injuries
secondary to motor vehicle injuries,
pedestrian injuries, falls from height,
motorcycle accidents and crush
injuries. In the elderly, it may occur
following a fall from a sitting position.
Pelvic trauma-related mortality is
between 3-20%. It reaches 40-50% in Illustration 12.4 Superior view of
pelvic bones
hemodynamically unstable patients.

The pelvis is composed of three bones:


Two innominate bones and sacrum.
İlium, ischium and pubis bones form
the innominate. (See Illustration 12.3 ,
12.4, 12.5)

488
Illustration 12.5 Posterior view of Illustration 12.6 Pelvic ligaments Illustration 12.8 Acetabulum
pelvic bones anathomy

The posterior sacroiliac, sacrotuberous Illustration 12.7 Pelvic vessels


and sacrospinous ligaments located
between the sacrum and two innominate
bones provide the pelvic stability.
Symphysis pubis supports pelvis frontally.
(See Illustration 12.6)

The pelvis has a complex vasculature. pubis form the anterior part of the
Iliac arteries and main veins are close to acetabulum. (See Illustration 12.8)
both sides of the sacroiliac joints. (See
Illustration 12.7) The thin-walled venous Cauda equina courses through the sacral
structures have limited contraction spinal cord and leaves at the sacral spinal
capabilities. Therefore, patients with Three bones form the acetabulum. The foramina to form the lumbar and sacral
pelvic fractures may have life-threatening ilium forms the upper boundary; ischium plexus. Lumbosacral plexus is the
bleedings. forms the posterior part and ilium and thickest peripheral nerve of the body and

489
is frequently injured in posterior hip Acetabular fractures are often associated
Image 12.19 Iliac and anterior column
dislocation with acetabular fracture. with the femur fracture, hip fracture and fracture of acetabulum
Watch this video for detailed information. dislocations, and knee injuries. Posterior
wall fracture is the most common
Pelvic fractures are divided into three
acetabular injury and is usually
groups:
associated with posterior dislocation of
a) pelvic fractures which cause separation the hip. Posterior hip dislocation is
of pelvic ring, generally associated with sciatic nerve
injury. Watch this video for detailed
b) no separation of pelvic ring with one information. (See image 1, 2, 3 )
bone fracture (see Illustration 12.9) and

c) acetabular fractures. Image 12.17 Posterior acetabular


fracture associated with posterior
dislocation of the right hip Image 12.18 Iliac and anterior column
fracture of acetabulum. 3D
Illustration 12.9 Nondisplacement reconstruction in the CT of of the
fractures of pelvis patient with image 12.19.

490
Several classifications are used to identify A P compression or open book injury is
Illustration 12.10 Lateral
pelvic fractures (see video) compression fractures of pelvis the second most common mechanism
and corresponds to 25% of injuries. An
Young-Burgess Classification classifies
example is a frontal impact of the motor
fractures according to the direction of the
vehicle. (See Illustration 12.11) (See
force that caused the injury and the
image 12.21 and 12.22)
mechanisms of injury . There are four
different fracture models according to this Left to right. Type 1, Type 2, and Type 3. Type
1 includes sacral compression fracture on Illustration 12.11 Anteroposterior
classification system: ipsilateral side. Type 2 is a sacral injury with
disruption of posterior sacroiliac ligaments.
compression fractures of pelvis
• lateral compression (LC), Iliac wing fracture on impaction side can be
seen. Type 3 includes Type 1 and 2 injuries on
impaction side with open book fracture/injury
• anteroposterior compression (AP), on contralateral side.

• vertical scissor (VS) and


Anteroposterior compression fractures of pelvis.
Left to right Type 1, 2, and 3. Type 1: symphysis
• combined mechanisms. Image 12.20 pubis diastasis less than 2.5 cm, ligaments are
stretched (anterior sacroiliac, sacra-tuberous,
Lateral compression is the most common sacrospinous) but intact. Type 2: Symphysis
separated more than 2.5 cm. ligaments (sacra-
mechanism. It corresponds to 50% of the tuberous, sacrospinous) are disrupted.
injuries. An example is a side impact of a Sacroiliac ligaments are intact. Type 3:
Symphysis separated more than 2.5 cm and all
motor vehicle to a pedestrian (see ligaments are disrupted.
Illustration 12.10, image 12.20).

Lateral compression type 3 injury; inferior and


superior pubic rami fractures on impaction side
and contralateral sacroiliac widening

491
The least common is Coexistence of other injuries constitutes 20-25% of injuries.
Image 12.21
Antero-posterior VS as it generates 5%
Tile classification is about the mechanical stability of the pelvis.
compression type 2 of the injuries. Falls
injury with right from heights are Type A – Stable pelvic ring injuries, posterior stability is intact: Avulsion fractures,
femur bone fracture
examples. (See isolated iliac wing fractures, isolated pubic rami fractures, transverse fractures of
Illustration 12.12, sacrum or coccyx.
image 12.23)
Type B – Partially stable pelvic ring injury (incomplete disruption of the posterior
Illustration 12.12 pelvis) rotationally unstable, vertically stable: Open-book fractures, lateral
Vertical compression compression fractures, double rami fractures and posterior injury
injury. Pubis and
sacroiliac joints are
Type C – Unstable pelvic ring injury: Vertical shearing fractures, rotationally and
disrupted.
vertically unstable.

Tile classification system predicts the need for surgical intervention. Young and
Image 12.22 Type 3 Burgess determines the pattern of the fracture and predicts the chance of
injury
associated injuries and mortality risk.

Image 12.23 Left iliac


wing, acetabulum and
inferior pubis fracture
of pelvis

492
Case Presentation Critical Bedside Actions and General
A 38-year-old male presents to the emergency Approach
In multi-trauma patient, start with general trauma care including
department following a motor vehicle accident.
ABC.
The patient has left femoral and hip pain. His
Mechanical stabilization and immobilization of the patient are
vitals are as follows: Blood pressure 100/60
important because they reduce the risk of bleeding and
mmHg, heart rate 108 beats per minute, pulse secondary organ injuries.
oxygen saturation at room air 99%. His physical
Consider other organ injuries, especially with unstable pelvic
examination reveals suprapubic tenderness, fractures (e.g., intraabdominal injuries, gastrointestinal tract
limitation of motion in the left hip joint, pelvic injuries, genitourinary injuries, major vascular injuries, and
neurological injuries)
tenderness and hemorrhage at urethral meatus.
Point-of-care ultrasonography shows no • Check vital signs

intraabdominal free fluid. Plain pelvic radiography • Physical examination


and retrograde urethrography show superior
• Take medical history
pubic ramus fracture, sacral fracture, 3 cm
• Learn mechanism of injury
displacement of the symphysis pubis, left femur
bone fracture and urethral injury, respectively. • Ensure an iv line (except lower extremity)

Computerized tomography confirms • Type and crossmatch


retroperitoneal hemorrhage. The patient is • Order necessary imaging, labs, etc..
brought to the operating room.
• Determine the need for operation and type of pelvic fracture,
stable or unstable (mechanically and hemodynamically).

493
Differential Diagnoses forces (such as falls from heights) may
Image 12.24 Cullen’s Sign.
• Abdominal pain in elderly lead to damage to the ligaments and
pelvic floor that lead to significant
• Blunt abdominal trauma instability in the posterior pelvis.

• Hip dislocation Patients should also be questioned for


bladder tenderness, last urination and
• Hip fracture
defecation time, last oral intake time,
History And Physical medical history, drugs and allergies,
Examination Hints menstruation time and pregnancy status
of females. By Herbert L. Fred, MD and Hendrik A. van Dijk
The mechanism of injury plays a vital role – http://cnx.org/content/m14904/latest/, CC BY
in identifying pelvic fracture risk and 2.0, https://commons.wikimedia.org/w/
Rotation of the iliac wings indicates a index.php?curid=5038484
determining the severity of the fracture.
severe pelvic fracture on inspection.
Low-energy injuries (such as falling on the Image 12.25 Gray Turner sign
Differences in limb length may be
ground) typically lead to a stable injury.
associated with hip injury or unstable or
High-energy injuries (such as motor
displaced hemipelvis. Careful inspection
vehicle accidents) increase the risk of
of the skin and skin folds is important for
unstable pelvic fractures and other organ
the detection of open fractures. Perineal
injuries.
ecchymosis or hematoma, Cullen’s sign,
The direction of the force may give an and Grey Turner sign are late findings and
idea about the type of injury. Antero- may be visible due to retroperitoneal By Herbert L. Fred, MD and Hendrik A. van Dijk
hemorrhage and intraabdominal – http://cnx.org/content/m14942/latest/, CC BY
posterior forces may lead to open book
2.0, https://commons.wikimedia.org/w/
injuries (such as motor vehicle accidents). hemorrhage on the periumbilical or flank index.php?curid=5038419
The pelvic floor generally remains intact section. (Image 12.24 and 12.25)
In conscious patients without distracting
while lateral forces (side impacts) lead to
injury, tenderness on palpation between
injuries of the posterior ligaments. Vertical
the symphysis pubis, sacrum and the
494
sacroiliac joint may be a symptom of of pelvic injury on physical examination,
Image 12.27 Inlet view
pelvic injury. Manipulation of the pelvis severe trauma mechanism, suppressed
should be kept minimum because of the awareness or distracting injury.
increased risk of hemorrhage due to the
The method for evaluation of plain pelvic
break down of clots that formed around
radiography is described in here.
the pelvic fracture. Hence, in an unstable
pelvic fracture, the recurrent physical Sacral fractures and sacroiliac joint
examination is contraindicated. injuries may not be seen on AP view. Inlet
and outlet x-ray views increase sensitivity
Presence of blood in the penile meatus
and specificity for the diagnosis of pelvic
should be noted. A digital rectal
fractures in patients who have posterior
examination should be made to reveal the
pelvic tenderness but normal findings in
anal sphincter tone, prostate position and
anterior evaluation. (see image 12.26,
integrity, presence of mucosal disruption
12.27, and 12.28)
and bleeding caused by bone ends. Also,
a digital vaginal examination should be
done to detect open fractures in females.
Image 12.26 Inlet view Image 12.28 Outlet view
Pulse and motor and sensory
examination of the lower extremities
should be evaluated.

Emergency Diagnostic Tests


and Interpretation
Plain Radiography
ATLS guidelines recommend plain pelvic
radiography for patients who have signs

495
• CT should be used when the clinical Resuscitation
Image 12.29 Outlet view
suspicion is high, but the plain pelvic All critically ill patients should be
radiograph is negative. given oxygen and intravenous
fluids.
• CT identifies secondary injuries in
patients with pelvic fractures on x-ray. Lower extremity veins should not
be preferred as an intravenous
• It is preferred in suspected acetabular
line in patients with severe pelvic
fractures.
fracture because of the risk of
• Contrast-enhanced CT gives useful leak into the retroperitoneal
data for evaluation of soft tissue space.
injuries, vascular injuries, and pelvic
Opioids may be given for pain
hematoma.
control.
Avulsion fractures of the L5 transverse • The presence of arterial bleeding is
process, avulsion fractures of adhesion Antibiotics should be given for
80-90% recognizable with CT.
places of sacrospinous and patients with bowel rupture,
urogenital injury, and an open
sacrotuberous ligaments, avulsion of Emergency Treatment
fracture.
lower lateral lip of the sacrum and vertical Options
sacral fractures that extend to medial part There is no standardized protocol for the Tetanus prophylaxis is applied to
of sacral foraminae may show an treatment of pelvic injuries. Treatment appropriate patients.
unstable pelvic fracture exclusively. options should be based on the
hemodynamic status, the severity of Crystalloid fluids and blood
Computed Tomography trauma, the mechanism of injury, the type products may be required in
• CT is the gold standard for evaluation of patients with a pelvic injury.
of fracture, and concomitant injuries.
pelvic injuries.
Open book injuries, fractures that
cause separation of more than 0.5

496
cm in the pelvic ring, and fracture • Open book injuries get the most benefit Angiographic embolization is
findings that include displacement from bed linen wrapping method. reported to be effective at controlling
at symphysis pubis or obturator arterial bleeding, while external fixation is
• This maneuver may aggravate the
ring may need the blood reported to be effective at controlling
degree of the displacement in lateral
transfusion. venous hemorrhage. However, it is
compression injuries because of the
difficult to determine the origin of the
Hemodynamically unstable internal rotational strain.
hemorrhage whether venous or arterial
patients, due to hemorrhagic
• External fixation and extraperitoneal until angiography is applied.
shock caused by trauma, should
packing may be preferred by the
be treated considering the ATLS Complications
o r t h o p e d i c s u rg e o n a s i n v a s i v e
guidelines. Timely intervention is crucial for
treatment options.
prevention of complications.
Control of Hemorrhage
Treatment choice should be selected Angiographic Embolisation Life-threatening hemorrhage, deformity,
P o s t e r i o r p e l v i c r i n g i n j u r i e s a re
according to the capacity of the health neurological and genitourinary injuries are
associated with the most severe
center with an emergency physician, an complications that should be diagnosed
hemorrhages. The majority of pelvic
orthopedic surgeon and an interventional and treated in pelvic traumas.
bleeding has the venous origin. Arterial
radiologist to take control of pelvic
hemorrhages account for 10-15%. Early Complications
hemorrhage.
Shock and death are associated more • Hemorrhagic shock
Mechanical stabilization
with arterial bleeding.
• The bed linen is wrapped tightly around • Urethral injury
the pelvis as a simple non-invasive Angiography is indicated in patients with
technique. Please see videos 1 and 2. a major pelvic fracture who have resistant • Bladder injury

hypovolemia, although other resources • Vaginal laceration


• Sheets must be wrapped to pass
for bleeding are under control .
through the center of the trochanters
• Rectal injury
instead of iliac crests.

497
• Perineal injury Discharge Criteria
• Hemodynamically stable Type A pelvic fractures with no
• Limbo-Sacral nerve root injury
evidence of other system injuries.
Late complications

• Chronic pain
References and Further Reading, click here
• Sexual dysfunction

• Shortening of extremity

• Malunion or nonunion

Pediatric, Geriatric, Pregnant Patients and


Other Considerations
The risk of hemorrhage is higher in pediatric patients. Child abuse
should be considered. There is an increased risk of uterine
rupture in pregnant patients. Consider deep vein thrombosis
prophylaxis in non-ambulatory geriatric patients with stable pelvic
fractures.

Disposition Decisions
Admission criteria
• Tile type B or C pelvic fractures

• Acetabular fractures

• Pelvic fractures with other system injuries

498
Section 4

Spine Injuries

by Ozge Can Cervical Spine Injuries


Case Presentation
A 19-year-old male presented to the emergency department
with a fall from height. Emergency Medical Services Providers
stabilized him on a backboard and cervical collar. On
examination, his BP: 100/70 and HR: 60 GCS 13. There is
swelling and ecchymosis right side of the neck but no
vertebral step-off sign. His upper and lower extremities motor
function was 2/5. The cranial and cervical computed
tomography showed cervical dislocation and subarachnoid
hemorrhage.

Critical Bedside Actions and General Approach


• Check vital signs

• Check immobilization

499
• Order monitorization and IV line palpate the posterior structure of •Examine the motor and sensory
vertebrae behind the neck (Adam) after function. Motor function is assessed
• Learn mechanism of injury
unfastening the patient’s cervical collar from 0 to 5.
• Examine the vertebra and preventing the reflexive movement
0 – is total paralyzed,
of the head with the other hand. Check
• Examine motor and sensory function the posterior neck for midline sensitivity, 1 – is palpable contraction,
swelling, ecchymosis, step-off sign.
• Check exclusion rules
Examine motor and sensory function. 2 – motion with gravidity,
• Examine other injuries
• If the patient is fully conscious and has 3 – motion against gravidity,
• Order imaging and labs. no posterior midline tenderness, the
4 – motion is present but less power,
emergency physician may remove the
Differential Diagnosis cervical collar. Then rotate the head left 5 – normal power.
• Spinal cord injuries and right, caudal and cephalad slowly
• To assess sensory function, examine
• Cervical spine injuries and check if the patient is feeling any
deltoid muscle for C5, the thumb for
pain.
C6, the middle finger for C7, and the
• Cervical Ligamentous injuries
• Evaluate the exclusion criteria if the little finger for C8.
• Vertebral artery injuries patient is conscious and has no
• Check deep tendon reflexes (biceps,
posterior midline tender ness on
• Torticollis triceps).
examination. Nexus and Canadian C-
• Cervical hematomas, masses spine rules are the main rule-out criteria • Spinal cord injuries may lead to
of a cervical spine injury. neurogenic or spinal shock (See the
History and Physical shock and spinal cord injuries).
Examination Hints For nexus criteria, watch this video.
Hypotension and bradycardia are the
• The examination must start with general main symptoms of shock.
For Canadian C-spine rules, watch this
trauma care. On physical examination,
video
the emergency physician should
500
• Motorcycle accidents, falls from height • Simple wedge fracture generally occurs • Type 2: base of dens fracture
and sports injuries are common causes anteriorly when longitudinal ligament
• Type 3: dens and vertebra fracture
of cervical spine injuries. pulls vertebrae body and ruptures due
to flexion forces. Extension
• Victims may be under effects from drug,
alcohol, and unconsciousness • Flexion teardrop is an unstable fracture • C1 posterior arch fracture an unstable
associated with head trauma. c a u s e d b y fle x i o n f o rc e s . I t i s fracture of the atlas
associated with severe ligamentous
• Inspect for any other injuries such as • Hangman’s fracture is the fracture of
injury, anterior cervical cord syndrome
maxillofacial and head injuries. bilateral C2 pedicles
and quadriplegia.
Unconscious and vitally unstable
patients with a head, abdominal and • Clay Shoveler’s fracture is the stable Vertical compression fracture
thoracic injuries should be considered fracture of the C7 spinous process.
• Jefferson Jefferson fracture is C1 burst
to have a cervical injury.
• Spinal subluxation is characterized by a fracture. It is characterized by widened
• Learn the mechanism of injury. Cervical bone fracture with enlargement of predental space on open mouth
spine injuries are categorized according interspinous and intervertebral space. odontoid X-ray.
to mechanism into flexion, extension,
• Bilateral facet dislocation is associated • A burst fracture is mostly seen in lower
and vertical compression.
with soft tissue, annulus fibrosis and vertebrae. Lateral views show the
Flexion anterior ligament injury. fracture best.

• C1-2 atlantooccipital or atlantoaxial • Simultaneous flexion and rotation Emergency Diagnostic Test
dislocation is caused by displacement forces may produce unilateral facet and Interpretation
of the head anteriorly and posteriorly. It dislocation. C2 dens fracture. • Decide the need for imaging using
is diagnosable by plain radiography. exclusion criteria.
• C2 dens fracture has three types:
Atlantooccipital dislocation is a life-
threatening injury and more frequent in • Type 1: avulsion fracture
children.Flexion
501
• Choose the best test for the patient • Magnetic Resonance Imaging (MRI): injury is excluded by clinical or
according to your examination, findings Spinal Cord Injury without Radiographic radiologic means.
and mechanism of injury. Abnormality (SCIWORA) defines the
• Consider full monitoring. Monitorize the
presence of neurological deficit with no
• Imaging is indicated if pain and midline patient for spinal and neurogenic shock
radiographic or computed tomographic
tenderness, neurologic deficit or or phrenic nerve paralysis.
features of spinal fracture or instability.
intoxication/altered mental status is
Therefore, the presence of neurologic • Apply sedation to prevent self-injury or
present.
deficit necessitates MRI for the other complications in agitated patients
• Computed Tomography (CT): Cervical diagnosis of traumatic myelopathy. secondary to additional injuries or
CT is indicated if the patient is substance effects. (See sedation
unconscious, the physical examination
Emergency Management chapter)
• Trauma surveys should be applied any
is unclear, the neurologic deficit is
c-spine injury patients. See this chapter. • Provide cervical immobilization with in-
present or CT is planned for another
F o l l o w i n g re c o m m e n d a t i o n s a re line stabilization during intubation. (See
injury (especially head maxillofacial
specific to c-spine, not general trauma intubation indications in a trauma
t r a u m a ) . H o w e v e r, c u r r e n t A J R
management. patient).
guideline recommends CT scan in the
presence of any violation of NEXUS or • Immobilize the patient at the first • In-line stabilization (video 1 and 2):
Canadian-C-spine rules. contact. Watch this video Have the assistant stand at the head of
the patient and stabilize the patient’s
• Because you may not have CT scan • Immobilization is the first step of
neck using both hands and prevent
availability in some institutions, knowing management. If not done at the
hyperextension.
how to interpret c-spine x-rays is prehospital setting, immobilize the
important. Please see this chapter. You patient’s neck, place the collar • Intubation with video laryngoscope is
will also see many c-spine injury posteriorly with an assistant and fasten. recommended, if available.
samples in that chapter. Unfasten the collar as soon as possible
to prevent complications after the spinal

502
• Continue immobilization until the The patients with neurologic or spinal The patient reported chest
imaging if the patient needs an shock should be admitted to the ICU. The
pain. Blood Pressure: 130/80
emergent operation. patients with unstable fractures should be
admitted and/or operated immediately by mmHg. Heart Rate:120 bpm.
Medications neurosurgery. His Glasgow Coma Scale was
Corticosteroid treatment for spinal cord
injuries secondary to spine injuries has Discharge 15. There was no midline
been shown that having many flaws, Patients with stable fractures and no tenderness on cervical
therefore is not recommended anymore. neurologic deficits may be discharged.
examination. The respiratory
Recommend Philadelphia or Miami collar
Analgesics should be applied to awake
to the patients with suspected ligament
sound was normal. His E-FAST
patient.
injury. examination showed pleural
Pediatric, Geriatric, Refer the patients to neurosurgery clinic.
fluid. The chest x-ray revealed
Pregnant Patıents and Other hemothorax. Thorax CT

Thoracic
Considerations showed thoracic vertebrae
Some diseases may predispose a person
to cervical injury. Rheumatoid arthritis spinous process fracture.
may cause C2 transverse ligament
rupture. Atlantooccipital dislocation is
Spine Critical Bedside Actions and
General Approach
Injuries
seen with Down syndrome
Please refer to cervical spinal injuries
Patients with long-term corticosteroid use section.
or osteoporosis are predisposed to
Case Presentation
fractures. Differential Diagnosis
A 40-year-old male presented • Spinal cord injuries
Disposition Decision to the emergency department
• Thoracic Ligamentous injuries
Admission
after a motorcycle accident.
503
• Vertebral artery injuries • A scapular injury is an indicator of high •Flexion-rotation injuries: occur with a
energy trauma. In case of scapular posterior ligament injury.
• Rib fracture
injury, consider a thoracic spine injury.
• Shear injuries: Posterior anterior, lateral
• Pneumothorax
• Remember that spinal cord damage listezis occur with ligament injury.
• Scapula fracture may lead to spinal and neurological
shock. Emergency Diagnostic Test
History and Physical and Interpretation
• Learn the mechanism: Thoracic Patients who have vertebrae pain, midline
Examination Hints
vertebra injuries are classified as sensitivity, bone deformity, neurologic
• Thoracic injuries mostly occur with
flexion, extension, rotation, shear, deficit, more than 60 years old and high-
high-energy mechanisms, namely,
distraction and axial compression energy mechanism requires imaging.
motorcycle accidents, fall from height
injuries according to their mechanisms.
and gunshot injuries.
Anteroposterior (AP) and Lateral X-rays:
• Flexion injuries: occur with anterior AP images show lateral pedicles. Lateral
• First, immobilize the patient if he is not
compression. Instability is associated images show subluxations, compression
on a backboard. A vacuum splint or
with the posterior ligament injury. fractures (Image 1) and chance fractures.
scoop stretchers is useful. Logroll
Wedge fracture is an example. It is the
(video) the patient for examination. For
most common fracture in the thoracic Computed Tomography (CT): Patients
logrolling, a leader and three assistants
spine. with a neurologic deficit or altered mental
should be available.
status require CT.
• Extension injuries: are anterior ligament,
• Examine the patient’s vertebrae for
facet, laminar, spinous process injuries. Magnetic Resonance Imaging (MRI):
m i d l i n e s e n s i t i v i t y, s w e l l i n g ,
Patients with the suspected ligament,
ecchymosis, step-off sign. Additionally, • Axial compression injuries: are burst disk or epidural space injuries require
check the motor and sensory function fractures and occur with high-energy MRI.
and deep tendon reflexes. mechanisms.

504
Emergency Management Disposition Decision a history of corticosteroid
• Check immobilization
Admission usage. Vital signs are normal.
• Remove backboard if there are no signs Patients with spinal and neurogenic On her examination, she had
of injury. Prefer a sliding board instead shock symptoms should be admitted to
pain on her back at the level of
of backboard when prolonged the intensive care unit.
immobilization needed (i.e. the risk of
lumbar 2-3. The lateral X-ray
injury continues). Discharge showed an L2 compression
Patients with stable fractures and no
• Provide full monitoring, especially in neurologic deficits may be discharged
fracture.
patients with spinal or neurogenic after the consultation with neurosurgery
shock or phrenic nerve paralysis. or orthopedic department. Referral to
Critical Bedside Actions and
these clinics should also be planned.
General Approach
• Flexion restriction braces (Jewett or Please refer to spinal injuries section.
Knight-Taylor) is recommended if there
is no stable angle fracture.
Lumbar Differential Diagnosis
• Spinal cord injuries

Spinal
Medications
• Lumbar spine injuries
Please refer to cervical spinal injuries
section. • Spinal epidural hematoma

Pediatric, Geriatric, Injuries • Paraspinal hematoma


Pregnant Patıents and Other
Considerations Case Presentation • Retroperitoneal hematoma

Long-term corticosteroid use, A 70-year-old woman arrived in • Renal injuries


malignancies or osteoporosis is the emergency department
predisposing factors for fractures. • Soft tissue injuries
after a fall from stairs. She had
• İntraabdominal trauma
505
• Pelvic injuries fractures may be associated with
Image 12.31 Sagittal CT view of L2
abdominal injuries. compression fracture.
History and Physical
Examination Hints • Transverse process fracture: is the most
Examination findings and history are common lumbar spine fracture. An x-
similar to those of thoracic injuries. ray may miss most of the transverse
process fractures. The abdominal or
Lumbar vertebrae fracture is the most pelvic injury may accompany.
frequent of all and may happen after
relatively minor trauma. Emergency Diagnostic Test
and Interpretation
Examine hip flexion, leg extension, ankle
Please refer to thoracic spine injury
dorsiflexion, and toe extension to assess
section.
motor function.

Evaluate rectal tonus by a rectal exam in Image 12.30 L1 burst fracture on


lateral lumbar vertebrae view
patients with suspected lumbar injuries.

Lumbar fracture types:

• Wedge fracture: Isolated anterior


column fracture

• Burst fracture: Fracture of the anterior


and middle column

• Flexion-distraction: injuries are most


frequently seen in seat belt injuries.
Fractures at T12-L1 junction is called
chance fracture. Flexion-distraction
506
Image 12.32 Sagittal MRI view of L2 Emergency Management b e discharged. Patients with a simple
compression fracture and extending to Please refer to cervical injuries section. wedge fracture and no neurologic deficit
spinal canal. may be discharged with pain control.
Medications Discharge decisions should be taken with
Please refer to the cervical thoracic neurosurgery and/or orthopedic
injuries section. consultations. Referral to these clinics
should also be planned.
Pediatric, Geriatric,
Pregnant Patients and Other
Considerations
References and Further Reading, click
A lumbar fracture is rare in infants.
here
Consider child abuse, especially in
fractures with paralysis.

Consider epidural hematoma in geriatric


patients on anticoagulation

Disposition Decision
Admission
Admit patients with shock and
intraabdominal organ injuries to intensive
care unit.

Discharge
Patients with a simple transverse sacral
fracture, isolated spinous fracture or
isolated transverse process fracture may

507
Section 5

Upper extremity injuries

by  Meltem Songur Kodik Injuries of Shoulder


Case Presentation
A 50-year-old male was brought to the emergency department
after he fell onto his outstretched arm while working in a
construction pit. He complained of severe pain, instability and
weakness on his right shoulder. His vital signs were normal.
On the physical examination, the arm was in abduction and
external rotation. Normal contour of the deltoid and acromion
was lost. The patient was leaning away from the injured side
and cannot adduct or internally rotate the shoulder even
slightly without severe pain. His humeral head was palpable
anteriorly, and all movements were limited and painful.
Additionally, there was palpable fullness below the coracoid
process, towards the axilla. The neurovascular examination
was normal. Initial anteroposterior (AP) radiogram showed an

508
anterior glenohumeral Differential Diagnosis Image 12.34 Clavicle fracture
Fractures: Proximal end of the humerus
dislocation. The dislocation
(the most frequently injured bone of the
was reduced in the emergency shoulder) [Image 12.33] , clavicle (80 %
room. being middle-third fractures) [Image
12.34], scapula [Image 12.35].
Critical Bedside and General
Approach Image 12.33 Proximal humerus
Preserving function, preventing infection fracture Image 12.35 Scapular fracture
and assuring perfusion of the limb should
be the goals. Proper diagnosis and
treatment are essential for establishing
these goals.

• Check vital signs

• Learn mechanism of injury

• Take medical history

• Make an orderly and thorough


examination

• Order necessary imaging and labs.

• Noncritical orthopedic injuries should


be treated only after more threatening Dislocations: Glenohumeral (mostly
injuries have been addressed. anterior, rarely posterior, inferior and

509
s u p e r i o r ) [ F i g u re 4 ] , f o l l o w e d b y Shoulder instability may be subtle or usually misreferred to as cardiac,
acromioclavicular, sternoclavicular. obvious subluxation or dislocation. biliary or abdominal pathology.

• The young athletes usually sustain Emergency Diagnostic Tests


Image 12.36 Anterior humeral head
dislocation anterior glenohumeral dislocation and Interpretation
during athletic activities with rapid For traumatic injuries, AP (45-degree
movements. The avulsion of the lateral), transscapular lateral (“Y” view)
anteroinferior glenohumeral ligament is [Image 12.37], and axillary lateral views
f r e q u e n t . C h a r a c t e r i s t i c a l l y, are helpful. The true AP view provides a
capsulolabral detachment occurs. glenohumeral joint image without bony
(Bankart’s lesion) In the elderly, a fall overshadow, so it is preferable over the
onto the outstretched arm is a more standard AP view. The axillary lateral view
common mechanism of injury. is beneficial to evaluate the fractures of
the glenoid fossa and the acromion or
• With an anterior glenohumeral
coracoid process.
dislocation, the patients hold their arms
in slight abduction with external rotation The CT views are especially useful in
Soft tissue injuries: Impingement
with the help of their opposite hand. determining scapular injuries. 3D
syndrome (Subacromial tendinitis and
The patients with an acute reconstruction from chest CT provides
bursitis with rotator cuff tears, tendinitis
acromioclavicular sprain usually have a requested soft tissue data details as well.
of the long head of the biceps tendon)
step-off at the joint and hold their arms
to their side. Bedside ultrasonography is particularly
History and Physical
useful in clavicular fractures.
Examination Hints • Patients seeking medical care for their
• Combination of pain, instability,
shoulder symptoms usually have a
stiffness and loss of power are the main
previous shoulder pathology. For
complaints. The complaints usually
patients with an obscure history of
start after trauma with acute pain.
shoulder trauma, the patients’ pain is
510
with time. Therefore, the reduction of dose: 40 mg/kg/day. Children ≥12
Image 12.37 ransscapular lateral (“Y”
view) dislocation should be performed years: Refer to adult dosing.
immediately. Before performing the
• For pregnant patients: Paracetamol is
reduction, the type of dislocation and
still considered safe in pregnancy and
any accompanying bone fractures
should remain the first-line treatment for
should be shown radiographically.
pain and fever. Dose: 325 to 650 mg
2. M e d i c a t i o n s : F o r a l l re d u c t i o n every 4 to 6 hours or 1000 mg every 6
techniques, adequate sedation/ to 8 hours.
analgesia or anesthesia is required. For
• For elderly: patients Acetaminophen
many reductions, sedation with IV
325 to 650 mg every 4 to 6 hours or
fentanyl (50 to 100 mcg) and IV
1000 mg every 6 to 8 hours.
midazolam (1 to 3 mg) is sufficient.
Deep sedation with propofol or 4. Procedures:
etomidate is required for some
patients. Rarely, general anesthesia Shoulder Dislocations
may be necessary. Injection of a local
Glenohumeral. Various techniques such
anesthetic intraarticularly is another
as traction, leverage, or scapular
good alternative and facilitates the
manipulation can be used in combination
reduction.
for reduction. Some methods such as
3. Analgesia: Hippocrates and Kocher maneuvers must
Emergency Treatment not be used because of their high
Options • For children: Ibuprofen; Infants and incidence of complications (axillary nerve
1. Initial Stabilization: Please refer to Children <50 kg: Limited data available injury, humeral shaft and neck fractures,
“Critical Bedside And General in infants <6 months: 4 to 10 mg/kg/ and capsular damage). Watch video for
Approach” part. The incidence of dose every 6 to 8 hours; maximum one of the simple techniques.
neurovascular complications increases single dose: 400 mg; maximum daily
511
One of the most frequently used Pediatric, Geriatric, features of this method provide no
adequate leverage technique is Pregnant Patient, and Other adverse pregnancy outcomes.
Leidelmeyer’s external rotation method.
Considerations
While the patient is in supine position, the Disposition Decisions
arm is gently adducted and the elbow is Pediatric considerations
Admission Criteria
flexed to 90 degrees and gentle external In the pediatric age group, closed
If the closed reduction of shoulder
rotation is applied until the reduction reduction is similar to the adults. Under
dislocation fails, or when the patients
provided. The rate of success of this 10 years of age, shoulder dislocations are
have a neurovascular compromise, the
method is 80-90 %. There are other rare. Because of concomitant physeal
patient may need admission for reduction
techniques such as Cunningham and ( i . e . , g ro w t h p l a t e ) f r a c t u re s , a n
under general anesthesia or open
scapular rotation techniques which orthopedic consultation is recommended
reduction.
requires almost no analgesia or sedation before reduction.
with high success rate. Discharge Criteria
Elderly considerations After the reduction is accomplished, the
Fractures I n t h e e l d e r l y, a n t e r i o r s h o u l d e r
shoulder is immobilized with velpeau
dislocation is more common; of those,
bandage and the patient is discharged.
• Clavicle: After closed reduction of the 20% suffer redislocation and 60% have a
fracture, clavicular (figure-of-eight cuff tear. To stabilize the shoulder the torn The patient is referred to an orthopedic
splint) splint should be applied. cuff should be repaired; patients with surgeon within one week.
multiple redislocations usually require
• Scapula: To support the ipsilateral The patient should be informed if there is
both procedures.
upper extremity, a sling immobilization redislocation.
is used.
Pregnant considerations
Procedural sedation in pregnant women Referral
• Proximal Humerus: A sling and swathe Accompanying fractures of shoulder
provides relief from significant pain,
device is necessary for immobilization.
dislocations are Hill Sachs deformities
distress or fear, and exposure to the
(cortical depression in the humeral head
medications used in procedural sedation
created by the glenoid rim during
is short and are relatively low. Such
512
dislocation), Bankart lesions and fractures elbow appeared angulated and Critical Bedside and
o f g r e a t e r t u b e r o s i t y. I n s u c h General Approach
the upper extremity shortened.
circumstances, orthopedic referral is These steps are the same as those
required. The neurovascular examination mentioned above in the topic “shoulder.”
was normal. Radiogram,

Injuries of including anteroposterior (AP)


and lateral films, revealed a
Differential Diagnosis
Fractures

Elbow supracondylar extension


fracture. The prominence of the
Fractures of the distal humerus:

Supracondylar fractures [Image 12.38];


Case Presentation olecranon attached to the
Extension type (mostly) and Flexion type

An 8-year-old boy fell onto his


posteriorly displaced distal • Type 1: Minimal or no displacement
left outstretched arm with a
fragment is similar to that seen • Type 2: Slightly displaced
hyperextended elbow, while he
with posterior dislocation of the
• Type 3: Totally displaced
was riding his bike to school.
elbow. The fracture was stable
When he arrived at the ED, his Transcondylar, intercondylar, lateral
after reduction and immobilized condyle, and medial condyle fractures
left elbow was tender and
with a splint; elbow in a flexed
swollen, and he was unable to Articular surface fractures: Capitellum,
position. Trochlea, Epicondyle, Little Leaguer’ s
move the forearm. His vital
Elbow, Olecranon
signs were normal. On physical
Radial head and neck fractures
examination, he was holding
the upper extremity immobile in Dislocation and Subluxation:

extension to the side. The


513
Elbow: Posterior (Mostly) [Image 12.39], Very few fractures or dislocations
Image 12.39 Elbow dislocation
medial, lateral, anterior (Rarely) occur by direct blows.

Radial head (nursemaid’ s elbow) Numbness or weakness distal to the


injury should indicate the possibility of
Soft tissue disorders: Epicondylitis
neurovascular injury.
(Tennis Elbow), Olecranon bursitis, biceps
tendon rupture. Extension type supracondylar fracture
has a characteristic S-shaped
Image 12.38 Supracondylar fracture. configuration of the arm held at the side.
Type 2 O n t h e o t h e r s i t e , fle x i o n t y p e
supracondylar fractures leads the
supporting of the forearm with the
opposite hand where the elbow is flexed
at 90°. When the prominence of
olecranon is increased, it suggests
posterior dislocation of the elbow or
extension supracondylar fracture.
A n t e r i o r d i s l o c a t i o n o r fle x i o n
supracondylar fracture leads to the loss
of normal olecranon prominence.
History and Physical
When the prominence of olecranon is
Examination Hints increased, it suggests posterior
Indirect trauma transmitted through the
dislocation of the elbow or extension
bones of forearm causes many of the
supracondylar fracture. Anterior
elbow injuries. (e.g., FOOSH= Fall on
dislocation or flexion supracondylar
outstretched hand )

514
fracture leads to the loss of regular Emergency Diagnostic Tests epiphyses and ossification centers.
olecranon prominence. and Interpretation To identify fractures, physeal injuries and
Plain radiographs should be obtained in dislocations ultrasound is a useful tool.
Gradual onset of dull ache at the elbow
three views of plain x-rays – AP, lateral, Children with lateral condyle fractures
happens with epicondylitis. This pain
and lateral oblique are necessary. AP may benefit from an MRI.
increases with grasping and twisting of
view shows the epicondyles (medial and
the elbow. Emergency Treatment
lateral) and the articular surfaces.
With the elbow in a 90° flexion, the radial (radiocapitellar and ulnotrochlear) The Options
head, tip of the olecranon and the lateral lateral view provides the relation of the Initial Stabilization
epicondyle normally form an equilateral bones of the distal humerus and proximal Before taking radiographs, to prevent
triangle. Fracture of the radial head, forearm. The radiocapitellar joint, medial further injury, immobilization should be
olecranon or the lateral epicondyle alters epicondyle, radioulnar joint, and coronoid performed. Please refer to “Critical
this relationship. process view with the lateral oblique. Bedside And General Approach” part in
the topic “shoulder.”
A complete neurovascular examination Fat pad sign: With a history of known or
should be made for the elbow and distal suspected trauma of the elbow, if there is
Medications
extremity. This examination should an abnormal fat pad sign this should be Please refer to the topic “shoulder.’’
include: sensation and strength tests of considered as an indication of an occult
t h e m e d i a n , r a d i a l , u l n a r, a n d fracture. Wide anterior fat pad, also, Procedures
musculocutaneous nerves, deep tendon known as ‘’sail sign’’ indicates an occult Supracondylar fractures:
reflexes of the biceps (C5), brachioradialis fracture. Posterior fat pad sign in an adult
• Nondisplaced fractures of children do
(C6), and triceps (C7), palpation of the indicates radial head fracture; in children
not require immediate orthopedic
brachial, radial, and ulnar pulses, the indicates supracondylar fracture. Watch
evaluation. These patients may be
range of motion and strength of the this video.
referred for follow up within a week
elbow should be examined.
X-rays of the uninjured elbow help after splinting. The family should be
distinguish fractures from the normal informed to return if an unmanageable

515
p a i n o r c o m p a r t m e n t s y n d ro m e supination/flexion and hyperpronation. olecranon, external (lateral)
happens. The latter is more successful for the epicondyle (mnemonic: CRITOE).
reduction and may be less painful.
• Displaced supracondylar fractures Disposition Decisions
generally require open or closed Epicondylitis: Once a clinical diagnosis of
reduction and percutaneous pinning for epicondylitis is made, the initial treatment Admission Criteria
monitoring of pulses, never function consists of activity modification, counter • Open fractures accompanying vascular

and forearm compartments; the patient f o rc e b r a c i n g , n o n s t e ro i d a l a n t i - injuries

should be hospitalized. The patient is inflammatory drugs (NSAIDs) (if not


• When an operative reduction or internal
discharged with a splint or cast with contraindicated) and physical therapy. fixation is indicated
percutaneous pins. Severe cases can be splinted.
• All the patients with gross swelling and
Transcondylar, intercondylar, condylar, Pediatric, Geriatric, ecchymosis should be hospitalized for
epicondylar fractures, articular surface Pregnant Patient, and Other observation to monitor and decrease
fractures: orthopedic consultation is
Considerations the risk for compartment syndrome.
necessary for all these.
Pediatric Discharge Criteria
Elbow dislocations: to avoid vascular In children younger than ten years of age, • Reduced dislocations and stable
damage, reduce immediately, flex to 90° posterior elbow dislocation is the most fractures with none of the above
and place a posterior splint. common. Due to the different stages of features
ossification and predominance of
Radial head fracture: for all fractures for
c a r t i l a g e , t h e r e i s d i ffic u l t y i n • Splinted patients with an arrangement
24 to 48 hours a sling or posterior splint
interpretation of pediatric elbow X-rays. In of orthopedic follow up in 24-48 hour
and cold application, analgesic etc.
the pediatric elbow, the six ossification
applied, and the patient should be • Simple soft tissue injuries
centers are shown numbered in the order
referred to an orthopedic surgeon.
of appearance: Capitellum, radial head, Referral
Radial head subluxation: for reduction of internal (medial) epicondyle, trochlea,
RHS, there are two techniques –
516
For a close follow-up, all fractures radius fracture, called Colles’ Image 12.40 Colles’ Fracture
discharged from ED should be referred to
fracture is evident. The
an orthopedic surgeon.
emergency physician

Injuries of performed an early anatomic


reduction with the restoration

Wrist of radial length and correction


of the dorsal angulation to a
Case Presentation neutral position.
A 63-year-old woman fell on
her outstretched hand while Differential Diagnosis
Distal radius and ulna injuries:
she was replacing a bulb at
• Colles’ Fracture (the most frequent):
home. She presented to the ED
Transverse fracture of the distal radial • Hutchinson’s Fracture (Chauffeur’s
with tenderness, severe pain, metaphysis, which is dorsally displaced fracture): Intra-articular fracture of the
swelling, numbness, and and angulated. [Image 12.40] radial styloid.

coldness in the region of the • Smith’s Fracture (Reverse Colles’ • Distal radioulnar joint disruption
wrist. On physical examination, Fracture): Transverse fracture of the
• Pediatric fractures of the distal radius:
distal radial metaphysis with associated
she had swelling and ‘’dinner- Torus [Image 12.41], Greenstick,
volar displacement and angulation.
fork’’ deformity. The complete fractures [Image 12.42]
• Barton’s Fracture: Oblique intra-
neurovascular examination was
articular fracture of the rim of the distal
normal. On the PA and lateral radius with associated displacement of
views of the wrist, a distal the distal radial fragment.
517
Image 12.41 Torus fracture Image 12.42 Complete shaft fracture Image 12.43 Scaphoid fracture
of radius and ulna

Carpal injuries:

Fractures: Scaphoid [Image 12.43],


lunate, triquetral, pisiform, hamate,
trapezium, capitate, trapezoid

Clinical features: Carpal instability,


midcarpal and intercalated segment
instability, radiocarpal instability

Soft tissue injuries: Carpal tunnel


syndrome (due to distal radius fractures
and repetitive strain), de Quervain’ s
Disease and intersection syndrome
518
History and Physical The examination should include Distal radius and ulna fractures:

Examination Hints assessment of neurovascular status


• Colles’ and Smith’s fractures: Closed
Falling on an Out-Stretched Hand motor and sensory function of the
reduction and immobilization in a sugar-
(FOOSH) is the most common median, radial and ulnar nerves. Because
tong splint. Because the Smith’ s
mechanism of wrist injuries, with the wrist acute median nerve compression is a
fracture, unlike Colles’ fracture, is more
in extension. common occurrence, the sensation of
unstable, it requires operative
thumb and index fingers is important,
Immature, weaker epiphyseal plate or intervention and splint reduction.
especially with severely displaced
metaphysis of the radius in children are fractures. In all injuries to the wrist, radial • Barton’ s Fracture: Since those
more likely to sustain injuries, sparing the and ulnar pulses should be evaluated. fractures do not benefit close reduction,
still-cartilaginous carpal bones.
they should be referred to an
Emergency Diagnostic Tests orthopedic surgeon immediately.
Young adults with active lifestyles are
and Interpretation
more likely to be injured with greater
Standard views of the wrist include • Hutchinson’ s Fracture: If the fracture is
forces.
posteroanterior (PA), lateral, and oblique non-displaced, a sugar-tong splint
In the elderly, especially in women with views. MRI and CT provide the diagnosis immobilization is sufficient. In such
some degree of osteoporosis, distal radial of the radiographically occult fractures. cases, a short arm cast is a definitive
metaphysis is more fragile resulting in treatment. On the other hand, displaced
Colles fracture.
Emergency Treatment fractures accompanied with a
Options scapholunate ligament injury, open or
“Anatomic snuffbox’’ on the dorsum of Initial Stabilization: Please refer to closed reduction and fixation is
the wrist is an important landmark. “Critical Bedside And General Approach” necessary. Since many ligaments of the
Because the scaphoid is palpable with its part in the topic “shoulder.” wrist are attached to the styloid
triangle by styloid, extensor pollicis brevis process, it should be reduced
Medications: Please refer to the topic
tendon and the extensor pollicis longus accurately for wrist function.
“shoulder.’’
tendon. Tenderness in this area may
indicate a scaphoid fracture. Procedures:

519
Distal radioulnar joint disruption: A long Pediatric, Geriatric, •F r a c t u r e s w i t h c o m p a r t m e n t
arm cast is applied after closed reduction Pregnant Patient, and Other syndrome or neurovascular
compromise
Pediatric fractures of the distal radius: A Considerations
short arm splint is required for • Fractures needing immediate operative
Pediatric
immobilization. The most common fractures in children management or general anesthesia for
and adolescents are the distal radial reduction
Carpal injuries: A short arm splint with a
fractures (44). Commonly growth plate
thumb spica is required for • Fractures associated with soft tissue
injuries occur with the distal radius
immobilization. complications
fractures leading to physeal injuries (45).
Carpal tunnel syndrome: The wrist is The most important aspect of these • Fractures associated with circulatory
splinted in a neutral position and physeal fractures is premature closure deterioration in the hand
cortisone injections are given additionally and growth arrest of the injury site.
into the carpal tunnel. Discharge Criteria
Geriatric • Appropriate reduction and
De Quervain’s Disease: for mild and Distal radius fractures in older patients immobilization
moderate forms conservative should be screened for osteoporosis.
• Orthopedic follow-up should be
measurements such as rest of the arm in Also, distal radius fractures in an senior
arranged
elevated position, splinting, NSAIDs and man is an early and sensitive marker of
corticosteroid injections into dorsal skeletal fragility (47). Subsequent • Pain control measures should be taken
extensor compartment of the wrist may fractures are prevalent and treatment of adequately
be sufficient. underlying osteoporosis in this population
group is required. • Cast or splint care instructions should
be given at discharge and should be
Disposition Decisions assured the patient understands them.

Admission Criteria
• Open fractures
520
• After ED treatment, the documentation of intact neurovascular
function is performed.

Referral
For a close follow-up, all fractures discharged from ED should be
referred to an orthopedic surgeon.

References and Further Reading, click here

521
Chapter 13

Selected
Infectious
Problems
Section 1

Epiglottitis

Case Presentation
by Kuan Win Sen A 62-year-old man presents to the ambulatory area of the
emergency department complaining of sore throat, fever, and
chills. He has history of type 2 diabetes mellitus, hypertension,
and obesity. He was seen by the general practitioner (GP) 2
days prior and was prescribed thymol gargle and paracetamol.
Further history reveals progressive difficulty and pain in
swallowing, decrease in appetite, and worsening sore throat
since the GP visit two days ago. On physical examination, he
is alert, has a temperature of 39.1C, heart rate of 112 per
minute, blood pressure of 136/74mmHg, respiratory rate of 18
per minute and oxygen saturation of 98% on room air. He
speaks with a muffled voice and has drooling of saliva. There
are no obvious findings in the oropharynx and no cervical
lymphadenopathy. The rest of the physical examination is
unremarkable.

523
Critical Bedside Actions and • Laryngotracheitis (croup) Patients with croup have a “barking”

General Approach cough and are generally comfortable in


Every patient who presents to the ED will
History and Physical the supine position (watch the video).
receive initial bedside evaluation (ABC…), Examination Hints
and vital signs check. Depending on the
Have a high index of suspicion for Emergency Diagnostic Tests
history and general look of the patient,
epiglottitis in patients reattending ED for and Interpretation
worsening sore throat. Severe sore throat Obtain a lateral neck X-ray (“thumb” sign)
our first priority is to evaluate some
with odynophagia, high fever, muffled
symptoms and signs of impending airway
voice (“hot potato voice”) and drooling Image 13.1
obstruction. This evaluation can be done
a re c o m m o n c l i n i c a l f e a t u re s o f
in the triage or inside the ED. If there are
epiglottitis. Patients with impending
findings, the patient should be triaged to
airway compromise would refuse to lie
high acuity area. Airway equipment and
down for examination. Suspect
monitoring devices should be placed to
epiglottitis if the severity of sore throat is
the patient. Depending on the patient’s
out of proportion to findings in the
initial evaluation, any abnormality found in
oropharyngeal examination.
ABC evaluation should be treated
immediately. However, suspicion of • Assess the soft palate for swelling and/
epiglottitis may change some or deviation of the uvula to exclude
management strategies such as involving peritonsillar abscess (quinsy)
airway specialists earlier.
• Assess the sublingual and
Differential Diagnoses submandibular regions for swelling to
• Peritonsillar abscess exclude Ludwig’s angina Typical findings of epiglottitis with enlarged
epiglottis and aryepiglottic folds.
• Diphtheria • Assess the tonsils for grey
Case courtesy of Dr Maxime St-Amant,
pseudomembranes that are classically Radiopaedia.org. From the case rID: 26840
• Submandibular space infection
seen in diphtheria
(Ludwig’s angina)
524
Prepare for laryngoscopic visualization Administer antibiotics (intravenous Individualize examination according
using a flexible nasolaryngoscopy (gold ceftriaxone 2g and intravenous to the severity of illness
standard). Watch the video. clindamycin 600mg)
• those with classic symptoms of
Diagnostic laboratory investigations Other options such as Racemic epiglottitis or stridor should have
(complete blood count and blood culture) epinephrine, eta-agonists, and involvement of pediatric airway experts
should not be performed in patients corticosteroids have not been shown to before attempts at visualization as there
where suspicion of epiglottitis is high until be beneficial in epiglottitis treatment. have been reports of cardiorespiratory
the airway is secured. arrest in children during these attempts
Pediatric Considerations
Emergency Treatment Epiglottitis used to be more common in Disposition Decisions
Options children with incomplete or lack of Hib Intubated patients or those who require
Airway maintenance is paramount immunization. However, the adult to child very close monitoring should be admitted
ratio has increased since the turn of the to the surgical intensive care unit or high
• Provide humidified supplemental
century. dependency unit.
oxygen
Typical findings in children:
• Consider prophylactic intubation to
secure the airway in progressive • Look toxic References and Further Reading, click
disease here
• Reluctant to lie down
• Prepare the surgical airway set for the
• Classical drooling, dysphagia, distress
possibility of encountering a difficult
(3D’s)
airway
• Tripod position
• Avoid supraglottic devices as they may
compress the swollen epiglottis • Sniffing position

525
Section 2

Meningitis

Case Presentation
by Alja Parežnik A 55-year-old previously healthy woman presented with fever,
headache, vomiting, and photophobia for three days. One
week earlier, she started to complain about a sore throat and
pain in the right ear. Neurological examination revealed
diminished consciousness and neck rigidity. Lumbar puncture
was performed and in CSF found >10.000 leukocytes/mm3.
Direct examination of CSF showed Gram-positive cocci in
chains and culture yielded S. pyogenes. The patient had
treated with Ceftriaxone (4 gr/day).

526
Introduction • through stool (enteroviruses), common pathogen since routine
Meningitis is an inflammation of the immunization of infants with H. influenzae
• through coughing and sneezing,
membranes of the brain and spinal cord. type B began in 1992. Table 13.1
It can be related to infectious and • through kissing, sexual contact or presents the most common bacteria and
noninfectious causes. The infection agent contact with infected blood, their specification. M. tuberculosis,
is usually bacteria or virus, and S.aureus, Borrelia burgdorferi and gram
• from eating a specific food (Listeria negative bacilli are among the rare
occasionally fungus. Additionally;
monocytogenes), causes.
physical injury, autoimmune disorders,
cancer or certain drugs can cause • from rodents and insects (leptospirosis Viral meningitis is much more common
meningitis . by mice, hamsters, rats and West Nile than bacterial. It tends to be less severe
virus through mosquito bites). and usually recovers completely without
Pathogenesis
Bacteria can breach the blood-brain specific therapy. Most common viral
Etiology
barrier (BBB) to infect the meninges by pathogens causing meningitis are;
The severity of illness and the treatment
direct spread, or contiguous infection differ depending on the cause. • Enteroviruses (Coxsackie, echoviruses)
(from a source such as the sinuses or
middle ear), trauma, neurosurgery, or Bacterial meningitis is a life-threatening • Arboviruses (KME, West Nile),
indwelling medical devices. neurological and infectious emergency. It
can lead to death within hours. Bacterial • Herpes viruses (HSV-1,2, VZV, EBV,
Nasopharyngeal colonization from
meningitis can lead to long-term CMV)
infected droplets of respiratory secretions
or distant localized infection (lungs, urine) problems, like hearing loss, vision loss,
• Others (mumps, HIV, parvovirus,
with subsequent bloodstream invasion, problems with memory and rotavirus, etc)
are other sources of infection. concentration, epilepsy, coordination,
movement and balance problems, Fungal meningitis is rare form and
Pathogens causing meningitis can be learning difficulties and behavioral generally occurs only in
spread in different ways: problems. In community-acquired immunocompromised people.
meningitis, S. pneumoniae is the most
• during birth from mother to her baby,
527
•malaria,
Table 13.1 Main common bacterial pathogens in meningitis and their specifications
NEISSERIA STREPTOCOCCUS HAEMOPHILUS LISTERIA • cancer of meninges,
MENINGITIDIS PNEUMONIAE INFLUENZE MONOCYTOGENES

Age Children, adults Children, adults Adults, not older, newborns, • vasculitis of CNS.
(living in crowded vaccinated pregnant women,
spaces) children immunocompromised
History and Physical
Vaccine yes yes yes
Examination Hints
Associated Sore throat Ear infection, Sinusitis The classic triad with fever, neck
diseases sinusitis,
pneumonia stiffness and altered mental status is
Characteristics Petechial rash, Rash, Rash present in only 44% of cases. However,
muscle pain and Neurological the absence of all of the triad almost
weakness changes (seizures,
focal) eliminates the possibility of meningitis.

Original by author More than 95% of patients have two out


of four criteria: a headache, fever, neck
Critical Bedside Actions and Differential Diagnoses stiffness and altered mental status.
General Approach • Encephalitis, Some patients may show dislike of bright
Stabilization of an unstable patient is the lights, rash, sleepy or difficulty to wake,
• meningitis, or seizure. Babies may present with
priority. Check the airway and breathing
(respiratory rate, saturation) and give • septicemia, refusing to feed, irritable, high pitched or
oxygen if needed. Check circulation moaning cry, stiff body with jerking
(pulse, capillary refill time, urine output, • brain abscess, movements.
blood pressure) and give fluids or
• subdural empyema, Some people are at the highest risk for
medications if needed. Then, check
developing meningitis. Risky age groups
disability (Glasgow coma scale or AVPU • subarachnoid bleeding,
are children under age 5, teenagers from
(alert, voice, pain, unresponsive), focal
• tetanus, 16-25 years and adults over the age of
neurological signs, seizures,
55. Certain medical conditions, such as
papilledema, glucose).
528
damaged or absent spleen, chronic • Altered mental status: 69% B ) Brudzinski’s sign is neck stiffness
disease or immune system disorders causes a patient’s hips and knees to flex
• Headache: 87%
increase the risk. Traveling to areas where when the neck is flexed. Watch the video.
meningitis is common is another risk • Vomiting 35%
In sum, ask about typical clinical signs:
factor.
• Focal neurological exam: 23-33% fever, headache, photophobia, stiff neck,
Search for meningismus sign using one vomiting, and neurological changes.
or more of the following tests. In jolt • Seizures 15-30% Previous diseases, associated diseases,
accentuation test, the patient rotates his family history, vaccination, allergy, social
• Kernig’s sign 9%
head horizontally at a frequency of two to status, medications and traveling to other
three rotations per second. The test is • Brudzinski’ sign 1% countries are also important. Pay
positive if there is the exacerbation of an attention to age, immunocompromised
• Jolt accentuation of headache 100%
existing headache. The absence of jolt state, living in dorms, traveling to states
accentuation has a specificity of 97% for Abnormal neurologic exam, photophobia, with common meningitis infection,
ruling out meningitis. Kernig’s sign is the and lethargy are among the other related contacts with infected people.
inability to straighten the leg when the hip signs. Petechial rash, muscle aches, and
In the physical exam, we look for the
is flexed to 90 degrees. Brudzinski’s sign weakness are characteristic for
meningeal signs with different tests (jolt
is positive when forced flexion of the meningococcal meningitis.
accentuation of headache, Kerning’s sign,
neck elicits a reflex flexion of the hips.
Tests for confirming physical meningeal Brudzinski’s sign) and neurological
Both Kernig and Brudzinski have reported
signs: changes (AVPU, GCS).
low sensitivity (5%) but high specificity
(95%). Most common meningeal
A) Kernig’s sign is an failure to straighten Emergency Diagnostic Tests
symptoms and their sensitivity are
the leg when the hip is flexed to 90 and Interpretation
presented below.
degrees because of the stiffness of the Routine blood work is often obtained but
• Fever >38 °C: 75-85% hamstrings. Watch the video. frequently unrevealing. Blood cultures
should be drawn in all patients.
• Stiff neck: 70-83%
529
T h e g o l d s t a n d a rd o f c o n fir m i n g hours. In meningococcal infections, it is neurologic deficit. CSF analysis is
meningitis is an analysis of cerebral- only 1 hour. important, table 13.2 below shows the
spinal fluid (CSF) obtained by lumbar main charactheristics of CSF in different
Computer tomography (CT) has to be
puncture (LP). It should be obtained in all types of meningitis.
performed before lumbar puncture (LP) in
patients with suspected meningitis unless
order to exclude increased intracranial
contraindicated. The first results are gram
pressure (ICP) or alternate mass lesion
stain and latex agglutination tests. The
when the patient has any of these criteria:
culture of CSF later confirms the
immunocompromised state, history of
diagnosis. The combination of all three
CNS disease, new-onset seizure,
mentioned tests is proved to be more
papilledema, abnormal level of
productive than any of the single test
consciousness (GCS<12) or focal
alone. Specific findings (white and red
blood cell counts, glucose and protein
levels) in CSF helps us to differentiate Table 13.2 Common CSF findings in different types of meningitis
between the types of meningitis.
INDEX NORMAL BACTERIAL VIRAL FUNGAL

It is essential to perform LP as soon as WBC (white blood cells) <5 >1000 <1000 <1000
possible. In some cases, LP is delayed Differential (neutrophils) <15% >80% <15% <15%
due to imaging, limited resources, signs CSF glucose (mg/dL) 45-65 reduced normal reduces
of severe sepsis or rapidly evolving rash, CSF protein (mg/dL) 20-45 >250 50-250 >250
severe respiratory or cardiac compromise
Opening pressure <20 Normal to high, Normal to high
and significant bleeding risk. It is prudent Normal to high
(cmH20) typically 15-30
to give empiric antibiotic therapy first. In Gram stain +
pneumococcal meningitis, an PCR +
approximate window to perform an LP
Quattromani EN, Aldeen AZ. Focus on: emergent evaluation and management of bacterial meningitis.
after antibiotic administration is 4-10 American college of emergency physicians news. 2008 May[updated 2014]. https://www.acep.org/
Clinical---Practice-Management/Focus-On--Emergent-Evaluation-and-Management-of-Bacterial-
Meningitis/. Accessed April 18, 2016.
530
Emergency Treatment • Penicillin/Cephalosporin anaphylaxis: Dexamethasone, 0,15 mg/kg (max.

Options Chloramphenicol 25mg/kg IV 10 mg) IV every 6 hours for 4 days in


adults and 0,4 mg/kg IV every 12 hours
Initial stabilization Below list has recommendations for
for 2 days for children. There is no
Obtain venous access, give oxygen and therapy after pathogen identification by
evidence of mortality benefits but
fluids and resuscitate if necessary. positive Gram stain. Multiple studies have
prevention of hearing loss and long-term
shown an association between time of
neurologic squeals.
Medications antibiotic administration and poor
Antibiotics are the cornerstone of outcome, is greater above 4-6 hours. As we are treating meningitis in the
treatment of bacterial meningitis. emergency department, it is reasonable
Administer empiric antibiotic therapy as • S. pneumonie = Ceftriaxon +
to give a dose of dexamethasone with the
soon as possible, before LP. Vancomycin
first round of antibiotics for the agent is
• N. meningitidis = Ceftriaxone unknown.
Empirical antibiotic therapy
options; • Listeria monocytogenes = Ampicillin or Primary treatment of viral
• Ceftriaxon/Ceftazidime 4g IV daily meningoencephalitis is symptomatic. In
penicillin G
immediately (child 100mg/kg up to 4g) meningitis caused by HSV-1 or 2, or
• S. agalacticae = Ampicillin or penicillin severe EBV and VZV Acyclovir is added in
• + Vancomycin 12,5mg/kg up to 500mg
G therapy. Seriously ill patients should
IV 6 hours if S.pneumonie or S.aureus is
receive Acyclovir IV 15-30 mg/kg per day
suspected, recent travel, risk of • H. influenzae = Cefriaxone
in 3 divided doses, which can be followed
resistance
While controversial, adjunctive steroids by an oral dose, 800 mg, five times daily
• + Ampicillin 2,4g IV 4hours if are recommended in all suspected cases for the total course of 7-14 days. Patients
immunosuppressed, if Listeria of bacterial meningitis especially with who are less ill can be treated with oral
monocytogenes (adults >50years) is Streptococcus pneumoniae  meningitis drug alone.
suspected (child 60mg/kg to 2,4g) and should be given before or with the
fi r s t d o s e o f a n t i b i o t i c . G i v e

531
Prevention and prophylaxis Pediatric, Geriatric and and has a fever, has bacterial meningitis
Patients hospitalized with suspected  N. Pregnant Patients until proven otherwise.
meningitidis  infection or meningitis of An atypical presentation is common in
Laboratory findings in blood and CSF can
uncertain etiology require droplet elderly (>65 years) as lethargy, the
be normal in extreme ages.
precautions for the first 24 hours of absence of fever and minimal signs of
treatment or until  N. meningitidis  can be meningismus. Older adults  and people Disposition Decisions
ruled out. with additional medical conditions may Admission criteria: If there is clinical
only present with a slight  headache and suspicion of meningitis, patients should
Those who came in close contacts with
fever or general weakness. be admitted for further workup and
an infected person, especially with N.
treatment.
meningitidis or H. influenzae, give Neonates, infants and young children
Rifampin (600mg/12h oral for 2 days; usually show poor feeding, irritability, and ICU Referral: Patients with signs of shock
children >1year 10 mg/kg/12h, <1year 5 fever. In babies,  a fever, irritability, or septicemia must be admitted to
mg/kg/12h) or Ciprofloxacin (500mg oral, decreased appetite, rash, vomiting, and a Intensive unit care (ICU). These signs
1 dose 250mg for child 5-12 years). In shrill cry may point to meningitis. Other include capillary refill time more than 4
pregnancy give a single dose of signs include stiff body and bulging soft seconds, unusual skin colour or rapidly
Ceftriaxone 250mg IM or ciprofloxacin spots on the head that aren’t caused by progressive rash, systolic hypotension
500 mg oral. crying. Babies with meningitis may cry <90mmHg, pulse rate <40 or >140/min,
when handled. respiratory rate <8 or >30/min, acidosis
Procedures pH < 7,3 or base excess more negative
If there is suspicion of bacterial Young children  with meningitis may have
than -5, white blood count < 4×109/L,
meningitis, the emergency physician flu-like symptoms,  cough or respiratory
lactate > 4mmol/L, GCS < 12 or a drop of
should perform tasks in the following distress. In children, history of respiratory
2 points, moribund state, altered mental
order: blood cultures, steroids, tract infection is common, and they are
state/decreased conscious level, poor
antibiotics, CT and LP also more likely than adults to experience
urine output, poor response to initial fluid
a seizure. When a child is looking sick
resuscitation.

532
References and Further Reading, click here

533
Section 3

Sinusitis

Case Presentation
by Katja Žalman and Gregor Prosen The 32-year-old married woman presented with nasal
stuffiness with yellow nasal drainage, pain over the cheek,
obstructed nose, facial pain and pressure, subjective fever
and chills, mildly productive cough and overall malaise for ten
days. She has used over-the-counter medication without
significant benefit. She smoke three packs of cigarettes per
week. She takes no medications and denies chronic medical
diseases.

She is conscious (GCS 15), alert and oriented. BP 125/78, HR


77/min, RR 19 breaths/min, body temperature 38 °C. HEENT
exam shows intact external ocular muscles, pupils are equal,
round and reactive to light, red swollen nasal mucosa with
thick yellow-green discharge. No polyps noted. Right maxillary
and frontal sinuses are tender to palpation. Mild erythema
noted on posterior oropharynx. External ear canals have no

534
edema or erythema. The tympanic membranes Introduction
Sinusitis is one of the most common infections treated by
are neither bulging or retracted; the ear
emergency physicians and affects about 1 in 8 adults in the north
landmarks are easily identifiable. The neck is America. It is the fifth most common diagnosis for which
supple without lymphadenopathy. The chest is antibiotics are prescribed.

clear to auscultation bilaterally. Sinusitis is an inflammation of the paranasal sinuses (frontal,


maxillary, ethmoid and sphenoid).

A healthy sinus is sterile and lined with a thin layer of mucus that
traps dust, germs and other particles in the air. Tiny, hair-like
projections in the sinuses seep the mucus towards ostial opening
that leads to the back of the throat, and then they slide down to
the stomach.

Sinusitis rarely occurs without concurrent rhinitis and


inflammation of the contiguous, nasal mucosa is simultaneously
involved, and therefore the preferred term for this condition is
rhinosinusitis.

Rhinosinusitis is classified according to the new guidelines (2015)


by duration as;

• acute rhinosinusitis (ARS) – when the illness appears less than 4


week

• chronic rhinosinusitis (CRS) when lasting more than 12 weeks,


with or without acute exacerbations. Chronic rhinosinusitis
should be confirmed as the clinical diagnosis. Objective

535
documentation of sinonasal from tumors, abnormal anatomy, weaker with acute rhinosinusitis is to
inflammation may be achieved using immune system, nasal polyps and also eradicate infection, decrease severity and
anterior rhinoscopy, nasal endoscopy or nasogastric and nasotracheal intubation. duration of symptoms and prevent
computed tomography. The primary pathogens responsible for complications.
acute bacterial and recurrent ARS are
The different subgroups of acute Differential Diagnosis
Streptococcus pneumonia, non-typable
rhinosinusitis are based on the duration The diagnosis of rhinosinusitis consists of
H. influenza, and M. catarrhalis.
of symptoms and signs, into acute the combination of clinical history,
bacterial rhinosinusitis (ABRS) or viral In chronic sinusitis, however, anaerobic physical examination, imaging studies,
rhinosinusitis (VRS). Four or more bacteria, streptococcal species, S. aureus and laboratory tests.
episodes of rhinosinusitis per year, and also fungi (Rhizopus, Aspergillus,
without persistent symptoms in between, Candida, Histoplasma, Blastomyces, Conditions that predispose to
the state is termed as recurrent ARS. Coccidioides, and Cryptococcus species) rhinosinusitis are
play role. • Allergic and nonallergic rhinitis
The acute rhinosinusitis is most
frequently (90%) associated with viral • Anatomic abnormality of the
Critical Bedside Actions and
upper respiratory tract infection. It is the ostiomeatal complex
General Approach
most important risk factor for the First, check conditions and vital signs of • Aspirin sensitivity
development of acute bacterial sinusitis the patient, and stabilize them if
and it is most often caused by rhinovirus, necessary. The most of the patients are • Associated conditions: asthma, otitis
c o r o n a v i r u s , i n flu e n z a A a n d B , rarely need any intervention during the media
parainfluenza, respiratory syncytial virus, primary evaluation (ABC) stage. When the
• Churg Strauss sydrome
adenovirus, and enterovirus. patient is stable, we can continue with
taking the history and physical exam, list • Cilliary dyskinesia,
The most common occlusions that leads
of differential diagnoses, general
to bacterial overgrowth and excess • Cocaine abuse
diagnostic and appropriate treatments.
mucus production are allergies, trauma
The acute care of a patient diagnosed • Cystic fibrosis
and fractures, mechanical obstruction
536
• GERD • Concha bullosa and other middle •signs and symptoms (major and
turbinate abnormalities minor)
• Immune diseases and
immunocompromised status • Infectious rhinitis (viral upper tract • questions on allergic symptoms
infections) (sneezing, watery rhinorrhea, nasal
• Instumentation (nasogastric and
itching and itchy watery), asthma and
nasotracheal intubation) • Nonallergic rhinitis (vasomotor rhinitis,
immunocompromising disorders
aspirin tolerance, eosinophilic
• Kartagener syndrome,
nonallergic rhinitis • history of previous episodes of
• Nasal anatomic variants rhinosinusitis
• Rhinitis medicamentosa
• Nasal polyps (decongestants, β – blockers, birth • history or possibility of trauma,
control pills, antihypertensives) fractures, nasal anatomic variants and
• Rhinitis medicamentosa anatomic abnormality of the
• Rhinitis secondary to: pregnancy,
ostiomeatal complex
• Trauma hypothyroidsm, horner sindrom, weger
granulomatosis – midline granuloma • active or passive smoking
• Tumors
• Tumors • current medications
• Young syndrome
• Vascular headache (migraine) Symptoms associated with rhinosinusitis
Differential diagnosis of
are divided into major and minor groups.
rhinosinusitis History and Physical Combinations of these symptoms provide
• Allergic rhinitis (seasonal, perennial)
Examination Hints a diagnosis based on the patient’s
• Anatomic abnormalities (foreign body, Before we start the focused physical history, viewed by anterior rhinoscopy, or
nasal polyps, nasal septal deviation, exam, which is based on an examination as a postnasal discharge on pharyngeal
enlarged tonsils and adenoids) of the respiratory system, we have to take examination.
a look at a patient’s history and have to
• Cerebral spinal fluid rhinorrhea be especially focused on: Major Symptoms

537
• Facial pain/pressure/fullness should distinguish between viral •When symptoms or signs (PODS) –
rhinosinusitis (VRS) and bacterial ABRS. It the presence of ≥ 2 PODS symptoms,
• Fever (for acute sinusitis only)
can be difficult to distinguish between one of which must be O or D of ARS
• Hyposmia/anosmia acute viral from acute bacterial sinusitis. lasting for more than 10 days but less
than 30 days without any evidence of
• Nasal obstruction/blockage P – Facial pain, pressure or fullness (may
clinical improvement.
involve the anterior face, periorbital
• Nasal or postnasal discharge/purulence region or manifest with headache) • Onset with severe symptoms or signs of
high fever (≥39°C [102°F]) and purulent
Minor Symptoms O – Nasal obstruction (congestion, (infected, colored or oozing) nasal
• Cough blockage, stuffiness) discharge or facial pain lasting for at
• Dental pain D – Nasal purulence or discolored least 3–4 consecutive days at the
beginning of the illness.
• Ear pain/pressure/fullness postnasal discharge (infected, colored,
oozing) • Onset with worsening symptoms or
• Fatigue
signs characterized by the new fever
S – Hyposmia or anosmia (smell)
• Fever (for subacute or chronic sinusitis) (fever is present in some patients with
Acute rhinosinusitis typically progresses VRS in the first few days of illness but
• Halitosis over a period of 7 to 10 days; it is mostly does not predict bacterial infection as
self-limited and resolves spontaneously. an isolated diagnostic criterion – it has
• Headaches
a sensitivity and specificity of only
During a viral upper respiratory tract
Acute rhinosinusitis is diagnosed when a about 50% for ABRS), headache, dental
i n f e c t i o n , t h re e c o m m o n c l i n i c a l
patient presents with up to 4 weeks of pain, or increase in nasal discharge,
presentations should guide the clinician
purulent nasal drainage, nasal following a typical viral upper
to think that it is an episode of acute
obstruction, facial pain-pressure-fullness, respiratory infection (URI) that lasted 5–
bacterial sinusitis: persistent  symptoms,
or all of these symptoms. When a patient 6 days and were initially improving
s e v e r e  s y m p t o m s , o r w o r s e n i n g
meets the criteria for ARS, the clinician (“double-worsening”).
symptoms.
538
Chronic rhinosinusitis is diagnosed when deemed unreliable. Endoscopy provides Ultrasound is safe, rapid and
a patient presented greater than 12 ideal direct visualization of the nasal noninvasive for evaluating only the
weeks of anterior or posterior cavity, and anatomical structures such as maxillary and frontal sinuses. The A-
mucopurulent drainage, nasal Eustachian tube orifice, tonsils, posterior mode may be useful for screening the
obstruction, facial-pain-pressure-fullness tongue, epiglottis, glottis, and vocal fluid in the maxillary sinus, and the B-
and decreased the sense of smell. cords. The nasal polyps can be identified, mode detecting fluid in the cavity,
as well as the presence of purulent ostial mucosal thickening, or soft tissue mass in
Invasive fungal sinusitis usually occurs in
s e c re t i o n s . E n d o s c o p y i s u s u a l l y the maxillary sinus.
immunocompromised patients and
performed by otolaryngologists.
patients with diabetes. It is generally X-rays is not recommended for patients
Therefore, emergency physicians should
associated with fever, nasal pain, cloudy who have already met the clinical
chose the patients who needs proper
rhinorrhea, and affected turbinates by diagnostic criteria for ABRS. Radiography
referral.
dark, thick and greasy material. cannot be used to distinguish between
Emergency Diagnostic Tests bacterial and viral etiologies.
The anterior rhinoscopic examination is
best performed after the application of a and Interpretation
X-rays includes 3 different
topical decongestant. The status of the Imaging projections:
nasal mucosa, the presence and color of In the majority of patients with • Waters view (occipitofrontal) – for
nasal discharge should be evaluated. rhinosinusitis, radiographic imaging is maxillary and frontal sinuses
Predisposing anatomical variations can unnecessary in case of meeting
• Caldwell view (angled posteroanterior) –
also be noted during anterior rhinoscopy. diagnostic criteria for acute rhinosinusitis.
only that visualizes the ethmoid air cells
Imaging procedures are useful when
The endoscopic examination should be
symptoms are vague, in poor response to • Lateral view – visualize the sphenoid
used in selected patients with chronic or
initial management, comorbidities that sinus and primary for adenoids in
recurrent sinusitis, in the patient with
predispose complications, atypical children
rhinosinusitis who do not respond to
presentation and a history of trauma.
therapy as expected, and in younger
children in whom a medical history is
539
Radiographic findings of acute CT is not used for routine evaluation and 50% of patients with a recent upper
sinusitis are; is limited to chronic and recurrent respiratory infection have abnormal
sinusitis, causes of questionable findings on CT scan. On the CT with
Image 13.2 diagnose, patients with unresponsive
disease, immunocompromised patients Image 13.4
with fever, dentomaxillary pain or
investigation of complications (severe
headache, facial swelling, cranial nerve

Image 13.3

X-ray – The Waters view shows air fluid level on


the right maxillary sinus, and loss of air on the
left maxillary sinus. Air presence is also
decreased in ethmoidal sinuses. Case courtesy of Dr Bruno Di Muzio,
Radiopaedia.org. From the case rID: 31870

• Diffuse opacification,
acute rhinosinusitis, we can find
• Mucosal thickening (>4 mm), or an air- opacification, air-fluid level, and severe
fluid level. mucosal thickening.
Case courtesy of A.Prof Frank Gaillard,
• Mild-to-moderate mucosal thickening, Radiopaedia.org. From the case rID: 4890 MRI is not used for routine evaluation.
however, is a nonspecific finding. MRI is a sensitive technique for
palsies, or forward displacement or
evaluating suspected fungal sinusitis and
bulging of the eye – proptosis). More than

540
for differentiating between inflammatory Emergency Treatment The clinician must also consider the
disease and malignancy. Options p a t i e n t ’s a g e , g e n e r a l h e a l t h ,
cardiopulmonary status, and comorbid
Laboratory Tests Viral rhinosinusitis (VRS) conditions when assessing suitability for
Complete blood cell count (CVC) is treatment watchful waiting.
g e n e r a l l y n o t s p e c i fic , a n d i t i s Viral rhinosinusitis is a self-limited disease
unnecessary for the majority of patients that occurs from 2 to 5 times per year in When we decide to treat ABRS with an
with uncomplicated rhinosinusitis. In the average adult. Decongestant therapy antibiotic, the commonly used drug for
most cases, the results show that the such as topical steroids, topical and/or children and adults is amoxicillin (with or
CBC may be within normal ranges. oral decongestants, which can not be without clavulanate as first-line therapy).
used more than 3 to 5 day, mucolytics, A period from 5 to 10 days regimen of
Higher level of erythrocyte sedimentation
and intranasal saline spray. They may be amoxicillin 500 mg, 2 times a day is
and C-reactive protein level can be seen
used alone or in varying combinations. recommended by many as the first-line
in patients. Both of them are not specific.
Analgesics or antipyretic drugs therapy. The acute sinusitis generally
Nasal cytology can be useful with variety (acetaminophen, ibuprofen, or other responds to treatment from 10 to 14
of syndromes, including allergic rhinitis, nonsteroidal anti-inflammatory agents) days. Some physicians continue
bacterial sinusitis, eosinophilia, nasal may be given for pain and fever. treatment for 7 days after the patient is
polyposos, and aspirin sensitivity. well to ensure complete eradication of the
Bacterial rhinosinusitis (BRS) organism and prevent relapse.
The culture of secretions from the nasal treatment
cavity or nasopharynx do not differentiate Delaying antibiotic treatment of ABRS for For patients who do not respond to
ABRS from VRS and are not routinely up to 7 days after diagnosis is the current amoxicillin, allergic to or intolerant of
obtained unless in immunocompromised, approach. This allows the infection get amoxicillin, live in communities with a
intensive care patients and patients with better on its own. If not, prescribe initial high incidence of resistant organisms,
complications of rhinosinusitis. antibiotic therapy for adults with failure to respond within 48-72 hours,
uncomplicated ABRS. persistence of symptoms beyond 10-14
day the second-line therapy is the most

541
commonly used, which include cephalosporins, macrolides or
quinolones.
References and Further Reading, click here
Adjunct therapy such as intranasal saline irrigations, intranasal
corticosteroids and local topic decongestants (oxymetazoline
hydrochloride) is recommended. Topical agents should be used
for up to 5 days; as extended use results in rebound vasodilation
and nasal obstruction, the condition is termed as “rhinitis
medicamentosa.” Antihistamines are not recommended as
adjunct therapy unless there are patients with a history of allergic
rhinosinusitis.

In patients with complications of acute rhinosinusitis high-dose


intravenous antibiotics, including cefuroxime, ceftriaxone, or
ampicillin-sulbactam is recommended.

Pediatric, Geriatric, Pregnant Patient, and


Other Considerations
The described procedures can also be used in the pediatric,
geriatric and pregnant patient with rhinosinusitis.

Disposition Decisions
Patients with uncomplicated rhinosinusitis can be discharged
home with prescription for decongestant therapy, nonsteroidal
anti-inflammatory drugs and in the case of ABRH with
appropriate antibiotics. All other patients with complications
require additional work-up or admission.

542
Section 4

Sepsis

Case Presentation
by Emilie J. Calvello Hynes 74 y/o female with history of diabetes, hypertension and
coronary stent placement presents with confusion and cough.
She has had a cough for 2 days and saw her primary care
doctor who prescribed an antibiotic. Her husband describes
her as behaving normally until today 3 hours prior to
presentation. She is taking Insulin, Lisinopril, Aspirin,
Metoprolol, and Azithromycin. There are no allergies to
medications.

On exam, the patient is spontaneously breathing with eyes


closed but opens to verbal command. She is lethargic and
oriented only to self. Blood pressure is 92/48mmHg, heart rate
122/min, respiratory rate 24 cycles/min, temperature is 37.5
°C and oxygen saturation is 89% on room air. She has dry
mucous membranes, and her skin is cool and diaphoretic. The
lung exam reveals crepitations in the left base and heart

543
sounds are regular but Introduction and Definitions t oprompt the right treatment
In the last 20 years, the collective interventions.
tachycardic. Her abdomen is
understanding of sepsis care has gone
flat and non-tender, and her P r i o r d e fin i t i o n s o f s e p s i s w e r e
through a major transformation. The term
predicated on the inflammatory response
neurological exam reveals no sepsis describes a physiologic syndrome
from the host, termed the systemic
focal deficits. with characteristic biochemical
inflammatory response syndrome (SIRS).
abnormalities initiated by infection. While
the mortality from sepsis in many high- Systemic Inflammatory
income countries is decreasing, the Response Syndrome
reported incidence has found to be
increasing due to aging populations as
well as greater attention paid to early Two or more of the following

recognition of a potentially deadly


• Heart rate > 90 beats/min
syndrome. The true incidence of sepsis
worldwide is unknown, and there is no • Respiratory rate > 20 cycles/min or
doubt, even with the most conservative PaCO2 <32 mm Hg
estimates, that sepsis is a leading cause
• Temperature > 38°C or < 36°C
of critical illness and death on the planet.
• WBC > 12,000/mm3, < 6,000/mm3 or >
Sepsis recognition and appropriate care
10% bandemia
have a huge possible impact on human
mortality, and yet the public health SIRS can be caused by a large variety of
awareness of sepsis is quite poor. Sepsis inciting agents that induce host
can have many presentations making it inflammatory response, such as burns,
sometimes difficult for even the most pancreatitis, a variety of infectious
experienced physicians to detect, further organisms and toxins. Sepsis, for the last
emphasizing the need for clear definitions two decades, has been understood to be
544
SIRS plus an infectious source. Severe
Table 13.3 Comparison of Sepsis Definition Evolution
sepsis was defined as sepsis plus organ
dysfunction. Whereas, septic shock was
OLD DEFINITION (PRE 2016) CURRENT DEFINITION
defined as hypotension induced by
sepsis that persisted despite adequate Sepsis = SIRS + infectious source Sepsis = life-threatening organ dysfunction due to
a dysregulated host response to infection
fluid resuscitation. However, a recent task • ED patients: 2 or more of the following qSOFA
force has fundamentally shifted the score may identify patients with increased
mortality
previously understood definitions of • SBP less than or equal to 100 mm Hg
sepsis in the following important ways. • RR ≥ 22
• Altered mental status (GCS <15)
(Table 13.3)

SIRS criteria is not adequately sensitive


Severe sepsis = sepsis + organ system The term severe sepsis is NO LONGER USED.
or specific in identifying those who may dysfunction
go on to have significant morbidity and
Septic shock Septic Shock = subset of patients with sepsis
mortality from overwhelming infection. Adults = persistent arterial hypotension despite and profound circulatory, cellular, and metabolic
adequate fluid resuscitation abnormalities
Rather, sepsis is now defined as life- Pediatrics* = tachycardia with decreased signs of
threatening organ dysfunction due to a perfusion Clinical Criteria
Despite adequate volume resuscitation,
dysregulated host response to infection. Persistent hypotension requiring vasopressors to
Organ dysfunction is best defined in the maintain MAP ≥ 65 mm Hg
Lactate ≥ 2 mmol/L
undifferentiated emergency department
patient by the quickSOFA score (qSOFA).
[SOFA =  Sepsis related Organ Failure
Original by the author
Assessment Score]
Read for new definitions from - Rubulotta FM, et al. Crit Care Med. 2009;37(1): 167-170.

Identification of these patients should * Because of higher vascular tone, neonates and children may be in a shock state long before
manifestation of hypotension.
prompt further diagnostic evaluation for
end-organ damage. The definition of septic shock has been simplified as a subset of sepsis in which underlying

545
circulatory and cellular/metabolic • Check glucose •Congestive heart failure
abnormalities are profound enough to
• Start 2 L IV Fluid bolus (LR or NS) for • Pulmonary embolism
increase mortality substantially. These
adults or 20 mL/kg for pediatrics
patients are identified by persistent • Acute respiratory distress syndrome
(unless malnourished)
hypotension and having lactate greater
than 2 mmol/L after fluid resuscitation. Of • Be prepared to assist with airway Environmental
note, the term severe sepsis should no • Heat stroke
patency or protection (i.e., intubation) if
longer be used as all sepsis is considered necessary • Burns
to have a high probability of being severe.
• Prepare broad-spectrum antibiotics for
Endocrine
Critical Bedside Actions and administration within the first hour
• DKA
General Approach
For the critically ill patient, you must Differential Diagnosis • Adrenal crisis
make a rapid determination of syndromic
Differential Diagnosis of Sepsis • Thyrotoxicosis
category of their acute illness. If they
– non infectious etiologies
meet the above definition of sepsis and • Pancreatitis
there is a concern for ongoing shock, Shock states
• Cardiogenic shock • Hypoglycemia
proceed to the critical bedside actions.

• Hypovolemic shock Toxicologic


First 5 – 10 minutes if high
• Salicylate toxicity
suspicion for septic shock • Hemorrhagic shock
• Transfer to critical care room if available • Neuroleptic Malignant Syndrome
• Obstructive shock
• Vital signs, primary bedside evaluation • Serotonin syndrome
with ABC…, administer O2, attach to • Distributive shock
the cardiac monitor • Sympathomimetic toxidrome
Cardiac/pulmonary
• 2 large bore IVs • Myocardial infarction • Delerium tremens

546
Neurologic Table 13.4 Findings Associated with an Physical Exam Signs
• Status epilepticus Infectious Source • Vital Signs: Tachycardia, fever, low BP,
FINDING SOURCE tachypnea
• Cerebral hemorrhage
Pulmonary findings Pneumonia,
empyema, • Poor perfusion: hot or cool skin, altered
History and Physical Exam parapneumonic mental status, poor urine output (<0.5
effusion
Hints mL/kg/hr), weak pulses,
Urine appearance URI, pyelonephritis,
infected renal calcluli
History • Pediatric-specific: skin mottling,
• History of immunocompromise (HIV, Skin findings Cellulitis, abscess,
(wounds, rash, meningococcus, viral delayed capillary refill skin, poor urine
chemotherapy, etc.), alcoholism, crepitus, bullae) or tick borne disease, output (<1 mL/kg/hr)
malignancy, liver disease, diabetes, gangrene, necrotizing
fasciitis
ongoing steroid use, intravenous drug • Associated finding less likely to be non-
Focal neurologic Cerebral abscess,
use deficit epidural abscess infectious source: chest pain, evidence
of DVT, evidence of ingestion (pill
Bony findings (pain or Myositis, discitis,
• Travel history, vaccination status asymmetry in osteomyelitis, septic fragments)
extremities) joint
• Recent illness, sick contacts Peritoneal signs Intra-abdominal Emergency Diagnostic Tests
abscess/
• History associated with source: cough, inflammation, and Interpretation
perforation,
dysuria, shortness of breath, chest pain, spontaneous bacterial
peritonitis
Labs
abdominal pain, vomiting, diarrhea,
• Specific derangements in sepsis:
back pain, decreased urine output, Heart findings (rub, Pericarditis,
murmur) endocarditis creatinine, LFTs, bilirubin, platelets,
focal neurological deficits, rash or skin
Stridor Epiglotitis, tracheitis, coagulation studies
changes, change in mental status croup
• Serial serum lactates
• Pediatric-specific: increased work of Vaginal discharge Pelvic inflammatory
disease, endometritis,
breathing, decreased PO intake, septic abortion,
chorioamnionitis
change in behavior
original by the author
547
• Source testing (guided by H&P): UA, • spine MRI, Hydroxyethyl starch should be
CSF, pleural, intraperitoneal, synovial avoided.
• extremity X-ray
fluid
Albumin may be used if indication AFTER
• Cultures: Blood, urine Emergency Treatment adequate crystalloid resuscitation.
Options
• CSF, body fluid if indicated Mortality for sepsis and septic shock can Monitor for signs of fluid overload
be as high as 40-50%. Decreases in (increasing hypoxia, rales, hepatomegaly
• Cultures are positive in sepsis only
mortality are accomplished via two goals: in children).
30-40% of the time
1. Restore tissue perfusion Oxygenation
• Special tests (if indicated): malaria,
dengue, viral hemorrhagic fever, etc. 2. Locate and treat infectious source Target saturation of > 90%

Imaging Restore tissue perfusion Intubation for respiratory failure and


• Guided by history and physical consider for those in refractory septic
shock.
• Chest x-ray for all Fluids
Vasopressors
• Bedside ultrasound of IVC to monitor Resuscitation with normal saline
resuscitation Provide for those with MAP < 65 mmHg
• General recommendations for adults d e s p i t e a d e q u a t e fl u i d
Video tutorial (1 and 2) for IVC include 20-40 mL/kg in defined volumes resuscitation.  (MAP = [(2 x diastolic) +
measurement amounts (500 mL) with reevaluation systolic]/ 3)
after every bolus administration.
Consider based on history:
Initial vasopressors may be given
• Pediatrics – start with 20 mL/kg bolus peripherally initially, although central
• abdominal CT,
venous access preferred.
• S i g n i fic a n t c a u t i o n i n t h o s e
• head CT,
malnourished or severely anemic

548
Norepinephrine (0.01-3 mcg/kg/min) the • Remove catheters and lines Pediatric, Geriatric,
preferred choice, with the recommended associated with infection. Pregnant Patient and Other
second agent of epinephrine (0.1 – 1
Timing Considerations
mcg/kg/min) or vasopressin (0.03 units/
min) Critically ill patients should have
Pediatric
Neonates and immunocompromised
antibiotics given within 1 hour.
Steroids (sickle cell, oncology, diabetic and HIV)
Do not delay antibiotics for testing! patients are at particular risk for
Consider for those with MAP < 65 mmHg
overwhelming infection.
despite fluid and vasopressor therapy. Antibiotics
Controversy exists regarding fluid
Hydrocortisone (adults – 200 mg, Choice of antibiotics driven by local
management in developing countries with
pediatric 1-2 mg/kg) or equivalent resistance patterns, region-specific
high malaria prevalence rates with fluid
epidemiology (HIV, malaria, influenza, etc.
Locate and treat infectious boluses found to increase mortality.
prevalence), and availability of drugs
source
For respiratory distress and hypoxemia,
Those with septic shock should always
make early use of high flow nasal cannula
include broad-spectrum coverage (gram
Location and CPAP while starting resuscitation.
positive, gram negative, anaerobes).
• Guided by history and physical Extracorporeal Membrane Oxygenation
Consider antimalarials where appropriate.
(ECMO) may be considered for refractory
• Source control: pediatric shock and respiratory failure.
Consider specific anti-viral therapy when
• Debride or drain any localized source appropriate (i.e., acyclovir for
Geriatric
of infection; surgical consult for meningoencephalitis, oseltamivir for
Increased risk factors due to
deeper infections such as influenza in the immunocompromised
comorbidities, endocrine deficiencies,
intrabdominal abscess or empyema. host).
pre-existing malnutrition, and age-related
immunosenescence.

549
Diagnosis may be more difficult as the References and Further Reading, click
initial inflammatory response to infection here
may be blunted or absent.

1.5 higher mortality rates than younger


patients.

Pregnant
The etiology of sepsis in pregnant women
is expanded and includes septic abortion,
chorioamnionitis/endometritis, group A
Streptococcus infection, particular
susceptibility to influenza and necrotizing
vulvitis.

Adequate resuscitation of the mother


often will improve outcomes for the fetus;
however, in the critically ill patient, early
delivery of the fetus should be considered
with appropriate consultation.

Disposition Decisions
All patients with suspected sepsis should
be admitted.

ICU level care for those who meet criteria


for septic shock.

550
Chapter 14

Selected
Toxicologic
Problems
Section 1

Opioid Overdose

Case Presentation
by Aldo Emigdio Bartolini Salinas and Jesús A 22-year-old male was brought to the emergency room by
Daniel López Tapia
EMS at 7 pm. His parents arrived at the hospital and
mentioned that this was not the first time their son had a
similar event. The patient was lethargic upon his arrival so
clinical history was difficult to obtain from the patient and the
parents had no additional information to provide.

On physical examination, the patient was lethargic and slightly


bradycardic with 59 bpm, his respiratory rate was 14 per
minute. Body temperature was stable at 36.6°C and blood
pressure was 105/80 mmHg. Oxygen saturation at room air
was 94%. Bilateral pupils were miotic, but no clear
measurement was obtained. On auscultation, cardiac and lung
sounds were rhythmic and stable. Neurologic examination was
accurate. Gastrointestinal auscultation showed hypoactive
bowel sounds. When blood pressure was measured there

552
were needle marks on both of his forearms. The and medicinal settings. Unfortunately, opioid derivatives such
as prescription drugs and “recreational” drugs like heroin have
rest of the systematic evaluation was normal.
been motive of abuse and intoxication, being the number one
Electrocardiogram and laboratory studies were illicit drug on the market with more deadly outcomes due to
made to rule out other possible diagnoses. abuse.

Opiate pharmacokinetics, toxicology, clinical manifestations, and


Critical Bedside Actions and General
management will be broadly discussed in this chapter.
Approach
In an emergency setting, evaluating circulation, airway, breathing
Epidemiology
and vital signs are a priority.
Worldwide opioid abuse, dependence, and intoxication have
Attention must be paid to respiratory rate; pulse oximetry must be been increasing in numbers. As of 2015, the United States had an
done to evaluate oxygen saturation while additional studies are approximate of 2.6 million opioid consumers and addicts. This
being performed. A ventilation mask with supplementary oxygen number includes opioid addicts due to pain relievers and non-
to 100% must be given. Eyes, mental status, and skin should be prescription opioid consumption. It is vital to consider that heroin
examined. Miotic pupils, euphoria and skin marks are usually use via injection is responsible for the transmission of HIV and
found in opioid-intoxicated patients. hepatitis C. In Russia, Central Asia, and Ukraine, it is responsible
for 60-70% of all HIV infections.
A thorough clinical history and physical examination must be
done. Further details will be discussed in this chapter. Gender distribution of opioid abuse is more common in men than
women, ranging in a 3:1 (M: F ratio) for heroin and 1.5:1 for
Introduction prescription opiates. The most common age of consumers is of
According to the United Nations Office on Drugs and Crime, in 26 years or younger.
2010 there were more than 15 million opioid consumers
worldwide. Opiates are withdrawn from the poppy plant (Papaver Pharmacokinetics and Toxicology
somniferum), these substances have psychoactive properties and Opioid-related sites in the brain are hypothalamus, thalamus and
are processed into natural, synthetic and semi-synthetic methods the limbic system. They bind competitively to an opioid specific
of use. These substances have been used in cultural, recreational receptor. To this date, three specific receptors have been

553
identified: mu (µ), delta (δ) and kappa (Κ). • Nalmefene Three main effects caused by opioid
An opioid-receptor like-1 is still under consumption are analgesia, euphoria, and
• Naloxone
investigation. These subtypes have a anxiolysis. Analgesia occurs by inhibiting
specific and different effect on the body. • Naltrexone transmission from the peripheral nerve to
They are capable of producing cAMP the spinal cord. Anxiolysis happens when
(adenylate cyclase), closing/opening Opioid Agonist-Antagonist opioids act upon noradrenaline releasing
calcium and potassium channels, leading neurons located in the locus coeruleus.
• Buprenorphine
to the ability to hyperpolarize the cell and Euphoria is related to the mesolimbic
modulating neurotransmitter release. • Nalbuphine s y s t e m d o p a m i n e i n c r e a s e .

Most opioids are metabolized by the liver
There are three action category; • Pentazocine
to active metabolites and excreted by the
Opioid Agonists Opioids can be consumed orally, by kidneys. They have a large distribution
snorting and by subcutaneous or volume of 1-10L/kg and are protein-
• Codeine bonded in most cases, which makes
intravenous injection. Its effects vary
depending on their site of administration, hemodialysis a problematic way for
• Diphenoxylate-atropine
dose and the type of opioid consumed. If opioid clearance.
• Fentanyl it is taken orally, it may take about six
hours to have its maximum effect History and Physical
• Heroin
(methadone) and its clinical effect may Examination Hints
Opioids affect the body in various ways.
• Hydrocodone persist for 24 to 48 hours. When snorted,
The more consistent clinical effects of
its peak effect is usually 30 minutes after.
• Loperamide opioids are a depressed respiratory rate,
When injected subcutaneously, it may
changes in mental status, decreased
• Meperidine take just about 15 minutes. It may show
bowel sounds and pupillary constriction
a n i m m e d i a t e e ffe c t w h e n d o n e
• Methadone (miosis). Additional findings include
intravenously.
hypothermia, bradycardia, hyporeflexia,
Opioid Antagonists dermal marks.
554
Pupillary constriction (miosis) is frequent intoxication). It is usually resolved once a Drug-induced toxicity
in opioid intoxication. However, normal normal respiratory rate and ventilation are
• Ethanol toxicity produces none or little
pupils or mydriasis (pupil dilation) are obtained. Cardiovascular changes,
miosis and no gastrointestinal changes.
possible when the patient takes a mostly bradycardia and hypotension is
Withdrawal produces hyperthermia and
stimulant simultaneously or when the in due to an increase in parasympathetic
seizures.
case of extended respiratory depression. activity and release of histamine. Lethal
Therefore, normal pupil examination does ventricular tachyarrythmias might occur. • Sedative-Hypnotics toxicity causes less
not exclude the possibility of intoxication. respiratory depression and ataxia in
G a s t ro i n t e s t i n a l c h a n g e s i n c l u d e
A thorough and careful examination is a children. Withdrawal produces
decreased bowel sounds, peristalsis, and
must. hyperthermia and seizures.
constipation. Additionally, renal changes,
Check respiratory rate to evaluate a particularly renal failure due to • Clonidine toxicity causes miosis,
suspected opioid-intoxication. A rhabdomyolysis may be present (from hypotension, bradycardia and no
respiratory rate below 12 bpm is a great heroin and methadone abuse). Skin gastrointestinal changes.
predictor of toxicity. The pulse oximeter marks due to “skin popping” may be
shows oxygen saturation, but a normal present secondary to subcutaneous • Hypoglycemic agents
reading does not exclude hypercapnia. injection. Changes in the reproductive
Organophosphate toxicity causes miosis,
Monitoring respiratory ventilation via end- system include changes in menstrual
vomiting, diarrhea, bradycardia,
tidal CO2 monitoring and capnography cycles, infertility, abnormal prolactin
hypotension or tachycardia and
helps to assess diagnosis and possible secretion, and decreased libido.
hypertension.
complications.
Differential Diagnoses Any medical condition that causes coma.
Noncardiogenic pulmonary edema is Generally, clinical manifestation is enough
frequent in opioid intoxication. The for diagnosis. When in doubt, laboratory Emergency Diagnostic Tests
symptoms include cyanosis, pink and imaging findings may be helpful. The and Interpretation
bronchial secretions, and rales (with all most common differential diagnoses are History and physical examination are
the additional symptomatology of the following. generally sufficient to make a diagnosis.

555
In some complicated cases, laboratory Routine urine toxicologic screens are not •L o p e r a m i d e : Q T o r Q R S
studies, urine screening tests and cardiac recommended. They confirm recent prolongation, ventricular tachycardia.
screening are recommended. abuse, but not acute toxicity.
• Oxycodone: QT prolongation.
Laboratory tests They can report many false positive
• S e r u m o r fin g e r- s t i c k g l u c o s e results. Imaging tests
Plain chest X-rays are reserved for
measurement (to rule out hypoglycemia)
Cardiac screening patients who present with symptoms of
• Serum acetaminophen concentration (in aspiration pneumonia, respiratory distress
Electrocardiographic (ECG) evaluation is
case of suspected concurrent use with syndrome, uncorrected hypoxia or
strongly recommended in patients who
opioids, to rule out suicidal attempt with abnormal sounds during lung
present palpitations, syncope, chest pain
acetaminophen) auscultation.
and dysrhythmias.
• Serum creatine phosphokinase, blood Emergency Treatment
If the initial ECG is normal, a control ECG
urea nitrogen, creatinine, urinalysis and Options
should be repeated 4-6 hours after.
s e r u m e l e c t ro l y t e s ( t o r u l e o u t
rhabdomyolysis). If the initial ECG shows abnormalities
Initial Stabilization
In an emergency setting, evaluating
such as QT or QRS prolongation, cardiac
• Serum ethanol level (to rule out circulation, airway, breathing and vital
monitoring should be done until a normal
intoxication) signs are a priority. Ventilatory support
rhythm.
should be given with a bag mask and
• Blood gas measurement.
Some specific cardiac disturbances are 100% oxygen to patients with respiratory
• Basic metabolic panel presented in the following abused distress or when the proper respiratory
substances: function has been jeopardized. If the
Urine toxicologic screenings
patient has oxygen saturation above 90%
• Methadone: torsade de pointes, QT
Opioids can be detected in urine samples and more than 12 breaths per minute at
prolongation.
in a maximum period of two days. room air, observation is adequate.
However, if oxygen saturation drops

556
below 90%, supplemental oxygen, bag mask and 0.05 mg Procedures
intravenous naloxone must be administered to restore normal Naloxone might be administered intravenously, nebulized and
ventilation. If the blood glucose is 60mg/dl or less, administer intranasally. A summary of dosage and administration route is
glucose intravenously. discussed in Table 14.1.

Medications
The first line treatment is Naloxone, a lipophilic, short-acting Table 14.1 Naloxone Administration
opioid antagonist, that can effectively reverse opiate-related INTRAVENOUS NEBULIZED INTRANASAL
NALOXONE NALOXONE NALOXONE
symptoms. Full patient history is essential to determine whether
Recommended 0.4 mg in 10 mL 2 mg in 3ml 1 mg/ml per
the patient is a long-term or short-term opioid user, the type of Dosage of normal saline normal saline naris
opioid consumed, time of administration and dosage. (0.04mg/ml). solution with a (total dose: 2mg)
Administration in standard face
separate boluses mask.
The recommended naloxone dose is 0.4 mg for most patients, of 1ml.
including those with methadone abuse. It should be diluted in 10 Pros. A gentle method Easy titration, Can be used in a
mL of normal saline to reach 0.04mg/ml dilution. Administer 1 ml of opioid lower risk of patient with
intoxication withdrawal complicated
separate boluses to improve the patients’ respiratory rate above reversal, no symptoms. intravenous
eight breaths per minute. Its clinical effects last up to acute withdrawal access.
symptoms
approximately 70 minutes.
Cons. Close monitoring No clinical data It might be
is is available. difficult to titrate
In patients that have abused fentanyl or other synthetic opioids, recommended. because of
an increased dosage is recommended. No established dosing is unknown
absorption rates
available, but some recommend increasing naloxone dose every and
bioavailability in
2-3 minutes, starting with 0.5mg -2mg-4mg-10mg and
humans.
administering a maximum dose of 15mg. Another suggested
Adopted and developed from 
method of use is by administering naloxone every 2-3 minutes
Li, K., et al (2018). Annals of Emergency Medicine, 72(1), 9–11., Stolbach,
starting with 0.04mg-0.08mg-0.16mg. If respiratory rate is not A., & Hoffman, R. (2018, April 18). Acute Opioid Intoxication in Adults.
improved after maximum dose, a different diagnosis must be Retrieved from UpToDate: www.uptodate.com, and Yin, S. (2018, January
10). Opioid Intoxication in Children and Adolescents. Retrieved from
considered. UpToDate: www.uptodate.com

557
Pediatric, Geriatric, for pregnant women are codeine, o f toxicity or abuse, care must be done

Pregnant patient, and other morphine, pethidine, and propoxyphene. by a geriatrician or addictionologist.
In lactation, morphine can be
Considerations
administered, but it must be interrupted Respiratory or Hepatic
Pediatric population every 4 to 6 hours. Buprenorphine and pathology
Morphine and fentanyl might be used for Opioid use should be avoided. If used,
fentanyl are not recommended due to
moderate pain in children under 12 years close monitoring is of importance.
their high concentration in breast milk.
of age and infants, although respiratory
depression is frequent. Codeine and Opioid intoxication during pregnancy can Disposition Decisions
tramadol may be used in children older result in severe respiratory distress in
Admission criteria
than 12 years old. both the mother and the neonate. Opioid
• Opioid overdoses with short-acting
abuse causes neonatal abstinence agents may be treated in the
The treatment of opioid intoxication is syndrome characterized by low birth emergency department.
dependent on children’s weight. 0.1 mg/ weight, CNS hyperirritability, myoclonus,
kg IV naloxone (max.2mg per dose) hyperreflexia, sweating, vomiting, • Opioid overdoses with a long-acting
should be administered below 20 kg. 2 diarrhea, death, and others. Opioid (ex. methadone) agents or
mg IV naloxone is recommended over 20 intoxication during pregnancy requires a combinations must be admitted to the
kg. neonatologist in the team. ICU.

Pregnancy Geriatric population Discharge criteria


Most opioids are category C and Geriatric population may be particularly • A psychiatric evaluation is
teratogenic, especially in the first vulnerable to opioid side effects or recommended once the patients’
trimester. If used for an extended period toxicity. Lowered doses must and well- mental status and respiration rate
or during delivery (except for pethidine), established times and days are become normal.
opioids may cause respiratory depression recommended. Caretakers should be
in both the mother and the neonate. For • Naloxone should not be administered
capable and adequately knowledgeable
2-3 hours before discharge.
chronic pain, opioids should only be about medication administration. In case
given as a last resort. The safer opioids
558
Referral
In case of intentional opioid overdose (as a suicide attempt or
drug addiction), the patient must be referred to a psychiatric
evaluation or rehabilitation facility.

References and Further Reading, click here

559
Chapter 15

Selected Eye
Problems
Section 1

Eye Trauma

Introduction of protective goggles reduces the


by Serpil Yaylaci and Kamil Kayayurt The most common cause of unilateral number of those injuries by 70%.
blindness in industrialized countries is
eye trauma, most of which can be Anatomy
The eyeball consists primarily of three
prevented with the use of protective
layers. The outermost layer is a fibrous
goggles. Every year, more than 55
structure made up of the sclera and
million people worldwide experience
the cornea. The middle layer is the
eye traumas, and more than one
vascular layer, called the uvea. The iris,
million suffer total loss of vision. Eye
ciliary body, and choroid are located
traumas constitute approximately 3%
inside the uvea. The innermost layer is
of total emergency department cases;
the neural layer, called the retina.
most of these traumas are minor
injuries. Major injuries are less The eyeball is located in the bony
frequent; however, the rate of recovery cavity, called the orbit. The superior
of visual impairments is quite low in wall of the orbit consists of the frontal
these cases. Sixty-five to eighty bone, the lateral wall consists of the
percent of eye traumas are observed in zygoma, and the medial and
males, typically 25–44 years of age. anteromedial walls consist of the
About half of eye traumas are the result maxilla, lacrimal, and ethmoid bones.
of occupational accidents, and the use The video shows bones of the orbit.

561
Please visit this  link  to see detailed Eye Globe Injuries •Eyelid lacerations
Anatomy videos. 1. Open-globe injuries ( Rupture of globe)
• Retrobulbar hematoma
2. Closed-globe injuries
Illustration 15.1 Anatomy of the eye • Traumatic optic neuropathy
• Conjunctival laserations
• Ophthalmic arter injuries
• Partial thickness corneal and scleral
• Extraocular muscle entrapment
lacerations

• Corneal and conjuntival abrasions Globe Injuries

• Hyphema Globe Rupture


It is an ophthalmologic emergency,
• Lens dislocation consisting of a full-thickness injury in the
cornea or sclera caused by penetrating or
Blausen.com staff (2014). “Medical gallery of • Traumatic iritis
Blausen Medical 2014“. WikiJournal of blunt trauma. Anterior rupture is usually
Medicine 1 (2). DOI:10.15347/wjm/
2014.010. ISSN2002-4436 • Retinal detachment observed, as this is the region where the
sclera is the thinnest. Posterior rupture is
• Vitreous hemorrhage
Orbital Traumas rare and difficult to diagnose. It can be
Eye traumas can be divided into three diagnosed through indirect findings such
• Commotio retina
groups: globe injuries, periorbital injuries, as contraction in the anterior chamber
and chemical injuries. Only selected Periocular trauma and decrease in intraocular pressure (IOP)
injuries observed most frequently in 1. Orbita fractures in the affected eye. If there is a risk of
emergency departments and that can globe rupture, a slit lamp test with 10%
2. Ekstraocular muscle, vascular, and
lead to total loss of vision will be fluorescein must be conducted. Normal
eyelid injuries
discussed. tissue is dark orange under a blue cobalt
• Orbital compartment syndrome filter; a lighter color is observed in the
Classification of Eye Trauma damaged zone due to a lower dye

562
concentration. Ultrasonography (USG) Image 15.1 ectopic pupil after Image 15.2 corneal foreign body
can be useful in making a diagnosis, penetrated eye trauma
especially with posterior ruptures.
Computed tomography (CT) sensitivity
ranges 56–75%. In cases of anterior
globe injuries, USG use, and if there is a
risk of a foreign metal body, magnetic
resonance imaging, are contraindicated.
Prompt ophthalmology consultation is
required. While in the emergency
d e p a r t m e n t , t e t a n u s p ro p h y l a x i s ,
analgesics, bed rest, head elevation, and Image 15.3 corneal foreign body
systemic antibiotic therapy are required. Foreign Bodies
Orbital foreign bodies are classified as
The most commonly preferred antibiotics
superficial or intraorbital. Superficial
are cefazolin and vancomycin. Age over
foreign bodies constitute the second
60 years; injury sustained by assault, on
most common general eye injury, after
the street/highway, during a fall, or by
corneal abrasions, and are the most
gunshot; and posterior injuries are
common work-related injuries. They
indications of a poor prognosis.
usually consist of earth, stone, wood and
metal pieces. Organic foreign bodies
have a higher risk of infection. Intraocular
foreign bodies (IOFB) are most commonly
observed in young males in the form of
hammering injuries. Blast injuries and
combat injuries are also frequently
observed.

563
Image 15.4 superglued eye lamp. Any buried foreign bodies should might be caused as a result of the
be removed by an ophthalmologist. If defect, relieving pain, and speeding up
there is a risk of IOFB, the patient should recovery must be targeted.
be referred to an ophthalmologist. If there Fluoroquinolones are preferred as
is an IOFB, the patient should be treated antibiotics, and oral analgesics are
as a case of globe rupture. preferred for relieving pain. Topical non-
steroids and anesthetics should not be
Corneal Abrasions
These are epithelium defects following a
Image 15.5 Fluorescein staining
non-penetrating eye trauma; they
confirms the presence of a corneal
constitute the most common eye abrasion
Diagnosis usually depends on patient pathology caused by trauma. The most
history. The patient should be asked the common cause is chronic contact lens
location and intensity of the trauma, as use. Other causes include blunt trauma,
well as time elapsed since the injury. In foreign bodies, burns, and radiation.
addition, the patient’s tetanus risk should Symptoms include stinging, burns, pain,
be determined. There may be multiple and a feeling that there is a foreign object
foreign bodies. During the diagnosis, a present. During diagnosis, a slit lamp
microscopic examination with fluorescein examination must be conducted with
must be conducted, and it must be fluorescein, and the dimensions and
determined if there is an intraocular shape of the defect must be ascertained.
foreign object. After applying local Linear defects indicate the possibility of a Courtesy of Simon Arunga. Retrieved
foreign body located in the inner part of from https://flic.kr/p/NZrhfH.
anesthesia, superficial foreign bodies can
be removed with saline irrigation or a wet the eyelid. Therefore, the inner parts of
used.
cotton swab. If unsuccessful, an attempt the eyelid must be examined for foreign
can be made to remove the object with a bodies as well. In the treatment,
25G needle under direct vision using a slit preventing bacterial superinfections that

564
Hyphema Image 15.7 Hyphaema from blunt
It is defined as bleeding in the anterior trauma.
chamber; the source of the bleeding is
the iris root or ciliary body. Although
hyphema can be caused by many
medical conditions, the most common
one is trauma. Among the causes of

Image 15.6 Hyphaema

Some iris detail is visible. A level cannot be


seen. Secondary glaucoma is likely until the
blood has absorbed. Courtesy of International
Centre for Eye Health. Retrieved from https://
flic.kr/p/dP3EPE.

lead to loss of vision; however, the source


of the bleeding is important. There is no
correlation between the amount of
Courtesy of Allen Foster. Retreived from https:// bleeding and tissue damage; small
flic.kr/p/CPv1Nf.
amounts of bleeding can be observed in
major injuries. When evaluating a patient
with hyphema, the manner in which the
trauma occurred, its intensity, and time
trauma, athletic injuries rank the highest. should be determined. There is usually
In children without underlying diseases, more than one eye pathology in patients
non-accidental injuries should come to with hyphema, and missing them could
mind. Bleeding itself does not usually have serious consequences. For this
565
reason, these patients must be evaluated Lens dislocation years after trauma. Although it can
by an ophthalmologist. During emergency Rupture of the fibrils attaching the lens to also be observed immediately after
department treatment, bed rest, head the ciliary body, following trauma can trauma due to distracting injuries and
elevation, eye patching, mydriatic/ result in subluxation or dislocation of the poor patient cooperation and poor
cycloplegic agents, antifibrinolytics lens. The most frequently posterior visualization as a result of periocular
(tranexamic acids), and anti-glaucoma dislocations; due to the restrictive effect edema, it is difficult to diagnose at the
drugs (beta blockers, carbonic anhydrase of the iris, anterior dislocations are less acute stage. While the most common
frequent. Sudden onset of loss of vision, cause is closed blunt trauma, it has a
Image 15.8 Dislocated lens after monocular diplopia, photosensitivity, red higher incidence of open globe injuries.
trauma. Symptoms include unilateral floaters,
eye, subconjunctival hemorrhage, and
periorbital ecchymosis can be observed photopsia, and visual impairment. During
in these patients. Anterior dislocations the physical examination, visual acuity,
can cause acute angle closure glaucoma visual field, and light reflex must be
by disrupting the flow of aqueous humor. evaluated, and IOP should be measured.
USG and CT can be used for diagnosis. Retinal detachment and vitreous bleeding
Treatment can vary from observation to can be seen via ophthalmoscopy. Retinal
surgical treatment, based on whether detachment can be detected with USG,
there are additional injuries and on the and a characteristic detachment pattern
Courtesy of Bruce Noble. Retrieved region of dislocation. in the form of a “V” can be observed with
from https://flic.kr/p/9oxpTD CT. Early diagnosis is to ensure surgical
Retinal Detachment intervention by an ophthalmologist. The
Traumatic retinal detachment (TRD) is the prognosis depends on the size of the
separation of the neurosensorial retina TRD, whether it involves the macula and
inhibitors) can be used. Complications
from the retinal pigment epithelia length of time.
such as glaucoma, corneal blood
underneath it, leading to a disruption of
staining, rebleeding, and optic atrophy
its nourishment, and thus, a loss of Vitreous hemorrhage: is bleeding in the
can develop.
vision. TRD typically develops months or posterior segment as a result of damage

566
to the retinal veins following blunt trauma. Examination findings can vary from a enophthalmos greater than 2 mm,
It is usually accompanied by retinal simple ecchymosis and edema to loss of significant hypoglobus or diplopia, and an
detachment, and the patient complains of vision. Sensitivity in the orbital wall, increase inorbital volume greater than 1
a sudden loss of vision. Fundoscopy, subcutaneous emphysema, and cm3.
USG, and CT can be used for diagnosis. irregularity in orbital rhythms can be
Treatment is surgical; although there are observed. Pupil diameters and light Orbital Compartment Syndrome
This condition is an ophthalmologic
differing views on its timing, due to high reflexivity must be evaluated. Upper and
surgical emergency that develops
TRD frequency, it is suggested that the lower eyesight restriction and diplopia
following an acute increase in intraorbital
surgery is performed at an early stage. can develop if the inferior rectus and
volume and pressure. A sudden increase
inferior oblique muscles are caught in the
Periorbital Injuries fracture line. In cases of medial wall
in pressure can cause blindness via
compression of the optic nerves and/or
fractures, patients can suffer epistaxis.
Orbital Fractures vascular structures when not diagnosed
It can occur as isolated incidents, as well When orbital pressure increases to very
at an early stage and treated. The most
as together with other facial bone high levels, optic nerve damage and loss
common causes are trauma, intraocular
fractures. The most commonly observed of vision can occur.
injections, and surgery. Orbital cellulite or
isolated orbital fracture is a blow-out The gold standard in diagnosis is CT, and abscess, orbital emphysema, foreign
fracture. There are three theories it should be used to evaluate the axial bodies, and tumors can also lead to this
regarding its formation: indirect impact and coronal planes. Fractures can condition. Orbital volume is about 30 ml,
related to increasing intraorbital pressure manifest in two ways in CT: the first one and it is surrounded by the bony orbit,
caused by trauma (hydraulic mechanism); is direct visualization of irregularity and which prevents expansion. The only
direct conveying of energy during orbital dislocations in bone cortexes, and the possibility is to expand toward the
wall trauma (the buckling mechanism); second one is visualization of air-liquid anterior, but that movement is limited by
and a combination of the two level in the sinuses around the orbit and the canthal ligaments attached to the
mechanisms. The most common air in the orbital cavity. Immediate surgery eyelids. Diagnosis is clinical. In patients
fractures are in the inferior and medial is rarely necessary for treatment. Surgical with the predisposing causes mentioned
walls. treatment indications include above, it should be considered as a

567
possibility if there are findings such as Eyelid Lacerations and toxoid should be administered
reduced eyesight, diplopia, pain, and Eyelids anatomically consist of five layers. together. Superficial lacerations can be
proptosis. Reduced eyesight, afferent The outermost layer is the thin skin layer; sutured with 6.0 nylon or polypropylene.
pupil defect, elevated IOP, painful eye beneath it is the subcutaneous tissue, Eyelids have a risk of edema, and a cold
movement, and proptosis can be and beneath that are the orbicularis oculi compress after repair can decrease
detected during a physical examination. muscle, which allows the eyelids to be swelling and wound tension. Sutures can
Widened blind spot, reduced color sight shut, the meibomian glands and the be removed on the fifth day. Ptosis in the
(especially red), and afferent pupil defect tarsal plate containing the eyelashes, and eye, lacerations closer than 1 cm to the
detected during a visual field test are the the innermost layer is the conjunctiva. medial canthus, and cuts reaching the
most reliable findings that suggest optic Eyelid lacerations (ELLs) are injuries tarsal plate should be evaluated by an
nerve damage. The possibility of optic caused by blunt or penetrating ophthalmologist or plastic surgeon.
disc edema and retinal vein occlusion or mechanisms. Because the eyelid is
congestion should be investigated with anatomically thin, it provides little Retrobulbar hematoma
Retrobulbar hematomas are hemorrhages
fundoscopy. If the patient history and protection against penetrating injuries,
formed behind the globe due to trauma,
physical examination support the and the risk of globe injury is high in
surgery, and posterior injections. They are
findings, no time should be lost with penetrating traumas. ELLs are injuries
usually arterial in origin; the inferior orbital
visualization methods. Normal IOP level is that require special attention, and certain
arteries and anterior ethmoidal arteries
3–6 mmHg. Although there is not a points must be considered. Before
are most commonly injured. The clinical
specific pressure limit defined for orbital repairing the laceration, a complete
importance is that this condition leads to
compartment syndrome, values ≥30 physical examination must be
compartment syndrome by causing
mmHg are considered to be high. The undertaken. During the examination, lid
increased pressure inside the orbital
most important factor in making a and globe movements, visual field,
cavity. Patients can suffer decreased
treatment decision is the presence of corneal injuries, foreign bodies, and globe
visual acuity, sluggish light reflex,
clinical findings. Treatment is surgical, perforation should be evaluated. All
restricted eye movement, painful
and lateral canthotomy and cantholysis patients must be asked about tetanus
proptosis, and afferent pupil defect. CT is
are the surgeries preferred most often. immunization. If there has been no
the most commonly preferred
immunization, immunoglobulin (250 U)
568
visualization method for diagnosis. Grade 1: Only epithelial damage; no While the prognosis is good for
Although there is not a globally accepted limbal ischemia. grades 1 and 2, it is poor for grades 3
algorithm for treatment, there are and 4. The first thing to do when a
Grade 2: Obscurity on the cornea;
medical and surgical treatment options. chemical substance contacts the eye is
however, iris details can be spotted and
Corticosteroids are used in medical to irrigate it with normal saline or Ringer’s
there is ischemia in less than 1/3 of the
treatments, and lateral canthotomy and lactate solution in order to neutralize the
limbus.
cantholysis are the surgical treatments. eye’s pH. Applying a local anesthetic will
Grade 3: Total loss of corneal epithelia. relieve the patient’s pain. If care is being
Chemical Injuries administered at the scene, tap water can
Stromal obscurity prevents spotting iris
Eye traumas caused by chemical
be used for irrigation. Grade 1 and 2
substances constitute a wide spectrum
Image 15.9 corneal chemical burn injuries can be treated with antibiotics,
ranging from corneal abrasions, which
steroids, and cycloplegic drugs. As
are simple burn symptoms, to serious
antibiotics, preparations containing
burns that can result in permanent
tobramycin or quinolone (ciprofloxacin,
blindness. The most commonly
ofloxacin) can be used 4–5 times per
encountered chemicals are cleaning
day. Steroids decrease inflammation and
materials, personal care items, and
prevent neutrophil activation. Grade 3
automobile chemicals. Alkaline chemical
and 4 injuries may require surgical
injuries are more common than acidic
treatment.
ones. Because acidic materials lead to
coagulation necrosis and scar formation,
deep penetration is restricted. Alkaline
details. 1/3–1/2 limbal ischemia. References and Further Reading, click
materials cause deeper wounds due to
here
liquefaction necrosis. Burns are grouped Grade 4: The cornea is completely
into four grades, based upon intensity. opaque and there is >50% limbal
ischemia.

569
Section 2

The Red Eye

Case Presentation
by David Brian Wood A 27-year-old female with no past medical problems presents
to the emergency room complaining of 2 days of a red, itchy
and burning left eye. She notes that she has had a lot of
watery discharge during this time and that her vision is blurry
on occasion but improves after blinking a few times. She
works in a day care where many of the children have been sick
lately. She does not use corrective lenses and does not recall
any trauma to the eye. She also denies systemic symptoms
such as fever, photophobia, or joint pain. Vitals: T 98.5oF, HR
78, BP 124/68, RR14, SpO2 100% on room air. Visual acuity:
20/20 in both eyes. Peripheral fields: intact. Forehead/maxilla:
no erythema or swelling. Lids/lashes: left eyelid mildly swollen.
Otherwise, lids and lashes are normal, and eversion of eyelid
demonstrates no foreign bodies. Conjunctiva: conjunctiva on
the left is diffusely injected, and there is watery discharge.

570
Pupils: round, reactive to light, Figure 15.1 Approach to red eye.
and equal bilaterally. Slit lamp:
lids, lashes, and conjunctiva as
above. No abrasions or
lacerations visualized with
fluorescein. Anterior chamber
without cell and flare.
Intraocular pressure: 18 mmHg
bilaterally.

Critical Bedside Actions and


General Approach
The initial step in the assessment is to
determine if there is chemical exposure.
The eye should be irrigated for at least 30
minutes or longer. If there has been
exposure to an alkaline substance,
irrigation can be stopped once the pH of
the tears is neutral.

If there has been no chemical exposure, a


thorough history and physical should be
obtained. One practical approach is to
first subdivide the etiologies into painful Approach to the presentation of a red eye in the Emergency Department. Adapted from Life in the Fast
or painless. Lane: The Red Eye Challenge.
571
Differential Diagnoses • Diffuse- usually caused by an eyelid •Systemic symptoms including fever,
issue such as blepharitis, ectropion, URI symptoms, weight loss, or joint
Painful red eye trichiasis, entropion, stye, or tumor. pain
If the eye is painful, break it down based
on the anatomic structure involved. • Localized- based on the structure • History of inflammatory disease (SLE,
involved. Examples include MS, ankylosing spondylitis, etc.)
• Cornea (abrasions, ulcer, keratitis, subconjunctival hemorrhage, trauma,
herpes simplex keratitis) The patient’s  description of eye
corneal foreign body, chalazion, and
pterygium. sensation  can also help with the
• Eyelid (internal hordeolum, chalazion,
evaluation of the red-eye.
external hordeolum, blepharitis, HZ
History and Physical
ophtalmicus, preseptal cellulitis) • Itching/burning: conjunctivitis, dry eye
Examination Hints syndrome, or blepharitis.
• Conjunctiva (conjunctivitis, dry eyes, A thorough history should be obtained by
glaucoma) paying particular attention to the • Foreign body  sensation: cor neal
following: irritation or inflammation
• Ciliary or scleral (scleritis, episcleritis)
• Trauma to the face or eye (including • Sharp pain: anterior chamber process
• Anterior chamber (anterior uveitis/iritis, possible foreign bodies) such as keratitis, uveitis, acute angle-
endophtalmitis, hyphema)
closure glaucoma
• Pain and the specific location of the
• Posterior chamber—usually does not pain • Dull pain: increased intraocular pressure
cause a red eye and is more likely to
or an extra orbital process
interfere with vision or cause deep dull • Photophobia
pain. Concerning findings on history:
• Pain with extraocular movements
Painless red eye • Change in vision
• Severe ocular pain
If the red eye is painless, determine if the
• Persistently blurred vision
erythema is localized or diffuse.

572
• Soft contact use-more susceptible to 3. Extra-ocular eye movements:  Look for including vitreous hemorrhage, retinal
bacterial infection disconjugate gaze and ask if the detachment, or optic neuritis.
patient develops any diplopia when
• Immunocompromise 8. Slit lamp exam:  Re-examine the lids,
looking in a certain direction. Either of
lashes, conjunctiva, and cornea. Use
The  ocular examination  has numerous these findings suggests an entrapped
fluorescein to evaluate for foreign
components, requires particular technical extraocular muscle or nerve deficit.
bodies, abrasions, lacerations, or
skill and therefore should be approached
4. Surrounding structures (forehead and Siedels sign (streaming of aqueous
systematically so that no part of the exam
maxilla):  Assess for surrounding humor from punctured globe site).
is overlooked.
erythema, induration, or rash. Always have the patient move the eyes
One approach to examining the eye is for in all directions to visualize the entire
5. Eyelids and lashes: Remember to evert
the examiner to begin peripherally and conjunctiva. Visualize the anterior
the eyelids as well. Look for localized
progress inward, ending with the chamber to look for cell and flare,
swelling or redness.
fundoscopic examination as shown white blood cells (hypopyon), or red
below: Order of Exam 6. Conjunctiva blood cells (hyphema).

1. Visual Acuity : Remember to test each 7. P u p i l s :  N o t e t h e s i z e , s h a p e , 9. Intraocular pressure assessment:  Do


eye with and without corrective lenses. symmetry, clarity, and reactivity to not measure if there may be a
If they do not have corrective lenses, a light. Assess for an afferent papillary penetrating injury as this would
pinhole can be used which will correct defect (APD) using the swinging exacerbate aqueous humor leakage.
most refractive disturbances. If vision flashlight test. If the pupil dilates when
• A pressure of 10-20mmgHg is
is too poor to read an eye chart, the light is shining in it, that eye has
considered normal.
assess if the patient can detect the and APD and is not transmitting as
following: count fingers, detect hand strong of a response to the light • A pressure >20mmHg warrants an
motion, any light perception. compared to the opposite eye. An APD ophthalmology consult.
can be caused by many problems
2. Peripheral fields • Pressures >30 necessitate rapid
treatment of underlying etiology (lateral
573
canthotomy for retrobulbar hematoma concern for an underlying autoimmune a concern for retrobulbar hematoma
or medication administration for acute disorder or temporal arteritis erythrocyte and the intraocular pressure is
closed-angle glaucoma). sedimentation rate (ESR), C-reactive >40mmHg, a lateral canthotomy should
protein (CRP) levels can be assessed. be performed. If neither of these
10.Fundoscopic exam:  Often difficult to
conditions exists then treatment can be
do a thorough exam in the emergency Imaging tailored to the specific diagnosis.
department, as the eye is typically not The two most common scenarios of
dilated. obtaining imaging of the red eye include Painful Red Eye
facial fracture and penetrating foreign
“The application of topical body. Plain radiographs can be obtained Cornea
anesthetics to the eye helps to to assess for facial fractures. However,
Corneal abrasions
CT of the facial bones has replaced this
differentiate between Most corneal abrasions will heal rapidly
imaging modality as it is more sensitive independent of intervention. Treatment
superficial and deeper causes and specific for fractures. If there is a focuses on preventing secondary
of pain. Pain that is not concern for a penetrating foreign body, a i n f e c t i o n a n d c o n t ro l l i n g p a i n . A
completely relieved by topical CT of the orbits can also be obtained. An significant component of the pain from a
MRI is an excellent modality for assessing corneal abrasion comes from ciliary
anesthetics is more likely to be
foreign bodies but is contraindicated if spasm that can be relieved by topical
from the sclera or anterior there is any concern that the object may cycloplegics.
chamber.” be metallic.
• Cyclopentolate is a short acting agent,
Emergency Diagnostic Tests Emergency Treatment Repeat every 4-6 hours.
and Interpretation Options
As stated previously, any eye that has • Homatropine is a long acting. Its effect
Laboratory tests lasts 2 days.
come into contact with a chemical should
In most cases, laboratory tests are rarely be irrigated with 1-2L of NS until the pH In addition, oral NSAIDs or opiates may
indicated or of much utility. If there is a of tears has returned to neutral. If there is be needed to control the pain adequately.

574
To prevent secondary infection, topical Antibiotics Image 15.11 Corneal ulcer with
antibiotic ointments can be used (below circumcorneal congestion.
recommendations are adopted from • Gentamicin ointment/solution 0.3%

Harwood-Nuss and Rosen). If the • Ciprofloxacin solution 0.3% –  Effective


abrasion is large or crosses the central against Pseudomonas; prescribe to
visual axis, the patient should follow up contact lens wearers
with ophthalmology within 24 hours.
Otherwise, follow-up can occur within 72 • Erythromycin ointment 0.5% – Effective
hours to ensure the abrasion is healing. against Pseudomonas; prescribe to
contact lens wearers

• Ofloxacin 0.3% Courtesy of P Vijayalakshmi. Retrieved from


https://www.flickr.com/photos/
Image 15.10 A corneal abrasion after communityeyehealth/5444946687
staining with florescine. • Polymyxin/trimethoprim

Antivirals Herpes Simplex Keratitis


Herpes Simples infections can be
• Trifluridine 1% diagnosed based on its characteristic
dendritic pattern seen with fluorescein
• Vidarabine 3%
staining. Conjunctival infections can be

Corneal ulcers treated with trifluridine one drop up to


Corneal ulcers are more serious and can nine times per day and antibiotic
pose a significant threat to the patient’s ointment such as erythromycin can be
vision. Ophthalmology should be added to prevent secondary infections.
consulted emergently for cultures of the All patients for which there is a concern
ulcer and initiation of antibiotics as well for herpes keratitis should be seen by an
Courtesy of James Heilman, MD – Own work, as, in certain cases, antifungals. ophthalmologist within 48 hours.
CC BY-SA 3.0, https://commons.wikimedia.org/
w/index.php?curid=11918476

575
Image 15.12 Herpes simplex virus do not improve after 1-2 weeks of Image 15.14 Posterior blepharitis
treatment.

Image 15.13 Chalazion

Top left: Child with measles and severe herpes Courtesy ofJohn KG Dart Retrieved from https://
simplex keratitis affecting the right eye. Top www.flickr.com/photos/communityeyehealth/
right: Dendritic ulcer stained with fluorescein dye 32271350840/in/photolist-aBgGJh-aBgGZo-
Bottom left: Geographic ulcer stained with aBe1Hi-aBgGR1-aBe1cn-aBgFWu-aBgH3S-
fluorescein dye Bottom right: Inflamed aBe1gM-aBgFV9-
© International Centre for Eye Health
conjunctiva and geographic ulcer Photo aBe1We-9J2LcJ-9J2L9d-9A9VAo-9HYUGv-
www.iceh.org.uk, London School of Hygiene &
(clockwise from top-left): John Sandford-Smith, RaH7r9-RaH7hG

Tropical Medicine. Retrieved from https://
Allen Foster, David Yorston). Retrieved from
www.flickr.com/photos/communityeyehealth/
https://www.flickr.com/photos/
8423448539/in/photolist-dQmNx6- Herpes zoster ophthalmicus
communityeyehealth/5616320250/in/
dQmnMn-9Ezoru
photolist-9yi7kL-cAjBe7-dQmoJB-9yi7rQ- Patients with herpes zoster ocular
cAjAUY-cAjAQj-cAjBbA-dQmoR2-
cAjB8s-9EwFS5-9AaetU-9wqCZN-cAjAAC Acute Blepharitis infections should be treated with artificial
Blepharitis is caused by inflammation of tears and erythromycin ointment to
Eyelid an eyelash follicle due to an overgrowth prevent secondary infection. Oral antiviral
of bacterial skin flora. The mainstay of medication can be used if there is skin
Internal/External hordeolum and
treatment consists of daily cleaning of the involvement and, after consultation with
acute chalazion
edge of the eyelashes. an ophthalmologist, topical antivirals may
Initial treatment for these conditions
be prescribed as well. The significant pain
consists of warm compresses and
from herpes zoster infections may require
erythromycin ointment twice daily for
opiate treatments or the use of an
7-10 days. Referral to ophthalmology as
an outpatient can be made if symptoms
576
antidepressant such as amitriptyline Image 15.16 Acute glaucoma, red eye. parasympathetic agonist, causes
25mg P.O. TID. myosis. Rarely causes sweating,
bradycardia, hypotension.
Image 15.15 Herpes zoster
ophthalmicus • Apraclonidine  – 1% – 1gtt q5min x3
doses. Decreases production of
aqueous humor (alfa-2 agonist). Used
most often in chronic glaucoma but
may be useful in AACG.

Photo: International Centre for Eye Health


• Latanoprost  – 0.005% 1gtt daily.
www.iceh.org.uk, London School of Hygiene &
Tropical Medicine. Increases outflow.

Acute closed-angle glaucoma and IOP S y s t e m i c a g e n t s



Courtesy of John Sandford-Smith. Retreived lowering agents  can be devided into 2
from https://www.flickr.com/photos/
communityeyehealth/5686390813/in/ category (adopted from Tintinalli)

photolist-9yKTSq-9EueSV-9yxBrw • Acetazolamide  – 500mg IV q12h or
500mg PO q6h. It is a carbonic
Conjunctiva T o p i c a l a g e n t s
 a n h y d r a s e i n h i b i t o r. D e c r e a s e s
production of aqueous humor. It should
Acute closed-angle glaucoma
The overall goal is to reduce intraocular be avoided  in patients with respiratory
• Timolol  – 0.5% 1 gtt q5min x3 doses
p re s s u re b y d e c re a s i n g a q u e o u s disease as it causes respiratory
then 1gtt q12h, decreses production of
production and increasing outflow. acidosis.
aqueous humor. Should be avoided in
Aqueous outflow is improved through
patients with asthma, heart block, and • Mannitol  20%   – 1-2 gram/kg IV over
miosis as this pulls the iris away from the 30-60 minutes. Decreases Aqueous
heart failure.
trabecular meshwork. Definitive treatment humor by increasing serum osmolality.

is an iridectomy performed by an • Pilocarpine – 2% 2 gtt q5min until pupil
ophthalmologist. constricts. Then 1gtt q6h. It is outflow

577
Conjunctivitis Ciliary/scleral Image 15.18 Recurrent scleritis
Most cases of conjunctivitis will be due to
allergic or viral causes and can be treated Episcleritis
Artificial tears can be used up to four
with artificial tears 5-6 times per day. If
times per day to help lubricate the eye. A
there is a concern for a bacterial cause of
trial of oral NSAIDs can be given in the
conjunctivitis the patient can be treated
emergency room and if pain resolves can
four times daily for 5-7 days with topical
be continued as an outpatient. If the
antibiotic drops such as trimethoprim or
patient continues to have significant pain
polymyxin B. If the patient wears soft
after NSAID a topical steroid can be used
contact lenses, then Pseudomonal
to relieve the discomfort. The steroid
coverage is necessary with a
drops can be continued as an outpatient
fluoroquinolone or aminoglycoside.
until seen by ophthalmology in 2-3 By Imrankabirhossain – Own work, CC BY-SA
4.0, Retrieved from https://
weeks. commons.wikimedia.org/w/index.php?
curid=60068874

Image 15.17 Lorem Ipsum dolor amet, Scleritis


consectetur Oral NSAIDs can be used for pain control Anterior chamber
as in episcleritis. Topical steroids are
Uveitis/Iritis
ineffective in scleritis, however, and oral
Often related to a systemic process such
steroids may be used starting at
as a rheumatologic condition,
prednisone 60mg daily for 1 week and a
malignancy, or infection. Iritis and Uveitis
slow taper over the next 4-6 weeks.
can be treated symptomatically with
Ophthalmology should be consulted and
cycloplegics, which paralyze the ciliary
may recommend starting additional
body and pupillary sphincter. A long-
immunosuppressive agents.
acting agent such as homatropine will
Courtesy of Rbmorley – Robert Morley,
Retrieved from https://commons.wikimedia.org/ last for 2-3 days after one dose and can
w/index.php?curid=8658905 control the pain until the patient can be
578
seen by an ophthalmologist. These and administer intraocular antibiotics and glaucoma. Patients with hyphema
patients should be seen by an steroids. should have ophthalmology consult in the
ophthalmologist within 48 hours.
Image 15.20 Endophthalmitis Image 15.21 Hyphema
Image 15.19 Acute anterior uveitis

Endophthalmitis with extensive hypopyon


consistent with active infection. © International
Centre for Eye Health iceh.lshtm.ac.uk, London By Rakesh Ahuja, MD – Own work, CC BY-SA
45-year-old female. Complains of painful eye School of Hygiene & Tropical Medicine. 2.5, https://commons.wikimedia.org/w/
and discomfort in bright light with watery Retrieved from https://www.flickr.com/photos/ index.php?curid=1270407
discharge. VA 6/12. Photo: International Centre communityeyehealth/7608314920/in/
for Eye Health www.iceh.org.uk, London School photolist-9oAtpo-cAjAXs-cAjAHs-9Ex9Yq-
MGPFgy ED.
of Hygiene & Tropical Medicine. Retrieved from
https://www.flickr.com/photos/
communityeyehealth/5594571189/in/
Hyphema Others
photolist-9wnD6X-czQdY9
Initial treatment consists of elevating the
Retrobulbar hematoma
Endophthalmitis patient’s head to allow the red blood cells
In patients in which there is a concern for
Usually leads to vision loss and therefore to settle inferiorly where they are less
retrobulbar hematoma or other space-
requires an emergent ophthalmology likely to obscure the trabecular meshwork
occupying retro-orbital lesions in which
consult. Admission is necessary to and raise intraocular pressure. If
the intraocular pressure is >40mmHg, a
administer IV antibiotics. In addition, the i n t r a o c u l a r p re s s u re i s i n c re a s e d
lateral canthotomy should be performed
ophthalmologist may aspirate the vitreous >30mmHg, the same treatment options
to relieve the pressure and spare the
can be employed as described in
579
optic nerve from further damage. If a Subconjunctival Hemorrhage recommendations adopted from
retrobulbar hematoma is present, but the Harwood-Nuss) or topical NSAIDs.
intraocular pressure is below 40mmgHg, Surgical removal by an ophthalmologist
N o t re a t m e n t i s n e c e s s a r y f o r a
and there is no vision loss, may be indicated if the visual axis is
subconjunctival hemorrhage as the blood
ophthalmology should be consulted. impaired or for persistent irritation.
Similar medications to those used in
Image 15.23 Pterygium Topical steroids
acute closed-angle glaucoma can also be
used to help decrease the intraocular • Prednisolone acetate 0.125% – Potent.
pressure. E ffe c t i v e f o r a n t e r i o r c h a m b e r
inflammation

• Medrysone 1% – Mild potency. Use for


Image 15.22 Subconjunctival
Hemorrhage allergies

By unknown photographer. Retrieved from • Rimexolone 1% – Mild potency


https://commons.wikimedia.org/w/index.php?
curid=56208402
• Dexamethasone 0.1% (solution), 0.05%
will resolve within 2 weeks. (ointment) – Potent

Pterygium Corneal foreign body


The management of a pterygium focuses If a small corneal body is seen on the
largely on preventing it from enlarging exam with the slit lamp, the eye should
through the use of sunglasses or goggles first be anesthetized prior to removal with
By Daniel Flather – Own work, CC BY-SA
3.0, Retrieved from https:// to prevent ultraviolet light, dust, and other a topical anesthetic. The foreign body can
commons.wikimedia.org/w/index.php? then be removed using a small-gauge
curid=15651313 irritants. If the pterygium is inflamed,
artificial tears can be used four times per needle, fine forceps, or irrigation. A
Painless Red Eye day. For severe inflammation, topical metallic foreign body will usually leave a
steroids can be used (below rust ring that should be removed with an

580
ophthalmic burr if available. After the bony orbit and lower eyelid to allow the having the child fixate on an object of
foreign body is removed, the resulting orbit to move forward to compensate for interest. If the vision is normal, the child
the increased pressure placed on it. To will continue to fixate on the object. By
Image 15.24 Corneal foreign body perform a lateral canthotomy the canthus around 3 years old visual acuity can be
is first anesthetized, then crushed with tested more effectively using an Allen
curved forceps, and then cut with chart or Tumbling E chart.
scissors. The inferior canthus tendon can
Neonatal conjunctivitis  is usually caused
then be identified by strumming it with
by exposure to an infectious agent while
the scissors and can then be incised.
exiting the birth canal. Most commonly
Following severing the tendon, the inferior
neonatal conjunctivitis is caused by
eyelid should be released completely
Chlamydia trachomatis or Neisseria
from the orbit. Depending on the amount
gonorrhoea but can also be due to
This is a ‘rust ring’ which shows signs of having of time the retina was ischemic, there
been present for some days. The iron particle or herpes simplex virus. Treatment is based
may be rapid improvement in vision as
‘rust’ will lift off the cornea easily but will leave a on the incubation period as shown below
stained area beneath. Removal with a needle or the pressure is reduced.
drill (burr) will be necessary. Retrieved from (adopted from Harwood-Nuss). In
https://www.flickr.com/photos/
communityeyehealth/8408519738 Pediatric Patients addition, neonates with HSV will require
Pediatric patients suffer from many of the ophthalmology consultation. HSV and
defect can be treated as a corneal same etiologies of red eyes as adults but gonococcal conjunctivitis will require
abrasion with topical antibiotic ointment. may be more difficult to obtain an inpatient therapy with ophthalmology
adequate exam on. Visual acuity in serving as a consult.
Procedures children begins around 20/100 and
While rare, the main ophthalmologic N. gonorrhoeae has 2-7 days incubation
improves to 20/20 by approximately 8
procedure performed in the emergency period. Ceftriaxone 25-50mg/kg IV or IM,
years of age. Before 5 years of age, most
department is a lateral canthotomy. The once is effective for treatment. For
children will be unable to read a Snellen
overall goal of the lateral canthotomy is prophylaxis, 1% silver nitrate, 0.5%
chart. Visual acuity can be grossly tested
too severe the connection between the
by covering each eye separately and

581
erythromycin ointment, 1% tetracycline can be used.

C. trachomatis has 5-14 days incubation period. Treatment is


erythromycin 50mg/kg/day divided q.i.d. PO x14 days. There is
no prophylactic agent recommendation.

HSV has 1-2 weeks incubation period. Acyclovir 60 mg/kg/day


divided q.i.d. for 2-3 weeks AND atopical anti-viral agent are used
for treatment. There is no prophylactic agent recommendation.

Disposition Decisions
The vast majority of patients presenting for red-eye will be
discharged home. Even many of the ocular emergencies will be
able to be discharged following evaluation by ophthalmology.
There are a few conditions requiring admission such as
endophthalmitis, retrobulbar hematoma, and globe rupture.
Disposition and urgency of consultation are covered in the above
clinical management of ocular problems.

References and Further Reading, click here

582
Chapter 16

Selected
Procedures
Section 1

Automated External Defibrillator (AED) Use

Introduction should be used in combination with


by Mehmet Ali Aslaner AED is a portable electronic device that cardiopulmonary resuscitation (CPR).

produced to detect and treat the life- Pulseless ventricular tachycardia and
threatening cardiac arrhythmias such ventricular fibrillation are the most
as like ventricular fibrillation and common and treatable causes of SCA.
ventricular tachycardia in case of Therefore, the use of an AED is very
sudden cardiac arrest (SCA). AED important and vital.
defibrillate (electrical therapy) the
patient to stop life-threatening Where to find an AED?
Public areas, corporate or government,
arrhythmias and allow an effective
should keep an AED in offices,
rhythm.
shopping centers, airports, airplanes,
AED device gives simple orders and restaurants, casinos, hotels, sports
can be used by a layperson who was stadiums, community centers, fitness
previously trained before, a certified centers, health clubs, theme parks,
first responder, and health care schools and universities, workplaces
professionals. and any other location where people
may gather.
Indication
2015 European Resuscitation Council AED Kit
Guidelines recommend using an AED An AED kit contains
for adult basic life support (BLS). It

584
• a face shield to provide a barrier 4. As soon as the AED arrives, CPR providers should continue CPR
between the patient and first aid with minimal interruption of chest
• switch on the AED and attach the
provider during rescue breathing, compressions while attaching an AED.
electrode pads on the patient’s bare
Standard AEDs are suitable for use in
• rubber gloves, chest.
children older than 8 years. AEDs are safe
• trauma shears for cutting through a • CPR should be continued while to use. Currently, There are no published
patient’s clothing to expose the chest, electrode pads are being attached to reports of AEDs’ harmful effects on
the chest, if there is two rescuer. bystanders. Also, there are no reports of
• a towel for wiping away any moisture AEDs delivering inappropriate shocks. If
on the chest, and • Follow the spoken/visual directions, someone has a sudden cardiac arrest,
using an AED and giving CPR can
• a razor for shaving extensively hairy • ensure that no one is touching the
improve the person’s chance of survival.
chests. patient while the AED is analyzing the
rhythm.
How to use an AED?
1. recognize abnormal status in case of • If a shock is indicated, push the References and Further Reading, click
BLS. When facing an unconscious shock button as directed (fully here
person, you should decide if he/she is automatic AEDs will deliver the
alive or not (unresponsive and not shock automatically). Immediately
breathing normally) by the BLS restart CPR 30-2 and continue as
algorithm. directed by the voice and visual
directions.
2. If the person is not breathing normally,
call the emergency services and send • If no shock is indicated, continue
someone to get AED. CPR until emergency medical service
(EMS) arrives.
3. begin chest compressions with rescue
breaths 30-2 (if trained or able to do). Please watch the video

585
Section 2

Arterial Blood Gas (ABG) Sampling

Case Presentation
by Matija Ambooz and Gregor Prosen A 23 years old pregnant woman was admitted with a history of
polyuria, dysuria, fever, and thirst. She is an insulin dependent
diabetic patient. She is febrile. Her chest is clear, and
circulation is adequate. Urinalysis shows the presence of
ketones, glucose, and leukocytes. Her lab results on
admission are:

Na+ 136 mmol/L, K+ 4.8 mmol/L, Cl- 101 mmol/L, Glucose
23.2 mmol/L, Urea 8.1 mmol/L, Creatinine 0.09 mmol/L

Her ABG results are:



pH: 7.26

pCO2 = 14 mmHg

pO2 = 133 mmHg

HCO3- = 7.1 mmol/L

586
Low pCO2 and low HCO3- Partial pressures of carbon dioxide Image 16.1 An example of an arterial
(PaCO2) and oxygen (PaO2), hydrogen blood gas analysis result.
indicates metabolic acidosis.
ion activity (pH), total hemoglobin (Hb),
Hyperglycemia, glycosuria, and oxyhemoglobin saturation (HbO2), and
ketonuria indicate DKA. There carboxyhemoglobin (COHb) and
methemoglobin (MetHb) are directly
might be an underlying UTI that
measured.
triggered DKA. Respiratory
Oxygen (O2) and carbon dioxide (CO2)
alkalosis is a compensation.
are the most important respiratory gases,
and their partial pressures in arterial
Introduction
Arterial blood gas (ABG) analysis is an blood show the overall adequacy of gas
important investigation to monitor the exchange. pH, which measures hydrogen
acid-base balance of critically ill patients. ion activity, is a regular part of every
arterial blood gas sampling (Image 16.1).
ABG help to determine treatment may To learn how to evaluate ABG analysis
indicate the severity of the condition and please click here.
can help to diagnose a disease. The
re s p i r a t o r y s t a t u s a n d a c i d - b a s e
equilibrium of individuals with pulmonary
disorders, drug overdose, and metabolic
disorders may be evaluated through this
procedure.

Blood is drawn from a peripheral artery


via single percutaneous needle puncture,
or from an indwelling arterial cannula or
catheter for multiple samples.
587
ABG Sampling Procedure • Raynaud’s syndrome, f o r a radial artery puncture, Also 23
gauge and 25 gauge needle can be used.
Indications • Full-thickness burns. Relative
A 23 gauge syringe may be used as it
• to evaluate ventilation, (PacO2) acid- contraindications include:
allows faster filling than 25 gauge one,
base status (pH and PaCO2), but does not affect more pain to the
• Skin infection at the site of puncture,
oxygenation status (PaO2 and SaO2), patient (Figure 16.2).
and the oxygen-carrying capacity of • Previous surgery in the area,
blood (PaO2, HbO2, Hbtotal, and Image 16.2 Needle and syringe before
dyshemoglobins) • Inadequate collateral flow, assembly

• Partial-thickness burns,
• to quantitate the patient’s response to
therapeutic intervention and/or • Atherosclerosis,
diagnostic evaluation (e.g., oxygen
therapy, exercise testing) • Anticoagulation or coagulopathy*.

• to monitor the progression and severity *ABG sampling can be performed safely
of the observed disease. We usually in patients who are on anticoagulants or
evaluate these parameters in patients have other coagulopathies. In patients
• 70% isopropyl alcohol or an antiseptic
with multi-organ failure, both chronic with severe disseminated coagulopathies,
solution,
and acute respiratory failure, ventilated extreme caution is required.
patients, critically ill trauma patients, • gauze or cotton-wool ball to be applied
Equipment and Patient over puncture site,
septic patients, patients with burns and
Preparation
poisoned patients.
Equipment used in arterial puncture • well-fitting non-sterile gloves
include;
Contraindications • puncture-resistant container.
• Inadequate circulation,
ABG syringe, for an adult, use a 20-
With an adult patient who is conscious,
• Burger’s disease, gauge, 2.5-inch needle for a femoral
follow the steps below (adapted from
sample and a 22 gauge, 1.25-inch needle
588
W.H.O. best phlebotomy practice commonly punctured for blood gas inadequate, repeat the test on the
guidelines). sampling in adults. The first choice is the other hand.
radial artery  due to its superficial
• Introduce yourself to the patient and • Video 1: Modified Allen test;
anatomical location. It has good collateral
ask their full name. Radial and Ulnar Artery are both
circulation and is not surrounded by
pressed to prevent blood flow.
• Check that the laboratory form matches structures that could be easily damaged
Ulnar artery is released after the
the patient’s identity. by puncturing.
hand becomes pale. If the hand
• Ask whether the patient has allergies, The procedure as defined by W.H.O. flushes after 5s – 15s, the ulnar
phobias or has ever fainted during guidelines consists of 16 steps for radial artery has sufficient blood flow
previous injections or blood draws. artery puncture. and radial artery may be
punctured. If it takes more than
• Discuss the procedure and obtain 1. Approach the patient, introduce
15s for hand to flush, the ulnar
verbal consent. yourself and ask the patient to state
artery has inadequate blood flow
their full name.
and this hand should not be
• If the patient is afraid or anxious, help
2. Place the patient on their back, lying punctured.
him relax and make him more
comfortable. flat. Ask the nurse for assistance if the
4. Perform hand hygiene, clear off a
patient’s position needs to be altered
bedside work area and prepare
• Make the patient comfortable in a to make them comfortable. If the
supplies.
supine position. patient is clenching their fist, holding
their breath or crying, this can change 5. Disinfect the sampling site on the
• Place a clean paper or towel under the
breathing and thus alter the test result. patient with 70% alcohol and allow it
patient’s arm.
to dry.
3. Locate the radial artery by performing
Procedure Steps an Allen test for collateral circulation 6. Assemble the needle and heparinized
Various arteries can be used for blood
(Video 1). If the test fails to locate the syringe and pull the syringe plunger to
c o l l e c t i o n . T h e  r a d i a l ,  b r a c h i a l ,
radial artery or collateral flow is
and  femoralarteries are the sites most
589
the required fill level recommended by 9. Withdraw the needle and syringe; a l t e r n a t i v e l y, u s e a l c o h o l r u b
the local laboratory. (1 – 3 mL) place a clean, dry piece of gauze or solution.Check the patient site for
cotton wool over the site and have the bleeding and thank the patient.
patient or an assistant apply firm
Image 16.3 Syringe and needle 15.Check the patient site for bleeding and
pressure for sufficient time to stop the
prepared for puncturing. thank the patient.
bleeding. Check whether bleeding has
stopped after 2–3 minutes. 16.Transport the sample immediately to
the laboratory, following laboratory
10.Activate the mechanisms of a safety
handling procedures.
needle to cover the needle before
placing it in the ice cup. ABG Sampling video 1 and video 2

11.Expel air bubbles, cap the syringe and


Hints and Pitfalls
7. Holding the syringe like a dart, use the roll the specimen between the hands
• Precooling proved to be useful for
index finger to locate the pulse again, to gently mix it. Cap the syringe to patients who had problems with anxiety
inform the patient that the skin is about prevent contact between the arterial and pain due to arterial blood
to be pierced then insert the needle at blood sample and the air, and to puncturing. Cryoanalgesia can be
a 45-degree angle, approximately 1 cm prevent leaking during transport to the provided by ice bag applied to wrist 3
distal to the index finger, to avoid laboratory. minutes prior to arterial puncture.
contaminating the area where the
12.Label the sample syringe.
needle enters the skin. • ABG measurements are particularly
13.Dispose appropriately of all used vulnerable to pre-analytic errors.
8. Advance the needle into the radial
material and personal protective Problems include air bubbles, improper
artery until a blood flashback appears,
equipment. anticoagulation, delayed analysis, non-
then allow the syringe to fill to the
arterial samples and other transport or
appropriate level. DO NOT pull back 14.Remove gloves and wash hands handling related problems.
the syringe plunger. thoroughly with soap and water, then
dry using single-use towels;
590
• After collection, the sample should be alter all three ABG measurements. After •T h e h e m a t o m a i s a c o m m o n
analyzed quickly. If a delay of more than flushing the syringe with heparin, a complication without a serious sequel.
10 minutes is anticipated, the sample sufficient amount usually remains in the It may be prevented by inserting the
must be embedded in an ice bath. dead space of the syringe and needle needle without puncturing the fat side
Leukocytes and platelets continue to for anticoagulation without distortion of of the vessel and by applying firm
consume oxygen in the sample after it the ABG determination. pressure at the site of a puncture. Due
is drawn and can cause a significant fall to high pressure present in arteries,
in PaO2 over time at room temperature, Post Procedure Care and pressure should be applied for a longer
especially in the setting of leukocytosis Recommendations time than in venipuncture.
or thrombocytosis. Cooling decrease After collection of at least 1 to 2 mL of
sample, the needle is removed, and firm • Nerve damage may be prevented by
the metabolic activity of leukocytes and
pressure is applied at the site of puncture choosing an appropriate site for
platelets and thus prevent the clinically
for 3 – 5 minutes. If the patient is on puncturing and avoiding redirection of
important effect of oxygen consumption
anticoagulants or has any coagulopathy, the needle.
for at least 1 hour.
the pressure is required for 10 – 15
• Fainting may be prevented by ensuring
• Room air has a PO2, of approximately minutes.
that the patient is lying down with their
150-160 mmHg (at sea level) and a
feet elevated.
PCO2 of essentially zero. Thus, air Complications
bubbles that mix and equilibrate with There are some potential complications
Geriatrics, Pediatrics,
arterial blood will shift the PaO2 toward related to arterial blood sampling.
Pregnant Patients and Other
150 mmHg and PaCO2 toward zero.
• Temporary arterial occlusion and spasm Considerations
• Heparin must be added to the syringe may be prevented by helping the Pediatrics should be mentioned as a
as an anticoagulant. Because the pH of patient relax. One can achieve this by special consideration due to the
heparin is near 7.0, and the PO2 and explaining the procedure, positioning challenge that they present in the form of
PCO2 of the heparin solution are near the patient comfortably and using obtaining vascular access and blood
room air values, excess heparin can precooling or other forms of analgesia. samples. Fear and anticipation of pain
associated with procedures may the
591
hospital experience traumatic for References and Further Reading, click
children. The procedure should be here
explained before starting and consent
taken. Parents may provide comfort to
the child, but there is also a potential for
parents to faint. Products to decrease the
pain may be considered in stable
patients. Capillary blood obtained from
heel is another option and can be used
for gas analysis when arterial access is
unavailable or when the clinician is not
comfortable obtaining a percutaneous
arterial blood sample. Arterial blood may
be obtained from radial, brachial, dorsalis
pedis and in newborn infants, the
umbilical arteries. The radial artery is the
site of choice. For arterial puncture in
infants and children, a small-gauge
butterfly needle is preferable to a needle
and syringe as used in adults. In contrast
to arterial puncturing in adults,
continuous, but gentle suction should be
provided in infants. Pulsating blood is a
good sign that the radial artery has been
punctured.

592
Section 3

Arthrocentesis

Introduction • Evaluation of therapeutic response


by Tanju Tasyurek Arthrocentesis is an acknowledged, for septic arthritis
useful procedure to puncture and
• Diagnosis of traumatic bony or
aspiration of a joint. It is usually
ligamentous injury
performed both as a diagnostic and
therapeutic tool for various clinical • Installation of medications for acute
situations. Arthrocentesis (synovial or chronic arthritis
fluid aspiration) of a joint can be
• Relief of the pain of acute
performed either diagnostically (for
hemarthrosis
identification of the etiology of acute
arthritis) or therapeutically (for pain • Determination of communication
relief, drainage of effusion, or injection between the laceration and joint
of medications). Arthrocentesis is space
required procedure in majority of
patients with monoarthritis and is Contraindications
mandatory if an infection is suspected. • Absolute contraindication to
arthrocentesis is an  infection in the
Indications of tissue overlying the site  to be
Arthrocentesis punctured. However, inflammation
• Diagnosis of septic or crystal- with warmth, swelling, and
induced arthritis tenderness may overlie an acutely

593
arthritic joint, and this condition may • Various syringes (5 mL, 20 mL, 30 mL, b e taken. To avoid infection, aseptic
mimic a soft tissue infection. 60 mL) technique is essential, including the use
of sterile gloves and instruments. After
• C o a g u l o p a t h y  i s a n a b s o l u t e • Various size of needles, 18 or 20 G and
s k i n p re p a r a t i o n w i t h a n t i s e p t i c
contraindication. However, few studies 25 or 27 G
solutions, the clinician should allow the
are demonstrating whether it is
• Morbidly obese patients might require solution to dry for several minutes
dangerous performing arthrocentesis in
a 2 1 - g a u g e s p i n a l
 because the bactericidal effects of iodine
patients using anticoagulants. It was
needle for arthrocentesis are dependent on both concentration
found  safe  even in those who have
and time. Iodine solution should be
international normalized ratios as high • Specimen tubes removed with an alcohol sponge. This
as 4.5.
will prevent iodine transfer into the joint
• Bandage
• Prosthetic joints  increase the risk for space, which can cause an inflammation.
infection. Therefore arthrocentesis General Arthrocentesis Without anesthesia, arthrocentesis may
should be avoided for these joints. Technique be quite painful. Entire route of the
However, if an infected prosthesis is Arthrocentesis is a relatively simple
needle should be anesthesized from skin
suspected, arthrocentesis should be procedure. Knowledge of anatomic
to joint capsule. 1% or 2 % lidocaine can
performed. landmarks and patient positioning will
b e u s e d .

aid in the successful completion of joint
Rigid needles are preferred whereas
Equipment aspiration. Defining the anatomy is the
some clinicians can use sturdy catheters.
• Sterile gloves and drapes most important part of the procedure.
As a general rule, one should try to
The clinician should be familiar with the
• Gauze pads (5), 4 × 4 inches. remove as much fluid or blood as
anatomy of the specific joint and
possible.
• Skin cleaning agent landmarks in order to avoid puncture of
tendons, blood vessels, and nerves. Arthrocentesis of the hip joint is generally
• Local anesthetic such as Lidocaine 1%
performed by an orthopedic surgeon.

The procedure should be explained to
İt may be difficult to aspirate fluid from
the patient and written consent should

594
small joints. If only one drop of fluid is • Pronate the patient’s forearm and rest it parallel to the radial shaft. The
obtained from small joints, it is best to with the palm down on a side table set landmarks can be found easily if the arm
send it for culture. at the appropriate height for comfort. is first extended. At this point, the
depression can be located. Then flex and
The common complications • Identify the olecranon process, lateral
pronate the arm for the procedure.
of procedure epicondyle, and radial head, and find
• Iatrogenic infection the depression (or bulge, if the effusion Because of the risk of ulnar nerve and
is large) in the soft triangle. This site is superior ulnar collateral artery injury, the
• Iatrogenic hemorrhage used for all approaches. medial approach should not be used.

• Pain during the time of the procedure • Identify the entry site, and mark the site How to locate the entry site; please
with a plastic needle sheath or a sterile watch the video.
• Reaccumulation of the joint fluid
surgical marker.
Real patient example (watch the video)
Specific Arthrocentesis
• Carefully examine the elbow before
Techniques arthrocentesis. Radiocarpal Joint (Wrist)
Landmarks and positioning are important The wrist joint is anatomically complex.
while performing arthrocentesis. For small • Olecranon bursitis is located posteriorly The dorsal site is the preferred site of
joints, application of traction is often very over the olecranon and can be aspiration of the wrist joint.
helpful in obtaining fluid. confused with the elbow joint.
The landmark of this joint is the dorsal
Radiohumeral Joint (Elbow) The alternative is the posterolateral radial tubercle (Lister’s tubercle). The
approach can be used. However, there is extensor pollicis longus tendon runs in a
Lateral approach an increased risk of injury to the radial groove on the radial side of the tubercle.
• The patient sits upright on a stretcher.
nerve and triceps tendon. This approach The tendon can be palpated by active
• Bend the patient’s elbow to 90º. is useful if the bulge of effusion is extension of the wrist and thumb.
palpated inferior to the lateral epicondyle.
• The wrist should be slightly palmar
In the posterolateral approach, insert the
flexed to facilitate the performance of
needle perpendicular to the skin but
595
t h e p r o c e d u r e .
 • First of all arthrocentesis of this joint is F o r the parapatellar approach, identify
The positioning of the wrist is moderately difficult. the midpoint of either the medial or the
approximately 20 to 30 degrees of lateral border of the patella. Insert an 18-
• The patient should sit upright with the
flexion with accompanying ulnar gauge needle 3-4 mm below the midpoint
arm at the side, with the shoulder held
deviation. of either the medial or the lateral border
in external rotation.
of the patella. Direct the needle toward
• Applying traction to the hand might be
• To find the landmark clinician should the intercondylar notch of the femur by
helpful.
palpate the coracoid process medially perpendicular to its’ long axis.
• Insert the needle dorsally just distal to and the proximal end of the humerus
For the suprapatellar approach, identify
the radius and just ulnar to the laterally.
the midpoint of either side of the
anatomic snuff box.
• The clinician should insert a 20-gauge superomedial or the superolateral border
• Avoid the associated tendons (extensor needle at a point inferior and lateral to of the patella. Insert an 18-gauge needle
carpi radialis brevis and extensor the coracoid process and direct it through the midpoint of either superior
pollicis longus). posteriorly toward the glenoid rim. borders. Direct the needle toward the
intercondylar notch of the femur.

• Direct the needle perpendicular to the The video shows posterior approach. The needle enters the suprapatellar
skin. bursa. Remember that in 10% of the
Knee Joint, Anteromedial
population, the suprapatellar bursa does
• If the bone is hit, pull the needle back Approach
not communicate with the knee joint.
and redirect it slightly toward the The medial surface of the patella at the
thumb. middle or superior portion of the patella is For the infrapatellar approach, position
the landmark for the knee joint. Knee the patient sitting upright with the knee
Watch the video 1 and video 2.
arthrocentesis may be done via the bent at 90° over the edge of the bed.
parapatellar approach (which is generally
Glenohumeral Joint (Shoulder), Identify either side of the inferior border
Anterior Approach preferred), suprapatellar approach, or of the patella and the patellar tendon.
infrapatellar approach. Insert an 18-gauge needle 5 mm below

596
the inferior border of the patella and just proximal interphalangeal and distal
lateral to the edge of the patellar tendon. interphalangeal joints. The extensor
Be careful not to go through the patellar tendon of the great toe can be located by
tendon while inserting the needle. active extension of the toe. The clinician
should insert the needle into the skin at a
Please watch the video 1 and video 2.
90-degree angle and enter the
dorsomedial aspect of the great toe
Tibiotalar Joint (Ankle)
The medial malleolar sulcus is bordered (MTP) joint, just medial to the extensor
medially by the medial malleolus and tendon.
laterally by the anterior tibial tendon. The
Please watch video.
tendon can easily be identified with active
dorsiflexion of the foot. The clinician
should insert the needle at a point just
References and Further Reading, click
medial to the anterior tibial tendon and
here
directed into the hollow at the anterior
edge of the medial malleolus. The needle
must be inserted 2 to 3 cm to penetrate
the joint space.

Please watch the video.

Metatarsophalangeal and
Interphalangeal Joints
For the first digit, landmarks are the distal
metatarsal head and the proximal base of
the first phalanx. For the other toes, the
landmarks are the prominences at the

597
Section 4

Basics of Bleeding Control

Types of wounds If the wound is deep, it may need a


by Ana Spehonja and Gregor Prosen surgical consult or referral for better
Contusion (Contusio) cosmetic healing. Please check the
It is a result of minor forces, usually wound care chapter.
over clothes on 90°angle. Capillaries
beneath the skin can rupture due to a Wound (Vulnus)
blunt blow or punch. There can also be • caused by sharp object – vulnus
a hematoma. Analgesics and RICE scissum
(rest, ice, compression, and elevation)
• laceration – vulnus lacerum
will be enough for management.
• puncture wound – vulnus ictum
Abrasion (Excoriation)
It is a result of forces obliquely hit skin. • bite wound – vulnus morsum
The topmost layers of skin are scraped
off, leaving a raw, tender area. Clean • gunshot wound – vulnus
sclopetarium
the wound, put a sterile bandage, give
an analgesic, tetanus protection, and • explosive wound – vulnus
RICE (rest, ice, compression, and explosivum
elevation) are the parts of
management. • contusion – conquassatio

598
Types are based on time, place, the • Bruit or thrill Damaged subcutaneous capillaries.
cause of injury. A clean wound which is Slow dotted bleeding.
• Active or pulsatile bleeding
not older than 6-8 hours (18-24 hours on
face) can be closed right away. Wounds • Signs of limb ischemia
Assessment and Simple
that are older than 8 hours should be Procedures
thoroughly cleaned and cover with wet • Pulsatile or expanding hematoma
Initial evaluation when
gauze. They can be closed after 3-5 days
Soft Signs assessing wounds that are not
when they are clean, and there is no sign
• Proximity of injury to vascular structures life- or limb-threatening:
of infection. • past medical history and circumstances
• Major single nerve deficit surrounding the injury,
If blood vessels affected,
• Non-expanding hematoma • remove rings or other jewelry that
Arterial bleeding
encircle the injured body part,
It is a consequence of injury to the artery. • Reduced pulses
The blood is pulsating out of the wound • review the mechanism of injury,
• Posterior knee or anterior elbow
and has a bright red color. If the artery is
dislocation • ask about the presence of a foreign
lacerated through the whole lumen, it will
spontaneously shrink and limit the body sensation,
• Hypotension or moderate blood loss at
bleeding. However, if there is only injury the scene • determine the time that the injury
to the wall of an artery and it is not occurred
thoroughly dissected, this cause even Venous bleeding
more harm. It is a consequence of injury to the vein. • determine if the wound was the result of
The blood is leaving the wound more intentional, unintentional or workplace
How to recognize arterial bleeding slowly and is not pulsating; it has a dark event
red color.
Hard Signs • examine nerves’ motor and sensorial
• No pulses
Capillary bleeding function, and tendons.

599
Assessment of bleeding wounds Eschmarch tourniquet Eschmarch tourniquet. It can limit
that are potentially life- or limb- breathing when applied on the thorax. Lift
We can use cuff from blood pressure
threatening the injured limb. Place clean gauze over
monitor. It should be inflated with
Direct pressure the wound and maintain direct pressure
pressure over 250mmHg, especially on
on the wound. Place one bandage over
We should provide equal pressure over a lower extremities. However, inflating
the wound and wrap the other on around
gauze that covers the complete wound. It 20-30 mmHg over the systolic blood
the limb. Make sure to have firm and
is the first step of immediate bleeding pressure levels are also acceptable in
constant pressure. Place the limb in a
control, and applicable anywhere on the most of the bleeding. Use only to stop
brace and keep it elevated. Check pulse,
body. Replace skin flaps to their original life-threatening exsanguination or when a
mobility, and sensation distal from the
position, before applying pressure if tourniquet is needed for a short period of
dressing. Check the dressing every 5-10
possible. time to create a bloodless field for wound
minutes.
inspection. This technique can be used id
Some areas of the body can be painful,
above measures are not effective to stop Clamping and Cauterisation
and it is a limitation for some patients.
a fast bleeding. It has a time limit up to 2
Direct pressure has time limitations. It is fast and on point bleeding control.
hours. It is a painful procedure. Apply
Therefore, application of pressure One of the final steps, if the above
blood pressure cuff proximal to the
bandage may be necessary. measures do not work to stop bleeding. It
bleeding point, inflate it above systolic
should not be applied any wound and
Pressure on arteries blood pressure and clamp the tubing with
vessel having an amputation and possible
a hemostat. After procedure record the
We can stop blood flow to extremities re-anastomosis chance. But it is ideal for
time of application, do a neurological
w i t h p r e s s u r e o n m a i n a r t e r i e s .
 continuously bleeding superficial arteries
exam and do not leave the tourniquet on
It is the second step of immediate in some wounds. Do not try to clamp
for more than 120 minutes.
b l e e d i n g c o n t r o l .
 deeper vessels because clamping may
It is useful only on extremities. It is a Compression bandage damage other structures.
painful application and has time limitation
It is very fast, but a temporary bleeding Cauterization is a final step of bleeding
as direct pressure.
control maneuver. It is less effective than control in the ED, and applied by surgical
600
teams when the other measures used by
ER team are not effective to stop
bleeding.

References and Further Reading, click


here

601
Section 5

Cardiac Monitoring

Case Presentation
by Stacey Chamberlain A 44-year-old male patient with a history of hypertension and
end-stage renal disease on hemodialysis presents with
shortness of breath after missing dialysis for 6 days. He
reported gradual onset shortness of breath associated with
orthopnea and increased lower extremity edema. He denies
chest pain or palpitations. He does not have any cough or
fever. On physical exam, he is in no distress, afebrile with a
heart rate of 60, respiratory rate of 20, blood pressure of
140/78 and oxygen saturation of 98% on room air. He has a
regular rate and rhythm without murmurs and has crackles
bilaterally to the inferior 1/3 of the lung bases and 1+ pitting
edema of the bilateral lower extremities.

You decide to get an EKG which shows the following (EKG


from www.lifeinthefastlane.com):

602
Image 16.4 ECG 1 What are the indications for include pulse oximetry, end tidal CO2
monitoring, central venous pressure
cardiac monitoring in this
monitoring, and continuous arterial blood
patient? What EKG pressure monitoring. Of note, telemetry is
abnormalities do you see? the ability to do cardiac monitoring from a
remote location; in practice, this is often a
What does the rhythm strip
centralized system that might be located
https://i2.wp.com/lifeinthefastlane.com/wp- show? What is the treatment? at a nursing station where multiple
content/uploads/2011/02/ECG-Potassium-7-
peaked-T-waves.jpg?ssl=1 patients can be monitored remotely.
Case discussion is at the end
of the chapter. Cardiac monitoring differs from a 12-lead
You send a blood chemistry
electrocardiogram in that it is done
test, place the patient on a continuously over a period of time rather
Introduction
cardiac monitor and one hour Cardiac monitoring in the emergency than capturing one moment in time in a

later note the following on the setting is continuous monitoring of a static image. The benefit of this, of
patient’s cardiac activity in order to course, is for capturing transient
monitor (EKG from
identify conditions that may require arrhythmias, ectopic beats, or monitoring
www.lifeinthefastlane.com): emergent intervention. These conditions for changes over time. A disadvantage of
include certain arrhythmias, ischemia and cardiac monitoring is that typically only 2
Image 16.5 ECG 2 infarction, and abnormal findings that leads are displayed instead of a full 12
could signal impending decompensation. leads, giving a less comprehensive view
This chapter focuses specifically on of the heart and limiting its utility to look
cardiac monitoring or for anatomic patterns. For example, on
electrocardiography. the 12 lead EKG, ED practitioners usually
group the inferior, anterior and lateral
https://i0.wp.com/lifeinthefastlane.com/wp- Additional methods of continuous
content/uploads/2011/02/disappearance-p- leads when looking for ischemic or infarct
waves-hyperk.jpg?ssl=1 hemodynamic monitoring in the ED patterns. These may be less evident on a

603
monitor with only two leads. Additionally, three classes. Cardiac monitoring is 3.Patients with unstable coronary
the static EKG allows for the ED considered indicated in “most, if not all” syndromes and newly diagnosed high-
physician to carefully study it for subtle patients in Class I, which includes 16 risk coronary lesions (for 24 hours)
fin d i n g s , f o r e x a m p l e , t o m a k e subcategories. In Class II, cardiac
4. Adults or children who have undergone
measurements of intervals, whereas, in monitoring “may be of benefit in some
c a r d i a c s u r g e r y

real-time monitoring, this is very difficult. patients but is not considered essential
(minimum of 48 to 72 hours)
I n p r a c t i c e , b o t h m o d a l i t i e s a re for all patients” and has 10
commonly used in conjunction for many subcategories. For Class III, cardiac 5. Patients who have undergone non-
ED patients. monitoring is not indicated. urgent percutaneous coronary
intervention with complications
The American Heart Association (AHA) Indications for Cardiac
published a consensus document in 2004 Monitoring 6. P a t i e n t s w h o h a v e u n d e r g o n e
establishing practice standards for Adopted from AHA consensus document implantation of an automatic
electrocardiographic monitoring in defibrillator lead or a pacemaker lead
hospital settings. This comprehensive Class I Indications and are considered pacemaker
document outlines the indications for Cardiac monitoring is considered
dependent
cardiac monitoring, the specific skills indicated in “most, if not all” patients in
required of the practitioner for cardiac Class I 7. Patients with a temporary pacemaker
m o n i t o r i n g , a n d s p e c i fic E C G or transcutaneous pacing pads
1. Patients who have been resuscitated
abnormalities that the practitioner should
from cardiac arrest 8. Patients with AV block
recognize.
2. Patients in the early phase of acute 9. Patients with arrhythmias complicating
Cardiac monitoring is essential for those
coronary syndromes (ST-elevation or Wolff-Parkinson-White syndrome with
patients who are at risk for an acute, life-
n o n – S T- e l e v a t i o n M I , u n s t a b l e rapid anterograde conduction over an
t h re a t e n i n g a r r h y t h m i a . T h e A H A
angina/“rule-out” MI) accessory pathway
guidelines divide indications for cardiac
monitoring in the inpatient setting into

604
10.Patients with long-QT syndrome and 2. Patients with chest pain syndromes Class III
associated ventricular arrhythmias Cardiac monitoring is not indicated
3. P a t i e n t s w h o h a v e u n d e r g o n e
11.Patients receiving intra-aortic balloon uncomplicated, non-urgent 1. Postoperative patients who are at low
counter-pulsation percutaneous coronary interventions risk for cardiac arrhythmias (e.g. young
patients without heart disease who
12.Patients with acute heart failure/ 4. Patients who are administered an
undergo uncomplicated surgical
pulmonary edema antiarrhythmic drug or who require
procedures)
adjustment of drugs for rate control
13.Patients with indications for intensive
with chronic atrial tachyarrhythmias 2. Obstetric patients, unless heart
care
disease is present
5. P a t i e n t s w h o h a v e u n d e r g o n e
14.Patients undergoing diagnostic/
implantation of a pacemaker lead and 3. P a t i e n t s w i t h p e r m a n e n t , r a t e -
therapeutic procedures requiring
are not pacemaker dependent controlled atrial fibrillation
conscious sedation or anesthesia
6. P a t i e n t s w h o h a v e u n d e r g o n e 4. Patients undergoing hemodialysis
15.P a t i e n t s w i t h a n y o t h e r
uncomplicated ablation of an (unless they have a class I or II
hemodynamically unstable arrhythmia
arrhythmia indication)
16.Diagnosis of arrhythmias in pediatric
7. Patients who have undergone routine 5. Stable patients with chronic ventricular
patients
coronary angiography premature beats

Class II Indications 8. Patients with sub-acute heart failure


Cardiac monitoring “may be of benefit in
“Must Know” Arrhythmias
One of the most critical skills of an ED
some patients but is not considered 9. Patients who are being evaluated for
physician is interpreting both static EKGs
essential for all patients.” syncope
and interpreting arrhythmias on a cardiac
1. Patients with post-acute MI (24 to 48 10.Patients with do-not-resuscitate monitor. A skilled practitioner must be
hours after admission) orders with arrhythmias that cause able to diagnose common arrhythmias
discomfort and be well versed in the management of

605
acute arrhythmias, recognizing which • Non-conducted atrial premature • Junctional ectopic tachycardia
arrhythmias necessitate immediate action beats
• Accelerated ventricular rhythm
and which are less worrisome. AHA
• Junctional rhythm
guidelines list  the specific arrhythmias • Ventricular
that the ED physician must be able to • AV blocks
recognize. • Monomorphic and polymorphic
• 1st-degree ventricular tachycardia
Specific Arrhythmias (adopted
• 2nd-degree Mobitz I • Torsades de pointes
from AHA Scientific Statement)
• Normal rhythms (Wenckebach) or Mobitz II
• Ventricular fibrillation
• Normal sinus rhythm • 3rd-degree (complete heart block)
• Premature complexes
• Sinus bradycardia • Asystole
• Supraventricular (atrial, junctional)
• Sinus arrhythmia • Pulseless electrical activity (PEA)
• Ventricular
• Sinus tachycardia • Tachyarrhythmias
• Muscle or other artifacts simulating
• Intraventricular conduction defects • Supraventricular arrhythmias

• Right and left bundle-branch block • Paroxysmal supraventricular Approach


tachycardia (AV nodal reentrant, How and whether to treat an arrhythmia
• Aberrant ventricular conduction AV reentrant) depends on many factors. The AHA has
established algorithms for specific
• Bradyarrhythmias • Atrial fibrillation
rhythms including ventricular fibrillation
• Inappropriate sinus bradycardia • Atrial flutter (v-fib)/pulseless ventricular tachycardia
(v-tach) and pulseless electrical activity
• Sinus node pause or arrest • Multifocal atrial tachycardia
(PEA)/asystole, as well as for non-specific
rhythm categories such as bradycardia
606
and tachycardia. Additionally, they have rhythm is considered a supraventricular picture and patient’s vital signs are of
published algorithms for clinical scenarios rhythm (originating above the ventricles). utmost importance in determining the
including cardiac arrest, acute coronary Supraventricular tachycardia is a generic management for these patients. A nice
syndrome, and suspected stroke. term encompassing any narrow-complex summary of this issue with rhythm strip
tachycardias originating from above the examples is provided on the FOAM site
The first step in the assessment of any
AV node. Colloquially, when many “Life in the Fast Lane.”
rhythm is a clinical assessment of the
practitioners refer to “SVT” however, they
patient. The premier issue of concern is if While each rhythm has distinctive
are actually referring to a specific
the patient is perfusing vital organs. A management, it is worth noting for the
subcategory of supraventricular
quick survey of the patient assessing novice learner that only v-fib and
tachycardia called AV nodal re-entrant
mental status and pulses is essential to pulseless v-tach warrant asynchronized
tachycardia (AVNRT). Wide complex
determining management. The mechanical defibrillation (i.e., “shocking”
tachycardias either originate in the
management of a patient with v-tach will the patient). Many students are stunned
ventricles or could originate in the atria
be substantially different if the patient is upon observing an asystolic cardiac
and have an associated bundle branch
unresponsive and pulseless versus if the arrest code to learn that shocking a
block. Different criteria have been
patient is awake with good pulses. As “flatline” (i.e., asystolic) patient is an
developed to help the practitioner
another example, the physician can inappropriate treatment perpetrated by
distinguish between ventricular
quickly distinguish artifact from v-fib on fictitious TV shows and movies. For
tachycardia and an SVT “with
the cardiac monitor by assessing the unstable patients with arrhythmias in
aberrancy” (i.e., aberrant conduction
patient, as v-fib is not a perfusing rhythm. patients who still have palpable pulses,
either due to an accessory path such as
synchronized cardioversion may be used.
The initial assessment of in Wolff-Parkinson-White or with a bundle
tachyarrhythmias (heart rate > 100) is to branch block), the most well known of In regards to medications, for certain
determine if the rhythm is “narrow- which are the Brugada criteria. Practically rhythms and clinical scenarios, only
complex” (i.e., QRS duration < 0.12s) or speaking, many ED practitioners will vasopressor types of medications are
“wide-complex” (i.e., a QRS duration of assume the more dangerous and used (e.g., epinephrine for asystole). For
0.12s or greater). A narrow complex potentially unstable rhythm (v-tach) until other rhythms and scenarios, anti-
proven otherwise; of course, the clinical
607
arrhythmic medications are used (e.g., Some useful internet resources for the ED (unless an immediate point of care
amiodarone for v-tach). For practitioner are provided at the end of tests are available). If the patient’s
supraventricular tachyarrhythmias, this chapter for practice interpreting hyperkalemia progressed, the patient
atrioventricular (AV) nodal blocking EKGs and cardiac rhythms. could develop QRS widening with the
agents are often necessary. One author morphology as shown on the rhythm strip
suggests using a five “As” approach to Case Discussion called a “sine wave.” This dangerous
treating emergency arrhythmias, keeping The ED practitioner should recognize that finding could precipitously deteriorate
in mind the medications adenosine, the potentially life-threatening conditions into a life-threatening arrhythmia such as
amiodarone, adrenaline (epinephrine), that a patient who has missed pulseless v-tach with cardiac arrest and
atropine and ajmaline. Ajmaline is an hemodialysis is at risk for are fluid should prompt immediate action. It is
antiarrhythmic that is not commonly used overload (leading to pulmonary edema) important to note that hyperkalemia can
in English-speaking countries where and hyperkalemia. This patient could be manifest in a variety of different EKG
procainamide is more common as an considered to meet the Class I monitoring findings and does not always follow a
alternative to amiodarone for unstable v- criteria for “needing intensive care” and consistent pattern from peaked Ts to
tach. possibly with “pulmonary edema;” QRS widening to sine waves; therefore,
however, even if the patient had no the patient should be treated at the first
Additional interventions may include symptoms, the patient is at risk for an indication of any hyperkalemia-related
pacemaker placement for symptomatic acute life-threatening arrhythmia that EKG changes.
heart blocks and in many cases, would necessitate cardiac monitoring.
determining the underlying precipitant of
the arrhythmia and tailoring treatment to The EKG demonstrates peaked T waves
indicative of acute hyperkalemia. Given References and Further Reading, click
that cause. The emergency physician
the clinical picture of missed dialysis and here
must familiarize himself with each rhythm
and its unique management in any given the peaked Ts on the EKG, the ED

clinical scenario. physician should immediately initiate


treatment for acute hyperkalemia without
waiting for a confirmatory blood test

608
Section 6

Gastric Lavage and Activated Charcoal Application

Case Presentation
by Elif Dilek Cakal A 22-year old female presented to the emergency department
15 minutes after she had committed suicide by taking 30 pills
of 500 mg acetaminophen. She had no known chronic
diseases. Her blood pressure was 134/87 mmHg;
temperature, 36.4°C; heart rate of 70 bpm and regular;
respiration 15 bpm; and O2 saturation 99%. At the time of
arrival, she was asymptomatic. Nothing was remarkable on
examination. Gastric lavage was performed. 1 mg/kg of
activated charcoal was given to the patient. IV N-
acetylcysteine treatment was started. She was admitted to the
hospital.

609
Gastric Lavage Procedure • Hydrocarbons intake (unless containing •Local analgesics and lubricants
highly toxic substances such as
Emergency Indications • Intubation equipment
pesticides).
Gastric Lavage (GL) should not be
• Sedatives (if necessary)
undertaken routinely. Whether gastric • Oral intake of caustic substances.
lavage positively alters the morbidity or • Restraints (if necessary)
• Poisonings with toxic substances;
mortality of the poisoned patient, even
those are more toxic to lungs than to • Bite block or oral airway
applied shortly after the intake, is
gastrointestinal system.
controversial. GL is indicated only if: • Oral or nasogastric tubes
• Poisonings with pills that are known not
• Oral intake < 60 minutes • 36- to 40-French or 30 English-
to fit through the holes of the gastric
tube gauge tubes in adults (oral)
• The life-threatening dose of the toxic
substance is ingested • 24- to 28- French-gauge in
• Known esophageal structures.
children (oral)
Contraindications • History of gastric bypass surgery.
• Patients with compromised airway • Lavage systems
reflexes, unless they are intubated. If Emergency Physician (EP) must be
the critical situation of the patient cautious in combative patients and • Commercially available
indicates intubation, then, gastric patients with medical conditions such as
• Intermittent aliquots of lavage fluid
lavage may be performed. Intubation, bleeding disorders.
can be given and withdrawn
only for decontamination, is not
manually
recommended. Equipment and Patient
Preparation • Activated charcoal (see below)
• Non-toxic or non-life-threatening
intoxications. Equipment for GL includes: • Normal saline or water
• Intravenous access and monitoring
Before starting, the steps of the
• A large suction catheter procedure must be explained to patients

610
in an attempt to gain cooperation. If the This video explains the steps of the 9. 9. Continue lavage until the
patient is too agitated, sedatives in insertion of the gastric tube fluid becomes clear
anxiolytic doses may be used. EP must
1. Explain the procedure to the patient. 10.Administer activated charcoal via tube
keep in mind that significantly altered
level of consciousness due to sedation 2. Collect the equipment and place the 11.Clamp off and remove the tube
warrants intubation. patient in the left lateral decubitus
position. Hints and Pitfalls
Although there is no adequate data in
• The procedure is intended to be
humans to show that tube diameter or 3. Put a bite block or oral airway into the therapeutic, not punitive.
route is important, the oral route is patient’s mouth.
primarily preferred for the gastric lavage. • In some situations, Gastric lavage may
Nasogastric tubes are less traumatic for 4. Introduce to pass the tube gently be helpful for up to 2 hours:
patients and are preferred in liquid
5. When the pharynx is reached, put the • Highly toxic drugs
ingestions and children.
patient’s chin on the chest to facilitate
passage of the tube into the • Drugs not absorbed by activated
Place all patients in the left lateral
esophagus. charcoal
decubitus position in Trendelenburg to
facilitate the content removal and to • Sustained release or enteric-
6. Confirm the placement
decrease the aspiration risk. Supine coated products
position greatly increases aspiration risk, 7. Aspirate and remove the gastric
unless the patient is intubated. contents before gastric irrigation • Auscultation of the stomach generally
confirms the placement of the tube
The tube must be measured from the 8. Repeatedly introduce 200–300 mL of during injection of air with a 50-mL
corner of the mouth to the mid- lavage solution (10 mL/kg body weight syringe and aspiration of gastric
epigastrium in order to avoid kinking and in children up to a maximum of 300 contents. Radiographic confirmation
complications. mL) into the stomach and then remove should be considered, especially in
them children and intubated patients.
Procedure Steps
611
• A cough, stridor, or cyanosis indicates • Esophageal lacerations or perforation. Elderly patients are susceptible to
that the tube has entered the trachea; cardiac consequences of both procedure
• Gastric perforation.
withdraw the tube immediately and and the poisoning; therefore, their vital
reattempt passage. • Fluid and electrolyte disturbances, signs should be monitored closely.
especially in children.
Post Procedure Care and Gastric lavage and activated charcoal are

Recommendations • Hypothermia. considered safe for pregnant patients.


• For most patients, a short period of Poisonings that are toxic to the fetus as
• Nasal, oral, pharyngeal, pyriform sinus well as toxic to mother must be
observation of vital signs is adequate.
injuries. considered.
• The nature of the poisoning will lead the
• Pulmonary hemorrhage, pneumothorax,
management. Activated Charcoal
and empyema.
Application
Complications
GL is generally safe but not harmless. Pediatric, Geriatric, and Emergency Indications
Pregnant Patient • Oral intake < 60 minutes
Complications of GL include: Considerations
Gastric lavage is always a difficult • the life-threatening dose of the toxic
• Misplacement of the gastric tube into substance
procedure to apply to children.
the trachea.
Nasogastric tubes may be preferred. 10
Multi-Dose Activated Charcoal
• Pulmonary aspiration of gastric content mL/kg aliquots of lavage solution up to a
(MDAC) Indications
of lavage fluid, especially in patients maximum of 300 mL is given and
Life-Threatening Oral Intake of
with compromised airway reflexes. removed. Because electrolyte
disturbance has occurred in children who • Carbamazepine
• Aspiration pneumonia. were lavage with tap water, prewarmed
• Dapsone
• Laryngospasm and hypoxia, especially (45°C) normal saline is generally

in patients with lung diseases. recommended for children. • Phenobarbital

612
• Quinine for decreased peristalsis (relative •Give the  recurrent dose of charcoal
contraindication) by 0.5 g/kg (≤50 g) every 4 hours
• Theophylline
Equipment and Patient How to administer:
Contraindications
• For patients with compromised airway
Preparation • If the patient is awake and cooperative,
There is no specific equipment for
reflexes, unless they are intubated. If AC may be given orally. Alternatively, it
activated charcoal administration.
the critical situation of the patient may be given by gastric or nasogastric
However, drinking the charcoal can be
indicates intubation, then, gastric tube, if these procedures are indicated.
very unpleasant for many patients,
lavage may be performed. Intubation,
especially children. Therefore, mixing with • Mixing the activated charcoal with fruit
only for decontamination, is not
fruit juice can be an option. In addition, if juices increases tolerability.
recommended.
necessary nasogastric or orogastric tube
• If the patient is unconscious or airway is
• Oral intake of caustic substances placement can facilitate the active
compromised, gastric lavage should be
charcoal treatment.Procedure Steps
• Late presentation done, and activated charcoal should be
Procedure Steps g i v e n a f t e r i n t u b a t i o n . Tr a c h e a l
• Increased risk and severity of aspiration
Recommended empirical single-dose of intubation is not recommended solely in
associated with AC use (e.g.,
activated charcoal is as follows: order to give activated charcoal. Only
hydrocarbon ingestion)
activated charcoal is to be given, the
• <1 year – 0.5-1 g/kg or 10-25 g nasogastric tube is adequate and is
• Need for endoscopy (e.g., significant
caustic ingestion) preferred.
• 1-12 years – 0.5-1 g/kg or 25-50 g

• Toxins poorly adsorbed by AC (e.g., • If MDAC is indicated, the gastric tube


• >12 years – 1-2 g/kg or 25-100 g
metals including iron and lithium, alkali, should be withdrawn after gastric
mineral acids, alcohols) lavage and the first dose of activated
charcoal. Further doses should be given
Multidose activated charcoal
• Presence of intestinal obstruction via nasogastric tube.
(absolute contraindication) or concern
613
Hints and Pitfalls • Disopyramide •Look for traces of aspiration or
• The substances that cannot bind to gastrointestinal complications.
• Nadolol
activated charcoal are as follows:
• Phenobarbital
Complications
• Lithium Complications of AC and MDAC include:
• Phenytoin
• Strong acids and bases • Constipation, diarrhea, vomiting
• Piroxicam
• Metals and inorganic minerals • Pulmonary aspiration
• Quinine
• Alcohols Pediatric, Geriatric, and
• Sotalol Pregnant Patient
• Hydrocarbons
• Sustained-release thallium
Considerations
• Multi-dose activated charcoal enhances In pediatric and geriatric patients, extra
elimination of (But not necessarily • Theophylline caution should be exercised to avoid and
indicated in all) monitor complications.
• Valproate
• Amitriptyline Activated charcoal is considered safe for
• Vancomycin
pregnant women.
• Aspirin
• MDAC increase the risk of constipation
• Caffeine and bowel obstruction in some cases.
Therefore, consider adding a cathartic References and Further Reading, click
• Carbamazepine
agent to the  second or third dose of here
• Cyclosporine AC.

• Dapsone Post Procedure Care and


Recommendations
• Digoxin
• Control possible nausea and vomiting.

614
Section 7

Intravenous (IV) Line Access

Introduction
by Keith A. Raymond Peripheral Intravenous (IV) cannulation is a nursing skill. Few countries throughout
the world require physicians to perform this procedure on a regular basis. Mastery
of technique, understanding nuances and anatomy, and daily performance are
required to maintain this skill. Therefore, if a nurse reports that he is unable to
obtain IV access, and it is required urgently, establishing an IV access or
intraosseous (IO) line should be considered to avoid delay.

IV lines can safely remain in place safely for up to 72 hours. In some cases, this is
up to 7 days.

“There is no body cavity that cannot be reached with a


number fourteen needle and a good strong arm.”

― Samuel Shem, The House of God

IV Line Access and Procedure


Success rates in multiple attempts for admitted patients at a children’s hospital
range from 23% for physicians, 44% for nurses to 98% for IV nurse clinicians. The
average time required for peripheral IV cannulation is reported at 2.5 to 13
minutes, with difficult IV access requiring as much as 30 minutes. Therefore, we
will focus here on peripheral IV cannulation and line access for the easiest and

615
most commonly used sites in • Any site where there is a concern for • Topical anesthetic, eg. EMLA
emergencies, as we must provide high vascular flow. ( 2.5% lidocaine and prilocaine),
volumes and medications to the patient
Equipment and Patient • transilluminator light,
quickly.
Preparation
• ultrasound with a vascular probe.
Emergency Indications
Intravenous access is used when
Equipment
• gloves,
therapies cannot be used or are less
Illustration 16.1
effective by alternative routes. In critical • skin disinfectant (Povidine and Alcohol
situations, medication bioavailability, Swabs),
hydration, and blood products can be
given and provide rapid onset of action. • 16-18 gauge IV catheter (smaller

Peripheral access is typically safer, easier catheters may be used for pediatric

to obtain, and less painful than central patients, but larger is better in critical

a c c e s s . F i n a l l y, t w o l a r g e b o r e cases), (see Illustration 16.1)

intravenous catheters in place can


• tape,
provide the same or more fluids during
resuscitation as a central line. • syringe,

Contraindications • 3-way stopcock,


• Patients with anatomic disparities that
• isotonic crystalloid solution,
could lead to fluid or medication
extravasation locally or proximally. • intravenous tubing,

• Massive edema in extremities, burns, • an elastic tourniquet or blood pressure


cellulitis, or injuries at or proximal to (BP) cuff.
proposed insertion sites.
• Optional

616
Patient Preparation but prevent contamination of the clean 6.Attach the 3-way stopcock, then
• Obtain informed consent or implied, prepped site to be accessed. flush the stopcock and cannula of
f o l l o w i n g p ro c e d u re d i s c u s s i o n ,
 blood with 5 ml of saline to prevent
risks, and benefits. Procedure clotting, and assess the flow of fluid
1. Apply the tourniquet or BP cuff (inflate through the catheter. Watch for skin
• If possible, have the patients wash their above diastolic reading) proximal to bulge suggesting extravasation of fluid.
forearms, including the antecubital the intravenous site.
space, three times with soap and water, 7. Secure the catheter with tape and
2. Using ‘no-touch’ technique, insert the release the tourniquet or BP cuff.
then pat dry.
IV catheter distal to and along the line
• Select the site starting distally, preferred of the vein at a 10 to 15-degree angle 8. Attach intravenous tubing to 3 way
Cephalic vein in the forearm, then to the skin. stopcock, attached to the fluid of
Medial Brachial Vein in Antecubital choice and initiate flow, watching again
3. Advance the needle and the catheter for fluid extravasation. Medications
Sulcus.
slowly; in most cases, a ‘flash’ of may be administered through another
• Always apply universal precautions blood will enter the catheter (but not port of the stopcock or added to the IV
(gloves as a minimum) to the procedural always). solution as desired.
list. Both visualize and palpate the vein
4. SLOWLY advance the needle an 9. Make sure that you removed the
to be cannulated.
additional 1 to 2 millimeters, then slide tourniqet before you give drug or fluid
• There is a slight give to the vessel the cannula into the vein, while infusion.
compared to surrounding tissue. securing the needle in place.
10.If fluid extravasation occurs at any
• Disinfect overlying skin, and provide 5. Remove the needle while pressing on time, remove the catheter, and repeat
topical anesthetic to site as desired. the overlying skin over the cannula the procedure at the more proximal
proximal to the insertion site to stem site (never distal to the previous
• Transillumination and/or ultrasound may
the blood flow. attempt).
be used to provide additional guidance,

617
Please watch the video. • Use an arm board in pediatric patients,
to prevent catheter displacement from
Post Procedure Care movement.
• All medications administered should be
followed by a 20 ml saline flush. • D o n o t u s e fl a s h l i g h t s f o r
transillumination as they can burn skin,
• A three-way stopcock should remain use transilluminator only. Lowering the
attached to the IV line if it is not in room light during transillumination
active use. maximizes visualization.

• Clean surrounding skin of blood and • Following two failed attempts, seek
other contaminants following insertion. assistance and/or switch to
an Intraosseousline.
• All IV catheters should be removed
within 7 days or as soon as no longer
Complications
necessary.
• Thrombosis and Hemorrhage
• Be vigilant during infusions for tissue
• Air embolism
swelling or catheter displacement.
• Extravasation of Drugs
Hints and Pitfalls
• Palpation is more important than • Vasculitis and Contusions
visualization.

• Secure vein proximally and distally from


References and Further Reading, click
the insertion site if dealing with a ‘roller’
here
vessel.

618
Section 8

Intraosseous (IO) Line/Access

Introduction
by Keith A. Raymond Peripheral Intravenous (IV) cannulation is a nursing skill. Few countries throughout
the world require physicians to perform this procedure on a regular basis. Mastery
of technique, understanding nuances and anatomy, and daily performance are
required to maintain this skill. Therefore, if a nurse reports that he is unable to
obtain IV access, and it is required urgently, establishing an IV access or
intraosseous (IO) line should be considered to avoid delay.

Following medicine delivery and fluid resuscitation utilizing an IO line, transition to


peripheral intravenous or central intravenous access is easier to achieve, and the
intraosseous line may be discontinued.

Intraosseous lines can safely remain in place for up to 24 hours and are often a
bridge to either IV or Central Venous line placement.

Intraosseous Line Access and Procedure


Emergency Indications
When IV access cannot be achieved, IO access is safe, reliable, and quick. It can
be accomplished in 30 to 60 seconds and even faster with an IO gun. This is
especially helpful in pediatric emergencies when time is critical. Almost anything
that can be given IV such as medications, fluids, blood products and continuous

619
infusions of catecholamines (epinephrine, • skin disinfectant (Povidine or • Yellow (45 mm) for large patients
norepinephrine, and dopamine). Chlorhexidine and Alcohol Swabs), or dense bone sites such as
proximal humerus or anterior
Contraindications • 16-18 gauge IO or Jamshidi-type
superior iliac spine.
Absolute: needle,
Patient Preparation
• fracture or crush injuries near or • tape,
• Obtain informed consent or implied,
proximal to the access site,
• syringe, following procedure discussion, risks,
• fragile bone conditions such as and benefits.
• isotonic crystalloid solution, and
Osteogenesis Imperfecta,
intravenous tubing. • Select site: humeral head, proximal
• previous attempts in the same bone, tibia, medial malleolus, sternum, distal
• Optional: radius, distal femur, and/or anterior
• the presence of infection in or on the superior iliac spine. (see illistration 16.2)
• IO drill or gun, Infusion pump,
overlying tissue of the bone,
• 2% Lidocaine for topical and • Proximal Tibia and Humeral Head are
• demineralized or immature bone. most commonly used during arrests as
subcutaneous infiltration (awake
patients tend to report pain with placement does not interfere with
Relative:
fluid infusion rather than insertion). intubation or other activities.
• IV access can be obtained readily.
• NOTE: Color coding of IO needles is • Always apply universal precautions
• Use for only ultra short-acting (gloves as a minimum) to the procedural
common
medications such as Adenosine. list.
• Pink (15 mm) for patients 3 – 39
Equipment and Patient kg,
Preparation
• Blue (25 mm) for patients 40 kg
Equipment and greater,
• gloves,

620
Illustration 16.2 Procedure 8.If properly positioned, the needle
1. Once the patient is prepared, identify will stand without support and be fixed
the designated site with a sterile in place.
gloved finger.
• Remove the stylet and attach the
2. Disinfect overlying skin, and provide syringe and aspirate, marrow and
local anesthetic as desired. blood confirms placement but
may not always appear.
3. Be sure the stylet is in place on the
needle prior to insertion. 9. Gently flush saline through the needle
and watch the insertion site for
4. Have a 20 ml Saline syringe flush, IV swelling.
tubing, tape, medications, fluids, and
pump prepared, as required. 10.If the test injection is unsuccessful or
swelling is seen on the opposite side
5. Place the needle through the skin, of the bone, repeat the above
perpendicular and down to the bone. procedure with a new IO needle on

6. Activate the IO drill or gun until the IO another bone.

needle anchors in place, OR manually 11.If successful, stabilize the needle with
TWIST the needle clockwise (don’t the tape; gauze padding may be used
push) with gentle firm pressure until as desired.
the bone gives (loss of resistance
technique) and the needle locks into 12.Attach the IV tubing to the needle hub
place. and infuse fluids, blood products, or
medications.
7. The bone give is an indication the
needle has passed through cortical Video – Intraosseous Needle Line
bone into the marrow. Insertion

621
Video – Intraosseous Needle Line • IO needle selection should be
Insertion in A Real Patient consistent with the site and marrow
cavity.
Post Procedure Care
• All medications administered should be • IO needle displacement sometimes can
followed by a 20 ml Saline flush. be avoided by properly securing it to
the skin.
• A three-way stopcock should be
attached to the IO line if it is not in Complications
active use. • Bone fracture

• All IO needles should be removed within • Compartment Syndrome


24 hours or as soon as an IV or Central
• Extravasation of Drugs
line is placed.
• Osteomyelitis
• Be vigilant during infusions for tissue
swelling or needle displacement.

Hints and Pitfalls References and Further Reading, click


• Always used an uninjured limb; if none here
available, the sternum is best.

• An IO drill or gun should be used


preferentially to manual insertion

• In pediatric patients, if the bone is too


soft, needle displacement is inevitable
despite proper placement. Select
anterior superior iliac spine.

622
Section 9

Emergency Delivery

Case Presentation
by David F. Toro, Diana V. Yepes, Ryan H. As you begin the morning of your next weekend day shift in a
Holzhauer
small community hospital, the triage nurse comes in running
and asks you to evaluate a patient that is being registered in
the Emergency Department. You find a visibly pregnant 29-
year-old female patient complaining of having regular uterine
contractions for the last 10 hours and passed a significant
amount of clear liquid per vagina on the way to the hospital,
as well as a sensation of pelvic fullness and an increasing urge
to use the bathroom with every uterine contraction.

On your evaluation, you find the patient is having uterine


contractions at regular intervals, 4 times on a 10-minute
period, lasting around 3 minutes each. You are able to detect
a normal fetal heart rate and fetal movements. On the pelvic
exam, you find a fully effaced and dilated cervix and palpate
the fetal head at the level of the ischial spines. Only at this

623
moment, you remember your Introduction Identifying True Labor
Every year around 4 million babies are Labor is the process by which the fetus is
hospital does not have a
born in the US; unfortunately, there is no expelled from the uterus and can be a
gynecologist in-house, and information on how many of these are lengthy process on nulliparous women
your nearest transfer center is 1 born outside the regular delivery units, but becomes a shorter process on

hour away. What would you do including the Emergency Department. subsequent pregnancies. It begins when
Fortunately, however, it is an uncommon an organized uterine activity starts,
next?
occurrence in Emergency Medicine. Just causing gradual effacement or thinning of
as it applies to many other emergency the cervix and dilatation in order to allow
procedures, the Emergency Medicine passage of the fetus during the final
provider needs to be familiar with the stages.
normal vaginal delivery. The provider
The labor process can be divided
must know preparations for it as well as
into  latent  and  active phases. The latent
how to identify and treat immediate
phase begins when there is organized
complications for those cases where
and regular uterine activity causing a
immediate access to an obstetrician is
cervical dilatation and effacement; it is
not readily available or if delivery is
considered active phase when it causes 3
imminent before arrival to a birthing unit,
or more cm dilatation and/or effacement
such as in a patient arriving late to the
of 80%.
hospital or a precipitous delivery.
The active labor is normally divided into 4
This chapter describes the evaluation of
stages. The  first stage  concludes when
the patient in possible active labor, the
dilatation and effacement are complete.
normal delivery technique, and immediate
The second stage ends when the fetus is
post-delivery care.
delivered, the  third stage  ends when the
placenta is delivered, and the 4th stage is

624
the approximate period of 1 hour after the but may not occur until the moment of “Bloody show” is the common name
third stage concludes. delivery. given to the expulsion of the blood-tinged
cervical mucus plug as effacement and
Contractions occur since the 2nd Although Vaginal pH changes during
dilatation occur. Although by definition it
trimester as  Braxton-Hicks contractions, pregnancy, normal vaginal fluid tends to
is always present, in practice, it may not
but they become more common as the have an acidic pH (4.5-6.0) where as
be noticed as it can occur gradually
3rd trimester goes by, transforming amniotic fluid is alkaline (pH 7.0-7.5).
instead of all at once. When noticed, it
gradually into active labor. Braxton-Hicks Therefore, another way of identifying
can precede the initiation of active labor
contractions tend to be limited to the amniotic fluid is using nitrazine or pH
by several days.
suprapubic area and thighs, are short and paper. Under acidic environment, this
irregular in duration, have a low strength paper changes color from yellow to Initial Examination
and are sporadic in timing. orange, and when amniotic fluid is In order to plan ahead for the imminent
present, it changes from orange to yellow, delivery, it is important to perform a
True labor contractions, in contrast, are
green or blue (Image 16.6). vaginal and abdominal exam to determine
progressively longer in duration, radiated
the fetal well-being, lie, position,
to the back and pelvic area, occur at
presentation, dilatation, effacement, and
regular intervals that become more Image 16.6 Under the presence of station.
frequent, are progressively stronger and amniotic fluid, nitrazine paper turns
cause effacement and dilatation of the from orange to yellow, green or blue.
Abdominal exam
cervix. The Leopold maneuvers are part of the
abdominal exam.
Other signs that indicate true labor are
rupture of membranes and “bloody • First, palpate the uterine fundus to
show.” The spontaneous rupture of determine if the fetus is in a vertical or
membranes manifests by a sudden gush transverse lie by feeling if the fetal pole
of clear fluid or by continuous leakage of represents the head, breech (buttocks)
vaginal fluid, with bleach or semen smell, or back.

625
• Second, apply pressure to the sides of In the situation of imminent delivery, there contraction are called late
the uterus with the entire hand, being is little use for advanced fetal monitoring decelerations and constitute a sign of
sure to utilize both hands, to determine in the ED. Nevertheless, an initial fetal distress or placental insufficiency.
where the spine and extremities are. assessment of the fetal well-being is
Decelerations occurring at any moment
appropriate if time allows. The most basic
• Third, with your dominant hand index without relation to the contractions are
way to assess the fetal wellbeing is by
and thumb, palpate just above pubic called variable decelerations and
listening to the fetal heart rate (FHR). This
symphysis to locate the presenting part represent an indication of umbilical cord
can be done by auscultating with a
and determine if it is engaged on the compression or umbilical cord prolapse.
stethoscope, Doppler US or bedside
pelvis. If the presenting part is movable,
ultrasound, placed on the mother’s In the case of late or variable
it is not yet engaged. If it is not
abdomen and in the area where the fetal decelerations, the patient should be given
movable, it is engaged.
thorax is located. oxygen, IV fluid bolus, placed on lateral
• Forth, while facing the maternal legs decubitus and immediate OB
The normal fetal heart rate is 110-160
from the abdomen palpate, enter the consultation should be obtained as
BPM and should be measured over 2
presenting part with both hands moving immediate emergency delivery may be
minutes, as it is normally variable. Higher
towards the birth canal while applying indicated.
rates represent fetal distress.
deep pressure. When the head is the
Decelerations on FHR can be normal or
presenting part, you will feel a round Vaginal Exam
abnormal. The effacement, dilatation, station, and
prominence in one of your hands. If this
position should be determined. On
cephalic prominence is on the same Decelerations occurring during the
vaginal exam, while using lubricated
side as the back and spine, the fetus is uterine contractions are called early
sterile gloves, locate the cervix and the
in face presentation. If the prominence decelerations and are due to the vagal
presenting part. Palpate the cervix to
is on the same side as the small parts, response to the compression of the fetal
determine effacement and dilatation,
the fetus is on vertex presentation. head on the mother’s pelvis.
palpate the presenting part to locate
Fetal Monitoring Decelerations occurring towards the end anterior and posterior fontanel, chin or
of the contractions and peaking after the
626
sacrum and locate the ischial spines and Illustration 16.3 Effacement stages permeable to 2 fingers is considered
determine the station. dilated to 3cm, which is considered
active labor.
The position is the relation of the occiput
or posterior fontanel in relation to the Station  is the level of the presenting part
maternal pelvis. If the fetus is presenting in relation to the ischial spines. This
breech, the sacrum is used as fetal measurement is done by palpation, where
reference, and if it is presenting face, the the ischial spines are at 8- and 4-o’clock
chin is the point of reference. The most on the vaginal canal. It is also described
common presentation and what is in centimeters where 0 is at the level of
considered normal is left/right occiput the ischial spines, negative numbers (-1,
anterior. -2, -3, -4, -5) are above and positive
numbers (+1, +2, +3, +4, +5) are below
Effacement  is the progressive thinning
the spines. (Illustration 16.4).
and shortening of the cervix that occurs
slowly during early labor and
progressively faster during active labor. It Illustration 16.4 Stations
may occur simultaneously with dilatation,
especially on multiparous women. It is
measured qualitatively from 0% (long and
rubbery) to 100% (very thin and soft) by
palpation of the cervix. (Illustration 16.3 2)

Cervical  dilatation  is the measurement of


the cervical os diameter, expressed in
centimeters. 10cm is considered full
a. Normal long and thick cervix, b. Shorter and
dilatation. A cervix permeable to 1 finger slightly thinner cervix at around 50% effacement,
is considered dilated to 1cm and if it’s c. Fully effaced cervix

627
Fetal Movements During Engagement: During this stage, the bi- External rotation: The head returns to

Labor parietal diameter passes through the an anatomic position in relation to the
As the fetus descends on the birth canal, pelvic inlet and is considered engaged rest of the fetal torso. The head returns
several movements occur as a when the head reaches station 0. On now to a transverse position, just as
mechanical process where the fetus primigravid patient, this movement during engagement, while the fetal
follows the path of least resistance, occurs in the last 2 weeks of pregnancy, shoulders are passing between the ischial
adapting the position of the presenting but on multiparous patients, it may occur spines.
part to the dimensions of the birth canal when labor begins.
Expulsion: During this stage, the rest of
and producing the following movements:
Flexion: During this stage, the fetus neck the fetal body is born. The shoulders
(Illustration 16.5)
is flexed to present a shorter diameter on continue descending on an oblique
the pelvis. position as they finalize their descent on
Illustration 16.5 Fetal movements the pelvis and are delivered – first the
Internal Rotation: This occurs as the
anterior shoulder and then the posterior
presenting part crosses the ischial spines.
one at the level of the perineum. The fetal
At this point, the relative transverse
pelvis is the smallest of the large fetal
position on the head moves back to the
diameters and descends on the maternal
original occiput anterior position.
pelvis following the same path and is
Extension: The occiput reaches the delivered all at once, in contrast to the
vaginal introitus and passes under the fetal head and the shoulders.
Fetal positions for delivery. 1. Cephalic fetal symphysis pubis. During this stage, the
presentation before labor, 2. Engagement, 3.
head is born from the occipital area, the
The Delivery Procedure
Flexion, 4. Internal Rotation, 5. Extension, 6. The following materials are typically used
External rotation, 7. Expulsion of anterior bregma, forehead, nose and finally chin,
shoulder, 8. Expulsion of posterior shoulder. for a normal vaginal delivery procedure
at the perineal area of the vaginal
(does not include equipment for neonatal
introitus.
resuscitation):

628
• 0-0 absorbable (Chromic catgut or • Tissue Forceps provide additional space for the
undyed Vicryl ) suture material delivery maneuvers.
• Tissue scissors
• Basins The following videos describe and
• Umbilical cord clamp
illustrate a step by step guide to the
• Kelly clamps
Before you begin the procedure, as there normal vaginal delivery procedure.
• Light source is a high risk of exposure to body fluids,
This video demonstrates the hand
remember to wear sterile gloves, mask
• Long needle driver technique for spontaneous vaginal
with eye protection, waterproof sterile
delivery on a simulated environment.
• Mask with face shield gown, and shoe covers.
This video shows the baby’s process on a
• Material scissors Apply iodine solution to the perineal area
virtual simulation.
and clean with sterile water, then apply
• Povidone-Iodine solution sterile drapes to the patient’s thighs and Once the fetus has been delivered,
abdomen. It is likely that stool is expelled carefully place him/her on the sterile
• Shoe covers
during the birthing process; so, additional drape on the mother’s abdomen and
• Sterile drapes and towels sterile drapes should be available to stimulate while drying with sterile towels
prevent fecal contamination of the baby or gauze.
• Sterile gauze
or the perineal area.
After drying and stimulating, clamp the
• Sterile gloves
The ideal position of the patient is on a umbilical cord about 1 in or 3 cm from the
• Sterile lubricant gel birthing table with stirrups and lithotomy newborn’s abdominal skin using an
position. If this is not available, additional umbilical clamp, Kelly clamp with rubber
• Sterile waterproof gown personnel may help the patient maintain ligature or fabric ligature. Then place a
the knees flexed and the hips abducted. Kelly clamp about 1 in from the umbilical
• Suction device or bulb syringe
If a regular stretcher is used, it is helpful clamp and use the scissors to cut the
• Syringes (10-20mL), and needles (22-24 to place folded sheets or an inverted umbilical cord between the two clamps.
gauge) bedpan to elevate the patient’s pelvis and
629
Obtain a blood sample from the placental Once the placenta is delivered, inspect it During delivery of the head, gentle
end of the cord for neonatal testing.
 to ensure it was delivered completely, and upward pressure with a sterile towel or
At this moment, follow the neonatal there are no remaining parts in the uterus. drape to prevent anal contamination on
resuscitation guidelines, calculate the the perineal area helps elevate the
After delivering the placenta, inspect the
initial APGAR scale and wrap the presenting part and decrease the
cervix, vaginal mucosa and the perineum
newborn to prevent hypothermia. Then, pressure the fetal chin exerts on the
for tears that may need to be repaired.
place the newborn under radiated heat. perineal skin.
The main mechanism for hemostasis after
Immediate Post-Delivery Immediately following delivery of the
the placenta has detached is uterine
Care head, palpate the fetal neck to inspect for
muscle contraction over the blood
Delivery of the placenta occurs up to 30 umbilical cord encircling the neck. This
vessels, so an infusion of oxytocin,
to 40 minutes after fetal delivery, and it is, cord needs to be reduced over the fetal
ergonovine or methylergonovine may be
for the most part, a passive process. head before delivery can continue.
given to aid in the process. Oxytocin
Once you see a slight increase in vaginal
(Pitocin) is the most commonly used As with any other procedure, don’t stand
bleeding and the remaining umbilical cord
agent. Add 20 units to a 1 Liter Normal too close to the patient as fluids may be
protrudes slightly, ask the mother to bear
Saline bag and infuse at 10 mL/min until suddenly expelled risking contamination.
down and apply very gentle traction on
bleeding is controlled. Once bleeding is
the umbilical cord to gently advance the Be very careful when holding the
controlled, finish the infusion at 1-2 mL/
placenta through the vaginal canal, while newborn, as he/she will be very slippery.
min.
applying cephalad suprapubic massage It is advisable to hold him/her close to
to the contracted uterus to prevent Key Additional Points your body.
uterine inversion. A controlled and gentle delivery of every
fetal part is preferred to an explosive
Never force the expulsion of the placenta
delivery and decreases, to some extent, References and Further Reading, click
or apply more than gentle traction as
the probability of vaginal tears. here
umbilical cord separation and uterine
inversion can cause major bleeding.

630
Section 10

Pericardiocentesis

Case Presentation difficult lying flat in bed.


by David Wald and Lindsay Davis
A 52-year-old female with a Today she is experiencing
history of metastatic breast shortness of breath at rest.
cancer presents to the She reports no other active
emergency department with health conditions. Her initial
a complaint of progressive vital signs are blood pressure
shortness of breath 92/66 mmHg, heart rate of
throughout the past week. 108, respiratory rate of 20 per
She initially felt tired and out minute, temperature 37.1°C
of breath with walking and and a room air oxygen
climbing stairs. This has saturation of 96%. On
progressed to the point evaluation, the patient
where she would feel short of appears uncomfortable and
breath just walking around mildly dyspneic. She has JVD
her house. Over the last day, at 5cm. Her heart sounds are
she noted that it has become regular without a murmur but

631
are barely audible. Lungs are clear in all fields. Image 16.7

Her abdomen is soft and non-tender. She has no


lower extremity edema. She is awake, alert and
moves all of her extremities equally. A 12 lead
electrocardiogram shows sinus tachycardia with
low voltage in the limb leads. Her chest
radiograph is shown in Image 16.7. An
emergency medicine bedside cardiac ultrasound
is performed, see Image 16.8.

Pericardial effusion on chest radiograph demonstrating an enlarged “water


bottle” shaped heart. This finding can be seen in pericardial effusions that
have developed slowly, allowing time for the pericardial sac to stretch.

632
Image 16.8 A p e r i c a r d i a l e ffu s i o n develops when fluid accumulates in
the potential space between the visceral and parietal pericardium.
Pericardial effusion can be caused by a number of conditions
including trauma, malignancy, uremia, cardiac rupture, and
infectious causes such as tuberculosis and viral pathology. The
clinical effect of pericardial effusion can vary based on etiology,
volume, and particularly the speed at which the effusion
accumulates. If fluid accumulates very gradually, the pericardium
can remodel and stretch to accommodate the increased volume.
In these cases, symptoms are often insidious and progressive
over days to weeks. Alternatively, if fluid accumulates suddenly,
as in the case of penetrating chest trauma, the pericardium is not
able to stretch to accommodate the increased volume of fluid.
The result can be the rapid development of pericardial
tamponade and death.
Echocardiography of a pericardial effusion in the subcostal 4-chamber view.
Note the inward bowing of the right ventricle indicating cardiac tamponade Pericardial tamponade occurs when the pressure of a pericardial
and hemodynamic compromise. effusion becomes greater than the pressure in the right atrium,
resulting in the collapse of the right atrium during diastole.
Remember, the right side of the heart is a low flow system, and it
Pathophysiology and Indications does not take much pressure to impede flow. The increased
The heart is surrounded by a double layer fibrous sac known as
pressure within the pericardial space can eventually cause
the pericardial sac. The first layer, the visceral pericardium, is
compression of the entire right side of the heart, leading to the
adherent to the cardiac epicardium. The second layer, the parietal
restricted ventricular filling. This, in turn, can lead to decreased
pericardium, is separated by the visceral pericardium by 25-50
stroke volume and decreased cardiac output. If not treated, this
mL of physiologic serous fluid, allowing the heart to beat without
can result in hypotension, cardiogenic shock, and death. If an
friction.
effusion develops rapidly, as little as 150 mL of fluid, can cause

633
tamponade physiology. A pericardial described in every textbook discussion of nonspecific. In the case of a chronic
effusion that restricts cardiac output pericardial tamponade, studies have pericardial effusion, a chest radiograph
resulting in tamponade is a true shown that these exam findings have may demonstrate an enlarged “water
cardiovascular emergency and is the poor sensitivity and specificity and are bottle” shaped cardiac silhouette (see
primary indication for present together with a minority of the image 16.7). However, if the effusion is
emergency pericardiocentesis. time. The most common signs and from an acute traumatic etiology, the
symptoms exhibited by patients with pericardial sac will not have had time to
At times, making the diagnosis of
tamponade include dyspnea, tachycardia, stretch to accommodate the increased
pericardial tamponade can be difficult
JVD, and a narrowed pulse volume, leaving the cardiac silhouette
because the condition can present in an
pressure.  Pulsus paradoxus  (a decrease unchanged.
i n s i d i o u s f a s h i o n i f t h e e ffu s i o n
in systolic blood pressure of greater than
accumulates slowly over many days to
12 mmHg during inspiration) has been Image 16.9
weeks. In these cases, the differential
found to be one of the more sensitive and
diagnosis will include many other causes
specific exam findings associated with
of shortness of breath. Alternatively,
cardiac tamponade. However, the tedious
pericardial tamponade should always be
and time-consuming nature of this exam
considered as a potentially reversible
technique makes it difficult at best and
cause in the patient who develops
often an impractical tool for diagnosing a
cardiac arrest shortly after chest trauma.
life-threatening condition in an unstable
Electrocardiogram demonstrating electrical
Cardiothoracic surgeon Claude Schaeffer patient in the emergency department. alternans in a patient with a large pericardial
effusion. Note the alternating QRS amplitude
Beck originally described the physical due to the swinging motion of the heart within
Electrocardiographic findings can include
exam findings of pericardial tamponade the pericardial sac.
low voltage QRS complexes, PR segment
in 1935 as what is now commonly known
depression, ST elevation and electrical
as  Beck’s triad: muffled heart sounds,
alternans (see image 16.9), though these Ultrasound is the best and most
jugular venous distension (JVD), and
findings are also not specific or sensitive. applicable diagnostic imagining modality
hypotension. Though these findings are
Chest radiographs are equally used to identify a pericardial effusion or

634
tamponade. It is noninvasive and safe, Image 16.10 Image 16.12
with no risk of radiation to the patient.
The increased availability of bedside
ultrasound in the emergency department
has allowed for instant point of care
diagnosis of this potentially life-
threatening condition. In addition to
offering direct visualization of the effusion
itself, sonography allows the operator to
assess for hemodynamic compromise
secondary to increased pericardial
Echocardiography of a pericardial effusion (PE) Echocardiography of a pericardial effusion (PE)
pressure. The initial sign of this process is in the subxiphoid view. Note that the effusion is in the apical 4-champer view. The arrow
the collapse of the right atrium in diastole, seen as a large anechoic stripe surrounding the indicates collapse of the right ventricle (RV) seen
heart. in cardiac tamponade.
followed by bowing of the right ventricle.
A pericardial effusion on ultrasound Image 16.11 Contraindications
appears as a dark (anechoic) stripe
between the myocardium and the Absolute contraindication
pericardium. Pericardial effusion can Aortic dissection.

typically best be viewed by one of three


Relative contraindications
commonly used cardiac windows: 1)
• Uncorrected coagulopathy or
subxiphoid, 2) parasternal long axis, and
anticoagulant therapy
3) apical 4-chamber. Ultrasound images
of a pericardial effusion from these three • Thrombocytopenia (platelets <50)
views are seen in Image 16.10-11.
• Traumatic hemopericardium (managed
Echocardiography of a pericardial effusion (PE)
surgically)
with tamponade physiology as demonstrated by
the collapse of the right ventricle (RV) seen in
parasternal long-axis view.
635
Equipment and Patient • Sterile gowns, gloves, masks, and caps Before bedside ultrasound was

Preparation for all providers in the room readily available, practitioners would
If the time permits and the patient is perform emergent pericardiocentesis in a
• Sterile drapes
a w a k e , t h e p ro c e d u re s h o u l d b e “blind” fashion, relying on anatomic
explained to the patient, and the • Sterile ultrasound transducer cover landmarks to guide the placement and
physician performing the procedure direction of the needle. This approach
• Sterile ultrasound gel can put surrounding structures at greater
should obtain written informed consent.
risk of injury. Bedside ultrasound allows
• Chlorhexidine sponge
Cardiopulmonary resuscitation equipment direct visualization of the heart, not only
should be readily available in the event of • 1% lidocaine with epinephrine leading to faster, more accurate diagnosis
a life-threatening arrhythmia or further of tamponade, but it also allows
hemodynamic decompensation. • 25 gauge needle (1.5 inches) and 10mL
practitioners to choose the approach that
syringe for lidocaine injection
offers them the best access to the
Intravenous sedation should be
• 16 or 18 gauge spinal needle (5-10cm) pericardial effusion while avoiding
considered but must be reconciled with
surrounding structures. The three most
the urgency of the procedure and the
• 1 empty 5mL syringe commonly used ultrasound approaches
patient’s hemodynamic stability.
are the Subxiphoid, parasternal and
• 5 mL syringe filled with 8cc of saline
If possible, elevate the chest wall apical approach (see Images
30-45%; this brings the heart itself closer • 3-way stopcock 16.13-16.15). Details regarding probe
to the chest wall. positions and the pros and cons of each
• 60 mL syringe
approach can be found in Table 1. 
All providers involved in the procedure
• Central venous access kit/
should wear sterile protective clothing
Pericardiocentesis kit
including gown, gloves, and mask.
• Sterile gauze
Equipment
Procedure Steps
636
Image 16.13 Image 16.14

To achieve a subxiphoid view of the heart, place the probe inferior to the
xiphoid process and angle it cephalad and towards the patient’s left. For a parasternal long approach, position the probe to the left of the
sternum, in the 4th or 5th intercostal space.

637
Image 16.15
Table 16.1 Probe positions for ultrasound guided-
pericardiocentesis

PROBE
APPROACH PROS CONS
POSITION

Inferior to the Good acoustic Risk of hepatic


xiphoid process, window created puncture
angling the probe by liver, no
Subxiphoid cephalad and left overlying bony
towards the heart structures

To the left of the No overlying Risk of damage to


sternum, in the organs internal mammary
Parasternal 4th or 5th artery, which lies
intercostal space 3-5 cm lateral to
sternal border

Slightly lateral to The part of the Often most


midclavicular line, heart closest to challenging view
in 5th or 6th the needle is the due to body
intercostal space. left ventricle, habitus, increased
Insert needle which is thick risk of
above the rib to walled and will pneumothorax
Apical avoid suffer less
neurovascular significant injury if
bundle and aim inadvertently
toward right penetrated. The
shoulder coronary arteries
are also smallest
at the apex.

To use an apical approach, place the probe slightly lateral to the 1. Clean and drape the patient in a sterile fashion.
midclavicular line, in the 5th or 6th intercostal space.
2. Prepare the ultrasound transducer with a sterile sheath.
Table 16.1 shows. probe positions for ultrasound-guided
pericardiocentesis • The ideal probe for this procedure is the 5 to 1 MHz
transducer due to its small footprint.

638
• If that is not available, the 5 to 2 8. Once placement is confirmed, proceed Please watch video 1 and video 2.
MHz curvilinear probe should with Seldinger technique
provide adequate views. Hints and Pitfalls
• remove the syringe from the • Do not confuse the epicardial fat pad
3. V i s u a l i z e t h e a r e a o f m a x i m a l needle, with a pericardial effusion. Keep in mind
pericardial effusion with the ultrasound t h a t s i g n i fic a n t e ffu s i o n s a r e
• insert the guidewire into the
probe. circumferential, and the fat pad is only
needle tip and thread the wire.
an anterior structure. Also, fat pads will
4. If time allows and the patient is awake,
• Once the guidewire is in place, move with each contraction, whereas a
infiltrate the area of planned needle
remove the needle, keep the wire pericardial effusion does not and thus
insertion with 5 mL of 1% lidocaine.
in place. will appear to change size as the
5. Using an in-plane approach, insert the ventricular wall constricts inward away
• Thread a dilator over the
needle at a 45-degree angle (for from the pericardium with each
guidewire.
subxiphoid approach 30-45 degrees to contraction.
prevent liver puncture), maintaining • Remove the dilator, keep the wire
• Use the ultrasound to measure the
negative pressure on the syringe while in place.
depth of the pericardial effusion, and
keeping the needle tip in view on the
• Slide a catheter over the wire. make sure you use a needle that is long
screen at all times.
When the catheter is in place, the enough.
6. Observe the needle entering the guidewire can be removed.
• Keep your needle tip in view at all
pericardial sac.
9. Aspirate/drain the pericardial effusion/ times! This may require tilting or
7. Once the pericardial sac has been blood. adjusting the probe to maintain
penetrated, inject agitated saline and visualization around surrounding
look for the bubbles visible on 10.Check the vitals and check with structures as you progress.
ultrasound in the pericardial sac to ultrasound.
• Rapid drainage of pericardial effusion
confirm correct placement.
can lead to rapid increases in preload,

639
which can rarely cause flash pulmonary • Immediately after the procedure, a s i g n i fic a n t l y d e c r e a s e d t h e s e
edema, bradycardia and rebound chest x-ray should be obtained to complications. A study of 1127
hypertension. ensure there is no pneumothorax or air ultrasound-guided pericardiocentesis at
under the diaphragm. Mayo Clinic showed a procedure success
• If there is a question about the source
rate of 97%. The rate of major
of bloody aspirate, look for clotting • If a pigtail catheter was inserted for
c o m p l i c a t i o n s , w h i c h w e re t h o s e
ability. If the blood is from a traumatic continuous drainage, it should be
requiring intervention, was 1.2%. The
effusion or intracardiac, it will clot sutured in place. Avoid tying the sutures
minor complication rate was 3.5%.
easily. If the blood has migrated into the so tightly that it occludes the catheter.
Similar findings have been repeated in
pericardial space and results from a
• Cover the catheter insertion site with more recent studies.
non-traumatic effusion, it will be
sterile gauze and tape.
defibrinated and thus will not clot or will Specific complications to be
take much longer to clot. • The patient should remain on a cardiac aware of include:
monitor. • Dry tap (often caused by blockage of
• Agitated saline can be prepared by
the needle with clot or tissue)
connecting two 5mL syringes to the • Vital signs and cardiac rhythm should
needle catheter via a 3-way stopcock frequently be reassessed to monitor for • Dysrhythmias, though the literature
valve. One syringe contains saline, the findings that would suggest re- suggests dysrhythmias related directly
other air. Agitate the saline by rapidly accumulation of the effusion and to the pericardiocentesis procedure
pushing the saline from one syringe to inadvertent procedural complications. itself are rare
the other with the stopcock closed to
• Myocardial or coronary artery puncture
the needle catheter. The saline is Complications
Most complications of pericardiocentesis leading to hemopericardium
sufficiently agitated when it appears
cloudy. are related to needle penetration of either
• Liver laceration
the heart or surrounding structures.
Post Procedure Care and Studies have cited serious complication • Pneumothorax/hemothorax
Recommendations rates of 20-30% using the blind
• Pneumopericardium
approach. The use of ultrasound has
640
• Vascular injury, most likely the internal mammary artery and the
intercostal neurovascular bundle

• Flash pulmonary edema from rapidly increase in left ventricular


preload after pericardial decompression

• Suppurative pericarditis

• Costochondritis

References and Further Reading, click here

641
Section 11

Lumbar Puncture

Case Presentation
by Khuloud Alqaran A16-year-old male, without a known case of any medical
illness, presented to the ED accompanied by his mother. His
chief complaint was altered mental status. Three days earlier
to his presentation, he had a fever, nausea, vomiting, and
headache. The symptoms worsened over time. His mother
noted that 2 weeks earlier he visited his grandmother at the
intensive care unit. On physical examination, he opened his
eyes once the doctor called his name; then, he said, “Where
am I, what is the time?” He was moving in the bed with no
neurological focal deficit. Vital signs as following: Temperature
38C, heart rate of 110/min and blood pressure of 100/45
mmHg. Nuchal rigidity was positive, and he had skin rashes
over his shins as shown in the picture below.

642
Image 16.16 IIH) by measuring the opening pressure investigations may be needed with
of the Cerebrospinal fluid (CSF) the CT scan.

Therapeutic Indications for CT head prior to


• Administration of anesthetic spinal the LP are as follows;
agent or intrathecal drug (e.g., • Age > 60 years
Chemotherapy in case of Leukemia).
• Immunocompromise status
• Drain away excess of CSF in case of IIH
• Altered mental status or presence of
Contraindications any neurological deficit
Courtesy of Dr. DB Calheiros. Retrieved from
http://www.atlasdermatologico.com.br/ Absolute contraindication • Any sign/symptoms of elevated ICP
disease.jsf?diseaseId=287 Infected skin or soft tissue at the entry (e.g., headache, papilledema, or
site of the needle bradycardia)

Indications Relative contraindication • History of the Central nervous system


• Increased Intracranial Pressure (ICP) (CNS) lesion (e.g., old stroke, SOL such
Diagnostic as tumor or abscess)
• Presumption of meningitis • Deranged coagulation profile (INR > 1.4
or Severe thrombocytopenia, platelet < • Recent seizure activity (within a week
• Presumption of subarachnoid
40,000) from presentation)
hemorrhage (SAH).

Other contraindication Equipment and Patient


• Presumption of any syndrome such as
Multiple sclerosis or Guillain-Barre
Brain abscess or any space-occupying Preparation
lesions (SOL)
Equipment
• Presumption of Pseudotumor Cerebri
Lumbar puncture should not be delayed
(Idiopathic Intracranial Hypertension,
for any reason in the clear indication.
H o w e v e r, i n s o m e c a s e s , f u r t h e r
643
Pre-packed LP procedure kit may include Image 16.17 LP set
the following items:

• Sterile dressing

• Sterile drape

• Alcohol Swabs

• Spinal needle size 18 gauges

• Lidocaine 1% without epinephrine

• Syringe 3mL

• 3 Way stopcock

• Monometer

• 4 plastic test tubes (numbered from 1-4)

• Syringe 10mL

• 2×2 Gauze

• Small Adhesive plaster Retrieved from https://i2.wp.com/blogs.brown.edu/emergency-medicine-residency/files/2015/08/


Figure-2.jpg
• Extra supplies
Maintain universal precautions (sterile The spinal needle should be replaced by
• Non-sterile marking pen gloves, surgical facemask, cap and sterile Styletted spinal needle size 20 or 22

• Chlorhexidine swabs for gown) gauge. However, the length should be

sterilization chosen depending on the patient’s age.

644
• Infant 1.5in or 3.8cm • Lateral recumbent with spine parallel to columns of L4 at the level of the
bed: in this position, the patient’s hips, posterior superior iliac crests. (Adult
• Child 2.5in or 6.3cm
knees, and chin are flexed toward his/ injection site – any interspinous space
• Adult 3.5in or 8.8cm her chest (fetal position). Analgesia, from L2-S1 as the spinal cord
sedation or anxiolytic (e.g., terminates at L1 level. However,
The registered nurse or any doctor benzodiazepine) can be considered if pediatrics injection site should be only
colleague to help position the patient appropriate to reduce patient anxiety. from L3-L4/L4-L5 interspinous space
during the procedure as the conus medularis ends at the
• Sitting upright with hips flexed with feet
level of L1-L3).
Place/order CSF tests needed prior to the on a stool: in this position, the patient is
procedure awake and cooperative. It’s preferred in 3. U s e a s k i n - m a r k i n g p e n t o
obese patients when it would ease approximate the entry site.
Patient preparation
midline localization. The patient would
• Informed consent needs to be taken 4. Gown up and maintain universal
sit upright; his/her lumbar spine should
from the patient or the legal guardian. precautions (sterile gloves, surgical
be perpendicular to the table. His/her
facemask, and head cap).
• Speak to your patient during the foot should be supported by a stool and
procedure; the patient is already not hanging down. A pillow can be 5. Apply the antiseptic solution in a
anxious and can’t see what you are placed at the patient lap so he/she can circular motion starting from the entry
doing. Talk to the patient and explain bend forward and keep his/her chin site to the periphery.
what you are doing in a calm manner. towards his/her chest (angry cat
For example, say: “ Now I am going to position). 6. Apply the sterile drape.
numb the site with a smaller needle; it
7. Create a skin wheal of 1% Lidocaine;
may have a burning sensation. Please Procedure Steps
make sure it is no more than 1mL to
don’t move.” 1. Position the patient as mentioned
avoid losing the landmark. Then inject
above
• Patient placed in two positions upon into the deeper tissues.
preference and patient condition or age 2. Identify the landmarks anatomically by
palpating the midline vertebral
645
8. Advanced the needle at the midline Please watch video 1 and video 2. • Ask someone to help you hold the
with your dominant hand holding the patient and maintain his position
hub and your non-dominant hand Hints and Pitfalls during the procedure.
supporting the needle by placing the • Like any other procedure, preparation is
a must. Position the patient, palpate his • If still anxious, give him/her some
thumb/index finger on the shaft of the
back, get to know his anatomy, then anxiolytic or even sedation if
needle for balance, parallel to the bed.
mark it with a marking pen. necessary.
The angle should be facing upward,
aiming at the umbilicus. • Injecting lidocaine can sometimes
• Your patient is elderly, and you are
hitting a bone only after insertion of obscure your landmark.
9. Characteristic “pop” is occasionally
felt when the needle passes the dura. 25% of your needle. In most patients,
• Try not to inject more than 1mL to
If there is no sound, draw the stylet the needle should be inserted 50-75%
make a wheal; then, inject the
periodically checking for the CSF after of its length prior to obtaining CSF flow.
remaining in the deeper tissues.
approximately 4-5 cm.
• You may be hitting calcified
• The traditional teaching “feel the first
10.Once CSF starts to drain, attach the supraspinal ligament.
pop then the second pop, CSF will
manometer to measure the pressure. flow.”
• Try to enter from the lateral aspect
Then, start collecting the fluid from to avoid the calcified ligament.
tube number 1 to 4 in sequence • Never depend on the pop. Most of
pattern. No more than 1mL in 1-3 • The patient can get very anxious and the time a series of pops are felt
tubes, then 3-4mL in tube 4. alarmed. instead, as several spinal
ligaments are encountered prior to
11.Replace the stylet before removing the • Talk to the patient; he can’t see entering the space.
needle; then, remove both of them what you are doing. Tell him what
together. step you are going to do. • In obese patients,

12.Cover the injection site with the


adhesive plaster.

646
• Use the full length of the needle to • RBC counts taper down from • Protein (mg/dL)
3.5-inch, or use the 6-inch tube 1 to tube 4. This is not
• Culture or gram stain
“harpoon” needle. fully reliable unless it is
completely clear by the 4th • Normal values include 5-20 cmH2O
• Try to do it in a sitting position.
tube, but classically, the RBC opening pressure, equal to or less
• CSF is red or tinged red count decreases by 30%. than 5 WBC per mm3, no
neutrophils, 50-80 mg/dL glucose
• The needle is too deep and hits a • Examining the 4th tube as a
and 20-45 mg/dL protein.
venous plexus leading to a separate entity can also help
traumatic tap. On the other hand, rule out SAH • The bacterial CSF sample shows
it’s subarachnoid hemorrhage or elevated opening pressure >500
• <100 RBC: almost certainly
meningitis. WBC per mm3, >80% PMNL, low
traumatic
glucose (<40 mg/dL), and increased
• Signs of a traumatic tap
• <500 RBC: probably protein (>50 mg/dL). In addition,
• The absence of traumatic culture or gram stain indicates
xanthochromia (shows up bacteria. The viral sample, however,
• >10K RBC: likely SAH
within 12 hours and persists shows normal or slightly elevated
2-4 weeks). If present at the • Is it bacterial or viral? opening pressure, 100 – 500 WBC,
time of the taping then its neutrophils less than 50% and
• There are couple measures helping lymphocytic predominance, normal
highly suggest SAH.
us to identify the cause. These are glucose, protein is normal or slightly
• RBC count 400-500 RBCs or elevated, and culture or gram stain
• Opening pressure (cmH2O)
less suggestive of the indicates the virus.
traumatic tap. Must become • WBC count (per mm3)
zero at one of the last tubes. Post-procedure Care and
• Neutrophils (%) Recommendations
• Glucose (mg/dL)
647
• The patient may lie flat; however, there (e.g. bed rest, oral analgesia, in some
is controversial evidence that it may cases caffeine drinks can help). A
reduce headache incident. refractory headache, an epidural blood
patch is recommended. Using non-
• Vital sign should be recorded
cutting, smaller diameter needle can
depending on the hospital guidelines.
decrease the occurance. 20-22 gauge
• Neurological examination at least every atraumatic needles are the best choice.
4 hours within the first 24 hours.
Other complications are;
• Encourage fluid intakes up to 3L/24hrs • Infection
(tea and caffeine may help).
• Heriniation syndrome
• Monitor the puncture site for any
• Formation of subarachnoid epidermal
bleeding, CSF leakage or infection.
cyst
• Ensure the patient void after 8 hours
• Backache and radicular syndrome
post procedure.
• Spinal epidural hemorrhage
• Administer analgesia accordingly.

• Do not forget the documentation.


References and Further Reading, click
Complications here
Post–LP headache  is the most common
complication. It results from the reduction
of CSF below the cisterna magna. It can’t
be prognostic or prevented. Starts within
the first 48hrs. A mild headache, self-
limited, only needs conservative therapy
648
Section 12

Nasogastric Tube Placement

Case Presentation
by Sara Nikolić and Gregor Prosen A 47-year-old man presents to your ER complaining of nausea
and vomiting. He tells you that vomiting started a couple of
hours after eating dinner the night before. It was a normal
vomit, consist of digested food; however, it is not associated
with meals. He has barely eaten during the past 36 hours. The
pain consists of cramping and is vaguely umbilical, but it is
not well localized. He gets mild relief from vomiting and says
the pain is severe (9/10). He has felt generally unwell and has
not taken his temperature. None of his close contacts have
reported any vomiting. His last bowel movement was
yesterday morning, and he cannot recall passing any flatus
today. About 20 years ago he had an appendicectomy.

On examination, the patient is lying in bed in some discomfort


from the stomach cramps. He has an appendicectomy scar

649
and mild distension of his small bowel with numerous Introduction
Nasogastric (NG)  tube placement is one
abdomen. You hear high- valvule conniventes and
of the most common procedures
pitched bowel sounds on increased peristalsis with performed in intensive care settings, the
auscultation. whirling motion of the bowel emergency department, and hospital

contents. wards. It is frequently used for the


Vital signs are as follows: management of patients who require
temperature of 36.6°C, heart See video. compression of the gastrointestinal (GI)
tract, diagnosis and assessment,
rate of 66 bpm, blood pressure
nutritional support and medication
126/63 mmHg, respiratory rate administration. The procedure is rapid,
11/min, oxygen saturation 98% simple, and straightforward. The goal is
to conduit the NG tube to the stomach
on room air.
through the nasal cavity, nasopharynx,
Blood tests come back and oropharynx, and esophagus that enters
the stomach below the diaphragm. The
show: white cell count 7.7 ×
very vascular nasal mucosa lines the
109 cells/L, CRP 23 mg/L, Na nasal cavity. This is important to
137 mM, K 3.7 mM, AST 27 U/ remember because, after each
unsuccessful insertion, incidences of
L, ALT 23 U/L, ALP 29 U/L,
mucosal bleeding and hemodynamic
amylase 152 U/dL, urea 4.2 complication increase.
mM, creatinine 92 µM.

The bedside ultrasound shows


distended (>3 cm diameter)
650
Procedure • Feeding introduced, but an NG tube can be
used for lower-grade varices) or
Indications • Bowel irrigation
stricture
• NG tube can be kept following
• Recent banding of esophageal varices
Diagnostic corrosive ingestion for the development
of a tract in the esophagus. It • Alkaline ingestion (the tube may be kept
• Evaluation of upper gastrointestinal (GI) subsequently can be used for balloon if the injury is not severe)
bleeding dilatation.
Equipment and Patient
• Identification of the esophagus and
Contraindications Preparation
stomach on a chest x-ray Absolute  contraindications for NG tube
placement are: Equipment
Procedural/Therapeutic
• Choose the size of nasogastric tube
• Severe midface trauma – rare that is appropriate for the patient. A size
• Aspiration of gastric fluid content
perforation through the thin cribriform 16 to 18 French is typically used for an
• Administration of radiographic contrast plate of the ethmoid bone and into the adolescent or adult patient.
to the GI tract brain can occur. Patients with facial
trauma are best served with orogastric • Personal protective equipment
• Gastric decompression
intubation.
• A water-soluble lubricant is often
• Relief of symptoms and bowel rest in enough to facilitate passage.
• Recent nasal surgery
the small-bowel obstruction
Relative  contraindications for NG • If topical anesthesia is available: 1.5 mL
• Gastric content aspiration for recent of atomized lidocaine may be atomized
placement are:
toxic ingestions into the nasopharynx, with 3 mL applied
• Coagulation abnormality to the oropharynx and swallowed, or 5
• Administration of medications such as
ml of 2% lidocaine jelly can be injected
active charcoal • E s o p h a g e a l v a r i c e s ( u s u a l l y, a
Sengstaken-Blakemore tube
651
into the nostril. Do not use oil-based • Explain the procedure to the patient, •Gather equipment for the procedure
lubricant. including route, purpose, and
• Inspect the nostrils for septal deviation
anticipated duration of intubation.
• Cup of water with a straw – to determine which nostril is more
• Have the patient sit upright or raise the patent, ask the patient to occlude each
• Emesis basin and towel
head of the bed. If this is not possible, nostril and breathe through the other
• Tongue blade and flashlight passing the tube with the patient in the
• Position the patient sitting upright with
left lateral decubitus position has less
• Aspirating and irrigating syringes (10ml their neck partially flexed
risk of aspiration than if the patient is
and Toomey syringe, 60 mL) supine. • Put the gloves on
• Tincture of benzoin, scissors, and tape • Check for nasal obstruction or • If available, put the anesthetic spray to
or semipermeable transparent maxillofacial trauma. Have the patient the back of the patient’s throat for
membrane dressing inhale briskly through each nostril and comfort
use the more patent nostril for
• pH indicator paper
intubation. • Estimate the length of insertion by
• Stethoscope measuring the distance from the tip of
• Test the gag reflex. Patients unable to the nose, around the ear, and down to
• Suction tubing and container gag are at increased risk of pulmonary 5cm below the xiphisternum. This point
aspiration. can be marked with a piece of tape on
• Wall suction, set to low intermittent
the tube.
suction Procedure steps
• Wash hands • Ask the patient to hold the cup of water
General patient preparation
and put the straw in his or her mouth.
• Give the patient nothing by mouth for • Introduce yourself and confirm patient
several hours (ideally). details • Lubricate the tip of the NG tube

• Explain the procedure and gain consent • Place the kidney plate near the patient
in case there is leakage.

652
• War n patient you will start the auscultating a rush of air over the inability to speak, or significant nasal
procedure and in case of pain they stomach using the 60 mL Toomey hemorrhage occurs.
should tip you on the hand. syringe is not as helpful because the
sounds of air in the bronchial tree can Hints and Pitfalls
• Gently insert the NG tube along the • During insertion, if concern exists that
be mistaken for gastric insufflation.
floor of the nose. Advance NG tube the NG tube is in the wrong place, ask
parallel to the nasal floor (not angled up • Apply benzoin or another skin the patient to speak. If the patient can
into the nose) until it reaches the back preparation solution to the nose bridge. speak, then the tube has not passed
of the nasopharynx, where resistance Tape the NG tube to the nose to secure through the vocal cords and/or lungs.
will be met (10-20 cm). it in place.
• To improve the success rate of
• At this moment, ask the patient to sip • If clinically indicated, attach the tube to nasogastric tube placement, provide
water through the straw and start wall suction after verification of correct external and medially directed pressure
swallowing. With each sip you continue placement. on the ipsilateral neck at the level of the
to advance the NG tube until the thyrohyoid membrane. It will collapse
• Dispose of used equipment into a
distance of the previously estimated the piriform sinus and eliminate it as a
clinical waste bin and wash hands.
length is reached. potential site for impaction. This
• Explain to patient that the procedure is maneuver was successful for difficult
• Confirm the tube’s placement in the
over. Reassure that the NG tube will nasogastric intubation in 85 percent of
stomach by radiographic imaging.
become more comfortable over the patients.
Alternatively, gently aspirate gastric
next few hours. Offer patient paper
contents with a 3-cc syringe, and check • The nasogastric tube may coil in the
towels to clean their face and nose.
the pH. A pH < 4 suggests the tip is in a oropharynx, mouth, or hypopharynx.
Document clearly the procedure.
gastric location. A pH > 5 does not Cool the tube in cold tap water or ice
reliably predict location because the Please watch video 1 and video 2. water for 5 minutes to make the tube
respiratory system and intestinal tract stiffer and then reinsert it. A larger bore
distal to the pylorus often have a pH > Withdraw the nasogastric tube if, at any
tube may be inserted more easily. A
5 . Ve r i f y i n g t u b e p o s i t i o n b y time resistance, respiratory distress, the
final option is to place several fingers
653
through the patient’s mouth and into the tube in anesthetized and tracheally perforation or errant placement of the
oropharynx. The fingers can be used to intubated patients after the first nasogastric tube.
guide the tube against the posterior attempt.
• Although auscultation of air in the
oropharyngeal wall and into the
• American Association of Critical-Care stomach has been classically used to
hypopharynx. Do not attempt this
Nurses partook a survey about feeding determine correct placement, air
unless the patient is unconscious or
tube practices in adult intensive care insufflated into the pleural space or the
paralyzed to prevent them from biting
units. The recommendations were to esophagus after misplacement of the
the fingers.
obtain radiographic confirmation that tube can be just as easily heard over
• The risk for tube misplacement is each blindly inserted tube is correctly the upper abdomen.
greater in the intubated patient who is positioned before the first use, which is
• Gastric contents should be able to be
unable to assist with nasogastric currently not adequately implemented.
aspirated through the nasogastric tube.
intubation. Observe that there are no Also, auscultation is widely used
changes in the patient’s oxygen despite recommendations to the • Testing the pH of the gastric contents
saturation when inserting the contrary. can help predict the placement of the
nasogastric tube. It is very easy for the nasogastric tube. However, in one trial,
nasogastric tube to pass by the cuff of Post Procedure Care and pH of 4 was able to accurately identify
an endotracheal tube without much Recommendations the location of only 56% of all NG
resistance. • The patient should be able to speak
feeding tubes when compared with the
without respiratory distress immediately
reference standard radiography. The
• GlideScope facilitates NG tube insertion after placement of the nasogastric tube.
use of H2 blockers makes the
and reduces the duration of the Observe the patient for complaints of
assessment of gastric pH difficult.
procedure in anesthetized patients. neck pain, substernal chest pain,
Radiographic demonstration of the tube
Also, esophageal guidewire-assisted dysphagia, drooling, trismus, fever, or
in the antral or fundal portion of the
insertion with manual forward laryngeal subcutaneous and mediastinal air.
stomach is the preferred method of
displacement technique most frequently These would be signs of esophageal
confirmation.
results correct positioning of the NG

654
Complications a cuffed endotracheal tube does not often results in mediastinitis with a
• The  most common  complication of preclude passage into the respiratory subsequent mortality rate of up to 30%.
nasogastric intubation is  discomfort  in tree. The nasogastric tube will pass the Prompt recognition, surgical repair, and
the nasopharynx and oropharynx. cuff of the endotracheal tube without parenteral antibiotics can reduce the
significant resistance. Advancing the mortality rate to less than 10%. The use
• Placement in the nares can result in tube into the airway can result in of softer and smaller nasogastric tubes
epistaxis if the nasal mucosa is irritated, perforation of a bronchus or the lung with generous lubrication can reduce
abraded, or ulcerated. a n d re s u l t i n a p n e u m o t h o r a x , the risk of esophageal perforation.
hydropneumothorax, pulmonary
• These complications can be reduced or
hemorrhage, empyema, or Pediatric, Geriatric,
avoided with generous lubrication of the
bronchopulmonary fistula. These Pregnant Patient, and Other
nasogastric tube and the installation of
topical anesthetics and complications are increased if Considerations
medication or alimentation is infused The placement of a nasogastric tube in
vasoconstrictors.
into the respiratory tree. children is often difficult. Their large
• Sinusitis may occur from the tonsils and adenoids may hinder the
nasogastric tube obstructing the sinus • The  most serious  complication of passage of the nasogastric tube. These
ostia. These complications are usually nasogastric tube placement is tissues are soft, easily injured, and may
of no clinical significance. the  esophageal perforation. This most bleed as the nasogastric tube is passed.
often occurs in the posterior wall of the The tongue, large by comparison with
• A more serious consequence of cervical portion of the esophagus and adults, may push into the oropharynx and
nasogastric intubation is misplacement through the cricopharyngeus muscle. impede the passage of the nasogastric
into the respiratory tree. This is Risk factors for esophageal perforation tube. Their nostrils and nasal passage are
estimated to occur in up to 15% of include a preexisting esophageal quite small and limit the size of
cases. The incidence increases in abnormality, altered mental status, nasogastric tube that may be passed.
frequency with a patient who has a cervical osteophytes, cardiomegaly, Also, size is calculated by the formula
diminished gag reflex or a decreased tracheal intubation, a rigid nasogastric ((age in years + 16) / 2). Typical sizes
level of consciousness. The presence of tube, and multiple attempts. Perforation include 8 French for infants, 10 to 12
655
French for small children, and 12 to 14 French for older children.
Most common complications are nasal ala pressure sores that are
usually not associated with significant morbidity and mortality.

References and Further Reading, click here

656
Section 13

Procedural Sedation and Analgesia

Introduction Definition
by Nik Rahman When working in the emergency room, Procedural sedation is defined as the
one often finds himself in a situation use of short-acting analgesic and
where painful diagnostic or therapeutic sedative agents in order to enable
procedures are needed to be clinicians to perform procedures
performed. These procedures cause effectively while monitoring the patient
major pain and anxiety to the patient closely for potential adverse effects.
and using local anesthesia on its own
does not suffice in some situations. Terminology
Procedural sedation reduces anxiety, A n x i o l y s i s  i s a s t a t e i n w h i c h

pain and potential unpleasant decreasing apprehension regardings a

memories associated with such particular situation without affecting

procedures while also facilitating the patient’s level of awareness.

execution of the procedure. Therefore, Without the intentional alteration of


procedural sedation and analgesia mental status, relief of pain is
(PSA) has become a fundamental and called  Analgesia. However, the altered
required skill for emergency physicians mental state may be a secondary
to learn for day to day use. This effect of the medications administered
chapter is also important for many for this purpose.
physicians who are facing painful
procedures.

657
Dissociation  is a trancelike cataleptic may appear somnolent but is 5.General anesthesia  refers to the
state induced by an agent such as arousable to voice or light touch. drug-induced loss of consciousness
Ketamine and is characterized by (Reflex withdrawal from the painful which patients are not arousable to
profound analgesia and amnesia. In this stimulus is not considered a painful stimulation. In this state, the
state, protective reflexes, spontaneous purposeful response.) ventilatory function is often impaired,
respirations, and cardiopulmonary and assistance may be required to
3. Dissociative sedation  is a cataleptic
stability are preserved. maintain the airway and respiration.
state induced by a dissociative agent
Positive-pressure ventilation may be
The controlled reduction of environmental (i.e., Ketamine). This state is
required as spontaneous ventilation is
awareness is called sedation. characterized by profound analgesia
o f t e n i m p a i re d . C a rd i o v a s c u l a r
and amnesia. This state is achieved
function may be impaired as well.
Levels of Sedation while airway protective reflexes are
1. Minimal sedation (anxiolysis)  refers a maintained along with spontaneous The different levels of sedation need to be
patient in this state responds normally respiration. Cardiopulmonary stability understood as each scenario may require
to verbal commands, although is also maintained. a certain level to be achieved to facilitate
cognitive functions and coordination t h e p e r f o r m a n c e o f t h e re q u i re d
may be impaired. Respiratory and 4. Deep sedation/analgesia  is a state
procedure.
cardiovascular functions are where the patient has a depressed
unaffected as this state essentially level of consciousness in which he/she Steps for PSA
involves mild anxiolysis or pain control. requires painful or repeated stimulation Early planning and preparation are key to
to evoke a purposeful response. preventing adverse events that may occur
2. Moderate sedation/analgesia  involves Patients may require assistance to and can be catastrophic if procedural
depression of consciousness while maintain a patent airway. Spontaneous sedation is poorly managed.
patients still respond purposefully to v e n t i l a t i o n m a y b e i n e ffic i e n t .
verbal commands or light tactile Cardiovascular function is usually Adequate staffing needs to be ensured;
stimulation. Airway, ventilation and preserved. this is done by having a nurse or another
cardiovascular functions are all qualified individual present for continuous
spontaneously maintained. The patient monitoring of vital signs and airway
658
patency as well as having a separate provider performing the Anesthesiology consultation may be required for a
procedure. patient with an anticipated difficult airway or an ASA classification
of III. It may be wise to have the anesthesiologist perform the
Pre-procedural evaluation sedation in the operating room which is a better-controlled
The patient should be evaluated for sedation. PSA may not be
environment.
suitable for every patient in the ED. This can be done objectively
by using the ASA classification and difficult airway assessment Consent
(Table 16.2). When possible, a written consent should be obtained that
discusses the risks, benefits, and potential side effects of PSA.
Table 16.2 American Society of Anesthesiologists Physical The patient or direct family members need to sign the consent
Status Classification
before the procedure takes place.
SEDATION
CLASS DESCRIPTION EXAMPLES
RISK
Equipment Preparation
Normal and No past medical Minimal 1. High-flow oxygen source: ASA guidelines recommend
I healthy patien history
considering oxygen for moderate sedation and strongly
Mild systemic Mild asthma, Low
recommend it for deep sedation.
disease without controlled
II functional diabetes
limitations 2. Suction should be prepared in case any secretions accumulate
Severe systemic Pneumonia, Intermediate in the airway, which in turn need to be suctioned.
disease with poorly controlled
III functional seizure disorder
limitations 3. Airway management equipment: there may be a need for
Severe systemic Advanced cardiac High airway support during PSA. An appropriate size endotracheal
disease that is a disease, renal
IV tube should be prepared with an intubating blade, an oral
constant threat to failure, sepsis
life airway and a bag valve mask (BVM).
Moribund patient Septic shock, Extremely high
who may not severe trauma 4. Monitoring equipment includes a pulse oximeter, ECG monitor/
V
survive without defibrillator, transcutaneous pacing pads, Blood pressure
procedure monitor, and Capnography. Capnography measures end-tidal
carbon dioxide (CO2) partial pressure. The ASA recommends
659
the use of capnography for monitoring patients during PSA • Respiratory depression and hypoxia are possible
whether they are on supplemental oxygen or not. The aim is to
Generally, Etomidate has been shown to be safe and effective
detect if the patient’s ventilatory drive is affected during the
when used for procedural sedation.
procedure in order to perform corrective measures when
needed.
KETAMINE
Ketamine is a rapidly acting dissociative anesthetic that also
5. Vascular access equipment
produces a profound analgesic effect. Doses can be repeated
Medications used in PSA and titrated to effect with no risk of cumulative adverse events.
Onset, duration, and dosing vary according to the route of
ETOMIDATE administration.
Etomidate is a fast-acting sedative with little analgesic effect. The
onset of action is usually within 1 minute with a short duration of Contraindications:
action. It lasts between 3 to 5 minutes with standard dosing.
• Hypersensitivity to the medication
Elimination is done rapidly by the liver; therefore, duration of
action may be longer in patients with liver failure. It has few • Can lead to a hyper-sympathetic state, which might be
hemodynamic effects, and its neutral cardiovascular profile deleterious especially in patients with Coronary Artery Disease
makes it one of the most appealing agents for use.
• Avoided in patients who are predisposed to psychotic behavior
The contraindication is hypersensitivity to the medication.
Ketamine is a Pregnancy Category C medication
Etomidate is a Pregnancy Category C medication
Side Effects:
Side Effects:
• Ketamine is a derivative of the street drug Phencyclidine; it
• Muscle twitching is a well-known side effect that is generally causes an increase in systemic and pulmonary blood pressures,
well tolerated heart rate, cardiac output, cardiac workload and myocardial
oxygen demand. It should be avoided in elderly or patients with
• Nausea and vomiting may occur after emergence
cardiac diseases.

660
• Most common side effect seen with Ketamine is the emergence should be preceded by an opioid when performing painful
phenomenon. It occurs in approximately 15% of patients and is procedures.
mild in almost all of them. Less than 1 to 2% of patients have
Contraindications: in patients with allergy to eggs or soy
significant emergence agitation.
Side Effects:
• Transient airway laryngospasm (0.4%)
• Respiratory depression
• Emesis
• Apnea
FENTANYL
A rapid-acting synthetic opioid administered intravenously. • Hypotension
Duration of action may last from 30 to 60 minutes. It is a pure
analgesic with no sedative properties; therefore, it must not be • Pain over the injection site
used alone for PSA.
MIDAZOLAM
It is a Pregnancy Category C medication Midazolam is a benzodiazepine sedative, amnestic and anxiolytic
agent with no analgesic properties. It is usually combined with
Side Effects: opioids like Fentanyl to provide a good combination of sedation
and analgesia during PSA. It is eliminated by hepatic metabolism
• Respiratory depression is more likely at higher doses
and renal excretion; therefore, prolonged effects may be seen
• Hypotension and bradycardia are rare but may occur with high with dysfunction of any of those two organs.
doses
Side Effects:
PROPOFOL
• Cardiopulmonary depression
This is an ultra-short-acting sedative-hypnotic agent that has no
analgesic properties. It is quickly cleared from the body, It is a Pregnancy Category D medication
permitting superior titration, earlier recovery, and discharge. It
also possesses potent antiemetic properties and decreases The following table 16.3 summarized the list of all medications

intracranial pressure. Because of lack of analgesic effect, it used during procedural sedation.

661
Table 16.3 Agents Used in Procedural Sedation
ONSET OF DURATION OF
AGENT RECOMMENDED DOSE SIDE EFFECTS AND COMMENTS
ACTION ACTION

In adults < 1 minute 3 to 5 minutes Advantages: no cardiovascular or respiratory depression 


Etomidate 0.2 mg/kg over 30 to 60 Disadvantages: action is too short for some procedures
seconds

1 to 2 mg/kg intravenously 1 minute 10 to 20 minutes Nystagmus, hypersecretions, agitation, emergence delirium,


vomiting, myoclonus, laryngospasm, cardiovascular stimulation
Ketamine 4 to 5 mg/kg intramuscularly 5 minutes 15 to 45 minutes Contraindications: hypertension, ischemia, psychosis, infants
younger than three months

Initial dose in adults: 1 to 1.5 1 to 2 minutes 30 minutes Cough, hiccup, itching, vomiting, respiratory depression
mcg/kg intravenously
Fentanyl Titrate: 1 mcg/kg every 3 Requires another agent for sedation, repeat dosing may be required
minutes intravenously

Initial dose in adults: 0.02 mg/kg 1 to 2 minutes 30 minutes Respiratory depression, hypotension 
Titrate: 1 mg intravenously every
Midazolam 3 minutes Requires another agent for analgesia, poor reliability, repeat dosing
Initial dose in children (6 months may be required
- 5 years of age): 0.1 mg/kg

1 mg/kg Intravenously, then 0.5 15 to 30 seconds 5 to 10 min Respiratory depression, apnea, hypotension, pain over injection site
mg every 3 minutes if needed Advantages: Rapid onset, short duration, antiemetic, cerebral
Propofol protective. 
Contraindications: allergy to eggs or soy

Reversal Agents exceed that of the reversal agents. If used, it should be followed
Fentanyl and Midazolam both have antagonists that can be used by an extended observation period to ensure recovery.
to reverse their effects.
Caution: when using Flumazenil, it may lead to Status Epilepticus,
Agents like Naloxone for opioids (i.e., Fentanyl) and Flumazenil for especially in patients with unidentified benzodiazepine use or in
benzodiazepines (i.e., Midazolam) are the ones commonly used patients with a known seizure disorder.
for this purpose. However, the routine use of reversal agents
should be avoided as the duration of the sedation agents may

662
Recovery and Discharge
Finally, it is important to monitor all patients until the moment of
recovery. Drowsy patients should not be left unattended. Patients
should be monitored until they spontaneously wake up and are
able to perform their normal functions independently. Complete
recovery to baseline function may not be necessary for discharge.
Generally, an awake patient who is able to drink without vomiting,
able to ambulate and voids normally is capable of going home
ideally with family members or friends as an escort. Appropriate
discharge instructions should be given.

References and Further Reading, click here

663
Section 14

Rapid Sequence Intubation (RSI)

Introduction • Failure of airway maintenance or


by Qais Abuagla Airway management is one of the most protection or anticipated deteriorated
important skills for an Emergency clinical course
Department practitioner to master
• Failure of ventilation or oxygenation
because failure to secure an airway
can lead to mortality or morbidity. • Minimize oxygen consumption and
Therefore, all junior doctors and optimize oxygen delivery (e.g.,
trainees should be aware of this sepsis)
procedure in the early of their career.
• Prevent secondary brain injury or
RSI is an advanced airway terminate seizure (status epilepticus)
management technique that induces
immediate unresponsiveness Contraindication
(induction agent) and muscular
Absolute
relaxation (neuromuscular blocking
• Total upper airway obstruction
agent). It is the method of choice for
intubations in the Emergency • Total loss of facial/oropharyngeal
Department due to its high success landmarks
rates and the fewest complications.
Relative
Indications for RSI • Anticipating difficult airway

664
• Cardiac/respiratory arrest (this will go to • Assessment of airway: Anticipating hyoid bone? Submandibular space is
crash intubation) difficulty in establishing an airway in adequate to accommodate the tongue
emergency patients is the first step in making the visualization of the glottis
Steps of RSI (7 Ps) avoiding major complications. This easy. Is the larynx low enough in the neck
1. Preparation & Plan helps us to think about alternatives to be accessible? The distance from the

2. Preoxygenation of RSI. For example, neuromuscular hyoid to the thyroid. 2 fingers are what
paralysis should generally be we are looking for.
3. Pre-treatment avoided in patients with a high level
M – Mallampati: Oral access is assessed
of intubation difficulty.
4. Paralysis and induction with the Mallampati scale. Visibility of the
LEMON oral pharynx ranges from complete
5. Protection and positioning
visualization, including the tonsillar pillars
L – Look externally: Look for external
6. Placement with proof (class I), to no visualization at all, with the
markers of difficult intubation; these may
tongue pressed against the hard palate
7. Post-intubation management include the following body habitus, head
(class IV). Class I and class II predict
and neck anatomy (short neck), mouth
adequate oral access, class III predicts
Preparation (small opening, loose teeth or prominent
moderate difficulty, and class IV predicts
• Equipment  (tube, blade, Oxygen, teeth), jaw abnormalities (significant
suction, capnography, monitoring (ECG, a high degree of difficulty. (illustration
malocclusion), and beards.
BP, SpO2)) 16.6)
E – Evaluate 3-3-2: 3-3-2 rule is to assess
O – Obstruction or obesity. Upper airway
• Peds tube size: (age+4) /4 or use the patient’s airway geometry to
Braselow tape obstruction can make visualization of the
determine his or her suitability for direct
glottis, or intubation itself, mechanically
• The depth of the tube: size x 3 laryngoscopy. Can the patient fit 3 fingers
impossible. This may present as stridor,
between the incisors? For optimum
inability to swallow secretions or
• Asses for difficult airway, and set plan B glottis visualization, an adequate mouth
alteration in voice quality. Conditions
for failed airway opening is required.  Is the mandible
such as epiglottitis, head and neck
length 3 fingers from the mentum to the
665
Illustration 16.6 Mallampati extension is the most important •Very ill patient     <2 minutes
maneuver, and simple extension may be
as effective as the “sniffing” position in Pretreatment
It is used to blunt the adverse effect of
achieving an optimal laryngeal view. Neck
laryngoscopy and intubation but scant
mobility can be significantly reduced in
evidence
patients with trauma (cervical collar) or
the elderly and those with arthritis. Medication of pretreatment

Equipment and Preparation video part 1


Atropine
and video part 2.
• 0.02 mg/kg IV
Preoxygenation
Three minutes on 100% nonrebreather • For  all children <10 to prevent
mask or six vital capacity breaths bradycardia
p ro v i d e s 8 m i n u t e s o f a d e q u a t e
Fentanyl
oxygenation during apnea, but this time
Retrieved from https://commons.wikimedia.org/
wiki/File:Mallampati.svg period decreases in pregnancy, obesity, • 2-3 mcg/kg IV
and extreme of ages.
c a n c e r, L u d w i g ’s a n g i n a , n e c k • For elevated ICP, myocardial ischemia,
hematoma, foreign body or thermal injury Maintaining SpO2 > 90% differs between aortic dissection, subarachnoid
can compromise laryngoscopy, the patients. hemorrhage
passage of the endotracheal tube (ETT),
• Healthy 70 kg adult     8 minutes Lidocaine
BMV, or all three.
• Moderately ill adult     5 minutes • 1.5mg/kg
N – The NECK mobility. Neck mobility is
desirable for any intubation technique • 10 kg child     4 minutes • For increased ICP and bronchospasm
and is essential for positioning the patient
for optimal direct laryngoscopy. Neck • Obese adult     3 minutes

666
Indication for pretreatment: • Rapid action, and short duration. It has approximately 1 minute, and has a
PREMED no analgesic effect. clinical duration of 10 to 15 minutes.
• Pediatric
• It’s the most hemodynamically stable • Many protective reflexes are preserved
• Resistance (asthma) induction agent. This is an advantage with Ketamine, including airway
over other agents in shock, anaphylaxis reflexes. Ketamine has a direct
• Elevated ICP
or any case where the further drop in bronchodilator effect  and causes
• MI blood pressure can be catastrophic. catecholamine release. Therefore, it is
mainly used in patients with asthma,
• Elevated BP • Etomidate has a potential
anaphylaxis and hemodynamically
cerebroprotective effect as it decreases
unstable patients. Because of its
• Dissection cerebral metabolic oxygen consumption
features, it’s an excellent alternative to
and reduces cerebral blood flow and
Paralysis and Induction etomidate.
intracranial hypertension while
You don’t want an awake paralyzed
maintaining cerebral perfusion pressure • Side effects are the raise of BP (avoid in
patient!!
elderly) and emergence phenomena
• Side effects are nausea, vomiting,
First induction agent is given, then it is (visual, auditory, proprioceptive and
m y o c l o n u s a n d a d re n a l c o r t i c a l
followed by a paralytic agent. The confessional illusions which may
depression with multiple doses.
induction agents main aim is to induce progress to delirium after waking up
rapid loss of consciousness to facilitate Ketamine from sedation)
ease of intubation.
• 1-2mg/kg IV, 4-5mg/kg IM. Propofol
Medication for induction
• NMDA receptor antagonist. • 1.5 to 2.0 mg/kg IV,
Etomidate
• Ketamine produces a loss of awareness • It produces significant venous dilation,
• 0.2-0.3 mg/kg IV within 30 seconds, peaks in myocardial depression and can reduce
cerebral perfusion pressure.

667
• Because of the propensity of propofol • It can rise serum potassium levels, and minutes. It can be reversed by
to cause hypotension, through both is contraindicated in the following Sugammadex which can be an
vasodilation and direct myocardial circumstances: advantage in some circumstances.
depression, the dosage is reduced, or
• Hyperkalemia A video about RSI drugs.
the drug is avoided altogether in
hemodynamically compromised • Patient ≥5 hours post burn Protection and Positioning
patients. In-line Stabilization: In cases of trauma in
• Patient ≥5 days post crush injury
which cervical spine injury is suspected
Other agents: Benzodiazepines like or denervation
and not yet ruled out, protection of the
midazolam and barbiturates like
• Neuromuscular diseases cervical spine is a priority and intubation
thiopental and methohexital
(amyotrophic lateral sclerosis, must be performed without movement of
Paralytic agents multiple sclerosis, muscular the head. An assistant is required to
dystrophy) maintain inline stabilization. This allows
Neuromuscular blocking agent (NMB) are
the cervical collar to be opened giving
mainly divided into depolarizing agents • Denervation (stroke, spinal cord better access. The head and neck are
(DPA) and non-depolarizing agents injury) >5 days until 6 months post maintained in the neutral position.
(NDPA) injury
If no cervical spine injury is suspected
Succinylcholine • Intra-abdominal sepsis >5 days flexing the neck and extending the head
until resolution to the so-called sniffing position helps to
• 1.5 -2 mg/kg.
align the axes and facilitates visualization
Rocuronium
• It is the only DPA used in the of the glottic opening.
emergency room having a rapid onset • 1-1.2 mg/kg.
and short half-life. It takes 45-60 Placement with proof
seconds to induce paralysis and takes • NDPA. Intubation should be performed carefully
8-10 min to recover. and gently. After flaccidity is achieved
• It has a comparable time to paralysis
laryngoscopy glottis is visualized, the
but a longer recovery time of 35-45
668
clinician places the endotracheal tube prepared and started as soon as
between the cords, inflates the cuff, possible. A post-procedural chest x-ray is
withdraws the stylet, and confirms obtained to confirm the depth of tube
placement. placement and to evaluate for evidence
of barotrauma as a consequence of
A video about intubation details.
positive pressure ventilation. Oro-Gastric
Intubation video. tube and urinary catheter insertion used
to decompress the stomach and monitor
Confirmation of proper endotracheal tube urine output respectively.
(ETT) placement is crucial; unrecognized
esophageal intubation leads to
devastating complications.
References and Further Reading, click
C o n fir m t h e p l a c e m e n t b y a here
combination of

• Visualizing the passage of the ET tube


between the cords

• Listening to both sides of the chest and


over the stomach

• End-tidal CO2 (EtCO2) determination


(either colorimetric or quantitative).

Post-intubation management
After intubating the patient, the tube is
tied or taped in place. Maintaining
sedation is essential; infusions should be
669
Section 15

Reduction of Common Fractures and Dislocations

Introduction
by Dejvid Ahmetović and Gregor Prosen Most of the orthopedic injuries can be predicted considering the chief complaint,
the age of the patient and the mechanism of the injury itself. Additionally, a careful
physical examination and the patient’s history can often predict radiographic
findings with great accuracy. If an injury is suspected by clinical examination but
cannot be completely confirmed by evaluating the radiograph, the patient should
be treated as if the injury is present and discharged with detailed instructions on
how to look out for any additional signs of neurovascular complications,
compressions, and cast care.

Injuries to the musculoskeletal apparatus include one or more of the following


structures: bone, joints, ligaments, tendons and in some cases vasculature and
nerves.

Simple definitions of the injuries to the musculoskeletal system include the


following:

Fracture: A disruption of bone tissue, which may be caused by the application of a


force that exceeds the strength of the bone tissue itself, repetitive stress to the
bone tissue, or invasive processes that weakens the bone structure and integrity.

670
Dislocation: Complete disruption of a joint, whereby articular Illustration 16.7 Type of fractures
surfaces are forced from their normal position, which immobilizes
the joint temporarily. In the case of a  subluxation, there is still a
partial contact of the articular surfaces.

Fractures
Orthopedic injuries commonly result from accidents and often
involve otherwise healthy individuals, especially in the younger
population. Accurate diagnosis and treatment are of great
importance both economically and medically.

Most fractures result from excessive force applied to otherwise


healthy bone tissue, resulting in disruption of the bone cortex.
Disruption may occur from a variety of forces, including a direct
blow, axial loading, bending forces, torque forces and
combinations of these.

All fractures are either simple or multifragmentary (communited).


A simple fracture (spiral, oblique or transverse), is a single
circumferential disruption of any part of the diaphysis, metaphysis
or articular surface. A multifragmentary fracture (communited) is
any fracture with one or more completely separated fragments,
which can be further classified as either wedge or complex. A
wedge fracture consists of fragments that, after reduction, main
fragments still have some contact between themselves; in a
complex fracture, however, one or more intermediate fragments
and main fragments are no longer in contact after reduction.

671
Description of Common adolescents. Damage to the growth plate Illustration 16.8 SALTER-HARRIS
during growth may destroy part or all of Classification
Fractures
Pathologic fractures:  A type of injury its ability to produce new bone, thus
that results from a relatively small force preventing elongation of the bone, which
applied to otherwise diseased or may lead to anatomical and functional
weakened bone, which in normal deformities.
circumstances would not disrupt the
Conveniently, the Salter-Harris fracture
cortex. Examples of such types of injuries
types can be memorized by the
are fractures through metastatic lesions,
mnemonic SALTR.
fractures through benign bone cyst and
vertebral compression fractures in S – (slipped), fracture plane passes all the
individuals with advanced osteoporosis. w a y t h r o u g h t h e g r o w t h p l a t e .

A – (above), Fracture passes through
Stress fractures:  These types of
most of the growth plate and up to the
fractures involve ‘fatigued’ bone tissue
m e t a p h y s i s .

that was exposed to repetitive forces. The
L – (lower), A fracture that passes through
bone and supportive tissue did not have
the growth plate and extends down
enough time to adequately accommodate
t h r o u g h t h e e p i p h y s i s .

such forces. A common example is the Open fractures:  An open fracture is a
T – (through, transverse or together), A
fracture of the metatarsal shaft in fracture associated with overlying soft
f r a c t u re p a s s i n g d i re c t l y t h ro u g h
unconditioned foot soldiers and athletes. t i s s u e i n j u r y, c a u s i n g a n o p e n
metaphysis, growth plate and epiphysis.

It is known as ‘march fracture.’ communication between the fracture or
R – (rammed, ruined), An uncommon
crushing type of injury that does not dislocation and the environment.
Salter-Harris fractures:  Fractures
involving the physis and cartilaginous displace the growth plate but damages it
by direct compression.
Reduction
epiphyseal plate near the ends of the long Reduction of fractures  includes many
bones in still growing children and options, some of which are appropriate
672
for one type of injury and some for reduction must be anatomical, or it could o r closed fixation of the bone defect.
another. The reduction can be either lead to joint incongruity and arthrosis. O p e n re d u c t i o n re q u i re s s u rg i c a l
anatomical or non-anatomical. intervention for alignment of the fracture
Treatment Options
fragments; however, in closed reduction,
Non-anatomical reduction  in children is
General steps in fracture treatment the fracture is reduced by manual
used for extra-articular fractures.
a r e  r e d u c t i o n ,  i m m o b i l i z a t i o n , manipulation of the affected area. There
Because of the remodeling potential in
and rehabilitation. is also a difference between internal and
children, most deviations and anatomical
external fixation. The term internal fixation
positions will be corrected spontaneously, Conservative treatment  involves either itself suggests that the immobilizing
but only if no rotation is present. In functional treatment or closed implant is under the skin (bone surface or
adults, for example, fractures of the immobilization with or without any closed intramedullary), and external fixation
humeral shaft, deviations, and non- reduction. It is indicated in non-displaced presents in the case when most of the
anatomical positions are well tolerated fractures and when a certain degree of fixation material is outside the skin. (With
both functionally and cosmetically. The displacement is acceptable. Examples this method, the risk of infection of the
same applies to femoral and tibial shaft include clavicular, scapular and rib fracture is minimal. It is mostly used in
fractures when length, rotation, and axis fractures, most stable vertebral types of severe open fractures.)
remain the same. fractures and pelvic fractures, also when
the pelvis is stable, most extra-articular Generally, the  indications for surgical
Anatomical reduction  in children is
fractures in children because of their treatment  are open fractures, displaced
indicated in the case of some epiphyseal
remodeling potential, and any extra- intra-articular fractures, avulsion fractures
fractures, especially in those that are
articular fractures when the anatomical and all femoral shaft fractures.
intra-articular because if the reduction is
position can be reached by closed
not perfect, the gap will be filled with Reductions of Selected
reduction and maintained by closed
callus, which can consequently cause Fractures
external immobilization.
premature closure of the growth plate. In Any standard reduction procedure should
intraarticular fractures in adults, the Surgical treatment  involves open or include these steps
closed reduction of the fracture and open

673
1. Confirming fracture with  imaging  if 7. Axial traction, rotation, or angulation Image 16.18
there is no neurovascular compromise maneuvers may be necessary for the
and immediate reduction is needed different type of fractures. Therefore,
apply proper technique accordingly.
2. Defining the need  of reduction
procedure 8. After the reduction, please make sure
the reduction is acceptable. Therefore,
3. Explaining the procedure to the patient
you can use imaging for the
and getting his/her  consent  for
confirmation.
reduction and sedation and analgesia.
You may also prefer to use hematoma 9. S t a b i l i z e t h e e x t r e m i t y a s
or regional blocks. recommended in the  Splinting /
Casting chapter.
4. Prepare the team and the equipment.
Some fracture reductions may need 10.If you are going to discharge the
more than one person if you are not patient do not forget to give discharge Image shows comminuted fracture of the middle
phalanx of 2nd finger.
using special traction devices for the instructions and arrange follow up with
reduction purpose. Prepare the post- orthopedic clinic. Pain, swelling, typical angulation because
reduction splinting/casting equipment of extensor tendon are typical
as discussed in the Splinting / Casting Fracture of the middle phalanx
presentation.
chapter.
Treatment is conservative in case of
The mechanisms;
5. Properly place the patient and injured proper reduction. However often surgical
extremity. • direct force caused by fall, fixation required. In general,  simple axial
traction  is enough to align the fractured
6. Properly position yourself and other • blow from a heavy object, phalanx. However,  keeping the fractured
team members.
• twisting force. parts in an acceptable alignment can be

674
difficult. Therefore, immediate splinting/ different specific maneuvers, please fracture in the elderly. Fall on
casting required. watch sample videos (video 1 and video 2 an  outstretched hand  is the primary
and Video 3) mechanism of injury.
Boxer’s fracture
Colles’ fracture Treatment options are conservative
Image 16.19 treatment with marginally displaced
Image 16.20 fractures and surgical (open reduction
internal fixation (ORIF), external fixation)
with severe displacement and unstable
reduction.

Watch video 4 and video 5.

Nightstick fracture

Image 16.21
It is a fracture of the neck of the 4th. or
5th metacarpal. Image shows 5th
metacarpal neck fracture. Swelling, pain
and obvious deformity are seen in the
presentation. Striking a clenched fist into
Fracture of the distal radius, with dorsal
an immovable object is the most
displacement and volar angulation.
common mechanism.
Swelling and reduced movement and
Treatment is conservative for acceptable
characteristic clinical deformity named
angulation (30° for 5th metacarpal) after
‘dinner fork deformity’  are seen in the
reduction, surgical with severe
presentation. It is the most common wrist
displacement. The reduction requires

675
It is a fracture of the shaft of either radius Fracture of the femoral shaft T o learn how to apply Hare Traction
or ulna or both. Splint, please watch video.
Image 16.22
Deformity and pain are prominent in the Dislocations
presentation. The name of the fracture Any standard reduction procedure should
derived from citizen trying to defend include these steps
against baton or nightstick, offering
forearm. Caused by direct force, blow or 1. Confirming dislocation with  imaging  if

impact. there is no neurovascular compromise


and immediate reduction is needed
Treatment is the conservative/closed
reduction in stable and only slightly 2. Defining the need  for reduction

displaced fractures. Long arm cast with procedure

elbow 90°. The surgical option is ORIF 3. Explaining the procedure to the patient
with plate fixation in unstable fractures. The figure shows a complex segmental
and getting his/her  consent  for
fracture of the shaft.
Forearm reduction video. reduction and sedation and analgesia.
Swelling, deformity, loss of function, pain, Yo u m a y a l s o p r e f e r t o u s e
exter nal rotation are presentation the intraarticular anesthetic agent.
findings.
4. Prepare the team and the equipment.
The mechanism is a direct or axial force Many dislocation reductions may need
of high energy. The reduction with axial more than one person if you are not
traction should be applied in order to using special traction devices for the
d e c re a s e p a i n , h e m o r r h a g e , a n d reduction purpose. Prepare the post-
anatomical alignment. After the reduction, reduction splinting/casting/sling
traction splint should be placed. The equipment as discussed in
definitive treatment is surgical (ORIF) with the Splinting / Casting chapter.
plate or nail.
676
5. Properly place the patient and injured Dislocation of the Pain, deformity are two main
extremity. interphalangeal joints of the characteristics at the presentation.
fingers
6. Properly position yourself and other Axial loading and hyperextension cause
team members. the dislocation. Dislocations are usually
Image 16.23
dorsal.
7. Axial traction, rotation, or angulation
maneuvers may be necessary for the Longitudinal traction and hyperextension
d i ffe r e n t t y p e o f d i s l o c a t i o n s . with applying dorsal pressure to the base
Therefore, apply proper technique of the dislocated phalanx usually
accordingly. reduce the dislocation.

8. After the reduction, please make sure Finger dislocation reduction video.
the joint is in normal anatomy.
Figer dislocation metacarpal block video.
Therefore, you can use imaging for the
confirmation.
Shoulder dislocation
9. S t a b i l i z e t h e e x t r e m i t y a s
recommended in the  Splinting / Image 16.24
Casting chapter.

10.If you are going to discharge the The image shows a fracture and subluxation at
patient do not forget to give discharge the distal and fracture and dislocation at the
 proximal phalangeal joint.
instructions and arrange  a follow up
with the orthopedic clinic. Dislocations of the PIP joint are a The images show loss of shoulder curve on the
common hand injury, as opposed to DIP left, and X-ray of the same patient with anterior
Reductions of Selected dislocation, which is rare, because of the
shoulder dislocation and severe Hill Sacks
deformity (cortical depression in the
Dislocations firm attachments of the skin and posterolateral head of the humerus) + fracture.

surrounding tissue to the bone.


677
Anterior glenohumeral dislocation, the Tr y w a t c h i n g t h i s v i d e o o n I n the anterior dislocation, the leg is in
most common type of shoulder www.youtube.com, or enable JavaScript extension, abduction and external
dislocation. Pain and swelling are if it is disabled in your browser. rotation.
common, and normal rounded contour of
10 ways to reduce dislocated shoulder - High-energy trauma with the flexed knee
the shoulder is lost. The patient is unable
video.  (e.g., dashboard injury).
to move, supported with healthy arm.

The mechanism is generally a fall on the Cunningham technique to reduce Conservative reduction is necessary, if
hand when in the externally rotated shoulder dislocation - video. unsuccessful, reoccurring or any type of
position. It is common in age 18-25, acetabular injury, surgical treatment is
mostly due to sport or motorbike injury. Hip dislocation required.

The majority of the cases are successfully Watch the video for reduction of the hip.
Image 16.25
reduced with simple maneuvers
(conservative reduction). Traction and Artificial hip dislocation - reduction -
external rotation-elevation, scapular video.
rotation, Cunningham are the most
popular techniques. However, there are
many other successful methods. Please References and Further Reading, click
do not use the Hippocrates and Kocher here
techniques because of their high The images show posterior hip dislocation.

complication rate. If the reduction is


In approximately 95% femoral head is
unsuccessful, it can be done under
dislocated posteriorly.
sedation or even in general anesthesia in
some cases. In the posterior dislocation, the leg
position is in flexion, adduction and
internal rotation.
An error occurred.
678
Section 16

Splinting and Casting

Case Presentation 1 Image 16.26


by Joseph Pinero, Timothy Snow, Suzanne 65-year-old female with a
Bentley
history of hypertension and
diabetes presenting with right
wrist pain and swelling after
suffering a fall from standing,
landing on an outstretched
hand. Plain radiographs of
the wrist will show a distal
radius fracture, otherwise Procedure: Sugar Tong
known as a Colles fracture. Emergency Indication

 
 • Colles Fracture (Distal Radius


Fracture)

• Distal Ulna Fracture

• Smith Fracture (Reverse Colles


Fracture)

679
• Barton Fracture (Dorsal or volar rim • Contraindicated if overlying cellulitis •The plaster should be measured
fracture of the distal radius) or grossly contaminated wound prior to placement and should be
8-10 layers thick
Precaution • Clean skin with an antiseptic solution
4. Place the patient in a sling and perform
The presence of an open fracture requires • First, anesthetize skin with a small
post-reduction plain radiographs
a n e m e rg e n t o r t h o p e d i c s u rg i c a l wheal of lidocaine
consultation and surgical fixation with Please watch the video.
• Then dive deeper into the largest
open reduction and internal fixation.
area of swelling and hematoma and Hints and Pitfalls
Equipment and Patient Preparation aspirate blood. Once confirmed that
• Using too much padding can
you are within the hematoma, gently
• Syringe cause your splint not to provide enough
inject 10-15 cc of anesthetic.
support, resulting in malunion.
• Lidocaine • Wait at least 10 minutes for the
• Using too little padding can
anesthetic to be absorbed prior to
• 4” Plaster result in plaster burning the skin.
beginning your manipulation.
• Soft web roll lining
3. Perform a closed reduction with the Post Procedure Care and
• 4” ace wrap aim of creating a neutral volar tilt (15- Recommendations
degree angulation in wrist flexion)

• Sling Always take post-reduction radiographs
○ The sugar tong splint should be
Procedure Steps applied by placing a U-shaped splint Complications
from the dorsal metacarpal-phalangeal
1. Obtain standard radiographs including • Malunion
joints down around the elbow joint and
posteroanterior and lateral films
wrap back around to come up to just • Nonunion
2. Local anesthesia via hematoma block below the metacarpal-phalangeal
joints on the palmar surface. • Median nerve injury

680
Case Presentation 2 Procedure: Posterior Long •4” Plaster

11-year-old male with no past Arm Splint • Soft web roll lining
Emergency Indication
medical history presents with
• 4” ace wrap
right arm pain around the • Supracondylar fracture
• Sling
elbow after falling off of the • Distal Humerus fracture
monkey bars at the playground Procedure Steps
• Monteggia’s fracture
earlier today. Plain radiographic 1. Obtain standard radiographs including
• Proximal forearm fractures
films will show a supracondylar posteroanterior and lateral films

fracture. • Radial head and neck fractures


2. Local anesthesia is typically not
• Olecranon fractures effective in this group as these
Image 16.27 fractures are typically seen in young
• Severe ligamentous injuries of the children that require minimal
elbow manipulation if any at all.

Precaution 3. If a child cannot tolerate the placement


of the splint, it is possible to utilize
The presence of an open fracture requires
procedural sedation (administration of
a n e m e rg e n t o r t h o p e d i c s u rg i c a l
a small amount of sedative making the
consultation and surgical fixation with
child less aware of the procedure).
open reduction and internal fixation
4. Placing the splint requires pre-
Equipment and Patient Preparation
measurement of plaster casting
 
 • Syringe material from the proximal posterior
humerus at the axillary crease to the
• Lidocaine

681
wrist joint without crossing into the Complications Case Presentation 3
hand.
• Malunion
26-year-old male with no past
• The plaster should be layered at medical history presents with
8-10 layers thick, as this is a long • Nonunion
left ankle pain after landing on
arm splint and will be heavier than • Median nerve injury
the average short arm splint.
another player’s foot while
• Pressure ulcers jumping up during a basketball
• The elbow should be placed at 90
degrees of flexion with the wrist in a • Decreased range of motion game.
neutral position, which is neither
supinated nor pronated.
Image 16.28
5. Place the patient in a sling and
perform post-reduction plain
r a d i o g r a p h s .

Hints and Pitfalls

6. Be sure to place the arm in the proper


p o s i t i o n o f n e u t r a l i t y .

Post Procedure care and
recommendations

7. Make sure to provide adequate


analgesia both before and after the
placement of the splint.

Please watch the video.

682
Procedure: Short Leg Splint • 4” or 6” Plaster (depending on the size •Wait at least 10 minutes for the
Emergency Indication of the leg) anesthetic to be absorbed prior to
beginning your manipulation.
• Ankle fracture • Soft web roll lining
3. Placing a short leg splint for this type
• Tibia fracture • 6” ace wrap
of fracture involves pre-measuring two
• Crutches separate strips of plaster.
• Severe ankle sprain

Procedure Steps • The first strip is measured from the


• Metatarsal fractures
superior calf to just beyond the toes
Precaution 1. Obtain standard radiographs including of the foot for your posterior support
posteroanterior and lateral films
The presence of an open fracture requires • The second strip is measured from
a n e m e rg e n t o r t h o p e d i c s u rg i c a l 2. Local Anesthesia via hematoma block the head of the fibula, down and
consultation and surgical fixation with around the heel, up to the tibial
• Contraindicated if overlying cellulitis
open reduction and internal fixation. plateau medially.
or grossly contaminated wound
Equipment and Patient Preparation • Once both strips are measured, wrap
• Clean the skin with an antiseptic
the leg in web roll padding and place
• Chucks (or any material that you can solution
the plaster on the leg with the U-
use to keep the counters and floors
• First, anesthetize skin with a small shaped splint above the posterior
clean)
wheal of lidocaine support strip.

• Water source (basin half full of water will


• Then dive deeper into the wound • Cover both with a thin layer of the
suffice)
overlying the largest area of swelling web roll, followed by your ace

• Syringe and hematoma and aspirate blood. bandage.


Once confirmed that you are within
• Lidocaine • The leg will typically require more
the hematoma, gently inject 10-15 cc
t h a n o n e 6 ” a c e w r a p

of anesthetic.
683
Perform post-reduction plain Case Presentation 4 Procedure: Long Leg
radiographs and educate the patient Splint
A 8-year-old male presenting
on the use of crutches
with thigh pain after falling from
Please watch video.
a bicycle. Plain radiographs Emergency Indication

Hints and Pitfalls show a fracture of the distal • Distal femur fracture

Test for neurovascular function femur.


• Tibia plateau fracture

Post Procedure care and Image 16.29 Precaution


recommendations
The presence of an open fracture requires
Make sure to provide adequate analgesia a n e m e rg e n t o r t h o p e d i c s u rg i c a l
both before and after the placement of consultation and surgical fixation with
the splint. open reduction and internal fixation.

Complications Equipment and Patient Preparation

• Malunion • Chucks (or any material that you can


use to keep the counters and floors
• Nonunion
clean)
• Pressure ulcers
• Water source (basin half full of water will
• Decreased range of motion suffice)

• Early onset arthritis


 
 • Syringe

• Lidocaine

684
• 4” or 6” Plaster (depending on the size • Wait at least 10 minutes for the Please watch video.
of the leg) anesthetic to be absorbed prior to
beginning your manipulation.
• Soft web roll lining
Post Procedure Care and
3. Placing a long leg splint for this type of
• 6” ace wrap Recommendations
fracture involves pre-measuring three
• Crutches strips of plaster: one from the plantar Make sure to provide adequate analgesia
surface of the toes to the gluteal fold, both before and after the placement of
Procedure Steps one support strut from the medial the splint.
ankle up to the proximal inner thigh,
1. Obtain standard radiographs including
and the last support strut from the Complications
posteroanterior and lateral films
lateral ankle up to the greater
• Malunion
2. Local Anesthesia via hematoma block trochanter of the femur.
• Nonunion
• Contraindicated if overlying cellulitis 4. Wrap the leg in web roll padding
or grossly contaminated wound • Pressure ulcers
5. Firmly secure the splint with a top layer
• Clean skin with an antiseptic solution o f w e b r o l l a n d a c e w r a p
 • Decreased range of motion
Hints and Pitfalls
• First, anesthetize skin with a small • Early onset arthritis
wheal of lidocaine 6. Test for neurovascular function
 
• Then dive deeper into the wound 7. If the fracture is in the midshaft of the
overlying the largest area of swelling femur or proximal femur, casting is not
and hematoma and aspirate blood. an appropriate option. Orthopedic
Once confirmed that you are within consultation and traction fixation will
the hematoma, gently inject 10-15 cc be required temporarily, prior to
of anesthetic. surgical fixation.

685
Case Presentation 5 Procedure: Short Leg Cast •Soft web roll lining

54-year-old female with no • 6” ace wrap X 2


past medical history presents Emergency Indication
• Crutches
with right ankle pain after
• Definitive treatment for nondisplaced
Procedure Steps
stepping off of a curb and a n k l e a n d f o o t f r a c t u r e s

“rolling over her ankle.” X-ray Precaution 1. Obtain standard radiographs including
posteroanterior and lateral films
shows a non-displaced • The presence of an open fracture
malleolar fracture. requires an emergent orthopedic 2. Local anesthesia via hematoma block,
surgical consultation and surgical and for more severe fractures,
Image 16.30 fixation with open reduction and internal systemic analgesia may be required.
fixation.
• Contraindicated if overlying cellulitis
Equipment and Patient Preparation or grossly contaminated wound

• Chucks (or any material that you can • Clean the skin with an antiseptic
use to keep the counters and floors solution
clean)
• First, anesthetize skin with a small
• Water source (basin half full of water will wheal of lidocaine
suffice), lukewarm temperature
• Then dive deeper into the wound
• Syringe overlying the largest area of swelling
  and hematoma and aspirate blood.
• Lidocaine
Once confirmed that you are within
• 4” and 6” Plaster (depending on the the hematoma, gently inject 10-15 cc
size of the leg) of anesthetic.

686
• Wait at least 10 minutes for the prevent “bananaing” of the plaster tibial plateau) to prevent pressure
anesthetic to be absorbed prior to when applying. ulcers.
manipulation.
• Plaster or fiberglass is then wrapped • Test for neurovascular function after
3. Placing a short leg cast for this type of around the foot and ankle up to the casting
fracture involves first ensuring the foot proximal tibia ensuring it remains in
• Short leg casts are often “bi-valved” or
is sitting in proper anatomical 90 degrees of flexion.
cut in half prior to discharge from the
alignment ankle flexed at 90 degrees.
• A foot plate can be added by placing hospital. This is done in order to allow
• The first step is to adequately pad 6 layers of plaster on the bottom of for some room for swelling.
the entire area of casting (from them for support.
Post Procedure Care and
1-2cm distal to the tibial plateau) to
• To prevent cutting off blood flow to Recommendations
the distal foot (covering the base of
the distal foot when the injury swells,
the phalanges) leaving the tips of the Make sure to provide adequate analgesia
bi-valving of the cast should be done
toes uncovered. Extra padding both before and after the placement of
by cutting along the medial and
should be placed at the areas of the splint.
lateral shin, through the plaster or
pressure (the ends of the cast) and
fiberglass.
the heal to prevent ulcers. Complications
4. Give the patient crutches and perform
• The second step is to apply the • Malunion
post-reduction plain radiographs
plaster ensuring the ankle remains at
• Nonunion
90 degrees of flexion. The entire rolls Please watch video.
of plaster are dipped and soaked in • Pressure ulcers
lukewarm water and then squeezed Hints and Pitfalls
to remove some of the water. The • Decreased range of motion
• Extra padding should be applied to
thumb and index finger should be
areas of pressure (tips of toes, heal at • Early onset arthritis
placed at each end of the plaster to
the malleoli and top of the cast at the

687
• Contracture of the Achilles tendon if the foot is <90 degrees of
flexion

References and Further Reading, click here

688
Section 17

Urinary Catheter Placement

Case Presentation
by Gul Pamucu Gunaydin A 75-year-old male patient was admitted to the emergency
department with difficulty voiding. He had this complaint for
over a year, and tonight, although he felt pain and distention in
his lower abdomen, he could not urinate at all. On his physical
exam, the patient had a palpable mass that was thought to be
the distended bladder. He was agitated and tachycardic. He
was diagnosed with acute urinary retention, and initial attempt
to insert urinary indwelling catheter was failed. The second
attempt with a Coude catheter was successful and 2 liters of
urine was drained gradually. His rectal exam revealed prostate
enlargement. He was discharged with instructions,
uneventfully.

Procedure: Urinary Catheter Placement


Urinary catheter insertions is a common procedure in the ED. They may be
external (condom) or indwelling (urethral, suprapubic). Condom catheters are
indicated in men with functional disabilities such as restricted mobility or dementia

689
with incontinence, who can void • Draining urine in acute urinary retention, Absolute
spontaneously. Suprapubic catheters are urinary obstruction, inability to void
an option if urethral catheters fail. This • Trauma patient presenting with the
• Irrigation of bladder to remove gross following signs (known or suspected
chapter focuses solely on urethral urinary
hematuria and clots/debris urethral damage):
catheterization.
• Palliative care for terminally ill (e.g.to • Blood at meatus
Emergency Indications
assist treatment of decubitus ulcers in
incontinent patients by maintaining • Penile deformity
Short-term catheterization
moisture free environment) • High riding prostate

Diagnostic • To warm hypothermic patients • Perineal hematoma

• Diagnostic sampling (sterile urine • Intubated patient • Allergy to latex, rubber or lubricants
sampling)
• Emergency Surgery Relative
• Monitoring urinary output (trauma,
critically ill, burns) Long-term catheterization • Uncooperative patient
• Bladder outlet obstruction
• Filling the bladder prior to pelvic • Recent bladder or urethral surgery
• To reduce changes in patients who are
ultrasound
terminally ill or cannot care for • Urethral Stricture
• Cystogram, cystourethrogram themselves
Equipment and Patient
• Urine collection • Neurogenic bladder Preparation
Urinary catheter: Catheters are classified
• Monitoring core body temperature • Urinary incontinence according to the material it is made of,
number of lumens and shape of the tip.
Therapeutic Contraindications
Number of lumens

690
• One way-non balloon also known as • The Roberts tip catheter has an eye •Sterile local anesthetic lubricant gel:
straight, Nelaton or Robinson catheters above and below the balloon to reduce (% 2 lidocaine gel) anesthetizing the
are used for one time or intermittent the residual urine. urethra with topical lidocaine gel
drainage. instilled through a pre-loaded syringe
Catheter size is described in French units.
reduces discomfort. The catheter tip is
• Two-way catheters have a balloon It refers to the catheter’s circumference in
also lubricated prior to its insertion.
inflation channel and a urine drainage millimeters. Start with 12-16 F for adults.
channel. Choose the smallest size that is enough • 10 ml syringe filled with sterile saline or
for adequate drainage. If obstruction of sterile water
• Foley catheter, which has a self-
the catheter due to blood or debris is
retaining balloon, is the most commonly • Sterile urine bag
expected, use a larger bore catheter (e.g.,
used.
18-24 F). • Tape to secure the urine collection
• The triple lumen (three-way) indwelling system
Catheter length: Adult indwelling
catheter is used for bladder irrigation.
catheters are available in a standard
Procedure Steps
Shape of Tip (male) length (40-45cm) and a shorter Universal precautions should be taken in
female length (20-26cm). Female length all steps. Patient consent should be
• Coude or Tieman catheter curves 45 catheters should not be used in male obtained before starting any procedure.
degrees at the tip and is designed to patients because of the risk of inflating Ensure the privacy of the patient. Aseptic
pass urethra in patients with prostatic the balloon in the urethra. insertion technique is recommended.
enlargement; it offers rigidity too.
Other Equipment:
• The Whistle Tip (Couvelaire Tip)
Female Patients
• Sterile gloves and drapes 1. Prepare all equipment on a tray
catheter has a terminal and a lateral
covered with a sterile drape in a sterile
drainage eye used for large blood clots.
• Sterile gauze sponge or cotton balls fashion.

• Antiseptic solution (Povidone-iodine or 2. Place the patient in the lithotomy


chlorhexidine) position.

691
3. Wear your sterile gloves. 11.Inflate the balloon with 10 ml of sterile solution and paint the area in a sterile
water or saline using the filling port. fashion with the antiseptic solution.
4. Check the balloon for patency.
12.Pull the catheter back until resistance 7. Alternatively, you may change gloves
5. Place a fenestrated drape over the
is felt. after cleansing external genitals.
perineum.
13.Attach the urine collection bag. 8. Inject 10 mL of 2% lidocaine gel into
6. Spread the labia with your non-
the urethra through the meatus before
dominant hand. 14.Secure the catheter to the anterior
insertion of the catheter.
thigh.
7. Use the forceps/pickups to hold the
9. P e r f o r m s t e p 1 0 - 1 5 o f f e m a l e
sterile sponge, soak it in the antiseptic 15.Remove gloves, dispose of waste
catheterization.
solution, and clean the area from appropriately, and wash hands.
anterior to posterior and central to 10.When the procedure is finished, don’t
Please watch below videos (manikin and
peripheral. forget to reduce foreskin to prevent
patient examples)
iatrogenic paraphimosis.
8. Alternatively, you may change gloves
after cleansing external genitals. Male Patients Please watch below videos (manikin and
Perform step 1 to 4 of female patient
patient examples)
9. Lubricate the tip of the catheter with catheterization.
%2 lidocaine gel. Hints and Pitfalls
5. Firmly hold the penis with the non-
• Universal availability and ease of
10.Pass the catheter through the meatus dominant hand, and position the penis
insertion of urinary catheters often lead
and advance it until the hub meets the 45 to 90 degrees to the coronal plane,
to the inappropriate and prolonged use
urethral meatus, you should be able to apply gentle traction. Retract the
of these catheters. Insert catheters only
see urine flowing. Insert the catheter foreskin if the patient is not
for appropriate indications and leave
2-3 inch or 5-7.5 cm more, preferably circumcised.
catheters in place only as long as
until the hub to avoid inflating the
6. Use the forceps/pickups to hold the needed.
balloon inside the urethra.
sterile sponge, soak it in antiseptic
692
• A tense patient means a tight urethral • Place the urinary drainage bag below •Patients’ follow up with urology
sphincter; encourage the patients to the level of the patient’s bladder, not should be arranged.
relax by taking deep breaths and relax allowing it to touch the floor.
• Discharge instructions:
urinary sphincter muscles as if going to
• For difficult urinary catheterization,
void. • If you develop any symptoms of a
change the size: 20-24 F catheter for
urinary tract infection, contact your
• Always be gentle; never force the benign prostate hyperplasia, small
doctor immediately.
catheter since this may cause urethral caliber for the urethral stricture (12-16
trauma. F). • Take enough fluids to maintain
adequate urine flow.
• If no urine has returned, do not inflate • If catheterization is unsuccessful, it is
the balloon. best to avoid multiple blind attempts • Be careful not to pull the catheter
since they increase the risk of infection, accidentally, avoid twisting and
• Even when urine is flowing, it is
exacerbate the patient’s discomfort, kinking of the catheter.
possible for the eye of the catheter to
and produce urethral congestion and
lie within the bladder while the balloon • Keep the bag lower than the bladder
edema, rendering further attempts even
remains within the prostatic urethra; so, to prevent back flowing.
more challenging.
always advance the catheter until the
hub. • Patients occasionally experience • Avoid disconnecting the catheter and
hypotension and hematuria when the drain tube.
• If there is pain during inflation of the
large volume from the bladder is
balloon, stop immediately since the • Empty the bag regularly. The
drained rapidly but has little clinical
balloon may still be in the urethra. drainage spout should not touch
significance, and gradual emptying is
anything while emptying the bag.
• Once inserted, the catheter should be not necessary.
secured to prevent traction and damage • Alpha blockers may be started to
Post-Procedure Care and patients with prostate enlargement.
from movement and catheter kinks.
Recommendations
Complications

693
• Discomfort, pain • Vena cava air embolism • Pyelonephritis

• Inability to pass the catheter • Infections: UTI accounts for 32% of all • Bacteriemia, urosepsis
healthcare-associated infections. A
• Misplacement of the catheter • Latex allergies
majority of these infections are
• Vagina attributable to the use of an indwelling • Obstruction or blockage of catheter
c a t h e t e r. U s e o f b e s t p r a c t i c e results from precipitated mucus,
• Ureter techniques by emergency nurses can protein, crystals, blood clots, and
help prevent UTIs from occurring as a bacteria.
• Renal Pelvis
result of urinary catheter insertions in
• Traumatic complications to lower the emergency department. Earlier • Urine leakage around the catheter
urinary tract – proper insertion catheter removals, use of smaller bore
• Fragmentation or fracture and
technique is the single most important catheters, a closed drainage system,
retainment of the catheter
factor for preventing injury. optimal hygienic techniques (hand-
washing, sterile catheterization • Catheter knotting
• Passage of the catheter into a false
techniques) by health care workers, and
lumen • Balloon rupture
removal of the catheter when infection
• Intraurethral balloon distention is suspected are effective in minimizing • Calculi formation
the incidence of infection.
• Hematuria • Bladder spasms contraction
• Urinary tract infection
• Rupture of urethra 11 (may cause • Accidental removal of the catheter
urethral stricture in the long term) 5 • Urethritis
• Stricture formation in long-term
• Bladder perforation • Prostatitis
Pediatric, Geriatric, Pregnant
• Hydro uterus • Epididymoorchitis Patients and Other
Considerations
• Paraphimosis • Cystitis

694
• Use 6-10 F catheters for pediatric
patients, 12F for patients age >12
years, 5F for infants

• Difficult urethral catheterization (DUC) is


where the urological consult is
requested to insert a urinary catheter.
Many causes of DUC have been
identified including anxiety, poor
technique, urethral stricture, phimosis,
b l a d d e r n e c k c o n t r a c t u re , f a l s e
passages, benign prostatic hyperplasia,
unfavorable body habitus and patient
positioning.

• To prevent infections:

• Insert catheters using aseptic


technique and sterile equipment

• Maintain a closed drainage system

• Maintain unobstructed urine flow

References and Further Reading, click


here

695
Chapter 17

Selected
Diagnostic Tests
Section 1

Arterial and Venous Blood Gas Analysis

Introduction • Determination of the need for


by Kemal Gunaydin Measurements of PaO2, PaCO2, mechanical ventilation
SaO2, pH, and bicarbonate values are
• Evaluation of the indication for
made with arterial blood gas (ABG)
admission to intensive care
analysis in order to determine the acid-
base balance and respiratory • Determination of the effectiveness of
regulation. Arterial blood gas (ABG) the given treatment
analysis is an important laboratory
• Indication and follow-up of oxygen
method that provides reliable
treatment
information about the patient’s
metabolic status and respiratory • Evaluation of the reason for sudden
physiology. and unexplained dyspnea

Indications for arterial blood Generally, the radial, brachial and the
gas (ABG) analysis are femoral arteries are used for this
• Diagnosis and follow-up of metabolic purpose. The choice of the artery is
and respiratory acidosis and alkalosis associated with many factors. It mainly
depends on the physician’s experience
• Determination of the type of
and the patient’s clinical condition.
respiratory failure
Primarily, the radial artery is preferred.
The Allen test should be performed

697
prior to the procedure to evaluate the PaCO2: Alveolar ventilation •PaO2: Between 40-59 mm Hg,
adequacy of the collateral circulation in “moderate hypoxemia.”
PaO2 and PCO2: Gas exchange
hand. The obtained blood gas sample
• PaO2: Below 40 mmHg, “severe
should be delivered to the laboratory as Ph, PCO2, and HCO3: These are used to
hypoxemia.”
soon as possible. evaluate the acid-base status.

The normal values of the arterial blood Arterial partial pressure of


gases (Please refer to the agreed norms carbon dioxide (PaCO2)
from your lab); pH This is the partial pressure of carbon
In ABG, pH shows a status of acidosis or dioxide in the arterial blood. It is the
pH    7.35 – 7.45 alkalosis. However, it is not possible to indicator of alveolar ventilation. Its normal
understand its type with pH. pH is also value is 40 mmHg at sea level, while it is
PaCO2     35 – 45 mmHg
the only parameter showing 46.5 mmHg in venous blood. Increased
PaO2     80 – 100 mmHg compensation. Its normal values are values show respiratory acidosis, while
between 7.35-7.45. It is decompensated decreased values demonstrate
SaO2     %95 – 97
acidosis if pH<7.35, and decompensated respiratory alkalosis.
Standard HCO3     22 – 26 mEq/L alkalosis if pH>7.45.
Alveolar-arterial oxygen
Actual HCO3     22 – 26 mEq/L Arterial partial pressure of gradient – p(A-a) O2
oxygen (PaO2) This is the difference between the
BE (Base excess)     ±3 mmol/L
This is the partial pressure of oxygen in alveolar and arterial partial pressures of
the arterial blood. It is used in the oxygen, providing general information
evaluation of oxygenation. about the function of gas exchange in the
[H+]: Hydrogen ion concentration. lungs. Its normal value is 5 mmHg, which
• PaO2: Between 60-79 mm Hg, “ mild increases with age. A 4 mmHg increase is
pH: The negative logarithm of the
hypoxemia.” observed for every 10 years after 20
hydrogen ion concentration.
years of age.
PaO2: Oxygenation
698
p(A-a) O2: [150-(1.25x PaCO2)]-PaO2 metabolic status. If BE is <-2.5, it is Delta-Delta Gap (ΔAG/
metabolic acidosis, if BE >+2.5, it is ΔHCO3-)
Expected p(A-a) O2 value for age: 2.5+
metabolic alkalosis. In the presence of high AG metabolic
[0.25xage(years)]
acidosis, the “delta-delta gap” is
Anion Gap (AG) calculated to determine a second
Bicarbonate (HCO3-) The anion gap represents the difference
This is the serum concentration of the metabolic acid-base balance imbalance.
between the serum cations and the
bicarbonate ion. It is an important buffer In this case, the increase in AG is
anions. In daily practice, the measured
in the blood, and it is used to evaluate the compared with the decrease in HCO3.

cation is sodium, and the anions are
metabolic component of the acid-base AG/ΔHCO3- = (Calculated AG-12) / (24-
chloride and the bicarbonate. The normal
balance. Standard bicarbonate is the measured HCO3-)
AG is 12±4 mEq/L. Albumin constitutes
bicarbonate value that should be present
the majority of the immeasurable anions. • In the presence of high AG metabolic
in the blood under standard conditions
In patients with low levels of albumin, AG acidosis, ΔAG/ΔHCO3- = 1.
(37°C temperature and 40 mmHg PCO2).
should be considered according to the
Its normal value is 22-26 mEq/L. Actual • If there is also hyperchloremic acidosis,
level of albumin. It shows whether the
bicarbonate is the real bicarbonate value ΔAG/ΔHCO3- <1.
metabolic acidosis develops due to the
in the blood. Its normal value is 22-26
accumulation of non-volatile acids (lactic • If there is also metabolic alkalosis, ΔAG/
mEq/L. Increased values indicate
acid, ketoacids, etc.) (increased AG ΔHCO3- >1.
metabolic alkalosis, while decreased
metabolic acidosis), or due to loss of
values show metabolic acidosis.
bicarbonate (normal AG or Lactate
hyperchloremic metabolic acidosis). Lactate is a surrogate anaerobic indicator
Base excess (BE)
of metabolism, which is increased under
Metabolic acidosis or alkalosis may be AG = Na+ – (HCO3- + Cl-)
stress and hypoperfusion. It is also used
determined by looking at the base
Expected AG = Calculated AG+2.5X [4.5- as an indicator of the resuscitative efforts
excess. BE is the amount of required acid
albumin level] in patients with shock and an indicator of
or base to bring the pH of the totally
survival in patients with septic shock.
oxygenated blood to 7.40 at 37°C and 40
mmHg PCO2; it is the indicator of the
699
Levels above 4 mmol/L are associated • If PCO2 ↓ = Respiratory alkalosis- If it is more than the expected value,
with a mortality rate of 28%. metabolic acidosis concomitant respiratory acidosis is
present.
SYSTEMATIC INTERPRETATION OF If PaCO2 is normal, the change in the
THE ARTERIAL BLOOD GASES direction of pH defines a metabolic If it is less than the expected value,
disorder. concomitant respiratory alkalosis is
1th STEP present.
If pH or PaCO2 are out of normal range, • If pH ↑ = Metabolic alkalosis-
acid-base balance imbalance is present respiratory acidosis 5th STEP
If there is respiratory acidosis or alkalosis
2th STEP • If pH ↓ = Metabolic acidosis- present, the expected pH is calculated.
If pH is abnormal, and pH and PaCO2 respiratory alkalosis
move in opposite directions, the
 In acute respiratory acidosis;
primary disorder is RESPIRATORY 4th STEP
If primary metabolic acidosis or alkalosis • Expected pH= 7.4 – [ 0.008 x
If pH is abnormal, and pH and PaCO2 is detected, the expected PaCO2 is (PaCO2-40)]
move in the same direction, the
 calculated.
Chronic respiratory acidosis;
primary disorder is METABOLIC
• For metabolic acidosis;
• Expected pH = 7.4 – [ 0.003 x
3th STEP (PaCO2-40)]
• Expected PaCO2= (1.5 x HCO3) +
If one of pH or PaCO2 is normal, a mixed
8±2
acid-base disorder is present. If pH is below the expected value in acute
• For metabolic alkalosis; respiratory acidosis, there is concomitant
If pH is normal, the change in the
metabolic acidosis present.
direction of PaCO2 defines a respiratory • Expected PaCO2= 40 + (0.6 x
disorder. ▲HCO3) If it is above the expected value in
chronic respiratory acidosis, there is
• If PCO2 ↑ = Respiratory acidosis- If PaCO2 is within the expected range, concomitant metabolic alkalosis present.
metabolic alkalosis full compensation is present.
700
In acute respiratory alkalosis; • Ketoacidosis Causes of Alkalosis with Decreased
Amount of Fluid
• Expected pH =7.4 + [ 0.008 x (40 – • Uremic Acidosis
PaCO2)] • Gastric acid loss
• Methanol poisoning
In chronic respiratory alkalosis; • Vomiting
• Ethanol poisoning
• Expected pH =7.4 + [ 0.003 x (40 – • Gastric aspiration
• Ethylene Glycol poisoning
PaCO2)]
• Renal Cl loss
• Propyl Alcohol Poisoning
If pH is above the expected value in
• Use of diuretics
acute respiratory alkalosis, there is • Salicylate Poisoning
concomitant metabolic alkalosis present. • Urine Cl < 20mmol/L
• Iron Poisoning
If it is below the expected value in chronic
respiratory alkalosis, there is concomitant
metabolic acidosis present. Causes of Alkalosis with Normal Amount
Causes of Acidosis with Normal Anion
of Fluid
Gap
6TH STEP
The evaluation of the anion gap in • Mineralocorticoid excess
• Diarrhea
metabolic acidosis
• Hyperaldosteronism
• Isotonic Saline Infusion
Acid-base disorders • Bartter syndrome
• Renal failure
Metabolic • Cushing syndrome
• Renal tubular acidosis
Causes of Acidosis with Increased Anion • K loss
Gap • Acetazolamide
• Urine Cl > 20 mmol/L
• Lactic Acidosis • Ureteroenterostomy
 
701
Respiratory • Liver failure ventilation, comments can be made
easily by checking the PvCO2, pH and
Causes of Acidosis • Sepsis (+ met. acidosis)
HCO3 levels. In addition, SvO2 levels in

• Hypoventilation • Psychiatric diseases patients with central venous catheters are


very important indicators in evaluating
• CNS diseases Venous Blood Gas patients in shock. However, it is not a
Arterial blood gas sampling is an useful method to evaluate oxygenation.
• Muscle diseases
uncomfortable, painful, difficult and an To overcome this problem, it would be a
• Severe V/Q mismatch invasive procedure for the patient. sensible approach to measure the
Furthermore, the success rate of the saturation with pulse oximetry
• Chronic lung diseases p ro c e d u re m a y d e c re a s e d u e t o simultaneously.
movement of the patient or low arterial
blood pressure. Therefore, the question Arterial blood gas is a more reliable and
Causes of Alkalosis “can venous blood gas be used instead accurate method for assessing the

of arterial blood gas?” has been raised, oxygenation. Arterial and venous blood
• Brain lesion or diseases gases provide similar and very close
and many studies have been performed
on this subject. Since venous blood gas measurements in terms of PC02, HCO3,
• Centrally acting drugs or chemicals
is easy to sample from the peripheral and pH levels.
• Salicylate,
veins or the central veins in patients with

• Endotoxin, central venous catheters, it is a more


comfortable and an easy procedure for
• Progesterone some patients and the physicians.

• Hypoxemia compensation In many studies, a very good correlation


has been shown between venous blood
• Pneumonia, pulmonary edema, PTE
gas and the arterial blood gas. To
• Lack of fluid volume evaluate the acid-base disorders and

702
Table 17.1 The comparison of arterial, peripheral vein and
central blood gases
PERIPHERAL
CENTRAL VENOUS
VENOUS BLOOD
BLOOD GAS
GAS
3 to 8 mmHg higher than 4 to 5 mmHg higher than
PCO2 the arterial pH the arterial pH

0.02 to 0.04 pH units 0.03 to 0.05 pH units


pH lower than the arterial pH lower than the arterial pH

1 to 2 mEq/L higher than little or no increase in


HCO3 the arterial pH HCO3

References and Further Reading, click here

703
Section 2

Cerebrospinal fluid analysis

Introduction foramen of Luschka and foramen


by Arwa Alburaiki and Rouda Salem Alnuaimi CSF is a colorless fluid that is present Magendie, where it will be reabsorbed
within the subarachnoid space, central again by the Arachnoid villi back to the
canal of the spinal cord and the brain venous drainage system of the brain.
ventricles. It is produced at a rate of
500 ml per day, by the choroid plexus Normal CSF Composition
Color: Clear
epithelial cells that are found in the
brain ventricles [lateral, third and fourth WBC
ventricles], and reabsorbed back into
circulation, by arachnoid granulation • < 5 cells/mm3 with less than 3PMN/
into dural venous sinuses. CSF is mm3 in adults
recycled about 2-3 times per 24 hr.
• <20 cells/mm3 with < 1 PMN cells/
CSF circulates from the choroid plexus mm3 in neonate
in the lateral ventricle into the third
RBCs: < 10 cells/mm3
ventricle through the foramen of
Monro, from the third ventricle into the Glucose: 45-80 mg/dl [CSF: serum
fourth ventricle through the cerebral ratio is 0.6] .
aqueduct of Sylvius. Then, it travels
Protein
from the fourth ventricle into
subarachnoid space through the • <45 mg/dl in adults

704
• <20 mg/dl in children Color
• Purulent :bacterial /TB manengitis.
Normal LP opening pressure
• Xanthochromia [yellow color] in case of SAH,
• In adults: 60-200 mmH2O [6-20 cm H2O].
Hyperbilirubinemia.
• In children who are < 8 years 10-100 mmH2O.
Image 17.2 Xanthochromia
• In neonate: 30-60 mmH2O

Image 17.1

CSF Analysis and Interpretation


Opening pressure
• Low Opening pressure: CSF leakage, or dehydration.

• High Opening pressure: overproduction, infection, bleeding, The sample on the left represents xanthochromia. Retrieved from http://
www.medfriendly.com/xanthochromia.html
tumor, false measurement [sitting position, Valsalva or crying].
705
Cytology • Blood [traumatic Taps /SAH). References and Further Reading,
High WBC count click here
• Multiple sclerosis.
• Viral meningitis [ predominant
• Guillain-Barre syndrome.
lymphocytes].
Miscellaneous test
• Bacterial meningitis [ predominant
PMNs ]. • India ink for Cryptococcus

• Fungal infection • VDRL/RPR for neurosyphilis

• Vasculitis • PCR for HSV or CMV

• Traumatic tap Table 17.2 CSF Analysis

High RBC count NORMAL BACTERIAL VIRAL SAH


Bloody or
Color Clear Purulent Clear/purulent
• Taumatic Tap xanthochromic

Opening
• SAH Pressure (cm 7-18 >20 cm H2O Normal/high High
H2O)
Biochemistry WBC/mm3 0-5 25-10000+ 10-500 Slightly high
Glucose level
WBC/RBC ratio
Differentials Lymphocytes PMNs Lymphocytes
same to serum
• Decreases in case of bacterial/TB
RBC/mm3 0-5 Normal Normal >500
meningitis or CNS tumor.
Glucose mg/
45-80 <20 Normal/low Normal
100ml
Protein
Protein mg/
15-50 50-10000 50-200 60-150
• Increases in: 100ml

The source is not provided by authors.


• Bacteria/TB meningitis.
706
Section 3

Urine Analysis

Indications • Postprandial sample – DM


by Jan Zajc Urinalysis should be performed to
• Catheterise – infants, bedridden
evaluate the following
patients
• Evaluation of renal & lower urinary
• Suprapubic needle aspiration
tract abnormalities

• Assessment of some metabolic/


Urine Examination
endocrine disorders Macroscopic Examination
Normal Volume – 600 – 2000mls
• Assessment of hydration status
• Polyuria – Diabetes Mellitus,
Urine Collection Diabetes Insipidus, Polycystic
• Early Morning sample – qualitative
Kidney, Chronic Renal Failure,
• Random sample – routine Diuretics.

• 24hr sample – quantitative estimation • Oliguria – Dehydration, diarrhea,


of proteins, Vanillyl mandelic acid, 5- Excessive sweating, Acute
hydroxyindole acetic acid, glomerulonephritis, Acute tubular
metanephrines, hormones in urine, necrosis, Complete urinary tract
microalbumin obstruction

• Midstream sample – UTI Color

707
• Clear or yellow pale – normal due to • Reflects the ability of the kidney to •High
pigments called urochrome maintain normal hydrogen ion
• All causes of oliguria,
concentration in plasma & ECF
• Milky – Purulent UTI, chyluria
• Glycosuria,
• Acidic urine
• Orange/Red – Urobilinogen, Red
• DM,
Beetroot ingestion, Hemoglobinuria, • Ketosis-diabetes,
Haematuria • Dehydration,
• starvation, fever,
• Brown/Black – alkaptonuria, melanin • nephrotic syndrome
• systemic acidosis,
Odor • Low
• UTI by E.coli,
• Normal – aromatic due to the volatile • All causes of polyuria except
• acidification therapy and high
fatty acids glycosuria DI,
protein diet
• Ammonical – bacterial action(E. coli) • pyelonephritis,
• Alkaline urine
Fruity- ketonuria, starvation
• glomerulonephritis
• Strict vegetarian,
• Musty – Phenylketonuria
Osmolality
• Systemic alkalosis,
• Fishy – UTI with Proteus
• Normal – able to produce 500-850
• UIT by pseudomonas or Proteus,
• Rancid – Tyrosinemia mOsm/kg water
• alkalinization therapy,
Urinary pH • Dehydrated with normal renal function –
• CRF 800 – 1400mOsm/kg water
• Normal pH 4.6 – 8
Specific Gravity • Diuresis with normal renal function – 40
– 80 mOsm/kg water
• Normal range- 1.003 to 1.035

708
Chemical Examination • A prognostic marker for kidney • acromegaly,
disease
Proteinuria • Cushing’s disease,
• in diabetes mellitus (earliest sign
• Glomerular proteinuria, e.g., nephrotic • hyperthyroidism,
of renal damage in DM)
syndrome
• drugs like corticosteroids
• in hypertension (sign of end-organ
• Tubular proteinuria: e.g., acute n damage) • Glycosuria without hyperglycemia
chronic pyelonephritis, heavy metal
poisoning, TB kidney • i n c re a s i n g m i c ro a l b u m i n u r i a • renal tubular dysfunction
during the first 48 hours after
• Overflow proteinuria: Bence Jones admission predicts an elevated Ketones
p ro t e i n s ( p l a s m a c e l l d y s c r a s i a ) , risk for acute respiratory failure,
hemoglobin( intravascular hemolysis), • Acetone, Acetoacetic acid, β-
multiple organ failure, and overall
myoglobin(skeletal muscle trauma) hydroxybutyric acid
mortality

• Hemodynamic proteinuria: seen in high • Non-diabetic causes- high fever,


• Bence Jones proteins – monoclonal
fever, hypertension, heavy exercise, starvation, severe vomiting/ diarrhea,
immunoglobulin light chains (kappa or
CCF etc. Glycogen storage disease
lambda) synthesized by neoplastic
plasma cells, seen in multiple myeloma, Bilirubin
• Post-renal proteinuria: caused by
inflammatory or neoplastic conditions in macroglobulinemias, primary
amyloidosis • Liver diseases
renal pelvis, ureter, bladder, prostate or
urethra. • Injury,hepatitis
Sugars – Benedict’s test and Reagent

• Microalbuminuria – Defined as urinary Strip test • Obstruction to biliary tract


excretion of 30 to 300 mg/24 hrs of
• Glycosuria with hyperglycemia • Urobilinogen
albumin in the urine
• diabetes, • hemolytic jaundice

709
• Early hepatitis • trauma, •Pigment casts – include those
produced endogenously, such as
• hepatocellular jaundice • tumors of the urinary tract
hemoglobin in hemolytic anemia,
Blood myoglobin in rhabdomyolysis, and
Microscopic Examination
The centrifuged sample of urine sediment bilirubin in liver disease.
• Prerenal
is examined on a glass slide under high
Cellular casts  – Red cell casts, White
• bleeding diathesis, magnification after the supernatant is
cell casts, and Epithelial cell cast
discarded
• hemoglobinopathies, • Red Cell casts – The presence of red
Acellular casts – Hyaline casts, Granular,
blood cells within the cast is always
• malignant hypertension Waxy, Fatty, Pigment casts and Crystal
pathologic, and is strongly indicative of
casts
• Renal glomerular damage, usually associated

• Hyaline casts – Seen in fever, strenuous with nephritic Syndrome


• trauma,
exercise, damage to the glomerular
• White Cell casts – Indicative of
• calculi, capillary
i n fl a m m a t i o n o r i n f e c t i o n ,
• acute & chronic • Granular casts – indicative of chronic pyelonephritis, acute allergic interstitial
glomerulonephritis, renal disease nephritis, nephrotic syndrome, or post-
streptococcal acute glomerulonephritis
• renal TB, • Waxy casts – severe longstanding
kidney disease (end-stage renal • Epithelial casts – seen in acute tubular
• renal tumors necrosis and toxic ingestion, such as
disease)
from mercury, diethylene glycol, or
• Postrenal
• Fatty casts – nephrotic syndrome, salicylate
• severe UTI, diabetic or lupus nephropathy, Acute
tubular necrosis Other structure – Bacteria, Microfilaria,
• calculi, Trichomonas Vaginalis, Schistosoma
haematobium, Spermatozoa, Yeast.
710
How to perform urinalysis Urine analysis video.
Patients should be instructed clearly – using clean-catch,
References and Further Reading, click here
midstream specimen method is as accurate as catheterization.
Urine should be checked immediately or refrigerated, but never
left at room temperatures. In the tables are normal values.

Table 17.3 Normal Urine Charactheristics


CHARACTERISTICS FINDINGS

Color Pale to dark yellow

Clarity Clear

pH 4.5-7.4

Glucose Negative

Protein Negative

Ketones Negative

Blood Negative

Bilirubin Negative

Urobilinogen 0.2 – 1.0

Specific gravity 1.005 – 1.025

Nitrite Negative

Leukocyte esterase Negative

711
Section 4

Whole blood cell count – CBC

Introduction • patients with acute illnesses of non-


by Kaja Cankar and Gregor Prosen The whole blood cell count is one of traumatic origin,
the most commonly ordered tests in
• which includes patients in
medicine. It is a routine hematological
emergency room presenting
screening study, performed to evaluate
w i t h f e v e r, c h e s t p a i n ,
the status and overall health of a
abdominal pain, gastrointestinal
patient.
bleeding, constipation, severe
Whole blood cell count includes total diarrhea, vomiting, severe
red blood cell count (RBC) with nosebleed, irritability and crying
indices, hemoglobin (Hb), hematocrit (infants), throat pain,
(HCT), white blood cell count (WBC) hypertensive urgencies and
with or without a differential and a emergencies, severe joint pain
platelet level. or low back pain, skin rash,
scrotal pain, seizures, syncope,
Indications vaginal bleeding, weakness and
Simplified guidelines suggest that patients with lightning injuries,
whole blood cell count is indicated for patients after near-drowning
and patients with terrestrial
• trauma patients with acute blood loss
venomous bites and stings.
and GCS of 8 or below,

712
Red blood cell (RBC) count • RDC Red cell distribution width result in abnormalities due to cell

and indices measures the range of cell size. lysis or clotting.


RBC count and indices can assist in
White blood cell count CBC in trauma patients may sometimes
determining whether our patient has
WBC count includes differential with be misleading. It presents normal initial
anemia, polycythemia, and
detail information about neutrophils, levels of hemoglobin which do not
erythrocytosis.
lymphocytes, monocytes, eosinophils, exclude a significant hemorrhage.
Hemoglobin (Hb) reflects the amount of and basophils. Patient’s hemoglobin value is not a real-
hemoglobin or oxygen-carrying potential time indicator of his or hers intravascular
available in the blood. Platelets blood volume, and it takes quite some
Platelet level is the number of platelets or time (minutes-hours) before hemoglobin
Hematocrit (Hct) indicates the proportion thrombocytes in a given volume of whole value reflects the degree of blood loss in
of whole blood that is occupied with red blood. Both increased and decreased trauma patients accurately. Following the
cell mass. levels can point to abnormal conditions of trend of serial hemoglobin
excess clotting or bleeding. measurements, every 15 to 30 minutes
Special measurements of red cells, called
can provide useful information regarding
indices, include:
Variations In The Test ongoing blood loss.
• MCV Mean corpuscular volume or Results
In a hospital setting, it is important to In trauma patients elevated white blood
average size of the red cell.
avoid taking blood from the same side as cell can often be found, but this
• MCH Mean corpuscular hemoglobin or an infusion in order to avoid occurrence is unlikely due to infection.
average hemoglobin content. hemodilution. It should be taken into WBC is elevated due to demargination of
consideration that some samples that WBCs during the stress response.
• MCHC Mean corpuscular hemoglobin
were difficult to obtain, e.g., lengthy
concentration or average hemoglobin Interpretation of Test
venipuncture using a narrow gauge
concentration.
needle, such as a small butterfly, may
Increased WBC
• Infection (localized and generalized)

713
• Inflammation (i.e., vasculitis) • Underlying hematopoietic disease •H e m o c o n c e n t r a t e d s t a t e s
(aplastic anemia, agranulocytosis) (dehydration, burns, diarrhea)
• Myeloproliferative disorder
• Immunosuppression, • High altitude,
• Tissue necrosis (burns)
• Medications (antibiotics, • Exercise,
• Myocardial infarction
chemotherapeutic agents)
• Polycythemia Vera
• Physiological stress (e.g., exercise,
Patient presenting with neutropenia is at
pain, surgery, prolonged crying in • Chronic obstructive lung disease
risk of infections from common and
infants)
opportunistic organisms.
Decreased Hb
• Medications (steroids) • Iron deficiency, vitamin deficiencies,
Decreased HCT
e.g., vitamin B12
• Vomiting • Blood loss
• Bleeding,
• Dysrhythmias • Hemolysis
• Kidney disease
• Acute myocardial infarction (AMI), • Long-standing anemia
• Inflammatory disorders (rheumatoid
• Pregnancy • Pregnancy
arthritis or infections)
The physician should look for a “left shift” If suspecting acute loss, the physician
• Hemolysis (accelerated loss of red
which indicates the presence of immature should look for  schistocytes  on the
blood cells through destruction)
forms in the peripheral circulation peripheral blood smear. Long-standing
(bands). Usually, this represents an anemia can be evaluated by the RBC • Inherited hemoglobin defects
infectious state. indices. Administration of fluids in (thalassemia or sickle cell anemia)
hypovolemic patients or trauma
Decreased WBC resuscitation will cause a decreased HCT. • Cirrhosis of the liver
• Infection (overwhelming sepsis or viral),
Increased HCT
714
• Bone marrow failure and cancers that • Infections (SBE, HIV, septicemia, Toxic granulations, Döhle Bodies,
affect the bone marrow mononucleosis) and cytoplasmic vacuolization are
remnants of phagocytosis found in
Causes of increased Hb  are similar to • Drug-induced destruction (penicillin,
neutrophils. These are indicative of more
HCT. heparin, sulfonamide, quinine)
serious bacterial infections.

Increased platelet count • Idiopathic, thrombocytopenic purpura,


Cutoff values of white blood cell (WBC)
• Myeloproliferative diseases thrombotic thrombocytopenic purpura,
counts greater than 15,000/mm3 suggest
disseminated intravascular coagulation
a higher likelihood of serious illness.
• Malignancy
• SLE
Acute hemorrhage will not be reflected in
• Infection
• Toxemia of pregnancy the Hgb or HCT early on.
• Recent surgery (splenectomy)
• Renal insufficiency Stress can cause the level of white blood
• Chronic inflammation (i.e., irritable cells to elevate, which can be
bowel syndrome) • Bone marrow failure due to carcinoma, misinterpreted as an infection.
leukemia, lymphoma, or fibrosis
• Tr a u m a ( m a s s i v e h e m o r r h a g e , In patients presenting with abdominal
thrombus) • Other: menses, poor nutritional states pain, an elevated WBC does not
such as iron, folate, and vitamin B12 necessarily imply a serious disease.
• Secondary to iron deficiency anemia or deficiencies.
hemolytic anemia Pediatric, Geriatric,
Hints And Pitfalls Pregnant Patient, And Other
In thrombocytosis, there is an excess of
Patients with serious infections may have
platelets (more than 1 million), but they Considerations
completely normal, or even low WBC
are usually large and nonfunctioning. • Geriatric patients will more than likely
counts. Overreliance on normal WBC
demonstrate normal to low WBC counts
counts in the setting of acute infections
Decreased platelet count in sepsis.
may lead to misdiagnosis and delays in
patient care.
715
• Elevated WBC can be found in prolonged crying in infants, pain,
vomiting dysrhythmias, and pregnant patients.

• Pregnancy can lower the hematocrit by 10%.

CBC Tips and Notation – Simple Explanation of CNC


Interpretation - video. 

References and Further Reading, click here

716
Chapter 18

Selected Imaging
Modalities
Section 1

eFAST

Case Presentation Image 18.1 Subcostal Cardiac


by Ashley Bean, Brian Hohertz and Gregory R. A 40-year-old man involved
Snead
in a car crash presents to
your emergency department
by ambulance. His vital signs
are pulse 118 beats/minute,
blood pressure 80/45 mmHg,
respiratory rate 30 breaths/
Normal Subcostal Cardiac 4 chamber view
minute, oxygen saturation
Image 18.2 Right Upper Quadrant
98%, and temperature 37C.
He is awake and oriented,
complaining of epigastric
abdominal pain and difficulty
breathing.

The following images are


Audio is available here
obtained.

718
Image 18.4 Left Upper Quadrant to the operating room where he abdominal and cardiac injuries at
your institution, transfer the patient to a
undergoes a midline
facility with this capability. A stable
laparotomy. A spleen injury is patient with free intraabdominal fluid
identified intraoperatively. should undergo further diagnostic testing
such as CT to ascertain the specific
Introduction injury.
The objective of the extended focused
While peritoneal lavage has been
assessment for sonography in trauma
traditionally utilized to evaluate for
(eFAST) is to detect free fluid in the
intraabdominal blood in the hypotensive
peritoneal, pleural and pericardial spaces,
trauma patient, the eFAST exam offers
and also to detect free air in thoracic
Image 18.3 Suprapubic view several advantages over peritoneal
cavities. In the setting of trauma, we
lavage. The eFAST exam is non-invasive,
assume this fluid is blood; however, it can
repeatable, rapid and sensitive for injuries
be urine or bowel contents as a result of
requiring surgical intervention. It also
organ rupture or it can be pre-existing
does not interfere with computed
ascites. An eFAST exam should take less
tomography (CT) interpretation. Rozycki
than 5 minutes to complete.
et al. reported that the FAST exam was
In the peritoneal cavity, 200 ml of fluid shown to be 100% sensitive and 100%
can be detected via ultrasound in the specific for hypotensive blunt abdominal
ideal patient. In reality, however, the trauma patients. Conversely, peritoneal
smallest detectable amount is usually lavage is invasive, can only be performed
After 1 Liter of normal saline, around 500 ml. Hypotensive trauma once, may require laboratory processing
patients with free abdominal fluid need and has a high false positive rate.
the patient remains
urgent operative intervention. (Protocol 1) Peritoneal lavage may also confound
hypotensive and is transferred If there is not a surgeon who can repair interpretation of abdominal CT imaging.

719
Rapid detection of pericardial tamponade detected by the eFAST exam in which hemithorax for the presence of
and cardiac injuries is of critical case, the eFAST should be repeated. hemothorax or pneumothorax. Finally, the
importance in the trauma patient. inferior vena cava is imaged to estimate
Fortunately, ultrasound is very sensitive Procedure the patient’s volume status. While
for the detection of pericardial fluid. As The premise behind the eFAST exam is performing the exam, we ask four yes/no
little as 10 to 20 ml of fluid can be readily that free fluid accumulates in the questions. These are;
identified in the pericardium. In one of the dependent areas of the abdomen. An
extended FAST exam involves several 1. Is there fluid in the peritoneal cavity?
original studies on the FAST exam,
pericardial fluid had a sensitivity of 100% views. These are;
2. Is there a pericardial effusion?
and a specificity over 99% for cardiac 1. Subcostal or Parasternal Long Axis
injury. Therefore, trauma patients with a 3. Is there fluid in the thorax?
Cardiac
pericardial effusion have a presumed
4. Is there a pneumothorax?
cardiac injury requiring evaluation in the 2. Right Upper Quadrant
operating room. (Protocol 2)
3. Left Upper Quadrant Emergency Indications
Indications for the eFAST include both
Hypotensive trauma patients who do not
4. Suprapubic blunt and penetrating traumatic injuries
have free fluid in their abdominal,
as well presentations of unexplained
pericardial, or plural spaces should be 5. T h o r a x f o r H e m o t h o r a x a n d
hypotension as part of an ultrasound
investigated for further injury. For Pneumothorax
shock protocol (i.e., RUSH exam) to
instance, the patient may have spinal
6. IVC for volume status rapidly diagnose the cause of low blood
shock, a long bone fracture causing
pressure.
blood loss, or lost a significant amount of Perform either a subcostal view or
blood at the scene of the trauma. Other parasternal long axis view of the heart to Contraindications
non-traumatic causes should also be l o o k f o r p e r i c a r d i a l e ffu s i o n o r Contraindications to the eFAST are
considered such as myocardial infarction. tamponade. Views of the abdomen primarily situations in which performing
Another possibility is that there is not yet include the right and left upper quadrants the study would delay or interfere with
a large enough amount of blood to be as well as a suprapubic view. Image each
720
critical life-saving interventions including Image 18.6 Curvilinear Transducer Image 18.7 Linear Transducer
emergent surgical intervention.

Equipment and Patient


Preparation
Ultrasound equipment should be readily
available in the emergency department
setting. Machines should have low-
frequency probes (2-5MHz) that may be
either curvilinear or phased array in
design (Image 5, 6). A high-frequency
linear probe (5-10MHz) (Image 7) may Prior to patient arrival, the ultrasound
also be used to aid diagnosis of machine should be positioned to the left
Image 18.5 Phased Array Transducer
pneumothorax in challenging cases and of the trauma bed and a patient identifier
are preferred by some practitioners. placed for the exam along with probe and
Higher frequency curvilinear or phased study type selection (Image 8).
array probes may be advantageous in Ultrasound gel should be stocked on the
pediatric cases. All machines should have machine at all times. Following the
t h e c a p a b i l i t y o f i m a g e c a p t u re , primary survey, sufficient ultrasound gel
preferably in digital format. should be applied to the epigastrium to
facilitate all views. Please do not forget,
ultrasound gel is cold. Therefore, awake
patients should be informed. The exam
should then be performed to include the
recording of appropriate images and
video clips.

721
Image 18.8 Ultrasound and Patient side. When the transducer marker is point Image 18.9 Transducer marker
Bed Position towards the patient’s head (longitudinal
orientation), the patient’s head will be
toward the left side of the screen, and
their feet will be toward the right side of
the screen. When the probe marker is
pointed to the patient’s right side, the
patient’s right will be toward the left side
of the screen, and the patient’s left will be
toward the right side of the screen.

The orientation of the parasternal long


axis view aligns with the axis of the heart
rather than the external body. The probe
marker is pointed toward the patient’s
right shoulder. (Image 18.9)

Procedure Steps
Perform the eFAST exam immediately Image 18.10 Longitudinal Orientation
after the primary ATLS survey. Some
Transducer Orientation
Each transducer has a marker, which is authorities recommend applying e-FAST

oriented in the same direction as the during the circulation phase of the

probe marker indicator on the screen. For p r i m a r y s u r v e y, e s p e c i a l l y i n

all of the views of the eFAST exam, hemodynamically unstable patients. It

except the parasternal long axis cardiac should be performed before the patient is

view, the probe marker should be pointed rolled to minimize shifts independent fluid

either towards the patient’s head or right collections.

722
Image 18.11 Transverse Orientation Image 18.12 Subcostal Transducer Video: Normal Subcostal Cardiac
(pelvis) Position View. Again, the right ventricle is the
closest cardiac chamber to the chest
wall. The left ventricle and both atria are
also visible. The bright white line is the
pericardium. No anechoic fluid is
visualized between the heart and the
pericardium. There is a normal heart rate
and good contractility.

Video: Large Pericardial Effusion. In this


To obtain this view, use the liver as the acoustic subcostal view, a pericardial effusion
window. forms a black, anechoic rim around the
1. Imaging of the Heart for
Pericardial Effusion and Image 18.13 Normal Subcostal View heart.
Tamponade
Video: Pericardial Tamponade. The right
The first image obtained should be the
heart. To obtain a view of the pericardial ventricle, the closest chamber to the

space, use either a subcostal or a transducer, is collapsed, indicating that

parasternal long axis view. A low- pressure from the pericardial fluid is

frequency phased array or curvilinear inhibiting ventricular filling.

probe should be used.


Video: Hypodynamic Heart. This patient
has a hypodynamic heart with a low
ejection fraction. To estimate ejection
The right ventricle is the closest cardiac fraction, concentrate on the left ventricle.
chamber to the chest wall. The left ventricle and
both atria are also visible. The bright white line is
Decreased cardiac contraction may
the pericardium. indicate that cardiogenic shock rather
than hypovolemic shock is the cause of
723
the patient’s hypotension. There is also a Video: Normal Parasternal Long Axis Video: Hypodynamic Heart. This
small pericardial effusion. View patient has congestive heart failure and a
low ejection fraction. Look at the left
Video: Hyperdynamic Heart. The left
Image 18.15 Normal Parasternal Long ventricle to get a gross estimation of the
ventricle has a high ejection fraction Axis heart’s ejection fraction. Obtain an overall
typical of someone with significant blood
gestalt to label the ejection fraction as
loss. The heart is attempting to circulate
“normal,” “high,” “decreased,” or
the remaining intravascular volume.
“severely decreased.”

Image 18.14 Parasternal Long Axis Video: Hyperdynamic Heart. This patient
Transducer Position has a hyperdynamic heart with an
ejection fraction close to 100%.

Normal Parasternal Long Axis Cardiac View. If an


adequate subcostal view of the heart cannot be
obtained, attempt the parasternal long axis view.
A subcostal view might be difficult in some
trauma patients if they have an abdominal injury,
epigastric tenderness or abdominal distention. In
this view, the right ventricle is still the closest
cardiac chamber to the anterior chest wall. The
left ventricle left atrium, and aortic outflow tract
are also visible.

Video: Pericardial Effusion. There is a


large anechoic pericardial effusion

Video: Large Pericardial Effusion

724
2. Imaging of the Abdomen for Image 18.17 Normal Right Upper n o fluid collection between the kidney
Quadrant View and the liver. As a patient breathes, the
Free Intraperitoneal Fluid
diaphragm lowers the position of the liver
Image 18.16 Right Upper Quadrant and kidney into a more inferior position.

Image 18.18 Abnormal Right Upper


Quadrant

Normal Right Upper Quadrant View. First,


identify the kidney and the liver. The kidney is
often the easiest structure to identify. It has a
bright white center surrounded by the less
Right Upper Quadrant. Next, the abdomen is echogenic cortex. The hepatorenal space, the
imaged for free fluid. The right upper quadrant is interface between the kidney and liver, is a
the abdominal view that is the most likely to be potential space that may contain free fluid. In
positive. The transducer is placed in the mid- this image, the patient’s head is toward the left
axillary line with the probe marker pointed side of the screen, and the feet are toward the Abnormal Right Upper Quadrant. Note the small
toward the patient’s head. A low-frequency, right side of the screen. The diaphragm is the anechoic fluid collections between the liver and
curvilinear or a phased array probe should be bright white line just superior to the liver. It is the kidney.
used to obtain this view. important to visualize the tip of the liver as well
as the inferior pole of the kidney to have an Video: Abnormal Right Upper Quadrant.
adequate assessment of the hepatorenal space.
In this abnormal view of the right upper
quadrant, there is a dark, anechoic stripe
Video: Normal Right Upper Quadrant. between the liver and the kidney. The liver
Again, the probe marker is pointed is floating in a large amount of free fluid.
t o w a r d t h e p a t i e n t ’s h e a d . T h i s
Video: Free Fluid. Free fluid typically has
orientation corresponds to the probe
pointy edges. In contrast, fluid contained
marker indicator on the screen. There is

725
within a lumen has rounded edges. The Image 18.19 Left Upper Quadrant Image 18.20 Normal Left Upper
image shows free “pointy” fluid between Quadrant
the liver and the kidney.

Video: Luminal Fluid. The anechoic


structure in this image is the gallbladder.
Bile is an anechoic liquid, but since it is
contained within the lumen the
gallbladder, the edges appear rounded.

There are several other free fluid mimics.


These are; Left Upper Quadrant. To obtain the left upper
quadrant view, position the probe at the left Normal Left Upper Quadrant. This normal view of
posterior axillary line near ribs nine and ten. the left upper quadrant shows the spleen and
• Gallbladder Again, the probe marker is pointed toward the the kidney. As with the right upper quadrant
patient’s head. Since rib shadows may obscure view, the patient’s head is to the left of the
• Perinephric fat your view, it is sometimes helpful to angle the screen, and the patient’s feet are toward the right
probe obliquely in line with the intercostal space. of the screen. The diaphragm is a bright white,
Many novice sonographers do not position the hyperechoic curving line superior to the spleen.
• Stomach or duodenum probe posteriorly enough. The sonographer’s On the left side, examine the space between the
hand should be parallel to and be resting on the kidney and the spleen for free fluid. However, it is
• Inferior vena cava bed. Since the spleen is smaller than the liver, more likely that fluid will accumulate around the
the interface between the spleen and the kidney dome of the spleen and, therefore, you must
will be higher on the left side of the body. In image the dome of the spleen.
Perinephric fat can be mistaken for awake, cooperative patients, asking the patient
echogenic clot. However, this fat is to take a deep breath may lower the spleen into
the field of view.
usually symmetric, so compare with the
Video: Normal Left Upper Quadrant. In
opposite side. Fluid within the lumen of
this video of a normal left upper quadrant,
the stomach or duodenum should also
there is no collection of free fluid either
have rounded edges. Fluid within the IVC
between the spleen and the kidney or
should not only be rounded, but should
between the spleen and the diaphragm.
show vascular flow with color Doppler.
There appears to be tissue with the same

726
echogenicity as the spleen superior to Image 18.21 Pelvic Tranvers View Video: Normal Longitudinal
the diaphragm (the left of the screen). Suprapubic View
However, this is a mirror image artifact
and, later in this chapter, we will discuss Video: Abnormal Pelvic View. This video

how the absence of this artifact can demonstrates free fluid adjacent to the

indicate fluid within the chest cavity. bladder. The uterus is visualized floating
within the fluid.
Video: Abnormal Left Upper Quadrant. In
this abnormal left upper quadrant view, Video: Abnormal Pelvis View. Often the

there is fluid superior to the dome of the collection of free fluid is subtle as

spleen. Imaging only the splenorenal demonstrated by this retrovesicular


Pelvic View. The pelvic view is obtained by
interface would have missed this very placing the transducer in a suprapubic position collection.
in either a transverse or longitudinal orientation.
abnormal finding. In the trauma patient Since the bladder is the acoustic window, it is
helpful to image the patient before the
3. Imaging of the Thorax for
with free intraperitoneal fluid, the spleen
placement of a Foley catheter. Hemothorax and Pneumothorax
is the most commonly injured organ. To image the patient for the presence of
However, the most likely location for fluid Here the transducer is in a transverse a hemothorax, start with either the right
to accumulate is in the right upper orientation with the probe marker or left upper quadrant view while angling
quadrant. While this may seem pointed toward the patient’s right side the probe cephalad to image each
counterintuitive, in a supine patient, the (image 18.21). Angle the transducer so hemithorax. The probe may need to be
right upper quadrant is the most that its beam is pointed inferiorly in order moved one rib space toward the
dependent area of the upper abdomen. to visualize the pelvic organs. patient’s head. Focus on the diaphragm
So, in a patient with a positive intra- and look for the mirror image artifact of
abdominal fast view, the location of the Video: Normal Pelvic View. This normal,
the liver or the spleen on the cranial side
injury cannot be ascertained by the transverse suprapubic view fans through
of the diaphragm. If the mirror image is
location of the fluid. the bladder. Look for free fluid lateral or
absent, there is fluid within the pleural
inferior to the bladder.
space.

727
Image 18.22 Hemithorax Tranducer Image 18.24 Hemothorax or Pleural Video: Small Pleural Effusion or
Position Effusion Hemothorax. In this video of the chest,
there is an anechoic, pleural effusion
rather than the mirror image. The spine is
visualized in the chest cavity. The tip of
the lung floats into the picture as the
patient breaths.

Video: Free fluid within both the right


thoracic and abdominal cavities. Fluid is
visible both superior to and inferior to the
Transducer Position. Make sure to angle the Hemothorax or Pleural Effusion. In this image, diaphragm.
beam of the transducer into the chest cavity. there is a normal hepatorenal space. The
diaphragm is visualized at its insertion. Rather
Image 18.23 Normal Hemithorax than a mirror image, there is an anechoic fluid Image 18.25 Transducer Position to
collection, and the thoracic vertebrae are visible.
In addition, there is a “spine sign.” Usually, the Evaluate for Pneumothorax
spine cannot be visualized within the thoracic
cavity because air scatters the ultrasound beam.
If you are able to visualize the spine, then there is
a medium present (free fluid) which can transmit
the sound wave.

Video: Normal hemithorax. This video


demonstrates a normal right upper
quadrant and hemithorax with a mirror
Normal Hemithorax. This image shows the image artifact.
kidney, liver, and diaphragm. However, as you The extended FAST exam also images each
transition to the chest cavity at the insertion of hemithorax for pneumothorax.
the diaphragm, air scatters the ultrasound beam,
and you lose visualization of the spine. In
addition, you can see a mirror image artifact.
728
Lung sliding rules out pneumothorax in Video: Normal Lung. In this video of Video: Pneumothorax. Recognize the
the segment of lung the transducer is normal lung, identify the superior rib (left inferior and superior ribs with their
imaging. Since air rises, the transducer side of the screen) and the inferior rib corresponding rib shadows and the bright
should be placed at the most superior (right side of the screen) with their light line is the parietal pleura. Note this
region of the chest. In the supine, trauma corresponding rib shadows. The bright line is not sliding with respirations.
patient, this position is usually the third white line between the two is the plural
intercostal space. If the patient is sitting, Video: Pneumothorax. This video shows
line and can be seen sliding or
the apices of the lungs should be imaged. another example of a pneumothorax.
shimmering.
A high-frequency probe should be placed Absent lung sliding can be associated
in a longitudinal position with the with pneumothorax; however, lack of lung
indicator pointed toward the patient’s Image 18.27 Pneumothorax
sliding does not “rule in” a
head. pneumothorax. Pneumothorax would be
the most likely diagnosis; however, there
Image 18.26 Normal Lung
are several conditions you should
consider. Lack of sliding may be
observed in patient who are not
breathing, who have a mainstem
intubation, or in cases where the pleura is
adherent to the chest wall.

The inferior rib and rib shadow are still visible, 4. Vena Cava Imaging for
but only the visceral pleura is visualized. The
parietal pleura covering the surface of the lung Volume Assessment
has dropped away from the chest wall. The two The inferior vena cava (IVC) normally has
pleural layers no longer slide over each other.
The patient’s head is to the right of the screen, respiratory variation. In a patient with
and his feet are to the left. Look for a rib and a
rib shadow as landmarks to help find the pleural normal volume status, the IVC will
line. The bright, white light line is the opposition collapse 30-70 percent as the patient
of both the visceral and the parietal pleura and
should shimmer, moving back and forth (a sliding inhales. The IVC caliber of hypovolemic
motion) with respirations.
729
patients will be smaller and collapse Video: Hypovolemia. The walls of the •Always remember that free fluid may
greater than 70%. Conversely, patients vena cava completely collapse with not be blood – consider ascites,
with fluid overload will have an enlarged respiration in this hypovolemic patient. bladder rupture, and bowel rupture as
IVC with minimal collapse. causes of free intraperitoneal fluid.
IVC collapse estimates the patient’s
volume status. It does not predict the • Since the bladder is your acoustic
Image 18.28 Transducer Position for
Volume Assessment patient’s response to hydration. window, the pelvic view should be
imaged prior to insertion of catheter.
Video: Volume Overload. There is a large
vena cava with minimal change with • A normal echo does not definitively rule
respiration. out major pericardial injury.

• An epicardial fat pad may easily be


Hints and Pitfalls
• If initial eFAST exam is negative, but misinterpreted as “clot.”
you have continued concern, repeat the
• Hemothorax may be confused with
exam.
pericardial effusion.

• Repeat the eFAST exam if there is a


Place the transducer on the abdomen with the
probe marker pointed towards the patient’s change in the clinical status.
Post Procedure Care and
head. Visualize the vena cava about 3 cm Recommendations
proximal to the cavoatrial junction.
• Perinephric fat may be mistaken for Post-procedure care consists of clear
clot; however, it is usually symmetrical. communication of the results to the
Examine the opposite side and trauma team (positive, negative, or
Video: Euvolemia. This video
compare. indeterminate for abdomen, thorax, and
demonstrates a normal inferior vena cava
pericardium). Limited views should be
in a patient who is euvolemic. There is • Placing the patient in reverse discussed and scanning repeated by the
respiratory variation in the vena cava, but Trendelenburg may help visualize free
m o s t e x p e r i e n c e d s o n o g r a p h e r.
the collapse is not greater than 30%. fluid.
Adjunctive maneuvers to improve

730
visualization such as repositioning the patient, or filling the Pregnant patients present s e v e r a l c h a l l e n g e s i n c l i n i c a l
bladder via foley catheter to obtain a better view of the pelvis assessment and use of the eFAST exam. Clinical instability may
should be considered. Clean ultrasound gel off the patient to help require placing the patient in the lateral position to maximize
maintain body temperature. Clean and decontaminate the blood flow to the uterus and require repositioning to complete the
ultrasound machine based on your institutionally approved exam. Uterine enlargement can limit the view of the bladder but
process by removing surface gel and using an appropriate also result in displacement of bowel loops making pelvic views
surface wipe or process. Complete any additional documentation variable and occasionally dependent on fetal positioning. Late
of the images along with a note describing the procedure and gestation is accompanied by other changes in addition to uterine
findings for inclusion into the medical record. enlargement including diaphragmatic elevation that may require
repositioning the probe to achieve adequate views. Test
Cautionary Note performance has been reported to mirror those in non-pregnant
Complications of the eFAST are typically a result of incorrect patients in spite of these challenges. Another important
performance or interpretation of results leading to false positive or requirement is rapid fetal assessment in trauma presentations.
false negative results. Difficult or limited exams should be Rapidly determining the fetal heart rate should be determined on
discussed or repeated by the most experienced sonographer on arrival and will likely precede initiation of continuous fetal
the resuscitation team. Team leadership should also interrupt or monitoring by the obstetric team. Fetal reassessment should be
delay the eFAST for critical interventions in the care of the patient. regularly performed until continuous monitoring is available.
And, please keep in your mind, e-FAST should not delay the Remember that ultrasound cannot exclude placental abruption –
definitive treatment of trauma patient. even in seemingly low force scenarios. Obstetric consultation and
prolonged fetal monitoring is advised in all trauma cases involving
Pediatric, Geriatric and Pregnant Patient
a fetus of potentially viable gestational age.
Considerations
In pediatric patient, the eFAST is highly specific but has
insufficient sensitivity to exclude intra-abdominal injury. Though
no change in test performance characteristics have been reported References and Further Reading, click here

related to probe choice, consider using a higher frequency probe


in smaller patients.

731
Section 2

PoCUS – RUSH Protocol

Why use POCUS in undifferentiated hypotension?


by Rasha Buhumaid Hypotension is a high-risk sign which is associated with increased morbidity and
mortality rate. The differential diagnosis for hypotension is broad and the treatment
depends on the underlying etiology. In most cases of hypotension, patients
present with limited history and physical examination may be inaccurate making
the management of the condition a great challenge for emergency physicians.

The use of POCUS in undifferentiated hypotension has been shown to help


correctly and rapidly identify the etiology and therefore initiate the appropriate
management. Since 2001, there are many protocols published describing a
systematic approach to the use of POCUS in undifferentiated hypotension.  Table
18.1  summarizes the components of these protocols. In this chapter, we will
review the Rapid Ultrasound in Shock (RUSH) protocol.

Equipment: Ultrasound machine with a linear probe (10-5 MHz) and phased array
(5-1 MHz) or curved array (5-2 MHz) probe.

The RUSH protocol


The  RUSH protocol  (Rapid Ultrasound in Shock and Hypotension) uses the
analogy of plumping system to categorize the causes of shock into three
categories: The pump, the tank, and the pipes. The categories are assessed using
Audio is available here
specific and straightforward questions described below.

732
Table 18.1 Protocols for undifferentiated shock used to assess for pericardial
effusion.

Interpretation: Pericardial effusion will


a p p e a r a s a n e c h o i c ( b l a c k ) flu i d
surrounding the heart (Image 18.29). The
ultrasound finding of cardiac tamponade
includes circumferential pericardial
effusion with hyperdynamic heart
swinging in the pericardial sac (Video).
Right atrial wall diastolic collapse and
right ventricular early diastolic collapse is
also known as scalloping (Video).

Image 18.29 Pericardial effusion

Step 1 – Pump: Cardiac P e r i c a r d i a l e ffu s i o n w i l l l e a d t o

Evaluation hypotension when the fluid surrounding


The basic echocardiography views the pericardium causes obstructive
summarized in this tutorial (Video). physiology preventing the venous return
to the heart. This is known as cardiac
This step is used to evaluate the pump tamponade. Pericardial effusion and
(the heart) for the following: tamponade can easily be identified using
POCUS.
A. Determine the presence of
pericardial effusion Technique: any of the above described Pericardial effusion marked by * in parasternal
basic echocardiography views can be long view (A), parasternal short view (B),
subxiphoid view (C) and apical 4 chamber view
(D)
733
Video: Circumferential pericardial effusion Image 18.30 Pericardial fat pad B. Assess Left Ventricular (LV)
with hyperdynamic left ventricle and contractility
swinging of the heart in the pericardial In the setting of hypotension, severely
sac in a parasternal long view (A) and decreased LV contractility can help
parasternal short view (B) identify possible cardiogenic shock while
hyperdynamic LV could result from severe
Video: Subxiphoid view demonstrating
hypovolemia, acute blood loss or early
pericardial effusion with diastolic collapse septic shock.
of the Right Atrial (RA) wall
Te c h n i q u e : A n y o f t h e b a s i c
Pitfalls:  Two important conditions can be echocardiography views can be used to
confused with pericardial effusion. Fat pad marked by * above the Right Ventricle
(RV) wall in a parasternal short view assess LV contractility.

Pericardial fat pad (Image 18.30) which is Interpretation: The contractility of the LV
not usually anechoic (black) but rather Image 18.31 pericardial effusion can be evaluated by visual assessment of
has some echogenicity (gray) and it is the difference in the LV volume between
usually confined to the anterior wall the end of systole and diastole. In normal
above the right ventricle. LV contractility, there is a significant

Pleural effusion can be confused with change in the volume of the LV between

pericardial effusion in a parasternal long systole and diastole (Video). However, in


view. This can be differentiated using the a poorly contracting LV, there is a small
descending aorta as a landmark in change in the size of the LV volume
parasternal long view. A pericardial during the cardiac cycle (Video). Using
effusion will track above the descending this method, the LV contractility can be
Parasternal long view demonstrating pericardial broadly categorized into 4 categories;
aorta while pleural effusion will track effusion tracking above the Descending Aorta
below the descending aorta (Image (DA) and pleural effusion tracking below the DA normal, mild to moderately decreased,
18.31). severely decreased and hyperdynamic LV.

734
Hyperdynamic LV is when the volume In the right clinical setting, hypotension Pitfalls:
changes between systole and diastole with signs of RV strain could be a sign of
• The apical 4 chamber view can be a
more than 70% or the walls of the LV massive pulmonary embolism causing
challenging view to obtain due to
touch during systole (Video). Studies obstructive shock.
technical difficulty the view might be
have shown that qualitative visual
Technique: using parasternal short view foreshortened (the apex appears
assessment correlates well with the
and apical 4 chamber view can identify rounded rather than bullet shape), this
quantitative techniques used by
signs of RV strain. will lead to overestimation of the RV
cardiologists.
size.
Interpretation: the signs of RV strain
Video: Normal Left Ventricular (LV)
include 1) RV enlargement: best assessed • An important pitfall to consider when
contractility in parasternal lone view (A)
in apical 4 chamber view. Normally, the evaluating for signs of RV strain is that
and parasternal short view (B)
RV is smaller than the LV with a normal RV dilation is not specific for massive
V i d e o : S e v e r e l y d e c r e a s e d LV RV:LV ratio of 0.6:1. If RV/LV ration > 0.9 pulmonary embolism. Any condition
contractility in parasternal long view (A) it suggests RV enlargement (Video). 2) that increases the pressure of the right
and parasternal short view (B) The D sign: In a parasternal short view, heart will lead to dilated RV including
bowing of the interventricular septum chronic COPD and pulmonary
Video: Hyperdynamic LV contractility in towards the LV indicate increase pressure hypertension. However, the RV wall is
parasternal long view in the RV (Video). likely to be thick in chronic conditions.

Pitfalls: In some cases of cardiogenic • It is important to note that normal RV on


Video: Apical 4 chamber views with
shock, LV contractility will be preserved, echocardiography does not rule out
normal Right ventricle (RV) size (A) and
for example, acute severe valvular pulmonary embolism.
enlarged RV (B)
regurgitation and acute right ventricular
infarction. Video: Parasternal short views with Step 2 – Tank: Volume Status
normal interventricular septum (A) and D Evaluation
 C. Assess for Right Ventricle sign: bowing of the interventricular
(RV) strain septum towards the LV (B)

735
A. Tank fullness: Inferior Vena percentage of IVC collapsibility with Image 18.32 M mode evaluation of the
Cava (IVC) evaluation respiratory variation also known as the IVC
In spontaneously breathing patient, Caval Index (CI). Using M mode, the IVC
during inspiration, the negative intra- size measured 2 cm from the right atrial
thoracic pressure will increase venous junction during inspiration and expiration.
return to the heart leading to IVC collapse The CI (%) = (IVC expiratory diameter –
which is reversed during expiration. This IVC inspiratory diameter)/ IVC expiratory
physiology is reversed in ventilated diameter × 100 (Image 18.32).
patients. During insufflation, the intra-
Video:  Tutorial on ultrasound of the
thoracic pressure increases, decreasing
Inferior Vena Cava
the venous return and the IVC expands,
this is reversed during expiration. Studies M mode evaluation of the IVC: (A) shows a small
Video:  A: small and collapsible IVC with and collapsible IVC with respiration and Caval
have shown that the degree of IVC size respiration. B: large and non-collapsible Index= 60%. (B) shows a large IVC with minimal
change during the respiratory cycle can respiratory variation and Caval Index = 3%
IVC with respiration
be used to predict volume
responsiveness. Interpretation: In spontaneously breathing
right atrium while the abdominal aorta
patients, collapsible IVC or caval index
Technique: using a low frequency probe passes behind the heart.
more than 40% is associated with volume
in a longitudinal plane in the subxiphoid responsiveness. While dilated non- • Interpreting the findings of IVC
area the IVC is identified as a vessel collapsible IVC does not rule out volume assessment alone can be misleading as
draining into the right atrium (Video). The responsiveness. there are numerous causes of dilated
IVC can be assessed using 2 methods:
non-collapsible IVC including tension
qualitative and quantitative. The Pitfalls:
pneumothorax, massive pulmonary
qualitative method is by visual
• The IVC can be mistaken for the embolism and pericardial tamponade.
assessment of the size and collapsibility
abdominal aorta. This can be avoided
with respiratory cycle (Video). The  
by visualizing the IVC drain into the
quantitative method is by calculating the

736
B. Tank Leakiness: FAST Exam Image 18.33 FAST exam I n the setting of hypotension,
In atraumatic hypotension, intrabdominal demonstrating free fluid decreased LV contractility, dilated non-
free fluid could be due to ruptured collapsible IVC, signs of pulmonary
abdominal aortic aneurysm, ruptured edema and bilateral pleural effusion
ectopic pregnancy or ruptured suggests cardiogenic shock.
hemorrhagic ovarian cyst.
Technique: Using a low frequency probe,
Technique: the scanning technique for assess for pleural effusion by obtaining
Intraperitoneal free fluid is described in the same views used for the FAST exam.
details in the eFAST chapter. Start with the coronal views of the right
upper quadrant and left upper quadrant
Interpretation: Intraperitoneal free fluid is then move the probe cephalad (towards
FAST exam demonstrating free fluid marked by
identified as anechoic (black) fluid (*) in the hepatorenal space (A), the head) to visualize the pleural cavity
collection in any of the following areas. In subdiaphragmatic and spleenorenal space (B),
rectovesical space(C) and pouch of Douglas (D) better; space above the diaphragm
the right upper quadrant view, fluids will
(Video). To assess for pulmonary edema,
accumulate in the hepatorenal space also
use a low frequency probe to obtain a
known as Morrison’s pouch. In the left P i t f a l l s :
 longitudinal lung view of the anterior
u p p e r q u a d r a n t v i e w, flu i d s w i l l Free fluid in the peritoneal cavity can be chest wall, locate two ribs (hyperechoic
accumulate in the subdiaphragmatic urine or previous ascites. Ultrasound area with posterior shadowing) and
space that can extend into the cannot differentiate these fluids. However, identify the pleural line (hyperechoic line)
splenorenal space. In the pelvic view, in a patient with shock and hypotension, between the rib shadows (Video).
fluid will accumulate in rectouterine space this free fluid is considered as blood until
also known as the pouch of Douglas in proven otherwise. Video: Tutorial on lung ultrasound for
females and rectovesical space in males. pleural effusion
(Image 18.33) C. Tank Overload: Assessment
of pleural effusion and Video: Tutorial on lung ultrasound for
pulmonary edema pulmonary edema

737
I n t e r p re t a t i o n : P l e u r a l e ffu s i o n i s alveoli, lung ultrasound will produce B Video: Tutorial on lung ultrasound for
confirmed if a black/anechoic fluid is lines: hyperechoic-white vertical lines pneumothorax
identified above the diaphragm or by extending from the pleural line into the far
Interpretation: Assess the pleural line for
visualizing the thoracic vertebra body field. In case of pulmonary edema, B lines
sliding with respiration either by visual
(hyperechoic-white line with posterior will be bilateral and in all lung zones
evaluation or using M-Mode. The
shadowing) extending above the (Video).
presence of lung sliding, seen as
diaphragm also known as positive spine
Video: Lung ultrasound (A) A-lines in the seashore sign on M mode rules out
sign (Image 18.34).
normal aerated lung. (B) B lines extended pneumothorax with 100% negative
Image 18.34 from the pleural line to the far field predictive value. In the absence of lung
indicating fluid-filled alveoli sliding also seen as barcode sign on M
mode (Video) continue following the
Pitfalls: B-lines are not specific for
pleural line inferiorly and laterally to
pulmonary edema. Any pathology that
identify the boundary of pneumothorax
will fill the alveoli with fluids including
known as lung point. A lung point on
ARDS and bilateral pneumonia will
ultrasound will appear as a boundary
produce bilateral B-lines on lung
between the absence of pleural sliding
ultrasound.
and normal pleural sliding (Video). The

D. Tank Compromise: l u n g p o i n t i s v e r y s p e c i fic f o r


(A) Right upper quadrant view with normal Assessment for pneumothorax pneumothorax.
pleural space. (B) Right upper quadrant view
with evidence of pleural effusion (*) and positive Tension pneumothorax is another cause
spine sign Video: (A) Lung ultrasound with normal
of obstructive shock.
pleural sliding. (B) Lung ultrasound with
Technique: using a high frequency (linear) absence of pleural sliding. (C) M mode
In normally aerated alveoli, the pleural line probe, obtain a longitudinal view of the demonstrating seashore sign indicating
will produce reverberation artifact known lung between 3rd and 4th intercostal normal pleural sliding. (D) M mode
as A-lines. However, in case of fluid-filled space, mid-clavicular line (Video).

738
demonstrating barcode sign indicating the vertebral body. The aorta is traced 18.35) and the risk of rupture
absence of lung sliding caudad (toward the feet) until it bifurcates increases when it is greater than 5 cm.
into the common iliac arteries at the level
Video: Lung point which is the boundary Pitfalls: The majority of ruptured AAA are
of the umbilicus (Video).
between absence of lung sliding and retroperitoneal therefore when performing
normal lung sliding Video: Tutorial on ultrasound for the FAST exam, no intraperitoneal free
abdominal aortic aneurysm fluid will be identified.  Always make sure
Pitfalls: The absence of lung sliding could
that you are measuring the aorta from the
be from numerous causes other than Interpretation: The abdominal aorta is outer wall to outer wall.
pneumothorax including: pleurodesis, measured from outer wall to outer wall at
pleural bleb, poor respiratory effort and the proximal, middle and distal aorta. B. Assessment for clogged
mainstem intubation. Normal abdominal aorta measures less pipes: Deep Venous Thrombosis
than 3 cm. AAA is defined as abdominal (DVT)
Step 3 – Pipes: Vascular aorta measuring greater than 3 cm (Image When considering pulmonary embolism
system Evaluation as a cause of obstructive shock,
obtaining adequate echocardiography
A. Assessment for Abdominal Image 18.35 Aorta evaluation
views to assess for RV strain can
Aortic Aneurysm (AAA)
challenging. In this situation, evaluating
A ruptured aortic aneurysm is a cause of
the extremities for DVT can be used as a
non-traumatic hemorrhagic shock.
surrogate marker for possible pulmonary
Technique: Using a low frequency probe. embolism as literature shows that
The aorta is scanned in a transverse view majority of pulmonary embolism originate
starting in the subxiphoid area. The from DVT.
landmark used to identify the aorta is the
Technique: using a low-frequency probe
vertebral body (hyperechoic-white
t h e 2 z o n e g r a d e d c o m p re s s i o n
structure with posterior shadowing). The
(A) Normal abdominal aorta. (B) Abdominal aortic technique is used to identify DVT.
aorta is located anterior and to the left of
aneurysm measuring 4.8 cm with intramural
thrombus
739
1) Assess the common femoral vein zone: Video: (A) Normal compressible left t h e protocol may be altered based on the
place the probe in a transverse plane just Common Femoral Vein (CFV). (B) Non- clinician’s assessment of the clinical
below the inguinal ligament, identify the compressible left CFV suggesting a DVT condition.
common femoral vein and greater
Video: Summary of the RUSH protocol
saphenous vein. Trace the common Image 18.36 DVT
femoral vein distally until it divides into
the superficial and deep femoral vein.
References and Further Reading, click
2) Assess the popliteal zone: place the here
probe in a transverse plane in the
popliteal fossa, identify the popliteal vein
which is located on top of the popliteal
artery. Trace the popliteal vein until it
t r i f u rc a t e s d i s t a l l y. A p p l y g r a d e d
compression on all the veins identified to
Echogenic material in the right Common
ensure complete collapsibility of the veins Femoral Vein (CFV) indicating a DV
(Video).
Pitfalls: The 2 zone technique can only be
Video: Tutorial on ultrasound for DVT used in ambulatory patients as studies
have shown that it might miss isolated
Interpretation: Normally the veins are
deep femoral vein DVTs which are seen in
collapsible, failure to compress the vein
patients with prolonged immobilization.
(Video) or identification of echogenic
material in the vein lumen suggests DVT The RUSH protocol provides a systematic
(Image 18.36). stepwise approach to help rapidly identify
the etiology of undifferentiated shock
summarized in  video. The sequence of

740
Section 3

BLUE protocol

Case Presentation
by Toh Hong Chuen A 68-year-old man with a history of congestive cardiac failure
(CCF) and chronic obstructive pulmonary disease (COPD)
presented with breathlessness and a newly productive cough
for 3 days. He was non-compliant with neither medication nor
fluid restriction. At triage, he dyspneic and immediately
brought to the resuscitation bay. His vitals were BP
188/92mmHg, PR 119/min, RR 23/min, Temp 37.9C, SpO2
91% on 3L intranasal oxygen. Clinically, the JVP was elevated
to the earlobes. Heart sounds were S1S2, breath sounds were
diminished with prominent wheezing. There was mild pitting
edema in the lower limbs to the knee. The diagnostic dilemma
of acute exacerbation of CCF versus COPD needed to be
addressed urgently.

While cardiac monitors and peripheral IVs were being set up,
lung ultrasound was performed using the BLUE protocol.

741
Bilateral lung sliding were seen in Stage 1, Emergency Indication
Patients presenting with dyspnea or respiratory distress
negative DVT scan in Stage 2 and negative
posterior lateral alveolar pleural syndrome Contraindication
(PLAPS) in Stage 3. This clinched the diagnosis Absolute contraindication: NONE

of acute exacerbation of COPD, and he was Relative contraindication:


immediately put on nebulization with salbutamol • Lung ultrasound should not delay immediate interventions
and ipratropium. IV steroids and slow required for recognized life threats.

maintenance fluids were started, and since he • Nevertheless, lung ultrasound can often provide information
fulfilled the Anthonisen criteria for infective that leads to the diagnosis of these life threats.

exacerbation, broad-spectrum antibiotics were


Equipment and Patient Preparation
given. The CXR performed demonstrated
Patient Preparation
hyperinflated lungs, cardiomegaly, and no
• Position: Supine, or semi-recumbent
consolidation; while the blood tests were
• Consent: Verbal consent is adequate
unremarkable. The patient improved significantly
after the 3rd cycle of nebulization and did not • Others: Apply universal precaution

require non-invasive ventilation. He was admitted Equipment Preparation


for continued management and subsequently Probe: Curvilinear (preferred), or linear

discharged well after 2 days. Preset: Lung preset if available.

• If lung preset is not available, use the abdominal preset, but


switch off all B mode optimization settings (e.g., tissue
harmonics)
742
• Others: Gel C. Stage 3: Posterior Lateral Chest 1.Put two hands side by side with
Wall index fingers touching each other (i.e.,
Procedure Steps excluding the palm). The examiner
1. Explain the procedure to the patient • Two sites: right/left PLAPS
hand size should approximate that of
and get consent. point.
the patient’s hands.

2. Set to lung preset, and apply gel to the Blue Protocol Scanning 2. Place the upper hand just below the
probe Sites clavicle, with the fingertips in the mid-
There are many proposed lung ultrasound sternum.
3. Scan sequentially, and up to the 3rd
scanning sites. In the original paper on
stage if required.
BLUE Protocol, the chest wall is divided • Upper BLUE Point is in the middle

A. Stage 1: Anterior Chest Wall into 6 zones (Anterior, Lateral and of the upper hand (i.e., between
Posterior zones of the right and left chest the root of 3rd and 4th fingers)
• Four sites: Right and Left – walls), which is further subdivided into
Upper and Lower Blue Points upper and lower halves (i.e., 12 sites). Image 18.37 Upper and Lower BLUE
Points
• Proceed to Stage 2 only if A Currently, the scanning sites have been
p r o fil e ( i . e . , “ n o r m a l ” ) i s simplified to the 2 BLUE Points on the
identified anterior chest wall (stage 1) and 1 PLAPS

B. Stage 2: DVT scan point in the posterior-lateral chest wall


(stage 3), performed on each side of the
• Refer to the section on DVT thorax.
scanning
Locate the Upper and Lower BLUE
• Proceed to Stage 3 only if the (Image 18.37) and PLAPS Points (Image
scan is negative for DVT (i.e., 18.38) as follow:
normal)

743
• Lower BLUE Point is in the middle The bat sign is critical for correct o n B mode at the pleural line, termed
of the palm of the lower hand. identification of the pleural line. Always lung sliding (Video: Lung Sliding).
begin lung ultrasound by identifying the
• PLAPS Point is the horizontal This motion artifact produces the sea-
bat sign before proceeding to look for
continuation of the lower BLUE shore sign on M-Mode (Image 18.39:
artifacts and pathologies.
Point, as posterior as possible to Seashore Sign).
the posterior axillary line with the This sign is formed when scanning across
patient remaining supine. 2 ribs with the intervening intercostal Image 18.39 Seashore Sign
space.
Image 18.38 PLAPS Points
The wings are formed by the 2 ribs,
casting an acoustic shadow. The body is
t h e fir s t c o n t i n u o u s h o r i z o n t a l
hyperechoic line that starts below one rib
and extends all the way to the other.
(Video: Bat Sign) The body is the pleural
line, i.e., parietal pleural. Normally, the
pleural line is opposed to and hence
indistinguishable from the lung line
Lung Ultrasound Findings
(formed by the visceral pleura). Absent lung sliding is always abnormal
These are the building blocks of the
BLUE Protocol. As they are mostly and occurs when the two pleural are
Lung Sliding
artifacts, settings on B mode imaging When the lung expands and contracts • Separated, for example by air in the
which minimizes artifacts (so as to with respiration, the parietal (pleural line) case of pneumothorax
improve image resolution) should be and visceral pleural (lung line) move and
switched off. slide over each other, creating a • Opposed but stuck to each other
shimmering or sparkling motion artifact (pleurodesis)
Bat Sign

744
• Opposed but not moving (mainstem Image 18.41 Lung Point on M Mode repeated at regular intervals below
intubation) the pleural line, at a distance which is
equal to the distance between the probe-
The absence of lung sliding is readily
skin interface and pleural.
apparent in B mode (Video: Absent Lung
Sliding) and produces the stratosphere The presence of A-lines indicates good
sign on M-Mode. (Image 18.40: scanning technique, as the probe is
Stratosphere Sign) perpendicular to the pleural line – a
requisite for the generation of this artifact.
Image 18.40 Stratosphere Sign The converse is also true. This effect is
demonstrated in the clip (Video: A Lines),
where the A lines disappear when the
Lung Point is pathognomonic of
probe is tilted away from its initial
pneumothorax.
perpendicular position.
It reflects the size of the pneumothorax
B-lines
(moderate if seen anteriorly, large if seen
B-lines are artifacts with 7 characteristics,
posteriorly, and total collapse if absent)
of which the first three are always
and may guide the need for intervention.
present.
Lung Point Most pneumothoraces with lung point in
They are comet-tail artifacts arising
This refers to the appearance and the lateral chest wall requires chest tube
strictly from the pleural line and always
disappearance of lung sliding (Video: (90%), compared to those with anterior
move in concert with lung sliding (if lung
Lung Point on B Mode) with respiration at location (8%).
sliding is present). They are most often
a specific point on the pleural. It is
A-lines hyperechoic, well defined, long and laser-
equivalent to having alternating sea-shore
These are horizontal reverberation like, and erases the A lines along its path.
and stratosphere signs on M Mode
artifacts arising from the pleural line. (Video: B Lines)
(Image 18.41: Lung Point on M Mode).
Consequently, they appear and are
745
They occur when the subpleural visceral interlobular septa are
Image 18.42 Pleural Effusion
edematous. This can be found in several conditions, such as
acute cardiogenic pulmonary edema, ARDS, pulmonary
contusion, and pneumonia.

The following terms are commonly encountered in literature but


are not included in the BLUE Protocol.

Lung rockets: 3 or more B lines within a rib space. Septal rockets


contain between 3-5 B lines per rib space. Ground glass rockets
have 6 or more, which often coalesce to form a bright curtain-like
artifact “hanging” from the pleural line.

Interstitial syndrome: bilateral anterior lung rockets. Posterior lung


rockets are not considered as they may be due to gravitational Shred sign: indicating non-translobular consolidation (Image
pull. 18.43: Shred Sign, Video: Shred Sign)

The interface between consolidated and aerated lung is irregular,


Pleural effusion
Presence of anechoic collection between the pleural and lung line and appeared as if it has been “shredded.”
(Image 18.42: Pleural Effusion).
Lung Profiles
Consolidation The lung findings described above are used to characterize the
Tissue-like sign: indicating translobular consolidation (Video: lung profile.
Tissue-like Sign)
Anterior Chest Wall
When the entire lobe is consolidated, it has a tissue-like There are 6 profiles on the anterior chest wall (i.e., Upper and
appearance similar to the liver. Lower BLUE Points)

1. A profile = Bilateral A-lines with lung sliding.

746
Image 18.43 Shred Sign 6. C profile = Shred sign or tissue-like 5.Pneumonia: Variable: A + PLAPS
sign (regardless of size or number) profile, A/B profile, B’ profile & C profile

Lateral Chest Wall There are two important caveats in


There are 2 profiles on the posterior relating lung profiles to specific diseases:
lateral chest wall (i.e., PLAPS Point)
1. The normal non-pathological lung has
1. PLAPS profile (or PLAPS positive): the the same findings as patients with
presence of either pleural effusion or COPD or asthma, i.e., A profile with
consolidation negative DVT scan and negative
PLAPS.
2. Nude profile (or PLAPS negative):
absence of pleural effusion and 2. Patients with A’ profile and no lung
consolidation sliding requires additional imaging
modalities
Lung Profiles of Common
Respiratory Diseases • This could still be due to a
1. Cardiogenic pulmonary edema: B pneumothorax (i.e., massive one
profile with a complete collapse of the
2. A’ profile = A lines without lung sliding lung) or other rare causes (e.g.,
2. COPD or asthma: A profile with pleurodesis).
3. B profile = Bilateral B lines with lung
negative DVT scan, negative PLAPS
sliding
Hint and Pitfalls
3. Pulmonary embolism: A profile with
4. B’ profile = B lines without lung sliding • The BLUE protocol is not an acronym. It
positive DVT scan highlights the indication (and utility) for
5. A/B profile: Half A profile on one lung using the protocol, i.e., a patient who is
4. Pneumothorax: A’ profile with a lung
and half B profile at the other “blue” from respiratory distress. It is a
point
rapid and efficient way of diagnosing

747
the 5 major respiratory diseases, with a • It cannot be used for patients with the lung findings, in the form
reported accuracy of 90.5%. mixed or multiple respiratory of the FALLS protocol.
disorders.
• Perform lung ultrasound immediately Post Procedure Care and
after clinical examination, prior to CXR. • It does not identify rare respiratory Recommendations
It yields diagnostic information rapidly disorders (defined as occurring None
and can expedited treatment. with a frequency of <2% of ICU
patients in the single center that Complications
• Recognizing the B profile, for None
was studied)
example, takes only less than 10
seconds. • Massive pleural effusion is not Pediatric, Geriatric, and
included in the protocol, though Pregnant Patient
• Completing the entire protocol
diagnosis is not an issue) Considerations
(i.e., up to Stage 3) requires less
than 3 minutes. • It cannot be used for non- Geriatric
respiratory causes of The BLUE protocol is derived from a
• Stay focus and scan only the BLUE
breathlessness, e.g., study of 301 consecutive adult patients
points and PLAPS points.
hyperventilation from metabolic and is applicable to the geriatric
• Other sites can be scan when the acidosis or profound anemia. population.
time is available.
• It is not designed to provide Pregnant
• Always interpret ultrasound findings in information on the patient’s While there are no pregnant patients
the context of clinical findings; and hemodynamic status. which are reported in the original paper2,
integrate both in the clinical decision- the principles in the diagnostic algorithm
• This could also be performed
making process. are applicable in pregnancy.
using point of care
• Pitfalls of the BLUE Protocol: ultrasound, by integrating the
Pediatric
focused cardiac and IVC with

748
In the same way, the BLUE protocol can also be adapted for use
in the pediatric patients.

References and Further Reading, click here

749
Section 4

How to Read C-Spine X-Ray

Introduction
by Dejvid Ahmetović and Gregor Prosen C-spine x-ray interpretation is one of the fundamental skills of emergency
physicians. Although current guidelines lead us to use CT scan for a suspected c-
spine injury, c-spine x-rays are still valuable in some low resource settings and
patient groups who are susceptible to radiation. Therefore, this chapter will
summarize the basics of c-spine x-ray interpretation.

Interpretation of radiographs has its limitations, which more or less depending on


the individual’s knowledge of anatomy and clinical experience.

Because anatomical landmarks for measurements can sometimes be difficult to


find or identify. A more systematic approach to reading cervical radiographs can
significantly reduce the chances of missing an important injury.

Visualisation
Plain radiographs, when they show the lateral projection of the cervical spine and
include an open mouth view, are fairly sensitive in identifying c-spine fractures. The
risk of missing a significant fracture is, according to statistics, less than 1%.
Addition of the anteroposterior (AP) projection increases sensitivity to
approximately 100%. All of the three essential above mentioned projections can
be seen in Image 18.44.

750
Image 18.44 C-spine essential views Image 18.45 Inadequate c-spine lateral x-rays

Lateral view with normal slight lordosis (A), Odontoid or open mouth view of
the atlas and axis (B), Standard anteroposterior or AP view with open mouth,
it can also be taken with closed mouth (C).

Before analyzing cervical radiographs, some additional facts need


to be presented.

Most spinal injuries occur at the junctions of the spine:


Two examples of a cervical x-ray that is not good enough for the evaluation
craniocervical, cervicothoracic, thoracolumbar and lumbosacral. of the possible injury of the neck.

Only c-spine radiograph one should be satisfied with is the one head to achieve the swimmer’s position, which better visualizes
showing all of the 7 cervical vertebrae (C1-Th1). the lower vertebrae.

The C7-Th1 vertebrae may be obscured in muscular or obese There are 3 basic views of c-spine
patients (Image 18.45), or in patients with spinal cord lesions that
1. Cross-Table Lateral View
affect the muscles which normally depress shoulders. Such
lesions that leave the trapezius muscle unopposed occur in the 2. Odontoid – Open Mouth View
lower cervical region. Shoulders can be depressed by pulling the
arms down slowly and steadily, or if the patient is capable, asking 3. Anteroposterior View
them to depress one shoulder and lift the other hand above his

751
Cross-Table Lateral View First, visualize the spine from the base of Image 18.47 Alignment and lines
The lateral (cross-table) view is the most the skull to the C7-Th1 junction. Next,
helpful x-ray study in diagnosing c-spine check if the x-ray is a real lateral view, or
injuries. Inspection of the x-ray should be if it is slightly rotated. Facet joints are
thorough, methodical and complete. At best visualized when we have a proper
this point it is not easy to differentiate lateral projection. (see Image 18.46).
‘ABCs’, because of all the acronyms
To check for proper alignment, look for a
across the field of medicine, but the
normal smooth lordotic curve and
‘ABCs’ in this case stands for: A –
imagine two lines, each running along the
alignment and adequacy, B – bone
anterior and posterior margins of
abnormalities, C – cartilage space
vertebral bodies. Additionally, a third line
assessment and S for soft tissues.
(spino-laminar line), running along the

A – Alignment and adequacy base of spinous processes and up to the


posterior aspect of the foramen magnum,
must be visualized (Image 18.47).
Image 18.46 Alignment and adequacy Always assess (AV) anterior vertebral, (PV)
posterior vertebral and (SL) spinolaminar lines,
All three lines should form a smooth and they should run smooth, without any disruptions,
lordotic curve of the cervical spine. Any and should form a slight lordotic shape.

disruption in the flow of these lines


injury if the C2 spinous process base lies
suggests either a bony or a ligamentous
more than 2 mm from this line. Also
injury (Image 18.48).
correlate with the soft tissue findings (see
An exception to this rule is a pseudo- below, under “S”). Furthermore, on the
subluxation of C2 and C3 in the pediatric lateral view, inspect the predental space,
Example of a slightly rotated not ideal lateral population, which can cause confusion. which is the distance between the
projection of the cervical spine in (A) and an x-ray
of an ideal lateral projection in (B). In these cases inspect the spino-laminar anterior surface of the odontoid process
line from C1-C3 and be suspicious of and the posterior aspect of the anterior

752
Image 18.48 Lorem Ipsum dolor amet, Image 18.49 Predental space Image 18.50
consectetur

Watch for a non-disrupted bony outline.


Disruption, as in the above examples means
Predental space, the distance between the fracture of the bone structure. Also search for
anterior surface of the odontoid process and any hypo- or hyper-dense areas in the bone, as it
posterior aspect of the anterior ring of C1, in may be the only indication of the compression
adult, it should not exceed 3 mm, or 5 mm in fracture. In (A) slight widening of the soft tissue
children. is visible just in front of the fracture, under the
white arrow, which may indicate that this is an
acute injury.
as it may indicate a compression fracture.
Areas with decreased bone density which
C – Cartilage space assessment
may be found in patients with rheumatoid Inspection of a good quality lateral view
Disruption in the shape of the AV line, that
indicates injury, and in this case a fracture of the arthritis, osteoporosis or metastatic x-ray in a healthy person should show
body of C7.
osteolytic lesions, are more prone to uniform intervertebral spaces. (Image
ring of C1. It should not exceed 3 mm in breaking under stress. Acute 18.51).
adults or 5 mm in children. (Image 18.49). compression fractures of the above-
mentioned changes show as areas of An emergency physician may diagnose
B – Bone increased bone density (Image 18.50). subluxations and dislocations of the facet
Watch for a normal bony outline of the joints through the assessment of cartilage
vertebras and bone density. Subtle space between corpora of vertebrae,
changes in bone density should be noted, facet joints, and space between spinous

753
Image 18.51 Cartilage space S – Soft tissues Image 18.52 Soft tissues
The prevertebral soft tissues can be used
as an indicator of an acute swelling or
hemorrhage resulting from an injury, and
may sometimes be the only indicator of
an acute injury on an x-ray. The normal
width of the prevertebral tissue decreases
down from C1 to C4 and increases from
C4 downwards. Normal measurements
from C1 to C4 are less than 7 mm (less
than half of the vertebral body at this
level), and less than 22 mm below the C5
(less than the vertebral body at this level)
see Image 18.52. Air within soft tissue
could suggest rupture of the esophagus
or trachea.

Odontoid – Open Mouth Retro-pharyngeal soft tissue, narrows down from


Uniform intervertebral cartilage spaces, also
facet joints must be inspected, for any unusual View C1 to C4, and should not exceed more than
7mm (less than third of the vertebral body).
alignment or increased space. This is usually the second standard view Bellow the C4 soft tissue starts widening, but
obtained in the emergency department. should not exceed 22mm (for easier thinking,
processes. Increased interspinous should not exceed the width of the body of the
The main goal is to picture the odontoid vertebrae.
distance by more than 50% suggests a
process of the C2 and the C1. It can be
ligamentous injury and the protective and the lateral masses of the C1 should
done with the mouth either open or
muscle spasm may make the be equal. If not, the inequality may be due
closed. Two things are assessed when
interpretation difficult. to a slight rotation of the head. Secondly,
inspecting the odontoid x-ray: the
and considering the previous point, the
distance between the odontoid process
754
margins of C1 and C2 should remain Anteroposterior View Image 18.54 Anteroposterior View
aligned (Image 18.53). Images taken in this projection are usually
much less clear than the two mentioned
Image 18.53 Odontoid - Open Mouth above. The tips of the spinous processes
View should lie in a straight line in the mid-line
and distances between the spinous
processes should also be checked.
Anomalies, such as bifid spinous
processes, can make interpretation
difficult. The laryngeal and tracheal
shadows should align down the middle.
The alignment of the lateral masses of the
vertebra should also be checked (Image
18.54).

Other Views Blue line connects the spinous processes, they


Oblique and flexion/extension views are should lie mid-line and have an equal amount of
space between. Red-line should smoothly
The distance between the odontoid process and useful only to an experienced physician. connect the lateral masses of the vertebrae.
the lateral masses of the C1 should be equal, if Always check the edges of the picture, in most
not inequality may be due to the slight rotation Flexion and extension are often either
cases, apexes of the lungs are visible, check for
of the head. (If the patient has the upper central contraindicated because of the pneumothorax.
incisor teeth, we can check if the space between
those two teeth aligns with the middle of the suspected unstable trauma or impossible
odontoid process, this might give the slight idea to accomplish because of the spastic
about rotation in case process itself is not
broken and misaligned). Even with the slight musculature post-injury. (Image 18.55).
 
rotation of the head we can still check alignment
by looking at the lateral margins of the C1 and
Unsupervised or even forced flexion or
C2, which should remain aligned. extension in a patient with ligamentous
injury may also lead to neurologic injury.
 

755
Image 18.55 Image 18.56

Suspected fracture of the odontoid process, but with closed mouth teeth
might affect the view.

Straightened normal lordotic curvature of the c-spine, may be due to the


muscle spasm as a protective mechanism, what also makes flexion and
extension views hard to capture.
756
Image 18.57 Image 18.58 Lorem Ipsum dolor amet,
consectetur

Lateral view of a type 2 odontoid process


fracture seen in A. Fracture of spinous
processes of C7 and Th1 vertebrae named Clay
– shoveler fracture in B.
Same patient as in Figure 13, but with open
mouth view, and the fracture through the body Even in adults, a normal cross-table
of C2 is visible, also note misalignment of lateral
lateral x-ray does not exclude a spinal
borders of C1 and C2 and difference in space
between odontoid process and lateral masses of cord injury. If in doubt, treat as if there is
C2 on both sides.
spinal cord injury until proven otherwise.
It is also worthwhile to memorize a short
SCIWoRA (Spinal Cord
mnemonic for children: SCIWoRA (Spinal
Injury Without Radiographic
Cord Injury Without Radiographic
Abnormality)
Abnormality).
Plain radiographs are negative in 25% of
pediatric patients with an injury to the
spinal cord. Tenderness of the neck and
careful neurologic examination must stay References and Further Reading, click
the main way of diagnosing a patient, here
especially in the pediatric population.

757
Section 5

How to read chest x-rays

Introduction Meanwhile, the X-ray tube should be


by  Ozlem Koksal Chest X-ray interpretation is one of the 180 cm away. Unfortunately, the
fundamental skills of every doctor. majority of the patients may not fit the
Emergency physicians are particularly ideal situation because of their acute
exposed to various chest x-rays during problems. Emergency physicians
a regular shift. Therefore, knowing the interpret many portable (bedside)
basics and pathologies in the ED anteroposterior chest x-rays with poor
setting is very important. This chapter quality, without lateral views to make
will summarize the basics of chest x- the diagnosis. The image quality is one
ray interpretation and give some of the most important things in image
pathologic examples. interpretation.

There are 3 types of chest films; Assessing The Image Quality


“RIPE” mnemonic is used; Rotation,
• AnteroPosterior (AP)
Inspiration, Position,

• PosteroAnterior (PA) Exposure(Penetration).

• Lateral Rotation: The clavicles should appear


symmetrical and be seen as equal
The ideal timing can be defined as
length. The distance between the
the  end of inspiration, and the patient
thoracic spinal process and clavicular
should hold his breath at that time.
heads should be equal (Image 18.59). If

758
there is a rotation, mediastinum may look Image 18.60 Image 18.61
abnormal.

Image 18.59

The chest x-ray shows adequate inspiration.

Position: PA, AP, or lateral view? The The right ribs (red arrows) and left ribs (green
The clavicular heads and spinous process standard chest X-Rays consists of a PA arrows) on the lateral chest X-Ray.
alignment. The x-ray shows minimal rotation.
Compare X and Y. and lateral chest X-Ray.
On the AP film, the chest has a different
The normal lateral chest x-ray view is appearance. The heart and mediastinal
Inspiration: On good inspiration, the
obtained with the left chest against the shadow are magnified because of
diaphragm should be seen at the level of
cassette. If the x-ray is a true lateral, the anterior structures, mainly sternum. This
the 8th – 10th posterior rib or 5th – 6th
right ribs are larger due to magnification view is taken mostly at the bedside as
anterior rib.
and usually projected posteriorly to the portable. Some patients are at semi-erect
left ribs (Image 18.61). or supine position. Therefore, mediastinal
structures are widened because of
 
gravity.

759
The pulmonary vasculature is altered Image 18.63 Image 18.64
when patients are examined in the supine
position. The size of the pulmonary
vasculature is more homogeneous
throughout the upper and the lower
lobes. (Image 18.62 and Image 18.63).
Supine views are less useful and should
be reserved for critical patients who
cannot stand erect position.

Image 18.62 The normal X-Ray film


Underexposed PA X-Ray film.You can not
appreciate thoracic vertebras.
The AP X-Ray shows magnification of the heart
and widening of the mediastinum. Interpretation
The interpretation of a chest X-Ray
Exposure / Penetration: Ideally, you
should be approached systematically. For
should be able to see the heart, the blood
chest X-Rays, there is a classic
vessels, and the intervertebral spaces.
schematic: “ABCDEF.” You should first
Exposure should be adequate if you are
check the patient’s name and date of the
able to see approximately T4 vertebra
film. You should also check the side
and spinal process. If the film is
marker, and the film position (PA or AP).
underexposed, you will not be able to see
Finally, you should check patient’s
them (Image 18.64). If the film is
position such as supine, erect or semi-
overexposed, details of bone structures
erect.
will be lost (Image 18.65).
  The analysis is ABCDEF:

760
Image 18.65 A – AIRWAY push the trachea to the opposite side
The trachea, carina and both main and resulting in a deviation that will show
bronchi are called the upper airway and up on chest X-Ray.
should all be visible on an AP view (Image
18.66). B – BONES
A chest X-Ray provides a good view to
Image 18.66 look for ribs and clavicle fractures.
Clavicular fractures are usually at the
middle 3rd of the clavicle, which is easy
to see in chest X-Rays. Rib fractures,
however, can sometimes be hard to see.
Each rib should be followed across its
length to look for fracture lines or step-
offs that could indicate a fracture.
Overexposed PA X-Ray film. You are able to see
all vertebral bodies with obvious intervertebral Hyperinflated lungs are seen as the result
spaces.
of chronic obstructive pulmonary disease
• Airways where the patient is unable to fully expel
the air that is inhaled with every breath.
• Bones Because of this, overinflation will result in
Airway structures on the chest X-Ray. (Red
a greater number of ribs that can be
• Cardiac Arrows: trachea, Green Arrow: carina, Pink
Arrows: left and right main bronchus) visible on the chest X-Rays. Normally,
• Diaphragm 8-10 ribs are expected to be seen on the
Look for if there is any deviation of the
chest X-Ray (Image 18.67).
• Extrathoracic tissues trachea away from the midline.
Introduction of air into one side of the
• Fields and Fissures
chest cavity will cause that side of the
lung to collapse. The collapsed lung will
761
Image 18.67 Ray, it refers to cardiomegaly or further investigation is considered if it
pericardial effusion. is more than 8 cm.

The aortic arch and the left pulmonary


artery should be visible as two semi-
Image 18.68
circles above the left atrium. There is a
space called the “Aortopulmonary
Window” that has the following borders:
a s c e n d i n g a o r t i c a rc h ( a n t e r i o r ) ,
descending aortic arch (posterior), left
pulmonary artery (inferior), inferior border
of aortic arch (superior). The window
should be “concave” in the lateral border
(Image 18.68). If it is not, mediastinal
Bone structures on the PA chest X-Ray.
(Numbers: ribs, red dashed line and arrows: lymphadenopathy and aorta/pulmonary
clavicle, yellow dashed line and arrows: medial artery aneurysms are possible. The left
border of scapula, green dashed line and arrows:
3rd rib, pink dashed line: vertebras) hilar point is slightly higher than the right
Heart borders on the AP chest X-Ray. (Pink
hilar point. The hilar point should be at
dashed lines and arrows: heart borders, Yellow
C –  CARDIAC the level of the lateral extent of the right dashed line and arrow: Aortic Arch, Blue
This part involves the heart and circle, and arrow: Aortopulmonary Window)
6th rib. The inferior vena cava lies end of
surrounding structures. The silhouette of t h e r i g h t c a r d i o p h r e n i c a n g l e .

the heart should be identified, and the D – DIAPHRAGM
The structures should be visible behind The outline of the diaphragm should be
heart borders should be clear. As a the heart especially the spine, paraspinal clear and smooth. Right hemidiaphragm
general rule, the heart base should not be region and azygoesophageal line. should be higher than the left (Image
wider than 1/2 the total width of the
18.69). It has 3 major characteristics that
diaphragm. If the heart base is 1/2 the In  ideal circumstances, mediastinum is
can be found on chest X-Ray:
width of the diaphragm on the chest X- maximum 6 cm in a PA chest x-ray, and

762
Image 18.69 1.The gastric air • Middle zone: between 2nd and 4th costal cartilage.
bubble on the left.
• Lower zone:
2.The diaphragmatic between 4th and
Image 18.70 Radiological lung zones.
contour looks like a 6th costal
“dome” shape, and cartilage.
the right side
So you should
located little higher
compare the lung
than the left.
parenchyma left to
3.The costophrenic right in the upper,
angle is the lateral middle and lower
point of attachment zones and see
for the diaphragm, whether there is a
The view of the diaphragm on the AP chest X- and it should be a difference.
Ray. (Yellow dashed lines and arrows:
diaphragm, red arrow: gastric air bubble, pink clear, sharp, and a
dashed lines: costophrenic angles) Look for equal
triangle-shaped at
radiolucency
either end. If the
between the left and the right lungs zones. The horizontal fissure
angle is closer to 90 degrees, then the lungs could be
on the right divides the upper and middle lobes; from the hilum to
hyperexpanded (e.g., COPD) and be pushing the diaphragm
the 6th rib at the axillary line.
down into the abdomen. If the costophrenic angle is blunting,
that usually is indicative of pleural effusion. You should also check soft tissues outside the thorax for
subcutaneous air, foreign body, bizarre density, etc. 
E – EXTRATHORACIC TISSUES
Mostly this means as the lung parenchyma. Lung fields can be F – FIELDS AND FISSURES
divided into zones: upper, middle, and lower zones (Figure-12); You should check lung fields for infiltrates. Identify the location of
infiltrates and identify the pattern of infiltration (interstitial or
• Upper zone: from the apex to 2nd costal cartilage.
alveolar pattern). Look for air bronchograms, nodules, Kerley B
763
lines. Pay attention to the apices. You should also check for
masses, consolidation, pneumothorax and vascular markings.
Vessels should be almost invisible at the lung periphery. Finally,
you should evaluate the major and minor fissures for fluid
collection (Image 18.71).

Image 18.71

Minor (A) and major (B) fissures of the lung.

Please visit our Flickr channel to see various  chest x-ray


pathologies.

References and Further Reading, click here

764
Section 6

How to read head CT

For a standard approach to read head/ Diagram 18.1


by Reza Akhavan and Bita Abbasi brain computed tomography (CT) scan,
one should adhere to systematic
algorithms.

The predefined algorithms are various,


and their main usage is building a
mental pathway that leads the novice
readers not to miss a point. Our
recommended algorithm is shown in
Diagram 18.1.

In the interpretation of head CT, the


most critical diagnoses for emergency
physicians are hemorrhage or ischemic
stroke, and midline shift. These
diagnoses or pathologies require
Step by step approach to brain CT in
immediate action. Therefore, checking
emergency situations
the brain parenchyma is the first step
for emergency physicians.

765
1.Brain Parenchyma posterior attachments of the falx cerebri.
 Image 18.73
*: Bold and underlined structures are
Midline shift marked in accompanying figure.
Presence of mass effect from edema or
space-occupying lesions may cause a
shift in midline structures. The shift of Image 18.72
midline may cause compression on the
anterior cerebral artery and eventually
infarct.  There are multiple sulci and
Axial non-contrast brain CT scan shows an ICH
cisterns in the brain that are filled with in the right parietotemporal lobe (arrow in a) with
CSF. The presence of effacement in these adjacent edema. SAH is seen in the brain sulci
(arrowhead in a). Red-line in (b) represents the
structures is another sign for the midline. Note the deviation of septum
pellucidum (blue line), third ventricle (yellow
presence of a space-occupying lesion or Axial brain C scan (a) and its corresponding line), and pineal gland (green line).
parenchymal edema. schematic view (b) depict the midline structures.
Falx cerebri (green), septum pellucidum
(magenta), third ventricle (yellow) and pineal Spontaneous hemorrhage
A note on anatomy (Image 18.72 and gland (orange) should be located in the midline.
Image 18.73): There are three midline Also known as hemorrhagic stroke,
structures that should be scrutinized Abnormal parenchymal spontaneous intracranial hemorrhages
when searching for midline shift: hyperdensity most commonly occur in hypertensive
Intraparenchymal hemorrhages manifest
patients. The most common locations are
1. Septum pellucidum: a membrane as hyperdense areas in brain parenchyma
basal ganglia, thalamus, pons, and
located between lateral ventricles and are really difficult to miss! They are
cerebellum. Hemorrhages outside these
generally categorized as spontaneous or
2. Third ventricle common locations may be secondary to
secondary to neoplasms, vascular
tumors or vascular malformation.
3. Pineal gland malformations or trauma, etc.
A note on anatomy (Image 18.74 and
All these three structures should be on Image 18.75)*: Deep grey matter nuclei
the line drawn between anterior and

766
are islands of grey matter located deep in *: Bold and underlined structures are i n the emergency departments are
the brain: marked in accompanying figure. traumatic. Traumatic hemorrhages may
be intra-axial (within brain parenchyma) or
• Thalamus Image 18.75 extra-axial. Intra-axial hemorrhages like
• Putamen contusions or hemorrhagic diffuse axonal
injuries are discussed here. Extra-axial
• Globus pallidus hemorrhaged will be discussed later.

• Caudate nucleus Contusion

Caudate nuclei, putamen and globus Contusions are caused by impaction of


pallidus are collectively known as basal b r a i n p a re n c h y m a o n h a rd b o n y
ganglia. Internal capsule is a white matter protrusions, so direct contact with bony
structure located adjacent to deep nuclei.
 protrusions affects cortical grey matter
(Illustration 18.1). They are most
commonly seen in frontal and temporal
Image 18.74 lobes (Image 18.76). They are often
hemorrhagic and easily seen on
computed tomography. In the control CT
Axial brain CT scan of a 57-year-old woman that scans after a few days, the perilesional
presented with left-sided paraparesis, shows a
hyperdense hemorrhage in the right basal edema progresses, and the lesions
ganglia (asterisk). Note the adjacent hypodense become more readily visible.
edema (black arrow) and mass effect on the
lateral ventricle (white arrow).
Diffuse axonal injury

Axial brain CT scan (a) and corresponding Secondary hemorrhage


In case of rotational acceleration traumas
schematic picture (b) represent the basal ganglia
and associated structures. Red: head of caudate The most common secondary (brain traumas associated with rotations
nucleus, Green: Globus pallidus, Blue: Putamen,
Pink: Thalamus, Yellow: internal capsule. parenchymal hemorrhages encountered and change of speed), white matter and

767
Illustration 18.1 Image 18.76 Unfortunately, only 15% of DAIs are
visible on CT scan. MRI remains the most
sensitive modality for detecting these
lesions. When visible on CT, they present
as hemorrhagic foci in the grey-white
interface, near deep nuclei of the brain
and in the corpus callosum (Image 18.77).

Axial brain CT scan in a trauma patients shows Image 18.77


multiple hemorrhagic lesions in the right frontal
(a) and right temporal (b) lobes consistent with
contusions. Note the perilesional hypodense
edema.

Illustration 18.2

Schematic representation of common locations


for the contusion. Contusions most commonly
occur in inferior portions of the frontal lobe,
temporal lobe, underneath the direct impact
(coup) or at the opposite site of direct impact
(countre-coup).

grey matter experience slightly different


changes of speed. This disrupts axons at
the grey-white interfaces (Illustration
18.2). This kind of lesion is called diffuse
axonal injury (DAI).
Axial non-contrast brain CT scan of a 34 y/o
male from a motor vehicle accident. Multiple
hyperdense hemorrhagic lesions are seen in the
Schematic representation of diffuse axonal grey-white junction (a), adjacent to thalamus (b)
injury. Rotational forces disrupt the axons at and corpus callosum (c). Multiplicity and location
grey-white junctions. of the lesions are compatible with diffuse axonal
injury.
768
Abnormal parenchymal Image 18.78 Image 18.79
hypodensity
In most cases, abnormal hypodensities
represent cerebrovascular accidents
(involves the cortex) or edema secondary
to other pathologies (usually without
cortical involvement). Stroke is defined as
acute onset of focal neurologic defect
due to cerebrovascular compromise.
Most strokes are ischemic (80%), and
some are hemorrhagic (15%).
Subarachnoid hemorrhage accounts for
the remaining 5%.

Brain CT scan findings in ischemic stroke


are mainly a factor of time and involved Axial non-contrast CT scan of a 63 y/o man with
artery. sudden onset right hemiparesis shows
hyperdensity in the middle cerebral artery. This Axial non-contrast brain CT in a 49 y/o woman
finding –being the result of arterial thrombosis- is presented to the emergency medicine with a 3-
In the first hours of a stroke, thrombosis the earliest finding in the CT scan of acute day history of dysarthria and right-sided motor
in the supplying artery creates a ischemic stroke. If you are able to discern the weakness. There is cortical hypodensity in the
sign and diagnose a stroke in this phase, be left frontal lobe (arrow) that represents ischemic
hyperdense artery sign. This is the proud of yourself! infarct in the territory of the anterior cerebral
earliest imaging finding of acute stroke in artery.
Cortical hypodensity (Image 18.79) and
non-contrast CT scan (Image 18.78).
sulcal effacement (Image 18.80) are
Another imaging finding in non-contrast relatively late signs of stroke.
CT scan is the loss of grey-white
differentiation

769
Image 18.80 with grey matter and contains supplying connection, and it takes a
capillaries), arachnoid network (that considerable amount of force for them to
contains CSF and absorbs it into dural be separated. In practice, epidural
veins via arachnoid granulations) and hematomas are mostly due to arterial
dura mater (that is in direct contact with hemorrhage (especially middle meningeal
periosteum). The layers are depicted artery) and are usually associated with
schematically in Illustration 18.3. skull fractures (Illustration 18.4).

Illustration 18.3
Illustration 18.4

Axial non-contrast CT shows a faint hypodense


area in the left frontoparietal area suggesting
ischemic stroke. Sulcus effacement is noted in
comparison to the other side (arrow). Note the
subtlety of findings in the acute phase of stroke.

2.Extra-axial spaces Schematic representation of meningeal layers.

Extra-axial spaces are defined as the


The most common extra-axial
space within the skull that is not part of
pathologies are traumatic.
brain parenchyma. Meningeal layers
engulf the parenchyma and separate it Epidural hematoma
from the calvarium. Epidural space is a potential space
located between the periosteum and dura Schematic representation of epidural
The meninges are composed of three hematoma. Note that epidural hematomas do
mater. These layers have a tight not cross suture lines and may be associated
layers: pia mater (that is in direct contact with skull fractures.
770
Epidural hematomas appear as biconvex or lentiform S u b d u r a l s p a c e i s a potential space between the inner
hyperdensity in the brain periphery (Image 18.81). They may cross layer of dura and arachnoid membrane (Illustration 18.5).
the midline but do not cross the sutures. This is because of the Hematomas within subdural space are usually due to rupture of
attachment of dura to the sutures. bridging veins located in this areas.

Subdural hematomas appear as crescent-shaped collections of


blood overlying the
Subdural cerebral Image 18.82
Image 18.81

Illustration 18.5

axial non-contrast brain CT scan shows an acute


SDH in the left hemisphere with midline shift.
Axial non-contrast brain CT scan shows a
lentiform epidural hematoma in the right
hemisphere. Schematic representation of subdural hemispheres (Image 18.82). They may cross
hematoma. Note that subdural hematomas may
cross the suture lines, but do not cross the the sutures or track along the falx or
hematoma midline. tentorium, but do not cross the midline.
771
As time progresses, the density of SDH decreases. Therefore Image 18.84
subacute SDH is isodense to brain parenchyma and becomes
gradually hypodense in chronic stages. In the subacute phase,
SDH may not be readily discernible, and attention to the midline
shift helps in diagnosis (Image 18.83).

Image 18.83

Axial non-contrast brain CT scan shows the chronic phase of epidural


hematoma. There are areas of increased density (arrows in a and b) in the
EDH that are compatible with active rebleeding.
Image 18.85

Axial brain CT-scan reveals a midline shift. As a novice image interpreter, you
may see no other pathologies. Look carefully at the left hemisphere! What
appears as a thickened cortex, is actually an isodense subacute SDH, hence
known as thick cortex sign.

The layered appearance of SDH is a sign of active rebleeding.


Call the neurosurgeon immediately (Image 18.84)!

Subarachnoid hemorrhage (SAH)


Subarachnoid space is the space between subarachnoid This patient presented to the ED complaining of a severe headache. Axial
membrane and pia mater. The space contains CSF and extends non-contrast brain CT-scan shows hyperdense SAH in the basal cisterns
(white arrow) and right-sided Sylvian fissure (black arrow). Note the
hypodensity in the adjacent parenchyma that represents secondary edema.
772
into brain sulci and cisterns. In the CT h i n d e r i n g C S F r e s o r p t i o n .
 Image 18.86
scan, SAH is seen only in the acute *: Bold and underlined structures are
phase and presents as hyperdensity marked in accompanying figure.
within the sulci and cisterns (Image
18.85). Illustration 18.6

Subarachnoid hemorrhages may be


traumatic or non-traumatic. Traumatic
SAH is usually associated with SDH.
Non-traumatic SAH is usually due to a
ruptured berry aneurysm.

Dural vein thrombosis


Dural veins are located between dural
layers. Increased density in these Axial non-contrast brain CT scan shows
increased density in the posterior part of
structures is a sign of dural vein superior sagittal sinus indicating thrombosis
thrombosis. (arrowhead). There is also a hyperdense
intracranial hematoma (ICH) in the left parietal
lobe (arrow) with adjacent edema. Remember
A note on anatomy (Illustration 18.6 and that intraparenchymal hematoma is due to
Image 18.86)*: Dural veins are venous impairment of blood flow and venous infarct
A schematic view of main dural veins and their secondary to dural vein thrombosis.
structures of the brain that are located correspondence on axial brain CT scans.
between dural layers. In addition of Superior sagittal sinus (a and b), straight sinus
enter the structure from adjacent
(c), transverse sinuses (d) and sigmoid sinuses
draining venous blood, they have the (e) are outlined on the CT images. parenchymal hemorrhage or blood in the
critical task of CSF resorption. Dural vein subarachnoid space (SAH) may enter the
thrombosis causes increased pressure in Intraventricular hemorrhage ventricle via CSF flow (Image 18.87).
the venous structures and may lead to (IVH)
The most common cause of IVH in adults
infarcts and hemorrhages. This may also
is trauma. Blood in the ventricles could
cause increased intracranial pressure by
773
Image 18.87 Image 18.88 Simple linear fractures are the most
common types of skull fractures and
appear as linear hypodense lines in the
skull. Depressed skull fractures are
usually accompanied with contusions in
the underneath brain parenchyma (Image
18.89).

A 55 y/o man presented with sudden loss of Image 18.89


consciousness. Axial non-contrast brain CT scan
(a) shows ICH in the right temporal lobe (black
arrow), SAH in basal cisterns (white arrows in a)
and IVH in the fourth ventricle (arrowhead). As
the ICH was located outside the common
locations of spontaneous ICH, a CT angiography
of cerebral vessels was performed (b). In CT
angiography an outpouching of contrast was
noted (white arrow in b) that is consistent with an
aneurysm.

Axial non-contrast brain CT scan shows


Hydrocephalus dilatation of lateral ventricles that suggests
hydrocephaly. Axial brain CT-scan in a young man with a
Hydrocephaly or dilatation of ventricles shotgun injury. Bone window (a) reveals a
(Image 18.88) may be the result of mass- depressed skull fracture (arrow in a). Note the
occupying lesion that obstructs CSF flow
3.Bones hyperdense pellet embedded near the fracture.
Parenchymal window (b) shows a contusion
The presence of skull fractures is not (arrow in b) in the adjacent parenchyma.
in ventricular structures or may be due to
always a sign of underlying brain injury.
impairment of CSF resorption via
However, the significant amount of force Warning signs in a skull fracture include
subarachnoid granulations.
that causes a skull fracture mandates the presence of intracranial air
careful evaluation of other associated (pneumocephalus)(figure 28), depression
intracranial injuries. of inner table of calvarium (depressed
  fracture), overlying scalp laceration (open

774
skull fracture), or fractures adjacent to Image 18.91 Illustration 18.7 How to read CT
dural veins or middle meningeal artery
(figure 28). Whenever you encounter a
skull fracture, look carefully at these red
flags!

Image 18.90

Axial non-contrast bone window CT scan of the


brain shows a fracture in the anterior wall of the
Axial non-contrast brain CT scan in a trauma left maxillary sinus (arrow). The air-fluid level
patient. Bone window (a) shows a linear skull within the sinus (arrowhead) indicates
fracture (arrow in a). The location of the fracture hemorrhage.
near middle meningeal artery and presence of
pneumocephalus (arrowheads in a and b) Another CT head interpretation
warrants further evaluation. Parenchymal
window (b) depicts an epidural hematoma mnemonic is BLOOD CAN BE VERY
(arrow in b) that is secondary to the rupture of
BAD. We will not go over all details of this
the middle meningeal artery.
mnemonic, but we have an infographic
Finally, note the visible parts of paranasal for you (Illustration 18.7). Please visit
sinuses and look for possible fractures. our  Flickr channel  for more pathologic
The presence of air-fluid levels or CT scan images.
hyperdensity within the sinuses might be
an indication of fracture in the trauma References and Further Reading, click

setting (Image 18.91) here

775
Section 7

How to read pelvic x-rays

Case Presentation
by Sara Nikolić and Gregor Prosen A 27-year-old woman was in a car accident. She is
hemodynamically stable with vital signs as follows:
temperature of 36.4°C, heart rate of 70 bpm, blood pressure
120/80 mmHg, respiratory rate 10/min, oxygen saturation 99%
on room air. During the secondary survey, pelvic bones are not
stable, and there is a pain on palpation. You placed a pelvic
binder and ordered a pelvic X-ray.

Introduction
Pelvic fractures carry life‐threatening injury potential which should be identified or
suspect during the primary assessment of patients with major trauma. The
prevalence of pelvic fracture in studies of patients with blunt trauma is between
5% and 11.9%. The mortality from pelvic fractures in patients who reach hospital
is reported to be between 7.6% and 19%. Usually, injuries are secondary to
massive force, such as a road traffic accident or fall from a height. Fractures may
be associated with vascular, soft tissue and visceral injuries. If the pelvic ring is
broken in two places, the fracture is likely to be unstable. Isolated ring fractures,
however, tend to be stable. Patients who survive a pelvic fracture are at risk for

776
significant complications such as chronic close to them, especially at the posterior, t h e femoral head impacts and fractures
pain, leg length discrepancy, sexual can also be injured. The bleeding is the posterior margin of the acetabular
dysfunction, or nerve palsy. usually venous and extraperitoneal and rim.
can be life-threatening.
Important Anatomical Lateral compression  produces a
Considerations If bones fracture but the ligaments remain horizontal fracture through the ipsilateral
The three bones compose the pelvis (the intact, a tamponade effect can be pubic symphysis and momentary medial
sacrum and the two innominate bones). achieved, and the degree of hemorrhage displacement of the hemipelvis. A lateral
Strong ligaments keep these three bones limited. compression force can also impinge on
together. These are crucial for maintaining the upper femur causing central
pelvic stability. A large array of ligaments
Mechanism of Injury dislocation of the hip.
The Young-Burgess system identifies four
traverses the interior and exterior surface
types of pelvic ring disruption, based on Vertical shear  forces the hemi-pelvis
of the posterior aspect of the pelvis.

interpretation of radiographic images: upwards and towards the midline and can
Two ligaments originate from the side and
anteroposterior compression, lateral tear all the sacroiliac ligaments on the
back of the sacrum and insert into the
c o m p re s s i o n , v e r t i c a l s h e a r a n d affected side as well as the pubic
ischial spine and ischial tuberosity.
combined mechanical injury. symphysis ligaments.
The pubic symphysis, a
Anteroposterior compression  causes Complex pattern  happens in less than
fibrocartilagenous joint, is supported by
“open book” look at one or both sides of 25% of cases. The pelvis is exposed to
ligaments. However, adds little to the
the pelvis. A diffuse force will disrupt the two or more of the forces mentioned
overall stability of the pelvis. The urethra
pubic symphysis, while a more direct above. A combination of injuries results in
and bladder lie close to the pubic
force fractures the pubic rami in a vertical a complex radiological picture.
symphysis, and there is a 20% risk of
plane. For the pubic bones to separate by
injury if symphysis is disrupted.
over 2,5 cm, one or both of the ligaments X-Ray Views
Torn or rupture of the ligaments can associated with sacroiliac joints have to The routine pelvic view is anteroposterior

cause separation of three bones. In this be torn. An anteroposterior force can also (AP) projection, and in 94% of cases, a

situation, the nerves and vessels running push the flexed femur backward so that correct diagnosis can be made from this

777
view. When the fracture is noted in the AP Normal findings •Check the pubic symphysis
view, special views (inlet and outlet view AP View Interpretation Summary
• Check the sacroiliac joints
and oblique views) for further
investigations are recommended. A • Check the acetabulum
• A d e q u a c y a n d q u a l i t y

Radiographic interpretation is Ensure that the whole of the pelvis is S
systematized with ABCS approach: visible Soft Tissues

• Alignment • A l i g n m e n t
 • Check the disruption of fat planes


Assess the borders of the three circles inside the pelvis
• Bones
namely, the pelvic brim and the two
• Check for soft tissue shadows outside
• Cartilage and joints obturator foramina.
the pelvis
• Soft Tissues B
• Bones: Check each of the following
Details
systematically: A
In this step, focus the three circles
• Pubis Sacrum
Image 18.92 enclosed by the pelvis. One is created by
• Acetabulum the pelvic brim (A) and the other two by
the obturator foramina (B) (Image 18.93).
• Femoral heads
Trace around the edge of the large circle.
• Iliac crest Normally this has a smooth edge which is

• Lumbar vertebrae not disrupted by the sacroiliac joint or


pubic symphysis unless the patient is
C very old. The pelvic brim cannot be
• Cartilage and joints disrupted in only one place. As the pelvis
is not completely rigid, this disruption
778
Image 18.93 Image 18.94 f o r “teardrop sign” (acetabular floor)
(Image 18.95 and 18.96).

Image 18.95

may take the form of a minimal diastasis.


B
The inner margins of both obturator Examine the outer edges of the pelvis
foramina should then be inspected in the and its bony structure for evidence of
same way as the pelvic brim. Again these Next, focus on the anterior inferior iliac
fractures. These may present as areas of
are rarely broken in only one place. spine, anterior superior iliac spine and
increased density, lucency, or alteration of
Complete the examination of the look for the iliac crest to the sacrum. The
the internal trabecular pattern. Fractures
foramina by tracing along its superior sacrum should also be examined for
away from the three bony circles can
border to the inferior surface of the neck symmetry of its foramina (Image 18.97).
occur in isolation. The start point of the
of the femur. This is known as Shenton’s examination is the pubic symphysis.
C
line (Image 18.94). Then, slowly progress to the right or left Check for either widening or overlapping
side. Focus on the posterior and anterior of bones at the level of the symphysis
joint margin, the ilioischial line (posterior pubis (A). If you see one of those,
column), and the iliopectineal line disruption in the pelvic brim should be
(anterior column). To finish the exam, look investigated. Sacroiliac joints (B) at the
right and left sides must also be checked
779
Image 18.96 Image 18.97 Image 18.98

Outlet views are used to detect the


degree of vertical displacement of the
for widening, defects in the cortical
fracture fragments.
surface, overlapping of bone, and lack of
congruity of the joint margin (Image plane. Loss of this line indicates extra- Oblique (Judet) views are used to define
18.98). peritoneal hemorrhage or soft tissue acetabular fracture patterns. If a fracture
edema. Conversely, intra-peritoneal or abnormality of the acetabulum is
S hemorrhage can displace the line. suspected computed tomography will
Check for soft tissue shadowing both usually be necessary once the patient has
inside and outside the pelvis because Inlet and outlet views should ideally be
been adequately resuscitated and
hematoma and tissue edema can requested if there is clinical or
stabilized.
produce swellings which are visible on radiological evidence of a pelvic fracture.
the anteroposterior radiograph. Normally An inlet view looks down the lumen of the
the obturator internus muscle is seen on true pelvis. It is better than the
both sides of the pelvis as a dark grey anteroposterior view for showing the
line, which is due to the muscle or fat orientation of fractures of the pubic rami.

780
Abnormal Findings Image 18.101 Complex pelvic fracture Image 18.104 Hip dislocation (antero-
– open book fracture. inferior)

Image 18.99 Pubic rami fracture

Pay ettention to symphisis pubis and sacroiliac


joint seperations.

Image 18.100 Pubic rami and ischium Image 18.102 Femoral neck fracture Image 18.103 Hip dislocation
fracture (posterior)

Right femoral neck fracture after fall. Check the


Shenton’s line alignement.

781
Image 18.105 Acetabular fracture joints. Failing to trace around the bony edges, especially
the iliac crests and sacral foramina, will lead to fractures being
missed.

Epiphyseal lines may be misinterpreted as fractures. Remember


that the Y-shaped (triradiate) cartilage separating the pubis,
ischium, and ilium in the acetabular floor does not fuse until
puberty.

Accessory ossification centers (in particular the one in the


posterior acetabulum) may also be mistaken for fractures.
However, apophyses are usually bilateral, have a sclerotic margin,
and are not associated with overlying soft tissue signs.

Being systematical is crucial to make possible for the non-


specialist to interpret pelvic radiographs accurately. Table 1
shows the summary of how to read a pelvic x-ray.

Hints and Pitfalls


Ensure that the whole of the pelvis can be seen, including the iliac References and Further Reading, click here
crests, both hips, and the femurs distal to the lesser trochanters.
The adequacy of the penetration should also be assessed. Pelvic
rotation is determined by lining up the symphysis pubis with the
midline of the sacrum.

It is common for part of the iliac crest to be missing or poorly


penetrated in the films so that fractures cannot be seen. A rotated
film causes asymmetry of the bony circles and the sacroiliac

782
Chapter 19

Selected
Emergency Drugs
Section 1

Antidotes

An antidote is an agent or drug that there are a few indications that


by Hamidreza Reihani and Elham Pishbin can reverse the toxic effects of antidotes should be administered as
poisoning. The base of clinical practice soon as possible to prevent major
in the treatment of toxicities is complications and death. Cyanide
cardiopulmonary stabilization, antidotes for cyanide toxicity, naloxone
decontamination, enhancing for severe opium overdose and
elimination and supportive atropine for organophosphate
management. Antidotes are indicated poisoning and gas agents are some
in some specific and well-defined examples. 
situations, and they are not routinely
The antidotes administered in clinical
administered in toxin exposures.
practice are not a great deal. There is a
Therefore, the physician should know
list of essential antidotes and a brief
the indications and contraindications of
explanation of their character and
each antidote. Administration of the
application.
pharmacologic antagonists may
worsen the outcome in some situations
and are not recommended. 

There is usually enough time to start


the treatment after supportive care and
evaluation of the patient. However,

784
Antidotes and their • Infusion of 10-20% of stabilizing dose/ • Hypermagnesaemia

characteristics hour 
Precautions
Atropine • Large doses may be required
• Calcium chloride extravasation can lead
General information
• There are auto-injectors for rapid use to soft tissue necrosis, preferably
• Anticholinergic agent administered via central line
Other
• Competitive muscarinic antagonist • Continuous monitoring is
• Drying of respiratory secretions is the
recommended 
Indications goal
Dose
• Organophosphate poisoning • Tachycardia is not the endpoint
• 1 gram calcium chloride (10 mL),
• Carbamates Calcium (0.15mL/kg in children)
General information
• Nerve agents • 10-30 mL of calcium gluconate
• Calcium chloride 10% (1 g /10 mL),
Precautions (27.2 mg/mL elemental Ca)  Administration

• Excessive doses lead to anticholinergic • Calcium gluconate 10% (9 mg/mL • IV bolus over 5 minutes
symptoms elemental Ca), one-third of the calcium in
• Repeated doses every 10-20 minutes if
strength of calcium chloride
Dose needed
Indications
• Start with 1-2 mg IV (adults), 0.02 mg/kg • Infusion can be administered
IV (children) • Calcium channel blockers toxicity 
Other
• Double the dose every 2-3 minute to • Hydrofluoric acid exposure
achieve atropinization
• Hyperkalemia
Administration
785
• Topical calcium gel or local • 8 mg (adults), 4 mg • Cardiac monitoring is • Reverse the dangerous
injection of calcium gluconate (children) not approved needed cardiac effects of digitalis
for hydrofluoric acid skin
Administration • Avoid infusion more than 24 Indications
burns 
hours
• Oral  • Acute and chronic digoxin
• Intra-arterial or IV with a Bier
Dose overdose
block for extremity exposure • Can be repeated every 8
hours until 24 hours • Start with15 mg/kg/h • Other cardiac glycosides
Ciproheptadine
poisoning
General information Administration
Deferoxamin
General information Precautions
• An antihistaminic and • IV infusion
antiserotonergic agent • Iron-chelating agent • Close monitoring; ready for
• Infusion rate could be
resuscitation
• Has anticholinergic effects • Converts it to a water- increased
as well soluble complex excreted by • Monitor serum free level of
Other
urine digoxin
Indications
• Evaluate patient after 6
Indications Dose
• Control of symptoms in hours
serotonin syndrome • Systemic iron toxicity • Acute overdose: 5 vials; for
• The urine color will become
unstable patients 10-20 vials;
Precautions • Iron levels > 500 µg/dL red
can be calculated if the
• May cause anticholinergic ingested dose is known 
• Multiple pills on radiography Digoxin immune Fab
effects General information
Precautions • Chronic overdose: can be
Dose • Fab fragments of antibodies calculated by serum digoxin
• Hypotension may occur at level; start with 2 vials
to digoxin
rapid rates

786
Administration • 3 mg/kg  • Oral: loading (1.5-2 mL/kg 80-proof
liquor), maintenance (0.2-0.5 mL/kg/h)
• Bolus in life threatening conditions, Administration
otherwise infusion Other
• IM every 4 hours for 48 hours
Other • Maintain blood ethanol concentration
Ethanol between 100-150 mg/dl
• For other cardiac glycoside poisoning General information
start with 5 vials  Flumazenil
• Blocks the formation of toxic
General information
Dimercaprol metabolites of alcohols
(BAL) • Competitive benzodiazepine antagonist
Indications
General information • Not used routinely in benzodiazepine
• Methanol and ethylene glycol poisoning
poisoning
• Heavy metal chelator
Precautions
Indications
Indications
• Serum ethanol levels monitored every
• Reversal of procedural sedation
• Severe lead, inorganic arsenic and 1-2 hours
mercury poisoning • Pediatric poisoning (limited use)
• The dose should be doubled during
Precautions dialysis Precautions

• Many severe adverse effects Dose/ Administration • May cause withdrawal

• Nephrotoxic • IV: loading (10 mL/kg of 10% IV • May induce seizure


solution), maintenance (1-2 mL/kg/h of
• Administered in ICU Dose
10% IV solution)
Dose • 0.2 mg, 0.01 mg/kg (children)

787
• Repeat up to the desired effect or 3 mg  • In dialyzed patients is given every 4 Hydroxocobalamin
hours or continuous infusion General information
Administration
Glucagon • A precursor of Vitamin B12
• IV over 30 seconds
General information
• Hydroxyl group is displaced by cyanide
Other
• Increase cAMP** and form cyanocobalamin
• Not used in mixed drug overdose
• Positive inotropic and chronotropic Indications
similar to beta-agonists
Fomepizole • Cyanide toxicity
General information
Indications
Precautions
• An alcohol dehydrogenase inhibitor
• β-blocker toxicity
• It's a safe drug
Indications • Calcium channel blocker toxicity
Dose
• Methanol and ethylene glycol toxicity
Precautions
• 5 g, repeat if needed; 70 mg/kg
Dose • Induces vomiting, consider airway (children)
• Loading dose = 15 mg/kg management
Administration
• Maintenance dose = 10 mg/kg q12 Dose
• Infusion in 100 normal saline in 15
hours
• 5-10 mg (adults), 0.05-0.1 mg/kg minutes
Administration (children)
Other
• Infusion in 100 ml normal saline or 5% Administration
• Skin and urine orange-red discoloration
dextrose in 30 minutes
• The first dose is IV bolus, if there is a

Other clinical response, start infusion 

788
Insulin (High dose) Intravenous Lipid Emulsion • Methemoglobin-forming agents
General information General information toxicity

• It has strong inotropic effects • 20% lipid emulsion as a parenteral • Symptomatic methemoglobinemia 
nutrient
Indications • MetHb levels >20% in asymptomatic
Indications patients
• Calcium channel blocker toxicity
• Overdose by drugs with high protein Precautions
• Βeta-blocker toxicity
binding and large volume of distribution,
• Pulse oximetry is unreliable in
Precautions e.g. Local anesthetics, β-blockers and
methemoglobinemia
calcium channel blockers
• Glucose level should be monitored • Hemolysis in G6PD deficiency
every 10 minutes Dose/Administration
Dose
• Hypokalemia be considered • 1.5 ml/kg IV bolus
• 1-2 mg/kg IV, 1 mg/kg (children)
Dose/ Administration • 0.25 ml/kg/minute
Administration
• 1 IU/kg IV bolus of short acting insulin, Other
followed by 0.5-1 IU/kg/hr • Slow IV injection, may repeat 30-60
• Until hemodynamic stability restored
minutes later
• Glucose 25 g (dextrose 50%) before
Methylen blue Other
starting insulin, then 25 g/hr according to
General information
glucose level 
• MetHb levels measured frequently
• It reduces methemoglobin (MetHb) to
Other
hemoglobin N-acetylcysteine
• Higher doses were administered in General information
Indications
studies

789
• Preventing hepatocellular injury in • For reversing the opioid effects, • Synthetic analogue of somatostatin
severe acetaminophen toxicity respiratory and CNS depression
Indications
Indications Precautions
• Hypoglycemia due to sulfonylurea
• Serum acetaminophen concentration • Re-sedation may occur due to short
Precautions
above toxic level half-life of naloxone
• Break through hypoglycemia may occur
• Hepatocellular injury • Withdrawal in chronic users

Dose/Administration
Precautions Dose
• 50 µg IV then 25 µg/h or
• Oral therapy may not be tolerated due • Start: 0.1 to 0.4 mg; 0.01 mg/kg
to its taste and odor (children) • 100 µg IM or SC every 6 hours

Dose/Administration • Repeat every 2-3 minutes up to 10 mg


Physostigmine
Administration General information
• Oral: loading (140 mg/kg), then (70 mg/
kg q 4 hours) for 17 doses • Reverse anticholinergic syndrome
• Intravenously, intramuscularly, or
• IV: loading 150 mg/kg in 30-60 minutes subcutaneously
Indications
then 50 mg/kg over 4 hours, 100 mg/kg
Other • For CNS symptoms (delirium, seizure)
infused over next 16 hours
• Start with larger doses if respiratory due to anticholinergic drugs
Naloxone
depression exist
General information Precautions

• Infusions may be required


• An opioid antagonist • Contraindicated in bradycardia, AV
block and bronchospasm
Indications
Octreotide
General information Dose

790
• 0.5 – 1 mg (adults), 0.02 mg/kg • Maintenance: 500 mg/hr or 1-2 g q4-6h; • 50 mg IV every 6 hours for ethylene
(children) 10-20 mg/kg/hour (children) glycol toxicity

Administration Administration • 0.5 g/min infusion until the seizure stops

• IV slowly in 2 minutes or IM • IV infusion in 0.9% saline

• Repeat in 10 to 30 minutes if needed Other Sodium bicarbonate


General information
Other • Should be administered in the early
phase before irreversible binding occurs • Hyperosmolar Sodium Bicarbonate
• In rapid administration cholinergic
Injection
symptoms may occur Pyridoxine
General information Indications
Pralidoxime
General information • Vitamin B6 is essential for GABA • Cardiotoxicity due to fast sodium
production channel blockade (e.g., TCA* poisoning)
• Reactivate cholinesterase inhibition due
to organophosphates Indications • Urine alkalinization

Indications • Isoniazid, hydrazine and Gyromitra Precautions


poisoning
• Organophosphates poisoning • Hypokalemia is a concern
• Ethylene glycol poisoning
• Nerve agents • Serum pH maintained between
Dose/Administration 7.50-7.55
Dose
• 1 gram for each gram of isoniazid, 70 Dose
• Loading: 1-2 g IV; 25-50 mg/kg
mg/kg (children), maximum 5 gram
(children) • Start with 1-2 mEq/kg, further doses
may be needed

791
Administration • 25-75 mg/kg/day • Repeat after 30 minutes if clinically
needed
• First dose administered bolus Administration
Succimer (DMSA)
• Other bolus doses or infusion if required • Continuous infusion for 5 days
General information
Other Other
• Oral metal chelator
• Given only if there is evidence of • Usually starts 4 hours after first
Indications
cardiotoxicity, such as QRS widening and dimercaprol (BAL) injection
ventricular dysrhythmias • Symptomatic lead poisoning
Sodium thiosulfate
Sodium calcium edetate (EDTA) General information • Asymptomatic lead poisoning, lead level
General information > 60 µg/dl (adults), > 45 µg/dl (children)
• Help the body to detoxify cyanide
• IV heavy metal chelator Precautions
Indications
Indications • May cause neutropenia, gastrointestinal
• Cyanide poisoning
upset and liver abnormalities
• Severe lead toxicity
Precautions
Dose
• Lead level > 70 µg/dl
• In severe cases with other antidotes
• 10 mg/kg three times a day for 1 week,
Precautions
Dose then two times a day for 2 weeks 
• Patient should be admitted in hospital
• 50 ml of 25% (12.5 g; 1 ampoule) in Administration
• Nephrotoxicity, ECG changes and liver adults; 1.65 ml/kg (children)
• Orally 
test disturbance may occur
Administration
Other
Dose
• IV over 10 minutes
• The serum level should be monitored
792
References and Further Reading, click here

793
Section 2

Drugs for Pain Relief

Introduction 2. Chronic pain


by Nik Ahmad Shaiffudin Nik Him, Azizul Fadzi A patient presented with pain at the
Emergency Department (ED) Acute versus Chronic Pain
Pain is an unpleasant feeling. Sensory
commonly un-recognized, under-
neurons convey it to the brain by as a
treated and delayed in getting
result of injury, disease, or emotional
treatment. Prompt recognition and
disorder. Acute pain is defined as pain
alleviation of pain should be a priority
less than 6 months duration with a
when treating patient suffered from
known cause and disappears when the
pain. There are many drugs available
problem resolves. Chronic pain occurs
for pain relief. Optimal control of pain is
when pain lasts more than 6 months
essential for good patient care. It
duration, persists beyond the healing
prevents an adverse physiological and
time and usually the cause may not be
psychological effects, reduce the
determined (International Association
incidence of chronic pain,
for the study of pain, 2007).
postoperative morbidity and facilitate
earlier discharge from the hospital. The scientific approach to pain
management demands a step-wise
Drugs for pain relief may be used
approach, which utilizes lower risk
for:
inte rve nt ions first (W HO, 1996)
1. Acute pain especially in acute pain management.
It is important to understand the

794
different pain mechanisms of chronic pain 5. Others: Medications with no direct
as well as evidence-based multi- pain-relieving properties may also be
mechanistic treatment. It is also essential prescribed as part of a pain
to provide individualized treatment. management plan e.g. laxative, anti-
Pharmacological and non- emetic, steroids, bisphosphonates,
pharmacological aspect is as equally muscle relaxant and anti-spasmodic
important in chronic pain management at
The tables below shows some specific
decreasing pain and increasing
information about these agents.
functioning of chronic pain patients
during activity of daily livings.
Table 19.1 Non-opiods (Simple analgesic)
The classes of medications used in the RECOMMENDED SIDE CAUTIONS AND
DRUG DOSAGES EFFECTS CONTRAINDICATIONS
COMMENTS
treatment of pain (Adapted from ACPA
Paracetamol 0.5 - 1gm, 6 - 8 hourly Rare Hepatic impairment Preferred drug in elderly.
resource guide to chronic pain Max: 4g/day
Reduce maximum dose Liver damage following over
medication & treatment, 2015) include: 50%-70% in patients with dosage.
hepatic impairment
Maximum dose 4 g daily.
1. Non-opioids (simple, non-selective and
Perfalgan (IV) >50 kg, 1 g 6 hourly up to max Hepatic impairment Important to consider the
selective COX-2 inhibitors) e.g. aspirin, Aqueous 4g/day total dosage of paracetamol
solution: 10mg/ 10-50 kg, 15 mg/kg/dose used i.e. to include dosage
NSAIDs, and acetaminophen and ml paracetamol, max 60mg/kg in 4 divided doses of suppositories and oral
available in 50ml preparations.
celecoxib. and 100ml vials Administration:
Infusion over 15 minutes.
Renal & hepatic impairement:
2. Opioids (weak and strong) e.g., minimum interval between doses
should not be less than 6 hours
tramadol, morphine, codeine,
Provided by authors
hydrocodone, and oxycodone.

3. A d j u v a n t a n a l g e s i c s e . g .
antidepressants, anticonvulsants

4. Local analgesia, e.g. lidocaine patch


795
Table 19.2 Non-Opioid (Non-Selective NSAIDs)
RECOMMENDED CAUTIONS AND
DRUG DOSAGES
SIDE EFFECTS
CONTRAINDICATIONS
COMMENTS

Aspirin 325 to 650 mg orally or rectally Peptic ulcer, Gastroduodenal ulcer Current data suggest that increased CVS risk may be
every 4 hours as needed, not to GI bleed, Asthma an effect of the NSAIDs/Coxib class.
exceed 4 g/day. Platelet dysfunction, Bleeding disorder
Renal failure, Renal dysfunction Physicians and patients should weigh the benefits
Hypertension Ischaemic heart disease and risks of NSAIDs/Coxib therapy.
Allergic reaction in susceptible individuals, Cerebrovascular disease
Increase in CVS events Inflammatory bowel disease Concurrent use with aspirin inhibits aspirin‟s
antiplatelet effect (mechanism unclear)
Same for below agents Same for below agents
Same for below agents

Diclofenac Sodium 50 - 150 mg daily,


8 - 12 hourly
Max: 200 mg/day

Mefenemic Acid 250-500 mg 8 hourly

Ibuprofen 200-400 mg, 8 hourly


Max: 2400 mg/day
Elderly patients: 200 mg 3 x a
day

Naproxen 500-550mg BD
Elderly patients; 220 mg BD

Ketoprofen Patch: 30 -60 mg BD


Topical; PRN

Ketorolac IV: 10-20 mg BD


( max 3days)

Meloxicam 7.5-15 mg daily


Max: 15 mg /day

Provided by authors

796
Table 19.4 Non-Opioids ( Selective Cox-2 Inhibitors)
CAUTIONS AND
DRUG RECOMMENDED DOSAGES SIDE EFFECTS
CONTRAINDICATIONS
COMMENTS

Celecoxib 400mg BD in acute pain (48 hours only) Renal impairment Ischaemic heart disease Associated with lower risk of serious upper
200-400 mg daily (for longer term use) Allergy reaction in susceptible individuals Cerebrovascular disease gastrointestinal side effects compared to traditional
<18 years : not recommended Increase in CVS events Hypersensitivity to sulfonamides. NSAIDs
Elderly patients: 100 mg daily Hypertension Higher doses associated with higher
incidence of GIT, CVS side effects. Use the lowest effective dose for the shortest duration
Same for below agents Patients with indications for necessary
cardioprotection require aspirin
supplement
Uncontrolled Hypertension

Same for below agents

Etoricoxib 120 mg daily in acute pain (48 hours only)


60 - 90 mg daily (for longer term use)
Elderly patients 30 mg daily

Parecoxib 20-40mg 6-12 hourly (max 80mg/day for max


duration of 48 hours )
Elderly (>65 years & <50kg) reduce to half the
dose with a maximum daily dose of 40mg.

Renal & hepatic impairment :


Do not use

Provide by authors
Table 19.3 Opioids (Weak opioids)
CAUTIONS AND
DRUG RECOMMENDED DOSAGES SIDE EFFECTS
CONTRAINDICATIONS
COMMENTS

Tramadol 50 - 100 mg, 6 - 8 hourly Dizziness Risk of seizures in patients with Interaction with TCA, SSRI and SNRI
Max: 400 mg/day Nausea history of seizures and with high
Vomitting doses
Constipation In elderly, start at lowest dose (50
Drowsiness mg) and maximum 300 mg daily

Dihydrocodeine 30 - 60 mg, Nausea Respiratory depression Metabolites can accumulate causing adverse effects
tartrate 6 - 8 hourly Vomiting Constipation Drowsiness Acute alcoholism
(DF118) Max: 240 mg/day Paralytic ileus In severe hepatic impairment, codeine may not be
Raised intracranial pressure converted to the active metabolite- morphine.
Renal dysfunction &:dialysis patient:
do not use

Hepatic dysfunction:
do not use

Provided by authors 797


Table 19.5 Combinations of opioids and paracetamol
RECOMMENDED CAUTIONS AND
DRUG DOSAGES
SIDE EFFECTS
CONTRAINDICATIONS
COMMENTS

Paracetamol 500 mg + 1 - 2 tablets, 6 - 8 hourly Constipation Hepatic impairment Decrease in side effect profile of Codein/ tramadol
Codeine 8 mg Max: 8 tablets/day respectively and paracetamol while maintaining
efficacy

Paracetamol 325 mg + 1 - 2 tablets, 6 - 8 hourly Nausea Hepatic impairment, Epilepsy Same as above
Tramadol 37.5 mg Max: 8 tablets/day Vomiting
Drowsiness

Provided by authors

798
Table 19.6 Opioids ( Strong opioids)
CAUTIONS AND
DRUG RECOMMENDED DOSAGES SIDE EFFECTS
CONTRAINDICATIONS
COMMENTS

Morphine SC (Adults): Nausea Acute bronchial asthma Metabolites can accumulate causing increased
<65 yrs: 5mg-10 mg 4 hrly Vomiting therapeutic and adverse effects
>65 yrs: 2.5 mg-5mg 4hrly Pruritus Respiratory depression
Sedation Both parent drug and metabolites can be removed with
IV: Follow morphine pain protocol (Appendix) Constipation Head injuries,Renal and hepatic dysfunction: dialysis, watch for “rebound” pain effect
Respiratory depression needs dose adjustment
Oral: Starting dose 5- 10 mg, Myoclonus
4 hourly of IR

Elderly: 2.5 - 5 mg, 4 - 6 hourly of IR

Fentanyl To be prescribed by APS team only Nausea No active metabolites and appears to have no added risk
Vomiting of adverse effects; monitor with high long term user
Renal dysfunction : appears safe, however, a Sedation
dose reduction is necessary Constipation Metabolites are inactive, but use caution because fentanyl
Respiratory depression is poorly dialysable

Dialysis patients : appears safe Decrease hepatic blood flow affects metabolism more
than hepatic failure.
Hepatic dysfunction : appears safe, generally
no dose adjustment necessary

Oxycodone Starting dose (oral): Nausea Acute bronchial asthma Metabolites and parent drug can accumulate causing
IR 5 -10 mg 4 - 6 hourly Vomiting Respiratory depression toxic and CNS-depressant effects
(oxynorm) Sedation Con-comittent used of sedative drugs
Renal dysfunction : Use cautiously with careful Constipation Head injuries,Renal and hepatic dysfunction: In severe hepatic impairment, the parent drug may not be
monitoring, adjust dose if necessary Respiratory depression needs dose adjustment readily converted to metabolites

Dialysis patients:
do not use

Hepatic dysfunction:
Use cautiously and monitor patient carefully
for symptoms of opioid overdose
Decrease initial dose by 1/2 to 1/3 of the usual
amount
Elderly patients : 2.5-5 mg every 4-6 h

Provided by authors

799
Table 19.7 Adjuvant Therapies
CAUTIONS AND
DRUG RECOMMENDED DOSAGES SIDE EFFECTS
CONTRAINDICATIONS
COMMENTS

Antidepressant

Amitriptyline Start with 10 - 25 mg nocte. Anticholinergic effects Not recommended in elderly patients with Nortriptyline may be a suitable alternative and better
Increase weekly by 25 mg/day to a max of e.g. dry mouth, drowsiness, cardiac disease, glaucoma, renal disease tolerated in elderly at similar doses
150 mg/day urinary retention, arrhythmias
Interaction with Tramadol
Elderly patients: 10 mg ON Significant risk of adverse effects for the elderly

Duloxetine 30 - 60 mg/day Gastrointestinal disorder Narrow-angle glaucoma Interaction with Tramadol


Max: 120 mg/day Excessive sweating CNS Potent CYP1A2 inhibitors
disorder Concomitant use of MAOIs
Hypertension

Anticonvulsants

Carbamazepine 100 - 1600 mg/day Dizziness Increased ocular pressure Well tolerated.
Ataxia Latent psychosis Serious adverse events are rare
Elderly patients: 100 mg daily Fatigue Confusion
Leucopenia Agitation
Nausea
Vomiting
Drowsiness

Gabapentin Day 1: start at 300mg Drowsiness Dose adjustment needed in renal impairment However, need to monitor sedation, ataxia, oedema,
Day 2: 300 mg 12 hourly dizziness hepatic trans-aminases, blood count , serum
Day 3: 300 mg 8 hourly GI symptoms creatinine, blood urea and electrolytes
Thereafter, increase by 300 mg/day every Mild peripheral oedema
1- 7 days
Max: 3600 mg/day

Elderly patients : 100mg daily

Pregabalin Start with 150 mg/day (in 2 divided doses). Same as above Same as above Same as above
If needed, increase to 300 mg/day after 3 -
7 days intervals,
then if needed, increase to 600 mg/day
after 7 days interval Max: 600 mg/day
Elderly patients : 50 mg at bedtime

Provided by authors

800
Table 19.8 Other agents used for analgesia or an adjunct to analgesics
RECOMMENDED CAUTIONS AND
DRUG DOSAGES
SIDE EFFECTS
CONTRAINDICATIONS
COMMENTS

Bisphosphonates

Pamidronate 60 - 90 mg as a single infusion over Asymptomatic hypocalcemia, Hypersensitivity to biphosphonates. Rehydrate patients with normal saline before or during
2 - 4 hrs every 4 weeks hypophosphataemia, hypomagnaesemia treatment.
Flu-like symptoms Hyperparathyroidism
Mild fever Not to be given as bolus injection
Local injection -site reactions In renal impairment, reduce dose and
Malaise increase infusion duration required
Rigor
In patients with poor dental hygiene,
there is higher risk of ONJ. Dental
referral is advised

Zoledronate Acid 4 mg as 15 min IV infusion every 3 Hypertermia Same as above Same as above
- 4 weeks Flu-like symptoms
Headache
Hypersensitivity
Osteonecrosis of jaw

Steroids as anti
inflammatory

Dexamethasone Oral/ IV/SC: Increased or decreased appetite Peptic ulcer disease Concomitant Should be given before 6 pm to reduce risk of insomnia
8 - 16 mg daily or divided doses Insomnia, Indigestion, Nervousness NSAIDs use
(initial dose), then to reduce to Myopathy, Oral candidiasis Liver or cardiac impairment Efficacy may reduce over 2 - 4 weeks
lowest possible dose (usually 2 Adrenal suppression Use lowest possible dose to prevent side effects.
mg/day)
Anticipate fluid retention and glycemic effects in short-
Elderly patients :5 mg daily and term use and CV and bone demineralization with long-
taper as soon as feasible term use
Monitor for rash or skin irritation

Lignocaine
(topical)

Lignocaine 5% Elderly patients : 1-3 patches for Monitor muscle weakness, urinary function, cognitive
12 hours per day effects, sedation

Muscle relaxant

Baclofen 5 mg -15 mg daily Avoid abrupt discontinuation because of CNS irritability

Laxatives
801

Provided by authors
Table 19.9 Other agents used for analgesia or an adjunct to analgesics
RECOMMENDED CAUTIONS AND
DRUG DOSAGES
SIDE EFFECTS
CONTRAINDICATIONS
COMMENTS

Lactulose 15 - 45 ml orally 6 - 8 hourly Bloating, Epigastric pain Hypersensitivity to lactulose products May be mixed with fruit juice, water or milk
Flatulence, Nausea, Vomiting Galactosemia Reasonable fluid intake is required for efficacy
Cramping
Patients requiring a galactose free diet

Bisacodyl 5 - 10 mg orally, 1 - 2 times daily Atony of colon Intestinal obstruction


Max: 30 mg/day

Antiemetic

Metoclopramide 10 - 20 mg Extrapyramidal reactions Epileptic patients Gastrointestinal


6 - 8 hourly Dizziness hemorrhage
Drowsiness

Haloperidol 0.5-3 mg ON Extrapyramidal Syndromes Concomitant use with other


Dystonia psychotropic drugs may increase Extra-
Prolonged QT interval pyramidal Syndromes
Neuroleptic Malignant
Syndrome

Granisetron 1 mg 12 hourly Constipation Progressive ileus and/or gastric Should not be used as first line.
distension may be masked Not for long term use.

Ondansetron 8 mg 12 hourly Headache Pregnancy and lactation


Sensation of flushing or warmth in the head Hepatic impairment
and epigastrium
Constipation

Prochlorperazine 10 - 30 mg daily in divided doses Extrapyramidal symptoms May increased risk of seizure with
Severe nausea and vomiting: 20 Dry mouth Tramadol
mg stat followed by 10 mg after 2
hours
For prevention: 5 - 10 mg 8 - 12
hourly

Provided by authors

802
Management of Major Severe Vomiting supplemented with regular pain

Opioid Complications 1. Before any antiemetic, always ensure reassessments.


that patient is adequately hydrated,
Hypoventilation* or Unarousable 4. Drugs for pain relief should be chosen
good analgesia, and that
1. Stop infusion appropriately and to keep it simple as
hypoglycemia and hypotension are not
polypharmacy is associated with more
2. Oxygen12L/min.via Hudson mask causative factors.
side effects.

3. N a l o x o n e ( N a r c a n ) 0 . 0 1 m g / k g
 2. Reduce or stop infusion if necessary.


5. Pain medication should be given within
*Hypoventilation if 3. Give Ondansetron 0.15mg/kg IV or 20–25 minutes of initial evaluation at
Granisetron 0.05mg/kg IV over 10 min. ED including the plan of treatment. The
• Respiratory rate < 10 / min. for > 5
benefits not only improving patient
years old
Hints and Pitfalls comfort but also had physiological
• Respiratory rate < 15 / min. for 1 – advantages, e.g. reduction of pain-
The hints
5 years old related tachycardia in acute coronary
1. More than 75% of ED presenting
syndrome and aortic dissection.
• Respiratory rate < 20 / min. for < 1 complain is related to pain.
year old. 6. History of medication that had been
2. Severe pain creates a barrier to obtain
taken and failed prior to ED
an adequate history and physical
Apnoea presentation is crucial. It should be
1. Stop infusion exam. It can be easily resolved by
known that medications which have
giving early pain medication thus
2. V e n t i l a t e w i t h b a g a n d failed at home are likely to fail in the
facilitate better patient care.
mask(100%oxygen) ED.
3. Assessment of pain severity is
3. Check pulse, if absent start CPR 7. Always consider targeted analgesia.
challenging that requires a holistic
approach. Thus, regardless of one’s • No doubt non-specific analgesics
4. Naloxone(Narcan) 0.01mg/kg
preferred approach, the assessment (e.g., NSAIDs, opioids) useful in
method should be used and the ED but the risk of side effects
803
may be significant in certain 10.Pain care is an ongoing process in the component in pain care. Pain
population thus targeted analgesia ED and after discharge. score should be monitored with
is the best approach. the aim of addressing relief
• Ignorance of the principle of
(“correcting” where possible).
• ED providers should consider a ongoing pain treatment lead to
specific and effective therapy risks of “wind-up” and increased • Pain is inevitable whereas
available, e.g. local nerve block analgesia requirements s u ffe r i n g i s o p t i o n a l , t h u s
emergency care provider’s needs
8. I n a c u t e s e v e r e p a i n , f a s t • Proper pain care saves time
to treat the pain or acknowledge
administration of drugs for analgesia is overall (as for a fracture), it will
the reason for non-treatment as
better and preferable.  likely be necessary for at least a
such should occur both in
few days and often more after
• The key with regard to analgesia conversations with the patient (or
discharge
administration route is neither family) and in the medical record.
“always use IV” and nor “the more The pitfalls 3. F a i l u r e t o a n t i c i p a t e m a j o r
severe the pain, the more likely IV 1. The response to drugs for pain relief
complications of pain relief medication,
is the right route.” varies for individual. Therefore, there is
e.g. toxicity, anaphylaxis reaction.
no uniform pain threshold.
• In difficult IV access or IV route’s
• Close monitoring during
disadvantages seem to outweigh • Heredity, socio-cultural level,
intravenous administration of pain
i t s b e n e fi t s , a l t e r n a t i v e energy level, coping skills, and
medication to identify major
approaches may be best prior experiences with pain define
complication is preferred.
pain tolerance among individuals.
9. Any suspicions of drug abuse, e.g.
• Early and systematic approach in
preference or insist on certain opioids, 2. Neglect of pain medications at ED
management of major
ECP should obtain a detailed history when busy in resuscitating the patient.
complication can improve the
and consider for referral to
• Assessment of pain is a morbidity and mortality
psychologist for evaluation for drug
necessary, but not a sufficient
abuse
804
4. Unrelieved pain has adverse physical use of pain medication in children increases the risk of major
and psychological consequences. especially opioids is a common problem. malformations. However, NSAIDs should
Therefore, alternative analgesia routes not be used after 32 weeks’ gestation
• ECP should encourage the
such as nasal medication administration because of the possibility of bleeding
reporting of pain by individuals
are helpful in younger patients. effects. If opioids are used with caution
who are reluctant to discuss pain,
during the pregnancy, the infant should
deny pain when it is likely to Geriatric be observed carefully for any signs of
present, or fail to follow through Being an elderly not only had a higher risk
withdrawal. This is called as neonatal
on prescribed pain relief for inadequate pain assessment but also
abstinence syndrome.
medications. to suffer untoward side effects of the pain
relief medication especially in the Drugs seeking behavior patient
Special considerations demented patient. ECP need to weight Healthcare providers should have a
out the risks and benefits of analgesia sound understanding of the anatomy,
Pediatric
Pain management in the pediatric and should be discussed with patients physiology, and psychology of addictive
population is challenging and they are at and family members. It may be releived behaviors. A focused history and
higher risk for under-recognized and through the use of opioid-sparing examination should concentrate on items
under-treated. Neonates and even analgesic regimens or employment of that can indicate inconsistencies or
premature babies can and do feel pain. specific therapies (e.g., regional nerve f a l s i fic a t i o n s a s s o c i a t e d w i t h
Pain experienced by children is no less blocks for hip fractures). inappropriate drug-seeking behavior. It
and may even be more than that was always difficult as a decision has to
Pregnant patient be made between “losing” to drug
experienced by an adult. Children react to
Poor acute pain management may lead to
and report pain in different ways e.g. seekers and denying analgesia to
chronic pain and is associated with
becomes quiet or withdrawn instead of patients who are genuinely in need. It is
hypertension, anxiety, and depression.
crying. best to give patients the benefit of the
Commonly prescribed pain medications
doubt with due diligence.
The lack of IV access (time-consuming are relatively safe in pregnancy. There is
and painful) and unwarranted fears on the no evidence showing analgesics

805
References and Further Reading, click here

806
Section 3

Paralysing Agents

General Information Indications


by Qais Abuagla Paralysis agents are neuromuscular- Paralysis drugs in the emergency
blocking agents (NMBA). They block department are used in rapid sequence
neuromuscular transmission at the intubation (RSI) to produce paralysis,
motor endplate. which helps in RSI in 2 ways:

Classification 1. paralyze the vocal cords, and permit


1. Non-depolarizing blocking agents intubation of the trachea
(NDBA): They act by competitively
2. relax the skeletal muscle to facilitate
blocking the binding of acetylcholine
intubation
to its receptors, e.g., rocuronium

2. Depolarizing blocking agents (DBA): Succinylcholine


Succinylcholine is the only DBA used
These agents act by depolarizing
in the ED. It is rapidly hydrolyzed by
the motor endplate of the skeletal
plasma pseudocholinesterase into
muscle fiber. This persistent
weak NMBA. Succinylcholine is rapidly
depolarization makes the muscle
active, typically producing intubating
fiber resistant to further stimulation
conditions within 45 seconds of
by acetylcholine, e.g.,
administration by rapid intravenous
succinylcholine. It has 2 phases of
bolus injection.
action – fasciculation and then
desensitization.

807
Contraindications • Malignant Hyperthermia: is a syndrome Category C
• Hyperkalemia characterized by rapid temperature rise
and rhabdomyolysis. Treatment for this Reversal agent
• Preexisting hyperkalemia Sugammadex
consists of cessation of any potential

• Burns >5 days old offending agents and administration of


dantrolene.
• Crush injury >5 days old References and Further Reading, click
Pregnant Patient Considerations here
• Severe infection >5 days old Category C

• Neuromuscular diseases (e.g.,


Rocuronium
Myasthenia Gravis)
Rocuronium is one of the NDBA. It works

• History of Malignant Hyperthermia in less than 1 minute after administration.

• Allergy to succinylcholine Contraindications


• No absolute contraindication to it
Dosing and Administration
• Adult 1.5 mg/kg IV • Anaphylaxis

• Pediatrics 1.5 mg/kg IV Dosing and Administration


• Adult 1 mg/kg
Adverse Effects
• Cardiovascular System: Succinylcholine • Pediatrics 1 mg/kg
can lead to bradycardia, significant in
Adverse Effects
pediatric patients that are 1 year and
• Hypertension in 1-2%
less. For that, some practitioners
recommend atropine prior to • Hypotension in 1-2%
succinylcholine administration, but
there is no evidence supporting that. Pregnant Patient Considerations
808
Chapter 20

Selected Clinical
Rules, Scores,
Mnemonics
Section 1

Clinical Decision Rules

Introduction CDRs have been directly compared


by Stacey Chamberlain Clinical Decision Rules (CDRs), also against clinician gestalt or clinical
known as Decision “Instruments” or practice, and they do not always fare
“Aids,” are evidence-based tools to better. Additionally, some rules actually
assist the practitioner in decision- incorporate clinician gestalt whereas
making for common complaints. In the the rule cannot even be applied unless
Emergency Department (ED) setting, the pre-test probability (based on
these decision aids are often used to physician’s judgment of the likelihood
help identify patients that might be of the disease) is below a pre-
higher risk for serious conditions such determined threshold. Also, for a CDR
as pulmonary embolism (PE) or to be useful to a practitioner, it must be
subarachnoid hemorrhage (SAH), or practical. If a CDR is developed that
they are used to prevent overuse of has too many complicated variables, it
unnecessary testing, which is how is unlikely to be applied in a busy
many of the orthopedic rules are clinical environment.
applied.
Another caveat to the application of
CDRs, despite being called “Rules,” CDRs is that they must be applied
are not meant to replace critical appropriately. CDRs evolve through a
thinking from experienced process of derivation to validation to
practitioners. In fact, many of the impact analysis of the tool. After the

810
tool is derived (level 4 evidence), the tool The practitioner must also understand the judgment as to what you should do if
is validated in a limited patient setting purpose of the CDR and whether it is a the patient is “PERC positive.”
(level 3 evidence), then a broader one-way or two-way rule. As noted by
In addition to CDRs, there are many risk
validation setting (level 2 evidence) and Green, for example, the Ottawa Ankle
stratification tools or scales that are
finally, the impact of the tool is assessed Rules are intended to be a two-way rule;
currently used for serious conditions such
(level 1 evidence). These levels are if the patient meets criteria, you do an X-
as pulmonary embolism (PE) and acute
important to caution the novice learner ray. If they don’t meet criteria, you do not
coronary syndrome (ACS). Others are
against applying every CDR derived and do an X-ray. There are two paths you can
being developed for use in the ED setting
published automatically into their clinical take after you apply your CDR.
for common conditions such as
practice. The tool must be validated in a Alternatively, the pulmonary embolism
congestive heart failure (CHF), chronic
patient population with similar rule-out criteria (PERC) demonstrate a
obstructive pulmonary disease (COPD)
characteristics to the practitioner’s one-way rule. This tool was developed to
and transient ischemic attack (TIA) to
patient population. For example, the tool identify a subset of patients at very low
identify patients at higher risk for acute
may not perform the same (have the risk for PE such that no further testing
severe complications. From a practical
same sensitivity and specificity) if the need be done. If the patient is “PERC
perspective, the ED physician will often
prevalence of disease is different positive,” this should not imply that
use these risk stratification devices to
between the study and actual patient further testing for PE such as a D-dimer
help determine which patients require
populations. Also, the practitioner must should be done. Whether or not
admission. However, these tools are less
be familiar with the inclusion and additional testing should be done remains
prescriptive in that they are not rules that
exclusion criteria for a particular tool. If up to the practitioner and depends on
suggest what a practitioner should or
not, the tool could be misused. For many variables including whether an
should not do; rather, they help the
example, if the tool was derived and alternate diagnosis is much more likely.
physician more objectively look at the risk
validated for a patient population over the PERC was simply designed to help “rule
for an individual patient. Then the
age of 18, it should not be inappropriately out” the diagnosis of PE, not “rule in.”
practitioner must decide what level of risk
applied in a pediatric setting. This rule only guides you down one path,
they are comfortable with in regards to
potentially to do no testing; it makes no
inpatient or outpatient management,

811
which may greatly depend on the use online calculators, additional no point tenderness to the
resources available in those information on inclusion and exclusion
distal posterior malleoli
environments. Most of the risk criteria, and pearls and pitfalls for each
stratification tools encompass multiple tool. bilaterally.
variables with more complicated scoring
Orthopedic CDRs Should you get an X-ray to rule out
s y s t e m s ; a s t h e y a re n o t e a s i l y
fracture?
memorized, most of these would typically
Case 1
be used by ED physicians with real-time
A 28-year-old man presents to Ottawa Ankle Rule
access to a computer or smartphone with • Pain in the malleolar zone and any one
appropriate apps. the ED with left ankle pain after
of the following:
twisting his ankle playing
Given the many pitfalls noted above • Bone tenderness along the  distal 6  cm
regarding CDRs, the goals of using basketball. He is able to bear
of the posterior edge or tip of the tibia
evidence-based medicine to reduce weight and notes pain and (medial malleolus), OR
practice variability, maximize use of
swelling to the lateral aspect of
resources, and help identify and diagnose • Bone tenderness along the distal 6  cm
high-risk conditions are important. It is the ankle (he points to just of the posterior edge or tip of the fibula
equally important that the ED physician below the lateral malleolus). He (lateral malleolus), OR
critically appraise these tools and denies weakness, numbness, • An inability to bear weight both
selectively apply them in appropriate
ways. The remainder of this chapter will
or tingling and has no other immediately after the trauma and in the
ED for four steps.
use case scenarios to review the most injuries. On exam, he is
commonly used CDRs in the ED setting. neurovascularly intact. Edema Ottawa Foot Rule
• Pain in the midfoot zone and any one of
The useful FOAM reference MDCalc.com and tenderness are noted
the following:
provides a summary of the most common slightly anterior and inferior to
tools that are being used with easy-to-
the lateral malleolus. There is
812
• Bone tenderness at the base of the Some of the longest standing and most b e 1 0 0 % s e n s i t i v e . Tw o s t u d i e s
fifth metatarsal, OR widely accepted CDRs are the Ottawa compared the PDR and Ottawa knee
knee, ankle, and foot rules. These rules rules and found the PDR to perform
• Bone tenderness at the navicular bone,
are to help practitioners identify patients better with similar sensitivities but better
OR
with an extremely low risk of fracture specificity for the PDR (51-60% versus
• An inability to bear weight both such that X-rays do not need to be done, 27%). However, one validation study for
immediately after the trauma and in the thus limiting the risks and costs of the PDR found the sensitivity to be as low
ED for four steps. unnecessary testing. The sensitivity of as 77%.  Additionally, while the Ottawa
these rules has been found to be rule has been validated in children as
Ottawa Knee Rule 98.5-100%. In impact study of the young as two years old, the PDR
• Knee injury with any of the following: Ottawa knee rule, application of the rule excludes children younger than 12.
decreased the use of knee radiography
• Age 55 years or older Case 1 Discussion
without patient dissatisfaction or missed
• Tenderness at head of fibula fractures and was associated with In the above case, using either CDR, an
reduced waiting times and costs. These X-ray is unnecessary.
• Isolated tenderness of patella
rules have been validated in pediatric
• Inability to flex to 90° populations as well with similar Trauma CDRs
sensitivities (98.5-100%).
• Inability to bear weight both Case 2
immediately after the trauma and in the A less studied rule for knee trauma to A 57-year-old man fell from a
ED (4 steps) determine the need for radiography is the
height of 12 feet while on a
Pittsburgh Decision Rule (PDR). It differs
Pittsburgh Knee Decision Rule from the Ottawa rule in that it looks at the ladder. He did not pass out; he
• Mechanism: blunt trauma or fall mechanism of injury and applies to a reports that he simply lost his
d i ffe r e n t a g e g r o u p ; a l s o , p o i n t
• Age < 12 or > 50 footing. He fell onto a grassy
tenderness is not used in the PDR. Its
• Unable to bear weight 4 steps in the ED original derivation study found this rule to
area, hitting his head and

813
complains of neck pain. He did Canadian C-spine Rule tenderness and additional factors
• Age ≥ 65 that might limit a practitioner’s exam. The
not lose consciousness and
C C R c a n b e d i ffic u l t f o r s o m e
denied headache, blurry vision, • Extremity paresthesias
practitioners to remember all the criteria
vomiting, weakness, numbness • Dangerous mechanism (fall from ≥ 3ft / that qualify as a dangerous mechanism
and is limited to ages > 16 and < 65.
or tingling in any extremities. 5 stairs, axial load injury, high-
speed  MVC/rollover/ejection, bicycle However, it can be used in intoxicated
He denies other injuries. He patients if the patients are alert and
collision, motorized recreational vehicle)
was able to get up and cooperative, allowing a full neurologic

ambulate after the fall and NEXUS Criteria for C-spine exam. The NEXUS Criteria are applicable
Imaging over any age range (> 1 year old), but the
came in by private vehicle. He • Focal neurologic deficit present sensitivity may be low in patients > 65
has not had previous spine years of age. A single comparison study
• Midline spinal tenderness present
surgery and does not have found the CCR to have better sensitivity
• Altered level of consciousness present (99.4% versus 90.7%); however, the
known vertebral disease. On
study was performed by hospitals
exam, he is neurologically • Intoxication present
involved in the initial CCR validation
intact with a GCS of 15, does • Distracting injury present study.

not appear intoxicated and has Both the Canadian C-spine Rule (CCR) Case 2 Discussion
moderate midline cervical and NEXUS Criteria are widely employed
By applying either criteria to this case,
spine tenderness. in clinical practice to reduce unnecessary
the patient would require C-spine imaging
cervical spine imaging in trauma patients
as by CCR, the patient would meet
Should you get imaging to rule out a with neck pain or obtunded trauma
criteria for dangerous mechanism, and by
cervical spine fracture? patients. The CCR uses mechanism and
NEXUS, the patient has midline
age criteria, whereas the NEXUS Criteria
tenderness to palpation.
incorporates criteria including midline
814
Case 3 Should you get a CT head for this The Canadian CT Head Rule (CCHR) only
patient to rule out a clinically applies to patients with an initial GCS of
A 36-year-old woman slipped
significant brain injury? 13-15, witnessed loss of consciousness
on ice and fell and hit her head. (LOC), amnesia to the head injury event,
She reports loss of Canadian CT Head Rule or confusion. The study was only for
consciousness for a minute • High-Risk Criteria (rules out the need patients > 16 years of age. Patients were
for neurosurgical intervention) excluded from the study if they had
after the event, witnessed by a
“minor head injuries” that didn’t even
bystander. She denies • GCS < 15 at two hours post-injury
meet these criteria. Patients were also
headache. She denies • Suspected open or depressed excluded if they had signs or symptoms
skull fracture of moderate or severe head injury
weakness, numbness or
including GCS < 13, post-traumatic
tingling in her extremities and • Any sign of basilar skull fracture seizure, focal neurologic deficits, or
(hemotympanum, Raccoon eyes,
no changes in vision or speech. coagulopathy. Other studies have looked
B a t t l e ’s s i g n , C S F o t o o r
at different CDRs for traumatic brain
She has not vomited. She rhinorrhea)
injury including the New Orleans Criteria
remembers the event except (NOC). However, CCHR has been found
• Medium Risk Criteria (rules out clinically
for the transient loss of important brain injury) to have superior sensitivity and
consciousness. She doesn’t specificity.
• Retrograde amnesia to event  ≥ 30
use any blood thinners. On minutes Case 3 Discussion
physical exam, she has a GCS By applying this rule to the above case,
• Dangerous mechanism
of 15, no palpable skull fracture (pedestrian struck by motor the patient should be considered for
and no signs of a basilar skull vehicle, ejection from the motor imaging due to the mechanism. A fall
vehicle, fall from > 3 feet or > 5 from standing for an adult patient would
fracture.
stairs) constitute a fall from > 3 feet; therefore,

815
although the patient would not likely be hematoma measuring The PECARN (Pediatric Emergency Care
high risk and need neurosurgical Applied Research Network) Pediatric
approximately 4×4 cm.
intervention, the patient might have a Head Trauma Algorithm was developed
positive finding on CT that in many Should you get CT imaging of this child as a CDR to minimize unnecessary
practice settings would warrant an to rule out clinically significant head radiation exposure to young children. The
observation admission. estimated risk of lethal malignancy from a
injury?
single head CT in a 1-year-old is 1 in
Case 4 PECARN Pediatric Head Trauma 1000-1500 and decreases to 1 in 5000 in
A 20-month-old female was Algorithm a 10-year-old. Due to these risks, in
going up some wooden stairs, • Age < 2 addition to costs, length of stay and
potential risks of procedural sedation, this
slipped, fell down four stairs, • GCS < 15, palpable skull fracture,
CDR is widely employed given the
and hit the back of her head on or signs of altered mental status
frequency of pediatric head trauma ED
the wooden landing at the • Occipital, parietal or temporal visits. This CDR has the practitioner use a
prediction tree to determine risk, but
bottom of the stairs. She did scalp hematoma; History
of LOC≥5 sec; Not acting normally unlike some other risk stratification tools,
not lose consciousness and the PECARN group does make
per parent or Severe Mechanism
cried immediately. She was of Injury? recommendations based on what they
consolable after a couple of consider acceptable levels of risk. In the
• Age ≥ 2 less than 2-year-old group, the rule was
minutes and is acting normal
found to be 100% sensitive with
• GCS < 15, palpable skull fracture,
per her parents. She has not sensitivities ranging from 96.8%-100%
or signs of altered mental status
vomited. On exam, she is well- sensitive in the greater than two-year-old
• History of  LOC  or history of group.
appearing, alert, and has a
vomiting or Severe headache or
normal neurologic exam. She is Severe Mechanism of Injury?
This algorithm does have some
complexity and ambiguity. It requires the
noted to have a left parietal
816
practitioner to know what were However, a sub-analysis of patients less •Thoracic wall trauma, complaints of
considered signs of altered mental status than two years old with isolated scalp abdominal pain, decreased breath
and what were considered severe hematomas suggests that patients were sounds, vomiting
mechanisms of injury. In addition, certain higher risk if they were < 3 months of age,
• 0.7% risk of intra-abdominal injury
paths of the decision tree lead to had non-frontal scalp hematomas, large
intervention
intermediate risk zones. In these cases, scalp hematomas (> 3cm), and severe
the recommendation is “observation mechanism of injury. Given the large A CDR for pediatric blunt abdominal
versus CT,” allowing for the ED physician hematoma in the case study patient and trauma has been derived by the PECARN
to base his/her decision to image or not a severe mechanism of injury (a fall of > 3 group but not yet validated. This CDR
based on numerous contributory factors feet in the under two age group), one uses a seven-point decision rule. If the
including physician experience, multiple might more strongly consider imaging patient does not have any of these
versus isolated findings, and parental due to these two additional higher risk findings, the patient would be considered
preference, among others. factors. “very low risk” with a 0.1% risk of intra-
abdominal injury intervention required. A
Other pediatric head trauma CDRs rules PECARN Abdominal Trauma
study did compare the PECARN CDR
have been derived and validated; • Evidence of abdominal wall trauma/
versus clinical suspicion and found that
however, in comparison trials, PECARN seatbelt sign or GCS < 14 with blunt
the CDR had significantly higher
performed better than the other CDRs.1 abdominal trauma (if no, go to next
sensitivity (97.0% vs. 82.8%) but lower
Of note, in this study, physician practice point)
specificity (42.5% vs. 78.7%). However,
(without the use of a specific CDR)
• 5.4% risk of needing intra- abdominal CTs were done in 33% of
performed as well as PECARN with only
abdominal injury intervention patients with clinical suspicion < 1%,
slightly lower specificity.
meaning that even though clinical
• Abdominal tenderness (if no, go to next
Case 4 Discussion suspicion had higher specificity, this often
point)
did not translate into clinical practice.
For purposes of the case study, the
• 1.4 % risk of intra-abdominal Validation of the PECARN rule has the
patient falls into an intermediate risk zone
injury intervention potential to therefore improve both
of clinically important brain injury.
817
sensitivity and specificity compared to anything for the headache. She A CDR to determine risk for sub-
physician practice, but this remains to be arachnoid hemorrhage (SAH) was derived
does not have a family history
seen. and has been externally validated in a
of cerebral aneurysms or single study. The CDR’s purpose was to
Additional CDRs polycystic kidney disease. On identify those at high risk for SAH and
included those with acute non-traumatic
Case 5 physical exam, she has a
headaches that reached maximal
A 24-year-old woman presents normal neurologic exam and intensity within one hour and who had
with headache that began normal neck flexion. normal neurologic exams. Of note, the
three hours prior to arrival to rule has many inclusion and exclusion
Should you do a head CT and/or a criteria that the ED physician must be
the ED. The patient was at rest lumbar puncture to evaluate for a sub- familiar with and was only derived for
when the headache began. The arachnoid hemorrhage in this patient? patients 16 years or older. The study
headache was not described authors note that the CDR is to identify
Ottawa SAH Rule
as “thunderclap,” but it did patients with SAH; it is not an acute
• Investigate if ≥1 high-risk variables
headache rule. In the validation study, of
reach maximum severity within present:
over 5,000 ED visits with acute headache,
the first 30 minutes. The • Age ≥ 40 only 9% of those met inclusion criteria.
headache is generalized and Also, clinical gestalt again plays a role as
• Neck pain or stiffness the authors suggest not to apply the CDR
rated 10/10. She denies head
• Witnessed loss of consciousness to those who are ultra-high risk with a
trauma, weakness, numbness, pre-test probability for SAH of > 50%.
and tingling in her extremities. • Onset during exertion
The Ottawa SAH Rule was 100%
She denies visual changes, • Thunderclap headache (instantly sensitive but did not lead to reduction of
changes in speech and neck peaking pain) testing vs. current practice. The authors
state that the value of the Ottawa SAH
pain. She has not taken • Limited neck flexion on exam
818
Rule would be to standardize physician surgery. She has no anterior Pulmonary Embolism Rule-
practice in order to avoid the relatively Out Criteria (PERC)
abdominal pain, no dysuria or
high rate of missed sub-arachnoid • Age ≥ 50
hemorrhages. hematuria and no personal or
• Heart rate ≥ 100
family history of gallstones,
Case 5 Discussion
kidney stones, or blood clots. • O2 sat on room air < 95%
By applying the Ottawa SAH Rule, this
She’s never had this pain • Prior history of venous
patient is low risk and does not require
before, has no significant past thromboembolism
further investigation for a SAH.
medical history and her only • Trauma or surgery within 4 weeks
Case 6
medication is birth control pills.
A 19-year-old female presents • Hemoptysis
On exam, her vital signs are
with sharp right flank pain and • Exogenous estrogen
within normal range, she has
shortness of breath that started
normal cardiac and pulmonary • Unilateral leg swelling
suddenly the day prior to
exams, no costovertebral angle The PERC CDR was originally derived
arrival. The pain is worse with
and validated in 2004 and with a
tenderness, no chest wall or
deep inspiration but not related subsequent multi-study center validation
abdominal tenderness and no
to exertion and not relieved in 2008. In the larger validation study, the
leg swelling. rule was only to be applied in those
with ibuprofen. She denies
patients with a pre-test probability of <
anterior chest pain, cough, and Do you need to do any studies to
15%, therefore incorporating clinical
fever. She denies leg pain or evaluate this patient for a pulmonary gestalt prior to using the rule. PERC is a
embolism? one-way rule, as mentioned above, which
swelling and recent travel,
tried to identify patients who are so low-
immobilization, trauma, or risk for pulmonary embolism (PE) as to

819
not require any testing. It does not imply Risk Stratification Tools are normal, and there is no
that testing should be done for patients
Case 7 calf tenderness or swelling.
who do not meet criteria, and it is not
meant for risk stratification, as opposed A 68-year-old male presents How should you proceed with this
to the Wells’ and Geneva scores. with acute onset of shortness patient’s work up for PE?

Case 6 Discussion of breath and right-sided sharp


chest pain, worse with deep Table 20.1 Wells’ Criteria for
In order to apply the PERC CDR to the Pulmonary Embolism
case study patient, the ED physician pre- breathing. He denies chest POINT
CRITERIA VALUE
supposes a pre-test probability of < 15%. pain with exertion, no cough,
If the ED physician has a higher pre-test Clinical signs and symptoms of +3
fever or hemoptysis, no leg DVT
probability than that, he/she should not
use the PERC CDR. If the ED physician, pain or swelling, no recent PE is #1 diagnosis, or equally +3
likely
in this case, did indeed have a pre-test travel, surgery or Heart rate > 100 +1.5
probability of < 15%, the case study
immobilization. He has a Immobilization at least 3 days, +1.5
patient would fail the rule-out due to her or Surgery in the Previous 4
use of oral contraceptives. In that case,
history of prostate cancer and weeks

the ED physician would need to completed his treatment with Previous, objectively diagnosed
PE or DVT
+1.5

determine if he/she would do further radiation therapy four months Hemoptysis +1


testing which could include a D-dimer, CT
ago. On exam, his heart rate is Malignancy w/ Treatment within +1
chest with contrast, ventilation/perfusion 6 mo, or palliative
scan, or lower extremity Doppler studies 90, O2 saturation is 98% with a
to evaluate for deep vein thromboses normal respiratory rate, blood
(DVTs). The PERC CDR gives no
pressure, and temperature. His
guidance in this case.
cardiac and pulmonary exams

820
The Wells’ Criteria for PE is a risk o ff of less than 6 for low risk was studied
Table 20.2 Geneva Score (Revised) for
Pulmonary Embolism stratification score with different point in pregnant patients with a negative
values assigned to different criterion. Its predictive value of 100%.
CRITERIA POINT
CATEGORY VALUE purpose is to identify patients who have a
The original Geneva score included the
Risk factors Age > 65 +1 lower risk for PE in order to potentially
use of chest radiography and an ABG,
Previous DVT or avoid unnecessary testing and the risks
PE
+3 whereas the revised score (rGeneva) uses
and costs associated with it. The criteria
only clinical criteria. A patient with a
Surgery (under have been validated in the ED setting. In
general anesthesia) rGeneva score of 0-3 is considered the
or lower limb +2 the initial three-tier model, a patient with
fracture in past 1 low risk with a < 10% prevalence of PE. A
0-1 points was considered to be in a low-
month score of 4-10 identifies intermediate-risk
risk group (1.3% prevalence of PE in an
Active malignant
+2 patients, and a score of 11+ is high risk
condition ED population) versus patients with a
(>60% prevalence or PE).
Symptoms Unilateral lower moderate score of 1-6 ( 16.2%
+3
limb pain prevalence), and those with a high score The Wells and rGeneva scores have been
Hemoptysis +2 of >6 ( 37.5% prevalence). Subsequent compared and found to have overall
Signs Heart rate < 75 0 studies have been done to apply a s i m i l a r a c c u r a c y. T h e s e P E r i s k
Heart rate 75 - 94 +3 simplified version of the Wells’ Criteria stratification tools are meant to be
Heart rate ≥ 95
and also to use the Wells’ Criteria along applied in those patients with concern for
+5
with D-dimer testing in a dichotomous PE as a diagnosis. If PE is not under
Pain on lower limb
deep venous manner (two-tier model) where a score of consideration, the tools should not be
+4
palpation and
4 or less (“PE Unlikely” group) combined applied. Practically speaking, for many
unilateral edema
with a negative D-dimer would achieve ED physicians, these tools are used to
sufficiently low probability of PE so as not help risk stratify patients to identify those
to pursue further workup. This two-tier who are very low-risk such that no testing
model is supported by the American should be done, low to intermediate risk
College of Physicians (ACEP) Clinical such that D-dimer testing would be a
Guidelines. A two-tier model using a cut-
821
useful diagnostic tool, or high risk such a “solid or hematologic malignant other cardiac risk factors.
that even if a D-dimer were negative, the condition, currently active or considered
His exam in the ED is normal,
post-test probability would remain high cured < 1 year.” Using the dichotomous
enough that further testing should be Wells’ approach, the patient would be and his EKG and initial troponin
pursued. One recent study found that considered “PE Unlikely;” using the are normal.
physician gestalt actually performed rGeneva, the patient would be
better than either the Wells or rGeneva intermediate risk. The ACP Guidelines Does this patient require additional
scores. However, guidelines from the would suggest that a D-dimer should be cardiac workup in the ED or admission
Clinical Practice Committee of the done in this patient, adjusted for age, to to hospital for additional workup? Can
American College of Physicians (ACP) determine the need for possible imaging this patient be safely discharged for
were published in 2015 that outline best to evaluate for PE. outpatient follow-up?
practice advice including advocating that
clinicians should use validated CPRs to Case 8 The HEART Score is used to risk stratify

estimate pre-test probability in patients in A 50-year-old male presents to chest pain patients in the ED to identify
those at risk for major adverse cardiac
whom acute PE is being considered. the ED complaining of chest
events (MACE) within six weeks. With the
Case 7 Discussion pain for two days. His pain is HEART Score, low-risk patients have a
substernal, non-radiating, score of 0-3 and have a less than 2% risk
This patient’s Wells’ score is 4. Although
of MACE at 6 weeks. The HEART Score
subject to ED physician judgment, PE described as a tightness, not
d i ffe r s f ro m t h e T h ro m b o l y s i s i n
could be considered at least equally as related to exertion. He has no Myocardial Infarction (TIMI), and Global
likely as any other diagnosis given the
associated shortness of breath, Registry of Acute Coronary Events
absence of other findings to explain his
nausea or diaphoresis. No (GRACE) scores as those scores measure
shortness of breath (no crackles or
the risk of death for patients with
wheezing on exam, no cough or fever). cough or fever. He’s never had
diagnosed acute coronary syndromes
The patient’s rGeneva score is 6. An this pain before. He has a (ACS) rather than identifying patients who
“active malignant condition” is defined as
history of hypertension but no have cardiac-related chest pain in the first

822
place. Additionally, even with low TIMI ED physician who finds this risk level Case 8 Discussion
scores for those diagnosed with ACS in unacceptable.
This patient’s HEART Score is 3 if the
the ED, there is still a 4.7% risk of a bad
physician considers the history
outcome. This may be of little utility to the
“moderately suspicious.” The patient is at
Table 20.3 HEART Score for Cardiac Events low risk for a major cardiac event in the
next six weeks so that the ED physician
CATEGORY CRITERIA POINT VALUE
could consider outpatient follow-up.
History Highly suspicious +2
Again, however, the risk stratification
Moderately suspicious +1 scores are not prescriptive, however.
Slightly suspicious 0 Decision-making must be done by the
EKG Significant ST depression +2 clinician based on his/her judgment,
Non specific repolarization resources available, and comfort with
+1
disturbance certain levels of risk.
Normal 0

Age ≥ 65 +2 Case 9
45-65 +1
A four-year-old boy presents to
≤ 45 0 the ED with a complaint of sore
Risk Factors (include: hypercholesterolemia, ≥ 3 risk factors or history of throat for one day associated
hypertension, diabetes mellitus, cigarette smoking, atherosclerotic disease +2
positive family history, obesity) with cough and fever. On
1-2 risk factors +1 exam, he is febrile to 38.5
No risk factors known 0 degrees Celsius, has bilateral
≥ 3× normal limit
Troponin +2
tonsillar exudates, and anterior
1-3× normal limit +1
cervical lymphadenopathy.
≤ normal limit 0

823
How should you proceed Antibiotics have been shown to
Table 20.4 Centor Score (Modified) for
Streptococcal Pharyngitis reduce suppurative (peritonsillar abscess,
with the workup for this child cervical lymphadenitis, and mastoiditis)
POINT
for possible strep? CATEGORY CRITERIA
VALUE and non-suppurative (e.g., acute
Age 3-14 rheumatic fever) complications of strep
+1
Options include treating pharyngitis and shorten the duration of
15-44
empirically, doing a rapid point- 0 clinical symptoms as well as reducing
transmission. Rapid antigen detection
of-care strep test, sending a 45 or
-1
older tests have been found to have a
throat culture, or supportive sensitivity between 70 and 90% and a
Exudate or swelling
+1
treatment. on tonsils specificity of ≥95%. Some authors
Tender/swollen recommend rapid antigen detection
The Centor Score is a risk stratification anterior cervical +1 testing (RADT) only for children with high
tool to look at clinical criteria that suggest lymph nodes
clinical scores (using Centor or other
a greater likelihood of strep pharyngitis Fever (T > 38°C, published clinical criteria) or if the results
+1
100.4°F)
that may prompt the ED physician to of the standard throat culture will not be
prescribe antibiotics. It was originally Present
Cough 0 available for more than 48 hours.
designed for use in adults, but a modified Additionally, the presence of particular
Absent
score has been validated for use in +1
clinical criteria may impact the ED
children > 2 years of age and adults that physician’s decision to test and/or treat.
includes age criteria as strep pharyngitis of strep is greater than 50%, and some Studies looking at different clinical
is a more common condition in children. would advocate for empiric antibiotics in prediction scores (including Centor) found
In the absence of any of the criteria at t h i s g r o u p . H o w e v e r, a s a r i s k that the presence of tonsillar
any age group, the risk of strep is less stratification tool, ED physicians can exudates  conferred the highest odds of
than 10% (< 2.5% if 15 or older) and adjust their practice according to their having streptococcus infection.
further testing is not necessary. With a interpretation of the risks.
score of 4 or more points, the probability Case 9 Discussion

824
The patient has a Centor Score of 4. diarrhea, fever, or syncope. He Table 20.5 Glasgow-Blatchford Risk
Some clinicians would use this high-risk Score
denies a history of liver or heart
clinical score to justify further testing with
CATEGORY SCORE
an RADT or a throat culture. Others would problems. On exam, he has
BUN in mg/dL
treat empirically, especially given the normal vital signs with an initial
presence of exudates which has a higher 18.2 to 22.4 2
blood pressure of 128/78 in the
specificity than some of the other clinical 22.5 to 28 3

findings. This decision may be based on ED, and his abdomen is non- 28.1 to 70 4

additional factors such availability and tender. His hemoglobin is 13.5, 70.1 or greater 6
processing times of diagnostic testing and BUN is 5. Hemoglobin, men g/dL
and ease of patient follow-up. 12 to 13 1
Does this patient need admission for 10 to 11.9 3
Case 10 further monitoring or evaluation of his 9.9 or less 6
A 30-year-old male presents to upper GI bleed? Hemoglobin, women g/dL
the ED with nausea, vomiting,
Glasgow-Blatchford Risk Score is useful 10 to 12 1
and epigastric discomfort for for predictive of inpatient mortality, blood 9.9 or less 6

one day. He vomited multiple transfusions, re-bleeding, ICU monitoring, Systolic Blood Pressure, mmHg

times, initially non-bloody, then and hospital length of stay. Patients with 100-109 1
a score of zero may be discharged home, 90-99 2
developed some blood in the those with score 2 or higher are usually <90 3
vomit during the last two admitted, and those with score of 10 or
Heartrate >100 peats per minute 1
episodes, which he quantified more are at highest risk for morbidity and
Melena 1
resource utilization. Maximum score is
as a teaspoon in each. He Syncope 2
23.
denies melena or Hepatic Diseases 2
  Heart failure 2
hematochezia. He has no
825
Glasgow-Blatchford Risk Score is useful Case 10 Discussion BP is 86/48 which improves
for predictive of inpatient mortality, blood
The patient does not meet any of the to 98/50 with 1L IVF. Her
transfusions, re-bleeding, ICU monitoring,
and hospital length of stay. Patients with
criteria in the GBS and would be hematocrit is 31%, and her
considered low risk. The patient does not
a score of zero may be discharged home, EKG and telemetry monitoring
demonstrate any signs of lower GI
those with score 2 or higher are usually
bleeding and could likely be safely in the ED are normal.
admitted, and those with score of 10 or
discharged home based on this risk
more are at highest risk for morbidity and Is this patient low risk for safe
stratification.
resource utilization. Maximum score is discharge home?
23.
Case 11
San Francisco Syncope Rule
The Glasgow-Blatchford Bleeding Score A 45-year-old woman presents • Congestive heart failure history
(GBS) uses clinical information as well as with syncope immediately prior
some diagnostic testing to risk stratify • Hematocrit < 30%
to arrival. She was feeling
upper GI bleeding patients. It should not
• EKG abnormal (new EKG change from
be used for lower GI bleeding patients or generalized fatigue prior to the
any source, any non-sinus rhythm on
patients in whom the source of GI syncopal episode. She denies
bleeding is unclear. A score of 0 is • EKG or monitoring)
chest pain, palpitations, or
considered low risk. Any score higher
than 0 is high risk for needing a medical shortness of breath. She has • Shortness of breath symptoms

intervention of transfusion, endoscopy, or not had vomiting or diarrhea. • Systolic BP < 90 mmHg at triage
surgery; therefore, the presence of any of She has been taking PO today
the above criteria would be considered The San Francisco Syncope Rule was
high risk. The tool assigns different point
but has a decreased appetite. derived in 2004. In its initial derivation
values to different gradations of the She has no known medical and validation studies, it was found to
h a v e 9 2 % a n d 9 8 % s e n s i t i v i t y,
variables present to a possible highest problems. On exam, her initial
possible score of 29. respectively. Its use has become

826
c o n t r o v e r s i a l , h o w e v e r, d u e t o Case 11 Discussion certainly have the potential to be
inconsistent validation studies where it useful adjuncts for the management of
Although the San Francisco Syncope
has not performed as well. A systematic ED patients with these common
Rule has failed to be consistently
review of the literature from 2011 conditions.
validated for use in identifying all high-
suggested that “the probability of a
risk patients, this patient fails the rule due
serious outcome given a negative score
to her initial SBP being less than 90.
with the San Francisco Syncope Rule References and Further Reading, click
Therefore, this patient would, in any case,
was 5% or lower, and the probability was here
not be considered low-risk, and the ED
2% or lower when the rule was applied
physician might consider additional
only to patients for whom no cause of
monitoring and/or evaluation.
syncope was identified after initial
evaluation in the emergency department.” Ottawa Heart Failure Risk Score,
However, a meta-analysis from 2013
COPD Risk Scale and Canadian TIA
suggests that it only had 87% sensitivity
Risk Score
for serious outcomes according to pooled
results and that there was a broad range These risk stratification tools are
of false-negative rates among the mentioned as they have all completed
included studies (range 0% to 48%). derivation studies and are in various
Although there is clearly no consensus on stages of validation studies. The intent of
use of this tool to safely discharge these tools is to help the clinician develop
patients with syncope home, if they do risk estimates of short-term serious
not meet these criteria, patients who do adverse events in ED patients. Although
have criteria would be considered higher not yet ready for widespread usage, the
risk, possibly warranting observation, ED physician should be aware of these.
admission and/or further diagnostic Additional studies need to be completed
studies. and published to determine the validity
and impact of these scores, but they
827
Section 2

Mnemonics

3 D’s: Beck’s triad (cardiac ABCDE: Supraventricular


by Ozlem Dikme tamponade) tachycardia (treatment)
D = Distant heart sounds
 A = Adenosine

D = Distended jugular veins
 B = Beta-blocker

D = Decreased arterial pressure C = Calcium channel antagonist

D = Digoxin

4 P’s: Arterial occlusion E = Excitation (vagal stimulation)
P = Pain

P = Pallor
 ABCDEFGHIJK:
P = Pulselessness
 Haematuria differential in
P = Paresthesias children
A = Anatomy (cysts, etc)

ABC/2: Volume of B = Bladder (cystitis)

intracranial bleed of CT C = Cancer (Wilm’s tumour)

A = Maximal diameter of the
D = Drug related (cyclophosphamide)

hematoma by CT

E = Exercise induced

B = Diameter 90° to A, and

F = Factitious (Munchausen by proxy)

C = Approximate number of CT slices
G = Glomerulonephritis

with hemorrhage multiplied by the slice
H = Haematology (bleeding disorder,
thickness

sickle cell)

Vol > 20 – 30 ml consider surgery
I = Infection (UTI)


828
J = inJury (trauma)
 APGAR: System to evaluate A = Aggravating and alleviating factors

K = Kidney stones (hypercalciuria) newborn’s condition S = Severity

A = Appearance (color)
 C = Character, quality

ABC HELP: Causes of ST P = Pulse (heart rate)
 L = Location

Elevation in ECG G = Grimmace (reflex, irritability)
 A = Associated

A = AMI
 S = Setting

A = Activity (muscle tone)

B = Brugada
 T = Timing
R = Respiratory effort
C = CNS Pathologies

H = Hypertrophy (LVH)
 APPENDICITIS: RLQ pain ASTHMA: Common
E = benign Early repolarization
 differential Medications used to treat
L = LBBB
 A = Appendicitis/ Abscess
 Asthma
P = Pericarditis P = PID/ Period
 A = Albuterol

P = Pancreatitis
 S = Steroid

AEIOU TIPS: Causes of E = Ectopic/ Endometriosis
 T = Theophylline

altered mental status N = Neoplasia
 H = Humidified Oxygen

A = Alcohol
 M = Magnesium (MgSO4)

D = Diverticulitis

E = Epilepsy, electrolytes
 A = Antileukotrienes
I = Intussusception

I = Infection

C = Crohns Disease/ Cyst (ovarian)

O = Overdose

I = IBD
 BATS: Subarachnoid
U = Urea

T = Torsion (ovary)
 hemorrhage causes
T = Trauma
 B = Berry aneurysm

I = Irritable Bowel

I = Insulin
 A = Arteriovenous malformation / Adult
S = Syndrome Stones
P = Psychiatric
 polycystic kidney disease

S = Sepsis, shock ASCLAST: Eliciting history of T = Trauma (e.g., being struck with
present illness and exploring baseball bat)

 
symptoms S = Stroke

829
BE FEVEER: Duke’s Criteria A = Anticoagulate
 L = Lactic acidosis


for Bacterial Endocarditis R = Reduce clot size E = Ethylene glycol



S = Salicylates poisoning
(BE)
CARDIAC RIND: Pericarditis
Major Criteria

causes CHADS2: Risk factors for
B = persistent Blood culture positive >2
times 12 hr part (each C&S taking should
C = Collagen vascular disease
 developing stroke in
be one hour apart, and 3 samples should
A = Aortic aneurysm
 patients with nonrheumatic
be taken)

R = Radiation
 atrial fibrillation
D = Drugs (such as hydralazine)
 C = Congestive Heart Failure

E = Endocardial involvement from Echo

I = Infections
 H = Hypertension or treated hypertension

Minor criteria

A = Acute renal failure
 A = Age 75 years or older

F = Fever >/= 38C

C = Cardiac infarction
 D = Diabetes Mellitus

E = Echo findings not fulfilling a major

R = Rheumatic fever
 S = Stroke or TIA previously
V = Vascular (vasculitis) – Janeway
I = Injury

lesions, mycotic aneurysm, etc

N = Neoplasms
 CHESS: San Francisco
EE = Evidences from microbiological/
D = Dressler’s syndrome Syncope Rule
immunology (2)
 C = history of Congestive heart failure,

R = Risk factors/predisposing factors – CAT MUD PILES: Causes of H = Hematocrit <30%,

drug abuse, valvular diseases High AG Metabolic Acidosis E = Electrocardiogram abnormality,

(predisposing factors) C = Carbon monoxide, Cyanide
 S = Shortness of breath, or

A = Alcoholic ketoacidosis
 S = Systolic blood pressure <90 mm Hg
BOOMAR: MI basic T = Toluene

management M = Methanol
 CLADE SPADE: Fall potential
B = Bed rest

U = Uremia
 causes
O = Oxygen
 C = Cardiovascular/ Cerebrovascular

D = Diabetic ketoacidosis

O = Opiate
 L = Locomotor (skeletal, muscular,
P = Paraldehyde, Phenformin

M = Monitor
 neurological)

I = Iron, Isoniazid


830
A = Ageing (increased body sway, Infarction
 haemorrhage)

decreased reaction time)
 Ignorance (poor control)
 D = Dilated cardiomyopathy

D = Drugs (esp. antihypertensives, Intoxication (alcohol) S = Shock

antipsychotics)
 T = Toxicity of digitalis, quinidine
E = Environmental
 DEMENTIA: Dementia, some
S = Sensory deficits (eg. visual problems)
 common causes DOPE: Acute Deterioration
P = Psychological/ Psychiatric D = Diabetes
 in Intubated Patient
(depression)
 E = Ethanol
 D = Displacement of the tube

A = Acute illness
 M = Medication
 O = Obstruction of the tube

D = Dementia
 E = Environmental (e.g., CO poisoning)
 P = Patient (this is the first priority, not the
E = Epilepsy N = Nutritional
 machines), Pneumothorax

T = Trauma
 E = Equipment failure
DCAP – BTLS: Things to look I = Infection

for in head-to-toe survey for A = Alzheimer’s DOTS: Signs to suspect
trauma fracture
D = Deformity
 DEPRESSED ST: Depressed D = Deformity


C = Contusions
 ST-segment (causes) O = Open wound


A = Abrasions
 D = Drooping valve (MVP)
 T = Tenderness


P = Punctures/penetrations
 E = Enlargement of LV with strain
 S = Swelling

B = Burns
 P = Potassium loss (hypokalemia)



R = Reciprocal ST- depression (in I/W ELEVATION: ST elevation
T = Tenderness

L = Lacerations
 AMI)
 causes in ECG
E = Embolism in lungs (pulmonary E = Electrolytes

S = Swelling
embolism)
 L = LBBB

E = Early repolarization

DKA precipitants (5 I’s) S = Subendocardial ischemia

Infection
 S = Subendocardial infarct
 V = Ventricular hypertrophy

Ischaemia (cardiac, mesenteric)
 E = Encephalon haemorrhage (intracranial A = Aneurysm


831
T = Treatment (e.g., pericardiocentesis)
 CING-KUF: Diabetic HEADS: Stroke risk
I = Injury (AMI, contusion)
 ketoacidosis management factors
O = Osborne waves (hypothermia)
 C = Creatinine (check it)/ Catheterize
 H = Hypertension/ Hyperlipidemia

N = Non-occlusive vasospasm I = Insulin (5u/hour. Note: sliding scale no E = Elderly

longer recommended in the UK)
 A = Atrial fibrillation

FAILURE: CHF causes of N = Nasogastic tube (if patient comatose)
 D = Diabetes mellitus/ Drugs (cocaine)

exacerbation G = Glucose (once serum levels drop to S = Smoking/ Sex (male)
F = Forgot medication

12)

A = Arrhythmia/ Anaemia
 HEAD HEART VESSELS:
K = K+ (potassium)

I = Ischemia/ Infarction/ Infection
 Syncope causes, by system
U = Urea (check it)

L = Lifestyle: taken too much salt
 CNS causes include HEAD:
F = Fluids (crytalloids)
U = Upregulation of CO: pregnancy,
hyperthyroidism
 H = Hypoxia/ Hypoglycemia

GET SMASHED: Causes of
R = Renal failure
 E = Epilepsy

acute pancreatitis
E = Embolism: pulmonary A = Anxiety

G = Gallstones

D = Dysfunctional brain stem
E = Ethanol

FAST HUG: Interventions for T = Trauma

(basivertebral TIA)
critically ill patients in ED S = Steroids

F = Fluid Resuscitation and balance
 Cardiac causes are HEART:
M = Mumps

A = Analgesia
 H = Heart attack

A = Autoimmune (PAN)

S = Sedation
 E = Embolism (PE)

S = Scorpion bites

T = Thromboembolic prophylaxis
 A = Aortic obstruction (IHSS, AS or
H = Hyperlipidemia

H = Head-of-bed elevation
 myxoma)

E = ERCP

U = stress Ulcer prophylaxis, and
 R = Rhythm disturbance

D = Drugs (azathioprine, diuretics)
G = Glucose/glycemic control T = ventricular Tachycardia
 
 
Vascular causes are VESSELS:

832
V = Vasovagal
 R = Renal tubular acidosis
 N = Nitrates

E = Ectopic (reminds one of hypovolemia)
 D = Diarrhea
 S = Stool Softeners
S = Situational
 U = Ureterosigmoidostomy

S = Subclavian steal
 P = Pancreatic fistulas and drainage
 KUSSMAL: Causes of High
E = ENT (glossopharyngeal neuralgia)
 S = Saline (in large amounts) AG Metabolic Acidosis
L = Low systemic vascular resistance (hyperchloremic metabolic acidosis) K = dKA

(Addison’s, diabetic vascular neuropathy)
 U = Uremia

S = Sensitive carotid sinüs HOLT: Jugular venous S = Salicylates poisoning

pressure elevation causes S = Sepsis

HEPATICS: Hepatic H = Heart failure
 M = Methanol poisoning

encephalopathy, O = Obstruction of venea cava
 A = Alcoholic ketoacidosis

precipitating factors L = Lymphatic enlargement – L = Lactic acidosis
H = Hemorrhage in GIT/ Hyperkalemia
 supraclavicular

E = Excess protein in diet
 T = intra-Thoracic pressure increase LEMON: Difficult
P = Paracentesis
 laryngoscopy
A = Acidosis/ Anemia
 INFARCTIONS: Myocardial L = Look externally, e.g. short neck, large

T = Trauma
 infarction treatment tongue, large teeth, etc


I = Infection
 I = IV access
 E = Evaluate 3-3-2


C = Colon surgery
 N = Narcotic analgesics (e.g., morphine, – 3 = adequacy of oral access


S = Sedatives pethidine)
 – 3 = to assess capacity of mandibular


F = Facilities for defibrillation (DF)
 space to accommodate tongue

HARD-UPS: Causes of A = Aspirin/ Anticoagulant (heparin)
 – 2 = distance of larynx to level of base of
Normal Anion Gap Metabolic R = Rest
 tongue

Acidosis C = Converting enzyme inhibitor
 M = Mallampati scoring

H = Hyperventilation (chronic)
 T = Thrombolysis
 O = Obstruction

A = Acetazolamide, Acids (e.g., I = IV beta blocker
 N = Neck mobility
hydrochloric), Addison’s disease
 O = Oxygen


833
LOAD: Rapid Sequence D = Diet
 Quicktrach, cricothyrodotomy set,

Intubation Premedications S = Spiritual Nursing LMAs



L = Lidocaine
 L = Laryngoscope blade: Straight (in BM,
O = Opioids

MOANS: Difficult BVM “L”urus, therefore, Mi”L”ler), “C”urved =
M = Mask seal not good, e.g. beard, Ma”C”intosh; good light source

A = Atropine

facial deformity, etc
 S = Syringe to test and inflate cuff balloon
D = Defasciculating dose of competitive
O = Obesity (difficult ventilate), 3rd of ETT

NMB
trimester pregnancy, or obstruction e.g. S = Stylet

MANTRELS: Clinical neck swelling, angioedema, hematomas, S = Suction catheter, Yankauer catheter
cancer, etc

Decision Rules for
Appendicitis A = Age, elderly, loss of muscle tone to MIDAS: Coma (conditions to
M = Migration of pain RLQ
 support the upper airway
 exclude as cause)
A = Anorexia
 N = No teeth (no teach causing caved in M = Meningitis

N = N/V
 face)
 I = Intoxication

T = Tenderness in RLQ
 S = Stiff lungs – upper airway obstruction D = Diabetes

R = Rebound pain
 – exacerbation of asthma, COPD, etc A = Air (respiratory failure)

E = Elevated temp >= 37.3 C
 S = Subdural/ Subarachnoid hemorrhage
MEALSSS: Rapid Sequence
L = Leukocytosis >=10

S = Shift of WBC to left
Intubation Equipments OLDER SAAB: Pain history
M = Mask – well fitting snugly
 checklist
E = ETT (appropriate size + 1 size above O = Onset

METHODS Discharge
and 1 size below); for children, ETT size = L = Location

Planning
M = Medication
 (age/4) + 4 or child’s little finger (less D = Description (what does it feel like)

E = Environment and exercise
 accurate)
 E = Exacerbating factors

T = Treatment
 A = Airway gadgets in case of difficult R = Radiation

H = Health Teachings
 airway or failed airway, e.g. S = Severity

O = Out Patient
 Oropharyngeal airways, surgical airways, A = Associated symptoms


834
A = Alleviating factors
 C = Cardiac tamponade
 PULSE: MI signs and
B = Before (ever experience this before) H = Hypokalemia/ Hyperkalemia/ symptoms
Hypoxia/ Hypothermia/ Hypovolemia
 P = Persistent chest pains

O NAVEL: Endotrachial tube M = Myocardial infarction
 U = Upset stomach

deliverable drugs E = Electrolyte derangements
 L = Lightheadedness

O = Oxygen
 D = Drugs S = Shortness of breath

N = Naloxone

E = Excessive sweating
A = Atropine
 PIRATES: Atrial fibrillation
V = Ventolin (albuterol), Vasopressin
 causes RATE: Hemolytic-Uremic
E = Epinephrine
 P = Pulmonary: PE, COPD
 Syndrome components
L = Lidocaine I = Iatrogenic
 R = Renal failure

R = Rheumatic heart: mirtral regurgitation
 A = Anemia (microangiopathic, hemolytic)

O SHIT: Management of A = Atherosclerotic: MI, CAD
 T = Thrombocytopenia

acute severe asthma T = Thyroid: hyperthyroid
 E = Encephalopathy (TTP)
O = Oxygen (high dose: >60%)
 E = Endocarditis

S = Salbutamol (5mg via oxygen-driven S = Sick sinus syndrome RESS: Principles of
nebuliser)
 management in toxicology
H = Hydrocortisone (or prednisolone)
 PQRST: Mnemonic for a R = Reduce absorption

I = Ipratropium bromide (if life threatening)
 complete pain history E = Enhance elimination

T = Theophylline (or preferably P3 = Positional, palliating, and provoking S = Specific antidote

aminophylline-if life threatening) factors
 S = Supportive treatment
Q = Quality

PATCH MED: Pulseless R3 = Region, radiation, referral
 RN CHAMPS: Shock types
electrical activity – causes S = Severity
 R = Respiratory

P = Pulmonary embolus
 T3 = Temporal factors (time and mode of N = Neurogenic

A = Acidosis
 onset, progression, previous episodes) C = Cardiogenic

T = Tension pneumothorax
 H = Hemorrhagic


835
A = Anaphylactic
 SAMPLE: Focused History in R = Radiation distortion

M = Metabolic
 Emergency Conditions/ T = Tumor
P = Psychogenic

Trauma
S = Septic S = signs and symptoms

SIMPLE: Criteria to define
A = allergies

simple febrile seizure
RODS: Difficult extraglottic M = medications

S = Seizure of focal type

devices P = pertinent past medical history

I = Intracranial infection

R = Restrictied mouth opening
 M = Multiple times a day

L = last oral intake

O = Obstruction upper airway
 P = Past history of afebrile seizure

E = events leading up to.
D = Disrupted or distorted upper airway
 L = Last longer than 15 minutes

S = Stiff lungs, spine of cervical Scared Lovers Try Positions E = Examination abnormalities

That They Can’t Handle:


SAD PUCKER: Structures in SINUS BRADICARDIA (sinus
Carpal (Wrist) Bones
retroperitoneal space S = Scaphoid

bradycardia): Sinus
S = Suprarenal glands (adrenals)

L = Lunate

bradycardia aetiology
A = Aorta/IVC
 S = Sleep

T = Triquetrum

D = Duodenum (2nd – 3rd, and 4th I = Infections (myocarditis)

P = Pisiform

segments)
 N = Neap thyroid (hypothyroid)

T = Trapezium

P = Pancreas (tail is intraperitoneal)
 U = Unconsciousness (vasovagal
T = Trapezoid

U = Ureters
 syncope)

C = Capitate

C = Colon (only the ascending and S = Subnormal temperatures
H = Hamate
descending parts)
 (hypothermia)

K = Kidneys
 B = Biliary obstruction

SHORT: Difficult
E = Esophagus
 R = Raised CO2 (hypercapnia)

cricothyrotomy
R = Rectum S = previous Surgery
 A = Acidosis

H = Hematoma/swelling around neck
 D = Deficient blood sugar (hypoglycemia)

 
O = Obesity
 I = Imbalance of electrolytes


836
C = Cushing’s reflex (raised ICP)
 F = Fracture
 R = Renal tubular acidosis

A = Aging
 E = Elderly
 P = Pancreatic fistula
R = Rx (drugs, such as high-dose R = Road trip
atropine)
 VOMITING: Vomiting, extra
D = Deep anaesthesia
 TV SPARC CUBE: Shock GI differential
I = Ischemic heart disease
 signs and symptoms V = Vestibular disturbance/ Vagal (reflex
A = Athletes T = Thirst
 pain)

V = Vomiting
 O = Opiates

SITTT: Causes of hematuria S = Sweating
 M = Migrane/ Metabolic (DKA,
S = Stone
 P = Pulse weak
 gastroparesis, hypercalcemia)

I = Infection
 A = Anxious
 I = Infections

T = Trauma
 R = Respirations shallow/rapid
 T = Toxicity (cytotoxic, digitalis toxicity)

T = Tumor
 C = Cool
 I = Increased ICP, Ingested alcohol

T = Tuberculosis C = Cyanotic
 N = Neurogenic, psychogenic

U = Unconscious
 G = Gestation
TOM SCHREPFER: B = BP low

Predisposing Conditions for E = Eyes blank
Pulmonary Embolism References and Further Reading, click
T = Trauma
 USED CARP: Causes of here
O = Obesity
 Normal Anion Gap Metabolic
M = Malignancy
 Acidosis
S = Surgery
 U = Ureteroenterostomy

C = Cardiac disease
 S = Small bowel fistula

H = Hospitalization
 E = Extra chloride

R = Rest (bed-bound)
 D = Diarrhea

E = Estrogen, pregnancy, post-partum
 C = Carbonic anhydrase inhibitors

P = Past hx
 A = Adrenal insufficiency


837
Section 3

Classifications and Scores

Case 1
by Sarah Attwa and Marwan Galal A 20-year-old male presents to your ED with a 5 cm wound
after he fell off his motorbike. On physical exam, the wound
overlays a fractured left tibia but does not show extensive soft
tissue damage nor any signs of periosteal stripping or vascular
injury. Which antibiotic should you give to this patient?

Table 20.6 Gustilo-Anderson Classification


TYPE DEFINITION

Type I Open fracture, clean wound, wound <1cm in length

Type II Open fracture, wound >1cm in length without extensive soft tissue damage, flaps, avulsions

Type III Open fracture with extensive soft tissue laceration, damage, or loss or an open segmental fracture. This
type also includes open fractures caused by farm injuries, fractures requiring vascular repair, or fractures
that have been open for 8 hours prior to treatment.

Type III A Type III fracture with adequate periosteal coverage of the fractured bone despite extensive soft tissue
laceration or damage

Type III B Type III fracture with extensive soft tissue loss and periosteal stripping and bone damage. Usually
associated with massive contamination. It will often need further soft tissue coverage procedure (i.e.
free or rotational flap).

Type III C Type III fracture associated with arterial injury requiring repair, irrespective of degree of soft tissue injury

838
Application: Gustilo-Anderson Case 2 Image 20.1
classification for open wounds and A 7-year-old boy was brought
antibiotic coverage
by his mother to the ED after a
Interpretation: According to the above heavy object fell on his right
classification, each class should receive
hand earlier the same day. On
the following antibiotics:
physical exam, there is bony
Type I: 1st generation cephalosporin
tenderness, swelling, and
Type II: 1st generation Cephalosporin +/- erythema over his right middle
Gentamycin
finger PIP joint. Distal pulses
Type III: 1st generation Cephalosporin + are intact and no neurological
Gentamycin +/- Penicillin
deficit. You decided to send
Hint: In farm and war wounds, all 3 the patient for imaging. XR is
antibiotics must be given
shown below. What is the
The answer to the above clinical classification of this fracture?
scenario: Type II, Cephazolin +/-
Gentamycin

839
Illustration 20.1 Salter-Harris Class V and I are the least commonly CXR is normal, CBC shows
Classification picked up
WBC of 3600 and urine
Class V carries the worst prognosis dipstick is positive for nitrites
The answer to the above clinical and leukocytes. What is the
scenario: Class II next step in management?
Case 3 SIRS (Systemic Immune
An 85-year-old female was Response Syndrome) Formula
1. Temp > 38 c or < 36 c
brought to the ED by her son
with a 2-day history of fever 2. HR > 90/min

and altered mental status. She 3. RR > 20/min or PaCO2 < 32 mmHg
is known to be diabetic and
4. WBC > 12000 or < 4000
hypertensive. Her vitals are
Application: Any patient with suspected
Temp 38.6 Celsius, BP 85/53,
systemic inflammatory response and can
HR 110/min and RR is 26/min, help guide critical decisions and
Application: This a classification for long
and O2 saturation is 98% on interventions
bone fractures involving epiphyseal
growth plates. room air. On examination, she Interpretation: SIRS is met when the
is alert but confused, and the patient has 2 or more criteria of the above
Hints:
rest of her physical exam is Hints:
Class II fractures are the most common
unremarkable. Random
injuries seen in the ED SIRS + source of infection = Sepsis (16%
glucose level is 8.5 mmol/L, Mortality)

840
Sepsis + more than one organ’s system Temp 37.6 Celsius, BP 100/55, Persistent high SI has been
dysfunction (e.g. Oliguria) = Severe associated with poor outcome
HR 110/min, RR 20/min and
Sepsis (20% Mortality)
O2 Saturation is 99% on room The answer to the above clinical
Severe Sepsis + Hypotension scenario: By applying the above
air. What level of care does this
(unresponsive to fluid resuscitation) = equation, (110/100 = 1.1), this patient has
Septic Shock (69% Mortality) patient require? a high shock index and requires a high
level of care.
The answer to the above clinical Formula
scenario: By applying the above criteria, Case 5
SHOCK INDEX (SI) = HR / SBP
this patient has SIRS + urinary tract
A 72-year-old female presented
infection; therefore, she is in sepsis. She Application: It can be used to identify
needs adequate fluid resuscitation +
with a fever, cough, and
patients needing a higher level of care
Antibiotics, and if still hypotensive, she despite vital signs that may not appear sputum for the last 4 days. She
will be classified as septic shock and will strikingly abnormal. This index is a has a past medical history of
require a higher level of care (e.g., sensitive indicator of left ventricular
DM and hypertension. Her
Vasopressors) and close monitoring. dysfunction and can become elevated
Those patients should be admitted to the following a reduction in left ventricular vitals are: Temp 38.9 Celsius,
ICU. stroke work. HR 110/min, BP 100/45, RR

Case 4 Interpretation:
27/min, and O2 sat 92% on
A 27-year-old female presented room air. On exam, she is alert
Normal SI = 0.5 to 0.7
to the ED with severe and oriented, and chest
If SI > 0.9 was helpful to identify patients
abdominal pain for 1 day. No auscultation reveals crackles
in the ED requiring admission and/or
allergies or significant past intensive care despite apparently stable
over the right lower chest. The
medical history. Her vitals are: vital signs remainder of the physical exam

841
was normal. CXR reveals right Application: Clinical scoring system used cardiac auscultation reveals
for risk stratification and guide
lower lung lobe infiltrate. Labs normal S1-S2 with tachycardia,
management in all adult patients
showed mildly elevated white presenting with evidence of pneumonia no lower limb edema or
cell count with normal renal tenderness. Her left leg is
The answer to the above clinical
function and metabolic panel. scenario: By applying the above tool, this swollen compared to right side.
How would you risk-stratify the patient has a CURB-65 score of 2 (age + 12 lead ECG shows sinus
severity of pneumonia in this diastolic BP). This patient will likely need rhythm with no abnormal
admission for further treatment.
patient? What would be her findings. What is your next step
appropriate disposition? Case 6 in diagnosis?
A 61-year-old female presents
Table 20.7 CURB-65 Application: Wells score is used to
to the ED with leg swelling over calculate pretest probability for all
CATEGORY CRITERIA SCORE
the past 2 days. Her past patients with clinically suspected DVT
C Confusion 1

U Urea > 7 mmol/L 1 medical history is positive for


Interpretation:
R Respiratory rate > 30 1
DM, hypertension, treatment
B Systolic BP <90mmhg 1 0: low pretest probability
or Diastolic BP <60
mmHg
for ovarian cancer 4 months
65 years Age > 65 years 1 ago. Her vital signs are Temp, 1-2: Moderate pretest probability

37.8 C, HR: 98, BP: 109/72, 3 or more: High pretest probability


CURB-65 score 30-day mortality Management
RR: 16, and O2 sat 98%on
0-1 <5% Home

2 <10% Likely to need room air. On exam, she is alert,


admission

3-5 15-30% Admit,


oriented and in acute distress.
manage as
severe Lung auscultation is clear;
842
Table 20.8 Wells Score for Deep Vein Thrombosis developing stroke? What is the recommended
CRITERIA SCORE therapy for him?
Active cancer(treatment ongoing or within previous 6 months or palliative 1
treatment)
Table 20.9 CHADS2 Score for Atrial Fibrillation
Paralysis, paresis, or recent plaster immobilization or of the lower extremities 1

Recently bedridden for 3 days or more or major surgery within the previous 1 CATEGORY CRITERIA POINT VALUE
12 weeks requiring general or regional anesthesia
C Congestive heart failure 1
Localized tenderness along the distribution of the deep venous system 1
H Hypertension (>140/90 mmHg) 1
Entire leg swollen 1
A Age > 75 years 1
Calf swelling > 3cm compared to asymptomatic leg (measuring 10 cm below 1
tibial tuberosity)
D Diabetes Mellitus 1
Pitting edema confined to the symptomatic leg 1
S2 Prior Stroke or TIA 2
Non varicose collateral superficial veins 1

Previously documented DVT 1

Alternative diagnosis at least as likely as DVT 1 Application: Clinical prediction rule for assessing the risk of
stroke in patients with non-rheumatic Atrial Fibrillation and is used
Case 7 to determine if treatment is required with anticoagulation therapy
or antiplatelet therapy or not.
A 54-year-old male with a past medical history of
peripheral vascular disease comes in with on/off
Table 20.10 CHADS2 Interpretation
palpitations and lightheadedness for the past 2
ANTICOAGULATION
SCORE RISK RECOMMENDATIONS
weeks. His vital signs are normal. On exam, he is THERAPY

alert, oriented and chest is clear to auscultation 0 Low No therapy OR Aspirin


(ASA)
No therapy, if patient prefers give ASA

and heart sounds are irregularly irregular. The 1 Moderate Oral anticoagulant OR Oral anticoagulant, alternatives are ASA
ASA with Clopidogrel or ASA alone
remainder of his physical exam is unremarkable. 2 or High Oral anticoagulant Oral anticoagulant, alternatives are ASA

His ECG shows Atrial Fibrillation with HR of 96 greater with Clopidogrel or ASA alone

beats/min. What is this patient’s risk for


843
Special considerations ECG. How do you interpret this depression in V2 and V3 > 1mm,
which gives him a Sgarbossa score of 3.
• In low-risk patients, female sex OR ECG?
This means this patient has Acute MI.
patients with vascular disease, ASA is
Sgarbossa Criteria
recommended Case 9
• Concordant ST elevation ≥1 mm = 5
• In low-risk patients, female sex AND points A 13-year-old boy comes in
vascular disease, oral anticoagulant is complaining of right lower
• ST depression ≥1 mm in V1-V3 = 3
preferred quadrant abdominal pain for
points
• In low-risk patients, if age > 65 years, the past 2 days, associated
• Discordant ST elevation ≥5 mm = 2
oral anticoagulant is preferred with nausea, vomiting, and loss
points
The answer to the above clinical of appetite. His vitals are
Application: Used in cases of left bundle
scenario: Applying the above score, the temperature 38.1, BP 110/77,
branch block (LBBB) and suspicion of
patient is at low risk for stroke (Score of
acute myocardial infarction (AMI) HR 100, RR 18, and oxygen
0), and the recommended therapy for
him, given his peripheral vascular disease Interpretation: At score-sum of 3 or
saturation 99% on RA. On
is ASA. greater, these criteria have specificity of physical exam, he has right iliac
90% for detecting AMI fossa tenderness with rebound.
Case 8
A 70-year-old male with known Hints: These criteria can also be applied His initial labs are significant for
ischemic heart disease and to Pacemaker rhythm leukocytosis with neutrophilic
permanent pacemaker The answer to the above clinical left shift. After pain relief, what
presents to the ED with chest scenario: This ECG is showing a paced is the next best step in
rhythm, and you can clearly see the
pain for 2 hours. His initial vitals management?
pacemaker spikes. By applying the above
are stable and below is his criteria, this patient has Concordant ST
844
Table 20.11 Alvarado Score for Appendicitis definitely has acute appendicitis requiring surgery. He

CATEGORY CRITERIA POINT VALUE


should be urgently referred to the surgical team.

Symptoms Migratory right iliac fossa pain 1


Case 10 and 11
Nausea/vomiting 1
A 70-year-old male was brought to the ED by his
Anorexia 1
son after he collapsed at home 2 hours ago. He
Signs Tenderness in right iliac fossa 2
has history of fever and URI symptoms for the
Rebound tenderness in right iliac 1
past 3 days. On exam, patient stuporous, and
fossa

Fever 1
opens his eye to a verbal prompt, moaning and
withdraws from painful stimuli. What’s his GCS
Lab findings Leukocytosis 2 score?
Neutrophil left shift 2
A 18-year-old male involved in a motor vehicle
Application: Used in all cases of clinically suspected acute collision was brought in by EMS with apparent
appendicitis facial and head injuries. On exam, with pinching
his chest, he does not open his eyes nor makes
Table 20.12 Interpretation of Alvarado Score
any sounds but flexes both arms inwards. What’s
SCORE SIGNIFICANCE

1-4 Unlikely his GCS score?


5-6 Possible

7-8 Acute appendicitis present

9-10 Definite acute appendicitis requiring surgery

The answer to the above clinical scenario: Using the above


scoring system, the patient has a score of 9, and therefore,
845
Table 20.13 Glasgow Coma Scale Application: Part of neurological
CHOOSE THE BEST RESPONSE OF examination for any patient (e.g., trauma,
PATIENT altered mental status, intoxication, etc.)
EYE OPENING

4: Spontaneously
Interpretation: Useful objective tool to
3: To verbal command
assess and quantify neurological function
of patients in ED to help guide critical
2: To pain
decisions and interventions (e.g.,
1: No response
Intubation to protect the airway )

Hints: Patients with a score of 8 or below


BEST VERBAL RESPONSE
due to irreversible causes need airway
5: Oriented and converses
protection via intubation
4: Disoriented and converses

3: Inappropriate words; cries


Special consideration: Modified GCS
2: Incomprehensible sounds
score for the pediatric population
1: No response

The answer to the above clinical


BEST MOTOR RESPONSE scenarios:
6: Obeys command
Case 10 – GCS score of 9
5: Localizes pain

4: Flexion withdrawal
Case 11 – GCS score of 5 (this patient
3: Flexion abnormal (decorticate)
needs airway protection)
2: Extension (decerebrate)

1: No response

Glasgow Coma Score (GCS) (Modified from


Teasdale, G., & Jennett, B. (1974). Assessment References and Further Reading, click
of coma and impaired consciousness: a
practical scale. The Lancet, 304(7872), 81-84.) -
here
Please read this article to get more insight
regarding GCS.
846

Das könnte Ihnen auch gefallen